Sei sulla pagina 1di 292

Sales – Chapter 3-12 Cases

HERMINIO TAYAG, petitioner, petitioner gave varied sums of money to the tenants as 4. That defendants Julio Tiamson, Renato Gozun, Rosita
vs. partial payments, and the latter issued receipts for the Hernandez, Bienvenido Tongol, Alfonso Flores, Norma
AMANCIA LACSON, ROSENDO LACSON, ANTONIO said amounts. Quiambao, Rosita Tolentino, Jose Sosa, Francisco
LACSON, JUAN LACSON, TEODISIA LACSON- Tolentino, Sr., Emiliano Laxamana, Ruben Torres, Meliton
ESPINOSA and THE COURT OF APPEALS, On July 24, 1996, the petitioner called a meeting of the Allanigue, Dominga Laxamana, Felicencia de Leon,
respondents. defendants-tenants to work out the implementation of the Emiliano Ramos are original farmers or direct tillers of
terms of their separate agreements.7 However, on August landholdings over parcels of lands covered by Transfer
Before us is a petition for review on certiorari of the 8, 1996, the defendants-tenants, through Joven Mariano, Certificate of Title Nos. 35922-R, 35923-R and 35925-R
Decision1 and the Resolution2 of respondent Court of wrote the petitioner stating that they were not attending which are registered in the names of defendants
Appeals in CA-G.R. SP No. 44883. the meeting and instead gave notice of their collective LACSONS; while defendants Felino G. Tolentino, Rica
decision to sell all their rights and interests, as Gozun, Perla Gozun, Benigno Tolentino, Rodolfo
The Case for the Petitioner tenants/lessees, over the landholding to the Quiambao, Roman Laxamana, Eddie San Luis, Alfredo
respondents.8 Explaining their reasons for their collective Gozun, Jose Tiamson, Augusto Tolentino, Sixto
Respondents Angelica Tiotuyco Vda. de Lacson,3 and her decision, they wrote as follows: Hernandez, Alex Quiambao, Isidro Tolentino, Ceferino de
children Amancia, Antonio, Juan, and Teodosia, all Leon, Alberto Hernandez, and Aurelio Flores are sub-
surnamed Lacson, were the registered owners of three Kami ay nagtiwala sa inyo, naging tapat at nanindigan sa tenants over the same parcel of land.
parcels of land located in Mabalacat, Pampanga, covered lahat ng ating napagkasunduan, hindi tumanggap ng
by Transfer Certificates of Title (TCT) Nos. 35922-R, ibang buyer o ahente, pero sinira ninyo ang aming 5. That on March 17, 1996 the defendants TIAMSON, et
35923-R, and 35925-R, registered in the Register of pagtitiwala sa pamamagitan ng demanda ninyo at al., entered into Deeds of Assignment with the plaintiff by
Deeds of San Fernando, Pampanga. The properties, pagbibigay ng problema sa amin na hindi naman which the defendants assigned all their rights and
which were tenanted agricultural lands,4 were nagbenta ng lupa. interests on their landholdings to the plaintiff and that on
administered by Renato Espinosa for the owner. the same date (March 17, 1996), the defendants received
Kaya kami ay nagpulong at nagpasya na ibenta na lang from the plaintiff partial payments in the amounts
On March 17, 1996, a group of original farmers/tillers, ang aming karapatan o ang aming lupang sinasaka sa corresponding to their names. Subsequent payments
namely, Julio Tiamson, Renato Gozun, Rosita Hernandez, landowner o sa mga pamilyang Lacson, dahil ayaw were also received:
Bienvenido Tongol, Alfonso Flores, Norma Quiambao, naming magkaroon ng problema.
Rosita Tolentino, Jose Sosa, Francisco Tolentino, Sr.,
Emiliano Laxamana, Ruben Torres, Meliton Allanigue, Kaya kung ang sasabihin ninyong ito’y katangahan, lalo 1st PAYMENT 2nd PAYMENT CHECK NO.
Dominga Laxamana, Felicencia de Leon, Emiliano sigurong magiging katangahan kung ibebenta pa namin TOTAL
Ramos, and another group, namely, Felino G. Tolentino, sa inyo ang aming lupang sinasaka, kaya pasensya na 1.Julio Tiamson - - - - - - P 20,000 P 10,621.54
Rica Gozun, Perla Gozun, Benigno Tolentino, Rodolfo lang Mister Tayag. Dahil sinira ninyo ang aming 231281 P 30,621.54
Quiambao, Roman Laxamana, Eddie San Luis, Ricardo pagtitiwala at katapatan.9 2. Renato Gozun - - - - - -
Hernandez, Nicenciana Miranda, Jose Gozun, Alfredo [son of Felix Gozun (deceased)] P 10,000
Sosa, Jose Tiamson, Augusto Tolentino, Sixto On August 19, 1996, the petitioner filed a complaint with 96,000 106,000.00
Hernandez, Alex Quiambao, Isidro Tolentino, Ceferino de the Regional Trial Court of San Fernando, Pampanga, 3. Rosita Hernandez - - - - P 5,000 14,374.24
Leon, Alberto Hernandez, Orlando Flores, and Aurelio Branch 44, against the defendants-tenants, as well as the 231274 P 19,374.24
Flores,5 individually executed in favor of the petitioner respondents, for the court to fix a period within which to 4. Bienvenido Tongol - - -
separate Deeds of Assignment6 in which the assignees pay the agreed purchase price of P50.00 per square [Son of Abundio Tongol (deceased)] P 10,000
assigned to the petitioner their respective rights as meter to the defendants, as provided for in the Deeds of 14,465.90 231285 24,465.90
tenants/tillers of the landholdings possessed and tilled by Assignment. The petitioner also prayed for a writ of 5. Alfonso Flores - - - - - - P 30,000 26,648.40
them for and in consideration of P50.00 per square meter. preliminary injunction against the defendants and the 231271 56,648.40
The said amount was made payable "when the legal respondents therein.10 The case was docketed as Civil 6. Norma Quiambao - - - - P 10,000 41,501.10
impediments to the sale of the property to the petitioner no Case No. 10910. 231279 51,501.10
longer existed." The petitioner was also granted the 7. Rosita Tolentino - - - - - P 10,000 22,126.08
exclusive right to buy the property if and when the In his complaint, the petitioner alleged, inter alia, the 231284 32,126.08
respondents, with the concurrence of the defendants- following: 8. Jose Sosa - - - - - - - - - P 10,000 14,861.31
tenants, agreed to sell the property. In the interim, the 231291 24,861.31
1
Sales – Chapter 3-12 Cases
9. Francisco Tolentino, Sr. P 10,000 24,237.62 34. Orlando Florez 10,000 ------ ------ ----- on selling/alienating their rights and interests over the
231283 34,237.62 - subject properties to their co-defendants (LACSONS) or
10. Emiliano Laxamana - - P 10,000 ------ ----- 35. Aurelio Flores 10,000 ------ ------ ------ any other persons to the damage and prejudice of the
- ------ 6. That on July 24, 1996, the plaintiff wrote the defendants plaintiff who already invested much money, efforts and
11. Ruben Torres - - - - - - TIAMSON, et al., inviting them for a meeting regarding the time in the said transactions;
[Son of Mariano Torres (deceased)] P 10,000 P negotiations/implementations of the terms of their Deeds
33,587.31 ------ P 43,587.31 of Assignment; 13. That the plaintiff is entitled to the reliefs being
12. Meliton Allanigue P 10,000 12,944.77 demanded in the complaint;
231269 P 22,944.77 7. That on August 8, 1996, the defendants TIAMSON, et
13. Dominga Laxamana P 5,000 22,269.02 al., through Joven Mariano, replied that they are no longer 14. That to prevent irreparable damages and prejudice to
231275 27,269.02 willing to pursue with the negotiations, and instead they the plaintiff, as the latter has no speedy and adequate
14. Felicencia de Leon 10,000 ------ ------ ----- gave notice to the plaintiff that they will sell all their rights remedy under the ordinary course of law, it is essential
- and interests to the registered owners (defendants that a Writ of Preliminary Injunction be issued enjoining
15. Emiliano Ramos 5,000 18,869.60 LACSONS). and restraining the defendants TIAMSON, et al., from
231280 23,869.60 rescinding their contracts with the plaintiff and from
16. Felino G. Tolentino 10,000 ------ ------ ----- A copy of the letter is hereto attached as Annex "A" etc.; selling/alienating their properties to the LACSONS or
- other persons;
17. Rica Gozun 5,000 ------ ------ ------ 8. That the defendants TIAMSON, et. al., have no right to
18. Perla Gozun 10,000 ------ ------ ------ deal with the defendants LACSON or with any third 15. That the plaintiff is willing and able to put up a
19. Benigno Tolentino 10,000 ------ ------ ----- persons while their contracts with the plaintiff are reasonable bond to answer for the damages which the
- subsisting; defendants LACSONS are inducing or have defendants would suffer should the injunction prayed for
20. Rodolfo Quiambao 10,000 ------ ------ ----- induced the defendants TIAMSON, et. al., to violate their and granted be found without basis.12
- contracts with the plaintiff;
21. Roman Laxamana 10,000 ------ ------ ----- The petitioner prayed, that after the proceedings,
- 9. That by reason of the malicious acts of all the judgment be rendered as follows:
22. Eddie San Luis 10,000 ------ ------ ----- defendants, plaintiff suffered moral damages in the forms
- of mental anguish, mental torture and serious anxiety 1. Pending the hearing, a Writ of Preliminary Injunction be
23. Ricardo Hernandez 10,000 ------ ------ ----- which in the sum of P500,000.00 for which defendants issued prohibiting, enjoining and restraining defendants
- should be held liable jointly and severally.11 Julio Tiamson, Renato Gozun, Rosita Hernandez,
24. Nicenciana Miranda 10,000 ------ ------ ----- Bienvenido Tongol, Alfonso Flores, Norma Quiambao,
- In support of his plea for injunctive relief, the petitioner, as Rosita Tolentino, Jose Sosa, Francisco Tolentino Sr.,
25. Jose Gozun 10,000 ------ ------ ------ plaintiff, also alleged the following in his complaint: Emiliano Laxamana, Ruben Torres, Meliton Allanigue,
26. Alfredo Sosa 5,000 ------ ------ ------ Dominga Laxamana, Felicencia de Leon, Emiliano
27. Jose Tiamson 10,000 ------ ------ ------ 11. That to maintain the status quo, the defendants Ramos, Felino G. Tolentino, Rica Gozun, Perla Gozun,
28. Augusto Tolentino 5,000 ------ ------ ----- TIAMSON, et al., should be restrained from rescinding Benigno Tolentino, Rodolfo Quiambao, Roman
- their contracts with the plaintiff, and the defendants Laxamana, Eddie San Luis, Ricardo Hernandez,
29. Sixto Hernandez 10,000 ------ ------ ----- LACSONS should also be restrained from accepting any Nicenciana Miranda, Jose Gozun, Alfredo Sosa, Jose
- offer of sale or alienation with the defendants TIAMSON, Tiamson, Augusto Tolentino, Ceferino de Leon, Alberto
30. Alex Quiambao 10,000 ------ ------ ----- et al., in whatever form, the latter’s rights and interests in Hernandez, Orlando Flores, and Aurelio Flores from
- the properties mentioned in paragraph 4 hereof; further, rescinding their contracts with the plaintiff and from
31. Isidro Tolentino 10,000 ------ ------ ----- the LACSONS should be restrained from alienating their rights and interest over the aforementioned
- encumbering/alienating the subject properties covered by properties in favor of defendants LACSONS or any other
32. Ceferino de Leon ------ 11,378.70 TCT No. 35922-R, 35923-R and TCT No. 35925-R, third persons; and prohibiting the defendants LACSONS
231270 ------ Registry of Deeds of San Fernando, Pampanga; from encumbering/alienating TCT Nos. 35922-R, 35923-R
33. Alberto Hernandez 10,000 ------ ------ ----- and 35925-R of the Registry of Deeds of San Fernando,
- 12. That the defendants TIAMSON, et al., threaten to Pampanga.
rescind their contracts with the plaintiff and are also bent
2
Sales – Chapter 3-12 Cases
2. And pending the hearing of the Prayer for a Writ of each of them received from the petitioner were in the form appear. In support of his plea for a writ of preliminary
Preliminary Injunction, it is prayed that a restraining order of loans, and that they were deceived into signing the injunction, the petitioner adduced in evidence the Deeds
be issued restraining the aforementioned defendants deeds of assignment: of Assignment,16 the receipts17 issued by the
(TIAMSON, et al.) from rescinding their contracts with the defendants-tenants for the amounts they received from
plaintiff and from alienating the subject properties to the a) That all the foregoing allegations in the Answer are him; and the letter18 the petitioner received from the
defendants LACSONS or any third persons; further, hereby repleaded and incorporated in so far as they are defendants-tenants. The petitioner then rested his case.
restraining and enjoining the defendants LACSONS from material and relevant herein;
encumbering/selling the properties covered by TCT Nos. The respondents, thereafter, filed a Comment/Motion to
35922-R, 35923-R, and 35925-R of the Registry of Deeds b) That the defendants Tiamson, et al., in so far as the dismiss/deny the petitioner’s plea for injunctive relief on
of San Fernando, Pampanga. Deeds of Assignment are concern[ed] never knew that the following grounds: (a) the Deeds of Assignment
what they did sign is a Deed of Assignment. What they executed by the defendants-tenants were contrary to
3. Fixing the period within which plaintiff shall pay the knew was that they were made to sign a document that public policy and P.D. No. 27 and Rep. Act No. 6657; (b)
balance of the purchase price to the defendants will serve as a receipt for the loan granted [to] them by the the petitioner failed to prove that the respondents induced
TIAMSON, et al., after the lapse of legal impediment, if plaintiff; the defendants-tenants to renege on their obligations
any. under the "Deeds of Assignment;" (c) not being privy to
c) That the Deeds of Assignment were signed through the the said deeds, the respondents are not bound by the said
4. Making the Writ of Preliminary Injunction permanent; employment of fraud, deceit and false pretenses of deeds; and, (d) the respondents had the absolute right to
plaintiff and made the defendants believe that what they sell and dispose of their property and to encumber the
5. Ordering the defendants to pay the plaintiff the sum of sign[ed] was a mere receipt for amounts received by way same and cannot be enjoined from doing so by the trial
P500,000.00 as moral damages; of loans; court.

6. Ordering the defendants to pay the plaintiff attorney’s d) That the documents signed in blank were filled up and The petitioner opposed the motion, contending that it was
fees in the sum of P100,000.00 plus litigation expenses of completed after the defendants Tiamson, et al., signed the premature for the trial court to resolve his plea for
P50,000.00; documents and their completion and accomplishment was injunctive relief, before the respondents and the
done in the absence of said defendants and, worst of all, defendants-tenants adduced evidence in opposition
Plaintiff prays for such other relief as may be just and defendants were not provided a copy thereof; thereto, to afford the petitioner a chance to adduce
equitable under the premises.13 rebuttal evidence and prove his entitlement to a writ of
e) That as completed, the Deeds of Assignment reflected preliminary injunction. The respondents replied that it was
In their answer to the complaint, the respondents as that the defendants Tiamson, et al., did assign all their the burden of the petitioner to establish the requisites of a
defendants asserted that (a) the defendant Angelica Vda. rights and interests in the properties or landholdings they writ of preliminary injunction without any evidence on their
de Lacson had died on April 24, 1993; (b) twelve of the were tilling in favor of the plaintiff. That if this is so, part, and that they were not bound to adduce any
defendants were tenants/lessees of respondents, but the assuming arguendo that the documents were voluntarily evidence in opposition to the petitioner’s plea for a writ of
tenancy status of the rest of the defendants was executed, the defendants Tiamson, et al., do not have any preliminary injunction.
uncertain; (c) they never induced the defendants Tiamson right to transfer their interest in the landholdings they are
to violate their contracts with the petitioner; and, (d) being tilling as they have no right whatsoever in the On February 13, 1997, the court issued an Order19
merely tenants-tillers, the defendants-tenants had no right landholdings, the landholdings belong to their co- denying the motion of the respondents for being
to enter into any transactions involving their properties defendants, Lacson, et al., and therefore, the contract is premature. It directed the hearing to proceed for the
without their knowledge and consent. They also averred null and void; respondents to adduce their evidence. The court ruled
that the transfers or assignments of leasehold rights made that the petitioner, on the basis of the material allegations
by the defendants-tenants to the petitioner is contrary to f) That while it is admitted that the defendants Tiamson, et of the complaint, was entitled to injunctive relief. It also
Presidential Decree (P.D.) No. 27 and Republic Act No. al., received sums of money from plaintiffs, the same were held that before the court could resolve the petitioner’s
6657, the Comprehensive Agrarian Reform Program received as approved loans granted by plaintiff to the plea for injunctive relief, there was need for a hearing to
(CARP).14 The respondents interposed counterclaims for defendants Tiamson, et al., and not as part consideration enable the respondents and the defendants-tenants to
damages against the petitioner as plaintiff. of the alleged Deeds of Assignment; and by way of:…15 adduce evidence to controvert that of the petitioner. The
respondents filed a motion for reconsideration, which the
The defendants-tenants Tiamson, et al., alleged in their At the hearing of the petitioner’s plea for a writ of court denied in its Order dated April 16, 1997. The trial
answer with counterclaim for damages, that the money preliminary injunction, the respondents’ counsel failed to court ruled that on the face of the averments of the
3
Sales – Chapter 3-12 Cases
complaint, the pleadings of the parties and the evidence plea in his complaint before the trial court, to fix a period On August 4, 1998, the CA issued a Resolution denying
adduced by the petitioner, the latter was entitled to within which to pay the balance of the amounts due to the the petitioner’s motion for reconsideration.23
injunctive relief unless the respondents and the tenants under said deeds after the "lapse" of any legal
defendants-tenants adduced controverting evidence. impediment, assumed that the deeds were valid, when, in Hence, the petitioner filed his petition for review on
fact and in law, they were not. According to the certiorari before this Court, contending as follows:
The respondents, the petitioners therein, filed a petition for respondents, they were not parties to the deeds of
certiorari in the Court of Appeals for the nullification of the assignment; hence, they were not bound by the said I
February 13, 1997 and April 16, 1997 Orders of the trial deeds. The issuance of a writ of preliminary injunction
court. The case was docketed as CA-G.R. SP No. 44883. would restrict and impede the exercise of their right to A MERE ALLEGATION IN THE ANSWER OF THE
The petitioners therein prayed in their petition that: dispose of their property, as provided for in Article 428 of TENANTS COULD NOT BE USED AS EVIDENCE OR
the New Civil Code. They asserted that the petitioner had BASIS FOR ANY CONCLUSION, AS THIS
1. An order be issued declaring the orders of respondent no cause of action against them and the defendants- ALLEGATION, IS STILL THE SUBJECT OF TRIAL IN
court dated February 13, 1997 and April 16, 1997 as null tenants. THE LOWER COURT (RTC).24
and void;
On April 17, 1998, the Court of Appeals rendered its II
2. An order be issued directing the respondent court to decision against the petitioner, annulling and setting aside
issue an order denying the application of respondent the assailed orders of the trial court; and permanently THE COURT OF APPEALS CANNOT ENJOIN THE
Herminio Tayag for the issuance of a Writ of Preliminary enjoining the said trial court from proceeding with Civil HEARING OF A PETITION FOR PRELIMINARY
Injunction and/or restraining order. Case No. 10901. The decretal portion of the decision INJUNCTION AT A TIME WHEN THE LOWER COURT
reads as follows: (RTC) IS STILL RECEIVING EVIDENCE PRECISELY TO
3. In the meantime, a Writ of Preliminary Injunction be DETERMINE WHETHER OR NOT THE WRIT OF
issued against the respondent court, prohibiting it from However, even if private respondent is denied of the PRELIMINARY INJUNCTION BEING PRAYED FOR BY
issuing its own writ of injunction against Petitioners, and injunctive relief he demands in the lower court still he TAYAG SHOULD BE GRANTED OR NOT.25
thereafter making said injunction to be issued by this could avail of other course of action in order to protect his
Court permanent. interest such as the institution of a simple civil case of III
collection of money against TIAMSON, et al.
Such other orders as may be deemed just & equitable THE COURT OF APPEALS CANNOT USE "FACTS" NOT
under the premises also prayed for.20 For all the foregoing considerations, the orders dated 13 IN EVIDENCE, TO SUPPORT ITS CONCLUSION THAT
February 1997 and 16 April 1997 are hereby NULLIFIED THE TENANTS ARE NOT YET "AWARDEES OF THE
The respondents asserted that the Deeds of Assignment and ordered SET ASIDE for having been issued with LAND REFORM.26
executed by the assignees in favor of the petitioner were grave abuse of discretion amounting to lack or excess of
contrary to paragraph 13 of P.D. No. 27 and the second jurisdiction. Accordingly, public respondent is permanently IV
paragraph of Section 70 of Rep. Act No. 6657, and, as enjoined from proceeding with the case designated as
such, could not be enforced by the petitioner for being null Civil Case No. 10901.22 THE COURT OF APPEALS CANNOT CAUSE THE
and void. The respondents also claimed that the PERMANENT STOPPAGE OF THE ENTIRE
enforcement of the deeds of assignment was subject to a The CA ruled that the respondents could not be enjoined PROCEEDINGS BELOW INCLUDING THE TRIAL ON
supervening condition: from alienating or even encumbering their property, THE MERITS OF THE CASE CONSIDERING THAT THE
especially so since they were not privies to the deeds of ISSUE INVOLVED ONLY THE PROPRIETY OF
3. That this exclusive and absolute right given to the assignment executed by the defendants-tenants. The MAINTAINING THE STATUS QUO.27
assignee shall be exercised only when no legal defendants-tenants were not yet owners of the portions of
impediments exist to the lot to effect the smooth transfer the landholdings respectively tilled by them; as such, they V
of lawful ownership of the lot/property in the name of the had nothing to assign to the petitioner. Finally, the CA
ASSIGNEE.21 ruled that the deeds of assignment executed by the THE COURT OF APPEALS CANNOT INCLUDE IN ITS
defendants-tenants were contrary to P.D. No. 27 and Rep. DECISION THE CASE OF THE OTHER 35 TENANTS
The respondents argued that until such condition took Act No. 6657. WHO DO NOT QUESTION THE JURISDICTION OF THE
place, the petitioner would not acquire any right to enforce LOWER COURT (RTC) OVER THE CASE AND WHO
the deeds by injunctive relief. Furthermore, the petitioner’s ARE IN FACT STILL PRESENTING THEIR EVIDENCE
4
Sales – Chapter 3-12 Cases
TO OPPOSE THE INJUNCTION PRAYED FOR, AND TO Petitioner’s Plea for a Writ against the respondents on the basis of the material
PROVE AT THE SAME TIME THE COUNTER-CLAIMS of Preliminary Injunction averments of the complaint. In its April 16, 1997 Order,
THEY FILED AGAINST THE PETITIONER.28 Was Not Premature. the trial court denied the respondents’ motion for
reconsideration of the previous order, on its finding that
VI Contrary to the ruling of the trial court, the motion of the the petitioner was entitled to a writ of preliminary
respondents to dismiss/deny the petitioner’s plea for a writ injunction based on the material allegations of his
THE LOWER COURT (RTC) HAS JURISDICTION OVER of preliminary injunction after the petitioner had adduced complaint, the evidence on record, the pleadings of the
THE CASE FILED BY TAYAG FOR "FIXING OF his evidence, testimonial and documentary, and had parties, as well as the applicable laws:
PERIOD" UNDER ART. 1197 OF THE NEW CIVIL CODE rested his case on the incident, was proper and timely. It
AND FOR "DAMAGES" AGAINST THE LACSONS bears stressing that the petitioner had the burden to prove … For the record, the Court denied the LACSONS’
UNDER ART. 1314 OF THE SAME CODE. THIS CASE his right to a writ of preliminary injunction. He may rely COMMENT/MOTION on the basis of the facts culled from
CANNOT BE SUPPRESSED OR RENDERED solely on the material allegations of his complaint or the evidence presented, the pleadings and the law
NUGATORY UNCEREMONIOUSLY.29 adduce evidence in support thereof. The petitioner applicable unswayed by the partisan or personal interests,
adduced his evidence to support his plea for a writ of public opinion or fear of criticism (Canon 3, Rule 3.02,
The petitioner faults the Court of Appeals for permanently preliminary injunction against the respondents and the Code of Judicial Ethics).30
enjoining the trial court from proceeding with Civil Case defendants-tenants and rested his case on the said
No. 10910. He opines that the same was too drastic, incident. The respondents then had three options: (a) file Section 3, Rule 58 of the Rules of Court, as amended,
tantamount to a dismissal of the case. He argues that at a motion to deny/dismiss the motion on the ground that enumerates the grounds for the issuance of a writ of
that stage, it was premature for the appellate court to the petitioner failed to discharge his burden to prove the preliminary injunction, thus:
determine the merits of the case since no evidentiary factual and legal basis for his plea for a writ of preliminary
hearing thereon was conducted by the trial court. This, the injunction and, if the trial court denies his motion, for them (a) That the applicant is entitled to the relief demanded,
Court of Appeals cannot do, since neither party moved for to adduce evidence in opposition to the petitioner’s plea; and the whole or part of such relief consists in restraining
the dismissal of Civil Case No. 10910. The petitioner (b) forgo their motion and adduce testimonial and/or the commission or continuance of the act or acts
points out that the Court of Appeals, in making its findings, documentary evidence in opposition to the petitioner’s complained of, or in requiring the performance of an act or
went beyond the issue raised by the private respondents, plea for a writ of preliminary injunction; or, (c) waive their acts, either for a limited period or perpetually;
namely, whether or not the trial court committed a grave right to adduce evidence and submit the incident for
abuse of discretion amounting to excess or lack of consideration on the basis of the pleadings of the parties (b) That the commission, continuance or non-performance
jurisdiction when it denied the respondent’s motion for the and the evidence of the petitioner. The respondents opted of the act or acts complained of during the litigation would
denial/dismissal of the petitioner’s plea for a writ of not to adduce any evidence, and instead filed a motion to probably work injustice to the applicant; or
preliminary injunction. He, likewise, points out that the deny or dismiss the petitioner’s plea for a writ of
appellate court erroneously presumed that the preliminary injunction against them, on their claim that the (c) That a party, court, agency or a person is doing,
leaseholders were not DAR awardees and that the deeds petitioner failed to prove his entitlement thereto. The trial threatening, or is attempting to do, or is procuring or
of assignment were contrary to law. He contends that court cannot compel the respondents to adduce evidence suffering to be done, some act or acts probably in violation
leasehold tenants are not prohibited from conveying or in opposition to the petitioner’s plea if the respondents opt of the rights of the applicant respecting the subject of the
waiving their leasehold rights in his favor. He insists that to waive their right to adduce such evidence. Thus, the action or proceeding, and tending to render the judgment
there is nothing illegal with his contracts with the trial court should have resolved the respondents’ motion ineffectual.
leaseholders, since the same shall be effected only when even without the latter’s opposition and the presentation
there are no more "legal impediments." of evidence thereon. A preliminary injunction is an extraordinary event
calculated to preserve or maintain the status quo of things
At bottom, the petitioner contends that, at that stage, it The RTC Committed a Grave ante litem and is generally availed of to prevent actual or
was premature for the appellate court to determine the Abuse of Discretion Amounting threatened acts, until the merits of the case can be heard.
merits of his case since no evidentiary hearing on the to Excess or Lack of Jurisdiction Injunction is accepted as the strong arm of equity or a
merits of his complaint had yet been conducted by the trial in Issuing its February 13, 1997 transcendent remedy.31 While generally the grant of a
court. and April 16, 1997 Orders writ of preliminary injunction rests on the sound discretion
of the trial court taking cognizance of the case, extreme
The Comment/Motion of the In its February 13, 1997 Order, the trial court ruled that the caution must be observed in the exercise of such
Respondents to Dismiss/Deny petitioner was entitled to a writ of preliminary injunction discretion.32 Indeed, in Olalia v. Hizon,33 we held:
5
Sales – Chapter 3-12 Cases
Q: Do you know that two (2) of the defendants are
It has been consistently held that there is no power the First. The trial court cannot enjoin the respondents, at the residents of the United States?
exercise of which is more delicate, which requires greater instance of the petitioner, from selling, disposing of and
caution, deliberation and sound discretion, or more encumbering their property. As the registered owners of A: I do not know, sir.
dangerous in a doubtful case, than the issuance of an the property, the respondents have the right to enjoy and
injunction. It is the strong arm of equity that should never dispose of their property without any other limitations than Q: You do not know also that Angela Tiotuvie (sic) Vda.
be extended unless to cases of great injury, where courts those established by law, in accordance with Article 428 of de Lacson had already been dead?
of law cannot afford an adequate or commensurate the Civil Code. The right to dispose of the property is the
remedy in damages. power of the owner to sell, encumber, transfer, and even A: I am aware of that, sir.39
destroy the property. Ownership also includes the right to
Every court should remember that an injunction is a recover the possession of the property from any other We are one with the Court of Appeals in its ruling that:
limitation upon the freedom of action of the defendant and person to whom the owner has not transmitted such
should not be granted lightly or precipitately. It should be property, by the appropriate action for restitution, with the We cannot see our way clear on how or why injunction
granted only when the court is fully satisfied that the law fruits, and for indemnification for damages.38 The right of should lie against petitioners. As owners of the lands
permits it and the emergency demands it.34 ownership of the respondents is not, of course, absolute. being tilled by TIAMSON, et al., petitioners, under the law,
It is limited by those set forth by law, such as the agrarian have the right to enjoy and dispose of the same. Thus,
The very foundation of the jurisdiction to issue writ of reform laws. Under Article 1306 of the New Civil Code, the they have the right to possess the lands, as well as the
injunction rests in the existence of a cause of action and in respondents may enter into contracts covering their right to encumber or alienate them. This principle of law
the probability of irreparable injury, inadequacy of property with another under such terms and conditions as notwithstanding, private respondent in the lower court
pecuniary compensation and the prevention of the they may deem beneficial provided they are not contrary sought to restrain the petitioners from encumbering and/or
multiplicity of suits. Where facts are not shown to bring the to law, morals, good conduct, public order or public policy. alienating the properties covered by TCT No. 35922-R,
case within these conditions, the relief of injunction should 35923-R and TCT No. 35925-R of the Registry of Deeds
be refused.35 The respondents cannot be enjoined from selling or of San Fernando, Pampanga. This cannot be allowed to
encumbering their property simply and merely because prosper since it would constitute a limitation or restriction,
For the court to issue a writ of preliminary injunction, the they had executed Deeds of Assignment in favor of the not otherwise established by law on their right of
petitioner was burdened to establish the following: (1) a petitioner, obliging themselves to assign and transfer their ownership, more so considering that petitioners were not
right in esse or a clear and unmistakable right to be rights or interests as agricultural farmers/laborers/sub- even privy to the alleged transaction between private
protected; (2) a violation of that right; (3) that there is an tenants over the landholding, and granting the petitioner respondent and TIAMSON, et al.40
urgent and permanent act and urgent necessity for the the exclusive right to buy the property subject to the
writ to prevent serious damage.36 Thus, in the absence of occurrence of certain conditions. The respondents were Second. A reading the averments of the complaint will
a clear legal right, the issuance of the injunctive writ not parties to the said deeds. There is no evidence that show that the petitioner clearly has no cause of action
constitutes a grave abuse of discretion. Where the the respondents agreed, expressly or impliedly, to the said against the respondents for the principal relief prayed for
complainant’s right is doubtful or disputed, injunction is not deeds or to the terms and conditions set forth therein. therein, for the trial court to fix a period within which to pay
proper. Injunction is a preservative remedy aimed at Indeed, they assailed the validity of the said deeds on to each of the defendants-tenants the balance of the
protecting substantial rights and interests. It is not their claim that the same were contrary to the letter and P50.00 per square meter, the consideration under the
designed to protect contingent or future rights. The spirit of P.D. No. 27 and Rep. Act No. 6657. The petitioner Deeds of Assignment executed by the defendants-
possibility of irreparable damage without proof of even admitted when he testified that he did not know any tenants. The respondents are not parties or privies to the
adequate existing rights is not a ground for injunction.37 of the respondents, and that he had not met any of them deeds of assignment. The matter of the period for the
before he filed his complaint in the RTC. He did not even petitioner to pay the balance of the said amount to each of
We have reviewed the pleadings of the parties and found know that one of those whom he had impleaded as the defendants-tenants is an issue between them, the
that, as contended by the respondents, the petitioner defendant, Angelica Vda. de Lacson, was already dead. parties to the deed.
failed to establish the essential requisites for the issuance
of a writ of preliminary injunction. Hence, the trial court Q: But you have not met any of these Lacsons? Third. On the face of the complaint, the action of the
committed a grave abuse of its discretion amounting to petitioner against the respondents and the defendants-
excess or lack of jurisdiction in denying the respondents’ A: Not yet, sir. tenants has no legal basis. Under the Deeds of
comment/motion as well as their motion for Assignment, the obligation of the petitioner to pay to each
reconsideration. of the defendants-tenants the balance of the purchase
6
Sales – Chapter 3-12 Cases
price was conditioned on the occurrence of the following Q : Did you explain to them? or disapprove the same. In fact, as alleged by the
events: (a) the respondents agree to sell their property to petitioner in his complaint, he was yet to meet with the
the petitioner; (b) the legal impediments to the sale of the A : Yes, sir. defendants-tenants to discuss the implementation of the
landholding to the petitioner no longer exist; and, (c) the deeds of assignment. Unless and until the Department of
petitioner decides to buy the property. When he testified, Q : What did you tell them? Agrarian Reform approved the said deeds, if at all, the
the petitioner admitted that the legal impediments referred petitioner had no right to enforce the same in a court of
to in the deeds were (a) the respondents’ refusal to sell A : I explain[ed] to them, sir, that the legal impediment law by asking the trial court to fix a period within which to
their property; and, (b) the lack of approval of the then especially if the Lacsons will not agree to sell their pay the balance of the purchase price and praying for
Department of Agrarian Reform: shares to me or to us it would be hard to (sic) me to pay injunctive relief.
them in full. And those covered by DAR. I explain[ed] to
Q : There is no specific agreement prior to the execution them and it was clearly stated in the title that there is [a] We do not agree with the contention of the petitioner that
of those documents as when they will pay? prohibited period of time before you can sell the property. I the deeds of assignment executed by the defendants-
explained every detail to them.41 tenants are perfected option contracts.43 An option is a
A : We agreed to that, that I will pay them when there are contract by which the owner of the property agrees with
no legal impediment, sir. It is only upon the occurrence of the foregoing conditions another person that he shall have the right to buy his
that the petitioner would be obliged to pay to the property at a fixed price within a certain time. It is a
Q : Many of the documents are unlattered (sic) and you defendants-tenants the balance of the P50.00 per square condition offered or contract by which the owner stipulates
want to convey to this Honorable Court that prior to the meter under the deeds of assignment. Thus: with another that the latter shall have the right to buy the
execution of these documents you have those tentative property at a fixed price within a certain time, or under, or
agreement for instance that the amount or the cost of the 2. That in case the ASSIGNOR and LANDOWNER will in compliance with certain terms and conditions, or which
price is to be paid when there are no legal impediment, mutually agree to sell the said lot to the ASSIGNEE, who gives to the owner of the property the right to sell or
you are using the word "legal impediment," do you know is given an exclusive and absolute right to buy the lot, the demand a sale. It imposes no binding obligation on the
the meaning of that? ASSIGNOR shall receive the sum of FIFTY PESOS person holding the option, aside from the consideration for
(P50.00) per square meter as consideration of the total the offer. Until accepted, it is not, properly speaking,
A : When there are (sic) no more legal impediment exist, area actually tilled and possessed by the ASSIGNOR, treated as a contract.44 The second party gets in
sir. less whatever amount received by the ASSIGNOR praesenti, not lands, not an agreement that he shall have
including commissions, taxes and all allowable deductions the lands, but the right to call for and receive lands if he
Q : Did you make how (sic) to the effect that the meaning relative to the sale of the subject properties. elects.45 An option contract is a separate and distinct
of that phrase that you used the unlettered defendants? contract from which the parties may enter into upon the
3. That this exclusive and absolute right given to the conjunction of the option.46
A : We have agreed to that, sir. ASSIGNEE shall be exercised only when no legal
impediments exist to the lot to effect the smooth transfer In this case, the defendants-tenants-subtenants, under
ATTY. OCAMPO: of lawful ownership of the lot/property in the name of the the deeds of assignment, granted to the petitioner not only
ASSIGNEE; an option but the exclusive right to buy the landholding.
May I ask, Your Honor, that the witness please answer my But the grantors were merely the defendants-tenants, and
question not to answer in the way he wanted it. 4. That the ASSIGNOR will remain in peaceful possession not the respondents, the registered owners of the
over the said property and shall enjoy the fruits/earnings property. Not being the registered owners of the property,
COURT: and/or harvest of the said lot until such time that full the defendants-tenants could not legally grant to the
payment of the agreed purchase price had been made by petitioner the option, much less the "exclusive right" to buy
Just answer the question, Mr. Tayag. the ASSIGNEE.42 the property. As the Latin saying goes, "NEMO DAT
QUOD NON HABET."
WITNESS: There is no showing in the petitioner’s complaint that the
respondents had agreed to sell their property, and that the Fourth. The petitioner impleaded the respondents as
Yes, Your Honor. legal impediments to the agreement no longer existed. parties-defendants solely on his allegation that the latter
The petitioner and the defendants-tenants had yet to induced or are inducing the defendants-tenants to violate
ATTY. OCAMPO: submit the Deeds of Assignment to the Department of the deeds of assignment, contrary to the provisions of
Agrarian Reform which, in turn, had to act on and approve Article 1314 of the New Civil Code which reads:
7
Sales – Chapter 3-12 Cases
A: I heard and sometime in [the] first week of August, sir, Under Section 12 of the law, if the property was sold to a
Art. 1314. Any third person who induces another to violate they went in the barrio (sic). As a matter of fact, that is the third person without the knowledge of the tenants thereon,
his contract shall be liable for damages to the other reason why they sent me letter that they will sell it to the the latter shall have the right to redeem the same at a
contracting party. Lacsons. reasonable price and consideration. By assigning their
rights and interests on the landholding under the deeds of
In So Ping Bun v. Court of Appeals,47 we held that for the Q: Incidentally, do you knew (sic) these Lacsons assignment in favor of the petitioner, the defendants-
said law to apply, the pleader is burdened to prove the individually? tenants thereby waived, in favor of the petitioner, who is
following: (1) the existence of a valid contract; (2) not a beneficiary under Section 22 of Rep. Act No. 6657,
knowledge by the third person of the existence of the A: No, sir, it was only Mr. Espinosa who I knew (sic) their rights of preemption or redemption under Rep. Act
contract; and (3) interference by the third person in the personally, the alleged negotiator and has the authority to No. 3844. The defendants-tenants would then have to
contractual relation without legal justification. sell the property.50 vacate the property in favor of the petitioner upon full
payment of the purchase price. Instead of acquiring
Where there was no malice in the interference of a Even if the respondents received an offer from the ownership of the portions of the landholding respectively
contract, and the impulse behind one’s conduct lies in a defendants-tenants to assign and transfer their rights and tilled by them, the defendants-tenants would again
proper business interest rather than in wrongful motives, a interests on the landholding, the respondents cannot be become landless for a measly sum of P50.00 per square
party cannot be a malicious interferer. Where the alleged enjoined from entertaining the said offer, or even meter. The petitioner’s scheme is subversive, not only of
interferer is financially interested, and such interest negotiating with the defendants-tenants. The respondents public policy, but also of the letter and spirit of the agrarian
motivates his conduct, it cannot be said that he is an could not even be expected to warn the defendants- laws. That the scheme of the petitioner had yet to take
officious or malicious intermeddler.48 tenants for executing the said deeds in violation of P.D. effect in the future or ten years hence is not a justification.
No. 27 and Rep. Act No. 6657. Under Section 22 of the The respondents may well argue that the agrarian laws
In fine, one who is not a party to a contract and who latter law, beneficiaries under P.D. No. 27 who have had been violated by the defendants-tenants and the
interferes thereon is not necessarily an officious or culpably sold, disposed of, or abandoned their land, are petitioner by the mere execution of the deeds of
malicious intermeddler. The only evidence adduced by the disqualified from becoming beneficiaries. assignment. In fact, the petitioner has implemented the
petitioner to prove his claim is the letter from the deeds by paying the defendants-tenants amounts of
defendants-tenants informing him that they had decided to From the pleadings of the petitioner, it is quite evident that money and even sought their immediate implementation
sell their rights and interests over the landholding to the his purpose in having the defendants-tenants execute the by setting a meeting with the defendants-tenants. In fine,
respondents, instead of honoring their obligation under Deeds of Assignment in his favor was to acquire the the petitioner would not wait for ten years to evict the
the deeds of assignment because, according to them, the landholding without any tenants thereon, in the event that defendants-tenants. For him, time is of the essence.
petitioner harassed those tenants who did not want to the respondents agreed to sell the property to him. The
execute deeds of assignment in his favor, and because petitioner knew that under Section 11 of Rep. Act No. The Appellate Court Erred
the said defendants-tenants did not want to have any 3844, if the respondents agreed to sell the property, the In Permanently Enjoining
problem with the respondents who could cause their defendants-tenants shall have preferential right to buy the The Regional Trial Court
eviction for executing with the petitioner the deeds of same under reasonable terms and conditions: From Continuing with the
assignment as the said deeds are in violation of P.D. No. Proceedings in Civil Case No. 10910.
27 and Rep. Act No. 6657.49 The defendants-tenants did SECTION 11. Lessee’s Right of Pre-emption. – In case
not allege therein that the respondents induced them to the agricultural lessor desires to sell the landholding, the We agree with the petitioner’s contention that the
breach their contracts with the petitioner. The petitioner agricultural lessee shall have the preferential right to buy appellate court erred when it permanently enjoined the
himself admitted when he testified that his claim that the the same under reasonable terms and conditions: RTC from continuing with the proceedings in Civil Case
respondents induced the defendants-assignees to violate Provided, That the entire landholding offered for sale must No. 10910. The only issue before the appellate court was
contracts with him was based merely on what "he heard," be pre-empted by the Land Authority if the landowner so whether or not the trial court committed a grave abuse of
thus: desires, unless the majority of the lessees object to such discretion amounting to excess or lack of jurisdiction in
acquisition: Provided, further, That where there are two or denying the respondents’ motion to deny or dismiss the
Q: Going to your last statement that the Lacsons induces more agricultural lessees, each shall be entitled to said petitioner’s plea for a writ of preliminary injunction. Not
(sic) the defendants, did you see that the Lacsons were preferential right only to the extent of the area actually one of the parties prayed to permanently enjoin the trial
inducing the defendants? cultivated by him. …51 court from further proceeding with Civil Case No. 10910 or
to dismiss the complaint. It bears stressing that the
petitioner may still amend his complaint, and the
8
Sales – Chapter 3-12 Cases
respondents and the defendants-tenants may file motions
to dismiss the complaint. By permanently enjoining the
trial court from proceeding with Civil Case No. 10910, the
appellate court acted arbitrarily and effectively dismissed
the complaint motu proprio, including the counterclaims of
the respondents and that of the defendants-tenants. The
defendants-tenants were even deprived of their right to
prove their special and affirmative defenses.

IN LIGHT OF ALL THE FOREGOING, the petition is


PARTIALLY GRANTED. The Decision of the Court of
Appeals nullifying the February 13, 1996 and April 16,
1997 Orders of the RTC is AFFIRMED. The writ of
injunction issued by the Court of Appeals permanently
enjoining the RTC from further proceeding with Civil Case
No. 10910 is hereby LIFTED and SET ASIDE. The
Regional Trial Court of Mabalacat, Pampanga, Branch 44,
is ORDERED to continue with the proceedings in Civil
Case No. 10910 as provided for by the Rules of Court, as
amended.
SO ORDERED.

9
Sales – Chapter 3-12 Cases
ADELFA PROPERTIES, INC., petitioner, FIFTY SIX THOUSAND ONE HUNDRED FIFTY PESOS suggested that private respondents settle the case with
vs. ONLY (P2,856,150.00) their nephews and nieces, adding that ". . . if possible,
COURT OF APPEALS, ROSARIO JIMENEZ- although November 30, 1989 is a holiday, we will be
CASTAÑEDA and SALUD JIMENEZ, respondents. 2. The sum of P50,000.00 which we received from waiting for you and said plaintiffs at our office up to 7:00
ADELFA PROPERTIES, INC. as an option money shall p.m."8 Another letter of the same tenor and of even date
The main issues presented for resolution in this petition be credited as partial payment upon the consummation of was sent by petitioner to Jose and Dominador Jimenez.9
for review on certiorari of the judgment of respondent the sale and the balance in the sum of TWO MILLION Respondent Salud Jimenez refused to heed the
Court of appeals, dated April 6, 1993, in CA-G.R. CV No. EIGHT HUNDRED SIX THOUSAND ONE HUNDRED suggestion of petitioner and attributed the suspension of
347671 are (1) whether of not the "Exclusive Option to FIFTY PESOS (P2,806,150.00) to be paid on or before payment of the purchase price to "lack of word of honor."
Purchase" executed between petitioner Adelfa Properties, November 30, 1989;
Inc. and private respondents Rosario Jimenez-Castañeda 6. On December 7, 1989, petitioner caused to be
and Salud Jimenez is an option contract; and (2) whether 3. In case of default on the part of ADELFA annotated on the title of the lot its option contract with
or not there was a valid suspension of payment of the PROPERTIES, INC. to pay said balance in accordance private respondents, and its contract of sale with Jose and
purchase price by said petitioner, and the legal effects with paragraph 2 hereof, this option shall be cancelled Dominador Jimenez, as Entry No. 1437-4 and entry No.
thereof on the contractual relations of the parties. and 50% of the option money to be forfeited in our favor 1438-4, respectively.
and we will refund the remaining 50% of said money upon
The records disclose the following antecedent facts which the sale of said property to a third party; 7. On December 14, 1989, private respondents
culminated in the present appellate review, to wit: sent Francisca Jimenez to see Atty. Bernardo, in his
4. All expenses including the corresponding capital capacity as petitioner's counsel, and to inform the latter
1. Herein private respondents and their brothers, gains tax, cost of documentary stamps are for the account that they were cancelling the transaction. In turn, Atty.
Jose and Dominador Jimenez, were the registered co- of the VENDORS, and expenses for the registration of the Bernardo offered to pay the purchase price provided that
owners of a parcel of land consisting of 17,710 square deed of sale in the Registry of Deeds are for the account P500,000.00 be deducted therefrom for the settlement of
meters, covered by Transfer Certificate of Title (TCT) No. of ADELFA PROPERTIES, INC. the civil case. This was rejected by private respondents.
309773,2 situated in Barrio Culasi, Las Piñas, Metro On December 22, 1989, Atty. Bernardo wrote private
Manila. Considering, however, that the owner's copy of the respondents on the same matter but this time reducing the
certificate of title issued to respondent Salud Jimenez had amount from P500,000.00 to P300,000.00, and this was
2. On July 28, 1988, Jose and Dominador Jimenez been lost, a petition for the re-issuance of a new owner's also rejected by the latter.
sold their share consisting of one-half of said parcel of copy of said certificate of title was filed in court through
land, specifically the eastern portion thereof, to herein Atty. Bayani L. Bernardo, who acted as private 8. On February 23, 1990, the Regional Trial Court
petitioner pursuant to a "Kasulatan sa Bilihan ng Lupa."3 respondents' counsel. Eventually, a new owner's copy of of Makati dismissed Civil Case No. 89-5541. Thus, on
Subsequently, a "Confirmatory Extrajudicial Partition the certificate of title was issued but it remained in the February 28, 1990, petitioner caused to be annotated
Agreement"4 was executed by the Jimenezes, wherein possession of Atty. Bernardo until he turned it over to anew on TCT No. 309773 the exclusive option to
the eastern portion of the subject lot, with an area of 8,855 petitioner Adelfa Properties, Inc. purchase as Entry No. 4442-4.
square meters was adjudicated to Jose and Dominador
Jimenez, while the western portion was allocated to herein 4. Before petitioner could make payment, it 9. On the same day, February 28, 1990, private
private respondents. received summons6 on November 29, 1989, together with respondents executed a Deed of Conditional Sale 10 in
a copy of a complaint filed by the nephews and nieces of favor of Emylene Chua over the same parcel of land for
3. Thereafter, herein petitioner expressed interest in private respondents against the latter, Jose and P3,029,250, of which P1,500,000.00 was paid to private
buying the western portion of the property from private Dominador Jimenez, and herein petitioner in the Regional respondents on said date, with the balance to be paid
respondents. Accordingly, on November 25, 1989, an Trial Court of Makati, docketed as Civil Case No. 89-5541, upon the transfer of title to the specified one-half portion.
"Exclusive Option to Purchase"5 was executed between for annulment of the deed of sale in favor of Household
petitioner and private respondents, under the following Corporation and recovery of ownership of the property 10. On April 16, 1990, Atty. Bernardo wrote private
terms and conditions: covered by TCT No. 309773.7 respondents informing the latter that in view of the
dismissal of the case against them, petitioner was willing
1. The selling price of said 8,655 square meters of 5. As a consequence, in a letter dated November to pay the purchase price, and he requested that the
the subject property is TWO MILLION EIGHT HUNDRED 29, 1989, petitioner informed private respondents that it corresponding deed of absolute sale be executed. 11 This
would hold payment of the full purchase price and was ignored by private respondents.
10
Sales – Chapter 3-12 Cases
or a contract to sell, but not to an option contract which it contract to sell, by agreement the ownership is reserved in
11. On July 27, 1990, private respondents' counsel opined was the nature of the document subject of the the vendor and is not to pass until the full payment of the
sent a letter to petitioner enclosing therein a check for case at bar. Said appellate court similarly upheld the price. In a contract of sale, the vendor has lost and cannot
P25,000.00 representing the refund of fifty percent of the validity of the deed of conditional sale executed by private recover ownership until and unless the contract is
option money paid under the exclusive option to respondents in favor of intervenor Emylene Chua. resolved or rescinded; whereas in a contract to sell, title is
purchase. Private respondents then requested petitioner retained by the vendor until the full payment of the price,
to return the owner's duplicate copy of the certificate of In the present petition, the following assignment of errors such payment being a positive suspensive condition and
title of respondent Salud Jimenez. 12 Petitioner failed to are raised: failure of which is not a breach but an event that prevents
surrender the certificate of title, hence private respondents the obligation of the vendor to convey title from becoming
filed Civil Case No. 7532 in the Regional Trial Court of 1. Respondent court of appeals acted with grave effective. Thus, a deed of sale is considered absolute in
Pasay City, Branch 113, for annulment of contract with abuse of discretion in making its finding that the nature where there is neither a stipulation in the deed that
damages, praying, among others, that the exclusive agreement entered into by petitioner and private title to the property sold is reserved in the seller until the
option to purchase be declared null and void; that respondents was strictly an option contract; full payment of the price, nor one giving the vendor the
defendant, herein petitioner, be ordered to return the right to unilaterally resolve the contract the moment the
owner's duplicate certificate of title; and that the 2. Granting arguendo that the agreement was an buyer fails to pay within a fixed period. 15
annotation of the option contract on TCT No. 309773 be option contract, respondent court of Appeals acted with
cancelled. Emylene Chua, the subsequent purchaser of grave abuse of discretion in grievously failing to consider There are two features which convince us that the parties
the lot, filed a complaint in intervention. that while the option period had not lapsed, private never intended to transfer ownership to petitioner except
respondents could not unilaterally and prematurely upon the full payment of the purchase price. Firstly, the
12. The trial court rendered judgment 13 therein on terminate the option period; exclusive option to purchase, although it provided for
September 5, 1991 holding that the agreement entered automatic rescission of the contract and partial forfeiture
into by the parties was merely an option contract, and 3. Respondent Court of Appeals acted with grave of the amount already paid in case of default, does not
declaring that the suspension of payment by herein abuse of discretion in failing to appreciate fully the mention that petitioner is obliged to return possession or
petitioner constituted a counter-offer which, therefore, was attendant facts and circumstances when it made the ownership of the property as a consequence of non-
tantamount to a rejection of the option. It likewise ruled conclusion of law that Article 1590 does not apply; and payment. There is no stipulation anent reversion or
that herein petitioner could not validly suspend payment in reconveyance of the property to herein private
favor of private respondents on the ground that the 4. Respondent Court of Appeals acted with grave respondents in the event that petitioner does not comply
vindicatory action filed by the latter's kin did not involve abuse of discretion in conforming with the sale in favor of with its obligation. With the absence of such a stipulation,
the western portion of the land covered by the contract appellee Ma. Emylene Chua and the award of damages although there is a provision on the remedies available to
between petitioner and private respondents, but the and attorney's fees which are not only excessive, but also the parties in case of breach, it may legally be inferred
eastern portion thereof which was the subject of the sale without in fact and in law. 14 that the parties never intended to transfer ownership to
between petitioner and the brothers Jose and Dominador the petitioner to completion of payment of the purchase
Jimenez. The trial court then directed the cancellation of An analysis of the facts obtaining in this case, as well as price.
the exclusive option to purchase, declared the sale to the evidence presented by the parties, irresistibly leads to
intervenor Emylene Chua as valid and binding, and the conclusion that the agreement between the parties is In effect, there was an implied agreement that ownership
ordered petitioner to pay damages and attorney's fees to a contract to sell, and not an option contract or a contract shall not pass to the purchaser until he had fully paid the
private respondents, with costs. of sale. price. Article 1478 of the civil code does not require that
such a stipulation be expressly made. Consequently, an
13. On appeal, respondent Court of appeals affirmed I implied stipulation to that effect is considered valid and,
in toto the decision of the court a quo and held that the therefore, binding and enforceable between the parties. It
failure of petitioner to pay the purchase price within the 1. In view of the extended disquisition thereon by should be noted that under the law and jurisprudence, a
period agreed upon was tantamount to an election by respondent court, it would be worthwhile at this juncture to contract which contains this kind of stipulation is
petitioner not to buy the property; that the suspension of briefly discourse on the rationale behind our treatment of considered a contract to sell.
payment constituted an imposition of a condition which the alleged option contract as a contract to sell, rather
was actually a counter-offer amounting to a rejection of than a contract of sale. The distinction between the two is Moreover, that the parties really intended to execute a
the option; and that Article 1590 of the Civil Code on important for in contract of sale, the title passes to the contract to sell, and not a contract of sale, is bolstered by
suspension of payments applies only to a contract of sale vendee upon the delivery of the thing sold; whereas in a the fact that the deed of absolute sale would have been
11
Sales – Chapter 3-12 Cases
issued only upon the payment of the balance of the that the document under discussion is entitled "Exclusive bound by his offer, and the option is at an end. A contract
purchase price, as may be gleaned from petitioner's letter Option to Purchase" is not controlling where the text of sale, on the other hand, fixes definitely the relative
dated April 16, 1990 16 wherein it informed private thereof shows that it is a contract to sell. rights and obligations of both parties at the time of its
respondents that it "is now ready and willing to pay you execution. The offer and the acceptance are concurrent,
simultaneously with the execution of the corresponding An option, as used in the law on sales, is a continuing since the minds of the contracting parties meet in the
deed of absolute sale." offer or contract by which the owner stipulates with terms of the agreement. 29
another that the latter shall have the right to buy the
Secondly, it has not been shown there was delivery of the property at a fixed price within a certain time, or under, or A perusal of the contract in this case, as well as the oral
property, actual or constructive, made to herein petitioner. in compliance with, certain terms and conditions, or which and documentary evidence presented by the parties,
The exclusive option to purchase is not contained in a gives to the owner of the property the right to sell or readily shows that there is indeed a concurrence of
public instrument the execution of which would have been demand a sale. It is also sometimes called an petitioner's offer to buy and private respondents'
considered equivalent to delivery. 17 Neither did petitioner "unaccepted offer." An option is not of itself a purchase, acceptance thereof. The rule is that except where a formal
take actual, physical possession of the property at any but merely secures the privilege to buy. 22 It is not a sale acceptance is so required, although the acceptance must
given time. It is true that after the reconstitution of private of property but a sale of property but a sale of the right to be affirmatively and clearly made and must be evidenced
respondents' certificate of title, it remained in the purchase. 23 It is simply a contract by which the owner of by some acts or conduct communicated to the offeror, it
possession of petitioner's counsel, Atty. Bayani L. property agrees with another person that he shall have the may be made either in a formal or an informal manner,
Bernardo, who thereafter delivered the same to herein right to buy his property at a fixed price within a certain and may be shown by acts, conduct, or words of the
petitioner. Normally, under the law, such possession by time. He does not sell his land; he does not then agree to accepting party that clearly manifest a present intention or
the vendee is to be understood as a delivery.18 However, sell it; but he does sell something, that it is, the right or determination to accept the offer to buy or sell. Thus,
private respondents explained that there was really no privilege to buy at the election or option of the other party. acceptance may be shown by the acts, conduct, or words
intention on their part to deliver the title to herein petitioner 24 Its distinguishing characteristic is that it imposes no of a party recognizing the existence of the contract of sale.
with the purpose of transferring ownership to it. They binding obligation on the person holding the option, aside 30
claim that Atty. Bernardo had possession of the title only from the consideration for the offer. Until acceptance, it is
because he was their counsel in the petition for not, properly speaking, a contract, and does not vest, The records also show that private respondents accepted
reconstitution. We have no reason not to believe this transfer, or agree to transfer, any title to, or any interest or the offer of petitioner to buy their property under the terms
explanation of private respondents, aside from the fact right in the subject matter, but is merely a contract by of their contract. At the time petitioner made its offer,
that such contention was never refuted or contradicted by which the owner of property gives the optionee the right or private respondents suggested that their transfer
petitioner. privilege of accepting the offer and buying the property on certificate of title be first reconstituted, to which petitioner
certain terms. 25 agreed. As a matter of fact, it was petitioner's counsel,
2. Irrefragably, the controverted document should Atty. Bayani L. Bernardo, who assisted private
legally be considered as a perfected contract to sell. On On the other hand, a contract, like a contract to sell, respondents in filing a petition for reconstitution. After the
this particular point, therefore, we reject the position and involves a meeting of minds two persons whereby one title was reconstituted, the parties agreed that petitioner
ratiocination of respondent Court of Appeals which, while binds himself, with respect to the other, to give something would pay either in cash or manager's check the amount
awarding the correct relief to private respondents, or to render some service. 26 Contracts, in general, are of P2,856,150.00 for the lot. Petitioner was supposed to
categorized the instrument as "strictly an option contract." perfected by mere consent, 27 which is manifested by the pay the same on November 25, 1989, but it later offered
meeting of the offer and the acceptance upon the thing to make a down payment of P50,000.00, with the balance
The important task in contract interpretation is always the and the cause which are to constitute the contract. The of P2,806,150.00 to be paid on or before November 30,
ascertainment of the intention of the contracting parties offer must be certain and the acceptance absolute. 28 1989. Private respondents agreed to the counter-offer
and that task is, of course, to be discharged by looking to made by petitioner. 31 As a result, the so-called exclusive
the words they used to project that intention in their The distinction between an "option" and a contract of sale option to purchase was prepared by petitioner and was
contract, all the words not just a particular word or two, is that an option is an unaccepted offer. It states the terms subsequently signed by private respondents, thereby
and words in context not words standing alone. 19 and conditions on which the owner is willing to sell the creating a perfected contract to sell between them.
Moreover, judging from the subsequent acts of the parties land, if the holder elects to accept them within the time
which will hereinafter be discussed, it is undeniable that limited. If the holder does so elect, he must give notice to It cannot be gainsaid that the offer to buy a specific piece
the intention of the parties was to enter into a contract to the other party, and the accepted offer thereupon of land was definite and certain, while the acceptance
sell. 20 In addition, the title of a contract does not becomes a valid and binding contract. If an acceptance is thereof was absolute and without any condition or
necessarily determine its true nature. 21 Hence, the fact not made within the time fixed, the owner is no longer qualification. The agreement as to the object, the price of
12
Sales – Chapter 3-12 Cases
the property, and the terms of payment was clear and herein petitioner. with the arrival of the period agreed
well-defined. No other significance could be given to such upon by the parties, petitioner was supposed to comply While there is jurisprudence to the effect that a contract
acts that than they were meant to finalize and perfect the with the obligation incumbent upon it to perform, not which provides that the initial payment shall be totally
transaction. The parties even went beyond the basic merely to exercise an option or a right to buy the property. forfeited in case of default in payment is to be considered
requirements of the law by stipulating that "all expenses as an option contract, 37 still we are not inclined to
including the corresponding capital gains tax, cost of The obligation of petitioner on November 30, 1993 conform with the findings of respondent court and the
documentary stamps are for the account of the vendors, consisted of an obligation to give something, that is, the court a quo that the contract executed between the parties
and expenses for the registration of the deed of sale in the payment of the purchase price. The contract did not is an option contract, for the reason that the parties were
Registry of Deeds are for the account of Adelfa properties, simply give petitioner the discretion to pay for the already contemplating the payment of the balance of the
Inc." Hence, there was nothing left to be done except the property. 32 It will be noted that there is nothing in the purchase price, and were not merely quoting an agreed
performance of the respective obligations of the parties. said contract to show that petitioner was merely given a value for the property. The term "balance," connotes a
certain period within which to exercise its privilege to buy. remainder or something remaining from the original total
We do not subscribe to private respondents' submission, The agreed period was intended to give time to herein sum already agreed upon.
which was upheld by both the trial court and respondent petitioner within which to fulfill and comply with its
court of appeals, that the offer of petitioner to deduct obligation, that is, to pay the balance of the purchase In other words, the alleged option money of P50,000.00
P500,000.00, (later reduced to P300,000.00) from the price. No evidence was presented by private respondents was actually earnest money which was intended to form
purchase price for the settlement of the civil case was to prove otherwise. part of the purchase price. The amount of P50,000.00 was
tantamount to a counter-offer. It must be stressed that not distinct from the cause or consideration for the sale of
there already existed a perfected contract between the The test in determining whether a contract is a "contract of the property, but was itself a part thereof. It is a statutory
parties at the time the alleged counter-offer was made. sale or purchase" or a mere "option" is whether or not the rule that whenever earnest money is given in a contract of
Thus, any new offer by a party becomes binding only agreement could be specifically enforced. 33 There is no sale, it shall be considered as part of the price and as
when it is accepted by the other. In the case of private doubt that the obligation of petitioner to pay the purchase proof of the perfection of the contract. 38 It constitutes an
respondents, they actually refused to concur in said offer price is specific, definite and certain, and consequently advance payment and must, therefore, be deducted from
of petitioner, by reason of which the original terms of the binding and enforceable. Had private respondents chosen the total price. Also, earnest money is given by the buyer
contract continued to be enforceable. to enforce the contract, they could have specifically to the seller to bind the bargain.
compelled petitioner to pay the balance of P2,806,150.00.
At any rate, the same cannot be considered a counter- This is distinctly made manifest in the contract itself as an There are clear distinctions between earnest money and
offer for the simple reason that petitioner's sole purpose integral stipulation, compliance with which could legally option money, viz.: (a) earnest money is part of the
was to settle the civil case in order that it could already and definitely be demanded from petitioner as a purchase price, while option money ids the money given
comply with its obligation. In fact, it was even indicative of consequence. as a distinct consideration for an option contract; (b)
a desire by petitioner to immediately comply therewith, earnest money is given only where there is already a sale,
except that it was being prevented from doing so because This is not a case where no right is as yet created nor an while option money applies to a sale not yet perfected;
of the filing of the civil case which, it believed in good faith, obligation declared, as where something further remains and (c) when earnest money is given, the buyer is bound
rendered compliance improbable at that time. In addition, to be done before the buyer and seller obligate to pay the balance, while when the would-be buyer gives
no inference can be drawn from that suggestion given by themselves. 34 An agreement is only an "option" when no option money, he is not required to buy. 39
petitioner that it was totally abandoning the original obligation rests on the party to make any payment except
contract. such as may be agreed on between the parties as The aforequoted characteristics of earnest money are
consideration to support the option until he has made up apparent in the so-called option contract under review,
More importantly, it will be noted that the failure of his mind within the time specified. 35 An option, and not a even though it was called "option money" by the parties.
petitioner to pay the balance of the purchase price within contract to purchase, is effected by an agreement to sell In addition, private respondents failed to show that the
the agreed period was attributed by private respondents to real estate for payments to be made within specified time payment of the balance of the purchase price was only a
"lack of word of honor" on the part of the former. The and providing forfeiture of money paid upon failure to condition precedent to the acceptance of the offer or to
reason of "lack of word of honor" is to us a clear indication make payment, where the purchaser does not agree to the exercise of the right to buy. On the contrary, it has
that private respondents considered petitioner already purchase, to make payment, or to bind himself in any way been sufficiently established that such payment was but
bound by its obligation to pay the balance of the other than the forfeiture of the payments made. 36 As an element of the performance of petitioner's obligation
consideration. In effect, private respondents were hereinbefore discussed, this is not the situation obtaining under the contract to sell. 40
demanding or exacting fulfillment of the obligation from in the case at bar.
13
Sales – Chapter 3-12 Cases
II courts, did their claim pertain exclusively to the eastern not necessary because these cases involve an exercise of
half adjudicated to the Jimenez brothers. a right or privilege (to buy, redeem or repurchase) rather
1. This brings us to the second issue as to whether than the discharge of an obligation, hence tender of
or not there was valid suspension of payment of the Such being the case, petitioner was justified in payment would be sufficient to preserve the right or
purchase price by petitioner and the legal consequences suspending payment of the balance of the purchase price privilege. This is because the provisions on consignation
thereof. To justify its failure to pay the purchase price by reason of the aforesaid vindicatory action filed against are not applicable when there is no obligation to pay. 47 A
within the agreed period, petitioner invokes Article 1590 of it. The assurance made by private respondents that contract to sell, as in the case before us, involves the
the civil Code which provides: petitioner did not have to worry about the case because it performance of an obligation, not merely the exercise of a
was pure and simple harassment 42 is not the kind of privilege of a right. consequently, performance or payment
Art. 1590. Should the vendee be disturbed in the guaranty contemplated under the exceptive clause in may be effected not by tender of payment alone but by
possession or ownership of the thing acquired, or should Article 1590 wherein the vendor is bound to make both tender and consignation.
he have reasonable grounds to fear such disturbance, by payment even with the existence of a vindicatory action if
a vindicatory action or a foreclosure of mortgage, he may the vendee should give a security for the return of the Furthermore, petitioner no longer had the right to suspend
suspend the payment of the price until the vendor has price. payment after the disturbance ceased with the dismissal
caused the disturbance or danger to cease, unless the of the civil case filed against it. Necessarily, therefore, its
latter gives security for the return of the price in a proper 2. Be that as it may, and the validity of the obligation to pay the balance again arose and resumed
case, or it has been stipulated that, notwithstanding any suspension of payment notwithstanding, we find and hold after it received notice of such dismissal. Unfortunately,
such contingency, the vendee shall be bound to make the that private respondents may no longer be compelled to petitioner failed to seasonably make payment, as in fact it
payment. A mere act of trespass shall not authorize the sell and deliver the subject property to petitioner for two has deposit the money with the trial court when this case
suspension of the payment of the price. reasons, that is, petitioner's failure to duly effect the was originally filed therein.
consignation of the purchase price after the disturbance
Respondent court refused to apply the aforequoted had ceased; and, secondarily, the fact that the contract to By reason of petitioner's failure to comply with its
provision of law on the erroneous assumption that the true sell had been validly rescinded by private respondents. obligation, private respondents elected to resort to and did
agreement between the parties was a contract of option. announce the rescission of the contract through its letter
As we have hereinbefore discussed, it was not an option The records of this case reveal that as early as February to petitioner dated July 27, 1990. That written notice of
contract but a perfected contract to sell. Verily, therefore, 28, 1990 when petitioner caused its exclusive option to be rescission is deemed sufficient under the circumstances.
Article 1590 would properly apply. annotated anew on the certificate of title, it already knew Article 1592 of the Civil Code which requires rescission
of the dismissal of civil Case No. 89-5541. However, it either by judicial action or notarial act is not applicable to a
Both lower courts, however, are in accord that since Civil was only on April 16, 1990 that petitioner, through its contract to sell. 48 Furthermore, judicial action for
Case No. 89-5541 filed against the parties herein involved counsel, wrote private respondents expressing its rescission of a contract is not necessary where the
only the eastern half of the land subject of the deed of willingness to pay the balance of the purchase price upon contract provides for automatic rescission in case of
sale between petitioner and the Jimenez brothers, it did the execution of the corresponding deed of absolute sale. breach,49 as in the contract involved in the present
not, therefore, have any adverse effect on private At most, that was merely a notice to pay. There was no controversy.
respondents' title and ownership over the western half of proper tender of payment nor consignation in this case as
the land which is covered by the contract subject of the required by law. We are not unaware of the ruling in University of the
present case. We have gone over the complaint for Philippines vs. De los Angeles, etc. 50 that the right to
recovery of ownership filed in said case 41 and we are not The mere sending of a letter by the vendee expressing the rescind is not absolute, being ever subject to scrutiny and
persuaded by the factual findings made by said courts. At intention to review by the proper court. It is our considered view,
a glance, it is easily discernible that, although the pay, without the accompanying payment, is not however, that this rule applies to a situation where the
complaint prayed for the annulment only of the contract of considered a valid tender of payment. 43 Besides, a mere extrajudicial rescission is contested by the defaulting
sale executed between petitioner and the Jimenez tender of payment is not sufficient to compel private party. In other words, resolution of reciprocal contracts
brothers, the same likewise prayed for the recovery of respondents to deliver the property and execute the deed may be made extrajudicially unless successfully impugned
therein plaintiffs' share in that parcel of land specifically of absolute sale. It is consignation which is essential in in court. If the debtor impugns the declaration, it shall be
covered by TCT No. 309773. In other words, the plaintiffs order to extinguish petitioner's obligation to pay the subject to judicial determination51 otherwise, if said party
therein were claiming to be co-owners of the entire parcel balance of the purchase price. 44 The rule is different in does not oppose it, the extrajudicial rescission shall have
of land described in TCT No. 309773, and not only of a case of an option contract 45 or in legal redemption or in a legal effect. 52
portion thereof nor, as incorrectly interpreted by the lower sale with right to repurchase, 46 wherein consignation is
14
Sales – Chapter 3-12 Cases
In the case at bar, it has been shown that although
petitioner was duly furnished and did receive a written
notice of rescission which specified the grounds therefore,
it failed to reply thereto or protest against it. Its silence
thereon suggests an admission of the veracity and validity
of private respondents' claim. 53 Furthermore, the
initiative of instituting suit was transferred from the
rescinder to the defaulter by virtue of the automatic
rescission clause in the contract. 54 But then, the records
bear out the fact that aside from the lackadaisical manner
with which petitioner treated private respondents' latter of
cancellation, it utterly failed to seriously seek redress from
the court for the enforcement of its alleged rights under
the contract. If private respondents had not taken the
initiative of filing Civil Case No. 7532, evidently petitioner
had no intention to take any legal action to compel specific
performance from the former. By such cavalier disregard,
it has been effectively estopped from seeking the
affirmative relief it now desires but which it had theretofore
disdained.

WHEREFORE, on the foregoing modificatory premises,


and considering that the same result has been reached by
respondent Court of Appeals with respect to the relief
awarded to private respondents by the court a quo which
we find to be correct, its assailed judgment in CA-G.R. CV
No. 34767 is hereby AFFIRMED.

SO ORDERED.

15
Sales – Chapter 3-12 Cases
SPOUSES JULIO D. VILLAMOR AND MARINA VILLAMOR, That, I Macaria Labingisa, am the owner in fee simple of a ROSALINDA S. EUGENIO
petitioners, parcel of land with an area of 600 square meters, more or
vs. less, more particularly described in TCT No. (18431) 18938 of ACKNOWLEDGMENT
THE HON. COURT OF APPEALS AND SPOUSES the Office of the Register of Deeds for the province of Rizal,
MACARIA LABINGISA REYES AND ROBERTO REYES, issued in may name, I having inherited the same from my REPUBLIC OF THE PHILIPPINES)
respondents. deceased parents, for which reason it is my paraphernal CITY OF MANILA ) S.S.
property;
This is a petition for review on certiorari of the decision of the At the City of Manila, on the 11th day of November, 1971,
Court of Appeals in CA-G.R. No. 24176 entitled, "Spouses That I, with the conformity of my husband, Roberto Reyes, personally appeared before me Roberto Reyes, Macaria
Julio Villamor and Marina Villamor, Plaintiffs-Appellees, have sold one-half thereof to the aforesaid spouses Julio Labingisa, Julio Villamor and Marina Ventura-Villamor, known
versus Spouses Macaria Labing-isa Reyes and Roberto Villamor and Marina V. Villamor at the price of P70.00 per sq. to me as the same persons who executed the foregoing Deed
Reyes, Defendants-Appellants," which reversed the decision meter, which was greatly higher than the actual reasonable of Option, which consists of two (2) pages including the page
of the Regional Trial Court (Branch 121) at Caloocan City in prevailing value of lands in that place at the time, which whereon this acknowledgement is written, and signed at the
Civil Case No. C-12942. portion, after segregation, is now covered by TCT No. 39935 left margin of the first page and at the bottom of the
of the Register of Deeds for the City of Caloocan, issued on instrument by the parties and their witnesses, and sealed with
The facts of the case are as follows: August 17, 1971 in the name of the aforementioned spouses my notarial seal, and said parties acknowledged to me that
vendees; the same is their free act and deed. The Residence
Macaria Labingisa Reyes was the owner of a 600-square Certificates of the parties were exhibited to me as follows:
meter lot located at Baesa, Caloocan City, as evidenced by That the only reason why the Spouses-vendees Julio Villamor Roberto Reyes, A-22494, issued at Manila on Jan. 27, 1971,
Transfer Certificate of Title No. (18431) 18938, of the and Marina V. Villamor, agreed to buy the said one-half and B-502025, issued at Makati, Rizal on Feb. 18, 1971;
Register of Deeds of Rizal. portion at the above-stated price of about P70.00 per square Macaria Labingisa, A-3339130 and B-1266104, both issued
meter, is because I, and my husband Roberto Reyes, have at Caloocan City on April 15, 1971, their joint Tax Acct.
In July 1971, Macaria sold a portion of 300 square meters of agreed to sell and convey to them the remaining one-half Number being 3028-767-6; Julio Villamor, A-804, issued at
the lot to the Spouses Julio and Marina and Villamor for the portion still owned by me and now covered by TCT No. 39935 Manila on Jan. 14, 1971, and B-138, issued at Manila on
total amount of P21,000.00. Earlier, Macaria borrowed of the Register of Deeds for the City of Caloocan, whenever March 1, 1971; and Marina Ventura-Villamor, A-803, issued
P2,000.00 from the spouses which amount was deducted the need of such sale arises, either on our part or on the part at Manila on Jan. 14, 1971, their joint Tax Acct. Number
from the total purchase price of the 300 square meter lot sold. of the spouses (Julio) Villamor and Marina V. Villamor, at the being 608-202-6.
The portion sold to the Villamor spouses is now covered by same price of P70.00 per square meter, excluding whatever
TCT No. 39935 while the remaining portion which is still in the improvement may be found the thereon; ARTEMIO M. MALUBAY
name of Macaria Labing-isa is covered by TCT No. 39934 Notary Public
(pars. 5 and 7, Complaint). On November 11, 1971, Macaria That I am willing to have this contract to sell inscribed on my Until December 31, 1972
executed a "Deed of Option" in favor of Villamor in which the aforesaid title as an encumbrance upon the property covered PTR No. 338203, Manila
remaining 300 square meter portion (TCT No. 39934) of the thereby, upon payment of the corresponding fees; and January 15, 1971
lot would be sold to Villamor under the conditions stated
therein. The document reads: That we, Julio Villamor and Marina V. Villamor, hereby agree Doc. No. 1526;
to, and accept, the above provisions of this Deed of Option. Page No. 24;
DEED OF OPTION Book No. 38;
IN WITNESS WHEREOF, this Deed of Option is signed in the Series of 1971. (pp. 25-29, Rollo)
This Deed of Option, entered into in the City of Manila, City of Manila, Philippines, by all the persons concerned, this
Philippines, this 11th day of November, 1971, by and 11th day of November, 1971. According to Macaria, when her husband, Roberto Reyes,
between Macaria Labing-isa, of age, married to Roberto retired in 1984, they offered to repurchase the lot sold by
Reyes, likewise of age, and both resideing on Reparo St., JULIO VILLAMOR MACARIA LABINGISA them to the Villamor spouses but Marina Villamor refused and
Baesa, Caloocan City, on the one hand, and on the other reminded them instead that the Deed of Option in fact gave
hand the spouses Julio Villamor and Marina V. Villamor, also With My Conformity: them the option to purchase the remaining portion of the lot.
of age and residing at No. 552 Reparo St., corner Baesa
Road, Baesa, Caloocan City. MARINA VILLAMOR ROBERTO REYES The Villamors, on the other hand, claimed that they had
expressed their desire to purchase the remaining 300 square
WITNESSETH Signed in the Presence Of: meter portion of the lot but the Reyeses had been ignoring
them. Thus, on July 13, 1987, after conciliation proceedings
MARIANO Z. SUNIGA
16
Sales – Chapter 3-12 Cases
in the barangay level failed, they filed a complaint for specific 5. FAILING TO CONSIDER THAT EQUITABLE The pivotal issue to be resolved in this case is the validity of
performance against the Reyeses. CONSIDERATION TILT IN FAVOR OF THE DEFENDANT- the Deed of Option whereby the private respondents agreed
APPELLANTS; and to sell their lot to petitioners "whenever the need of such sale
On July 26, 1989, judgment was rendered by the trial court in arises, either on our part (private respondents) or on the part
favor of the Villamor spouses, the dispositive portion of which 6. HOLDING DEFENDANT-APPELLANTS LIABLE TO of Julio Villamor and Marina Villamor (petitioners)." The court
states: PAY PLAINTIFF-APPELLEES THE AMOUNT OF P3,000.00 a quo, rule that the Deed of Option was a valid written
FOR AND BY WAY OF ATTORNEY'S FEES. (pp. 31-32, agreement between the parties and made the following
WHEREFORE, and (sic) in view of the foregoing, judgment is Rollo) conclusions:
hereby rendered in favor of the plaintiffs and against the
defendants ordering the defendant MACARIA LABING-ISA On February 12, 1991, the Court of Appeals rendered a xxx xxx xxx
REYES and ROBERTO REYES, to sell unto the plaintiffs the decision reversing the decision of the trial court and
land covered by T.C.T No. 39934 of the Register of Deeds of dismissing the complaint. The reversal of the trial court's It is interesting to state that the agreement between the
Caloocan City, to pay the plaintiffs the sum of P3,000.00 as decision was premised on the finding of respondent court that parties are evidence by a writing, hence, the controverting
and for attorney's fees and to pay the cost of suit. the Deed of Option is void for lack of consideration. oral testimonies of the herein defendants cannot be any
better than the documentary evidence, which, in this case, is
The counterclaim is hereby DISMISSED, for LACK OF The Villamor spouses brought the instant petition for review the Deed of Option (Exh. "A" and "A-a")
MERIT. on certiorari on the following grounds:
The law provides that when the terms of an agreement have
SO ORDERED. (pp. 24-25, Rollo) I. THE COURT OF APPEALS GRAVELY ERRED IN been reduced to writing it is to be considered as containing all
FINDING THAT THE PHRASE WHENEVER THE NEED such terms, and therefore, there can be, between the parties
Not satisfied with the decision of the trial court, the Reyes FOR SUCH SALE ARISES ON OUR (PRIVATE and their successors in interest no evidence of their terms of
spouses appealed to the Court of Appeals on the following RESPONDENT) PART OR ON THE PART OF THE the agreement, other than the contents of the writing. ...
assignment of errors: SPOUSES JULIO D. VILLAMOR AND MARINA V. (Section 7 Rule 130 Revised Rules of Court) Likewise, it is a
VILLAMOR' CONTAINED IN THE DEED OF OPTION general and most inflexible rule that wherever written
1. HOLDING THAT THE DEED OF OPTION DENOTES A SUSPENSIVE CONDITION; instruments are appointed either by the requirements of law,
EXECUTED ON NOVEMBER 11, 1971 BETWEEN THE or by the contract of the parties, to be the repositories and
PLAINTIFF-APPELLEES AND DEFENDANT-APPELLANTS II. ASSUMING FOR THE SAKE OF ARGUMENT memorials of truth, any other evidence is excluded from being
IS STILL VALID AND BINDING DESPITE THE LAPSE OF THAT THE QUESTIONED PHRASE IS INDEED A used, either as a substitute for such instruments, or to
MORE THAN THIRTEEN (13) YEARS FROM THE CONDITION, THE COURT OF APPEALS ERRED IN NOT contradict or alter them. This is a matter both of principle and
EXECUTION OF THE CONTRACT; FINDING, THAT THE SAID CONDITION HAD ALREADY of policy; of principle because such instruments are in their
BEEN FULFILLED; nature and origin entitled to a much higher degree of credit
2. FAILING TO CONSIDER THAT THE DEED OF than evidence of policy, because it would be attended with
OPTION CONTAINS OBSCURE WORDS AND III. ASSUMING FOR THE SAKE OF ARGUMENT great mischief if those instruments upon which man's rights
STIPULATIONS WHICH SHOULD BE RESOLVED AGAINST THAT THE QUESTIONED PHRASE IS INDEED A depended were liable to be impeached by loose collateral
THE PLAINTIFF-APPELLEES WHO UNILATERALLY CONDITION, THE COURT OF APPEALS ERRED IN evidence. Where the terms of an agreement are reduced to
DRAFTED AND PREPARED THE SAME; HOLDING THAT THE IMPOSITION OF SAID CONDITION writing, the document itself, being constituted by the parties
PREVENTED THE PERFECTION OF THE CONTRACT OF as the expositor of their intentions, it is the only instrument of
3. HOLDING THAT THE DEED OF OPTION SALE DESPITE THE EXPRESS OFFER AND evidence in respect of that agreement which the law will
EXPRESSED THE TRUE INTENTION AND PURPOSE OF ACCEPTANCE CONTAINED IN THE DEED OF OPTION; recognize so long as it exists for the purpose of evidence.
THE PARTIES DESPITE ADVERSE, (Starkie, EV, pp. 648, 655 cited in Kasheenath vs. Chundy,
CONTEMPORANEOUS AND SUBSEQUENT ACTS OF THE IV. THE COURT OF APPEALS ERRED IN FINDING W.R. 68, cited in Francisco's Rules of Court, Vol. VII Part I p.
PLAINTIFF-APPELLEES; THAT THE DEED OF OPTION IS VOID FOR LACK OF 153) (Emphasis supplied, pp. 126-127, Records).
CONSIDERATION;
4. FAILING TO PROTECT THE DEFENDANT- The respondent appellate court, however, ruled that the said
APPELLANTS ON ACCOUNT OF THEIR IGNORANCE V. THE COURT OF APPEALS ERRED IN HOLDING deed of option is void for lack of consideration. The appellate
PLACING THEM AT A DISADVANTAGE IN THE DEED OF THAT A DISTINCT CONSIDERATION IS NECESSARY TO court made the following disquisitions:
OPTION; SUPPORT THE DEED OF OPTION DESPITE THE
EXPRESS OFFER AND ACCEPTANCE CONTAINED Plaintiff-appellees say they agreed to pay P70.00 per square
THEREIN. (p. 12, Rollo) meter for the portion purchased by them although the
prevailing price at that time was only P25.00 in consideration
17
Sales – Chapter 3-12 Cases
of the option to buy the remainder of the land. This does not agreed to sell and convey to them the remaining one-half The Reyeses as well were granted an option to sell should
seem to be the case. In the first place, the deed of sale was portion still owned by me ... (p. 26, Rollo) the need for such sale on their part arise.
never produced by them to prove their claim. Defendant-
appellants testified that no copy of the deed of sale had ever The respondent appellate court failed to give due In the instant case, the option offered by private respondents
been given to them by the plaintiff-appellees. In the second consideration to petitioners' evidence which shows that in had been accepted by the petitioner, the promise, in the
place, if this was really the condition of the prior sale, we see 1969 the Villamor spouses bough an adjacent lot from the same document. The acceptance of an offer to sell for a price
no reason why it should be reiterated in the Deed of Option. brother of Macaria Labing-isa for only P18.00 per square certain created a bilateral contract to sell and buy and upon
On the contrary, the alleged overprice paid by the plaintiff- meter which the private respondents did not rebut. Thus, acceptance, the offer, ipso facto assumes obligations of a
appellees is given in the Deed as reason for the desire of the expressed in terms of money, the consideration for the deed vendee (See Atkins, Kroll & Co. v. Cua Mian Tek, 102 Phil.
Villamors to acquire the land rather than as a consideration of option is the difference between the purchase price of the 948). Demandabilitiy may be exercised at any time after the
for the option given to them, although one might wonder why 300 square meter portion of the lot in 1971 (P70.00 per execution of the deed. In Sanchez v. Rigos, No. L-25494,
they took nearly 13 years to invoke their right if they really sq.m.) and the prevailing reasonable price of the same lot in June 14, 1972, 45 SCRA 368, 376, We held:
were in due need of the lot. 1971. Whatever it is, (P25.00 or P18.00) though not
specifically stated in the deed of option, was ascertainable. In other words, since there may be no valid contract without a
At all events, the consideration needed to support a unilateral Petitioner's allegedly paying P52.00 per square meter for the cause of consideration, the promisory is not bound by his
promise to sell is a dinstinct one, not something that is as option may, as opined by the appellate court, be improbable promise and may, accordingly withdraw it. Pending notice of
uncertain as P70.00 per square meter which is allegedly but improbabilities does not invalidate a contract freely its withdrawal, his accepted promise partakes, however, of
'greatly higher than the actual prevailing value of lands.' A entered into by the parties. the nature of an offer to sell which, if accepted, results in a
sale must be for a price certain (Art. 1458). For how much the perfected contract of sale.
portion conveyed to the plaintiff-appellees was sold so that The "deed of option" entered into by the parties in this case
the balance could be considered the consideration for the had unique features. Ordinarily, an optional contract is a A contract of sale is, under Article 1475 of the Civil Code,
promise to sell has not been shown, beyond a mere privilege existing in one person, for which he had paid a "perfected at the moment there is a meeting of minds upon
allegation that it was very much below P70.00 per square consideration and which gives him the right to buy, for the thing which is the object of the contract and upon the
meter. example, certain merchandise or certain specified property, price. From that moment, the parties may reciprocally
from another person, if he chooses, at any time within the demand perform of contracts." Since there was, between the
The fact that plaintiff-appellees might have paid P18.00 per agreed period at a fixed price (Enriquez de la Cavada v. Diaz, parties, a meeting of minds upon the object and the price,
square meter for another land at the time of the sale to them 37 Phil. 982). If We look closely at the "deed of option" signed there was already a perfected contract of sale. What was,
of a portion of defendant-appellant's lot does not necessarily by the parties, We will notice that the first part covered the however, left to be done was for either party to demand from
prove that the prevailing market price at the time of the sale statement on the sale of the 300 square meter portion of the the other their respective undertakings under the contract. It
was P18.00 per square meter. (In fact they claim it was lot to Spouses Villamor at the price of P70.00 per square may be demanded at any time either by the private
P25.00). It is improbable that plaintiff-appellees should pay meter "which was higher than the actual reasonable respondents, who may compel the petitioners to pay for the
P52.00 per square meter for the privilege of buying when the prevailing value of the lands in that place at that time (of property or the petitioners, who may compel the private
value of the land itself was allegedly P18.00 per square sale)." The second part stated that the only reason why the respondents to deliver the property.
meter. (pp. 34-35, Rollo) Villamor spouses agreed to buy the said lot at a much higher
price is because the vendor (Reyeses) also agreed to sell to However, the Deed of Option did not provide for the period
As expressed in Gonzales v. Trinidad, 67 Phil. 682, the Villamors the other half-portion of 300 square meters of within which the parties may demand the performance of their
consideration is "the why of the contracts, the essential the land. Had the deed stopped there, there would be no respective undertakings in the instrument. The parties could
reason which moves the contracting parties to enter into the dispute that the deed is really an ordinary deed of option not have contemplated that the delivery of the property and
contract." The cause or the impelling reason on the part of granting the Villamors the option to buy the remaining 300 the payment thereof could be made indefinitely and render
private respondent executing the deed of option as appearing square meter-half portion of the lot in consideration for their uncertain the status of the land. The failure of either parties to
in the deed itself is the petitioner's having agreed to buy the having agreed to buy the other half of the land for a much demand performance of the obligation of the other for an
300 square meter portion of private respondents' land at higher price. But, the "deed of option" went on and stated that unreasonable length of time renders the contract ineffective.
P70.00 per square meter "which was greatly higher than the the sale of the other half would be made "whenever the need
actual reasonable prevailing price." This cause or of such sale arises, either on our (Reyeses) part or on the Under Article 1144 (1) of the Civil Code, actions upon written
consideration is clear from the deed which stated: part of the Spouses Julio Villamor and Marina V. Villamor. It contract must be brought within ten (10) years. The Deed of
appears that while the option to buy was granted to the Option was executed on November 11, 1971. The
That the only reason why the spouses-vendees Julio Villamor Villamors, the Reyeses were likewise granted an option to acceptance, as already mentioned, was also accepted in the
and Marina V. Villamor agreed to buy the said one-half sell. In other words, it was not only the Villamors who were same instrument. The complaint in this case was filed by the
portion at the above stated price of about P70.00 per square granted an option to buy for which they paid a consideration. petitioners on July 13, 1987, seventeen (17) years from the
meter, is because I, and my husband Roberto Reyes, have time of the execution of the contract. Hence, the right of
18
Sales – Chapter 3-12 Cases
action had prescribed. There were allegations by the
petitioners that they demanded from the private respondents
as early as 1984 the enforcement of their rights under the
contract. Still, it was beyond the ten (10) years period
prescribed by the Civil Code. In the case of Santos v.
Ganayo,
L-31854, September 9, 1982, 116 SCRA 431, this Court
affirming and subscribing to the observations of the court a
quo held, thus:

... Assuming that Rosa Ganayo, the oppositor herein, had the
right based on the Agreement to Convey and Transfer as
contained in Exhibits '1' and '1-A', her failure or the
abandonment of her right to file an action against Pulmano
Molintas when he was still a co-owner of the on-half (1/2)
portion of the 10,000 square meters is now barred by laches
and/or prescribed by law because she failed to bring such
action within ten (10) years from the date of the written
agreement in 1941, pursuant to Art. 1144 of the New Civil
Code, so that when she filed the adverse claim through her
counsel in 1959 she had absolutely no more right whatsoever
on the same, having been barred by laches.

It is of judicial notice that the price of real estate in Metro


Manila is continuously on the rise. To allow the petitioner to
demand the delivery of the property subject of this case
thirteen (13) years or seventeen (17) years after the
execution of the deed at the price of only P70.00 per square
meter is inequitous. For reasons also of equity and in
consideration of the fact that the private respondents have no
other decent place to live, this Court, in the exercise of its
equity jurisdiction is not inclined to grant petitioners' prayer.

ACCORDINGLY, the petition is DENIED. The decision of


respondent appellate court is AFFIRMED for reasons cited in
this decision. Judgement is rendered dismissing the
complaint in Civil Case No. C-12942 on the ground of
prescription and laches.

SO ORDERED.

19
Sales – Chapter 3-12 Cases
NICOLAS SANCHEZ, plaintiff-appellee, ART. 1479. A promise to buy and sell a determinate accepted unilateral promise to buy or to sell." In other
vs. thing for a price certain is reciprocally demandable. words, Article 1479 is controlling in the case at bar.
SEVERINA RIGOS, defendant-appellant.
An accepted unilateral promise to buy or to sell a (2) In order that said unilateral promise may be
Appeal from a decision of the Court of First Instance of determinate thing for a price certain is binding upon the "binding upon the promisor, Article 1479 requires the
Nueva Ecija to the Court of Appeals, which certified the promissor if the promise is supported by a consideration concurrence of a condition, namely, that the promise be
case to Us, upon the ground that it involves a question distinct from the price. "supported by a consideration distinct from the price."
purely of law. Accordingly, the promisee can not compel the promisor to
In his complaint, plaintiff alleges that, by virtue of the comply with the promise, unless the former establishes
The record shows that, on April 3, 1961, plaintiff Nicolas option under consideration, "defendant agreed and the existence of said distinct consideration. In other
Sanchez and defendant Severina Rigos executed an committed to sell" and "the plaintiff agreed and committed words, the promisee has the burden of proving such
instrument entitled "Option to Purchase," whereby Mrs. to buy" the land described in the option, copy of which consideration. Plaintiff herein has not even alleged the
Rigos "agreed, promised and committed ... to sell" to was annexed to said pleading as Annex A thereof and is existence thereof in his complaint.
Sanchez the sum of P1,510.00, a parcel of land situated quoted on the margin.1 Hence, plaintiff maintains that the
in the barrios of Abar and Sibot, municipality of San Jose, promise contained in the contract is "reciprocally (3) Upon the other hand, defendant explicitly averred
province of Nueva Ecija, and more particularly described demandable," pursuant to the first paragraph of said in her answer, and pleaded as a special defense, the
in Transfer Certificate of Title No. NT-12528 of said Article 1479. Although defendant had really "agreed, absence of said consideration for her promise to sell and,
province, within two (2) years from said date with the promised and committed" herself to sell the land to the by joining in the petition for a judgment on the pleadings,
understanding that said option shall be deemed plaintiff, it is not true that the latter had, in turn, "agreed plaintiff has impliedly admitted the truth of said averment
"terminated and elapsed," if "Sanchez shall fail to exercise and committed himself " to buy said property. Said Annex in defendant's answer. Indeed as early as March 14,
his right to buy the property" within the stipulated period. A does not bear out plaintiff's allegation to this effect. 1908, it had been held, in Bauermann v. Casas,3 that:
Inasmuch as several tenders of payment of the sum of What is more, since Annex A has been made "an integral
Pl,510.00, made by Sanchez within said period, were part" of his complaint, the provisions of said instrument One who prays for judgment on the pleadings without
rejected by Mrs. Rigos, on March 12, 1963, the former form part "and parcel"2 of said pleading. offering proof as to the truth of his own allegations, and
deposited said amount with the Court of First Instance of without giving the opposing party an opportunity to
Nueva Ecija and commenced against the latter the The option did not impose upon plaintiff the obligation to introduce evidence, must be understood to admit the truth
present action, for specific performance and damages. purchase defendant's property. Annex A is not a "contract of all the material and relevant allegations of the opposing
to buy and sell." It merely granted plaintiff an "option" to party, and to rest his motion for judgment on those
After the filing of defendant's answer — admitting some buy. And both parties so understood it, as indicated by the allegations taken together with such of his own as are
allegations of the complaint, denying other allegations caption, "Option to Purchase," given by them to said admitted in the pleadings. (La Yebana Company vs.
thereof, and alleging, as special defense, that the contract instrument. Under the provisions thereof, the defendant Sevilla, 9 Phil. 210). (Emphasis supplied.)
between the parties "is a unilateral promise to sell, and "agreed, promised and committed" herself to sell the land
the same being unsupported by any valuable therein described to the plaintiff for P1,510.00, but there is This view was reiterated in Evangelista v. De la Rosa4
consideration, by force of the New Civil Code, is null and nothing in the contract to indicate that her aforementioned and Mercy's Incorporated v. Herminia Verde.5
void" — on February 11, 1964, both parties, assisted by agreement, promise and undertaking is supported by a
their respective counsel, jointly moved for a judgment on consideration "distinct from the price" stipulated for the Squarely in point is Southwestern Sugar & Molasses Co.
the pleadings. Accordingly, on February 28, 1964, the sale of the land. v. Atlantic Gulf & Pacific Co.,6 from which We quote:
lower court rendered judgment for Sanchez, ordering Mrs.
Rigos to accept the sum judicially consigned by him and Relying upon Article 1354 of our Civil Code, the lower The main contention of appellant is that the option granted
to execute, in his favor, the requisite deed of conveyance. court presumed the existence of said consideration, and to appellee to sell to it barge No. 10 for the sum of
Mrs. Rigos was, likewise, sentenced to pay P200.00, as this would seem to be the main factor that influenced its P30,000 under the terms stated above has no legal effect
attorney's fees, and other costs. Hence, this appeal by decision in plaintiff's favor. It should be noted, however, because it is not supported by any consideration and in
Mrs. Rigos. that: support thereof it invokes article 1479 of the new Civil
Code. The article provides:
This case admittedly hinges on the proper application of (1) Article 1354 applies to contracts in general,
Article 1479 of our Civil Code, which provides: whereas the second paragraph of Article 1479 refers to "ART. 1479. A promise to buy and sell a determinate thing
"sales" in particular, and, more specifically, to "an for a price certain is reciprocally demandable.
20
Sales – Chapter 3-12 Cases
to be valid must be supported by a consideration distinct Lastly, even supposing that Exh. A granted an option
An accepted unilateral promise to buy or sell a from the price. which is not binding for lack of consideration, the
determinate thing for a price certain is binding upon the authorities hold that:
promisor if the promise is supported by a consideration We are not oblivious of the existence of American
distinct from the price." authorities which hold that an offer, once accepted, "If the option is given without a consideration, it is a mere
cannot be withdrawn, regardless of whether it is supported offer of a contract of sale, which is not binding until
On the other hand, Appellee contends that, even granting or not by a consideration (12 Am. Jur. 528). These accepted. If, however, acceptance is made before a
that the "offer of option" is not supported by any authorities, we note, uphold the general rule applicable to withdrawal, it constitutes a binding contract of sale, even
consideration, that option became binding on appellant offer and acceptance as contained in our new Civil Code. though the option was not supported by a sufficient
when the appellee gave notice to it of its acceptance, and But we are prevented from applying them in view of the consideration. ... . (77 Corpus Juris Secundum, p. 652.
that having accepted it within the period of option, the specific provision embodied in article 1479. While under See also 27 Ruling Case Law 339 and cases cited.)
offer can no longer be withdrawn and in any event such the "offer of option" in question appellant has assumed a
withdrawal is ineffective. In support this contention, clear obligation to sell its barge to appellee and the option "It can be taken for granted, as contended by the
appellee invokes article 1324 of the Civil Code which has been exercised in accordance with its terms, and defendant, that the option contract was not valid for lack of
provides: there appears to be no valid or justifiable reason for consideration. But it was, at least, an offer to sell, which
appellant to withdraw its offer, this Court cannot adopt a was accepted by letter, and of the acceptance the offerer
"ART. 1324. When the offerer has allowed the offeree a different attitude because the law on the matter is clear. had knowledge before said offer was withdrawn. The
certain period to accept, the offer may be withdrawn any Our imperative duty is to apply it unless modified by concurrence of both acts — the offer and the acceptance
time before acceptance by communicating such Congress. — could at all events have generated a contract, if none
withdrawal, except when the option is founded upon there was before (arts. 1254 and 1262 of the Civil Code)."
consideration as something paid or promised." However, this Court itself, in the case of Atkins, Kroll and (Zayco vs. Serra, 44 Phil. 331.)
Co., Inc. v. Cua Hian Tek,8 decided later that
There is no question that under article 1479 of the new Southwestern Sugar & Molasses Co. v. Atlantic Gulf & In other words, since there may be no valid contract
Civil Code "an option to sell," or "a promise to buy or to Pacific Co.,9 saw no distinction between Articles 1324 and without a cause or consideration, the promisor is not
sell," as used in said article, to be valid must be 1479 of the Civil Code and applied the former where a bound by his promise and may, accordingly, withdraw it.
"supported by a consideration distinct from the price." This unilateral promise to sell similar to the one sued upon Pending notice of its withdrawal, his accepted promise
is clearly inferred from the context of said article that a here was involved, treating such promise as an option partakes, however, of the nature of an offer to sell which,
unilateral promise to buy or to sell, even if accepted, is which, although not binding as a contract in itself for lack if accepted, results in a perfected contract of sale.
only binding if supported by consideration. In other words, of a separate consideration, nevertheless generated a
"an accepted unilateral promise can only have a binding bilateral contract of purchase and sale upon acceptance. This view has the advantage of avoiding a conflict
effect if supported by a consideration which means that Speaking through Associate Justice, later Chief Justice, between Articles 1324 — on the general principles on
the option can still be withdrawn, even if accepted, if the Cesar Bengzon, this Court said: contracts — and 1479 — on sales — of the Civil Code, in
same is not supported by any consideration. It is not line with the cardinal rule of statutory construction that, in
disputed that the option is without consideration. It can Furthermore, an option is unilateral: a promise to sell at construing different provisions of one and the same law or
therefore be withdrawn notwithstanding the acceptance of the price fixed whenever the offeree should decide to code, such interpretation should be favored as will
it by appellee. exercise his option within the specified time. After reconcile or harmonize said provisions and avoid a conflict
accepting the promise and before he exercises his option, between the same. Indeed, the presumption is that, in the
It is true that under article 1324 of the new Civil Code, the the holder of the option is not bound to buy. He is free process of drafting the Code, its author has maintained a
general rule regarding offer and acceptance is that, when either to buy or not to buy later. In this case, however, consistent philosophy or position. Moreover, the decision
the offerer gives to the offeree a certain period to accept, upon accepting herein petitioner's offer a bilateral promise in Southwestern Sugar & Molasses Co. v. Atlantic Gulf &
"the offer may be withdrawn at any time before to sell and to buy ensued, and the respondent ipso facto Pacific Co., 10 holding that Art. 1324 is modified by Art.
acceptance" except when the option is founded upon assumed the obligation of a purchaser. He did not just get 1479 of the Civil Code, in effect, considers the latter as an
consideration, but this general rule must be interpreted as the right subsequently to buy or not to buy. It was not a exception to the former, and exceptions are not favored,
modified by the provision of article 1479 above referred to, mere option then; it was a bilateral contract of sale. unless the intention to the contrary is clear, and it is not
which applies to "a promise to buy and sell" specifically. so, insofar as said two (2) articles are concerned. What is
As already stated, this rule requires that a promise to sell more, the reference, in both the second paragraph of Art.
1479 and Art. 1324, to an option or promise supported by
21
Sales – Chapter 3-12 Cases
or founded upon a consideration, strongly suggests that
the two (2) provisions intended to enforce or implement
the same principle.

Upon mature deliberation, the Court is of the considered


opinion that it should, as it hereby reiterates the doctrine
laid down in the Atkins, Kroll & Co. case, and that, insofar
as inconsistent therewith, the view adhered to in the
Southwestern Sugar & Molasses Co. case should be
deemed abandoned or modified.

WHEREFORE, the decision appealed from is hereby


affirmed, with costs against defendant-appellant Severina
Rigos. It is so ordered.

22
Sales – Chapter 3-12 Cases
PERCELINO DIAMANTE, petitioner, Pressed by urgent financial needs, petitioner, on 17 deed of absolute sale was executed merely with the end
vs. October 1960, sold all his remaining rights over the in view of circumventing the requirements for the approval
HON. COURT OF APPEALS and GERARDO property in question to the private respondent for of the transfer of leasehold rights of Diamante in favor of
DEYPALUBUS, respondents. P4,000.00. Deypalubos; and the subsequent execution of the "Option
to Repurchase" was made to assure the maintenance of a
Assailed in this petition for review is the Resolution of the On 25 October 1960, private respondent, with his wife's vendor a retro's rights in favor of Diamante. There was,
respondent Court of Appeals dated 21 March 1979 in consent, executed in favor of the petitioner an Option to therefore, a misrepresentation of an essential or material
C.A.-G.R. No. SP-04866 setting aside its earlier decision Repurchase the property in question within ten (10) years fact committed by the lessee-appellee (Deypalubos) in his
therein, promulgated on 6 December 1978, which from said date, with a ten-year grace period. application for the permit and the lease agreement,
reversed the decision of the then Court of First Instance without which the same could not have been issued. 1
(now Regional Trial Court) of Iloilo City. The latter nullified Private respondent submitted to the Bureau of Fisheries
the Orders of the Secretary of the Department of the definite deed of sale; he did not, however, submit the The Secretary based his action on Section 20 of Fisheries
Agriculture and Natural Resources (DANR) dated 29 Option to Repurchase. Administrative Order No. 60, the second paragraph of
August 1969, 20 November 1969 and 21 April 1970, which reads:
declared binding the Fishpond Lease Agreement (FLA) Thereafter, on 2 August 1961, the Bureau of Fisheries
issued to private respondent and disallowed petitioner issued to private respondent Fishpond Permit No. 4953-Q; Any and all of the statements made in the corresponding
from repurchasing from private respondent a portion of the on 17 December 1962, it approved FLA No. 1372 in the application shall be considered as essential conditions
fishery lot located at Dumangas, Iloilo, covered by the latter's favor. and parts of the permit or lease granted. Any false
FLA. statements in the application of facts or any alteration,
On 11 December 1963, petitioner, contending that he has change or modification of any or all terms and conditions
The pleadings of the parties and the decision of the a valid twenty-year option to repurchase the subject made therein shall ipso facto cause the cancellation of the
respondent Court disclose the factual antecedents of this property, requested the Bureau of Fisheries to nullify FLA permit or lease.
case. No. 1372 insofar as the said property is concerned. On 18
December 1964, his letter-complaint was dismissed. Private respondent moved for a reconsideration of this last
A fishery lot, encompassing an area of 9.4 hectares and Petitioner then sought a reconsideration of the dismissal; Order arguing that the DANR Secretary's previous Order
designated as Lot No. 518-A of the Cadastral Survey of the same was denied on 29 April 1965. His appeal to the of 30 October 1968 dismissing petitioner's letter-complaint
Dumangas, Iloilo, was previously covered by Fishpond Secretary of the DANR was likewise dismissed on 30 had already become final on the ground that he (private
Permit No. F-2021 issued in the name of Anecita Dionio. October 1968. Again, on 20 November 1968, petitioner respondent) was not served a copy of petitioner's 20
Upon Anecita's death, her heirs, petitioner Diamante and sought for a reconsideration; this time, however, he was November 1968 motion for reconsideration. On 20
Primitivo Dafeliz, inherited the property which they later successful. On 29 August 1969, the DANR Secretary November 1969, private respondent's motion for
divided between themselves; petitioner got 4.4. hectares granted his motion in an Order cancelling FLA No. 1372 reconsideration was denied; a second motion for
while Dafeliz got 5 hectares. It is the petitioner's share that and stating, inter alia, that: reconsideration was likewise denied on 20 April 1970.
is the subject of the present controversy. Primitivo Dafeliz
later sold his share to private respondent. Evidently, the application as originally filed, could not be On 5 May 1970, private respondent filed with the Court of
favorably acted upon by reason of the existing right of a First Instance of Iloilo City a special civil action for
On 21 May 1959, petitioner sold to private respondent his third party over a portion thereof. It was only the certiorari with preliminary injunction (docketed as Civil
leasehold rights over the property in question for submission of the deed of absolute sale which could Case No. 8209), seeking to annul the Secretary's Orders
P8,000.00 with the right to repurchase the same within eliminate the stumbling block to the approval of the of 20 April 1970, 20 November 1969 and 29 August 1969
three (3) years from said date. transfer and the issuance of a permit or lease agreement. on the ground that the Secretary: (1) gravely abused his
It was on the basis of this deed of sale, in fact, the one discretion in not giving him the opportunity to be heard on
On 16 August 1960, private respondent filed an entitled "option to repurchase" executed barely a week the question of whether or not the Option to Repurchase
application with the Bureau of Fisheries, dated 12 July from the execution of the deed of absolute sale, (which) was forged; and (2) has no jurisdiction to set aside FLA
1960, for a fishpond permit and a fishpond lease reverted, in effect, the status of the land in question to No. 1372 as the Order of the Bureau of Fisheries
agreement over the entire lot, submitting therewith the what it was after the execution of the deed of sale with dismissing petitioner's 11 December 1963 letter-complaint
deeds of sale executed by Dafeliz and the petitioner. right to repurchase; that is, the land was again placed had already become final.
under an encumbrance in favor of a third party.
Circumstantially, there is a ground (sic) to believe that the
23
Sales – Chapter 3-12 Cases
After issuing a temporary restraining order and a writ of With such documentary evidence duly certified by the
preliminary injunction, the lower court tried the case jointly Notary Public, which is in effect an affirmation of the On the other hand, as to the signature of his wife, the
with Criminal Case No. 520 wherein both the petitioner existence of the deed of "Option of Repurchase" (sic) and latter herself admitted the same to be her own. Thus —
and a certain Atty. Agustin Dioquino, the Notary Public its due execution, the Secretary may not be said to have
who notarized the 25 October 1960 Option to gravely abused his discretion in giving the document Q There is a signature below the typewritten words
Repurchase, were charged with falsification of a public enough evidentiary weight to justify his action in applying "with my marital consent" and above the name Edelina
document. the aforequoted provisions of Fisheries Adm. Order No. Duyo, whose signature is this?
60. This piece of evidence may be considered substantial
After due trial, the lower court acquitted the accused in the enough to support the conclusion reached by the A That is my signature. (T.s.n., Crim. Case No.
criminal case and decided in favor of the private respondent Secretary, which is all that is necessary to 520, April 5, 1971, p. 14).
respondent in Civil Case No. 8209; the court ruled that: (1) sustain an administrative finding of fact (Ortua vs.
the DANR Secretary abused his discretion in issuing the Encarnacion, 59 Phil. 635; Ang Tibay vs. CIR, 69 Phil. In not finding in favor of the perfect validity of the "Option
questioned Orders, (2) petitioner cannot repurchase the 635; Ramos vs. The Sec. of Agriculture and Natural to Repurchase," the court a quo merely indulged in
property in question as the Option to Repurchase is of Resources, et al. L-29097, Jan. 28, 1974, 55 SCRA 330). conjectures. Thus, believing the testimony of appellee that
doubtful validity, and (3) FLA No. 1372 in the name of Reviewing courts do not re-examine the sufficiency of the the later (sic) could not have executed the deed of option
private respondent is valid and binding. evidence in an administrative case, if originally instituted to repurchase after spending allegedly P12,000.00, and
as such, nor are they authorized to receive additional that if there was really a verbal agreement upon the
Petitioner appealed to the respondent Court which, on 6 evidence that was not submitted to the administrative execution of the deed of absolute sale, as alleged by
December 1978, reversed the decision of the trial court 2 agency concerned. For common sense dictates that the appellant, that appellant's right to repurchase, as was
on the ground that no grave abuse of discretion was question of whether the administrative agency abused its stipulated in the earlier deed of sale, shall be preserved,
committed by respondent Secretary inasmuch as private discretion in weighing evidence should be resolved solely such agreement should have been embodied in the deed
respondent was given the opportunity to be heard on his on the basis of the proof that the administrative authorities of sale of October 17, 1960 (Exh. D), the court doubted
claim that the Option to Repurchase is spurious, and that had before them and no other (Timbancaya vs. Vicente, L- the genuineness of the deed of Option to Repurchase
the trial court merely indulged in conjectures in not 19100, Dec. 27, 1963, 9 SCRA 852). In the instant case (sic).
upholding its validity. Said the respondent Court: the evidence presented for the first time before the court a
quo could be considered only for the criminal case heard It is highly doubtful if appellee had spent P12,000.00
With all the foregoing arguments appellee had jointly with this case. during the period from October 17, 1960 to October 25,
exhaustively adduced to show the spuriousness of the 1960 when the deed of option was executed. Likewise,
deed of "Option to Repurchase", appellee can hardly The lower court's action of acquitting the notary public, the right to repurchase could not have been embodied in
complain of not having been given an opportunity to be Agustin Dioquino, and appellant Diamante in Criminal the deed of absolute sale since, as the Secretary of DANR
heard, which is all that is necessary in relation to the Case No. 520 for falsification of public document is in itself found, the purpose of the deed of absolute sale is to
requirement of notice and hearing in administrative a finding that the alleged forgery has not been circumvent the law and insure the approval of appellee's
proceedings. Moreover, appellee never asked for a formal conclusively established. This finding is quite correct application, as with his right to 4.4 hectares appearing to
hearing at the first opportunity that he had to do so, as considering the admission of the NBI handwriting expert be subject to an encumbrance, his application would not
when he filed his first motion for reconsideration. He that admission of the NBI handwriting expert that he have been given favorable action.
asked for a formal hearing only in his second motion for cannot make any finding on the question of whether
reconsideration evidently as a mere afterthought, upon appellee's signature on the deed of "Option to Above all, the speculation and conjectures as indulged in
realizing that his arguments were futile without proofs to Repurchase" is forged or not, because of the lack of (sic) by the court a quo cannot outweigh the probative effect of
support them. specimen signature of appellee for comparative the document itself, a certified xerox copy thereof as
examination. The Secretary may have such signature in issued by the Notary Public, the non-presentation of the
The only remaining question, therefore, is whether the the application papers of appellee on file with the former's original having been explained by its loss, as was the
Secretary acted with grave abuse of discretion in giving office upon which to satisfy himself of (sic) the testimony of the same Notary Public, who justly won
weight to the alleged execution by appellee of the deed of genuineness of appellee's signature. It would be strange, acquittal when charged with falsification of public
Option to Repurchase, on the basis of the xerox copy of indeed, that appellee had not provided the NBI expert with document at the instance of appellee. The fact that the
said deed as certified by the Notary Public, Agustin a specimen of his signature when his purpose was to spaces for the document number, page and book
Dioquino. have an expert opinion that his signature on the numbers were not filled up in the photostatic copy
questioned document is forged. presented by the representative of the Bureau of Records
24
Sales – Chapter 3-12 Cases
Management does not militate against the genuineness of document should be afforded the opportunity to prove misrepresentation of an essential or material fact which,
the document. It simply means that the copy sent to the such claim, because, at most, the presumption of under the second paragraph of Section 29 of Fisheries
said Bureau happens to have those spaces unfilled up regularity in the performance of official duties is merely Administrative Order No. 60 earlier quoted, "shall ipso
(sic). But the sending of a copy of the document to the disputable and can be rebutted by convincing and positive facto cause the cancellation of the permit or lease." In
Bureau of Records Management attests strongly to the evidence to the contrary. short, the Secretary was of the opinion that the Option to
existence of such document, the original of which was Repurchase was an encumbrance on the property which
duly executed, complete with the aforesaid data duly His motion for reconsideration having been denied, the affected the absolute and exclusive character of private
indicated thereon, as shown by the xerox copy certified petitioner filed the instant petition for review. respondent's ownership over the 4.4 hectares sold to him
true by the Notary Public. by petitioner. This is a clear case of a misapplication of
Petitioner contends that the Rules of Court should not be the law on conventional redemption and a
Indeed, in the absence of positive and convincing proof of strictly applied to administrative proceedings and that the misunderstanding of the effects of a right to repurchase
forgery, a public instrument executed with the intervention findings of fact of administrative bodies, absent a showing granted subsequently in an instrument different from the
of a Notary Public must be held in high respect and of arbitrariness, should be accorded respect. original document of sale.
accorded full integrity, if only upon the presumption of the
regularity of official functions as in the nature of those While the petition has merit, petitioner's victory is hollow Article 1601 of the Civil Code provides:
upon the presumption of the regularity of official functions and illusory for, as shall hereafter be shown, even as We
as in the nature of those of a notary public (Bautista vs. reverse the assailed resolution of the respondent Court of Conventional redemption shall take place when the
Dy Bun Chin, 49 OG 179; El Hogar Filipino vs. Olviga, 60 Appeals, the questioned decision of the Secretary must, vendor reserves the right to repurchase the thing sold,
Phil. 17). nevertheless, be set aside on the basis of an erroneous with the obligation to comply with the provisions of article
conclusion of law with respect to the Option to 1616 and other stipulations which may have been agreed
Subsequently, the respondent Court, acting on private Repurchase. upon.
respondent's motion for reconsideration, promulgated on
21 March 1979 the challenged Resolution 3 setting aside The respondent Court correctly held in its decision of 6 In Villarica, et al. vs. Court of Appeals, et al., 4 decided on
the earlier decision and affirmed, in toto, the ruling of the December 1978 that the respondent Secretary provided 29 November 1968, or barely seven (7) days before the
trial court, thus: the private respondent sufficient opportunity to question respondent Court promulgated its decision in this case,
the authenticity of the Option to Repurchase and this Court, interpreting the above Article, held:
. . . the respondent (DANR) Secretary had gone beyond committed no grave abuse of discretion in holding that the
his statutory authority and had clearly acted in abuse of same was in fact executed by private respondent. We The right of repurchase is not a right granted the vendor
discretion in giving due weight to the alleged option to thus find no sufficient legal and factual moorings for by the vendee in a subsequent instrument, but is a right
repurchase whose (sic) genuiness (sic) and due execution respondent Court's sudden turnabout in its resolution of reserved by the vendor in the same instrument of sale as
had been impugned and denied by petitioner-appellee 21 March 1979. That private respondent and his wife one of the stipulations of the contract. Once the
(Deypalubos). While the certified true copy of the option to executed the Option to Repurchase in favor of petitioner instrument of absolute sale is executed, the vendor can no
repurchase may have been the basis of the respondent on 25 October 1960 is beyond dispute. As determined by longer reserve the right to repurchase, and any right
Secretary in resolving the motion for reconsideration, the the respondent Court in its decision of 6 December 1978, thereafter granted the vendor by the vendee in a separate
Court believes that he should have first ordered the private respondent's wife, Edelina Duyo, admitted having instrument cannot be a right of repurchase but some other
presentation of evidence to resolve this factual issue affixed her signature to the said document. Besides, the right like the option to buy in the instant case. . . .
considering the conflicting claims of the parties. As earlier trial court itself in Criminal Case No. 520 which was jointly
pointed out, all that was submitted to the Bureau of tried with the civil case, acquitted both the petitioners and In the earlier case of Ramos, et al. vs. Icasiano, et al., 5
Fisheries and consequently to the respondent Secretary, the notary public, before whom the Option to Repurchase decided in 1927, this Court had already ruled that "an
was a xerox copy of the questioned document which was was acknowledged, of the crime of falsification of said agreement to repurchase becomes a promise to sell when
certified to by a notary public to be a copy of a deed found document. made after the sale, because when the sale is made
in his notarial file which did not bear any specimen of the without such an agreement, the purchaser acquires the
signatures of the contracting parties. And assuming that a We hold, however, that the respondent Secretary gravely thing sold absolutely, and if he afterwards grants the
certification made by a notary public as to the existence of erred in holding that private respondent's non-disclosure vendor the right to repurchase, it is a new contract entered
a document should be deemed an affirmation that such and suppression of the fact that 4.4 hectares of the area into by the purchaser, as absolute owner already of the
document actually exists. Nevertheless, (sic) when such subject of the application is burdened with or encumbered object. In that case the vendor has not reserved to himself
claim is impugned, the one who assails the existence of a by the Option to Repurchase constituted a falsehood or a the right to repurchase."
25
Sales – Chapter 3-12 Cases
consideration and determination of the same essential
In Vda. de Cruzo, et al. vs. Carriaga, et al., 6 this Court Even if the promise was accepted, private respondent was and indispensable in order to arrive at a just decision in
found another occasion to apply the foregoing principle. not bound thereby in the absence of a distinct the case. 11 This Court, thus, has the authority to waive
consideration. 8 the lack of proper assignment of errors if the unassigned
Hence, the Option to Repurchase executed by private errors closely relate to errors properly pinpointed out or if
respondent in the present case, was merely a promise to It may be true that the foregoing issues were not squarely the unassigned errors refer to matters upon which the
sell, which must be governed by Article 1479 of the Civil raised by the parties. Being, however, intertwined with the determination of the questions raised by the errors
Code which reads as follows: issue of the correctness of the decision of the respondent properly assigned depend. 12
Secretary and, considering further that the determination
Art. 1479. — A promise to buy and sell a determinate of said issues is essential and indispensable for the The same also applies to issues not specifically raised by
thing for a price certain is reciprocally demandable. rendition of a just decision in this case, this Court does not the parties. The Supreme Court, likewise, has broad
hesitate to rule on them. discretionary power, in the resolution of a controversy, to
An accepted unilateral promise to buy or to sell a take into consideration matters on record which the
determinate thing for a price certain is binding upon the In Hernandez vs. Andal, 9 this Court held: parties fail to submit to the Court as specific questions for
promissor if the promise is supported by a consideration determination. 13 Where the issues already raised also
distinct from the price. If the appellants' assignment of error be not considered a rest on other issues not specifically presented, as long as
direct challenge to the decision of the court below, we still the latter issues bear relevance and close relation to the
A copy of the so-called Option to Repurchase is neither believe that the objection takes a narrow view of practice former and as long as they arise from matters on record,
attached to the records nor quoted in any of the pleadings and procedure contrary to the liberal spirit which pervades the Court has the authority to include them in its
of the parties. This Court cannot, therefore, properly rule the Rules of Court. The first injunction of the new Rules discussion of the controversy as well as to pass upon
on whether the promise was accepted and a consideration (Rule 1, section 2) is that they "shall be liberally construed them. In brief, in those cases wherein questions not
distinct from the price, supports the option. Undoubtedly, in order to promote their object and to assist the parties in particularly raised by the parties surface as necessary for
in the absence of either or both acceptance and separate obtaining just, speedy, and inexpensive determination of the complete adjudication of the rights and obligations of
consideration, the promise to sell is not binding upon the every action and proceeding." In line with the modern the parties and such questions fall within the issues
promissor (private respondent). trends of procedure, we are told that, "while an already framed by the parties, the interests of justice
assignment of error which is required by law or rule of dictate that the Court consider and resolve them.
A unilateral promise to buy or sell is a mere offer, which is court has been held essential to appellate review, and
not converted into a contract except at the moment it is only those assigned will be considered, there are a WHEREFORE, the instant petition is GRANTED. The
accepted. Acceptance is the act that gives life to a juridical number of cases which appear to accord to the appellate Resolution of respondent Court of Appeals of 21 March
obligation, because, before the promise is accepted, the court a broad discretionary power to waive the lack of 1979 in C.A.-G.R. No. SP-04866 and the Decision of the
promissor may withdraw it at any time. Upon acceptance, proper assignment of errors and consider errors not trial court in Civil Case No. 8209, insofar as they declare,
however, a bilateral contract to sell and to buy is created, assigned. And an unassigned error closely related to an for the reasons therein given, Fishpond Lease Agreement
and the offeree ipso facto assumes the obligations of a error properly assigned, or upon which the determination No. 1372, valid and binding, are hereby REVERSED and
purchaser; the offeror, on the other hand, would be liable of the question raised by the error properly assigned is SET ASIDE. The challenged Orders of the respondent
for damages if he fails to deliver the thing he had offered dependent, will be considered by the appellate court Secretary of Agriculture and Natural Resources of 29
for sale. notwithstanding the failure to assign it as error." (4 C.J.S., August 1969, 20 November 1969 and 21 April 1970 are
1734; 3 C.J., 1341, footnote 77). At the least, the likewise REVERSED and SET ASIDE and Fishpond
xxx xxx xxx assignment of error, viewed in this light, authorizes us to Lease Agreement No. 1372 is ordered REINSTATED.
examine and pass upon the decision of the court below.
. . . The contract of option is a separate and distinct No pronouncement as to costs.
contract from the contract which the parties may enter into In Insular Life Assurance Co., Ltd. Employees
upon the consummation of the option, and a consideration Association-NATU vs. Insular Life Assurance Co., Ltd., 10 IT IS SO ORDERED.
for an optional contract is just as important as the this Court ruled:
consideration for any other kind of contract. Thus, a
distinction should be drawn between the consideration for . . . (t)he Supreme Court has ample authority to review
the option to repurchase, and the consideration for the and resolve matter not assigned and specified as errors
contract of repurchase itself.7 by either of the parties in the appeal if it finds the
26
Sales – Chapter 3-12 Cases
BIBLE BAPTIST CHURCH and PASTOR REUBEN 5. That the title will remain in the safe keeping of the Bible
BELMONTE, petitioners, Baptist Church, Malate, Metro Manila until the expiration c) [F]inally, respondent court erred in not awarding
vs. of the lease agreement or the leased premises be petitioners Baptist Church and its pastor attorney's fees.7
COURT OF APPEALS and MR. & MRS. ELMER TITO purchased by the LESSEE, whichever comes first. In the
MEDINA VILLANUEVA, respondents. event that the said title will be lost or destroyed while in In sum, this Court has three issues to resolve: 1) Whether
the possession of the LESSEE, the LESSEE agrees to or not the option to buy given to the Baptist Church is
This petition for review on certiorari seeks to annul the pay all costs involved for the re-issuance of the title. founded upon a consideration; 2) Whether or not by the
Decision1 dated August 7, 1996, of the Court of Appeals terms of the lease agreement, a price certain for the
in CA-G.R. CV No. 45956, and its Resolution2 dated 6. That the leased premises may be renovated by the purchase of the land had been fixed; and 3) Whether or
September 12, 1996, denying reconsideration of the LESSEE, to the satisfaction of the LESSEE to be fit and not the Baptist Church is entitled to an award for
decision. In the questioned issuances, the Court of usable as a Church. attorney's fees.
Appeals affirmed the Decision3 dated June 8, 1993, of the
Regional Trial Court of Manila, Branch 3, in Civil Case No. 7. That the LESSOR will remove all other tenants from the The stipulation in the lease contract which purportedly
90-55437. leased premises no later than March 15, 1986. It is further gives the lessee an option to buy the leased premises at
agreed that if those tenants are not vacated by June 1, any time within the duration of the lease, is found in
The antecedents are: 1986, the rental will be lowered by the sum of Three paragraph 8 of the lease contract, viz:
Thousand Pesos (P3,000.00) per month until said tenants
On June 7, 1985, the Bible Baptist Church (petitioner have left the leased premises. 8. That the LESSEE has the option to buy the leased
Baptist Church) entered into a contract of lease4 with Mr. premises during the Fifteen (15) years of the lease. If the
& Mrs. Elmer Tito Medina Villanueva (respondent spouses 8. That the LESSEE has the option to buy the leased LESSEE decides to purchase the premises the terms will
Villanueva). The latter are the registered owners of a premises during the Fifteen (15) years of the lease. If the be: A) A selling Price of One Million Eight Hundred
property located at No. 2436 (formerly 2424) Leon Guinto LESSEE decides to purchase the premises the terms will Thousand Pesos (P1.8 million), Philippine Currency. B) A
St., Malate, Manila. The pertinent stipulations in the lease be: A) A selling Price of One Million Eight Hundred down payment agreed upon by both parties. C) The
contract were: Thousand Pesos (P1.8 million), Philippine Currency. B) A balance of the selling price may be paid at the rate of One
down payment agreed upon by both parties. C) The Hundred Twenty Thousand Pesos (P120,000.00),
1. That the LESSOR lets and leases to the LESSEE a balance of the selling price may be paid at the rate of One Philippine Currency, per year.
store space known as 2424 Leon Guinto Sr. St., Malate, Hundred Twenty Thousand Pesos (P120,000.00),
Manila, of which property the LESSOR is the registered Philippine Currency, per year. Under Article 1479 of the Civil Code, it is provided:
owner in accordance with the Land Registration Act.
x x x.5 Art. 1479. A promise to buy and sell a determinate thing
2. That the lease shall take effect on June 7, 1985 and for a price certain is reciprocally demandable.
shall be for the period of Fifteen (15) years. The foregoing stipulations of the lease contract are the
subject of the present controversy. An accepted unilateral promise to buy or to sell a
3. That LESSEE shall pay the LESSOR within five (5) determinate thing for a price certain is binding upon the
days of each calendar month, beginning Twelve (12) Although the same lease contract resulted in several promissor if the promise is supported by a consideration
months from the date of this agreement, a monthly rental cases6 filed between the same parties herein, petitioner distinct from the price.
of Ten Thousand Pesos (P10,000.00) Philippine submits, for this Court's review, only the following errors
Currency, plus 10% escalation clause per year starting on allegedly committed by the Court of Appeals: The second paragraph of Article 1479 provides for the
June 7, 1988. definition and consequent rights and obligations under an
a) Respondent Court of Appeals erred in finding that the option contract. For an option contract to be valid and
4. That upon signing of the LEASE AGREEMENT, the option to buy granted the petitioner Baptist Church under enforceable against the promissor, there must be a
LESSEE shall pay the sum of Eighty Four Thousand its contract of lease with the Villanuevas did not have a separate and distinct consideration that supports it.
Pesos (P84,000.00) Philippine Currency. Said sum is to consideration and, therefore, did not bind the latter;
be paid directly to the Rural Bank, Valenzuela, Bulacan for In this case, petitioner Baptist Church seeks to buy the
the purpose of redemption of said property which is b) [R]espondent court again also erred in finding that the leased premises from the spouses Villanueva, under the
mortgaged by the LESSOR. option to buy did not have a fixed price agreed upon by option given to them. Petitioners claim that the Baptist
the parties for the purchase of the property; and Church "agreed to advance the large amount needed for
27
Sales – Chapter 3-12 Cases
the rescue of the property but, in exchange, it asked the registered lessee of a property owned by the Manila other half to the same spouses. It was stated in the deed
Villanuevas to grant it a long term lease and an option to Railroad Co. She entered into an agreement whereby she that the only reason the spouses bought the first half of
buy the property for P1.8 million."8 They argue that the allowed Teodoro to occupy a portion of the rented the parcel of land at a much higher price, was the
consideration supporting the option was their agreement property and gave Teodoro an option to buy the same, undertaking of the sellers to sell the second half of the
to pay off the Villanueva's P84,000 loan with the bank, should Manila Railroad Co. decide to sell the property to land, also at the same price. This Court held that the
thereby freeing the subject property from the mortgage Ariola. In addition, Teodoro, who was occupying only a cause or consideration for the option, on the part of the
encumbrance. They state further that the Baptist Church portion of the subject rented property, also undertook to spouses-buyers, was the undertaking of the sellers to sell
would not have agreed to advance such a large amount pay the Manila Railroad Co., the full amount of the rent the other half of the property. On the part of the sellers,
as it did to rescue the property from bank foreclosure had supposed to be paid by the registered lessor Ariola. the consideration supporting the option was the much
it not been given an enforceable option to buy that went Consequently, unlike this case, Teodoro paid over and higher amount at which the buyers agreed to buy the
with the lease agreement. above the amount due for her own occupation of a portion property. It was explicit from the deed therein that for the
of the property. That amount, which should have been parties, this was the consideration for their entering into
In the petition, the Baptist Church states that "[t]rue, the paid by Ariola as lessor, and for her own occupation of the the contract.
Baptist Church did not pay a separate and specific sum of property, was deemed by the Court as sufficient
money to cover the option alone. But the P84,000 it paid consideration for the option to buy which Ariola gave to It can be seen that the Court found that the
the Villanuevas in advance should be deemed Teodoro upon Ariola's acquiring the property. buyer/optionee had parted with something of value, which
consideration for the one contract they entered into – the was the amount he paid over and above the actual
lease with option to buy."9 They rely on the case of Hence, in Teodoro, this Court was able to find that a prevailing price of the land. Such amount, different from
Teodoro v. Court of Appeals10 to support their stand. separate consideration supported the option contract and the price of the land subject of the option, was deemed
thus, its enforcement may be demanded. Petitioners, sufficient and distinct consideration supporting the option
This Court finds no merit in these contentions. therefore, cannot rely on Teodoro, for the case even contract. Moreover, the parties stated the same in their
supports the respondents' stand that a consideration that contract.
First, petitioners cannot insist that the P84,000 they paid is separate and distinct from the purchase price is
in order to release the Villanuevas' property from the required to support an option contract. Villamor is distinct from the present case because, First,
mortgage should be deemed the separate consideration this Court cannot find that petitioner Baptist Church parted
to support the contract of option. It must be pointed out Petitioners further insist that a consideration need not be a with anything of value, aside from the amount of P84,000
that said amount was in fact apportioned into monthly separate sum of money. They posit that their act of which was in fact eventually utilized as rental payments.
rentals spread over a period of one year, at P7,000 per advancing the money to "rescue" the property from Second, there is no document that contains an agreement
month. Thus, for the entire period of June 1985 to May mortgage and impending foreclosure, should be enough between the parties that petitioner Baptist Church's
1986, petitioner Baptist Church's monthly rent had already consideration to support the option. supposed rescue of the mortgaged property was the
been paid for, such that it only again commenced paying consideration which the parties contemplated in support of
the rentals in June 1986. This is shown by the testimony In Villamor v. Court of Appeals,14 this Court defined the option clause in the contract. As previously stated, the
of petitioner Pastor Belmonte where he states that the consideration as "the why of the contracts, the essential amount advanced had been fully utilized as rental
P84,000 was advance rental equivalent to monthly rent of reason which moves the contracting parties to enter into payments over a period of one year. While the Villanuevas
P7,000 for one year, such that for the entire year from the contract."15 This definition illustrates that the may have them to thank for extending the payment at a
1985 to 1986 the Baptist Church did not pay monthly consideration contemplated to support an option contract time of need, this is not the separate consideration
rent.11 need not be monetary. Actual cash need not be contemplated by law.
exchanged for the option. However, by the very nature of
This Court agrees with respondents that the amount of an option contract, as defined in Article 1479, the same is Noting that the option clause was part of a lease contract,
P84,000 has been fully exhausted and utilized by their an onerous contract for which the consideration must be this Court looked into its previous ruling in the early case
occupation of the premises and there is no separate something of value, although its kind may vary. of Vda. De Quirino v. Palarca,17 where the Court did say
consideration to speak of which could support the that "in reciprocal contracts, like the one in question,18
option.12 Specifically, in Villamor v. Court of Appeals,16 half of a the obligation or promise of each party is the
parcel of land was sold to the spouses Villamor for P70 consideration for that of the other."19 However, it must be
Second, petitioners' reliance on the case of Teodoro v. per square meter, an amount much higher than the noted that in that case, it was also expressly stated in the
Court of Appeals13 is misplaced. The facts of the Teodoro reasonable prevailing price. Thereafter, a deed of option deed that should there be failure to exercise the option to
case reveal that therein respondent Ariola was the was executed whereby the sellers undertook to sell the buy the property, the optionee undertakes to sell the
28
Sales – Chapter 3-12 Cases
building and/or improvements he has made on the conditions of the agreement, the aggrieved party can
premises. In addition, the optionee had also been paying WHEREFORE, judgment is rendered: collect reasonable attorney's fees.23
an amount of rent that was quite high and in fact turned
out to be too burdensome that there was a subsequent 1) Denying plaintiffs' application for writ of injunction; In view of this Court's finding that the option contract is not
agreement to reduce said rentals. The Court found that enforceable for being without consideration, the
"the amount of rentals agreed upon x x x – which amount 2) That defendant cannot be compelled to sell to plaintiffs respondents Villanueva spouses' refusal to comply with it
turned out to be so burdensome upon the lessee, that the the leased premises in accordance with par. 8 of the cannot be the basis of a claim for attorney's fees.
lessor agreed, five years later, to reduce it – as well as the contract of lease;
building and/or improvements contemplated to be Hence, this Court agrees with as the Court of Appeals,
constructed and/or introduced by the lessee, were, 3) Defendant is hereby ordered to reimburse plaintiffs the which affirmed the findings of the Regional Trial Court,
undoubtedly, part of the consideration for his option to sum of P15, 919.75 plus 12% interest representing real that such claim is to be dismissed for lack of factual and
purchase the leased premises."20 estate taxes, plaintiffs paid the City Treasurer's Office of legal basis.
Manila;
Again, this Court notes that the parties therein clearly WHEREFORE, the Decision and Resolution of the Court
stipulated in their contract that there was an undertaking 4) Declaring that plaintiff made a valid and legal of Appeals subject of the petition are hereby AFFIRMED.
on the part of the optionee to sell the improvements made consignation to the Court of the initial amount of
on the property if the option was not exercised. Such is a P18,634.00 for the month of November and December No costs.
valuable consideration that could support the option 1990 and every month thereafter.
contract. Moreover, there was the excessive rental SO ORDERED.
payments that the optionee paid for five years, which the All other claims of the plaintiffs are hereby dismissed for
Court also took into account in deciding that there was a lack of merit.
separate consideration supporting the option.
No pronouncement as to costs.
To summarize the rules, an option contract needs to be
supported by a separate consideration. The consideration SO ORDERED. 21
need not be monetary but could consist of other things or
undertakings. However, if the consideration is not On appeal, the Court of Appeals agreed with the Regional
monetary, these must be things or undertakings of value, Trial Court and found that the option to buy the leased
in view of the onerous nature of the contract of option. premises was not binding upon the Villanuevas for non-
Furthermore, when a consideration for an option contract compliance with Article 1479. It found that said option was
is not monetary, said consideration must be clearly not supported by a consideration as "no money was ever
specified as such in the option contract or clause. really exchanged for and in consideration of the option." In
addition, the appellate court determined that in the instant
This Court also notes that in the present case both the case, "the price for the object is not yet certain." Thus, the
Regional Trial Court and the Court of Appeals agree that Court of Appeals affirmed the Regional Trial Court
the option was not founded upon a separate and distinct decision and dismissed the appeal for lack of merit.22
consideration and that, hence, respondents Villanuevas
cannot be compelled to sell their property to petitioner Having found that the option to buy granted to the
Baptist Church. petitioner Baptist Church was not founded upon a
separate consideration, and hence, not enforceable
The Regional Trial Court found that "[a]ll payments made against respondents, this Court finds no need to discuss
under the contract of lease were for rentals. No money whether a price certain had been fixed as the purchase
[was] ever exchanged for and in consideration of the price.
option." Hence, the Regional Trial Court found the action
of the Baptist Church to be "premature and without basis Anent the claim for attorney's fees, it is stipulated in
to compel the defendant to sell the leased premises." The paragraph 13 of the lease agreement that in the event of
Regional Trial Court consequently ruled: failure of either of the parties to comply with any of the
29
Sales – Chapter 3-12 Cases
A. A. ADDISON, plaintiff-appellant, this purpose. She therefore asked that she be absolved up to that time the lands sold had not been registered in
vs. from the complaint, and that, after a declaration of the accordance with the Torrens system, and on the terms of
MARCIANA FELIX and BALBINO TIOCO, defendants- rescission of the contract of the purchase and sale of said the second paragraph of clause (h) of the contract,
appellees. lands, the plaintiff be ordered to refund the P3,000 that whereby it is stipulated that ". . . within one year from the
had been paid to him on account, together with the date of the certificate of title in favor of Marciana Felix, this
By a public instrument dated June 11, 1914, the plaintiff interest agreed upon, and to pay an indemnity for the latter may rescind the present contract of purchase and
sold to the defendant Marciana Felix, with the consent of losses and damages which the defendant alleged she had sale . . . ."
her husband, the defendant Balbino Tioco, four parcels of suffered through the plaintiff's non-fulfillment of the
land, described in the instrument. The defendant Felix contract. The appellant objects, and rightly, that the cross-complaint
paid, at the time of the execution of the deed, the sum of is not founded on the hypothesis of the conventional
P3,000 on account of the purchase price, and bound The evidence adduced shows that after the execution of rescission relied upon by the court, but on the failure to
herself to pay the remainder in installments, the first of the deed of the sale the plaintiff, at the request of the deliver the land sold. He argues that the right to rescind
P2,000 on July 15, 1914, and the second of P5,000 thirty purchaser, went to Lucena, accompanied by a the contract by virtue of the special agreement not only
days after the issuance to her of a certificate of title under representative of the latter, for the purpose of designating did not exist from the moment of the execution of the
the Land Registration Act, and further, within ten years and delivering the lands sold. He was able to designate contract up to one year after the registration of the land,
from the date of such title P10, for each coconut tree in only two of the four parcels, and more than two-thirds of but does not accrue until the land is registered. The
bearing and P5 for each such tree not in bearing, that these two were found to be in the possession of one Juan wording of the clause, in fact, substantiates the
might be growing on said four parcels of land on the date Villafuerte, who claimed to be the owner of the parts so contention. The one year's deliberation granted to the
of the issuance of title to her, with the condition that the occupied by him. The plaintiff admitted that the purchaser purchaser was to be counted "from the date of the
total price should not exceed P85,000. It was further would have to bring suit to obtain possession of the land certificate of title ... ." Therefore the right to elect to
stipulated that the purchaser was to deliver to the vendor (sten. notes, record, p. 5). In August, 1914, the surveyor rescind the contract was subject to a condition, namely,
25 per centum of the value of the products that she might Santamaria went to Lucena, at the request of the plaintiff the issuance of the title. The record show that up to the
obtain from the four parcels "from the moment she takes and accompanied by him, in order to survey the land sold present time that condition has not been fulfilled;
possession of them until the Torrens certificate of title be to the defendant; but he surveyed only two parcels, which consequently the defendant cannot be heard to invoke a
issued in her favor." are those occupied mainly by the brothers Leon and Julio right which depends on the existence of that condition. If
Villafuerte. He did not survey the other parcels, as they in the cross-complaint it had been alleged that the
It was also covenanted that "within one year from the date were not designated to him by the plaintiff. In order to fulfillment of the condition was impossible for reasons
of the certificate of title in favor of Marciana Felix, this make this survey it was necessary to obtain from the Land imputable to the plaintiff, and if this allegation had been
latter may rescind the present contract of purchase and Court a writ of injunction against the occupants, and for proven, perhaps the condition would have been
sale, in which case Marciana Felix shall be obliged to the purpose of the issuance of this writ the defendant, in considered as fulfilled (arts. 1117, 1118, and 1119, Civ.
return to me, A. A. Addison, the net value of all the June, 1914, filed an application with the Land Court for Code); but this issue was not presented in the defendant's
products of the four parcels sold, and I shall obliged to the registration in her name of four parcels of land answer.
return to her, Marciana Felix, all the sums that she may described in the deed of sale executed in her favor by the
have paid me, together with interest at the rate of 10 per plaintiff. The proceedings in the matter of this application However, although we are not in agreement with the
cent per annum." were subsequently dismissed, for failure to present the reasoning found in the decision appealed from, we
required plans within the period of the time allowed for the consider it to be correct in its result. The record shows
In January, 1915, the vendor, A. A. Addison, filed suit in purpose. that the plaintiff did not deliver the thing sold. With respect
Court of First Instance of Manila to compel Marciana Felix to two of the parcels of land, he was not even able to
to make payment of the first installment of P2,000, The trial court rendered judgment in behalf of the show them to the purchaser; and as regards the other
demandable in accordance with the terms of the contract defendant, holding the contract of sale to be rescinded two, more than two-thirds of their area was in the hostile
of sale aforementioned, on July 15, 1914, and of the and ordering the return to the plaintiff the P3,000 paid on and adverse possession of a third person.
interest in arrears, at the stipulated rate of 8 per cent per account of the price, together with interest thereon at the
annum. The defendant, jointly with her husband, rate of 10 per cent per annum. From this judgment the The Code imposes upon the vendor the obligation to
answered the complaint and alleged by way of special plaintiff appealed. deliver the thing sold. The thing is considered to be
defense that the plaintiff had absolutely failed to deliver to delivered when it is placed "in the hands and possession
the defendant the lands that were the subject matter of In decreeing the rescission of the contract, the trial judge of the vendee." (Civ. Code, art. 1462.) It is true that the
the sale, notwithstanding the demands made upon him for rested his conclusion solely on the indisputable fact that same article declares that the execution of a public
30
Sales – Chapter 3-12 Cases
instruments is equivalent to the delivery of the thing which It is evident, then, in the case at bar, that the mere payment, with the costs of both instances against the
is the object of the contract, but, in order that this symbolic execution of the instrument was not a fulfillment of the appellant. So ordered.
delivery may produce the effect of tradition, it is necessary vendors' obligation to deliver the thing sold, and that from
that the vendor shall have had such control over the thing such non-fulfillment arises the purchaser's right to
sold that, at the moment of the sale, its material delivery demand, as she has demanded, the rescission of the sale
could have been made. It is not enough to confer upon and the return of the price. (Civ. Code, arts. 1506 and
the purchaser the ownership and the right of possession. 1124.)
The thing sold must be placed in his control. When there
is no impediment whatever to prevent the thing sold Of course if the sale had been made under the express
passing into the tenancy of the purchaser by the sole will agreement of imposing upon the purchaser the obligation
of the vendor, symbolic delivery through the execution of a to take the necessary steps to obtain the material
public instrument is sufficient. But if, notwithstanding the possession of the thing sold, and it were proven that she
execution of the instrument, the purchaser cannot have knew that the thing was in the possession of a third
the enjoyment and material tenancy of the thing and make person claiming to have property rights therein, such
use of it himself or through another in his name, because agreement would be perfectly valid. But there is nothing in
such tenancy and enjoyment are opposed by the the instrument which would indicate, even implicitly, that
interposition of another will, then fiction yields to reality — such was the agreement. It is true, as the appellant
the delivery has not been effected. argues, that the obligation was incumbent upon the
defendant Marciana Felix to apply for and obtain the
As Dalloz rightly says (Gen. Rep., vol. 43, p. 174) in his registration of the land in the new registry of property; but
commentaries on article 1604 of the French Civil code, from this it cannot be concluded that she had to await the
"the word "delivery" expresses a complex idea . . . the final decision of the Court of Land Registration, in order to
abandonment of the thing by the person who makes the be able to enjoy the property sold. On the contrary, it was
delivery and the taking control of it by the person to whom expressly stipulated in the contract that the purchaser
the delivery is made." should deliver to the vendor one-fourth "of the products ...
of the aforesaid four parcels from the moment when she
The execution of a public instrument is sufficient for the takes possession of them until the Torrens certificate of
purposes of the abandonment made by the vendor; but it title be issued in her favor." This obviously shows that it
is not always sufficient to permit of the apprehension of was not forseen that the purchaser might be deprived of
the thing by the purchaser. her possession during the course of the registration
proceedings, but that the transaction rested on the
The supreme court of Spain, interpreting article 1462 of assumption that she was to have, during said period, the
the Civil Code, held in its decision of November 10, 1903, material possession and enjoyment of the four parcels of
(Civ. Rep., vol. 96, p. 560) that this article "merely land.
declares that when the sale is made through the means of
a public instrument, the execution of this latter is Inasmuch as the rescission is made by virtue of the
equivalent to the delivery of the thing sold: which does not provisions of law and not by contractual agreement, it is
and cannot mean that this fictitious tradition necessarily not the conventional but the legal interest that is
implies the real tradition of the thing sold, for it is demandable.
incontrovertible that, while its ownership still pertains to
the vendor (and with greater reason if it does not), a third It is therefore held that the contract of purchase and sale
person may be in possession of the same thing; entered into by and between the plaintiff and the
wherefore, though, as a general rule, he who purchases defendant on June 11, 1914, is rescinded, and the plaintiff
by means of a public instrument should be deemed . . . to is ordered to make restitution of the sum of P3,000
be the possessor in fact, yet this presumption gives way received by him on account of the price of the sale,
before proof to the contrary." together with interest thereon at the legal rate of 6 per
annum from the date of the filing of the complaint until
31
Sales – Chapter 3-12 Cases
NAVOTAS INDUSTRIAL CORPORATION, represented purchase price of ₱350,000.00 which the vendor
herein by its acting president DANIEL L. BAUTISTA, acknowledged to have received from the vendees.5 On July 30, 1977, Carmen Cruz, as lessor, and the NIC, as
Petitioners, lessee, executed a Supplementary Lease Agreement;12 the
vs. In a Letter6 dated November 22, 1976, Mariano Cruz, in his October 5, 1966 Contract of Lease earlier executed by the
GERMAN D. CRUZ, MARCELO D. CRUZ, ROSALINA behalf and in behalf of the other vendees, requested CBC to parties was modified, in that the terms of the
CRUZ-LAIZ, MARIANO A. CRUZ, JR., THE HEIRS OF conform to the sale of the property, a copy of which was lease was extended for another 15 years to expire on
ROGELIO D. CRUZ, namely, SYLVIA, ROSYL, ROGELIO, attached to the said letter. The CBC refused. October 1, 2005. The lessee was, likewise, given up to
JR., SERGIO and ESTRELLA, all surnamed CRUZ, the October 1, 1982 within which to construct the two slipways at
HEIRS OF SERAFIN D. CRUZ, namely, ADELAIDA, In the meantime, relations between Carmen Cruz and her a cost of not less than ₱600,000.00 and increasing the lease
MERCEDITAS and GABRIEL, all surnamed CRUZ, MARIA children became strained. She believed that her children had rental for the property. The lessee was granted the option to
CRISTINA CRUZ-YCASIANO, MONICA CRUZ-DADIVAS ignored her and failed to take care of her. buy the property for the price of ₱1,600,000.00. On the same
and CARMEN VDA. DE CRUZ, Respondent. day, the parties executed a Contract of Lease13 over an
On June 27, 1977, Mariano Cruz, for himself and in behalf of additional portion of the property, with an area of 590.58
This is a petition for review on certiorari of the Decision1 of the other vendees, presented the said deed of sale to the square meters, as shown in the sketch appended thereto.
the Court of Appeals (CA) in CA-G.R. CV No. 69818, Register of Deeds for registration purposes.7 In the same However, the said contracts were not presented for
reversing the Decision of the Regional Trial Court (RTC) in letter, they requested the Register of Deeds to request the registration to the Register of Deeds.
Civil Case No. 2427-MN. CBC for the transmittal of the owner’s TCT No. 81574 for the
annotation of the Deed of Sale with Assumption of Mortgage. On September 14, 1977, the aforesaid Cancellation of Real
The Antecedents However, on June 28, 1977, the CBC, through counsel, wrote Estate Mortgage the CBC had earlier executed (on June 29,
Mariano Cruz, informing him that Carmen Cruz had instructed 1977) was presented to the Register of Deeds and annotated
Carmen Vda. De Cruz was the owner of a parcel of land it not to conform to the Deed of Sale with Assumption of at the dorsal portion of TCT No. 81574 as Entry No. 27796.
located in Navotas, Rizal, with an area of 13,999 square Mortgage, and not to surrender the owner’s duplicate of the The following were, likewise, presented to the Register of
meters, covered by Transfer Certificate of Title (TCT) No. said title. Deeds for registration, and, thereafter, annotated at the
81574.2 dorsal portion of the said title: the Contract of Lease dated
In the meantime, the balance of the loan account secured by October 5, 1966 (Entry No. 27797), the July 30, 1977
On October 5, 1966, Carmen Cruz, as lessor, and the the mortgage was paid to the CBC. Thus, on June 29, 1977, Contract of Lease (Entry No. 27798), and the Supplementary
Navotas Industrial Corporation (NIC), through its president, the CBC executed a Cancellation of Real Estate Mortgage Lease Agreement (Entry No. 27799).14
Cipriano C. Bautista, as lessee, executed a contract of lease over the property.8 However, the deed was not presented to
over one-half portion of the said property, shown in the sketch the Register of Deeds for registration. In the meantime, Mariano Cruz and the other vendees
appended thereto as Annex "A." The lease was for the period presented the Deed of Sale with Assumption of Mortgage to
of October 1, 1966 to midnight of October 1, 1990. The On the same day, Mariano Cruz executed an Affidavit of the Register of Deeds for registration. On December 19,
property was to be used for shipyard slipways and the Adverse Claim9 stating, inter alia, that he and the others 1977, the Register of Deeds cancelled the said title and
lessee’s other allied businesses. The NIC obliged itself to named therein were the vendees of the property as issued TCT No. 11272 in the names of the new owners. TCT
construct two slipways, with all its accessories, within the first evidenced by a Deed of Sale with Assumption of Mortgage No. 11272 was later cancelled by TCT No. R-11830.
10 years of the lease with a total value of not less than appended thereto, and that, to protect their rights and
₱450,000.00.3 interests, the said affidavit of adverse claim was being In a Letter15 dated October 20, 1978, Mariano Cruz, et al.
executed as a cautionary notice to third persons and the informed the NIC that the property had been sold to them,
On March 14, 1973, the property was mortgaged to the China world that the property had been sold to them. It was, and gave it 30 days from receipt of the letter to vacate the
Banking Corporation (CBC) as security for a loan by two of likewise, stated that Carmen Cruz had ordered the CBC not property and return possession to them. The vendees,
Carmen Cruz’s children, Mariano and Gabriel.4 The owner’s to surrender the owner’s duplicate of TCT No. 81574. The likewise, informed the NIC that since the October 5, 1966
duplicate of the title was delivered to and kept by the CBC as aforesaid affidavit of adverse claim was inscripted at the Contracts of Lease and the July 30, 1977 Supplementary
mortgagee. dorsal portion of the title10 on June 30, 1977 as Entry No. Lease Agreement were annotated at the back of TCT No.
22178. 81574 only on September 14, 1977, after the affidavit of
On December 31, 1974, Carmen Cruz executed a Deed of adverse claim of Mariano Cruz, et al. was annotated on June
Absolute Sale of Realty with Assumption of Mortgage in In a Letter11 dated July 1, 1977, the Register of Deeds 29, 1977, such contracts were null and void. However, the
which she, as vendor, sold and conveyed the property to her requested CBC to surrender the owner’s duplicate of TCT No. NIC refused to vacate the property.
children, namely, Serafin D. Cruz (married 81574, pursuant to Section 72 of Act 496, in order that proper
to Adelaida Cruz), Mariano Cruz, Rogelio Cruz, Sr. memorandum be made thereon. The Register of Deeds was In the meantime, the property was subdivided into three lots:
Carmencita Cruz and Sr. Mary Carmela Cruz, for the obviously unaware that the CBC had already executed the Lots 1-A, 1-B and 1-C. Lot 1-A had an area of 6,307 square
cancellation of real estate mortgage on June 29, 1977. meters, covered by TCT No. 8509916 issued on July 5, 1982.
32
Sales – Chapter 3-12 Cases
Contract of Lease (Annex "D"), both dated July 30, 1977, for CA affirmed the decision of the RTC and dismissed the
Carmen Cruz filed a complaint with the RTC of Navotas having been entered into by the plaintiff who had long ceased petition.25 The decision became final and executory.
against Cipriano Bautista, in his capacity as president of the to be the owner of the property in question, awarding the sum
NIC, for the declaration of nullity of the July 30, 1977 of ₱450,000.00, actual damages, representing the value of In the meantime, Mariano Cruz died intestate and was
Supplementary Lease Agreement and Contract of Lease, and the improvements which the defendants bound themselves to survived by his son Mariano Cruz, Jr.; Rogelio Cruz, likewise,
for the cancellation of the annotation at the back of TCT No. introduce in the premises; awarding the plaintiffs-intervenors died and was survived by his children Sylvia, Rosyl, Rogelio,
81574 referring to the said contracts. The complaint was the sum of ₱100,000.00 as exemplary damages; the sum of Jr., Sergio and Estrella, all surnamed Cruz; Serafin Cruz also
amended to implead the NIC as party-defendant. Carmen ₱150,000.00 as moral damages; ₱50,000.00 as attorney’s died and was survived by his wife Adelaida, and his children
Cruz alleged therein that she was the owner-lessor of the fees and ₱10,000.00 as litigation expenses. Merceditas and Gabriel. TCT No. 81574 was reconstituted
property subject of the said contract; the NIC failed to and TCT No. R-85099 was issued.
construct the two slipways within the period stated in the Plaintiffs-intervenors further pray for such other relief and
lease contract; it took advantage of the animosity between remedies they are entitled to in the premises.18 On January 24, 1995, German and Marcelo Cruz, Rosalina
her and her children, and caused the preparation of the July Cruz-Laiz, Mariano Cruz, Jr. and the said heirs filed a
30, 1977 Supplementary Lease Agreement and Contract of However, Carmen Cruz filed a motion to dismiss the Complaint against Carmen Cruz, as unwilling plaintiff, and the
Lease; the NIC was able to insert therein blatantly erroneous, amended complaint. On February 6, 1984, the trial court NIC with the RTC of Malabon for the nullification of the July
one-sided and highly unfair provisions; and that the said issued an Order19 granting the motion and dismissing the 30, 1977 Supplementary Lease Agreement and Contract of
contracts were even extended for a period long beyond her amended complaint and the complaint-in-intervention. The Lease. The complaint was amended to allege that they were
life expectancy (the plaintiff was then almost 80 years old). order became final and executory. the co-owners of the property covered by TCT No. 85099
She further alleged that the provisions in the Contract of based on the Deed of Sale with Assumption of Mortgage
Lease and Supplementary Lease Agreement which granted On June 23, 1990, Mariano Cruz, et al. wrote the NIC that executed by Carmen Cruz on December 31, 1974; an
NIC the exclusive option to buy the property, was a sham. they would no longer renew the October 5, 1966 contract of affidavit of adverse claim was annotated at the dorsal portion
She prayed that, after due proceedings, judgment be lease which was to expire on October 1, 1990; as far as they of TCT No. 81574 on June 30, 1977, despite which NIC
rendered in her favor: were concerned, the July 30, 1977 Supplementary Lease caused Carmen Cruz to execute, on July 30, 1977, a
Agreement and Contract of Lease were null and void, the Supplementary Lease Agreement and Contract of Lease by
WHEREFORE, it is respectfully prayed that judgment be same having been executed and annotated on September taking advantage of her age, mental weakness and lack of
rendered declaring the Supplementary Contract of Lease 14, 1977 at the back of TCT No. 81574 long after the will; and that NIC failed to pay rentals for the property. The
dated July 30, 1977 as null and void ab initio; ordering the annotation of the affidavit of the adverse claim of Mariano plaintiffs prayed that:
defendant and all persons claiming possession of the Cruz, et al. on June 30, 1977.20
premises under it to vacate and turn over the premises to the WHEREFORE, it is respectfully prayed that, after trial on the
plaintiffs; ordering the defendant to pay the reasonable In a Letter21 dated January 11, 1991, Mariano Cruz, et al. merits, judgment be rendered in favor of the plaintiffs as
monthly rental of ₱10,000.00 for the occupancy of the wrote the NIC, demanding that it vacate the property within follows:
premises, beginning October 1, 1990, until it vacates the 30 days from notice thereof, otherwise, a complaint for
premises; ordering the defendant to pay the plaintiffs the sum unlawful detainer would be filed against it. However, the NIC 1. Under the First Alternative Cause of Action, declaring the
of ₱30,000.00 as moral damages; the sum of ₱50,000.00 as refused to vacate the property. Contract of Lease dated 30 July 1977 and the Supplementary
attorney’s fees, and the sum of ₱1,000.00 as appearance fee Lease Contract dated 30 July 1977, Annex "D" hereof, as null
of the undersigned counsel; to pay the sum of ₱5,000.00 as On April 18, 1991, Mariano Cruz and his siblings filed a and void ab initio; or, alternatively,
litigation expenses; plus costs of suit. Complaint22 against the NIC with the Municipal Trial Court
(MTC) of Navotas for ejectment. However, on June 11, 1992, Under the Second Alternative Cause of Action, annulling the
Plaintiffs further pray for such other relief and remedies they the trial court issued an Order23 said Contract of Lease and Supplementary Lease Contract.
are entitled to in the premises.17 dismissing the complaint, on the ground that it had no
jurisdiction over the case, it appearing that the validity of the Under the Third Alternative Cause of Action, rescinding and
Mariano Cruz and his siblings filed a complaint-in-intervention July 30, 1977 Supplementary Lease Agreement and the canceling the Contract of Lease and Supplementary Lease
in the said case, alleging that they were the co-owners of the Contract of Lease, in relation to the deed of absolute sale Agreement, ordering the defendants to vacate the leased
property, and praying that judgment be rendered in their with assumption of mortgage executed by Carmen Cruz, premises and to pay plaintiffs all unpaid rentals from 1
favor, as follows: were intertwined with the issue of NIC’s right of possession. October 1991 until defendants vacate the premises.
The plaintiffs sought a motion for reconsideration of the
WHEREFORE, it is respectfully prayed that judgment be decision, which the MTC denied on September 15, 1992. The 2. Under the Second Cause of Action, ordering defendants
rendered rescinding the Contract of Lease dated October 5, plaintiffs appealed to the RTC, which rendered a decision NAVOTAS and Bautista to vacate and surrender the
1966, (Annex "B"), declaring as null and void the granting the appealed decision.24 The plaintiffs-appellants possession of the subject property and all improvements
Supplementary Lease Agreement (Annex "C"), and the filed a petition for review with the CA. On July 13, 1993, the thereon to the plaintiffs;
33
Sales – Chapter 3-12 Cases
and Bautista, be made to reimburse her for all or part of the Navotas and Bautista on the portion of the property leased to
3. Under the Third Cause of Action, ordering defendants said judgment; it; (b) that the plaintiffs and Carmen Vda. de Cruz be ordered
NAVOTAS and Bautista, jointly and severally, to pay plaintiffs to accept the sum of ₱1,600,000.00 representing the option
the reasonable compensation for the use of the premises in c) Co-defendants be ordered to pay her moral as well as money for the purchase of the property subject of the lease
the amount of at least ₱10,000.00 a month from October exemplary damages in the amount which this Honorable contract specifically that which is now covered by
1990 up to the filing of this Complaint, totalling ₱500,000.00, Court may deem just and proper; TRANSFER CERTIFICATE OF TITLE NO. R-85099 and to
as well as ₱10,000.00 every month thereafter until execute and sign the necessary deed of conveyance
defendants shall have vacated and surrendered the premises d) Co-defendants, instead of answering defendants, be, therefore in favor of defendant Navotas and/or Bautista; and
to the plaintiffs. likewise, ordered to pay the plaintiffs, the rentals in arrears (c) that plaintiffs and Carmen Vda. de Cruz be ordered and
over the premises which now amounts to ₱147,000.00. condemned, jointly and severally, to pay defendants Navotas
4. Under the Fourth Cause of Action, ordering defendants and Bautista moral and exemplary damages of not less than
NAVOTAS and Bautista, jointly and severally, to pay the BOTH AS TO COUNTERCLAIM AND CROSS-CLAIM ₱80,000.00, attorney’s fees and litigation expenses of not
plaintiffs exemplary damages of at least ₱50,000.00 or such less than ₱50,000.00, and the costs of suit.
amount as the Honorable Court may deem just and equitable a) Plaintiffs and co-defendants be ordered, jointly and
in the premises; and severally, to reimburse answering defendant the sum of Herein answering defendants further pray for such other
₱30,000.00 which the latter paid her counsel as and for reliefs and remedies available in the premises.31
5. Under the Fifth Cause of Action, ordering defendants attorney’s fees for unnecessarily dragging her into this suit
NAVOTAS and Bautista to pay plaintiff attorney’s fees and including the amount of ₱1,000.00 which she will pay her In the meantime, Carmen Cruz died intestate on November
expenses of litigation in such amount as may be established lawyer for every appearance; 20, 1995 at the age of 97. She was survived by the plaintiffs
during the trial, but not less than ₱35,000.00. as her heirs.32
b) Likewise, the costs of suit and other litigation expenses.
Plaintiffs pray for such other reliefs just and equitable in the On March 7, 2000, the trial court rendered judgment in favor
premises.26 Other reliefs and remedies reasonable under the premises of the NIC and Bautista. The fallo of the decision reads:
are similarly prayed for.28
In her answer with cross-claim, Carmen Cruz alleged, inter WHEREFORE, premises considered, judgment is hereby
alia, that she was willing to be made a party-plaintiff, although In its amended answer, NIC alleged that its July 30, 1977 rendered:
she was initially reluctant to become one because of the Supplementary Lease Agreement and Contract of Lease
burden of a court hearing; she admitted that the plaintiffs were valid, whereas the deed of absolute sale with a) Affirming the validity of the Contract of Lease and the
were the co-owners of the property; Bautista was granted an assumption of mortgage executed by Carmen Cruz in favor of Supplementary Lease Agreement, both dated 30 July 1977,
"exclusive option to buy" the leased property at the the plaintiffs was null and void for being simulated and including the provision granting defendants exclusive option
ridiculously low fixed price of ₱1,600,000.00, which, fraudulent. NIC and Bautista further alleged that it was to buy the subject property.
according to Carmen Cruz, was an option unsupported by exercising its option to buy the subject property now covered
any consideration; hence, null and void.27 by TCT No. 85099;29 it, likewise, offered ₱1,600,000.00 as b) Affirming the full rental payments made by defendants
consideration for the sale to be paid upon the execution of a Navotas and Bautista for the lease of the subject property
Carmen Cruz prayed that, after due proceedings, judgment deed of transfer.30 until the expiration thereof.
be rendered in her favor:
NIC and Bautista prayed that, after due proceeding, judgment c) Denying the claims for actual and compensatory, moral
WHEREFORE, it is most respectfully prayed that the be rendered in their favor, thus: and exemplary damages as well as attorney’s fees interposed
complaint as against answering defendant be dismissed, and by plaintiffs against defendants.
that: WHEREFORE, premises considered, herein answering
defendants respectfully prayed that the complaint be d) Denying the claims for moral and exemplary damages
AS TO THE CROSS-CLAIM dismissed for lack of merit. interposed by defendants Navotas and Bautista against
plaintiffs.
a) The Contract of Lease and the Supplemental Lease On the Counterclaim: (a) that the "Contract of Lease" and the
Contract be declared null and void due to vitiated consent; "Supplementary Lease Agreement" be declared valid, legal e) The Deed of Absolute Sale with Assumption of Mortgage is
and binding between Carmen Vda. de Cruz and defendants hereby declared null and void as far as it prejudiced and is
b) In the event that monetary judgment be rendered by this Navotas and Bautista, as well as their respective heirs, adversely affecting the rights of defendants Navotas and
Honorable Court against answering defendant in favor of the successors or assigns, while the "Deed of Absolute Sale with Bautista on the portion thereof leased to them. The plaintiffs,
plaintiffs, her co-defendants, Navotas Industrial Corporation Assumption of Mortgage" be declared null and void so far as as heirs of defendant Cruz, are hereby ordered to accept the
it prejudiced and adversely affected the rights of defendants sum of ₱1,600,000.00 representing the option money for the
34
Sales – Chapter 3-12 Cases
purchase of the subject property subject of the lease contract WITH MRS. CRUZ IN 1977, THEY WERE DEALING WITH
specifically that which is now covered by Transfer Certificate ONE WHO WAS NO LONGER THE OWNER OF THE VI
of Title No. R-85099 and to execute and sign the necessary PROPERTY WHO CAN BIND THE SAME UNDER THE
deed of conveyance therefor in favor of defendants Navotas QUESTIONED LEASE CONTRACTS. THE TRIAL COURT ERRED IN HOLDING THAT THE
and/or Bautista. OPTION CONTRACT FOR APPELLEES’ PURCHASE OF
III THE SUBJECT PROPERTY WAS SUPPORTED BY A
f) Ordering plaintiffs to pay defendants Navotas and Bautista SEPARATE CONSIDERATION AND THUS VALID AND
₱20,000.00 by way of reasonable attorney’s fees. THE TRIAL COURT ERRED IN HOLDING THAT THE BINDING ON APPELLANTS.
CONSENT OF MRS. CRUZ TO THE SUBJECT LEASE
Costs against the plaintiffs.33 CONTRACTS HAD NOT BEEN VITIATED BY UNDUE AND VII
IMPROPER PRESSURE AND INFLUENCE ON THE PART
The trial court declared that when defendant Carmen Cruz OF APPELLEES CONSIDERING THAT: THE TRIAL COURT ERRED IN NOT HOLDING APPELLEES
executed the July 30, 1977 Supplementary Lease Agreement LIABLE TO APPELLANTS FOR ACTUAL AND
and Contract of Lease, she was still the owner of the A. THE UNDISPUTED EVIDENCE ON RECORD READILY COMPENSATORY DAMAGES CONSISTING OF THE
property; as such, NIC was not bound by the deed of sale BEARS OUT THE UNDUE AND IMPROPER PRESSURE REASONABLE RENTALS ON THE PROPERTY FROM 2
with assumption of mortgage executed by Carmen Cruz AND INFLUENCE EXERTED BY APPELLEES ON MRS. OCTOBER 1990 UNTIL THE RETURN THEREOF TO
because it was not a party thereto; and that such deed was CRUZ TO OBTAIN HER CONSENT TO THE SUBJECT APPELLANTS.
not registered with the Office of the Register of Deeds. The LEASE CONTRACTS;
trial court ruled that the plaintiffs failed to prove fraud and VIII
undue influence on Carmen Cruz and/or that NIC took B. THE VERY TERMS AND CONDITIONS OF THE LEASE
advantage of her mental weakness. The RTC ruled that only CONTRACTS, WHICH ARE GROSSLY THE TRIAL COURT ERRED IN ABSOLVING APPELLEES
Carmen Cruz had the right to rescind the contracts of lease DISADVANTAGEOUS TO MRS. CRUZ, POINT TO OF LIABILITY TO APPELLANTS FOR MORAL AND
and supplementary lease agreement. The option to buy the APPELLEES’ USE OF UNDUE PRESSURE AND EXEMPLARY DAMAGES AND ATTORNEY’S FEES.34
property granted to NIC was supported by a consideration, INFLUENCE ON HER TO OBTAIN HER CONSENT TO THE
more specifically the ₱42,000.00 rental payment it made SUBJECT LEASE CONTRACTS. On July 18, 2003, the CA rendered judgment granting the
upon the execution of the said contracts. appeal, and reversing the decision of the RTC. The CA ruled
IV that the appellees had constructive notice of the Deed of Sale
The plaintiffs appealed the decision to the CA wherein they with Assumption of Mortgage, which Carmen Cruz executed
alleged that: THE TRIAL COURT ERRED IN NOT HOLDING, IN THE in favor of the appellants, based on the affidavit of adverse
ALTERNATIVE, THAT THE SUBJECT LEASE CONTRACTS claim annotated on June 29, 1977 at the dorsal portion of
I WERE RENDERED RESCINDED BY REASON OF TCT No. 81574. The CA declared that the adverse claim
APPELLEES’ MATERIAL BREACHES OF THE TERMS AND annotated at the dorsal portion of the said title continued to
THE TRIAL COURT ERRED IN HOLDING THAT CONDITIONS CONSIDERING THAT: be effective and remained a lien until cancelled. The CA held
APPELLEES WERE NOT BOUND BY THE DEED OF that the option granted to the appellee NIC to purchase the
ABSOLUTE SALE OF REALTY WITH ASSUMPTION OF A. APPELLEES HAD ADMITTEDLY FAILED TO property was not effective because there was no
MORTGAGE WHICH APPELLANTS ANNOTATED AS AN CONSTRUCT THE SLIPWAYS AS REQUIRED UNDER THE consideration therefor, apart from NIC’s rental payments.
ADVERSE CLAIM ON THE CERTIFICATE OF TITLE OF LEASE CONTRACT; Besides, the CA emphasized, when Carmen Cruz executed
THE PROPERTY AS EARLY AS 30 JUNE 1977 BEFORE the July 30, 1977 Supplementary Lease Agreement and
APPELLEES REGISTERED THE QUESTIONED LEASE B. THE EVIDENCE FULLY ESTABLISHES THAT Contract of Lease, she was no longer the owner of the
CONTRACTS ON 14 SEPTEMBER 1977. APPELLEES HAVE NOT PAID THE RENTALS DUE ON property.
THE PROPERTY SINCE 1991.
II The CA denied NIC’s motion for reconsideration of the said
V decision; hence, it filed the instant petition for review on
THE TRIAL COURT ERRED IN COMPLETELY IGNORING certiorari, alleging that:
THE OVERWHELMING EVIDENCE ON RECORD THE TRIAL COURT ERRED IN DECLARING THE DEED OF
SHOWING THAT APPELLEES HAD ACTUAL AND ABSOLUTE SALE WITH ASSUMPTION OF MORTGAGE AS A.
CONSTRUCTIVE NOTICE OF THE SALE OF THE NULL AND VOID AS AGAINST APPELLEES CONSIDERING
SUBJECT PROPERTY TO THE CRUZ CHILDREN IN 1974, THAT THE SAME HAS BEEN CONFIRMED AND THE COURT OF APPEALS COMMITTED A CLEAR AND
AND THUS KNEW OR OUGHT TO HAVE KNOWN THAT IN RECOGNIZED IN SUBJECT TRANSFERS AFFECTING THE REVERSIBLE ERROR WHEN IT DECLARED THAT THE
EXECUTING THE QUESTIONED LEASE CONTRACTS SAME PROPERTY. QUESTIONED LEASE CONTRACTS WERE NULL AND
35
Sales – Chapter 3-12 Cases
VOID, IT APPEARING IN AN ADVERSE CLAIM in the Office of the Register of Deeds. The respondents cited protect the interest of a person over a piece of real property
ANNOTATED ON THE CERTIFICATE OF TITLE OF the ruling of this Court in Sajonas v. Court of Appeals36 to and serves as a notice and warning to third parties dealing
CARMEN VDA. DE CRUZ THAT SHE WAS NO LONGER support their claim. with said property that someone is claiming an interest on the
THE OWNER OF THE PROPERTY SUBJECT MATTER same or a better right than the registered owner thereof. A
THEREOF WHEN THE LEASE WAS EXECUTED ON JULY On the second issue, the petitioner avers that the exclusive subsequent transaction involving the property cannot prevail
30, 1977. option granted to it by Carmen Cruz under the Supplementary over the adverse claim which was previously annotated in the
Lease Agreement was essentially a mutual promise to buy certificate of title of the property. Here, the records are
B. and sell, equivalent to a reciprocal contract under the first obvious, the notice of adverse claim executed on June 29,
paragraph of Article 1479 of the New Civil Code. But in the 1977 was annotated on the title on June 30, 1977, that is, one
THE COURT OF APPEALS COMMITTED A CLEAR AND same breath, the petitioner argues that its exclusive option to month prior to the signing of the disputed lease contracts on
REVERSIBLE ERROR WHEN IT RULED THAT THE buy the property for ₱1,600,000.00 was supported by a July 30, 1977. Said contracts of lease were belatedly
OPTION TO BUY THE LEASED PROPERTY CONTAINED consideration apart from the said amount. The petitioner annotated two months after its execution or on September 14,
IN THE SUPPLEMENTARY LEASE CONTRACT IS NOT insists that the ₱42,000.00 which it paid to Carmen Cruz as 1977 only, after appellees were allegedly warned by
VALID AND BINDING FOR LACK OF CONSIDERATION rental upon the execution of the Supplementary Lease CARMEN that her children are desirous of the property
AND CAPACITY OF CARMEN VDA. DE CRUZ TO CONVEY Agreement was "advance money," which motivated Carmen leased in their favor. To say the least, this warning from
THE SAME. Cruz to grant the option to the petitioner. CARMEN should have aroused appellees’ suspicion
regarding the status of the prime property they intend to lease
C. On the third issue, the petitioner argues that the respondents’ for another fifteen (15) years. …38
action was barred by the order of the RTC in Civil Case No.
THE COURT OF APPEALS COMMITTED A CLEAR AND C-7040 dismissing the complaint and complaint-in- Section 110 of Act No. 496 was the law in force when
REVERSIBLE ERROR WHEN IT FAILED TO RECOGNIZE A intervention therein, based on a compromise agreement of Carmen Cruz executed the Deed of Sale with Assumption of
PRIOR JUDGMENT BASED ON A COMPROMISE AS A Carmen Cruz and petitioner NIC. Mortgage, and when the respondents executed the affidavit
BAR TO THE PROCEEDINGS IN THIS INSTANT CASE.35 of adverse claim and presented it to the Register of Deeds on
The Ruling of the Court June 30, 1977. The petitioner’s reliance on the said provision
On the first issue, the petitioner avers that the adverse claim is misplaced. Indeed, the Register of Deeds acted in accord
annotated at the dorsal portion of TCT No. 81574 was The annotation of an adverse claim is a measure designed to with Section 110 of Act No. 496 when he inscribed the
ineffective because the respondents failed to submit to the protect the interest of a person over a part of real property, affidavit of adverse claim at the dorsal portion of TCT No.
Register of Deeds the owner’s duplicate of TCT No. 81574, and serves as a notice and warning to third parties dealing 81574, despite the non-production of the owner’s duplicate of
as mandated by Section 110 of Act No. 496. The annotation with the said property that someone is claiming an interest TCT No. 81574 simultaneously with the presentation of the
of the adverse claim in the Office of the Register of Deeds on over it or has a better right than the registered owner affidavit of adverse claim. The law reads:
June 29, 1977 on TCT No. 81574 despite such failure to thereof.37
present the owner’s duplicate of the said title rendered such SEC. 110. Whoever claims any part or interest in registered
inscription ineffectual, not binding on it and Carmen Cruz. On the first issue, we agree with the ruling of the CA that the land adverse to the registered owner, arising subsequent to
Hence, the petitioner posits, Carmen Cruz remained the petitioner had constructive notice of the Deed of Sale with the date of the original registration, may, if no other provision
lawful owner of the property. Even Carmen Cruz maintained Assumption of Mortgage executed by Carmen Cruz in favor is made in this Act for registering the same, make a
that she was the owner of the property in her complaint in of the respondents. The affidavit of adverse claim the statement in writing setting forth fully his alleged right or
Civil Case No. C-7040 filed after the execution of the deed of respondents executed on June 29, 1977 was annotated at interest, and how or under whom acquired, and a reference to
absolute sale with assumption of real estate mortgage; she the dorsal portion of TCT No. 81574 on June 30, 1977, to wit: the volume and page of the certificate of title of the registered
even executed the July 30, 1977 Supplementary Lease owner, and a description of the land in which the right or
Agreement and Contract of Lease in its favor. According to A review of the facts and circumstances in the case at bar interest is claimed.
the petitioner, the said deed of sale was fictitious as, in fact, it reveals that at the time the Supplementary Lease Agreement
was rejected by Carmen Cruz. and Contract of Lease both dated July 30, 1977 were The statement shall be signed and sworn to, and shall state
executed by and between CARMEN and herein appellees, the adverse claimant’s residence, and designate a place at
For their part, the respondents aver that the petitioner had CARMEN was apparently no longer the owner of the land which all notices may be served upon him. This statement
constructive notice of the said sale, based on the inscription covered by TCT No. 81574 subject of this controversy. shall be entitled to registration as an adverse claim, and the
of the affidavit of adverse claim on June 29, 1977 at the Obviously, appellees cannot turn a blind eye on the court, upon a petition of any party-in-interest, shall grant a
dorsal portion of TCT No. 81574. Besides, the respondents inscription found on CARMEN’s certificate of title at the time speedy hearing upon the question of the validity of such
posit, Cipriano Bautista even admitted having known of the the Supplementary Lease Agreement and Contract of Lease adverse claim and shall enter such decree therein as justice
said adverse claim before the July 30, 1977 Contract of were signed on July 30, 1977. Basic is the rule that the and equity may require. If the claim is adjudged to be invalid,
Lease and Supplementary Lease Agreement were registered annotation of an adverse claim is a measure designed to the registration shall be cancelled. If in any case the court,
36
Sales – Chapter 3-12 Cases
after notice and hearing, shall find that a claim thus registered This Court explained the rationale of the requirement in L.P. said property that someone is claiming an interest on the
was frivolous or vexatious, it may tax the adverse claimant Leviste & Company, Inc. v. Noblejas:39 same or a better right than the registered owner thereof.42
double or treble costs in its discretion.
The basis of respondent Villanueva’s adverse claim was an Moreover, on June 29, 1977, the balance of Mariano Cruz
Irrefragably, the Deed of Sale with Assumption of Mortgage agreement to sell executed in her favor by Garcia Realty. An and Gabriel Cruz’s account with the CBC had already been
which Carmen Cruz executed on December 31, 1974 was a agreement to sell is a voluntary instrument as it is a willful act paid, presumably by Mariano Cruz; and the CBC had
voluntary act; and under Section 50 of the law, the act of of the registered owner. As such voluntary instrument, executed a cancellation of real estate mortgage. However,
registration shall be the operative act to convey and affect the Section 50 of Act No. 496 expressly provides that the act of the said deed was inexplicably not presented to the Register
land. Indeed, Section 55 of Act No. 496 provides that the registration shall be the operative act to convey and affect the of Deeds for registration.
presentation of the owner’s duplicate certificate of title for the land. And Section 55 of the same Act requires the
registration of any voluntary instrument is required: presentation of the owner’s duplicate certificate of title for the The general rule is that a person dealing with registered land
registration of any deed or voluntary instrument. As the is not required to go behind the register to determine the
SEC. 55. No new certificate of title shall be entered, no agreement to sell involves an interest less than an estate in condition of the property. However, such person is charged
memorandum shall be made upon any certificate of title by fee simple, the same should have been registered by filing it with notice of the burden on the property which is noted on
the register of deeds, in pursuance of any deed or other with the Register of Deeds who, in turn, makes a brief the face of the register or certificate of title.43 A person who
voluntary instrument, unless the owner’s duplicate certificate memorandum thereof upon the original and owner’s duplicate deals with registered land is bound by the liens and
is presented for such indorsement, except in cases expressly certificate of title. The reason for requiring the production of encumbrances including adverse claim annotated therein.44
provided for in this Act, or upon the order of the court for the owner’s duplicate certificate in the registration of a
cause shown; and whenever such order is made, a voluntary instrument is that, being a willful act of the In the present action, the petitioner caused the annotation of
memorandum thereof shall be entered upon the new registered owner, it is to be presumed that he is the July 30, 1977 Supplementary Lease Agreement and
certificate of title and upon the owner’s duplicate: Provided, interested in registering the instrument and would willingly Contract of Sale only on September 14, 1977, long after the
however, That in case the mortgagee refuses or fails to surrender, present or produce his duplicate certificate of title annotation of the respondents’ adverse claim at the dorsal
deliver within a reasonable time to the register of deeds the to the Register of Deeds in order to accomplish such portion of TCT No. 81574 on June 30, 1977. Thus, as of that
duplicate or copy of the certificate of title surrendered by the registration. …40 date, the petitioner had constructive knowledge of the Deed
owner, after advice by said officer, in order to enable him to of Sale with Assumption of Mortgage Carmen Cruz executed
register or annotate thereon another real right acquired by However, in this case, Carmen Cruz had ordered the CBC, on December 31, 1974 in favor of her children. Even before
said owner, the record or annotation made on the certificate the mortgagee and custodian of the owner’s duplicate of TCT July 30, 1977, the petitioner had knowledge that Carmen
in the register book shall be valid for all legal purposes. No. 81574, not to surrender the owner’s duplicate of the said Cruz was no longer the owner of the property, and had no
title to the Register of Deeds. The latter thus acted in accord more right to execute the July 30, 1977 Supplementary Lease
The production of the owner’s duplicate certificate whenever with law when the affidavit of adverse claim was inscribed at Agreement and Contract of Lease. The registration of the
any voluntary instrument is presented for registration shall be the dorsal portion of TCT No. 81574 on June 30, 1977. said lease contracts was of no moment, since it is understood
conclusive authority from the registered owner to the register Indeed, this Court ruled in L.P. Leviste & Company, Inc. v. to be without prejudice to the better rights of third parties.45
of deeds to enter a new certificate or to make a memorandum Noblejas41 that:
of registration in accordance with such instrument, and the While it is true that in the complaint and amended complaint
new certificate or memorandum shall be binding upon the … However, where the owner refuses to surrender the in Civil Case No. C-7040, Carmen Cruz alleged that she was
registered owner and upon all persons claiming under him, in duplicate certificate for the annotation of the voluntary the owner-lessor of the
favor of every purchaser for value and in good faith: Provided, instrument, the grantee may file with the Register of Deeds a property, such allegation cannot detract from the fact that the
however, That in all cases of registration procured by fraud statement setting forth his adverse claim, as provided for in property had already been registered under the names of the
the owner may pursue all his legal and equitable remedies Section 110 of Act No. 496. In such a case, the annotation of respondents under TCT No. 11272, later cancelled by TCT
against the parties to such fraud, without prejudice, however, the instrument upon the entry book is sufficient to affect the No. R-11830. The petitioner was informed by the respondents
to the rights of any innocent holder for value of a certificate of real estate to which it relates, although Section 72 of Act No. that they were the registered owners of the property.
title: And provided, further, That after the transcription of the 496 imposes upon the Register of Deeds the duty to require Moreover, the already aging Carmen Cruz and her children
decree of registration under this Act procured by the the production by the Registered owner of his duplicate had a domestic quarrel, and animosity that caused her to go
presentation of a forged duplicate certificate, or of a forged certificate for the inscription of the adverse claim. The into seclusion; she thought then that her children had
deed or other instrument, shall be null and void. In case of the annotation of an adverse claim is a measure designed to abandoned her. The attendant circumstances must have
loss or theft of an owner’s duplicate certificate, notice shall be protect the interest of a person over a piece of real property influenced Carmen Cruz to erroneously allege in her
sent by the owner or by someone in his behalf to the register where the registration of such interest or right is not, complaint that she was the owner of the property.46
of deeds of the province in which the land lies as soon as the otherwise, provided for by the Land Registration Act, and
loss or theft is discovered. serves as a notice and warning to third parties dealing with

37
Sales – Chapter 3-12 Cases
Even then, on February 23, 1988, Carmen Cruz executed an executed in favor of the plaintiffs involving the subject parcel ART. 1479. A promise to buy and sell a determinate thing for
Affidavit in which she swore that she had sold the property to of land with the Registry of Deeds; a price certain is reciprocally demandable.
her children:
2.7. In order to protect their rights and interests over the An accepted unilateral promise to buy or to sell a determinate
3. That among the parcels of land which I have sold was that subject property, the plaintiffs, through their appointed thing for a price certain is binding upon the promissor if the
parcel located in Barrio Almacen, Navotas, Rizal, then attorney-in-fact, Mariano A. Cruz, annotated an adverse claim promise is supported by a consideration distinct from the
covered by Transfer Certificate of Title No. 81574 of the on the title which was then still under answering defendant’s price.
Register of Deeds of Rizal in favor of my children Serafin D. name, as a cautionary notice to third persons and the whole
Cruz, Mariano D. Cruz, Rogelio D. Cruz, Sr. Carmencita Cruz world that said title has been transferred by answering In the first place, the petitioner insisted in its pleadings in the
and Sr. Mary Carmellas as vendees, with the agreement that defendant in favor of the plaintiffs herein and that any court a quo that under the Supplementary Lease Agreement
the then existing mortgage with the China Banking voluntary dealing thereon shall be considered subject to the and Contract of Lease, it was granted the exclusive option to
Corporation shall be assumed and settled by said vendees, said adverse claim.49 purchase the property leased. The petitioner maintained its
as embodied in a document entitled "Deed of Absolute Sale theory of the case in the CA. The petitioner cannot change its
of Realty with Assumption of Mortgage," which I executed on Carmen Cruz also alleged, in her amended complaint in Civil theory, and claim this time that it and Carmen Cruz entered
December 31, 1974 and entered in the notarial register of Case No. C-7040, that the July 30, 1977 Contract of Lease into a promise to buy and sell the property leased.53
Notary Public P. Dario Guevarra, Jr. as Doc. No. 198, Page and Supplementary Lease Agreement she executed in favor
No. 41, Book No. 198, Series of 1975.47 of the petitioner were fraudulent.50 Considering that Carmen Cruz was no longer the owner of
the property when she executed the July 30, 1977
… In her answer to the amended complaint in the court a quo, Supplementary Lease Agreement and Contract of Lease, and
Carmen Cruz alleged that the defendant therein (now the that the respondents had acquired ownership over the
7. That in view of these developments and considering my petitioner) was granted an "exclusive option to buy the leased property as of December 31, 1974 (which the petitioner had
advanced age and present physical condition and now property at the ridiculously low price of ₱1,600,000.00, constructive knowledge of since June 30, 1977), the
realizing that I may have been unduly taken advantage of by payable over an unspecified period – an option unsupported petitioner’s claim that it had the option to buy the property or
some parties to promote their own selfish interests, I now by any consideration – hence, null and void."51 She to compel the respondents to sell the property to it has no
hereby execute this sworn statement and hereby affirm the elaborated that: legal and factual basis.
validity of the sale of said parcel of land covered by TCT No.
81574 of the Register of Deeds of Rizal and hereby state that 15. That the above-quoted provision is not only a foolery, Even after a careful study of the merits of the petition, the
said sale was entered into by me of my own free will and for trickery and a product of deception because the exercise of Court finds that the petitioner’s claim is untenable. The
valuable consideration.48 the "option" is not fixed – the same maybe conveniently relevant portions of the Supplementary Lease Agreement
exercised by the defendant at anytime up to the year 2005. read:
In her answer to the respondents’ amended complaint in the Even the fixing of the sum worded as "flat sum" of One Million
trial court, Carmen Cruz reiterated that she had sold the Six Hundred Thousand – the valuation fifteen (15) years, 4. The LESSEE is hereby granted an exclusive option to buy
property to her children: hence, (2005) without providing for the "inflation and the property including all improvements already made by the
deflation" of the currency is grossly prejudicial and unfair. LESSEE (slipways and camarines) subject matter of this
2.5. On 31 December 1974, she sold the subject property to Moreover, the provision which states that if and when contract comprising SIX THOUSAND NINE HUNDRED
the plaintiffs for valuable consideration, free from all liens and defendants finally decides to exercise their option during the FORTY-NINE Point FIVE Square Meters (6,949.5) which is
encumbrances and claim of third parties, except that lifetime of the Lessor, the lessee will continue paying the one-half portion of the area covered by TCT No. 81574 and
pertaining to a real estate mortgage with China Banking rentals is not only illogical, untrue and deceptive, the same same property subject matter of this contract should also be
Corporation as evidenced by a notarized "Deed of Absolute being used mainly as a ploy to win the sympathy and titillate equally divided with one-half frontage along M. Naval Street
Sale of Realty with Assumption of Mortgage" dated 31 the ego of the old woman. It is rather unbelievable that being and along the Navotas River Bank shoreline during the period
December 1974, a photocopy of which is hereto attached and already the owner, defendants will still pay the rentals. This, of the lease. The price of the property is agreed to be fixed for
made an integral part hereof as Annex "B"; to our mind, is the height of hyprocracy.52 the duration of the Option to Buy at a flat sum of ONE
MILLION SIX HUNDRED THOUSAND PESOS
2.6. After she sold the subject lot to the plaintiffs herein, the On the second issue, we reject the petitioner’s contention that (₱1,600,000.00), Philippine Currency, payable over a period
latter tried to effect the registration and annotation of the said the exclusive option granted to it by Carmen Cruz under the to be mutually agreed upon. Should the LESSEE exercise the
transfer with the Registry of Deeds of Rizal sometime in 28 Supplementary Lease Agreement is essentially a mutual option to buy during the lifetime of the LESSOR, the LESSEE
June 1977 but China Banking Corporation, the mortgagee, promise to buy and sell, equivalent to a reciprocal contract will continue to pay the monthly rental to the LESSOR during
through its legal counsel, Atty. Arsenio Sy Santos, refused to under the first paragraph of Article 1479 of the New Civil her lifetime.
release the title thus the delay in the registration of the said Code, which reads:
"Deed of Sale with Assumption of Mortgage" which she
38
Sales – Chapter 3-12 Cases
5. The LESSEE shall pay to the LESSOR the sum of FORTY- On the third issue, the respondents’ action in the court a quo
TWO THOUSAND (₱42,000.00) PESOS upon signing of this was not barred by the order of the RTC dismissing the
contract as consideration thereof, to be applied as against the complaint of Carmen Cruz, and the respondents’ complaint-
rental for the period from October 1, 1990 to September 30, in-intervention in Civil Case No. 5114. Contrary to the
1991.54 petitioner’s claim, Carmen Cruz (the plaintiff therein) and the
petitioner (the defendant therein) did not enter into any
It must be stressed that an option contract is a contract compromise agreement in the said case. Moreover, the
granting a privilege to buy or sell within an agreed time and at dismissal of the complaint, and, consequently, the
a determined price. Such a contract is a separate and distinct respondents’ complaint-in-intervention was upon motion of
contract from the time the parties may enter into upon the plaintiff Carmen Cruz and without prejudice.
construction of the option.55 In Carceller v. Court of
Appeals,56 the Court held that an option contract is a IN LIGHT OF ALL THE FOREGOING, the petition is DENIED
preparatory contract in which one party grants to the other, for lack of merit. The Decision of the Court of Appeals in CA-
for a fixed period and under specified conditions, the power to G.R. CV No. 69818 is AFFIRMED. Costs against the
decide, whether or not to enter into a principal contract. The petitioner.
Court further stated that:
SO ORDERED.
… It binds the party who has given the option, not to enter
into the principal contract with any other person during the
period designated, and, within that period, to enter into such
contract with the one to whom the option was granted, if the
latter should decide to use the option. It is a separate
agreement distinct from the contract which the parties may
enter into upon the consummation of the option.57

It is only when the option is exercised may a sale be


perfected.58 An option contract needs to be supported by a
separate consideration. The Court defined consideration for
an option in Bible Baptist Church v. Court of Appeals,59 as
follows:

… The consideration need not be monetary but could consist


of other things or undertakings. However, if the consideration
is not monetary, these must be things or undertakings of
value, in view of the onerous nature of the contract of option.
Furthermore, when a consideration for an option contract is
not monetary, said consideration must be clearly specified as
such in the option contract or clause.

In the present case, there was no given period for the


petitioner to exercise its option; it had yet to be determined
and fixed at a future time by the parties, subsequent to the
execution of the Supplementary Lease Agreement. There
was, likewise, no consideration for the option. The amount of
₱42,000.00 paid by the petitioner to Carmen Cruz on July 30,
1977 was payment for rentals from October 1, 1990 to
September 30, 1991, and not as a consideration for the
option granted to the petitioner.

39
Sales – Chapter 3-12 Cases
PHILIPPINE NATIONAL OIL COMPANY AND PNOC requisite qualification to own land in the the obligation that each of the contracting party
DOCKYARD & ENGINEERING CORPORATION, Philippines.12chanrobleslaw assumed.27 Since Keppel was already a Filipino-owned
Petitioners, v. KEPPEL PHILIPPINES HOLDINGS, INC., corporation, it satisfied the condition that entitled it to
Respondent. Together with Keppel's lease rights and option to purchase the land.28chanrobleslaw
purchase, Lusteveco warranted not to sell the land or
Before the Court is a petition for review on certiorari filed assign its rights to the land for the duration of the lease Failing to secure a reconsideration of the CA decision,29
under Rule 45 of the Rules of Court, appealing the unless with the prior written consent of Keppel.13 PNOC filed the present Rule 45 petition before this Court
decision dated 19 December 20111 and resolution dated Accordingly, when the petitioner Philippine National Oil to assail the CA rulings.
14 May 20122 of the Court of Appeals (CA) in CA-G.R. Corporation14 (PNOC) acquired the land from Lusteveco
CV No. 86830. These assailed CA rulings affirmed in toto and took over the rights and obligations under the THE PARTIES' ARGUMENTS and THE ISSUES
the decision dated 12 January 20063 of the Regional Trial agreement, Keppel did not object to the assignment so
Court (RTQ of Batangas City, Branch 84, in Civil Case No. long as the agreement was annotated on PNOC's title.15 PNOC argues that the CA failed to resolve the
7364. With PNOC's consent and cooperation, the agreement constitutionality of the agreement. It contends that the
was recorded as Entry No. 65340 on PNOC's Transfer of terms of the agreement amounted to a virtual sale of the
Certificate of Title No. T-50724.16chanrobleslaw land to Keppel who, at the time of the agreement's
THE FACTS enactment, was a foreign corporation and, thus, violated
The Case and the Lower Court Rulings the 1973 Constitution.
The 1976 Lease Agreement and Option to Purchase
On 8 December 2000, Keppel wrote PNOC informing the Specifically, PNOC refers to (a) the 25-year duration of
Almost 40 years ago or on 6 August 1976, the respondent latter that at least 60% of its shares were now owned by the lease that was automatically renewable for another 25
Keppel Philippines Holdings, Inc.4 (Keppel) entered into a Filipinos17 Consequently, Keppel expressed its readiness years30; (b) the option to purchase the land for a nominal
lease agreement5 (the agreement) with Luzon to exercise its option to purchase the land. Keppel consideration of P100.00 if the option is exercised anytime
Stevedoring Corporation (Lusteveco) covering 11 reiterated its demand to purchase the land several times, between the 25th and the 30th year of the lease31; and
hectares of land located in Bauan, Batangas. The lease but on every occasion, PNOC did not favourably (c) the prohibition imposed on Lusteveco to sell the land
was for a period of 25 years for a consideration of P2.1 respond.18chanrobleslaw or assign its rights therein during the lifetime of the
million.6 At the option of Lusteveco, the rental fee could lease.32 Taken together, PNOC submits that these
be totally or partially converted into equity shares in To compel PNOC to comply with the Agreement, Keppel provisions amounted to a virtual transfer of ownership of
Keppel.7chanrobleslaw instituted a complaint for specific performance with the the land to an alien which act the 1973 Constitution
RTC on 26 September 2003 against PNOC.19 PNOC prohibited.
At the end of the 25-year Jease period, Keppel was given countered Keppel's claims by contending that the
the "firm and absolute option to purchase8the land for agreement was illegal for circumventing the constitutional PNOC claims that the agreement is no different from the
P4.09 million, provided that it had acquired the necessary prohibition against aliens holding lands in the lease contract in Philippine Banking Corporation v. Lui
qualification to own land under Philippine laws at the time Philippines.20 It further asserted that the option contract She,33 which the Court struck down as unconstitutional.
the option is exercised.9 Apparently, when the lease was void, as it was unsupported by a separate valuable In Lui She, the lease contract allowed the gradual
agreement was executed, less than 60% of Keppel's consideration.21 It also claimed that it was not privy to the divestment of ownership rights by the Filipino owner-
shareholding was Filipino-owned, hence, it was not agreement.22chanrobleslaw lessor in favour of the foreigner-lessee.34 The
constitutionally qualified to acquire private lands in the arrangement in Lui She was declared as a scheme
country.10chanrobleslaw After due proceedings, the RTC rendered a decision23in designed to enable the parties to circumvent the
favour of Keppel and ordered PNOC to execute a deed of constitutional prohibition.35 PNOC posits that a similar
If, at the end of the 25-year lease period (or in 2001), absolute sale upon payment by Keppel of the purchase intent is apparent from the terms of the agreement with
Keppel remained unqualified to own private lands, the price of P4.09 million.24chanrobleslaw Keppel and accordingly should also be
agreement provided that the lease would be automatically nullified.36chanrobleslaw
renewed for another 25 years.11 Keppel was further PNOC elevated the case to the CA to appeal the RTC
allowed to exercise the option to purchase the land up to decision.25cralawred Affirming the RTC decision in toto, PNOC additionally contends the illegality of the option
the 30th year of the lease (or in 2006), also on the the CA upheld Keppel's right to acquire the land.26 It contract for lack of a separate consideration, as required
condition that, by then, it would have acquired the found that since the option contract was embodied in the by Article 1479 of the Civil Code.37 It claims that the
agreement - a reciprocal contract - the consideration was option contract is distinct from the main contract of lease
40
Sales – Chapter 3-12 Cases
and must be supported by a consideration other than the introduced improvements worth P177 million.55 Taking
rental fees provided in the agreement.38chanrobleslaw THE COURT'S RULING these investments into account and the nature of the
business that Keppel conducts on the land, we find it
On the other hand, Keppel maintains the validity of both I. The constitutionality of the Agreement reasonable that the agreement's terms provided for an
the agreement and the option contract it contains. It extended duration of the lease and a restriction on the
opposes the claim that there was "virtual sale" of the land, The Court affirms the constitutionality of the Agreement. rights of Lusteveco.
noting that the option is subject to the condition that
Keppel becomes qualified to own private lands in the Preserving the ownership of land, whether public or We observe that, unlike in Lui She,56 Lusteveco was not
Philippines.39 This condition ripened in 2000, when at private, in Filipino hands is the policy consistently adopted completely denied its ownership rights during the course
least 60% of Keppel's equity became Filipino-owned. in all three of our constitutions.45 Under the 1935,46 of the lease. It could dispose of the lands or assign its
1973,47 and 198748 Constitutions, no private land shall rights thereto, provided it secured Keppel's prior written
Keppel contends that the agreement is not a scheme be transferred, assigned, or conveyed except to consent.57 That Lusteveco was able to convey the land in
designed to circumvent the constitutional prohibition. individuals, corporations, or associations qualified to favour of PNOC during the pendency of the lease58
Lusteveco was not proscribed from alienating its acquire or hold lands of the public domain. Consequently, should negate a finding that the agreement's terms
ownership rights over the land but was simply required to only Filipino citizens, or corporations or associations amounted to a virtual transfer of ownership of the land to
secure Keppel's prior written consent.40 Indeed, whose capital is 60% owned by Filipinos citizens, are Keppel.
Lusteveco was able to transfer its interest to PNOC constitutionally qualified to own private lands.
without any objection from Keppel.41chanrobleslaw II. The validity of the option contract
Upholding this nationalization policy, the Court has voided II.A
Keppel also posits that the requirement of a separate not only outright conveyances of land to foreigners,49: but An option contract must be supported by a separate
consideration for an option to purchase applies only when also arrangements where the rights of ownership were consideration that is either clearly specified as such in the
the option is granted in a separate contract.42 In the gradually transferred to foreigners.50 In Lui Shui,51 we contract or duly proven by the offeree/promisee.
present case, the option is embodied in a reciprocal considered a 99-year lease agreement, which gave the
contract and, following the Court's ruling in Vda. De foreigner-lessee the option to buy the land and prohibited An option contract is defined in the second paragraph of
Quirino v. Palarca,43 the option is supported by the same the Filipino owner-lessor from selling or otherwise Article 1479 of the Civil Code:ChanRoblesVirtualawlibrary
consideration supporting the main contract. disposing the land, amounted to - Article 14791 x x x An accepted promise to buy or to sell a
a virtual transfer of ownership whereby the owner divests determinate thing for a price certain is binding upon the
From the parties' arguments, the following ISSUES himself in stages not only of the right to enjoy the land promissor if the promise is supported by a consideration
emerge: (Jus possidendi, jus utendi, jus fruendi, and jus abutendi) distinct from the price.
but also of the right to dispose of it (jus disponendi) — An option contract is a contract where one person (the
chanRoblesvirtualLawlibraryFirst, the constitutionality of rights the sum total of which make up ownership.52 offeror/promissor) grants to another person (the
the Agreement, i.e., whether the terms of the Agreement [Emphasis supplied] offeree/promisee) the right or privilege to buy (or to sell) a
amounted to a virtual sale of the land to Keppel that was In the present case, PNOC submits that a similar scheme determinate thing at a fixed price, if he or she chooses to
designed to circumvent the constitutional prohibition on is apparent from the agreement's terms, but a review of do so within an agreed period.59chanrobleslaw
aliens owning lands in the Philippines. the overall circumstances leads us to reject PNOC's
claim. As a contract, it must necessarily have the essential
Second, the validity of the option contract, i.e., whether elements of subject matter, consent, and consideration.60
the option to purchase the land given to Keppel is The agreement was executed to enable Keppel to use the Although an option contract is deemed a preparatory
supported by a separate valuable consideration. land for its shipbuilding and ship repair business.53 The contract to the principal contract of sale,61 it is separate
industrial/commercial purpose behind the agreement and distinct therefrom,62 thus, its essential elements
If these issues are resolved in favour of Keppel, a third differentiates the present case from Lui She where the should be distinguished from those of a
issue emerges - one that was not considered by the lower leased property was primarily devoted to residential sale.63chanrobleslaw
courts, but is critical in terms of determining Keppel's right use.54 Undoubtedly, the establishment and operation of a
to own and acquire full title to the land, i.e., whether shipyard business involve significant investments. In an option contract, the subject matter is the right or
Keppel's equity ownership meets the 60% Filipino-owned Keppel's uncontested testimony showed that it incurred privilege to buy (or to sell) a determinate thing for a price
capital requirement of trie Constitution, in accordance with P60 million costs solely for preliminary activities to make certain,64 while in a sales contract, the subject matter is
the Court's ruling in Gamboa v. Teves.44chanrobleslaw the land suitable as a shipyard, and subsequently the determinate thing itself.65 The consent in an option
41
Sales – Chapter 3-12 Cases
contract is the acceptance by the offeree of the offerer's is one of the stipulations of the lease contract. It claims
promise to sell (or to buy) the determinate thing, i.e., the that a separate consideration is required only when an In Villamor v. CA,76 the parties executed a deed
offeree agrees to hold the right or privilege to buy (or to option to buy is embodied in an independent contract.70 It expressly acknowledging that the purchase price of
sell) within a specified period. This acceptance is different relies on Vda. de Quirino v. Palarca,71 where the Court P70.00 per square meter "was greatly higher than the
from the acceptance of the offer itself whereby the offeree declared that the option to buy the leased property is actual reasonable prevailing value of lands in that place at
asserts his or her right or privilege to buy (or to sell), supported by the same consideration as that of the lease that time."77 The difference between the purchase price
which constitutes as his or her consent to the sales itself: "in reciprocal contracts [such as lease], the and the prevailing value constituted as the consideration
contract. The consideration in an option contract may be obligation or promise of each party is the consideration for for the option contract. Although the actual amount of the
anything of value, unlike in a sale where the purchase that of the other.72chanrobleslaw consideration was not stated, it was ascertainable from
price must be in money or its equivalent.66 There is the contract whose terms evinced the parties' intent to
sufficient consideration for a promise if there is any benefit In considering Keppel's submission, we note that the constitute this amount as consideration for the option
to the offeree or any detriment to the Court's ruling in 1969 in Vda. de Quirino v. Palarca has contract.78 Thus, the Court upheld the validity of the
offeror.67chanrobleslaw been taken out of context and erroneously applied in option contract.79 In the light of the offeree's acceptance
subsequent cases. In 2004, through Bible Baptist Church of the option, the Court further declared that a bilateral
In the present case, PNOC claims the option contract is v. CA73 we revisited Vda. de Quirino v. Palarca and contract to sell and buy was created and that the parties'
void for want of consideration distinct from the purchase observed that the option to buy given to the lessee respective obligations became reciprocally
price for the land.68 The option is incorporated as Palarca by the lessor Quirino was in fact supported by a demandable.80chanrobleslaw
paragraph 5 of the Agreement and reads as separate consideration: Palarca paid a higher amount of
5. If within the period of the first [25] years [Keppel] rent and, in the event that he does not exercise the option When the written agreement itself does not state the
becomes qualified to own land under the laws of the to buy the leased property, gave Quirino the option to buy consideration for the option contract, the offeree or
Philippines, it has the firm and absolute option to the improvements he introduced thereon. These additional promisee bears the burden of proving the existence of a
purchase the above property for a total price of [P- concessions were separate from the purchase price and separate consideration for the option.81 The offeree
4,090,000.00] at the end of the 25th year, discounted at deemed by the Court as sufficient consideration to support cannot rely on Article 1354 of the Civil Code,82 which
16% annual for every year before the end of the 25th the option contract. presumes the existence of consideration, since Article
year, which amount may be converted into equity of 1479 of the Civil Code is a specific provision on option
[Keppel] at book value prevailing at the time of sale, or Vda. de Quirino v. Palarca, therefore, should not be contracts that explicitly requires the existence of a
paid in cash at Lusteveco's option. regarded as authority that the mere inclusion of an option consideration distinct from the purchase
contract in a reciprocal lease contract provides it with the price.83chanrobleslaw
However, if after the first [25] years, [Keppel] is still not requisite separate consideration for its validity. The
qualified to own land under the laws of the Republic of the reciprocal contract should be closely scrutinized and In the present case, none of the above rules were
Philippines, [Keppel's] lease of the above stated property assessed whether it contains additional concessions that observed. We find nothing in paragraph 5 of the
shall be automatically renewed for another [25] years, the parties intended to constitute as a consideration for Agreement indicating that the grant to Lusteveco of the
under the same terms and conditions save for the rental the option contract, separate from that of the purchase option to convert the purchase price for Keppel shares
price which shall be for the sum of P4,090,000.00... and price. was intended by the parties as the consideration for
which sum may be totally converted into equity of [Keppel] Keppel's option to buy the land; Keppel itself as the
at book value prevailing at the time of conversion, or paid In the present case, paragraph 5 of the agreement offeree presented no evidence to support this finding. On
in cash at Lusteveco's option. provided that should Keppel exercise its option to buy, the contrary, the option to convert the purchase price for
Lusteveco could opt to convert the purchase price into shares should be deemed part of the consideration for the
If anytime within the second [25] years up to the [30th] equity in Keppel. May Lusteveco's option to convert the contract of sale itself, since the shares are merely an
year from the date of this agreement, [Keppel] becomes price for shares be deemed as a sufficient separate alternative to the actual cash price.
qualified to own land under the laws of the Republic of the consideration for Keppel's option to buy?
Philippines, [Keppel] has the firm and absolute option to There are, however cases where, despite the absence of
buy and Lusteveco hereby undertakes to sell the above As earlier mentioned, the consideration for an option an express intent in the parties' agreements, the Court
stated property for the nominal consideration of contract does not need to be monetary and may be considered the additional concessions stipulated in an
[P100.00.00]...69 anything of value.74 However, when the consideration is agreement to constitute a sufficient separate
Keppel counters that a separate consideration is not not monetary, the consideration must be clearly specified consideration for the option contract.
necessary to support its option to buy because the option as such in the option contract or clause.75chanrobleslaw
42
Sales – Chapter 3-12 Cases
In Teodoro v. CA,84 the sub-lessee (Teodoro) who was promissor if the promise is supported by a consideration
given the option to buy the land assumed .the obligation What Teodoro, Dijamco, and Bible Baptist Church show is distinct from the price, [emphases supplied]
to pay not only her rent as sub-lessee, but also the rent of that the determination of whether the additional The Court en banc declared that there is no distinction
the sub-lessor (Ariola) to the primary lessor (Manila concessions in agreements are sufficient to support an between these two provisions because the scenario
Railroad Company).85 In other words, Teodoro paid an option contract, is fraught with danger; in ascertaining the contemplated in the second paragraph of
amount over and above the amount due for her own parties' intent on this matter, a court may read too much
occupation of the property, and this amount was found by or too little from the facts before it. Article 1479 is the same as that in the last clause of Article
the Court as sufficient consideration for the option 1324.98 Instead of finding a conflict, Sanchez v. Rigos
contract.86chanrobleslaw For uniformity and consistency in contract interpretation, harmonised the two provisions, consistent with the
the better rule to follow is that the consideration for the established rules of statutory
In Dijamco v. CA,87 the spouses Dijamco failed to pay option contract should be clearly specified as such in the construction.99chanrobleslaw
their loan with the bank, allowing the latter to foreclose the option contract or clause. Otherwise, the offeree must
mortgage.88 Since the spouses Dijamco did not exercise bear the burden of proving that a separate consideration Thus, when an offer is supported by a separate
their right to redeem, the bank consolidated its ownership for the option contract exists. consideration, a valid option contract exists, i.e., there is a
over the mortgaged property.89 The spouses Dijamco contracted offer100 which the offerer cannot withdraw
later proposed to purchase the same property by paying a Given our finding that the Agreement did not categorically from without incurring liability in damages.
purchase price of P622,095.00 (equivalent to their refer to any consideration to support Keppel's option to
principal loan) and a monthly amount of P13,478.00 buy and for Keppel's failure to present evidence in this On the other hand, when the offer is not supported by a
payable for 12 months (equivalent to the interest on their regard, we cannot uphold the existence of an option separate consideration, the offer stands but, in the
principal loan). They further stated that should they fail to contract in this case. absence of a binding contract, the offeror may withdraw it
make a monthly payment, the proposal should be II. B. any time.101 In either case, once the acceptance of the
automatically revoked and all payments be treated as An option, though unsupported by a separate offer is duly communicated before the withdrawal of the
rentals for their continued use of the property.90 The consideration, remains an offer that, if duly accepted, offer, a bilateral contract to buy and sell is generated
Court treated the spouses Dijamco's proposal to purchase generates into a contract to sell where the parties' which, in accordance with the first paragraph of Article
the property as an option contract, and the consideration respective obligations become reciprocally demandable 1479 of the Civil Code, becomes reciprocally
for which was the monthly interest payments.91 The absence of a consideration supporting the option demandable.102chanrobleslaw
Interestingly, this ruling was made despite the categorical contract, however, does not invalidate an offer to buy (or
stipulation that the monthly interest payments should be to sell). An option unsupported by a separate Sanchez v. Rigos expressly overturned the 1955 case of
treated as rent for the spouses Dijamco's continued consideration stands as an unaccepted offer to buy (or to Southwestern Sugar v. AGPC,103 which declared that
possession and use of the foreclosed property. sell) which, when properly accepted, ripens into a contract a unilateral promise to buy or to sell, even if accepted, is
to sell. This is the rule established by the Court en banc only binding if supported by a consideration... In other
At the other end of the jurisprudential spectrum are cases as early as 1958 in Atkins v. Cua Hian Tek,96 and upheld words, an accepted unilateral promise can only have a
where the Court refused to consider the additional in 1972 in Sanchez v. Rigos.97chanrobleslaw binding effect if supported by a consideration, which
concessions stipulated in agreements as separate means that the option can still be withdrawn, even if
consideration for the option contract. Sanchez v. Rigos reconciled the apparent conflict accepted, if the same is not supported by any
between Articles 1324 and 1479 of the Civil Code, which consideration.104 [Emphasis supplied]
In Bible Baptist Church v. CA,92 the lessee (Bible Baptist are quoted below:ChanRoblesVirtualawlibrary The Southwestern Sugar doctrine was based on the
Church) paid in advance P84,000.00 to the lessor in order Article 1324. When the offerer has allowed the offeree a reasoning that Article 1479 of the Civil Code is distinct
to free the property from an encumbrance. The lessee certain period to accept, the offer may be withdrawn at from Article 1324 of the Civil Code and is a provision that
claimed that the advance payment constituted as the any time before acceptance by communicating such specifically governs options to buy (or to sell).105 As
separate consideration for its option to buy the property.93 withdrawal, except when the option is founded upon a mentioned, Sanchez v. Rigos found no conflict between
The Court, however, disagreed noting that the P84,000.00 consideration, as something paid or promised. these two provisions and accordingly abandoned the
paid in advance was eventually offset against the rent due Southwestern Sugar doctrine.
for the first year of the lease, "such that for the entire year Article 1479. A promise to buy and sell a determinate
from 1985 to 1986 the [Bible Baptist Church] did not pay thing for a price certain is reciprocally demandable. Unfortunately, without expressly overturning or
monthly rent."94 Hence, the Court refused to recognize An accepted unilateral promise to buy or to sell a abandoning the Sanchez ruling, subsequent cases
the existence of a valid option contract.95chanrobleslaw determinate thing for a price certain is binding upon the reverted back to the Southwestern Sugar doctrine.106 In
43
Sales – Chapter 3-12 Cases
2009, Eulogio v Apeles107 referred to Southwestern lands."114 On account of the OGCC opinion, the PNOC Although the ruling was made in the context of ownership
Sugar v. AGPC as the controlling doctrine108 and, due to did not agree with Keppel's attempt to buy the land;115 and operation of public utilities, the same should be
the lack of a separate consideration, refused to recognize nonetheless, the PNOC made no categorical withdrawal applied to the ownership of public and private lands, since
the option to buy as an offer that would have resulted in a of the offer to sell provided under the Agreement. the same proportion of Filipino ownership is required and
sale given its timely acceptance by the offeree. In 2010, the same nationalist policy pervades.
Tuazon v. Del Rosario-Suarez109 referred to Sanchez v. By 2000, Keppel had met the required Filipino equity
Rigos but erroneously cited as part of its ratio decidendi proportion and duly communicated its acceptance of the The uncontested fact is that, as of November 2000,
that portion of the Southwestern Sugar doctrine that offer to buy to PNOC.116 Keppel met with the board of Keppel's capital is 60% Filipino-owned.127 However,
Sanchez had expressly abandoned.110chanrobleslaw directors and officials of PNOC who interposed no there is nothing in the records showing the nature and
objection to the sale.117 It was only when the amount of composition of Keppel's shareholdings, i.e., whether its
Given that! the issue raised in the present case involves purchase price was raised that the conflict between the shareholdings are divided into different classes, and 60%
the application of Article 1324 and 1479 of the Civil Code, parties arose,118 with PNOC backtracking in its position of each share class is legally and beneficially owned by
it becomes imperative for the Court [en banc] to clarify and questioning the validity of the Filipinos - understandably because when Keppel
and declare here which between Sanchez and option.119chanrobleslaw exercised its option to buy the land in 2000, the Gamboa
Southwestern Sugar is the controlling doctrine. ruling had not yet been promulgated. The Court cannot
Thus, when Keppel communicated its acceptance, the deny Keppel its option to buy the land by retroactively
The Constitution itself declares that "no doctrine or offer to purchase the Bauan land stood, not having been applying the Gamboa ruling without violating Keppel's
principle of law laid down by the court in a decision withdrawn by PNOC. The offer having been duly vested right. Thus, Keppel's failure to prove the nature
rendered en banc or in division may be modified or accepted, a contract to sell the land ensued which Keppel and composition of its shareholdings in 2000 could not
reversed except by the court sitting en banc.111Sanchez can rightfully demand PNOC to comply with. prevent it from validly exercising its option to buy the land.
v. Rigos was an en banc decision which was affirmed in III. Keppel's constitutional right to acquire full title to
1994 in Asuncion v. CA,112 also an en banc decision, the land Nonetheless, the Court cannot completely disregard the
while the decisions citing the Southwestern Sugar doctrine Filipinization is the spirit that pervades the constitutional effect of the Gamboa ruling; the 60% Filipino equity
are all division cases.113 Based on the constitutional rule provisions on national patrimony and economy. The proportion is a continuing requirement to hold land in the
(as well as the inherent logic in reconciling Civil Code Constitution has reserved the ownership of public and Philippines. Even in Gamboa, the Court prospectively
provisions), there should be no doubt that Sanchez v. private lands,120 the ownership and operation of public applied its ruling, thus enabling the public utilities to meet
Rigos remains as the controlling doctrine. utilities,121 and certain areas of investment122 to Filipino the nationality requirement before the Securities and
citizens, associations, and corporations. To qualify, sixty Exchange Commission commences administrative
Accordingly, when an option to buy or to sell is not per cent (60%) of the association or corporation's capital investigation and cases, and imposes sanctions for
supported by a consideration separate from the purchase must be owned by Filipino citizens. Although the 60% noncompliance on erring corporations.128 In this case,
price, the option constitutes as an offer to buy or to sell, Filipino equity proportion has been adopted in our Keppel must be allowed to prove whether it meets the
which may be withdrawn by the offeror at any time prior to Constitution since 1935, it was only in 2011 that the Court required Filipino equity ownership and proportion in
the communication of the offeree's acceptance. When the interpreted what the term capital constituted. accordance with the Gamboa ruling before it can acquire
offer is duly accepted, a mutual promise to buy and to sell full title to the land.
under the first paragraph of Article 1479 of the Civil Code In Gamboa v. Teves,123 the Court declared that the "legal
ensues and the parties' respective obligations become and beneficial ownership of 60 percent of the outstanding In view of the foregoing, the Court AFFIRMS the decision
reciprocally demandable. capital stock must rest in the hands of Filipino nationals." dated 19 December 2011 and the resolution dated 14
124 Clarifying the ruling, the Court decreed that the 60% May 2012 of the CA in CA-G.R. CV No. 86830 insofar as
Applied to the present case, we find that the offer to buy Filipino ownership requirement applies separately to each these rulings uphold the respondent Keppel Philippines
the land was timely accepted by Keppel. class of shares, whether with or without voting rights,125 Holdings, Inc.'s option to buy the land, and REMANDS the
thus:ChanRoblesVirtualawlibrary case to the Regional Trial Court of Batangas City, Branch
As early as 1994, Keppel expressed its desire to exercise Applying uniformly the 60-40 ownership requirement in 84, for the determination of whether the respondent
its option to buy the land. Instead of rejecting outright favour of Filipino citizens to each class of shares, Keppel Philippines Holdings, Inc. meets the required
Keppel's acceptance, PNOC referred the matter to the regardless of differences in voting rights, privileges and Filipino equity ownership and proportion in accordance
Office of the Government Corporate Counsel (OGCC). In restrictions, guarantees effective Filipino control of public with the Court's ruling in Gamboa v. Teves, to allow it to
its Opinion No. 160, series of 1994, the OGCC opined that utilities, as mandated by the Constitution.126 acquire full title to the land.
Keppel "did not yet have the right to purchase the Bauan
44
Sales – Chapter 3-12 Cases
SO ORDERED.

45
Sales – Chapter 3-12 Cases
POLYTECHNIC UNIVERSITY On June 13, 1988, before the expiration of the ten (10)-year prosecuting and Judge Francisco Brillantes, Jr. from
OF THE PHILIPPINES, period under the second lease contract, GHRC wrote a letter proceeding with the ejectment case.[12]
Petitioner, to NDC indicating its exercise of the option to renew the lease In its Second Amended and/or Supplemental Complaint,
- versus - for another ten (10) years. As no response was received from GHRC argued that Memorandum Order No. 214 is a nullity,
GOLDEN HORIZON REALTY NDC, GHRC sent another letter on August 12, 1988, for being violative of the writ of injunction issued by the trial
CORPORATION, reiterating its desire to renew the contract and also requesting court, apart from being an infringement of the Constitutional
Respondent. for priority to negotiate for its purchase should NDC opt to sell prohibition against impairment of obligation of contracts, an
the leased premises.[6] NDC still did not reply but continued encroachment on legislative functions and a bill of attainder.
The above-titled consolidated petitions filed under Rule 45 of to accept rental payments from GHRC and allowed the latter In the alternative, should the trial court adjudge the
the 1997 Rules of Civil Procedure, as amended, seek to to remain in possession of the property. memorandum order as valid, GHRC contended that its
reverse the Decision[1] dated June 25, 2008 and Resolution Sometime after September 1988, GHRC discovered that existing right must still be respected by allowing it to purchase
dated August 22, 2008 of the Court of Appeals (CA) in CA- NDC had decided to secretly dispose the property to a third the leased premises.[13]
G.R. CV No. 84399 which affirmed the Decision[2] dated party. On October 21, 1988, GHRC filed in the RTC a Pre-trial was set but was suspended upon agreement of the
November 25, 2004 of the Regional Trial Court (RTC) of complaint for specific performance, damages with preliminary parties to await the final resolution of a similar case involving
Makati City, Branch 144 in Civil Case No. 88-2238. injunction and temporary restraining order.[7] NDC, PUP and another lessee of NDC, Firestone Ceramics,
The undisputed facts are as follows: In the meantime, then President Corazon C. Aquino issued Inc. (Firestone), then pending before the RTC of Pasay
Petitioner National Development Company (NDC) is a Memorandum Order No. 214 dated January 6, 1989, ordering City.[14]
government- owned and controlled corporation, created the transfer of the whole NDC Compound to the National On November 14, 2001, this Court rendered a decision in
under Commonwealth Act No. 182, as amended by Com. Act Government, which in turn would convey the said property in G.R. Nos. 143513 (Polytechnic University of the Philippines v.
No. 311 and Presidential Decree (P.D.) No. 668. Petitioner favor of PUP at acquisition cost. The memorandum order Court of Appeals) and 143590 (National Development
Polytechnic University of the Philippines (PUP) is a public, cited the serious need of PUP, considered the Poor Mans Corporation v. Firestone Ceramics, Inc.),[15] which declared
non-sectarian, non-profit educational institution created in University, to expand its campus, which adjoins the NDC that the sale to PUP by NDC of the portion leased by
1978 by virtue of P.D. No. 1341. Compound, to accommodate its growing student population, Firestone pursuant to Memorandum Order No. 214 violated
In the early sixties, NDC had in its disposal a ten (10)-hectare and the willingness of PUP to buy and of NDC to sell its the right of first refusal granted to Firestone under its third
property located along Pureza St., Sta. Mesa, Manila. The property. The order of conveyance of the 10.31-hectare lease contract with NDC. We thus decreed:
estate was popularly known as the NDC Compound and property would automatically result in the cancellation of WHEREFORE, the petitions in G.R. No. 143513 and G.R.
covered by Transfer Certificate of Title Nos. 92885, 110301 NDCs total obligation in favor of the National Government in No. 143590 are DENIED. Inasmuch as the first contract of
and 145470. the amount of P57,193,201.64.[8] lease fixed the area of the leased premises at 2.90118
On September 7, 1977, NDC entered into a Contract of On February 20, 1989, the RTC issued a writ of preliminary hectares while the second contract placed it at 2.60 hectares,
Lease (C-33-77) with Golden Horizon Realty Corporation injunction enjoining NDC and its attorneys, representatives, let a ground survey of the leased premises be immediately
(GHRC) over a portion of the property, with an area of 2,407 agents and any other persons assisting it from proceeding conducted by a duly licensed, registered surveyor at the
square meters for a period of ten (10) years, renewable for with the sale and disposition of the leased premises.[9] expense of private respondent FIRESTONE CERAMICS,
another ten (10) years with mutual consent of the parties.[3] On February 23, 1989, PUP filed a motion to intervene as INC., within two (2) months from the finality of the judgment in
On May 4, 1978, a second Contract of Lease (C-12-78) was party defendant, claiming that as a purchaser pendente lite of this case. Thereafter, private respondent FIRESTONE
executed between NDC and GHRC covering 3,222.80 square a property subject of litigation it is entitled to intervene in the CERAMICS, INC., shall have six (6) months from receipt of
meters, also renewable upon mutual consent after the proceedings. The RTC granted the said motion and directed the approved survey within which to exercise its right to
expiration of the ten (10)-year lease period. In addition, PUP to file its Answer-in-Intervention.[10] purchase the leased property at P1,500.00 per square meter,
GHRC as lessee was granted the option to purchase the area PUP also demanded that GHRC vacate the premises, and petitioner Polytechnic University of the Philippines is
leased, the price to be negotiated and determined at the time insisting that the latters lease contract had already expired. ordered to reconvey the property to FIRESTONE
the option to purchase is exercised.[4] Its demand letter unheeded by GHRC, PUP filed an CERAMICS, INC., in the exercise of its right of first refusal
Under the lease agreements, GHRC was obliged to construct ejectment case (Civil Case No. 134416) before the upon payment of the purchase price thereof.
at its own expense buildings of strong material at no less than Metropolitan Trial Court (MeTC) of Manila on January 14,
the stipulated cost, and other improvements which shall 1991.[11] SO ORDERED.[16]
automatically belong to the NDC as lessor upon the Due to this development, GHRC filed an Amended and/or
expiration of the lease period. Accordingly, GHRC introduced Supplemental Complaint to include as additional defendants The RTC resumed the proceedings and when mediation and
permanent improvements and structures as required by the PUP, Honorable Executive Secretary Oscar Orbos and Judge pre-trial failed to settle the case amicably, trial on the merits
terms of the contract. After the completion of the industrial Ernesto A. Reyes of the Manila MeTC, and to enjoin the ensued.[17]
complex project, for which GHRC spent P5 million, it was afore-mentioned defendants from prosecuting Civil Case No. On November 25, 2004, the RTC rendered its decision
leased to various manufacturers, industrialists and other 134416 for ejectment. A temporary restraining order was upholding the right of first refusal granted to GHRC under its
businessmen thereby generating hundreds of jobs.[5] subsequently issued by the RTC enjoining PUP from lease contract with NDC and ordering PUP to reconvey the
46
Sales – Chapter 3-12 Cases
said portion of the property in favor of GHRC. The dispositive covering 3,222.80 square meters, and not to the first lease refusal, which incidentally appears only in the second lease
portion reads: contract, C-33-77 covering 2,407 square meters, which had contract and not in the first lease contract.[23]
WHEREFORE, premises considered, judgment is hereby already expired. It sustained the RTCs finding that the two (2) On its part, petitioner NDC assails the CA in holding that the
rendered in favor of the plaintiff and against the defendants lease contracts were interrelated because each formed part contracts of lease were impliedly renewed for another ten
ordering the plaintiff to cause immediate ground survey of the of GHRCs industrial complex, such that business operations (10)-year period. The provisions of C-33-77 and C-12-78
premises subject of the leased contract under Lease Contract would be rendered useless and inoperative if the first contract clearly state that the lessee is granted the option to renew for
No. C-33-77 and C-12-78 measuring 2,407 and 3,222.8 were to be detached from the other, as similarly held in the another ten (10) years with the mutual consent of both
square meters respectively, by a duly licensed and registered afore-mentioned case of Polytechnic University of the parties. As regards the continued receipt of rentals by NDC
surveyor at the expense of the plaintiff within two months Philippines v. Court of Appeals. and possession by the respondent of the leased premises,
from receipt of this Decision and thereafter, the plaintiff shall Petitioner PUP argues that respondents right to exercise the the impliedly renewed lease was only month-to-month and
have six (6) months from receipt of the approved survey option to purchase had expired with the termination of the not ten (10) years since the rentals are being paid on a
within which to exercise its right to purchase the leased original contract of lease and was not carried over to the monthly basis, as held in Dizon v. Magsaysay.[24]
property at P554.74 per square meter. And finally, the subsequent implied new lease between respondent and Petitioner NDC further faults the CA in sustaining the RTCs
defendant PUP, in whose name the property is titled, is petitioner NDC. As testified to by their witnesses Leticia decision which erroneously granted respondent the option to
hereby ordered to reconvey the aforesaid property to the Cabantog and Atty. Rhoel Mabazza, there was no agreement purchase the leased premises at the rate of P554.74 per
plaintiff in the exercise of its right of its option to buy or first or document to the effect that respondents request for square meter, the same rate for which NDC sold the property
refusal upon payment of the purchase price thereof. extension or renewal of the subject contracts of lease for to petitioner PUP and/or the National Government, which is
another ten (10) years was approved by NDC. Hence, the mere acquisition cost thereof. It must be noted that such
The defendant NDC is hereby further ordered to pay the respondent can no longer exercise the option to purchase the consideration or rate was imposed by Memorandum Order
plaintiff attorneys fees in the amount of P100,000.00. leased premises when the same were conveyed to PUP No. 214 under the premise that it shall, in effect, be a sale
pursuant to Memorandum Order No. 214 dated January 6, and/or purchase from one (1) government agency to another.
The case against defendant Executive Secretary is dismissed 1989, long after the expiration of C-33-77 and C-12-78 in It was intended merely as a transfer of one (1) user of the
and this decision shall bind defendant Metropolitan Trial September 1988.[21] National Government to another, with the beneficiary, PUP in
Court, Branch 20 of Manila. Petitioner PUP further contends that while it is conceded that this case, merely returning to the petitioner/transferor the cost
there was an implied new lease between respondent and of acquisition thereof, as appearing on its accounting books.
With costs against defendants NDC and PUP. petitioner NDC after the expiration of the lease contracts, the It does not in any way reflect the true and fair market value of
same did not include the right of first refusal originally granted the property, nor was it a price a willing seller would demand
SO ORDERED.[18] to respondent. The CA should have applied the ruling in and accept for parting with his real property. Such benefit,
Dizon v. Magsaysay[22] that the lessee cannot any more therefore, cannot be extended to respondent as a private
NDC and PUP separately appealed the decision to the exercise its option to purchase after the lapse of the one (1)- entity, as the latter does not share the same pocket, so to
CA.[19] By Decision of June 25, 2008, the CA affirmed in toto year period of the lease contract. With the implicit renewal of speak, with the National Government.[25]
the decision of the RTC.[20] the lease on a monthly basis, the other terms of the original The issue to be resolved is whether or not our ruling in
Both the RTC and the CA applied this Courts ruling in contract of lease which are revived in the implied new lease Polytechnic University of the Philippines v. Court of Appeals
Polytechnic University of the Philippines v. Court of Appeals under Article 1670 of the Civil Code are only those terms applies in this case involving another lessee of NDC who
(supra), considering that GHRC is similarly situated as a which are germane to the lessees right of continued claimed that the option to purchase the portion leased to it
lessee of NDC whose right of first refusal under the lease enjoyment of the property leased. The provision entitling the was similarly violated by the sale of the NDC Compound in
contract was violated by the sale of the property to PUP lessee the option to purchase the leased premises is not favor of PUP pursuant to Memorandum Order No. 214.
without NDC having first offered to sell the same to GHRC deemed incorporated in the impliedly renewed contract We rule in the affirmative.
despite the latters request for the renewal of the lease and/or because it is alien to the possession of the lessee. The second lease contract contained the following provision:
to purchase the leased premises prior to the expiration of the Consequently, as in this case, respondents right of option to
second lease contract. The CA further agreed with the RTCs purchase the leased premises was not violated despite the III. It is mutually agreed by the parties that this Contract of
finding that there was an implied renewal of the lease upon impliedly renewed contract of lease with NDC. Respondent Lease shall be in full force and effect for a period of ten (10)
the failure of NDC to act on GHRCs repeated requests for cannot favorably invoke the decision in G.R. Nos. 143513 years counted from the effectivity of the payment of rental as
renewal of the lease contract, both verbal and written, and and 143590 (Polytechnic University of the Philippines v. Court provided under sub-paragraph (b) of Article I, with option to
continuing to accept monthly rental payments from GHRC of Appeals) for the simple reason, among others, that unlike renew for another ten (10) years with the mutual consent of
which was allowed to continue in possession of the leased in said cases, the contracts of lease of respondent with NDC both parties. In no case should the rentals be increased by
premises. were not mutually extended or renewed for another ten (10) more than 100% of the original amount fixed.
The CA also rejected the argument of NDC and PUP that years. Thus, when the leased premises were conveyed to
even assuming that GHRC had the right of first refusal, said PUP, respondent did not any more have any right of first
right pertained only to the second lease contract, C-12-78
47
Sales – Chapter 3-12 Cases
Lessee shall also have the option to purchase the area to exercise his right of first priority could the lessor sell the July 1988 when respondents right of first refusal was still
leased, the price to be negotiated and determined at the time property to other buyers under the same terms and conditions subsisting and the lease contracts still in force. Petitioner
the option to purchase is exercised. [EMPHASIS SUPPLIED] offered to the lessee, or under terms and conditions more NDC did not bother to respond to respondents letter of June
favorable to the lessor.[30] 13, 1988 informing it of respondents exercise of the option to
An option is a contract by which the owner of the property Records showed that during the hearing on the application for renew and requesting to discuss further the matter with NDC,
agrees with another person that the latter shall have the right a writ of preliminary injunction, respondent adduced in nor to the subsequent letter of August 12, 1988 reiterating the
to buy the formers property at a fixed price within a certain evidence a letter of Antonio A. Henson dated 15 July 1988 request for renewing the lease for another ten (10) years and
time. It is a condition offered or contract by which the owner addressed to Mr. Jake C. Lagonera, Director and Special also the exercise of the option to purchase under the lease
stipulates with another that the latter shall have the right to Assistant to Executive Secretary Catalino Macaraeg, contract. Petitioner NDC had dismissed these letters as mere
buy the property at a fixed price within a certain time, or reviewing a proposed memorandum order submitted to informative in nature, and a request at its best.[32]
under, or in compliance with certain terms and conditions; or President Corazon C. Aquino transferring the whole NDC Perusal of the letter dated August 12, 1988, however, belies
which gives to the owner of the property the right to sell or Compound, including the premises leased by respondent, in such claim of petitioner NDC that it was merely informative,
demand a sale.[26] It binds the party, who has given the favor of petitioner PUP. This letter was offered in evidence by thus:
option, not to enter into the principal contract with any other respondent to prove the existence of documents as of that
person during the period designated, and, within that period, date and even prior to the expiration of the second lease August 12, 1988
to enter into such contract with the one to whom the option contract or the lapse of the ten (10)-year period counted from
was granted, if the latter should decide to use the option.[27] the effectivity of the rental payment -- that is, one hundred HON. ANTONIO HENSON
Upon the other hand, a right of first refusal is a contractual and fifty (150) days from the signing of the contract (May 4, General Manager
grant, not of the sale of a property, but of the first priority to 1978), as provided in Art. I, paragraph (b) of C-12-78, or on NATIONAL DEVELOPMENT COMPANY
buy the property in the event the owner sells the same.[28] October 1, 1988. 377 Se(n). Gil J. Puyat Avenue
As distinguished from an option contract, in a right of first Respondent thus timely exercised its option to purchase on Makati, Metro Manila
refusal, while the object might be made determinate, the August 12, 1988. However, considering that NDC had been
exercise of the right of first refusal would be dependent not negotiating through the National Government for the sale of REF: Contract of Lease
only on the owners eventual intention to enter into a binding the property in favor of PUP as early as July 15, 1988 without Nos. C-33-77 & C-12-78
juridical relation with another but also on terms, including the first offering to sell it to respondent and even when
price, that are yet to be firmed up.[29] respondent communicated its desire to exercise the option to Dear Sir:
As the option to purchase clause in the second lease contract purchase granted to it under the lease contract, it is clear that
has no definite period within which the leased premises will NDC violated respondents right of first refusal. Under the This is further to our earlier letter dated June 13, 1988
be offered for sale to respondent lessee and the price is premises, the matter of the right of refusal not having been formally advising your goodselves of our intention to exercise
made subject to negotiation and determined only at the time carried over to the impliedly renewed month-to-month lease our option for another ten (10) years. Should the National
the option to buy is exercised, it is obviously a mere right of after the expiration of the second lease contract on October Development Company opt to sell the property covered by
refusal, usually inserted in lease contracts to give the lessee 21, 1988 becomes irrelevant since at the time of the said leases, we also request for priority to negotiate for its
the first crack to buy the property in case the lessor decides negotiations of the sale to a third party, petitioner PUP, purchase at terms and/or conditions mutually acceptable.
to sell the same. That respondent was granted a right of first respondents right of first refusal was still subsisting.
refusal under the second lease contract appears not to have Petitioner NDC in its memorandum contended that the CA As a backgrounder, we wish to inform you that since the start
been disputed by petitioners. What petitioners assail is the erred in applying the ruling in Polytechnic University of the of our lease, we have improved on the property by
CAs erroneous conclusion that such right of refusal subsisted Philippines v. Court of Appeals pointing out that the case of constructing bodega-type buildings which presently house all
even after the expiration of the original lease period, when lessee Firestone Ceramics, Inc. is different because the lease legitimate trading and manufacturing concerns. These
respondent was allowed to continue staying in the leased contract therein had not yet expired while in this case business are substantial taxpayers, employ not less than 300
premises under an implied renewal of the lease and without respondents lease contracts have already expired and never employees and contribute even foreign earnings.
the right of refusal carried over to such month-to-month lease. renewed. The date of the expiration of the lease contract in
Petitioners thus maintain that no right of refusal was violated said case is December 31, 1989 which is prior to the It is in this context that we are requesting for the extension of
by the sale of the property in favor of PUP pursuant to issuance of Memorandum Order No. 214 on January 6, 1989. the lease contract to prevent serious economic disruption and
Memorandum Order No. 214. In contrast, respondents lease contracts had already expired dislocation of the business concerns, as well as provide
Petitioners position is untenable. (September 1988) at the time said memorandum order was ourselves, the lessee, an opportunity to recoup our
When a lease contract contains a right of first refusal, the issued.[31] investments and obtain a fair return thereof.
lessor has the legal duty to the lessee not to sell the leased Such contention does not hold water. As already mentioned,
property to anyone at any price until after the lessor has the reckoning point of the offer of sale to a third party was not Your favorable consideration on our request will be very
made an offer to sell the property to the lessee and the the issuance of Memorandum Order No. 214 on January 6, much appreciated.
lessee has failed to accept it. Only after the lessee has failed 1989 but the commencement of such negotiations as early as
48
Sales – Chapter 3-12 Cases
very truly yours, A-10-78 executed on 22 December 1978 which, as found by by PUP, to whom the lessor NDC sold it in violation of
the courts a quo, was interrelated to and inseparable from respondent lessees right of first refusal, as follows:
TIU HAN TENG their first contract denominated as C-30-65 executed on 24
President[33] August 1965 and their second contract denominated as C-26- It now becomes apropos to ask whether the courts a quo
68 executed on 8 January 1969. Thus - were correct in fixing the proper consideration of the sale at
As to petitioners argument that respondents right of first P1,500.00 per square meter. In contracts of sale, the basis of
refusal can be invoked only with respect to the second lease Should the LESSOR desire to sell the leased premises during the right of first refusal must be the current offer of the seller
contract which expressly provided for the option to purchase the term of this Agreement, or any extension thereof, the to sell or the offer to purchase of the prospective buyer. Only
by the lessee, and not in the first lease contract which LESSOR shall first give to the LESSEE, which shall have the after the lessee-grantee fails to exercise its right under the
contained no such clause, we sustain the RTC and CA in right of first option to purchase the leased premises subject to same terms and within the period contemplated can the
finding that the second contract, covering an area of 3,222.80 mutual agreement of both parties. owner validly offer to sell the property to a third person, again,
square meters, is interrelated to and inseparable from the first under the same terms as offered to the grantee. It appearing
contract over 2,407 square meters. The structures built on the In the instant case, the right of first refusal is an integral and that the whole NDC compound was sold to PUP for P554.74
leased premises, which are adjacent to each other, form part indivisible part of the contract of lease and is inseparable per square meter, it would have been more proper for the
of an integrated system of a commercial complex leased out from the whole contract. The consideration for the right is courts below to have ordered the sale of the property also at
to manufacturers, fabricators and other businesses. built into the reciprocal obligations of the parties. Thus, it is the same price. However, since FIRESTONE never raised
Petitioners submitted a sketch plan and pictures taken of the not correct for petitioners to insist that there was no this as an issue, while on the other hand it admitted that the
driveways, in an effort to show that the leased premises can consideration paid by FIRESTONE to entitle it to the exercise value of the property stood at P1,500.00 per square meter,
be used separately by respondent, and that the two (2) lease of the right, inasmuch as the stipulation is part and parcel of then we see no compelling reason to modify the holdings of
contracts are distinct from each other.[34] Such was a the contract of lease making the consideration for the lease the courts a quo that the leased premises be sold at that
desperate attempt to downplay the commercial purpose of the same as that for the option. price.[39] [EMPHASIS SUPPLIED]
respondents substantial improvements which greatly
contributed to the increased value of the leased premises. To It is a settled principle in civil law that when a lease contract
prove that petitioner NDC had considered the leased contains a right of first refusal, the lessor is under a legal duty In the light of the foregoing, we hold that respondent, which
premises as a single unit, respondent submitted evidence to the lessee not to sell to anybody at any price until after he did not offer any amount to petitioner NDC, and neither
showing that NDC issued only one (1) receipt for the rental has made an offer to sell to the latter at a certain price and disputed the P1,500.00 per square meter actual value of
payments for the two portions.[35] Respondent further the lessee has failed to accept it. The lessee has a right that NDCs property at that time it was sold to PUP at P554.74 per
presented the blueprint plan prepared by its witness, Engr. the lessors first offer shall be in his favor. square meter, as duly considered by this Court in the
Alejandro E. Tinio, who supervised the construction of the Firestone case, should be bound by such determination.
structures on the leased premises, to show the building The option in this case was incorporated in the contracts of Accordingly, the price at which the leased premises should be
concept as a one-stop industrial site and integrated lease by NDC for the benefit of FIRESTONE which, in view of sold to respondent in the exercise of its right of first refusal
commercial complex.[36] the total amount of its investments in the property, wanted to under the lease contract with petitioner NDC, which was
In fine, the CA was correct in declaring that there exists no be assured that it would be given the first opportunity to buy pegged by the RTC at P554.74 per square meter, should be
justifiable reason not to apply the same rationale in the property at a price for which it would be offered. adjusted to P1,500.00 per square meter, which more
Polytechnic University of the Philippines v. Court of Appeals Consistent with their agreement, it was then implicit for NDC accurately reflects its true value at that time of the sale in
in the case of respondent who was similarly prejudiced by to have first offered the leased premises of 2.60 hectares to favor of petitioner PUP.
petitioner NDCs sale of the property to PUP, as to entitle the FIRESTONE prior to the sale in favor of PUP. Only if Indeed, basic is the rule that a party to a contract cannot
respondent to exercise its option to purchase until October FIRESTONE failed to exercise its right of first priority could unilaterally withdraw a right of first refusal that stands upon
1988 inasmuch as the May 4, 1978 contract embodied the NDC lawfully sell the property to petitioner PUP.[37] valuable consideration.[40] We have categorically ruled that it
option to renew the lease for another ten (10) years upon [EMPHASIS SUPPLIED] is not correct to say that there is no consideration for the
mutual consent and giving respondent the option to purchase grant of the right of first refusal if such grant is embodied in
the leased premises for a price to be negotiated and As we further ruled in the afore-cited case, the contractual the same contract of lease. Since the stipulation forms part of
determined at the time such option was exercised by grant of a right of first refusal is enforceable, and following an the entire lease contract, the consideration for the lease
respondent. It is to be noted that Memorandum Order No. earlier ruling in Equatorial Realty Development, Inc. v. includes the consideration for the grant of the right of first
214 itself declared that the transfer is subject to such Mayfair Theater, Inc.,[38] the execution of such right consists refusal. In entering into the contract, the lessee is in effect
liens/leases existing [on the subject property]. Thus: in directing the grantor to comply with his obligation according stating that it consents to lease the premises and to pay the
...we now proceed to determine whether FIRESTONE should to the terms at which he should have offered the property in price agreed upon provided the lessor also consents that,
be allowed to exercise its right of first refusal over the favor of the grantee and at that price when the offer should should it sell the leased property, then, the lessee shall be
property. Such right was expressly stated by NDC and have been made. We then determined the proper rate at given the right to match the offered purchase price and to buy
FIRESTONE in par. XV of their third contract denominated as which the leased portion should be reconveyed to respondent the property at that price.[41]
49
Sales – Chapter 3-12 Cases
We have further stressed that not even the avowed public
welfare or the constitutional priority accorded to education,
invoked by petitioner PUP in the Firestone case, would serve
as license for us, and any party for that matter, to destroy the
sanctity of binding obligations. While education may be
prioritized for legislative and budgetary purposes, it is
doubtful if such importance can be used to confiscate private
property such as the right of first refusal granted to a lessee
of petitioner NDC.[42] Clearly, no reversible error was
committed by the CA in sustaining respondents contractual
right of first refusal and ordering the reconveyance of the
leased portion of petitioner NDCs property in its favor.
WHEREFORE, the petitions are DENIED. The Decision
dated November 25, 2004 of the Regional Trial Court of
Makati City, Branch 144 in Civil Case No. 88-2238, as
affirmed by the Court of Appeals in its Decision dated June
25, 2008 in CA-G.R. CV No. 84399, is hereby AFFIRMED
with MODIFICATION in that the price to be paid by
respondent Golden Horizon Realty Corporation for the leased
portion of the NDC Compound under Lease Contract Nos. C-
33-77 and C-12-78 is hereby increased to P1,500.00 per
square meter.
No pronouncement as to costs.
SO ORDERED.

50
Sales – Chapter 3-12 Cases
ANG YU ASUNCION, ARTHUR GO AND KEH TIONG, accepted by the plaintiffs for the reason that the parties did
petitioners, not agree upon the terms and conditions of the proposed SO ORDERED.
vs. sale, hence, there was no contract of sale at all. Nonetheless,
THE HON. COURT OF APPEALS and BUEN REALTY the lower court ruled that should the defendants subsequently The decision of this Court was brought to the Supreme Court
DEVELOPMENT CORPORATION, respondents. offer their property for sale at a price of P11-million or below, by petition for review on certiorari. The Supreme Court denied
plaintiffs will have the right of first refusal. Thus the dispositive the appeal on May 6, 1991 "for insufficiency in form and
Assailed, in this petition for review, is the decision of the portion of the decision states: substances" (Annex H, Petition).
Court of Appeals, dated 04 December 1991, in CA-G.R. SP
No. 26345 setting aside and declaring without force and WHEREFORE, judgment is hereby rendered in favor of the On November 15, 1990, while CA-G.R. CV No. 21123 was
effect the orders of execution of the trial court, dated 30 defendants and against the plaintiffs summarily dismissing pending consideration by this Court, the Cu Unjieng spouses
August 1991 and 27 September 1991, in Civil Case No. 87- the complaint subject to the aforementioned condition that if executed a Deed of Sale (Annex D, Petition) transferring the
41058. the defendants subsequently decide to offer their property for property in question to herein petitioner Buen Realty and
sale for a purchase price of Eleven Million Pesos or lower, Development Corporation, subject to the following terms and
The antecedents are recited in good detail by the appellate then the plaintiffs has the option to purchase the property or conditions:
court thusly: of first refusal, otherwise, defendants need not offer the
property to the plaintiffs if the purchase price is higher than 1. That for and in consideration of the sum of
On July 29, 1987 a Second Amended Complaint for Specific Eleven Million Pesos. FIFTEEN MILLION PESOS (P15,000,000.00), receipt of
Performance was filed by Ang Yu Asuncion and Keh Tiong, et which in full is hereby acknowledged, the VENDORS hereby
al., against Bobby Cu Unjieng, Rose Cu Unjieng and Jose SO ORDERED. sells, transfers and conveys for and in favor of the VENDEE,
Tan before the Regional Trial Court, Branch 31, Manila in his heirs, executors, administrators or assigns, the above-
Civil Case No. 87-41058, alleging, among others, that Aggrieved by the decision, plaintiffs appealed to this Court in described property with all the improvements found therein
plaintiffs are tenants or lessees of residential and commercial CA-G.R. CV No. 21123. In a decision promulgated on including all the rights and interest in the said property free
spaces owned by defendants described as Nos. 630-638 September 21, 1990 (penned by Justice Segundino G. Chua from all liens and encumbrances of whatever nature, except
Ongpin Street, Binondo, Manila; that they have occupied said and concurred in by Justices Vicente V. Mendoza and the pending ejectment proceeding;
spaces since 1935 and have been religiously paying the Fernando A. Santiago), this Court affirmed with modification
rental and complying with all the conditions of the lease the lower court's judgment, holding: 2. That the VENDEE shall pay the Documentary
contract; that on several occasions before October 9, 1986, Stamp Tax, registration fees for the transfer of title in his favor
defendants informed plaintiffs that they are offering to sell the In resume, there was no meeting of the minds between the and other expenses incidental to the sale of above-described
premises and are giving them priority to acquire the same; parties concerning the sale of the property. Absent such property including capital gains tax and accrued real estate
that during the negotiations, Bobby Cu Unjieng offered a price requirement, the claim for specific performance will not lie. taxes.
of P6-million while plaintiffs made a counter offer of P5- Appellants' demand for actual, moral and exemplary
million; that plaintiffs thereafter asked the defendants to put damages will likewise fail as there exists no justifiable ground As a consequence of the sale, TCT No. 105254/T-881 in the
their offer in writing to which request defendants acceded; for its award. Summary judgment for defendants was properly name of the Cu Unjieng spouses was cancelled and, in lieu
that in reply to defendant's letter, plaintiffs wrote them on granted. Courts may render summary judgment when there is thereof, TCT No. 195816 was issued in the name of petitioner
October 24, 1986 asking that they specify the terms and no genuine issue as to any material fact and the moving party on December 3, 1990.
conditions of the offer to sell; that when plaintiffs did not is entitled to a judgment as a matter of law (Garcia vs. Court
receive any reply, they sent another letter dated January 28, of Appeals, 176 SCRA 815). All requisites obtaining, the On July 1, 1991, petitioner as the new owner of the subject
1987 with the same request; that since defendants failed to decision of the court a quo is legally justifiable. property wrote a letter to the lessees demanding that the
specify the terms and conditions of the offer to sell and latter vacate the premises.
because of information received that defendants were about WHEREFORE, finding the appeal unmeritorious, the
to sell the property, plaintiffs were compelled to file the judgment appealed from is hereby AFFIRMED, but subject to On July 16, 1991, the lessees wrote a reply to petitioner
complaint to compel defendants to sell the property to them. the following modification: The court a quo in the aforestated stating that petitioner brought the property subject to the
decision gave the plaintiffs-appellants the right of first refusal notice of lis pendens regarding Civil Case No. 87-41058
Defendants filed their answer denying the material allegations only if the property is sold for a purchase price of Eleven annotated on TCT No. 105254/T-881 in the name of the Cu
of the complaint and interposing a special defense of lack of Million pesos or lower; however, considering the mercurial Unjiengs.
cause of action. and uncertain forces in our market economy today. We find
no reason not to grant the same right of first refusal to herein The lessees filed a Motion for Execution dated August 27,
After the issues were joined, defendants filed a motion for appellants in the event that the subject property is sold for a 1991 of the Decision in Civil Case No. 87-41058 as modified
summary judgment which was granted by the lower court. price in excess of Eleven Million pesos. No pronouncement by the Court of Appeals in CA-G.R. CV No. 21123.
The trial court found that defendants' offer to sell was never as to costs.
51
Sales – Chapter 3-12 Cases
On August 30, 1991, respondent Judge issued an order WHEREFORE, let there be Writ of Execution issue in the
(Annex A, Petition) quoted as follows: above-entitled case directing the Deputy Sheriff Ramon Among the sources of an obligation is a contract (Art. 1157,
Enriquez of this Court to implement said Writ of Execution Civil Code), which is a meeting of minds between two
Presented before the Court is a Motion for Execution filed by ordering the defendants among others to comply with the persons whereby one binds himself, with respect to the other,
plaintiff represented by Atty. Antonio Albano. Both defendants aforesaid Order of this Court within a period of one (1) week to give something or to render some service (Art. 1305, Civil
Bobby Cu Unjieng and Rose Cu Unjieng represented by Atty. from receipt of this Order and for defendants to execute the Code). A contract undergoes various stages that include its
Vicente Sison and Atty. Anacleto Magno respectively were necessary Deed of Sale of the property in litigation in favor of negotiation or preparation, its perfection and, finally, its
duly notified in today's consideration of the motion as the plaintiffs Ang Yu Asuncion, Keh Tiong and Arthur Go for consummation. Negotiation covers the period from the time
evidenced by the rubber stamp and signatures upon the copy the consideration of P15,000,000.00 and ordering the the prospective contracting parties indicate interest in the
of the Motion for Execution. Register of Deeds of the City of Manila, to cancel and set contract to the time the contract is concluded (perfected). The
aside the title already issued in favor of Buen Realty perfection of the contract takes place upon the concurrence
The gist of the motion is that the Decision of the Court dated Corporation which was previously executed between the of the essential elements thereof. A contract which is
September 21, 1990 as modified by the Court of Appeals in latter and defendants and to register the new title in favor of consensual as to perfection is so established upon a mere
its decision in CA G.R. CV-21123, and elevated to the the aforesaid plaintiffs Ang Yu Asuncion, Keh Tiong and meeting of minds, i.e., the concurrence of offer and
Supreme Court upon the petition for review and that the same Arthur Go. acceptance, on the object and on the cause thereof. A
was denied by the highest tribunal in its resolution dated May contract which requires, in addition to the above, the delivery
6, 1991 in G.R. No. SO ORDERED. of the object of the agreement, as in a pledge or
L-97276, had now become final and executory. As a commodatum, is commonly referred to as a real contract. In a
consequence, there was an Entry of Judgment by the On the same day, September 27, 1991 the corresponding writ solemn contract, compliance with certain formalities
Supreme Court as of June 6, 1991, stating that the aforesaid of execution (Annex C, Petition) was issued.1 prescribed by law, such as in a donation of real property, is
modified decision had already become final and executory. essential in order to make the act valid, the prescribed form
On 04 December 1991, the appellate court, on appeal to it by being thereby an essential element thereof. The stage of
It is the observation of the Court that this property in dispute private respondent, set aside and declared without force and consummation begins when the parties perform their
was the subject of the Notice of Lis Pendens and that the effect the above questioned orders of the court a quo. respective undertakings under the contract culminating in the
modified decision of this Court promulgated by the Court of extinguishment thereof.
Appeals which had become final to the effect that should the In this petition for review on certiorari, petitioners contend that
defendants decide to offer the property for sale for a price of Buen Realty can be held bound by the writ of execution by Until the contract is perfected, it cannot, as an independent
P11 Million or lower, and considering the mercurial and virtue of the notice of lis pendens, carried over on TCT No. source of obligation, serve as a binding juridical relation. In
uncertain forces in our market economy today, the same right 195816 issued in the name of Buen Realty, at the time of the sales, particularly, to which the topic for discussion about the
of first refusal to herein plaintiffs/appellants in the event that latter's purchase of the property on 15 November 1991 from case at bench belongs, the contract is perfected when a
the subject property is sold for a price in excess of Eleven the Cu Unjiengs. person, called the seller, obligates himself, for a price certain,
Million pesos or more. to deliver and to transfer ownership of a thing or right to
We affirm the decision of the appellate court. another, called the buyer, over which the latter agrees. Article
WHEREFORE, defendants are hereby ordered to execute the 1458 of the Civil Code provides:
necessary Deed of Sale of the property in litigation in favor of A not too recent development in real estate transactions is
plaintiffs Ang Yu Asuncion, Keh Tiong and Arthur Go for the the adoption of such arrangements as the right of first refusal, Art. 1458. By the contract of sale one of the
consideration of P15 Million pesos in recognition of plaintiffs' a purchase option and a contract to sell. For ready reference, contracting parties obligates himself to transfer the ownership
right of first refusal and that a new Transfer Certificate of Title we might point out some fundamental precepts that may find of and to deliver a determinate thing, and the other to pay
be issued in favor of the buyer. some relevance to this discussion. therefor a price certain in money or its equivalent.

All previous transactions involving the same property An obligation is a juridical necessity to give, to do or not to do A contract of sale may be absolute or conditional.
notwithstanding the issuance of another title to Buen Realty (Art. 1156, Civil Code). The obligation is constituted upon the
Corporation, is hereby set aside as having been executed in concurrence of the essential elements thereof, viz: (a) The When the sale is not absolute but conditional, such as in a
bad faith. vinculum juris or juridical tie which is the efficient cause "Contract to Sell" where invariably the ownership of the thing
established by the various sources of obligations (law, sold is retained until the fulfillment of a positive suspensive
SO ORDERED. contracts, quasi-contracts, delicts and quasi-delicts); (b) the condition (normally, the full payment of the purchase price),
object which is the prestation or conduct; required to be the breach of the condition will prevent the obligation to
On September 22, 1991 respondent Judge issued another observed (to give, to do or not to do); and (c) the subject- convey title from acquiring an obligatory force.2 In Dignos vs.
order, the dispositive portion of which reads: persons who, viewed from the demandability of the obligation, Court of Appeals (158 SCRA 375), we have said that,
are the active (obligee) and the passive (obligor) subjects. although denominated a "Deed of Conditional Sale," a sale is
52
Sales – Chapter 3-12 Cases
still absolute where the contract is devoid of any proviso that 43 Phil. 270). Where a period is given to the offeree within and the cause or consideration of the envisioned contract. In
title is reserved or the right to unilaterally rescind is stipulated, which to accept the offer, the following rules generally govern: a right of first refusal, while the object might be made
e.g., until or unless the price is paid. Ownership will then be determinate, the exercise of the right, however, would be
transferred to the buyer upon actual or constructive delivery (1) If the period is not itself founded upon or supported dependent not only on the grantor's eventual intention to
(e.g., by the execution of a public document) of the property by a consideration, the offeror is still free and has the right to enter into a binding juridical relation with another but also on
sold. Where the condition is imposed upon the perfection of withdraw the offer before its acceptance, or, if an acceptance terms, including the price, that obviously are yet to be later
the contract itself, the failure of the condition would prevent has been made, before the offeror's coming to know of such firmed up. Prior thereto, it can at best be so described as
such perfection.3 If the condition is imposed on the obligation fact, by communicating that withdrawal to the offeree (see merely belonging to a class of preparatory juridical relations
of a party which is not fulfilled, the other party may either Art. 1324, Civil Code; see also Atkins, Kroll & Co. vs. Cua, governed not by contracts (since the essential elements to
waive the condition or refuse to proceed with the sale (Art. 102 Phil. 948, holding that this rule is applicable to a establish the vinculum juris would still be indefinite and
1545, Civil Code).4 unilateral promise to sell under Art. 1479, modifying the inconclusive) but by, among other laws of general application,
previous decision in South Western Sugar vs. Atlantic Gulf, the pertinent scattered provisions of the Civil Code on human
An unconditional mutual promise to buy and sell, as long as 97 Phil. 249; see also Art. 1319, Civil Code; Rural Bank of conduct.
the object is made determinate and the price is fixed, can be Parañaque, Inc., vs. Remolado, 135 SCRA 409; Sanchez vs.
obligatory on the parties, and compliance therewith may Rigos, 45 SCRA 368). The right to withdraw, however, must Even on the premise that such right of first refusal has been
accordingly be exacted.5 not be exercised whimsically or arbitrarily; otherwise, it could decreed under a final judgment, like here, its breach cannot
give rise to a damage claim under Article 19 of the Civil Code justify correspondingly an issuance of a writ of execution
An accepted unilateral promise which specifies the thing to be which ordains that "every person must, in the exercise of his under a judgment that merely recognizes its existence, nor
sold and the price to be paid, when coupled with a valuable rights and in the performance of his duties, act with justice, would it sanction an action for specific performance without
consideration distinct and separate from the price, is what give everyone his due, and observe honesty and good faith." thereby negating the indispensable element of consensuality
may properly be termed a perfected contract of option. This in the perfection of contracts.11 It is not to say, however, that
contract is legally binding, and in sales, it conforms with the (2) If the period has a separate consideration, a the right of first refusal would be inconsequential for, such as
second paragraph of Article 1479 of the Civil Code, viz: contract of "option" is deemed perfected, and it would be a already intimated above, an unjustified disregard thereof,
breach of that contract to withdraw the offer during the agreed given, for instance, the circumstances expressed in Article
Art. 1479. ... period. The option, however, is an independent contract by 1912 of the Civil Code, can warrant a recovery for damages.
itself, and it is to be distinguished from the projected main
An accepted unilateral promise to buy or to sell a determinate agreement (subject matter of the option) which is obviously The final judgment in Civil Case No. 87-41058, it must be
thing for a price certain is binding upon the promissor if the yet to be concluded. If, in fact, the optioner-offeror withdraws stressed, has merely accorded a "right of first refusal" in favor
promise is supported by a consideration distinct from the the offer before its acceptance (exercise of the option) by the of petitioners. The consequence of such a declaration entails
price. (1451a)6 optionee-offeree, the latter may not sue for specific no more than what has heretofore been said. In fine, if, as it is
performance on the proposed contract ("object" of the option) here so conveyed to us, petitioners are aggrieved by the
Observe, however, that the option is not the contract of sale since it has failed to reach its own stage of perfection. The failure of private respondents to honor the right of first refusal,
itself.7 The optionee has the right, but not the obligation, to optioner-offeror, however, renders himself liable for damages the remedy is not a writ of execution on the judgment, since
buy. Once the option is exercised timely, i.e., the offer is for breach of the option. In these cases, care should be taken there is none to execute, but an action for damages in a
accepted before a breach of the option, a bilateral promise to of the real nature of the consideration given, for if, in fact, it proper forum for the purpose.
sell and to buy ensues and both parties are then reciprocally has been intended to be part of the consideration for the main
bound to comply with their respective undertakings.8 contract with a right of withdrawal on the part of the optionee, Furthermore, whether private respondent Buen Realty
the main contract could be deemed perfected; a similar Development Corporation, the alleged purchaser of the
Let us elucidate a little. A negotiation is formally initiated by instance would be an "earnest money" in a contract of sale property, has acted in good faith or bad faith and whether or
an offer. An imperfect promise (policitacion) is merely an that can evidence its perfection (Art. 1482, Civil Code). not it should, in any case, be considered bound to respect the
offer. Public advertisements or solicitations and the like are registration of the lis pendens in Civil Case No. 87-41058 are
ordinarily construed as mere invitations to make offers or only In the law on sales, the so-called "right of first refusal" is an matters that must be independently addressed in appropriate
as proposals. These relations, until a contract is perfected, innovative juridical relation. Needless to point out, it cannot proceedings. Buen Realty, not having been impleaded in Civil
are not considered binding commitments. Thus, at any time be deemed a perfected contract of sale under Article 1458 of Case No. 87-41058, cannot be held subject to the writ of
prior to the perfection of the contract, either negotiating party the Civil Code. Neither can the right of first refusal, execution issued by respondent Judge, let alone ousted from
may stop the negotiation. The offer, at this stage, may be understood in its normal concept, per se be brought within the the ownership and possession of the property, without first
withdrawn; the withdrawal is effective immediately after its purview of an option under the second paragraph of Article being duly afforded its day in court.
manifestation, such as by its mailing and not necessarily 1479, aforequoted, or possibly of an offer under Article 13199
when the offeree learns of the withdrawal (Laudico vs. Arias, of the same Code. An option or an offer would require, We are also unable to agree with petitioners that the Court of
among other things,10 a clear certainty on both the object Appeals has erred in holding that the writ of execution varies
53
Sales – Chapter 3-12 Cases
the terms of the judgment in Civil Case No. 87-41058, later
affirmed in CA-G.R. CV-21123. The Court of Appeals, in this
regard, has observed:

Finally, the questioned writ of execution is in variance with the


decision of the trial court as modified by this Court. As
already stated, there was nothing in said decision 13 that
decreed the execution of a deed of sale between the Cu
Unjiengs and respondent lessees, or the fixing of the price of
the sale, or the cancellation of title in the name of petitioner
(Limpin vs. IAC, 147 SCRA 516; Pamantasan ng Lungsod ng
Maynila vs. IAC, 143 SCRA 311; De Guzman vs. CA, 137
SCRA 730; Pastor vs. CA, 122 SCRA 885).

It is likewise quite obvious to us that the decision in Civil Case


No. 87-41058 could not have decreed at the time the
execution of any deed of sale between the Cu Unjiengs and
petitioners.

WHEREFORE, we UPHOLD the Court of Appeals in


ultimately setting aside the questioned Orders, dated 30
August 1991 and 27 September 1991, of the court a quo.
Costs against petitioners.

SO ORDERED.

54
Sales – Chapter 3-12 Cases
PARAÑAQUE KINGS ENTERPRISES, INCORPORATED, 2. Defendant Catalina L. Santos is the owner of eight handed by the attorney-in-fact of defendant Santos, Xerox
petitioner, (8) parcels of land located at (sic) Parañaque, Metro Manila copy of the letter is hereto attached as Annex "N".
vs. with transfer certificate of title nos. S-19637, S-19638 and S-
COURT OF APPEALS, CATALINA L. SANTOS, 19643 to S-19648. Xerox copies of the said title (sic) are 9. Upon learning of this fact plaintiff's representative
represented by her attorney-in-fact, LUZ B. PROTACIO, hereto attached as Annexes "B" to "I", respectively. wrote a letter to defendant Santos, requesting her to rectify
and DAVID A. RAYMUNDO, respondents. the error and consequently realizing the error, she had it
3. On November 28, 1977, a certain Frederick Chua reconveyed to her for the same consideration of FIVE
Do allegations in a complaint showing violation of a leased the above-described property from defendant Catalina MILLION (P5,000,000.00) PESOS. Xerox copies of the letter
contractual right of "first option or priority to buy the properties L. Santos, the said lease was registered in the Register of and the deed of reconveyance are hereto attached as
subject of the lease" constitute a valid cause of action? Is the Deeds. Xerox copy of the lease is hereto attached as Annex Annexes "O" and "P".
grantee of such right entitled to be offered the same terms "J".
and conditions as those given to a third party who eventually 10. Subsequently the property was offered for sale to
bought such properties? In short, is such right of first refusal 4. On February 12, 1979, Frederick Chua assigned all plaintiff by the defendant for the sum of FIFTEEN MILLION
enforceable by an action for specific performance? his rights and interest and participation in the leased property (P15,000,000.00) PESOS. Plaintiff was given ten (10) days to
to Lee Ching Bing, by virtue of a deed of assignment and with make good of the offer, but therefore (sic) the said period
These questions are answered in the affirmative by this Court the conformity of defendant Santos, the said assignment was expired another letter came from the counsel of defendant
in resolving this petition for review under Rule 45 of the Rules also registered. Xerox copy of the deed of assignment is Santos, containing the same tenor of (sic) the former letter.
of Court challenging the Decision 1 of the Court of Appeals 2 hereto attached as Annex "K". Xerox copies of the letters are hereto attached as Annexes
promulgated on March 29, 1993, in CA-G.R. CV No. 34987 "Q" and "R".
entitled "Parañaque Kings Enterprises, Inc. vs. Catalina L. 5. On August 6, 1979, Lee Ching Bing also assigned
Santos, et al.," which affirmed the order 3 of September 2, all his rights and interest in the leased property to Parañaque 11. On May 8, 1989, before the period given in the letter
1991, of the Regional Trial Court of Makati, Branch 57, 4 Kings Enterprises, Incorporated by virtue of a deed of offering the properties for sale expired, plaintiff's counsel
dismissing Civil Case No. 91-786 for lack of a valid cause of assignment and with the conformity of defendant Santos, the wrote counsel of defendant Santos offering to buy the
action. same was duly registered, Xerox copy of the deed of properties for FIVE MILLION (P5,000,000.00) PESOS. Xerox
assignment is hereto attached as Annex "L". copy of the letter is hereto attached as Annex "S".
Facts of the Case
6. Paragraph 9 of the assigned leased (sic) contract 12. On May 15, 1989, before they replied to the offer to
On March 19, 1991, herein petitioner filed before the provides among others that: purchase, another deed of sale was executed by defendant
Regional Trial Court of Makati a complaint, 5 which is Santos (in favor of) defendant Raymundo for a consideration
reproduced in full below: "9. That in case the properties subject of the lease of NINE MILLION (P9,000,000.00) PESOS. Xerox copy of the
agreement are sold or encumbered, Lessors shall impose as second deed of sale is hereto attached as Annex "T".
Plaintiff, by counsel, respectfully states that: a condition that the buyer or mortgagee thereof shall
recognize and be bound by all the terms and conditions of 13. Defendant Santos violated again paragraph 9 of the
1. Plaintiff is a private corporation organized and this lease agreement and shall respect this Contract of Lease contract of lease by executing a second deed of sale to
existing under and by virtue of the laws of the Philippines, as if they are the LESSORS thereof and in case of sale, defendant Raymundo.
with principal place of business of (sic) Dr. A. Santos Avenue, LESSEE shall have the first option or priority to buy the
Parañaque, Metro Manila, while defendant Catalina L. properties subject of the lease;" 14. It was only on May 17, 1989, that defendant Santos
Santos, is of legal age, widow, with residence and postal replied to the letter of the plaintiff's offer to buy or two days
address at 444 Plato Street, Ct., Stockton, California, USA, 7. On September 21, 1988, defendant Santos sold the after she sold her properties. In her reply she stated among
represented in this action by her attorney-in-fact, Luz B. eight parcels of land subject of the lease to defendant David others that the period has lapsed and the plaintiff is not a
Protacio, with residence and postal address at No, 12, San Raymundo for a consideration of FIVE MILLION privy (sic) to the contract. Xerox copy of the letter is hereto
Antonio Street, Magallanes Village, Makati, Metro Manila, by (P5,000,000.00) PESOS. The said sale was in contravention attached as Annex "U".
virtue of a general power of attorney. Defendant David A. of the contract of lease, for the first option or priority to buy
Raymundo, is of legal age, single, with residence and postal was not offered by defendant Santos to the plaintiff. Xerox 15. On June 28, 1989, counsel for plaintiff informed
address at 1918 Kamias Street, Damariñas Village, Makati, copy of the deed of sale is hereto attached as Annex "M". counsel of defendant Santos of the fact that plaintiff is the
Metro Manila, where they (sic) may be served with summons assignee of all rights and interest of the former lessor. Xerox
and other court processes. Xerox copy of the general power 8. On March 5, 1989, defendant Santos wrote a letter copy of the letter is hereto attached as Annex "V".
of attorney is hereto attached as Annex "A". to the plaintiff informing the same of the sale of the properties
to defendant Raymundo, the said letter was personally 16. On July 6, 1989, counsel for defendant Santos
informed the plaintiff that the new owner is defendant
55
Sales – Chapter 3-12 Cases
Raymundo. Xerox copy of the letter is hereto attached as 24. The defendants acted in a wanton, fraudulent, On September 2, 1991, the trial court issued the order
Annex "W". reckless, oppressive or malevolent manner and as a dismissing the complaint for lack of a valid cause of action. It
deterrent to the commission of similar acts, they should be ratiocinated thus:
17. From the preceding facts it is clear that the sale was made to answer for exemplary damages, the amount left to
simulated and that there was a collusion between the the discretion of the Court. Upon the very face of the plaintiff's Complaint itself, it
defendants in the sales of the leased properties, on the therefore indubitably appears that the defendant Santos had
ground that when plaintiff wrote a letter to defendant Santos 25. Plaintiff demanded from the defendants to rectify verily complied with paragraph 9 of the Lease Agreement by
to rectify the error, she immediately have (sic) the property their unlawful acts that they committed, but defendants twice offering the properties for sale to the plaintiff for ~1 5 M.
reconveyed it (sic) to her in a matter of twelve (12) days. refused and failed to comply with plaintiffs just and valid and The said offers, however, were plainly rejected by the plaintiff
(sic) demands. Xerox copies of the demand letters are hereto which scorned the said offer as "RIDICULOUS". There was
18. Defendants have the same counsel who attached as Annexes "KK" to "LL", respectively. therefore a definite refusal on the part of the plaintiff to accept
represented both of them in their exchange of communication the offer of defendant Santos. For in acquiring the said
with plaintiff's counsel, a fact that led to the conclusion that a 26. Despite repeated demands, defendants failed and properties back to her name, and in so making the offers to
collusion exist (sic) between the defendants. refused without justifiable cause to satisfy plaintiff's claim, sell both by herself (attorney-in-fact) and through her counsel,
and was constrained to engaged (sic) the services of defendant Santos was indeed conscientiously complying with
19. When the property was still registered in the name undersigned counsel to institute this action at a contract fee her obligation under paragraph 9 of the Lease Agreement. . .
of defendant Santos, her collector of the rental of the leased of P200,000.00, as and for attorney's fees, exclusive of cost ..
properties was her brother-in-law David Santos and when it and expenses of litigation.
was transferred to defendant Raymundo the collector was still xxx xxx xxx
David Santos up to the month of June, 1990. Xerox copies of PRAYER
cash vouchers are hereto attached as Annexes "X" to "HH", This is indeed one instance where a Complaint, after barely
respectively. WHEREFORE, it is respectfully prayed, that judgment be commencing to create a cause of action, neutralized itself by
rendered in favor of the plaintiff and against defendants and its subsequent averments which erased or extinguished its
20. The purpose of this unholy alliance between ordering that: earlier allegations of an impending wrong. Consequently,
defendants Santos and Raymundo is to mislead the plaintiff absent any actionable wrong in the very face of the Complaint
and make it appear that the price of the leased property is a. The Deed of Sale between defendants dated May itself, the plaintiffs subsequent protestations of collusion is
much higher than its actual value of FIVE MILLION 15, 1989, be annulled and the leased properties be sold to bereft or devoid of any meaning or purpose. . . . .
(P5,000,000.00) PESOS, so that plaintiff would purchase the the plaintiff in the amount of P5,000,000.00;
properties at a higher price. The inescapable result of the foregoing considerations point
b. Dependants (sic) pay plaintiff the sum of to no other conclusion than that the Complaint actually does
21. Plaintiff has made considerable investments in the P3,000,000.00 as actual damages; not contain any valid cause of action and should therefore be
said leased property by erecting a two (2) storey, six (6) as it is hereby ordered DISMISSED. The Court finds no
doors commercial building amounting to THREE MILLION c. Defendants pay the sum of P5,000,000.00 as moral further need to consider the other grounds of estoppel and
(P3,000,000.00) PESOS. This considerable improvement damages; laches inasmuch as this resolution is sufficient to dispose the
was made on the belief that eventually the said premises matter. 6
shall be sold to the plaintiff. d. Defendants pay exemplary damages left to the
discretion of the Court; Petitioners appealed to the Court of Appeals which affirmed
22. As a consequence of this unlawful act of the in toto the ruling of the trial court, and further reasoned that:
defendants, plaintiff will incurr (sic) total loss of THREE e. Defendants pay the sum of not less than
MILLION (P3,000,000.00) PESOS as the actual cost of the P200,000.00 as attorney's fees. . . . . Appellant's protestations that the P15 million price
building and as such defendants should be charged of the quoted by appellee Santos was reduced to P9 million when
same amount for actual damages. Plaintiff further prays for other just and equitable reliefs plus she later resold the leased properties to Raymundo has no
cost of suit. valid legal moorings because appellant, as a prospective
23. As a consequence of the collusion, evil design and buyer, cannot dictate its own price and forcibly ram it against
illegal acts of the defendants, plaintiff in the process suffered Instead of filing their respective answers, respondents filed appellee Santos, as owner, to buy off her leased properties
mental anguish, sleepless nights, bismirched (sic) reputation motions to dismiss anchored on the grounds of lack of cause considering the total absence of any stipulation or agreement
which entitles plaintiff to moral damages in the amount of of action, estoppel and laches. as to the price or as to how the price should be computed
FIVE MILLION (P5,000,000.00) PESOS. under paragraph 9 of the lease contract, . . . . 7

56
Sales – Chapter 3-12 Cases
Petitioner moved for reconsideration but was denied in an grounds for dismissal, claiming that petitioner's payment of complaint to constitute a cause of action is whether, admitting
order dated August 20, 1993. 8 rentals of the leased property to respondent Raymundo from the facts alleged, the court could render a valid judgment
June 15, 1989, to June 30, 1990, was an acknowledgment of upon the same in accordance with the prayer of the petition
Hence this petition. Subsequently, petitioner filed an "Urgent the latter's status as new owner-lessor of said property, by or complaint.
Motion for the Issuance of Restraining Order and/or Writ of virtue of which petitioner is deemed to have waived or
Preliminary Injunction and to Hold Respondent David A. abandoned its first option to purchase. A cause of action exists if the following elements are present:
Raymundo in Contempt of Court." 9 The motion sought to (1) a right in favor of the plaintiff by whatever means and
enjoin respondent Raymundo and his counsel from pursuing Private respondents likewise contend that the deed of under whatever law it arises or is created; (2) an obligation on
the ejectment complaint filed before the barangay captain of assignment of the lease agreement did not include the the part of the named defendant to respect or not to violate
San Isidro, Parañaque, Metro Manila; to direct the dismissal assignment of the option to purchase. Respondent such right, and (3) an act or omission on the part of such
of said ejectment complaint or of any similar action that may Raymundo further avers that he was not privy to the contract defendant violative of the right of plaintiff or constituting a
have been filed; and to require respondent Raymundo to of lease, being neither the lessor nor lessee adverted to breach of the obligation of defendant to the plaintiff for which
explain why he should not be held in contempt of court for therein, hence he could not be held liable for violation thereof. the latter may maintain an action for recovery of damages. 12
forum-shopping. The ejectment suit initiated by respondent
Raymundo against petitioner arose from the expiration of the The Court's Ruling In determining whether allegations of a complaint are
lease contract covering the property subject of this case. The sufficient to support a cause of action, it must be borne in
ejectment suit was decided in favor of Raymundo, and the Preliminary Issue: Failure to File mind that the complaint does not have to establish or allege
entry of final judgment in respect thereof renders the said Sufficient Copies of Brief facts proving the existence of a cause of action at the outset;
motion moot and academic. this will have to be done at the trial on the merits of the case.
We first dispose of the procedural issue raised by To sustain a motion to dismiss for lack of cause of action, the
Issue respondents, particularly petitioner's failure to file twelve (12) complaint must show that the claim for relief does not exist,
copies of its brief. We have ruled that when non-compliance rather than that a claim has been defectively stated, or is
The principal legal issue presented before us for resolution is with the Rules was not intended for delay or did not result in ambiguous, indefinite or uncertain. 13
whether the aforequoted complaint alleging breach of the prejudice to the adverse party, dismissal of appeal on mere
contractual right of "first option or priority to buy" states a technicalities — in cases where appeal is a matter of right — Equally important, a defendant moving to dismiss a complaint
valid cause of action. may be stayed, in the exercise of the court's equity on the ground of lack of cause of action is regarded as having
jurisdiction. 10 It does not appear that respondents were hypothetically admitted all the averments thereof. 14
Petitioner contends that the trial court as well as the appellate unduly prejudiced by petitioner's nonfeasance. Neither has it
tribunal erred in dismissing the complaint because it in fact been shown that such failure was intentional. A careful examination of the complaint reveals that it
had not just one but at least three (3) valid causes of action, sufficiently alleges an actionable contractual breach on the
to wit: (1) breach of contract, (2) its right of first refusal Main Issue: Validity of Cause of Action part of private respondents. Under paragraph 9 of the
founded in law, and (3) damages. contract of lease between respondent Santos and petitioner,
We do not agree with respondents' contention that the issue the latter was granted the "first option or priority" to purchase
Respondents Santos and Raymundo, in their separate involved is purely factual. The principal legal question, as the leased properties in case Santos decided to sell. If Santos
comments, aver that the petition should be denied for not stated earlier, is whether the complaint filed by herein never decided to sell at all, there can never be a breach,
raising a question of law as the issue involved is purely petitioner in the lower court states a valid cause of action. much less an enforcement of such "right." But on September
factual — whether respondent Santos complied with Since such question assumes the facts alleged in the 21, 1988, Santos sold said properties to Respondent
paragraph 9 of the lease agreement — and for not having complaint as true, it follows that the determination thereof is Raymundo without first offering these to petitioner. Santos
complied with Section 2, Rule 45 of the Rules of Court, one of law, and not of facts. There is a question of law in a indeed realized her error, since she repurchased the
requiring the filing of twelve (12) copies of the petitioner's given case when the doubt or difference arises as to what the properties after petitioner complained. Thereafter, she offered
brief. Both maintain that the complaint filed by petitioner law is on a certain state of facts, and there is a question of to sell the properties to petitioner for P15 million, which
before the Regional Trial Court of Makati stated no valid fact when the doubt or difference arises as to the truth or the petitioner, however, rejected because of the "ridiculous" price.
cause of action and that petitioner failed to substantiate its falsehood of alleged facts. 11 But Santos again appeared to have violated the same
claim that the lower courts decided the same "in a way not in provision of the lease contract when she finally resold the
accord with law and applicable decisions of the Supreme At the outset, petitioner concedes that when the ground for a properties to respondent Raymundo for only P9 million
Court"; or that the Court of Appeals has "sanctioned motion to dismiss is lack of cause of action, such ground without first offering them to petitioner at such price. Whether
departure by a trial court from the accepted and usual course must appear on the face of the complaint; that to determine there was actual breach which entitled petitioner to damages
of judicial proceedings" so as to merit the exercise by this the sufficiency of a cause of action, only the facts alleged in and/or other just or equitable relief, is a question which can
Court of the power of review under Rule 45 of the Rules of the complaint and no others should be considered; and that better be resolved after trial on the merits where each party
Court. Furthermore, they reiterate estoppel and laches as the test of sufficiency of the facts alleged in a petition or
57
Sales – Chapter 3-12 Cases
can present evidence to prove their respective allegations Without probing into whether petitioner is rightfully a
and defenses. 15 From the foregoing, the basis of the right of first refusal* must beneficiary under said law, suffice it to say that this Court has
be the current offer to sell of the seller or offer to purchase of previously ruled that under
The trial and appellate courts based their decision to sustain any prospective buyer. Only after the optionee fails to Section 6 18 of P.D. 1517, "the terms and conditions of the
respondents' motion to dismiss on the allegations of exercise its right of first priority under the same terms and sale in the exercise of the lessee's right of first refusal to
Parañaque Kings Enterprises that Santos had actually offered within the period contemplated, could the owner validly offer purchase shall be determined by the Urban Zone
the subject properties for sale to it prior to the final sale in to sell the property to a third person, again, under the same Expropriation and Land Management Committee. Hence, . . .
favor of Raymundo, but that the offer was rejected. According terms as offered to the optionee. . certain prerequisites must be complied with by anyone who
to said courts, with such offer, Santos had verily complied wishes to avail himself of the benefits of the decree." 19
with her obligation to grant the right of first refusal to This principle was reiterated in the very recent case of There being no allegation in its complaint that the
petitioner. Equatorial Realty vs. Mayfair Theater, Inc. 17 which was prerequisites were complied with, it is clear that the complaint
decided en banc. This Court upheld the right of first refusal of did fail to state a cause of action on this ground.
We hold, however, that in order to have full compliance with the lessee Mayfair, and rescinded the sale of the property by
the contractual right granting petitioner the first option to the lessor Carmelo to Equatorial Realty "considering that Deed of Assignment included
purchase, the sale of the properties for the amount of P9 Mayfair, which had substantial interest over the subject the option to purchase
million, the price for which they were finally sold to property, was prejudiced by its sale to Equatorial without
respondent Raymundo, should have likewise been first Carmelo conferring to Mayfair every opportunity to negotiate Neither do we find merit in the contention of respondent
offered to petitioner. within the 30-day stipulated period" (emphasis supplied). Santos that the assignment of the lease contract to petitioner
did not include the option to purchase. The provisions of the
The Court has made an extensive and lengthy discourse on In that case, two contracts of lease between Carmelo and deeds of assignment with regard to matters assigned were
the concept of, and obligations under, a right of first refusal in Mayfair provided "that if the LESSOR should desire to sell the very clear. Under the first assignment between Frederick
the case of Guzman, Bocaling & Co. vs. Bonnevie. 16 In that leased premises, the LESSEE shall be given 30 days Chua as assignor and Lee Ching Bing as assignee, it was
case, under a contract of lease, the lessees (Raul and exclusive option to purchase the same." Carmelo initially expressly stated that:
Christopher Bonnevie) were given a "right of first priority" to offered to sell the leased property to Mayfair for six to seven
purchase the leased property in case the lessor (Reynoso) million pesos. Mayfair indicated interest in purchasing the . . . . the ASSIGNOR hereby CEDES, TRANSFERS and
decided to sell. The selling price quoted to the Bonnevies was property though it invoked the 30-day period. Nothing was ASSIGNS to herein ASSIGNEE, all his rights, interest and
600,000.00 to be fully paid in cash, less a mortgage lien of heard thereafter from Carmelo. Four years later, the latter participation over said premises afore-described, . . . . 20
P100,000.00. On the other hand, the selling price offered by sold its entire Recto Avenue property, including the leased (emphasis supplied)
Reynoso to and accepted by Guzman was only P400,000.00 premises, to Equatorial for P11,300,000.00 without priorly
of which P137,500.00 was to be paid in cash while the informing Mayfair. The Court held that both Carmelo and And under the subsequent assignment executed between
balance was to be paid only when the property was cleared of Equatorial acted in bad faith: Carmelo for knowingly violating Lee Ching Bing as assignor and the petitioner, represented
occupants. We held that even if the Bonnevies could not buy the right of first option of Mayfair, and Equatorial for by its Vice President Vicenta Lo Chiong, as assignee, it was
it at the price quoted (P600,000.00), nonetheless, Reynoso purchasing the property despite being aware of the contract likewise expressly stipulated that;
could not sell it to another for a lower price and under more stipulation. In addition to rescission of the contract of sale, the
favorable terms and conditions without first offering said Court ordered Carmelo to allow Mayfair to buy the subject . . . . the ASSIGNOR hereby sells, transfers and assigns all
favorable terms and price to the Bonnevies as well. Only if property at the same price of P11,300,000.00. his rights, interest and participation over said leased
the Bonnevies failed to exercise their right of first priority premises, . . . . 21 (emphasis supplied)
could Reynoso thereafter lawfully sell the subject property to No cause of action
others, and only under the same terms and conditions under P.D. 1517 One of such rights included in the contract of lease and,
previously offered to the Bonnevies. therefore, in the assignments of rights was the lessee's right
Petitioner also invokes Presidential Decree No. 1517, or the of first option or priority to buy the properties subject of the
Of course, under their contract, they specifically stipulated Urban Land Reform Law, as another source of its right of first lease, as provided in paragraph 9 of the assigned lease
that the Bonnevies could exercise the right of first priority, "all refusal. It claims to be covered under said law, being the contract. The deed of assignment need not be very specific
things and conditions being equal." This Court interpreted this "rightful occupant of the land and its structures" since it is the as to which rights and obligations were passed on to the
proviso to mean that there should be identity of terms and lawful lessee thereof by reason of contract. Under the lease assignee. It is understood in the general provision
conditions to be offered to the Bonnevies and all other contract, petitioner would have occupied the property for aforequoted that all specific rights and obligations contained
prospective buyers, with the Bonnevies to enjoy the right of fourteen (14) years at the end of the contractual period. in the contract of lease are those referred to as being
first priority. We hold that the same rule applies even without assigned. Needless to state, respondent Santos gave her
the same proviso if the right of first refusal (or the first option unqualified conformity to both assignments of rights.
to buy) is not to be rendered illusory.
58
Sales – Chapter 3-12 Cases
Respondent Raymundo privy REVERSED and SET ASIDE. The case is REMANDED to
to the Contract of Lease the Regional Trial Court of Makati for further proceedings.

With respect to the contention of respondent Raymundo that SO ORDERED.


he is not privy to the lease contract, not being the lessor nor
the lessee referred to therein, he could thus not have violated
its provisions, but he is nevertheless a proper party. Clearly,
he stepped into the shoes of the owner-lessor of the land as,
by virtue of his purchase, he assumed all the obligations of
the lessor under the lease contract. Moreover, he received
benefits in the form of rental payments. Furthermore, the
complaint, as well as the petition, prayed for the annulment of
the sale of the properties to him. Both pleadings also alleged
collusion between him and respondent Santos which
defeated the exercise by petitioner of its right of first refusal.

In order then to accord complete relief to petitioner,


respondent Raymundo was a necessary, if not indispensable,
party to the case. 22 A favorable judgment for the petitioner
will necessarily affect the rights of respondent Raymundo as
the buyer of the property over which petitioner would like to
assert its right of first option to buy.

Having come to the conclusion that the complaint states a


valid cause of action for breach of the right of first refusal and
that the trial court should thus not have dismissed the
complaint, we find no more need to pass upon the question of
whether the complaint states a cause of action for damages
or whether the complaint is barred by estoppel or laches. As
these matters require presentation and/or determination of
facts, they can be best resolved after trial on the merits.

While the lower courts erred in dismissing the complaint,


private respondents, however, cannot be denied their day in
court. While, in the resolution of a motion to dismiss, the truth
of the facts alleged in the complaint are theoretically
admitted, such admission is merely hypothetical and only for
the purpose of resolving the motion. In case of denial, the
movant is not to be deprived of the right to submit its own
case and to submit evidence to rebut the allegations in the
complaint. Neither will the grant of the motion by a trial court
and the ultimate reversal thereof by an appellate court have
the effect of stifling such right. 23 So too, the trial court should
be given the opportunity to evaluate the evidence, apply the
law and decree the proper remedy. Hence, we remand the
instant case to the trial court to allow private respondents to
have their day in court.

WHEREFORE, the petition is GRANTED. The assailed


decisions of the trial court and Court of Appeals are hereby
59
Sales – Chapter 3-12 Cases
ROSENCOR DEVELOPMENT CORPORATION and Upon the death of the spouses Tiangcos in 1975, the deed of sale, the latter turned down their request and
RENE JOAQUIN, petitioners, management of the property was adjudicated to their heirs instead Atty. Aguila wrote them several letters demanding
vs. who were represented by Eufrocina de Leon. The lessees that they vacate the premises. The lessees offered to
PATERNO INQUING, IRENE GUILLERMO, FEDERICO were allegedly promised the same pre-emptive right by tender their rental payment to de Leon but she refused to
BANTUGAN, FERNANDO MAGBANUA and LIZZA the heirs of Tiangcos since the latter had knowledge that accept the same.
TIANGCO, respondents. this right was extended to the former by the late spouses
Tiangcos. The lessees continued to stay in the premises In April 1992 before the demolition can be undertaken by
This is a petition for review on certiorari under Rule 45 of and allegedly spent their own money amounting from the Building Official, the barangay interceded between the
the Rules of Court seeking reversal of the Decision1 of the P50,000.00 to P100,000.00 for its upkeep. These parties herein after which Rosencor raised the issue as to
Court of Appeals dated June 25, 1999 in CA-G.R. CV No. expenses were never deducted from the rentals which the rental payment of the premises. It was also at this
53963. The Court of Appeals decision reversed and set already increased to P1,000.00. instance that the lessees were furnished with a copy of
aside the Decision2 dated May 13, 1996 of Branch 217 of the Deed of Sale and discovered that they were deceived
the Regional Trial Court of Quezon City in Civil Case No. In June 1990, the lessees received a letter from Atty. by de Leon since the sale between her and Rene
Q-93-18582.1âwphi1.nêt Erlinda Aguila demanding that they vacate the premises Joaquin/Rosencor took place in September 4, 1990 while
so that the demolition of the building be undertaken. They de Leon made the offer to them only in October 1990 or
The case was originally filed on December 10, 1993 by refused to leave the premises. In that same month, de after the sale with Rosencor had been consummated. The
Paterno Inquing, Irene Guillermo and Federico Bantugan, Leon refused to accept the lessees’ rental payment lessees also noted that the property was sold only for
herein respondents, against Rosencor Development claiming that they have run out of receipts and that a new P726,000.00.
Corporation (hereinafter "Rosencor"), Rene Joaquin, and collector has been assigned to receive the payments.
Eufrocina de Leon. Originally, the complaint was one for Thereafter, they received a letter from Eufrocina de Leon The lessees offered to reimburse de Leon the selling price
annulment of absolute deed of sale but was later offering to sell to them the property they were leasing for of P726,000.00 plus an additional P274,000.00 to
amended to one for rescission of absolute deed of sale. A P2,000,000.00. xxx. complete their P1,000.000.00 earlier offer. When their
complaint-for intervention was thereafter filed by offer was refused, they filed the present action praying for
respondents Fernando Magbanua and Danna Lizza The lessees offered to buy the property from de Leon for the following: a) rescission of the Deed of Absolute Sale
Tiangco. The complaint-in-intervention was admitted by the amount of P1,000,000.00. De Leon told them that she between de Leon and Rosencor dated September 4,
the trial court in an Order dated May 4, 1994.3 will be submitting the offer to the other heirs. Since then, 1990; b) the defendants Rosencor/Rene Joaquin be
no answer was given by de Leon as to their offer to buy ordered to reconvey the property to de Leon; and c) de
The facts of the case, as stated by the trial court and the property. However, in November 1990, Rene Joaquin Leon be ordered to reimburse the plaintiffs for the repairs
adopted by the appellate court, are as follows: came to the leased premises introducing himself as its of the property, or apply the said amount as part of the
new owner. price for the purchase of the property in the sum of
"This action was originally for the annulment of the Deed P100,000.00."4
of Absolute Sale dated September 4, 1990 between In January 1991, the lessees again received another letter
defendants Rosencor and Eufrocina de Leon but later from Atty. Aguila demanding that they vacate the After trial on the merits, the Regional Trial Court rendered
amended (sic) praying for the rescission of the deed of premises. A month thereafter, the lessees received a a Decision5 dated May 13, 1996 dismissing the complaint.
sale. letter from de Leon advising them that the heirs of the late The trial court held that the right of redemption on which
spouses Tiangcos have already sold the property to the complaint. The trial court held that the right of
Plaintiffs and plaintiffs-intervenors averred that they are Rosencor. The following month Atty. Aguila wrote them redemption on which the complaint was based was merely
the lessees since 1971 of a two-story residential another letter demanding the rental payment and an oral one and as such, is unenforceable under the law.
apartment located at No. 150 Tomas Morato Ave., introducing herself as counsel for Rosencor/Rene The dispositive portion of the May 13, 1996 Decision is as
Quezon City covered by TCT No. 96161 and owned by Joaquin, the new owners of the premises. follows:
spouses Faustino and Cresencia Tiangco. The lease was
not covered by any contract. The lessees were renting the The lessees requested from de Leon why she had "WHEREFORE, in view of the foregoing, the Court
premises then for P150.00 a month and were allegedly disregarded the pre-emptive right she and the late DISMISSES the instant action. Plaintiffs and plaintiffs-
verbally granted by the lessors the pre-emptive right to Tiangcos have promised them. They also asked for a intervenors are hereby ordered to pay their respective
purchase the property if ever they decide to sell the same. copy of the deed of sale between her and the new owners monthly rental of P1,000.00 per month reckoned from May
thereof but she refused to heed their request. In the same 1990 up to the time they leave the premises. No costs.
manner, when they asked Rene Joaquin a copy of the
60
Sales – Chapter 3-12 Cases
SO ORDERED."6 resolution of the issue on the applicability of the statute of
I. frauds is important as it will determine the type of
Not satisfied with the decision of the trial court, evidence which may be considered by the trial court as
respondents herein filed a Notice of Appeal dated June 3, THE COURT OF APPEALS GRAVELY ERRED WHEN IT proof of the alleged right of first refusal.
1996. On the same date, the trial court issued an Order ORDERED THE RESCISSION OF THE ABSOLUTE
for the elevation of the records of the case to the Court of DEED OF SALE BETWEEN EUFROCINA DE LEON AND The term "statute of frauds" is descriptive of statutes
Appeals. On August 8, 1997, respondents filed their PETITIONER ROSENCOR. which require certain classes of contracts to be in writing.
appellate brief before the Court of Appeals. This statute does not deprive the parties of the right to
II. contract with respect to the matters therein involved, but
On June 25, 1999, the Court of Appeals rendered its merely regulates the formalities of the contract necessary
decision7 reversing the decision of the trial court. The THE COURT OF APPEALS COMMTITED MANIFEST to render it enforceable. Thus, they are included in the
dispositive portion of the June 25, 1999 decision is as ERROR IN MANDATING THAT EUFROCINA DE LEON provisions of the New Civil Code regarding unenforceable
follows: AFFORD RESPONDENTS THE OPPORTUNITY TO contracts, more particularly Art. 1403, paragraph 2. Said
EXERCISE THEIR RIGHT OF FIRST REFUSAL. article provides, as follows:
"WHEREFORE, premises considered, the appealed
decision (dated May 13, 1996) of the Regional Trial Court III. "Art. 1403. The following contracts are unenforceable,
(Branch 217) in Quezon City in Case No. Q-93-18582 is unless they are ratified:
hereby REVERSED and SET ASIDE. In its stead, a new THE COURT OF APPEALS GRIEVOUSLY ERRED IN
one is rendered ordering: CONCLUDING THAT RESPONDENTS HAVE xxx
ESTABLISHED THEIR RIGHT OF FIRST REFUSAL
(1) The rescission of the Deed of Absolute Sale executed DESPITE PETITIONERS’ RELIANCE ON THEIR (2) Those that do not comply with the Statute of Frauds as
between the appellees on September 4, 1990; DEFENSE BASED ON THE STATUTE OF FRAUDS. set forth in this number. In the following cases an
agreement hereafter made shall be unenforceable by
(2) The reconveyance of the subject premises to appellee Eufrocina de Leon, for herself and for the heirs of the action, unless the same, or some note or memorandum
Eufrocina de Leon; spouses Faustino and Crescencia Tiangco, did not appeal thereof, be in writing, and subscribed by the party
the decision of the Court of Appeals. charged, or by his agent; evidence, therefore, of the
(3) The heirs of Faustino and Crescencia Tiangco, thru agreement cannot be received without the writing, or a
appellee Eufrocina de Leon, to afford the appellants thirty At the onset, we not that both the Court of Appeals and secondary evidence of its contents:
days within which to exercise their right of first refusal by the Regional Trial Court relied on Article 1403 of the New
paying the amount of ONE MILLION PESOS Civil Code, more specifically the provisions on the statute a) An agreement that by its terms is not to be performed
(P1,000,000.00) for the subject property; and of frauds, in coming out with their respective decisions. within a year from the making thereof;
The trial court, in denying the petition for reconveyance,
(4) The appellants to, in turn, pay the appellees back held that right of first refusal relied upon by petitioners was b) A special promise to answer for the debt, default, or
rentals from May 1990 up to the time this decision is not reduced to writing and as such, is unenforceable by miscarriage of another;
promulgated. virtue of the said article. The Court of Appeals, on the
other hand, also held that the statute of frauds governs c) An agreement made in consideration of marriage, other
No pronouncement as to costs. the "right of first refusal" claimed by respondents. than a mutual promise to marry;
However, the appellate court ruled that respondents had
SO ORDERED".8 duly proven the same by reason of petitioners’ waiver of d) An agreement for the sale of goods, chattels or things
the protection of the statute by reason of their failure to in action, at a price not less than five hundred pesos,
Petitioners herein filed a Motion for Reconsideration of the object to the presentation of oral evidence of the said unless the buyer accept and receive part of such goods
decision of the Court of Appeals but the same was denied right. and chattels, or the evidences, or some of them, of such
in a Resolution dated October 15, 1999.9 things in action, or pay at the time some part of the
Both the appellate court and the trial court failed to purchase money; but when a sale is made by auction and
Hence, this petition for review on certiorari where discuss, however, the threshold issue of whether or not a entry is made by the auctioneer in his sales book, at the
petitioners Rosencor Development Corporation and Rene right of first refusal is indeed covered by the provisions of time of the sale, of the amount and kind of property sold,
Joaquin raise the following assignment of errors10: the New Civil Code on the statute of frauds. The terms of sale, price, names of purchasers and person on
61
Sales – Chapter 3-12 Cases
whose account the sale is made, it is a sufficient answered is whether or not the Court of Appeals erred in
memorandum; It is thus evident that the statute of frauds does not ordering the rescission of the Deed of Absolute Sale dated
contemplate cases involving a right of first refusal. As September 4, 1990 between Rosencor and Eufrocina de
e) An agreement for the leasing of a longer period than such, a right of first refusal need not be written to be Leon and in decreeing that the heirs of the spouses
one year, or for the sale of real property or of an interest enforceable and may be proven by oral evidence. Tiangco should afford respondents the exercise of their
therein; right of first refusal. In other words, may a contract of sale
The next question to be ascertained is whether or not entered into in violation of a third party’s right of first
f) A representation to the credit of a third person." respondents have satisfactorily proven their right of first refusal be rescinded in order that such third party can
refusal over the property subject of the Deed of Absolute exercise said right?
The purpose of the statute is to prevent fraud and perjury Sale dated September 4, 1990 between petitioner
in the enforcement of obligations depending for their Rosencor and Eufrocina de Leon. The issue is not one of first impression.
evidence on the unassisted memory of witnesses by
requiring certain enumerated contracts and transactions to On this point, we agree with the factual findings of the In Guzman, Bocaling and Co, Inc. vs. Bonnevie23, the
be evidenced by a writing signed by the party to be Court of Appeals that respondents have adequately Court upheld the decision of a lower court ordering the
charged.11 Moreover, the statute of frauds refers to proven the existence of their right of first refusal. Federico rescission of a deed of sale which violated a right of first
specific kinds of transactions and cannot apply to any Bantugan, Irene Guillermo, and Paterno Inquing uniformly refusal granted to one of the parties therein. The Court
other transaction that is not enumerated therein.12 The testified that they were promised by the late spouses held:
application of such statute presupposes the existence of a Faustino and Crescencia Tiangco and, later on, by their
perfected contract.13 heirs a right of first refusal over the property they were "xxx Contract of Sale was not voidable but rescissible.
currently leasing should they decide to sell the same. Under Article 1380 to 1381 (3) of the Civil Code, a
The question now is whether a "right of first refusal" is Moreover, respondents presented a letter20 dated contract otherwise valid may nonetheless be subsequently
among those enumerated in the list of contracts covered October 9, 1990 where Eufrocina de Leon, the rescinded by reason of injury to third persons, like
by the Statute of Frauds. More specifically, is a right of representative of the heirs of the spouses Tiangco, creditors. The status of creditors could be validly accorded
first refusal akin to "an agreement for the leasing of a informed them that they had received an offer to buy the the Bonnevies for they had substantial interests that were
longer period than one year, or for the sale of real disputed property for P2,000,000.00 and offered to sell prejudiced by the sale of the subject property to the
property or of an interest therein" as contemplated by the same to the respondents at the same price if they petitioner without recognizing their right of first priority
Article 1403, par. 2(e) of the New Civil Code. were interested. Verily, if Eufrocina de Leon did not under the Contract of Lease.
recognize respondents’ right of first refusal over the
We have previously held that not all agreements "affecting property they were leasing, then she would not have According to Tolentino, rescission is a remedy granted by
land" must be put into writing to attain enforceability.14 bothered to offer the property for sale to the respondents. law to the contracting parties and even to third persons, to
Thus, we have held that the setting up of boundaries,15 secure reparations for damages caused to them by a
the oral partition of real property16, and an agreement It must be noted that petitioners did not present evidence contract, even if this should be valid, by means of the
creating a right of way17 are not covered by the before the trial court contradicting the existence of the restoration of things to their condition at the moment prior
provisions of the statute of frauds. The reason simply is right of first refusal of respondents over the disputed to the celebration of said contract. It is a relief allowed for
that these agreements are not among those enumerated property. They only presented petitioner Rene Joaquin, the protection of one of the contracting parties and even
in Article 1403 of the New Civil Code. the vice-president of petitioner Rosencor, who admitted third persons from all injury and damage the contract may
having no personal knowledge of the details of the sales cause, or to protect some incompatible and preferent right
A right of first refusal is not among those listed as transaction between Rosencor and the heirs of the created by the contract. Rescission implies a contract
unenforceable under the statute of frauds. Furthermore, spouses Tiangco21. They also dispensed with the which, even if initially valid, produces a lesion or pecuniary
the application of Article 1403, par. 2(e) of the New Civil testimony of Eufrocina de Leon22 who could have denied damage to someone that justifies its invalidation for
Code presupposes the existence of a perfected, albeit the existence or knowledge of the right of first refusal. As reasons of equity.
unwritten, contract of sale.18 A right of first refusal, such such, there being no evidence to the contrary, the right of
as the one involved in the instant case, is not by any first refusal claimed by respondents was substantially It is true that the acquisition by a third person of the
means a perfected contract of sale of real property. At proven by respondents before the lower court. property subject of the contract is an obstacle to the action
best, it is a contractual grant, not of the sale of the real for its rescission where it is shown that such third person
property involved, but of the right of first refusal over the Having ruled upon the question as to the existence of is in lawful possession of the subject of the contract and
property sought to be sold19. respondents’ right of first refusal, the next issue to be that he did not act in bad faith. However, this rule is not
62
Sales – Chapter 3-12 Cases
applicable in the case before us because the petitioner is premises. It is undisputed that Carmelo did recognize this protect some incompatible and preferred right by the
not considered a third party in relation to the Contract of right of Mayfair, for it informed the latter of its intention to contract. The sale of the subject real property should now
Sale nor may its possession of the subject property be sell the said property in 1974. There was an exchange of be rescinded considering that Mayfair, which had
regarded as acquired lawfully and in good faith. letters evidencing the offer and counter-offers made by substantial interest over the subject property, was
both parties. Carmelo, however, did not pursue the prejudiced by the sale of the subject property to Equatorial
Indeed, Guzman, Bocaling and Co. was the vendee in the exercise to its logical end. While it initially recognized without Carmelo conferring to Mayfair every opportunity to
Contract of Sale. Moreover, the petitioner cannot be Mayfair’s right of first refusal, Carmelo violated such right negotiate within the 30-day stipulate periond.27
deemed a purchaser in good faith for the record shows when without affording its negotiations with Mayfair the full
that it categorically admitted that it was aware of the lease process to ripen to at least an interface of a definite offer In Paranaque Kings Enterprises, Inc. vs. Court of
in favor of the Bonnevies, who were actually occupying and a possible corresponding acceptance within the "30- Appeals,28 the Court held that the allegations in a
the subject property at the time it was sold to it. Although day exclusive option" time granted Mayfair, Carmelo complaint showing violation of a contractual right of "first
the occupying the subject property at the time it was sold abandoned negotiations, kept a low profile for some time, option or priority to buy the properties subject of the lease"
to it. Although the Contract of Lease was not annotated on and then sold, without prior notice to Mayfair, the entire constitute a valid cause of action enforceable by an action
the transfer certificate of title in the name of the late Jose Claro M. Recto property to Equatorial. for specific performance. Summarizing the rulings in the
Reynoso and Africa Reynoso, the petitioner cannot deny two previously cited cases, the Court affirmed the nature
actual knowledge of such lease which was equivalent to Since Equatorial is a buyer in bad faith, this finding of and concomitant rights and obligations of parties under
and indeed more binding than presumed notice by renders the sale to it of the property in question, a right of first refusal. Thus:
registration. rescissible. We agree with respondent Appellate Court
that the records bear out the fact that Equatorial was "We hold however, that in order to have full compliance
A purchaser in good faith and for value is one who buys aware of the lease contracts because its lawyers had, with the contractual right granting petitioner the first option
the property of another without notice that some other prior to the sale, studied the said contracts. As such, to purchase, the sale of the properties for the amount of
person has a right to or interest in such property without Equatorial cannot tenably claim that to be a purchaser in P9,000,000.00, the price for which they were finally sold to
and pays a full and fair price for the same at the time of good faith, and, therefore, rescission lies. respondent Raymundo, should have likewise been offered
such purchase or before he has notice of the claim or to petitioner.
interest of some other person in the property. Good faith XX X
connotes an honest intention to abstain from taking The Court has made an extensive and lengthy discourse
unconscientious advantage of another. Tested by these As also earlier emphasized, the contract of sale between on the concept of, and obligations under, a right of first
principles, the petitioner cannot tenably claim to be a Equatorial and Carmelo is characterized by bad faith, refusal in the case of Guzman, Bocaling & Co. vs.
buyer in good faith as it had notice of the lease of the since it was knowingly entered into in violation of the Bonnevie. In that case, under a contract of lease, the
property by the Bonnevies and such knowledge should rights of and to the prejudice of Mayfair. In fact, as lessees (Raul and Christopher Bonnevie) were given a
have cautioned it to look deeper into the agreement to correctly observed by the Court of Appeals, Equatorial "right of first priority" to purchase the leased property in
determine if it involved stipulations that would prejudice its admitted that its lawyers had studied the contract or lease case the lessor (Reynoso) decided to sell. The selling
own interests." prior to the sale. Equatorial’s knowledge of the stipulations price quoted to the Bonnevies was 600,000.00 to be fully
therein should have cautioned it to look further into the paid in cash, less a mortgage lien of P100,000.00. On the
Subsequently24 in Equatorial Realty and Development, agreement to determine if it involved stipulations that other hand, the selling price offered by Reynoso to and
Inc. vs. Mayfair Theater, Inc.25, the Court, en banc, with would prejudice its own interests. accepted by Guzman was only P400,000.00 of which
three justices dissenting,26 ordered the rescission of a P137,500.00 was to be paid in cash while the balance
contract entered into in violation of a right of first refusal. Since Mayfair had a right of first refusal, it can exercise was to be paid only when the property was cleared of
Using the ruling in Guzman Bocaling & Co., Inc. vs. the right only if the fraudulent sale is first set aside or occupants. We held that even if the Bonnevies could not
Bonnevie as basis, the Court decreed that since rescinded. All of these matters are now before us and so buy it at the price quoted (P600,000.00), nonetheless,
respondent therein had a right of first refusal over the said there should be no piecemeal determination of this case Reynoso could not sell it to another for a lower price and
property, it could only exercise the said right if the and leave festering sores to deteriorate into endless under more favorable terms and conditions without first
fraudulent sale is first set aside or rescinded. Thus: litigation. The facts of the case and considerations of offering said favorable terms and price to the Bonnevies
justice and equity require that we order rescission here as well. Only if the Bonnevies failed to exercise their right
"What Carmelo and Mayfair agreed to, by executing the and now. Rescission is a relief allowed for the protection of first priority could Reynoso thereafter lawfully sell the
two lease contracts, was that Mayfair will have the right of of one of the contracting parties and even third persons subject property to others, and only under the same terms
first refusal in the event Carmelo sells the leased from all injury and damage the contract may cause or to and conditions previously offered to the Bonnevies.
63
Sales – Chapter 3-12 Cases
for the loan mortgage includes the consideration for the Litonjua, within the one-year period should have been
X X X right of first refusal. L&R Corporation is in effect stating accepted as valid by the L & R Corporation. However,
that it consents to lend out money to the spouses Litonjua while the sale is, indeed, valid, the same is rescissible
This principle was reiterated in the very recent case of provided that in case they decide to sell the property because it ignored L & R Corporation’s right of first
Equatorial Realty vs. Mayfair Theater, Inc. which was mortgaged to it, then L&R Corporation shall be given the refusal."
decided en banc. This Court upheld the right of first right to match the offered purchase price and to buy the
refusal of the lessee Mayfair, and rescinded the sale of property at that price. Thus, while the spouses Litonjua Thus, the prevailing doctrine, as enunciated in the cited
the property by the lessor Carmelo to Equatorial Realty had every right to sell their mortgaged property to PWHAS cases, is that a contract of sale entered into in violation of
"considering that Mayfair, which had substantial interest without securing the prior written consent of L&R a right of first refusal of another person, while valid, is
over the subject property, was prejudiced by its sale to Corporation, they had the obligation under paragraph 9, rescissible.
Equatorial without Carmelo conferring to Mayfair every which is a perfectly valid provision, to notify the latter of
opportunity to negotiate within the 30-day stipulated their intention to sell the property and give it priority over There is, however, a circumstance which prevents the
period" other buyers. It is only upon the failure of L&R Corporation application of this doctrine in the case at bench. In the
to exercise its right of first refusal could the spouses cases cited above, the Court ordered the rescission of
In that case, two contracts of lease between Carmelo and Litonjua validly sell the subject properties to the others, sales made in violation of a right of first refusal precisely
Mayfair provided "that if the LESSOR should desire to sell under the same terms and conditions offered to L&R because the vendees therein could not have acted in
the leased premises, the LESSEE shall be given 30 days Corporation. good faith as they were aware or should have been aware
exclusive option to purchase the same." Carmelo initially of the right of first refusal granted to another person by the
offered to sell the leased property to Mayfair for six to What then is the status of the sale made to PWHAS in vendors therein. The rationale for this is found in the
seven million pesos. Mayfair indicated interest in violation of L & R Corporation’s contractual right of first provisions of the New Civil Code on rescissible contracts.
purchasing the property though it invoked the 30-day refusal? On this score, we agree with the Amended Under Article 1381 of the New Civil Code, paragraph 3, a
period. Nothing was heard thereafter from Carmelo. Four Decision of the Court of Appeals that the sale made to contract validly agreed upon may be rescinded if it is
years later, the latter sold its entire Recto Avenue PWHAS is rescissible. The case of Guzman, Bocaling & "undertaken in fraud of creditors when the latter cannot in
property, including the leased premises, to Equatorial for Co. v. Bonnevie is instructive on this point. any manner collect the claim due them." Moreover, under
P11,300,000.00 without priorly informing Mayfair. The Article 1385, rescission shall not take place "when the
Court held that both Carmelo and Equatorial acted in bad X X X things which are the object of the contract are legally in
faith: Carmelo or knowingly violating the right of first the possession of third persons who did not act in bad
option of Mayfair, and Equatorial for purchasing the It was then held that the Contract of Sale there, which faith."30
property despite being aware of the contract stipulation. In violated the right of first refusal, was rescissible.
addition to rescission of the contract of sale, the Court It must be borne in mind that, unlike the cases cited
ordered Carmelo to allow Mayfair to buy the subject In the case at bar, PWHAS cannot claim ignorance of the above, the right of first refusal involved in the instant case
property at the same price of P11,300,000.00. right of first refusal granted to L & R Corporation over the was an oral one given to respondents by the deceased
subject properties since the Deed of Real Estate spouses Tiangco and subsequently recognized by their
In the recent case of Litonjua vs L&R Corporation,29 the Mortgage containing such a provision was duly registered heirs. As such, in order to hold that petitioners were in bad
Court, also citing the case of Guzman, Bocaling & Co. vs. with the Register of Deeds. As such, PWHAS is presumed faith, there must be clear and convincing proof that
Bonnevie, held that the sale made therein in violation of a to have been notified thereof by registration, which petitioners were made aware of the said right of first
right of first refusal embodied in a mortgage contract, was equates to notice to the whole world. refusal either by the respondents or by the heirs of the
rescissible. Thus: spouses Tiangco.
X X X
"While petitioners question the validity of paragraph 8 of It is axiomatic that good faith is always presumed unless
their mortgage contract, they appear to be silent insofar as All things considered, what then are the relative rights and contrary evidence is adduced.31 A purchaser in good faith
paragraph 9 thereof is concerned. Said paragraph 9 obligations of the parties? To recapitulate: the sale is one who buys the property of another without notice
grants upon L&R Corporation the right of first refusal over between the spouses Litonjua and PWHAS is valid, that some other person has a right or interest in such a
the mortgaged property in the event the mortgagor notwithstanding the absence of L & R Corporation’s prior property and pays a full and fair price at the time of the
decides to sell the same. We see nothing wrong in this written consent thereto. Inasmuch as the sale to PWHAS purchase or before he has notice of the claim or interest of
provision. The right of first refusal has long been was valid, its offer to redeem and its tender of the some other person in the property.32 In this regard, the
recognized as valid in our jurisdiction. The consideration redemption price, as successor-in-interest of the spouses rule on constructive notice would be inapplicable as it is
64
Sales – Chapter 3-12 Cases
undisputed that the right of first refusal was an oral one of petitioners and, as such, it only shows that Eufrocina de SET ASIDE. The Decision dated May 13, 1996 of the
and that the same was never reduced to writing, much Leon was aware of the existence of the oral right of first Quezon City Regional Trial Court, Branch 217 is hereby
less registered with the Registry of Deeds. In fact, even refusal. It does not show that petitioners were likewise REINSTATED insofar as it dismisses the action for
the lease contract by which respondents derive their right aware of the existence of the said right. Moreover, the rescission of the Deed of Absolute Sale dated September
to possess the property involved was an oral one. letter was made a month after the execution of the Deed 4, 1990 and orders the payment of monthly rentals of
of Absolute Sale on September 4, 1990 between P1,000.00 per month reckoned from May 1990 up to the
On this point, we hold that the evidence on record fails to petitioner Rosencor and the heirs of the spouses Tiangco. time respondents leave the premises.
show that petitioners acted in bad faith in entering into the There is no showing that prior to the date of the execution
deed of sale over the disputed property with the heirs of of the said Deed, petitioners were put on notice of the SO ORDERED.
the spouses Tiangco. Respondents failed to present any existence of the right of first refusal.
evidence that prior to the sale of the property on
September 4, 1990, petitioners were aware or had notice Clearly, if there was any indication of bad faith based on
of the oral right of first refusal. respondents’ evidence, it would only be on the part of
Eufrocina de Leon as she was aware of the right of first
Respondents point to the letter dated June 1, 199033 as refusal of respondents yet she still sold the disputed
indicative of petitioners’ knowledge of the said right. In this property to Rosencor. However, bad faith on the part of
letter, a certain Atty. Erlinda Aguila demanded that Eufrocina de Leon does not mean that petitioner
respondent Irene Guillermo vacate the structure they were Rosencor likewise acted in bad faith. There is no showing
occupying to make way for its demolition. that prior to the execution of the Deed of Absolute Sale,
petitioners were made aware or put on notice of the
We fail to see how the letter could give rise to bad faith on existence of the oral right of first refusal. Thus, absent
the part of the petitioner. No mention is made of the right clear and convincing evidence to the contrary, petitioner
of first refusal granted to respondents. The name of Rosencor will be presumed to have acted in good faith in
petitioner Rosencor or any of it officers did not appear on entering into the Deed of Absolute Sale over the disputed
the letter and the letter did not state that Atty. Aguila was property.
writing in behalf of petitioner. In fact, Atty. Aguila stated
during trial that she wrote the letter in behalf of the heirs of Considering that there is no showing of bad faith on the
the spouses Tiangco. Moreover, even assuming that Atty. part of the petitioners, the Court of Appeals thus erred in
Aguila was indeed writing in behalf of petitioner Rosencor, ordering the rescission of the Deed of Absolute Sale dated
there is no showing that Rosencor was aware at that time September 4, 1990 between petitioner Rosencor and the
that such a right of first refusal existed. heirs of the spouses Tiangco. The acquisition by
Rosencor of the property subject of the right of first refusal
Neither was there any showing that after receipt of this is an obstacle to the action for its rescission where, as in
June 1, 1990 letter, respondents notified Rosencor or Atty. this case, it was shown that Rosencor is in lawful
Aguila of their right of first refusal over the property. possession of the subject of the contract and that it did not
Respondents did not try to communicate with Atty. Aguila act in bad faith.34
and inform her about their preferential right over the
disputed property. There is even no showing that they This does not mean however that respondents are left
contacted the heirs of the spouses Tiangco after they without any remedy for the unjustified violation of their
received this letter to remind them of their right over the right of first refusal. Their remedy however is not an action
property. for the rescission of the Deed of Absolute Sale but an
action for damages against the heirs of the spouses
Respondents likewise point to the letter dated October 9, Tiangco for the unjustified disregard of their right of first
1990 of Eufrocina de Leon, where she recognized the refusal35.
right of first refusal of respondents, as indicative of the
bad faith of petitioners. We do not agree. Eufrocina de WHEREFORE, premises considered, the decision of the
Leon wrote the letter on her own behalf and not on behalf Court of Appeals dated June 25, 1999 is REVERSED and
65
Sales – Chapter 3-12 Cases
DR. DANIEL VAZQUEZ and MA. LUIZA M. VAZQUEZ, "5.7. The BUYER hereby commits that it will develop the
petitioners, 'Remaining Property' into a first class residential subdivision xxx
vs. of the same class as its New Alabang Subdivision, and that it
AYALA CORPORATION, respondent. intends to complete the first phase under its amended 6. Representation and Warranties by the SELLERS
development plan within three (3) years from the date of this
The rise in value of four lots in one of the country's prime Agreement. x x x" The SELLERS jointly and severally represent and warrant to
residential developments, Ayala Alabang Village in the BUYER that at the time of the execution of this
Muntinlupa City, over a period of six (6) years only, 5.15. The BUYER agrees to give the SELLERS a first option Agreement and at the Closing:
represents big money. The huge price difference lies at the to purchase four developed lots next to the "Retained Area"
heart of the present controversy. Petitioners insist that the at the prevailing market price at the time of the purchase." xxx
lots should be sold to them at 1984 prices while respondent
maintains that the prevailing market price in 1990 should be The parties are agreed that the development plan referred to 6.2.3. There are no actions, suits or proceedings pending, or
the selling price. in paragraph 5.7 is not Conduit's development plan, but to the knowledge of the SELLERS, threatened against or
Ayala's amended development plan which was still to be affecting the SELLERS with respect to the Shares or the
Dr. Daniel Vazquez and Ma. Luisa Vazquez1 filed this formulated as of the time of the MOA. While in the Conduit Property; and
Petition for Review on Certiorari2 dated October 11, 2001 plan, the 4 lots to be offered for sale to the Vasquez Spouses
assailing the Decision3 of the Court of Appeals dated were in the first phase thereof or Village 1, in the Ayala plan 7. Additional Warranties by the SELLERS
September 6, 2001 which reversed the Decision4 of the which was formulated a year later, it was in the third phase,
Regional Trial Court (RTC) and dismissed their complaint for or Phase II-c. 7.1. With respect to the Audited Financial Statements
specific performance and damages against Ayala required to be submitted at Closing in accordance with Par.
Corporation. Under the MOA, the Vasquez spouses made several express 3.1.5 above, the SELLER jointly and severally warrant to the
warranties, as follows: BUYER that:
Despite their disparate rulings, the RTC and the appellate
court agree on the following antecedents:5 "3.1. The SELLERS shall deliver to the BUYER: 7.1.1 The said Audited Financial Statements shall show that
on the day of Closing, the Company shall own the
On April 23, 1981, spouses Daniel Vasquez and Ma. Luisa M. xxx "Remaining Property", free from all liens and encumbrances
Vasquez (hereafter, Vasquez spouses) entered into a and that the Company shall have no obligation to any party
Memorandum of Agreement (MOA) with Ayala Corporation 3.1.2. The true and complete list, certified by the Secretary except for billings payable to GP Construction & Development
(hereafter, AYALA) with AYALA buying from the Vazquez and Treasurer of the Company showing: Corporation and advances made by Daniel Vazquez for
spouses, all of the latter's shares of stock in Conduit which BUYER shall be responsible in accordance with Par. 2
Development, Inc. (hereafter, Conduit). The main asset of xxx of this Agreement.
Conduit was a 49.9 hectare property in Ayala Alabang,
Muntinlupa, which was then being developed by Conduit D. A list of all persons and/or entities with whom the 7.1.2 Except to the extent reflected or reserved in the Audited
under a development plan where the land was divided into Company has pending contracts, if any. Financial Statements of the Company as of Closing, and
Villages 1, 2 and 3 of the "Don Vicente Village." The those disclosed to BUYER, the Company as of the date
development was then being undertaken for Conduit by G.P. xxx thereof, has no liabilities of any nature whether accrued,
Construction and Development Corp. (hereafter, GP absolute, contingent or otherwise, including, without
Construction). 3.1.5. Audited financial statements of the Company as at limitation, tax liabilities due or to become due and whether
Closing date. incurred in respect of or measured in respect of the
Under the MOA, Ayala was to develop the entire property, Company's income prior to Closing or arising out of
less what was defined as the "Retained Area" consisting of 4. Conditions Precedent transactions or state of facts existing prior thereto.
18,736 square meters. This "Retained Area" was to be
retained by the Vazquez spouses. The area to be developed All obligations of the BUYER under this Agreement are 7.2 SELLERS do not know or have no reasonable ground to
by Ayala was called the "Remaining Area". In this "Remaining subject to fulfillment prior to or at the Closing, of the following know of any basis for any assertion against the Company as
Area" were 4 lots adjacent to the "Retained Area" and Ayala conditions: at closing or any liability of any nature and in any amount not
agreed to offer these lots for sale to the Vazquez spouses at fully reflected or reserved against such Audited Financial
the prevailing price at the time of purchase. The relevant 4.1. The representations and warranties by the SELLERS Statements referred to above, and those disclosed to
provisions of the MOA on this point are: contained in this Agreement shall be true and correct at the BUYER.
time of Closing as though such representations and
warranties were made at such time; and xxx xxx xxx
66
Sales – Chapter 3-12 Cases
offered to be sold to the Vasquez spouses at the prevailing failed to disclose Lancer's claims. Hence, even conceding
7.6.3 Except as otherwise disclosed to the BUYER in writing price in 1990. This was rejected by the Vasquez spouses who that Ayala Corporation was obliged to develop and sell the
on or before the Closing, the Company is not engaged in or a wanted to pay at 1984 prices, thereby leading to the suit four (4) lots in question within three (3) years from the date of
party to, or to the best of the knowledge of the SELLERS, below. the MOA, the obligation was suspended during the pendency
threatened with, any legal action or other proceedings before of the case filed by Lancer.
any court or administrative body, nor do the SELLERS know After trial, the court a quo rendered its decision, the
or have reasonable grounds to know of any basis for any dispositive portion of which states: Interpreting the MOA's paragraph 5.7 above-quoted, the
such action or proceeding or of any governmental appellate court held that Ayala Corporation committed to
investigation relative to the Company. "THEREFORE, judgment is hereby rendered in favor of develop the first phase of its own amended development plan
plaintiffs and against defendant, ordering defendant to sell to and not Conduit's development plan. Nowhere does the MOA
7.6.4 To the knowledge of the SELLERS, no default or plaintiffs the relevant lots described in the Complaint in the provide that Ayala Corporation shall follow Conduit's
breach exists in the due performance and observance by the Ayala Alabang Village at the price of P460.00 per square development plan nor is Ayala Corporation prohibited from
Company of any term, covenant or condition of any meter amounting to P1,349,540.00; ordering defendant to changing the sequence of the phases of the property it will
instrument or agreement to which the company is a party or reimburse to plaintiffs attorney's fees in the sum of develop.
by which it is bound, and no condition exists which, with P200,000.00 and to pay the cost of the suit."
notice or lapse of time or both, will constitute such default or Anent the question of delay, the Court of Appeals ruled that
breach." In its decision, the court a quo concluded that the Vasquez there was no delay as petitioners never made a demand for
spouses were not obligated to disclose the potential claims of Ayala Corporation to sell the subject lots to them. According
After the execution of the MOA, Ayala caused the suspension GP Construction, Lancer and Del Rosario; Ayala's to the appellate court, what petitioners sent were mere
of work on Village 1 of the Don Vicente Project. Ayala then accountants should have opened the records of Conduit to reminder letters the last of which was dated prior to April 23,
received a letter from one Maximo Del Rosario of Lancer find out all claims; the warranty against suit is with respect to 1984 when the obligation was not yet demandable. At any
General Builder Corporation informing Ayala that he was "the shares of the Property" and the Lancer suit does not rate, the Court of Appeals found that petitioners in fact waived
claiming the amount of P1,509,558.80 as the subcontractor of affect the shares of stock sold to Ayala; Ayala was obligated the three (3)-year period when they sent a letter through their
G.P. Construction... to develop within 3 years; to say that Ayala was under no agent, Engr. Eduardo Turla, stating that they "expect that the
obligation to follow a time frame was to put the Vasquezes at development of Phase I will be completed by 19 February
G.P. Construction not being able to reach an amicable Ayala's mercy; Ayala did not develop because of a slump in 1990, three years from the settlement of the legal problems
settlement with Lancer, on March 22, 1982, Lancer sued G.P. the real estate market; the MOA was drafted and prepared by with the previous contractor."7
Construction, Conduit and Ayala in the then Court of First the AYALA who should suffer its ambiguities; the option to
Instance of Manila in Civil Case No. 82-8598. G.P. purchase the 4 lots is valid because it was supported by The appellate court likewise ruled that paragraph 5.15 above-
Construction in turn filed a cross-claim against Ayala. G.P. consideration as the option is incorporated in the MOA where quoted is not an option contract but a right of first refusal
Construction and Lancer both tried to enjoin Ayala from the parties had prestations to each other. [Emphasis there being no separate consideration therefor. Since
undertaking the development of the property. The suit was supplied] petitioners refused Ayala Corporation's offer to sell the
terminated only on February 19, 1987, when it was dismissed subject lots at the reduced 1990 price of P5,000.00 per
with prejudice after Ayala paid both Lancer and GP Ayala Corporation filed an appeal, alleging that the trial court square meter, they have effectively waived their right to buy
Construction the total of P4,686,113.39. erred in holding that petitioners did not breach their the same.
warranties under the MOA6 dated April 23, 1981; that it was
Taking the position that Ayala was obligated to sell the 4 lots obliged to develop the land where the four (4) lots subject of In the instant Petition, petitioners allege that the appellate
adjacent to the "Retained Area" within 3 years from the date the option to purchase are located within three (3) years from court erred in ruling that they violated their warranties under
of the MOA, the Vasquez spouses sent several "reminder" the date of the MOA; that it was in delay; and that the option the MOA; that Ayala Corporation was not obliged to develop
letters of the approaching so-called deadline. However, no to purchase was valid because it was incorporated in the the "Remaining Property" within three (3) years from the
demand after April 23, 1984, was ever made by the Vasquez MOA and the consideration therefor was the commitment by execution of the MOA; that Ayala was not in delay; and that
spouses for Ayala to sell the 4 lots. On the contrary, one of Ayala Corporation to petitioners embodied in the MOA. paragraph 5.15 of the MOA is a mere right of first refusal.
the letters signed by their authorized agent, Engr. Eduardo Additionally, petitioners insist that the Court should review the
Turla, categorically stated that they expected "development of As previously mentioned, the Court of Appeals reversed the factual findings of the Court of Appeals as they are in conflict
Phase 1 to be completed by February 19, 1990, three years RTC Decision. According to the appellate court, Ayala with those of the trial court.
from the settlement of the legal problems with the previous Corporation was never informed beforehand of the existence
contractor." of the Lancer claim. In fact, Ayala Corporation got a copy of Ayala Corporation filed a Comment on the Petition8 dated
the Lancer subcontract only on May 29, 1981 from G.P. March 26, 2002, contending that the petition raises questions
By early 1990 Ayala finished the development of the vicinity Construction's lawyers. The Court of Appeals thus held that of fact and seeks a review of evidence which is within the
of the 4 lots to be offered for sale. The four lots were then petitioners violated their warranties under the MOA when they domain of the Court of Appeals. Ayala Corporation maintains
67
Sales – Chapter 3-12 Cases
that the subcontract between GP Construction, with whom the third phase (Phase II-C) where the subject lots are
Conduit contracted for the development of the property under located within the same time frame because there is no Likewise, petitioners aver that although Ayala Corporation
a Construction Contract dated October 10, 1980, and Lancer contractual stipulation in the MOA therefor. It is free to decide may change the sequence of its development plan, it is
was not disclosed by petitioners during the negotiations. on its own the period for the development of Phase II-C. If obliged under the MOA to develop the entire area where the
Neither was the liability for Lancer's claim included in the petitioners wanted to impose the same three (3)-year subject lots are located in three (3) years.
Audited Financial Statements submitted by petitioners after timetable upon the third phase of the amended development
the signing of the MOA. These justify the conclusion that plan, they should have filed a suit to fix the time table in They also assert that demand was made on Ayala
petitioners breached their warranties under the afore-quoted accordance with Article 119710 of the Civil Code. Having Corporation to comply with their obligation under the MOA.
paragraphs of the MOA. Since the Lancer suit ended only in failed to do so, Ayala Corporation cannot be declared to have Apart from their reminder letters dated January 24, February
February 1989, the three (3)-year period within which Ayala been in delay. 18 and March 5, 1984, they also sent a letter dated March 4,
Corporation committed to develop the property should only be 1984 which they claim is a categorical demand for Ayala
counted thence. Thus, when it offered the subject lots to Ayala Corporation further contends that no demand was Corporation to comply with the provisions of the MOA.
petitioners in 1990, Ayala Corporation was not yet in delay. made on it for the performance of its alleged obligation. The
letter dated October 4, 1983 sent when petitioners were The parties were required to submit their respective
In response to petitioners' contention that there was no action already aware of the Lancer suit did not demand the delivery memoranda in the Resolution12 dated November 18, 2002.
or proceeding against them at the time of the execution of the of the subject lots by April 23, 1984. Instead, it requested In compliance with this directive, petitioners submitted their
MOA on April 23, 1981, Ayala Corporation avers that the Ayala Corporation to keep petitioners posted on the status of Memorandum13 dated February 14, 2003 on even date,
facts and circumstances which gave rise to the Lancer claim the case. Likewise, the letter dated March 4, 1984 was while Ayala Corporation filed its Memorandum14 dated
were already extant then. Petitioners warranted that their merely an inquiry as to the date when the development of February 14, 2003 on February 17, 2003.
representations under the MOA shall be true and correct at Phase 1 will be completed. More importantly, their letter
the time of "Closing" which shall take place within four (4) dated June 27, 1988 through Engr. Eduardo Turla expressed We shall first dispose of the procedural question raised by the
weeks from the signing of the MOA.9 Since the MOA was petitioners' expectation that Phase 1 will be completed by instant petition.
signed on April 23, 1981, "Closing" was approximately the February 19, 1990.
third week of May 1981. Hence, Lancer's claims, articulated It is well-settled that the jurisdiction of this Court in cases
in a letter which Ayala Corporation received on May 4, 1981, Lastly, Ayala Corporation maintains that paragraph 5.15 of brought to it from the Court of Appeals by way of petition for
are among the liabilities warranted against under paragraph the MOA is a right of first refusal and not an option contract. review under Rule 45 is limited to reviewing or revising errors
7.1.2 of the MOA. of law imputed to it, its findings of fact being conclusive on
Petitioners filed their Reply11 dated August 15, 2002 this Court as a matter of general principle. However, since in
Moreover, Ayala Corporation asserts that the warranties reiterating the arguments in their Petition and contending the instant case there is a conflict between the factual
under the MOA are not just against suits but against all kinds further that they did not violate their warranties under the findings of the trial court and the appellate court, particularly
of liabilities not reflected in the Audited Financial Statements. MOA because the case was filed by Lancer only on April 1, as regards the issues of breach of warranty, obligation to
It cannot be faulted for relying on the express warranty that 1982, eleven (11) months and eight (8) days after the signing develop and incurrence of delay, we have to consider the
except for billings payable to GP Construction and advances of the MOA on April 23, 1981. Ayala Corporation admitted evidence on record and resolve such factual issues as an
made by petitioner Daniel Vazquez in the amount of that it received Lancer's claim before the "Closing" date. It exception to the general rule.15 In any event, the submitted
P38,766.04, Conduit has no other liabilities. Hence, therefore had all the time to rescind the MOA. Not having issue relating to the categorization of the right to purchase
petitioners cannot claim that Ayala Corporation should have done so, it can be concluded that Ayala Corporation itself did granted to petitioners under the MOA is legal in character.
examined and investigated the Audited Financial Statements not consider the matter a violation of petitioners' warranty.
of Conduit and should now assume all its obligations and The next issue that presents itself is whether petitioners
liabilities including the Lancer suit and the cross-claim of GP Moreover, petitioners submitted the Audited Financial breached their warranties under the MOA when they failed to
Construction. Statements of Conduit and allowed an acquisition audit to be disclose the Lancer claim. The trial court declared they did
conducted by Ayala Corporation. Thus, the latter bought not; the appellate court found otherwise.
Furthermore, Ayala Corporation did not make a commitment Conduit with "open eyes."
to complete the development of the first phase of the property Ayala Corporation summarizes the clauses of the MOA which
within three (3) years from the execution of the MOA. The Petitioners also maintain that they had no knowledge of the petitioners allegedly breached when they failed to disclose
provision refers to a mere declaration of intent to develop the impending case against Conduit at the time of the execution the Lancer claim:
first phase of its (Ayala Corporation's) own development plan of the MOA. Further, the MOA makes Ayala Corporation
and not Conduit's. True to its intention, Ayala Corporation did liable for the payment of all billings of GP Construction. Since a) Clause 7.1.1. – that Conduit shall not be obligated to
complete the development of the first phase (Phase II-A) of Lancer's claim was actually a claim against GP Construction anyone except to GP Construction for P38,766.04, and for
its amended development plan within three (3) years from the being its sub-contractor, it is Ayala Corporation and not advances made by Daniel Vazquez;
execution of the MOA. However, it is not obliged to develop petitioners which is liable.
68
Sales – Chapter 3-12 Cases
b) Clause 7.1.2. – that except as reflected in the audited The full text of the pertinent clauses of the MOA quoted
financial statements Conduit had no other liabilities whether hereunder likewise indicate that certain matters pertaining to Hence, petitioners' warranty that Conduit is not engaged in, a
accrued, absolute, contingent or otherwise; the liabilities of Conduit were disclosed by petitioners to Ayala party to, or threatened with any legal action or proceeding is
Corporation although the specifics thereof were no longer qualified by Ayala Corporation's actual knowledge of the
c) Clause 7.2. – that there is no basis for any assertion included in the MOA: Lancer claim which was disclosed to Ayala Corporation
against Conduit of any liability of any value not reflected or before the "Closing."
reserved in the financial statements, and those disclosed to 7.1.1 The said Audited Financial Statements shall show that
Ayala; on the day of Closing, the Company shall own the At any rate, Ayala Corporation bound itself to pay all billings
"Remaining Property", free from all liens and encumbrances payable to GP Construction and the advances made by
d) Clause 7.6.3. – that Conduit is not threatened with any and that the Company shall have no obligation to any party petitioner Daniel Vazquez. Specifically, under paragraph 2 of
legal action or other proceedings; and except for billings payable to GP Construction & Development the MOA referred to in paragraph 7.1.1, Ayala Corporation
Corporation and advances made by Daniel Vazquez for undertook responsibility "for the payment of all billings of the
e) Clause 7.6.4. – that Conduit had not breached any term, which BUYER shall be responsible in accordance with contractor GP Construction & Development Corporation after
condition, or covenant of any instrument or agreement to Paragraph 2 of this Agreement. the first billing and any payments made by the company
which it is a party or by which it is bound.16 and/or SELLERS shall be reimbursed by BUYER on closing
7.1.2 Except to the extent reflected or reserved in the Audited which advances to date is P1,159,012.87."22
The Court is convinced that petitioners did not violate the Financial Statements of the Company as of Closing, and
foregoing warranties. those disclosed to BUYER, the Company as of the date The billings knowingly assumed by Ayala Corporation
hereof, has no liabilities of any nature whether accrued, necessarily include the Lancer claim for which GP
The exchanges of communication between the parties absolute, contingent or otherwise, including, without Construction is liable. Proof of this is Ayala Corporation's
indicate that petitioners substantially apprised Ayala limitation, tax liabilities due or to become due and whether letter23 to GP Construction dated before "Closing" on May 4,
Corporation of the Lancer claim or the possibility thereof incurred in respect of or measured in respect of the 1981, informing the latter of Ayala Corporation's receipt of the
during the period of negotiations for the sale of Conduit. Company's income prior to Closing or arising out of Lancer claim embodied in the letter dated April 30, 1981,
transactions or state of facts existing prior thereto. acknowledging that it is taking over the contractual
In a letter17 dated March 5, 1984, petitioner Daniel Vazquez responsibilities of Conduit, and requesting copies of all sub-
reminded Ayala Corporation's Mr. Adolfo Duarte (Mr. Duarte) 7.2 SELLERS do not know or have no reasonable ground to contracts affecting the Conduit property. The pertinent
that prior to the completion of the sale of Conduit, Ayala know of any basis for any assertion against the Company as excerpts of the letter read:
Corporation asked for and was given information that GP at Closing of any liability of any nature and in any amount not
Construction sub-contracted, presumably to Lancer, a greater fully reflected or reserved against such Audited Financial …
percentage of the project than it was allowed. Petitioners Statements referred to above, and those disclosed to
gave this information to Ayala Corporation because the latter BUYER. In this connection, we wish to inform you that this morning we
intimated a desire to "break the contract of Conduit with GP." received a letter from Mr. Maximo D. Del Rosario, President
Ayala Corporation did not deny this. In fact, Mr. Duarte's xxx xxx xxx of Lancer General Builders Corporation apprising us of the
letter18 dated March 6, 1984 indicates that Ayala Corporation existence of subcontracts that they have with your
had knowledge of the Lancer subcontract prior to its 7.6.3 Except as otherwise disclosed to the BUYER in writing corporation. They have also furnished us with a copy of their
acquisition of Conduit. Ayala Corporation even admitted that on or before the Closing, the Company is not engaged in or a letter to you dated 30 April 1981.
it "tried to explore…legal basis to discontinue the contract of party to, or to the best of the knowledge of the SELLERS,
Conduit with GP" but found this "not feasible when threatened with, any legal action or other proceedings before Since we are taking over the contractual responsibilities of
information surfaced about the tacit consent of Conduit to the any court or administrative body, nor do the SELLERS know Conduit Development, Inc., we believe that it is necessary, at
sub-contracts of GP with Lancer." or have reasonable grounds to know of any basis for any this point in time, that you furnish us with copies of all your
such action or proceeding or of any governmental subcontracts affecting the property of Conduit, not only with
At the latest, Ayala Corporation came to know of the Lancer investigation relative to the Company. Lancer General Builders Corporation, but all subcontracts
claim before the date of Closing of the MOA. Lancer's letter19 with other parties as well…24
dated April 30, 1981 informing Ayala Corporation of its 7.6.4 To the knowledge of the SELLERS, no default or
unsettled claim with GP Construction was received by Ayala breach exists in the due performance and observance by the Quite tellingly, Ayala Corporation even attached to its Pre-
Corporation on May 4, 1981, well before the "Closing"20 Company of any term, covenant or condition of any Trial Brief25 dated July 9, 1992 a copy of the letter26 dated
which occurred four (4) weeks after the date of signing of the instrument or agreement to which the Company is a party or May 28, 1981 of GP Construction's counsel addressed to
MOA on April 23, 1981, or on May 23, 1981. by which it is bound, and no condition exists which, with Conduit furnishing the latter with copies of all sub-contract
notice or lapse of time or both, will constitute such default or agreements entered into by GP Construction. Since it was
breach."21 [Emphasis supplied] addressed to Conduit, it can be presumed that it was the
69
Sales – Chapter 3-12 Cases
latter which gave Ayala Corporation a copy of the letter Don't talk about standard.
thereby disclosing to the latter the existence of the Lancer Q: This Exhibit "D-5" was the plan that was being followed by
sub-contract. WITNESS GP Construction in 1981?

The ineluctable conclusion is that petitioners did not violate A Well, the word intent here, your Honor, was used to A: Yes, sir.
their warranties under the MOA. The Lancer sub-contract and emphasize the tentative character of the period of
claim were substantially disclosed to Ayala Corporation development because it will be noted that the sentence refers Q: And point of fact during your direct examination as of the
before the "Closing" date of the MOA. Ayala Corporation to and I quote "to complete the first phase under its amended date of the agreement, this amended development plan was
cannot disavow knowledge of the claim. development plan within three (3) years from the date of this still to be formulated by Ayala?
agreement, at the time of the execution of this agreement,
Moreover, while in its correspondence with petitioners, Ayala your Honor." That amended development plan was not yet in A: Yes, sir.32
Corporation did mention the filing of the Lancer suit as an existence because the buyer had manifested to the seller that
obstacle to its development of the property, it never actually the buyer could amend the subdivision plan originally As correctly held by the appellate court, this admission is
brought up nor sought redress for petitioners' alleged breach belonging to the seller to conform with its own standard of crucial because while the subject lots to be sold to petitioners
of warranty for failure to disclose the Lancer claim until it filed development and second, your Honor, (interrupted)31 were in the first phase of the Conduit development plan, they
its Answer27 dated February 17, 1992. were in the third or last phase of the Ayala Corporation
It is thus unmistakable that this paragraph merely expresses development plan. Hence, even assuming that paragraph 5.7
We now come to the correct interpretation of paragraph 5.7 of an intention on Ayala Corporation's part to complete the first expresses a commitment on the part of Ayala Corporation to
the MOA. Does this paragraph express a commitment or a phase under its amended development plan within three (3) develop the first phase of its amended development plan
mere intent on the part of Ayala Corporation to develop the years from the execution of the MOA. Indeed, this paragraph within three (3) years from the execution of the MOA, there
property within three (3) years from date thereof? Paragraph is so plainly worded that to misunderstand its import is was no parallel commitment made as to the timeframe for the
5.7 provides: deplorable. development of the third phase where the subject lots are
located.
5.7. The BUYER hereby commits that it will develop the More focal to the resolution of the instant case is paragraph
'Remaining Property' into a first class residential subdivision 5.7's clear reference to the first phase of Ayala Corporation's Lest it be forgotten, the point of this petition is the alleged
of the same class as its New Alabang Subdivision, and that it amended development plan as the subject of the three (3)- failure of Ayala Corporation to offer the subject lots for sale to
intends to complete the first phase under its amended year intended timeframe for development. Even petitioner petitioners within three (3) years from the execution of the
development plan within three (3) years from the date of this Daniel Vazquez admitted on cross-examination that the MOA. It is not that Ayala Corporation committed or intended
Agreement….28 paragraph refers not to Conduit's but to Ayala Corporation's to develop the first phase of its amended development plan
development plan which was yet to be formulated when the within three (3) years. Whether it did or did not is actually
Notably, while the first phrase of the paragraph uses the word MOA was executed: beside the point since the subject lots are not located in the
"commits" in reference to the development of the "Remaining first phase anyway.
Property" into a first class residential subdivision, the second Q: Now, turning to Section 5.7 of this Memorandum of
phrase uses the word "intends" in relation to the development Agreement, it is stated as follows: "The Buyer hereby We now come to the issue of default or delay in the fulfillment
of the first phase of the property within three (3) years from commits that to develop the remaining property into a first of the obligation.
the date of the MOA. The variance in wording is significant. class residential subdivision of the same class as New
While "commit"29 connotes a pledge to do something, Alabang Subdivision, and that they intend to complete the Article 1169 of the Civil Code provides:
"intend"30 merely signifies a design or proposition. first phase under its amended development plan within three
years from the date of this agreement." Art. 1169. Those obliged to deliver or to do something incur in
Atty. Leopoldo Francisco, former Vice President of Ayala delay from the time the obligee judicially or extrajudicially
Corporation's legal division who assisted in drafting the MOA, Now, my question to you, Dr. Vasquez is that there is no demands from them the fulfillment of their obligation.
testified: dispute that the amended development plan here is the
amended development plan of Ayala? However, the demand by the creditor shall not be necessary
COURT in order that delay may exist:
A: Yes, sir.
You only ask what do you mean by that intent. Just answer (1) When the obligation or the law expressly so declares; or
on that point. Q: In other words, it is not Exhibit "D-5" which is the original
plan of Conduit? (2) When from the nature and the circumstances of the
ATTY. BLANCO obligation it appears that the designation of the time when the
A: No, it is not.
70
Sales – Chapter 3-12 Cases
thing is to be delivered or the service is to be rendered was a three (3)-year period should be counted from the termination within the three (3)-year period much less to offer the subject
controlling motive for the establishment of the contract; or of the case filed by Lancer. The letter reads in part: lots for sale to petitioners. The letter dated January 23, 1984
reads in part:
(3) When demand would be useless, as when the obligor has I. Completion of Phase I
rendered it beyond his power to perform. You will understand our interest in the completion of the
As per the memorandum of Agreement also dated April 23, roads to our property, since we cannot develop it till you have
In reciprocal obligations, neither party incurs in delay if the 1981, it was undertaken by your goodselves to complete the constructed the same. Allow us to remind you of our
other does not comply or is not ready to comply in a proper development of Phase I within three (3) years. Dr. & Mrs. Memorandum of Agreement, as per which you committed to
manner with what is incumbent upon him. From the moment Vazquez were made to understand that you were unable to develop the roads to our property "as per the original plans of
one of the parties fulfills his obligation, delay by the other accomplish this because of legal problems with the previous the company", and that
begins. contractor. These legal problems were resolved as of
February 19, 1987, and Dr. & Mrs. Vazquez therefore expect 1. The back portion should have been developed before the
In order that the debtor may be in default it is necessary that that the development of Phase I will be completed by front portion – which has not been the case.
the following requisites be present: (1) that the obligation be February 19, 1990, three years from the settlement of the
demandable and already liquidated; (2) that the debtor delays legal problems with the previous contractor. The reason for 2. The whole project – front and back portions be completed
performance; and (3) that the creditor requires the this is, as you know, that security-wise, Dr. & Mrs. Vazquez by 1984.38
performance judicially or extrajudicially.33 have been advised not to construct their residence till the
surrounding area (which is Phase I) is developed and The letter dated February 18, 1984 is similarly worded. It
Under Article 1193 of the Civil Code, obligations for whose occupied. They have been anxious to build their residence for states:
fulfillment a day certain has been fixed shall be demandable quite some time now, and would like to receive assurance
only when that day comes. However, no such day certain was from your goodselves regarding this, in compliance with the In this regard, we would like to remind you of Articles 5.7 and
fixed in the MOA. Petitioners, therefore, cannot demand agreement. 5.9 of our Memorandum of Agreement which states
performance after the three (3) year period fixed by the MOA respectively:…39
for the development of the first phase of the property since II. Option on the adjoining lots
this is not the same period contemplated for the development Even petitioner Daniel Vazquez' letter40 dated March 5, 1984
of the subject lots. Since the MOA does not specify a period We have already written your goodselves regarding the does not make out a categorical demand for Ayala
for the development of the subject lots, petitioners should intention of Dr. & Mrs. Vazquez to exercise their option to Corporation to offer the subject lots for sale on or before April
have petitioned the court to fix the period in accordance with purchase the two lots on each side (a total of 4 lots) adjacent 23, 1984. The letter reads in part:
Article 119734 of the Civil Code. As no such action was filed to their "Retained Area". They are concerned that although
by petitioners, their complaint for specific performance was over a year has elapsed since the settlement of the legal …and that we expect from your goodselves compliance with
premature, the obligation not being demandable at that point. problems, you have not presented them with the size, our Memorandum of Agreement, and a definite date as to
Accordingly, Ayala Corporation cannot likewise be said to configuration, etc. of these lots. They would appreciate being when the road to our property and the development of Phase
have delayed performance of the obligation. provided with these at your earliest convenience.35 I will be completed.41

Even assuming that the MOA imposes an obligation on Ayala Manifestly, this letter expresses not only petitioners' At best, petitioners' letters can only be construed as mere
Corporation to develop the subject lots within three (3) years acknowledgement that the delay in the development of Phase reminders which cannot be considered demands for
from date thereof, Ayala Corporation could still not be held to I was due to the legal problems with GP Construction, but performance because it must appear that the tolerance or
have been in delay since no demand was made by petitioners also their acquiescence to the completion of the development benevolence of the creditor must have ended.42
for the performance of its obligation. of Phase I at the much later date of February 19, 1990. More
importantly, by no stretch of semantic interpretation can it be The petition finally asks us to determine whether paragraph
As found by the appellate court, petitioners' letters which construed as a categorical demand on Ayala Corporation to 5.15 of the MOA can properly be construed as an option
dealt with the three (3)-year timetable were all dated prior to offer the subject lots for sale to petitioners as the letter merely contract or a right of first refusal. Paragraph 5.15 states:
April 23, 1984, the date when the period was supposed to articulates petitioners' desire to exercise their option to
expire. In other words, the letters were sent before the purchase the subject lots and concern over the fact that they 5.15 The BUYER agrees to give the SELLERS first option to
obligation could become legally demandable. Moreover, the have not been provided with the specifications of these lots. purchase four developed lots next to the "Retained Area" at
letters were mere reminders and not categorical demands to the prevailing market price at the time of the purchase.43
perform. More importantly, petitioners waived the three (3)- The letters of petitioners' children, Juan Miguel and Victoria
year period as evidenced by their agent, Engr. Eduardo Vazquez, dated January 23, 198436 and February 18, The Court has clearly distinguished between an option
Turla's letter to the effect that petitioners agreed that the 198437 can also not be considered categorical demands on contract and a right of first refusal. An option is a preparatory
Ayala Corporation to develop the first phase of the property contract in which one party grants to another, for a fixed
71
Sales – Chapter 3-12 Cases
period and at a determined price, the privilege to buy or sell, An accepted unilateral promise to buy or to sell a determinate
or to decide whether or not to enter into a principal contract. It thing for a price certain is binding upon the promissor if the
binds the party who has given the option not to enter into the promise is supported by a consideration distinct from the
principal contract with any other person during the period price.
designated, and within that period, to enter into such contract
with the one to whom the option was granted, if the latter Consequently, the "offer" may be withdrawn anytime by
should decide to use the option. It is a separate and distinct communicating the withdrawal to the other party.47
contract from that which the parties may enter into upon the
consummation of the option. It must be supported by In this case, Ayala Corporation offered the subject lots for
consideration.44 sale to petitioners at the price of P6,500.00/square meter, the
prevailing market price for the property when the offer was
In a right of first refusal, on the other hand, while the object made on June 18, 1990.48 Insisting on paying for the lots at
might be made determinate, the exercise of the right would the prevailing market price in 1984 of P460.00/square meter,
be dependent not only on the grantor's eventual intention to petitioners rejected the offer. Ayala Corporation reduced the
enter into a binding juridical relation with another but also on price to P5,000.00/square meter but again, petitioners
terms, including the price, that are yet to be firmed up.45 rejected the offer and instead made a counter-offer in the
amount of P2,000.00/square meter.49 Ayala Corporation
Applied to the instant case, paragraph 5.15 is obviously a rejected petitioners' counter-offer. With this rejection,
mere right of first refusal and not an option contract. Although petitioners lost their right to purchase the subject lots.
the paragraph has a definite object, i.e., the sale of subject
lots, the period within which they will be offered for sale to It cannot, therefore, be said that Ayala Corporation breached
petitioners and, necessarily, the price for which the subject petitioners' right of first refusal and should be compelled by
lots will be sold are not specified. The phrase "at the an action for specific performance to sell the subject lots to
prevailing market price at the time of the purchase" connotes petitioners at the prevailing market price in 1984.
that there is no definite period within which Ayala Corporation
is bound to reserve the subject lots for petitioners to exercise WHEREFORE, the instant petition is DENIED. No
their privilege to purchase. Neither is there a fixed or pronouncement as to costs.
determinable price at which the subject lots will be offered for
sale. The price is considered certain if it may be determined SO ORDERED.
with reference to another thing certain or if the determination
thereof is left to the judgment of a specified person or
persons.46

Further, paragraph 5.15 was inserted into the MOA to give


petitioners the first crack to buy the subject lots at the price
which Ayala Corporation would be willing to accept when it
offers the subject lots for sale. It is not supported by an
independent consideration. As such it is not governed by
Articles 1324 and 1479 of the Civil Code, viz:

Art. 1324. When the offeror has allowed the offeree a certain
period to accept, the offer may be withdrawn at any time
before acceptance by communicating such withdrawal,
except when the option is founded upon a consideration, as
something paid or promised.

Art. 1479. A promise to buy and sell a determinate thing for a


price certain is reciprocally demandable.

72
Sales – Chapter 3-12 Cases
TANAY RECREATION CENTER AND DEVELOPMENT August 1, 1991 at a monthly rental of P10,000.00, and ownership and is deemed to have waived its right to
CORP., petitioner, vs. CATALINA MATIENZO dismissed petitioners claim for damages.[9] purchase the property.[13] The CA even further went on to
FAUSTO+ and ANUNCIACION FAUSTO PACUNAYEN, rule that even if the sale is annulled, petitioner could not
respondents. On appeal, docketed as CA-G.R. CV No. 43770, the Court achieve anything because the property will be eventually
of Appeals (CA) affirmed with modifications the trial courts transferred to Pacunayen after Faustos death.[14]
Petitioner Tanay Recreation Center and Development judgment per its Decision dated June 14, 1999.[10] The
Corp. (TRCDC) is the lessee of a 3,090-square meter dispositive portion of the decision reads: Petitioner filed a motion for reconsideration but it was
property located in Sitio Gayas, Tanay, Rizal, owned by denied per Resolution dated September 14, 1999.[15]
Catalina Matienzo Fausto,[1] under a Contract of Lease WHEREFORE, the appealed decision is AFFIRMED AND
executed on August 1, 1971. On this property stands the ACCORDINGLY MODIFIED AS DISCUSSED. Dissatisfied, petitioner elevated the case to this Court on
Tanay Coliseum Cockpit operated by petitioner. The lease petition for review on certiorari, raising the following
contract provided for a 20-year term, subject to renewal Furthermore, we resolved: grounds:
within sixty days prior to its expiration. The contract also
provided that should Fausto decide to sell the property, 1.0. That TRCDC VACATE the leased premises THE HONORABLE COURT OF APPEALS COMMITTED
petitioner shall have the priority right to purchase the immediately; SERIOUS REVERSIBLE ERROR IN HOLDING THAT
same.[2] THE CONTRACTUAL STIPULATION GIVING
2.0. To GRANT the motion of Pacunayen to allow her to PETITIONER THE PRIORITY RIGHT TO PURCHASE
On June 17, 1991, petitioner wrote Fausto informing her withdraw the amount of P320,000.00, deposited according THE LEASED PREMISES SHALL ONLY APPLY IF THE
of its intention to renew the lease.[3] However, it was to records, with this court. LESSOR DECIDES TO SELL THE SAME TO
Faustos daughter, respondent Anunciacion F. Pacunayen, STRANGERS;
who replied, asking that petitioner remove the 3.0. To order TRCDC to MAKE THE NECESSARY
improvements built thereon, as she is now the absolute ACCOUNTING regarding the amounts it had already THE HONORABLE COURT OF APPEALS COMMITTED
owner of the property.[4] It appears that Fausto had earlier deposited (for unpaid rentals for the extended period of SERIOUS REVERSIBLE ERROR IN HOLDING THAT
sold the property to Pacunayen on August 8, 1990, for the seven [7] years of the contract of lease). In case it had not PETITIONERS PRIORITY RIGHT TO PURCHASE THE
sum of P10,000.00 under a Kasulatan ng Bilihan yet completed its deposit, to immediately pay the LEASED PREMISES IS INCONSEQUENTIAL.[16]
Patuluyan ng Lupa,[5] and title has already been remaining balance to Pacunayen.
transferred in her name under Transfer Certificate of Title The principal bone of contention in this case refers to
(TCT) No. M-35468.[6] 4.0. To order TRCDC to PAY the amount of P10,000.00 petitioners priority right to purchase, also referred to as
as monthly rental, with regard to its continued stay in the the right of first refusal.
Despite efforts, the matter was not resolved. Hence, on leased premises even after the expiration of the extended
September 4, 1991, petitioner filed an Amended period of seven (7) years, computed from August 1, 1998, Petitioners right of first refusal in this case is expressly
Complaint for Annulment of Deed of Sale, Specific until it finally vacates therefrom. provided for in the notarized Contract of Lease dated
Performance with Damages, and Injunction, docketed as August 1, 1971, between Fausto and petitioner, to wit:
Civil Case No. 372-M.[7] SO ORDERED.[11]
7. That should the LESSOR decide to sell the leased
In her Answer, respondent claimed that petitioner is In arriving at the assailed decision, the CA acknowledged premises, the LESSEE shall have the priority right to
estopped from assailing the validity of the deed of sale as the priority right of TRCDC to purchase the property in purchase the same;[17]
the latter acknowledged her ownership when it merely question. However, the CA interpreted such right to mean
asked for a renewal of the lease. According to respondent, that it shall be applicable only in case the property is sold When a lease contract contains a right of first refusal, the
when they met to discuss the matter, petitioner did not to strangers and not to Faustos relative. The CA stated lessor is under a legal duty to the lessee not to sell to
demand for the exercise of its option to purchase the that (T)o interpret it otherwise as to comprehend all sales anybody at any price until after he has made an offer to
property, and it even asked for grace period to vacate the including those made to relatives and to the compulsory sell to the latter at a certain price and the lessee has failed
premises.[8] heirs of the seller at that would be an absurdity, and her to accept it. The lessee has a right that the lessor's first
(Faustos) only motive for such transfer was precisely one offer shall be in his favor.[18] Petitioners right of first
After trial on the merits, the Regional Trial Court of of preserving the property within her bloodline and that refusal is an integral and indivisible part of the contract of
Morong, Rizal (Branch 78), rendered judgment extending someone administer the property.[12] The CA also ruled lease and is inseparable from the whole contract. The
the period of the lease for another seven years from that petitioner already acknowledged the transfer of consideration for the lease includes the consideration for
73
Sales – Chapter 3-12 Cases
the right of first refusal[19] and is built into the reciprocal faith, and which is in violation of a right of first refusal a contract of sale entered into in violation of a right of first
obligations of the parties. granted to the lessee is not voidable under the Statute of refusal of another person, while valid, is rescissible.[24]
Frauds but rescissible under Articles 1380 to 1381 (3) of
It was erroneous for the CA to rule that the right of first the New Civil Code. It was also incorrect for the CA to rule that it would be
refusal does not apply when the property is sold to useless to annul the sale between Fausto and respondent
Faustos relative.[20] When the terms of an agreement Subsequently in 1994, in the case of Ang Yu Asuncion v. because the property would still remain with respondent
have been reduced to writing, it is considered as Court of Appeals, the Court en banc departed from the after the death of her mother by virtue of succession, as in
containing all the terms agreed upon. As such, there can doctrine laid down in Guzman, Bocaling & Co. v. Bonnevie fact, Fausto died in March 1996, and the property now
be, between the parties and their successors in interest, and refused to rescind a contract of sale which violated belongs to respondent, being Faustos heir.[25]
no evidence of such terms other than the contents of the the right of first refusal. The Court held that the so-called
written agreement, except when it fails to express the true right of first refusal cannot be deemed a perfected For one, Fausto was bound by the terms and conditions of
intent and agreement of the parties.[21] In this case, the contract of sale under Article 1458 of the New Civil Code the lease contract. Under the right of first refusal clause,
wording of the stipulation giving petitioner the right of first and, as such, a breach thereof decreed under a final she was obligated to offer the property first to petitioner
refusal is plain and unambiguous, and leaves no room for judgment does not entitle the aggrieved party to a writ of before selling it to anybody else. When she sold the
interpretation. It simply means that should Fausto decide execution of the judgment but to an action for damages in property to respondent without offering it to petitioner, the
to sell the leased property during the term of the lease, a proper forum for the purpose. sale while valid is rescissible so that petitioner may
such sale should first be offered to petitioner. The exercise its option under the contract.
stipulation does not provide for the qualification that such In the 1996 case of Equatorial Realty Development, Inc. v.
right may be exercised only when the sale is made to Mayfair Theater, Inc., the Court en banc reverted back to With the death of Fausto, whatever rights and obligations
strangers or persons other than Faustos kin. Thus, under the doctrine in Guzman Bocaling & Co. v. Bonnevie she had over the property, including her obligation under
the terms of petitioners right of first refusal, Fausto has the stating that rescission is a relief allowed for the protection the lease contract, were transmitted to her heirs by way of
legal duty to petitioner not to sell the property to anybody, of one of the contracting parties and even third persons succession, a mode of acquiring the property, rights and
even her relatives, at any price until after she has made from all injury and damage the contract may cause or to obligation of the decedent to the extent of the value of the
an offer to sell to petitioner at a certain price and said offer protect some incompatible and preferred right by the inheritance of the heirs. Article 1311 of the Civil Code
was rejected by petitioner. Pursuant to their contract, it contract. provides:
was essential that Fausto should have first offered the
property to petitioner before she sold it to respondent. It Thereafter in 1997, in Paraaque Kings Enterprises, Inc. v. ART. 1311. Contracts take effect only between the
was only after petitioner failed to exercise its right of first Court of Appeals, the Court affirmed the nature of and the parties, their assigns and heirs, except in case where the
priority could Fausto then lawfully sell the property to concomitant rights and obligations of parties under a right rights and obligations arising from the contract are not
respondent. of first refusal. The Court, summarizing the rulings in transmissible by their nature, or by stipulation or by
Guzman, Bocaling & Co. v. Bonnevie and Equatorial provision of law. The heir is not liable beyond the value of
The rule is that a sale made in violation of a right of first Realty Development, Inc. v. Mayfair Theater, Inc., held the property he received from the decedent.
refusal is valid. However, it may be rescinded, or, as in that in order to have full compliance with the contractual
this case, may be the subject of an action for specific right granting petitioner the first option to purchase, the A lease contract is not essentially personal in
performance.[22] In Riviera Filipina, Inc. vs. Court of sale of the properties for the price for which they were character.[26] Thus, the rights and obligations therein are
Appeals,[23] the Court discussed the concept and finally sold to a third person should have likewise been transmissible to the heirs. The general rule is that heirs
interpretation of the right of first refusal and the first offered to the former. Further, there should be identity are bound by contracts entered into by their
consequences of a breach thereof, to wit: of terms and conditions to be offered to the buyer holding predecessors-in-interest except when the rights and
a right of first refusal if such right is not to be rendered obligations arising therefrom are not transmissible by (1)
. . . It all started in 1992 with Guzman, Bocaling & Co. v. illusory. Lastly, the basis of the right of first refusal must their nature, (2) stipulation or (3) provision of law.[27]
Bonnevie where the Court held that a lease with a proviso be the current offer to sell of the seller or offer to purchase
granting the lessee the right of first priority all things and of any prospective buyer. In this case, the nature of the rights and obligations are,
conditions being equal meant that there should be identity by their nature, transmissible. There is also neither
of the terms and conditions to be offered to the lessee and The prevailing doctrine therefore, is that a right of first contractual stipulation nor provision of law that makes the
all other prospective buyers, with the lessee to enjoy the refusal means identity of terms and conditions to be rights and obligations under the lease contract
right of first priority. A deed of sale executed in favor of a offered to the lessee and all other prospective buyers and intransmissible. The lease contract between petitioner and
third party who cannot be deemed a purchaser in good Fausto is a property right, which is a right that passed on
74
Sales – Chapter 3-12 Cases
to respondent and the other heirs, if any, upon the death There is no personal act required from Fausto such that the other party; and (3) knowledge, actual or constructive,
of Fausto. respondent cannot perform it. Faustos obligation to deliver of the real facts.[33]
possession of the property to petitioner upon the exercise
In DKC Holdings Corporation vs. Court of Appeals,[28] the by the latter of its right of first refusal may be performed by The records are bereft of any proposition that petitioner
Court held that the Contract of Lease with Option to Buy respondent and the other heirs, if any. Similarly, waived its right of first refusal under the contract such that
entered into by the late Encarnacion Bartolome with DKC nonperformance is not excused by the death of the party it is now estopped from exercising the same. In a letter
Holdings Corporation was binding upon her sole heir, when the other party has a property interest in the subject dated June 17, 1991, petitioner wrote to Fausto asking for
Victor, even after her demise and it subsists even after her matter of the contract.[31] a renewal of the term of lease.[34] Petitioner cannot be
death. The Court ruled that: faulted for merely seeking a renewal of the lease contract
The CA likewise found that petitioner acknowledged the because obviously, it was working on the assumption that
. . . Indeed, being an heir of Encarnacion, there is privity of legitimacy of the sale to respondent and it is now barred title to the property is still in Faustos name and the latter
interest between him and his deceased mother. He only from exercising its right of first refusal. According to the has the sole authority to decide on the fate of the
succeeds to what rights his mother had and what is valid appellate court: property. Instead, it was respondent who replied, advising
and binding against her is also valid and binding as petitioner to remove all the improvements on the property,
against him. This is clear from Paraaque Kings Second, when TRCDC, in a letter to Fausto, signified its as the lease is to expire on the 1st of August 1991.
Enterprises vs. Court of Appeals, where this Court intention to renew the lease contract, it was Pacunayen Respondent also informed petitioner that her mother has
rejected a similar defense- who answered the letter on June 19, 1991. In that letter already sold the property to her.[35] In order to resolve the
Pacunayen demanded that TRCDC vacate the leased matter, a meeting was called among petitioners
With respect to the contention of respondent Raymundo premises within sixty (60) days and informed it of her stockholders, including respondent, on July 27, 1991,
that he is not privy to the lease contract, not being the ownership of the leased premises. The pertinent portion of where petitioner, again, proposed that the lease be
lessor nor the lessee referred to therein, he could thus not the letter reads: renewed. Respondent, however, declined. While petitioner
have violated its provisions, but he is nevertheless a may have sought the renewal of the lease, it cannot be
proper party. Clearly, he stepped into the shoes of the Furtherly, please be advised that the land is no longer construed as a relinquishment of its right of first refusal.
owner-lessor of the land as, by virtue of his purchase, he under the absolute ownership of my mother and the Estoppel must be intentional and unequivocal.[36]
assumed all the obligations of the lessor under the lease undersigned is now the real and absolute owner of the
contract. Moreover, he received benefits in the form of land. Also, in the excerpts from the minutes of the special
rental payments. Furthermore, the complaint, as well as meeting, it was further stated that the possibility of a sale
the petition, prayed for the annulment of the sale of the Instead of raising a howl over the contents of the letter, as was likewise considered.[37] But respondent also refused
properties to him. Both pleadings also alleged collusion would be its expected and natural reaction under the to sell the land, while the improvements, if for sale shall be
between him and respondent Santos which defeated the circumstances, TRCDC surprisingly kept silent about the subject for appraisal.[38] After respondent refused to sell
exercise by petitioner of its right of first refusal. whole thing. As we mentioned in the factual antecedents the land, it was then that petitioner filed the complaint for
of this case, it even invited Pacunayen to its special board annulment of sale, specific performance and
In order then to accord complete relief to petitioner, meeting particularly to discuss with her the renewal of the damages.[39] Petitioners acts of seeking all possible
respondent Raymundo was a necessary, if not lease contract. Again, during that meeting, TRCDC did not avenues for the amenable resolution of the conflict do not
indispensable, party to the case. A favorable judgment for mention anything that could be construed as challenging amount to an intentional and unequivocal abandonment of
the petitioner will necessarily affect the rights of Pacunayens ownership of the leased premises. Neither its right of first refusal.
respondent Raymundo as the buyer of the property over did TRCDC assert its priority right to purchase the same
which petitioner would like to assert its right of first option against Pacunayen.[32] Respondent was well aware of petitioners right to priority
to buy.[29] (Emphasis supplied) of sale, and that the sale made to her by her mother was
The essential elements of estoppel are: (1) conduct of a merely for her to be able to take charge of the latters
Likewise in this case, the contract of lease, with all its party amounting to false representation or concealment of affairs. As admitted by respondent in her Appellees Brief
concomitant provisions, continues even after Faustos material facts or at least calculated to convey the filed before the CA, viz.:
death and her heirs merely stepped into her shoes.[30] impression that the facts are otherwise than, and
Respondent, as an heir of Fausto, is therefore bound to inconsistent with, those which the party subsequently After June 19, 1991, TRCDC invited Pacunayen to
fulfill all its terms and conditions. attempts to assert; (2) intent, or at least expectation, that meeting with the officers of the corporation. . . . In the
this conduct shall be acted upon by, or at least influence, same meeting, Pacunayens attention was called to the

75
Sales – Chapter 3-12 Cases
provision of the Contract of Lease had by her mother with inequitable. The subject property was sold in 1990 to end of September 1991, aside from the expected rentals
TRCDC, particularly paragraph 7 thereof, which states: respondent Pacunayen for a measly sum of P10,000.00. from the cockpit space lessees in the amount of
Obviously, the value is in a small amount because the P11,000.00.[46]
7. That should the lessor decide to sell the leased sale was between a mother and daughter. As admitted by
premises, the LESSEE shall have the priority right to said respondent, the sale made in her favor by her mother Under Article 2199 of the Civil Code, it is provided that:
purchase the same. was just a formality so that she may have the proper
representation with TRCDC in the absence of her Except as provided by law or by stipulation, one is entitled
Of course, in the meeting she had with the officers of parents[42] Consequently, the offer to be made to to an adequate compensation only for such pecuniary loss
TRCDC, Pacunayen explained that the sale made in her petitioner in this case should be under reasonable terms suffered by him as he has duly proved. Such
favor by her mother was just a formality so that she may and conditions, taking into account the fair market value of compensation is referred to as actual or compensatory
have the proper representation with TRCDC in the the property at the time it was sold to respondent. damages. (Emphasis supplied)
absence of her parents, more so that her father had
already passed away, and there was no malice in her In its complaint, petitioner prayed for the cancellation of The rule is that actual or compensatory damages cannot
mine (sic) and that of her mother, or any intention on their TCT No. M-35468 in the name of respondent be presumed, but must be proved with reasonable degree
part to deceive TRCDC. All these notwithstanding, and for Pacunayen,[43] which was issued by the Register of of certainty. A court cannot rely on speculations,
her to show their good faith in dealing with TRCDC, Deeds of Morong on February 7, 1991.[44] Under ordinary conjectures, or guesswork as to the fact and amount of
Pacunayen started the ground work to reconvey circumstances, this would be the logical effect of the damages, but must depend upon competent proof that
ownership over the whole land, now covered by Transfer rescission of the Kasulatan ng Bilihan Patuluyan ng Lupa they have been suffered by the injured party and on the
Certificare (sic) of Title No. M-259, to and in the name of between the deceased Fausto and respondent best obtainable evidence of the actual amount thereof. It
her mother (Fausto), but the latter was becoming sickly, Pacunayen. However, the circumstances in this case are must point out specific facts, which could afford a basis for
old and weak, and they found no time to do it as early as not ordinary. The buyer of the subject property is the measuring whatever compensatory or actual damages are
they wanted to.[40] (Emphasis supplied) sellers own daughter. If and when the title (TCT No. M- borne.[47]
35468) in respondent Pacunayens name is cancelled and
Given the foregoing, the Kasulatan ng Bilihan Patuluyan reinstated in Faustos name, and thereafter negotiations In the present case, there is no question that the Tanay
ng Lupa dated August 8, 1990 between Fausto and between petitioner and respondent Pacunayen for the Coliseum Cockpit was closed for two months and TRCDC
respondent must be rescinded. Considering, however, purchase of the subject property break down, then the did not gain any income during said period. But there is
that Fausto already died on March 16, 1996, during the subject property will again revert to respondent nothing on record to substantiate petitioners claim that it
pendency of this case with the CA, her heirs should have Pacunayen as she appears to be one of Faustos heirs. was bound to lose some P111,000.00 from such closure.
been substituted as respondents in this case. Considering This would certainly be a winding route to traverse. Sound TRCDCs president, Ambrosio Sacramento, testified that
further that the Court cannot declare respondent reason therefore dictates that title should remain in the they suffered income losses with the closure of the cockpit
Pacunayen as the sole heir, as it is not the proper forum name of respondent Pacunayen, for and in behalf of the from August 2, 1991 until it re-opened on October 20,
for that purpose, the right of petitioner may only be other heirs, if any, to be cancelled only when petitioner 1991.[48] Mr. Sacramento, however, cannot state with
enforced against the heirs of the deceased Catalina successfully exercises its right of first refusal and certainty the amount of such unrealized income.[49]
Matienzo Fausto, represented by respondent Pacunayen. purchases the subject property. Meanwhile, TRCDCs accountant, Merle Cruz, stated that
based on the corporations financial statement for the
In Paraaque Kings Enterprises, Inc. vs. Court of Petitioner further seeks the award of the following years 1990 and 1991,[50] they derived the amount of
Appeals,[41] it was ruled that the basis of the right of the damages in its favor: (1) P100,000.00 as actual damages; P120,000.00 as annual income from rent.[51] From said
first refusal must be the current offer to sell of the seller or (2) P1,100,000.00 as compensation for lost goodwill or financial statement, it is safe to presume that TRCDC
offer to purchase of any prospective buyer. It is only after reputation; (3) P100,000.00 as moral damages; (4) generated a monthly income of P10,000.00 a month
the grantee fails to exercise its right of first priority under P100,000.00 as exemplary damages; (5) P50,000.00 as (P120,000.00 annual income divided by 12 months). At
the same terms and within the period contemplated, could attorneys fees; (6) P1,000.00 appearance fee per hearing; best therefore, whatever actual damages that petitioner
the owner validly offer to sell the property to a third and (7) the costs of suit.[45] suffered from the cockpits closure for a period of two
person, again, under the same terms as offered to the months can be reasonably summed up only to
grantee. The circumstances of this case, however, dictate According to petitioner, respondents act in fencing the P20,000.00.
the application of a different ruling. An offer of the property property led to the closure of the Tanay Coliseum Cockpit
to petitioner under identical terms and conditions of the and petitioner was unable to conduct cockfights and Such award of damages shall earn interest at the legal
offer previously given to respondent Pacunayen would be generate income of not less than P100,000.00 until the rate of six percent (6%) per annum, which shall be
76
Sales – Chapter 3-12 Cases
computed from the time of the filing of the Complaint on interest, whether the case falls under paragraph 1 or Petitioners claim for moral damages must likewise be
August 22, 1991, until the finality of this decision. After the paragraph 2, above, shall be 12% per annum from such denied. The award of moral damages cannot be granted
present decision becomes final and executory, the rate of finality until its satisfaction, this interim period being in favor of a corporation because, being an artificial
interest shall increase to twelve percent (12%) per annum deemed to be by then an equivalent to a forbearance of person and having existence only in legal contemplation, it
from such finality until its satisfaction, this interim period credit.[54] has no feelings, no emotions, no senses. It cannot,
being deemed to be equivalent to a forbearance of therefore, experience physical suffering and mental
credit.[52] This is in accord with the guidelines laid down Petitioner also claims the amount of P1,100,000.00 as anguish, which can be experienced only by one having a
by the Court in Eastern Shipping Lines, Inc. vs. Court of compensation for lost goodwill or reputation. It alleged that nervous system.[58] Petitioner being a corporation,[59]
Appeals,[53] regarding the manner of computing legal with the unjust and wrongful conduct of the defendants as the claim for moral damages must be denied.
interest, viz.: above-described, plaintiff stands to lose its goodwill and
reputation established for the past 20 years.[55] With regard to the claim for exemplary damages, it is a
II. With regard particularly to an award of interest in the requisite in the grant thereof that the act of the offender
concept of actual and compensatory damages, the rate of An award of damages for loss of goodwill or reputation must be accompanied by bad faith or done in wanton,
interest, as well as the accrual thereof, is imposed, as falls under actual or compensatory damages as provided fraudulent or malevolent manner.[60] Moreover, where a
follows: in Article 2205 of the Civil Code, to wit: party is not entitled to actual or moral damages, an award
of exemplary damages is likewise baseless.[61] In this
1. When the obligation is breached, and it consists in the Art. 2205. Damages may be recovered: case, petitioner failed to show that respondent acted in
payment of a sum of money, i.e., a loan or forbearance of bad faith, or in wanton, fraudulent or malevolent manner.
money, the interest due should be that which may have (1) For loss or impairment of earning capacity in cases of
been stipulated in writing. Furthermore, the interest due temporary or permanent personal injury; Petitioner likewise claims the amount of P50,000.00 as
shall itself earn legal interest from the time it is judicially attorneys fees, the sum of P1,000.00 for every
demanded. In the absence of stipulation, the rate of (2) For injury to the plaintiffs business standing or appearance of its counsel, plus costs of suit. It is well
interest shall be 12% per annum to be computed from commercial credit. settled that no premium should be placed on the right to
default, i.e., from judicial or extrajudicial demand under litigate and not every winning party is entitled to an
and subject to the provisions of Article 1169 of the Civil Even if it is not recoverable as compensatory damages, it automatic grant of attorney's fees. The party must show
Code. may still be awarded in the concept of temperate or that he falls under one of the instances enumerated in
moderate damages.[56] In arriving at a reasonable level of Article 2208 of the Civil Code. In this case, since petitioner
2. When an obligation, not constituting a loan or temperate damages to be awarded, trial courts are guided was compelled to engage the services of a lawyer and
forbearance of money, is breached, an interest on the by the ruling that: incurred expenses to protect its interest and right over the
amount of damages awarded may be imposed at the subject property, the award of attorneys fees is proper.
discretion of the court at the rate of 6% per annum. No . . . There are cases where from the nature of the case, However there are certain standards in fixing attorney's
interest, however, shall be adjudged on unliquidated definite proof of pecuniary loss cannot be offered, fees, to wit: (1) the amount and the character of the
claims or damages except when or until the demand can although the court is convinced that there has been such services rendered; (2) labor, time and trouble involved; (3)
be established with reasonable certainty. Accordingly, loss. For instance, injury to one's commercial credit or to the nature and importance of the litigation and business in
where the demand is established with reasonable the goodwill of a business firm is often hard to show which the services were rendered; (4) the responsibility
certainty, the interest shall begin to run from the time the certainty in terms of money. Should damages be denied imposed; (5) the amount of money and the value of the
claim is made judicially or extrajudicially (Art. 1169, Civil for that reason? The judge should be empowered to property affected by the controversy or involved in the
Code) but when such certainty cannot be so reasonably calculate moderate damages in such cases, rather than employment; (6) the skill and the experience called for in
established at the time the demand is made, the interest that the plaintiff should suffer, without redress from the the performance of the services; (7) the professional
shall begin to run only from the date the judgment of the defendant's wrongful act. (Araneta v. Bank of America, 40 character and the social standing of the attorney; and (8)
court is made (at which time quantification of damages SCRA 144, 145)[57] the results secured, it being a recognized rule that an
may be deemed to have been reasonably ascertained). attorney may properly charge a much larger fee when it is
The actual base for the computation of legal interest shall, In this case, aside from the nebulous allegation of contingent than when it is not.[62] Considering the
in any case, be on the amount finally adjudged. petitioner in its amended complaint, there is no evidence foregoing, the award of P10,000.00 as attorneys fees,
on record, whether testimonial or documentary, to including the costs of suit, is reasonable under the
3. When the judgment of the court awarding a sum of adequately support such claim. Hence, it must be denied. circumstances.
money becomes final and executory, the rate of legal
77
Sales – Chapter 3-12 Cases
WHEREFORE, the instant Petition for Review is
PARTIALLY GRANTED. The Court of Appeals Decision SO ORDERED.
dated June 14, 1999 in CA-G.R. CV No. 43770 is
MODIFIED as follows:

(1) the Kasulatan ng Bilihan Patuluyan ng Lupa dated


August 8, 1990 between Catalina Matienzo Fausto and
respondent Anunciacion Fausto Pacunayen is hereby
deemed rescinded;

(2) The Heirs of the deceased Catalina Matienzo Fausto


who are hereby deemed substituted as respondents,
represented by respondent Anunciacion Fausto
Pacunayen, are ORDERED to recognize the obligation of
Catalina Matienzo Fausto under the Contract of Lease
with respect to the priority right of petitioner Tanay
Recreation Center and Development Corp. to purchase
the subject property under reasonable terms and
conditions;

(3) Transfer Certificate of Title No. M-35468 shall remain


in the name of respondent Anunciacion Fausto
Pacunayen, which shall be cancelled in the event
petitioner successfully purchases the subject property;

(4) Respondent is ORDERED to pay petitioner Tanay


Recreation Center and Development Corporation the
amount of Twenty Thousand Pesos (P20,000.00) as
actual damages, plus interest thereon at the legal rate of
six percent (6%) per annum from the filing of the
Complaint until the finality of this Decision. After this
Decision becomes final and executory, the applicable rate
shall be twelve percent (12%) per annum until its
satisfaction; and,

(5) Respondent is ORDERED to pay petitioner the amount


of Ten Thousand Pesos (P10,000.00) as attorneys fees,
and to pay the costs of suit.

(6) Let the case be remanded to the Regional Trial Court,


Morong, Rizal (Branch 78) for further proceedings on the
determination of the reasonable terms and conditions of
the offer to sell by respondents to petitioner, without
prejudice to possible mediation between the parties.

The rest of the unaffected dispositive portion of the Court


of Appeals Decision is AFFIRMED.
78
Sales – Chapter 3-12 Cases
ROBERTO D. TUAZON, Petitioner, Leons), for a total consideration of only ₱2,750,000.00 as accepted by Roberto. Thus, in a Decision dated
vs. evidenced by a Deed of Absolute Sale7 executed by the November 18, 2002, the trial court dismissed the
LOURDES Q. DEL ROSARIO-SUAREZ, CATALINA R. parties. TCT No. 1779868 was then issued by the complaint. Its dispositive portion reads:
SUAREZ-DE LEON, WILFREDO DE LEON, MIGUEL Registry of Deeds of Quezon City in the name of the De
LUIS S. DE LEON, ROMMEL LEE S. DE LEON, and Leons. WHEREFORE, premises considered, judgment is hereby
GUILLERMA L. SANDICO-SILVA, as attorney-in-fact of rendered dismissing the above-entitled Complaint for lack
the defendants, except Lourdes Q. Del Rosario- The new owners through their attorney-in-fact, Guillerma of merit, and ordering the Plaintiff to pay the Defendants,
Suarez, Respondents. S. Silva, notified Roberto to vacate the premises. Roberto the following:
refused hence, the De Leons filed a complaint for
In a situation where the lessor makes an offer to sell to the Unlawful Detainer before the Metropolitan Trial Court 1. the amount of ₱30,000.00 as moral damages;
lessee a certain property at a fixed price within a certain (MeTC) of Quezon City against him. On August 30, 2000,
period, and the lessee fails to accept the offer or to the MeTC rendered a Decision9 ordering Roberto to 2. the amount of ₱30,000.00 as exemplary damages;
purchase on time, then the lessee loses his right to buy vacate the property for non-payment of rentals and
the property and the owner can validly offer it to another. expiration of the contract. 3. the amount of ₱30,000.00 as attorney’s fees; and

This Petition for Review on Certiorari1 assails the Ruling of the Regional Trial Court 4. cost of the litigation.
Decision2 dated May 30, 2005 of the Court of Appeals
(CA) in CA-G.R. CV No. 78870, which affirmed the On November 8, 2000, while the ejectment case was on SO ORDERED.16
Decision3 dated November 18, 2002 of the Regional Trial appeal, Roberto filed with the RTC of Quezon City a
Court (RTC), Branch 101, Quezon City in Civil Case No. Complaint10 for Annulment of Deed of Absolute Sale, Ruling of the Court of Appeals
Q-00-42338. Reconveyance, Damages and Application for Preliminary
Injunction against Lourdes and the De Leons. On On May 30, 2005, the CA issued its Decision dismissing
Factual Antecedents November 13, 2000, Roberto filed a Notice of Lis Roberto’s appeal and affirming the Decision of the RTC.
Pendens11 with the Registry of Deeds of Quezon City.
Respondent Lourdes Q. Del Rosario-Suarez (Lourdes) Hence, this Petition for Review on Certiorari filed by
was the owner of a parcel of land, containing more or less On January 8, 2001, respondents filed An Answer with Roberto advancing the following arguments:
an area of 1,211 square meters located along Tandang Counterclaim12 praying that the Complaint be dismissed
Sora Street, Barangay Old Balara, Quezon City and for lack of cause of action. They claimed that the filing of I.
previously covered by Transfer Certificate of Title (TCT) such case was a mere leverage of Roberto against them
No. RT-561184 issued by the Registry of Deeds of because of the favorable Decision issued by the MeTC in The Trial Court and the Court of Appeals had decided that
Quezon City. the ejectment case. the "Right of First Refusal" exists only within the
parameters of an "Option to Buy", and did not exist when
On June 24, 1994, petitioner Roberto D. Tuazon (Roberto) On September 17, 2001, the RTC issued an Order13 the property was sold later to a third person, under
and Lourdes executed a Contract of Lease5 over the declaring Lourdes and the De Leons in default for their favorable terms and conditions which the former buyer
abovementioned parcel of land for a period of three years. failure to appear before the court for the second time can meet.
The lease commenced in March 1994 and ended in despite notice. Upon a Motion for Reconsideration,14 the
February 1997. During the effectivity of the lease, Lourdes trial court in an Order15 dated October 19, 2001 set aside II.
sent a letter6 dated January 2, 1995 to Roberto where its Order of default.
she offered to sell to the latter subject parcel of land. She What is the status or sanctions of an appellee in the Court
pegged the price at ₱37,541,000.00 and gave him two After trial, the court a quo rendered a Decision declaring of Appeals who has not filed or failed to file an appellee’s
years from January 2, 1995 to decide on the said offer. the Deed of Absolute Sale made by Lourdes in favor of brief?17
the De Leons as valid and binding. The offer made by
On June 19, 1997, or more than four months after the Lourdes to Roberto did not ripen into a contract to sell Petitioner’s Arguments
expiration of the Contract of Lease, Lourdes sold subject because the price offered by the former was not
parcel of land to her only child, Catalina Suarez-De Leon, acceptable to the latter. The offer made by Lourdes is no Roberto claims that Lourdes violated his right to buy
her son-in-law Wilfredo De Leon, and her two grandsons, longer binding and effective at the time she decided to sell subject property under
Miguel Luis S. De Leon and Rommel S. De Leon (the De the subject lot to the De Leons because the same was not
79
Sales – Chapter 3-12 Cases
the principle of "right of first refusal" by not giving him 10 Mont., 5; 24 Am. St. Rep., 17) the following quotation governed not by contracts (since the essential elements to
"notice" and the opportunity to buy the property under the has been taken: establish the vinculum juris would still be indefinite and
same terms and conditions or specifically based on the inconclusive) but by, among other laws of general
much lower price paid by the De Leons. ‘An agreement in writing to give a person the ‘option’ to application, the pertinent scattered provisions of the Civil
purchase lands within a given time at a named price is Code on human conduct.
Roberto further contends that he is enforcing his "right of neither a sale nor an agreement to sell. It is simply a
first refusal" based on Equatorial Realty Development, contract by which the owner of property agrees with Even on the premise that such right of first refusal has
Inc. v. Mayfair Theater, Inc.18 which is the leading case another person that he shall have the right to buy his been decreed under a final judgment, like here, its breach
on the "right of first refusal." property at a fixed price within a certain time. He does not cannot justify correspondingly an issuance of a writ of
sell his land; he does not then agree to sell it; but he does execution under a judgment that merely recognizes its
Respondents’ Arguments sell something; that is, the right or privilege to buy at the existence, nor would it sanction an action for specific
election or option of the other party. The second party performance without thereby negating the indispensable
On the other hand, respondents posit that this case is not gets in praesenti, not lands, nor an agreement that he element of consensuality in the perfection of contracts. It
covered by the principle of "right of first refusal" but an shall have lands, but he does get something of value; that is not to say, however, that the right of first refusal would
unaccepted unilateral promise to sell or, at best, a is, the right to call for and receive lands if he elects. The be inconsequential for, such as already intimated above,
contract of option which was not perfected. The letter of owner parts with his right to sell his lands, except to the an unjustified disregard thereof, given, for instance, the
Lourdes to Roberto clearly embodies an option contract second party, for a limited period. The second party circumstances expressed in Article 19 of the Civil Code,
as it grants the latter only two years to exercise the option receives this right, or rather, from his point of view, he can warrant a recovery for damages. (Emphasis
to buy the subject property at a price certain of receives the right to elect to buy. supplied.)
₱37,541,000.00. As an option contract, the said letter
would have been binding upon Lourdes without need of But the two definitions above cited refer to the contract of From the foregoing, it is thus clear that an option contract
any consideration, had Roberto accepted the offer. But in option, or, what amounts to the same thing, to the case is entirely different and distinct from a right of first refusal
this case there was no acceptance made neither was where there was cause or consideration for the obligation in that in the former, the option granted to the offeree is for
there a distinct consideration for the option contract. x x x. (Emphasis supplied.) a fixed period and at a determined price. Lacking these
two essential requisites, what is involved is only a right of
Our Ruling On the other hand, in Ang Yu Asuncion v. Court of first refusal.
Appeals,20 an elucidation on the "right of first refusal" was
The petition is without merit. made thus: In this case, the controversy is whether the letter of
Lourdes to Roberto dated January 2, 1995 involved an
This case involves an option contract and not a contract of In the law on sales, the so-called ‘right of first refusal’ is an option contract or a contract of a right of first refusal. In its
a right of first refusal innovative juridical relation. Needless to point out, it entirety, the said letter-offer reads:
cannot be deemed a perfected contract of sale under
In Beaumont v. Prieto,19 the nature of an option contract Article 1458 of the Civil Code. Neither can the right of first 206 Valdes Street
is explained thus: refusal, understood in its normal concept, per se be Josefa Subd. Balibago
brought within the purview of an option under the second Angeles City 2009
In his Law Dictionary, edition of 1897, Bouvier defines an paragraph of Article 1479, aforequoted, or possibly of an
option as a contract, in the following language: offer under Article 1319 of the same Code. An option or January 2, 1995
an offer would require, among other things, a clear
‘A contract by virtue of which A, in consideration of the certainty on both the object and the cause or Tuazon Const. Co.
payment of a certain sum to B, acquires the privilege of consideration of the envisioned contract. In a right of first 986 Tandang Sora Quezon City
buying from, or selling to, B certain securities or properties refusal, while the object might be made determinate, the
within a limited time at a specified price. (Story vs. exercise of the right, however, would be dependent not Dear Mr. Tuazon,
Salamon, 71 N. Y., 420.)’ only on the grantor's eventual intention to enter into a
binding juridical relation with another but also on terms, I received with great joy and happiness the big box of
From Vol. 6, page 5001, of the work "Words and including the price, that obviously are yet to be later firmed sweet grapes and ham, fit for a king’s party. Thanks very
Phrases," citing the case of Ide vs. Leiser (24 Pac., 695; up. Prior thereto, it can at best be so described as merely much.
belonging to a class of preparatory juridical relations
80
Sales – Chapter 3-12 Cases
I am getting very old (79 going 80 yrs. old) and wish to live the offeror may withdraw his offer by communicating such obligation, because, before the promise is accepted, the
in the U.S.A. with my only family. I need money to buy a withdrawal to the offeree at anytime before acceptance; if promissor may withdraw it at any time. Upon acceptance,
house and lot and a farm with a little cash to start. it is founded upon a consideration, the offeror cannot however, a bilateral contract to sell and to buy is created,
withdraw his offer before the lapse of the period agreed and the offeree ipso facto assumes the obligations of a
I am offering you to buy my 1211 square meter at upon. purchaser; the offeror, on the other hand, would be liable
₱37,541,000.00 you can pay me in dollars in the name of for damages if he fails to deliver the thing he had offered
my daughter. I never offered it to anyone. Please shoulder The second paragraph of Article 1479 declares that "an for sale.
the expenses for the transfer. I wish the Lord God will help accepted unilateral promise to buy or to sell a determinate
you buy my lot easily and you will be very lucky forever in thing for a price certain is binding upon the promissor if xxxx
this place. You have all the time to decide when you can, the promise is supported by a consideration distinct from
but not for 2 years or more. the price." Sanchez v. Rigos21 provided an interpretation Even if the promise was accepted, private respondent was
of the said second paragraph of Article 1479 in relation to not bound thereby in the absence of a distinct
I wish you long life, happiness, health, wealth and great Article 1324. Thus: consideration. (Emphasis ours.)
fortune always!
There is no question that under Article 1479 of the new In this case, it is undisputed that Roberto did not accept
I hope the Lord God will help you be the recipient of multi- Civil Code "an option to sell," or "a promise to buy or to the terms stated in the letter of Lourdes as he negotiated
billion projects aid from other countries. sell," as used in said article, to be valid must be for a much lower price. Roberto’s act of negotiating for a
"supported by a consideration distinct from the price." This much lower price was a counter-offer and is therefore not
Thank you, is clearly inferred from the context of said article that a an acceptance of the offer of Lourdes. Article 1319 of the
unilateral promise to buy or to sell, even if accepted, is Civil Code provides:
Lourdes Q. del Rosario vda de Suarez only binding if supported by consideration. In other words,
"an accepted unilateral promise can only have a binding Consent is manifested by the meeting of the offer and the
It is clear that the above letter embodies an option effect if supported by a consideration, which means that acceptance upon the thing and the cause which are to
contract as it grants Roberto a fixed period of only two the option can still be withdrawn, even if accepted, if the constitute the contract. The offer must be certain and the
years to buy the subject property at a price certain of same is not supported by any consideration. Hence, it is acceptance absolute. A qualified acceptance constitutes a
₱37,541,000.00. It being an option contract, the rules not disputed that the option is without consideration. It can counter-offer. (Emphasis supplied.)
applicable are found in Articles 1324 and 1479 of the Civil therefore be withdrawn notwithstanding the acceptance
Code which provide: made of it by appellee. The counter-offer of Roberto for a much lower price was
not accepted by Lourdes. There is therefore no contract
Art. 1324. When the offerer has allowed the offeree a It is true that under Article 1324 of the new Civil Code, the that was perfected between them with regard to the sale
certain period to accept, the offer may be withdrawn at general rule regarding offer and acceptance is that, when of subject property. Roberto, thus, does not have any right
any time before acceptance by communicating such the offerer gives to the offeree a certain period to accept, to demand that the property be sold to him at the price for
withdrawal, except when the option is founded upon a "the offer may be withdrawn at any time before which it was sold to the De Leons neither does he have
consideration, as something paid or promised. acceptance" except when the option is founded upon the right to demand that said sale to the De Leons be
consideration, but this general rule must be interpreted as annulled.
Art. 1479. A promise to buy and sell a determinate thing modified by the provision of Article 1479 above referred
for a price certain is reciprocally demandable. to, which applies to "a promise to buy and sell" Equatorial Realty Development, Inc. v. Mayfair Theater,
specifically. As already stated, this rule requires that a Inc. is not applicable here
An accepted unilateral promise to buy or to sell a promise to sell to be valid must be supported by a
determinate thing for a price certain is binding upon the consideration distinct from the price. It is the position of Roberto that the facts of this case and
promissor if the promise is supported by a consideration that of Equatorial are similar in nearly all aspects. Roberto
distinct from the price. In Diamante v. Court of Appeals,22 this Court further is a lessee of the property like Mayfair Theater in
declared that: Equatorial. There was an offer made to Roberto by
It is clear from the provision of Article 1324 that there is a Lourdes during the effectivity of the contract of lease
great difference between the effect of an option which is A unilateral promise to buy or sell is a mere offer, which is which was also the case in Equatorial. There were
without a consideration from one which is founded upon a not converted into a contract except at the moment it is negotiations as to the price which did not bear fruit
consideration. If the option is without any consideration, accepted. Acceptance is the act that gives life to a juridical because Lourdes sold the property to the De Leons which
81
Sales – Chapter 3-12 Cases
was also the case in Equatorial wherein Carmelo and would show that what drove her to offer the property to In the above cited case, De Leon was the plaintiff in a
Bauermann sold the property to Equatorial. The existence Roberto was her immediate need for funds as she was Complaint for a sum of money in the RTC. He obtained a
of the lease of the property is known to the De Leons as already very old. Offering the property to Roberto was not favorable judgment and so defendant went to the CA. The
they are related to Lourdes while in Equatorial, the an act of liberality on the part of Lourdes but was a simple appeal of defendant-appellant was taken cognizance of by
lawyers of Equatorial studied the lease contract of Mayfair matter of convenience and practicality as he was the one the CA but De Leon filed a Motion to Dismiss the Appeal
over the property. The property in this case was sold by most likely to buy the property at that time as he was then with Motion to Suspend Period to file Appellee’s Brief. The
Lourdes to the De Leons at a much lower price which is leasing the same. CA denied the Motion to Dismiss. De Leon filed a Motion
also the case in Equatorial where Carmelo and Bauerman for Reconsideration which actually did not suspend the
sold to Equatorial at a lesser price. It is Roberto’s All told, the facts of the case, as found by the RTC and the period to file the appellee’s brief. De Leon therefore failed
conclusion that as in the case of Equatorial, there was a CA, do not support Roberto’s claims that the letter of to file his brief within the period specified by the rules and
violation of his right of first refusal and hence annulment Lourdes gave him a right of first refusal which is similar to hence he was deemed by the CA to have waived his right
or rescission of the Deed of Absolute Sale is the proper the one given to Mayfair Theater in the case of Equatorial. to file appellee’s brief.
remedy. Therefore, there is no justification to annul the deed of
sale validly entered into by Lourdes with the De Leons. The failure of the appellee to file his brief would not result
Roberto’s reliance in Equatorial is misplaced. Despite his to the rendition of a decision favorable to the appellant.
claims, the facts in Equatorial radically differ from the facts What is the effect of the failure of Lourdes to file her The former is considered only to have waived his right to
of this case. Roberto overlooked the fact that in appellee’s brief at the CA? file the Appellee’s Brief. The CA has the jurisdiction to
Equatorial, there was an express provision in the Contract resolve the case based on the Appellant’s Brief and the
of Lease that – Lastly, Roberto argues that Lourdes should be sanctioned records of the case forwarded by the RTC. The appeal is
for her failure to file her appellee’s brief before the CA. therefore considered submitted for decision and the CA
(i)f the LESSOR should desire to sell the leased properly acted on it.
properties, the LESSEE shall be given 30-days exclusive Certainly, the appellee’s failure to file her brief would not
option to purchase the same. mean that the case would be automatically decided WHEREFORE, the instant petition for review on certiorari
against her. Under the circumstances, the prudent action is DENIED. The assailed Decision of the Court of Appeals
There is no such similar provision in the Contract of Lease on the part of the CA would be to deem Lourdes to have in CA-G.R. CV No. 78870, which affirmed the Decision
between Roberto and Lourdes. What is involved here is a waived her right to file her appellee’s brief. De Leon v. dated November 18, 2002 of the Regional Trial Court,
separate and distinct offer made by Lourdes through a Court of Appeals,23 is instructive when this Court Branch 101, Quezon City in Civil Case No. Q-00-42338 is
letter dated January 2, 1995 wherein she is selling the decreed: AFFIRMED.
leased property to Roberto for a definite price and which
gave the latter a definite period for acceptance. Roberto On the second issue, we hold that the Court of Appeals SO ORDERED.
was not given a right of first refusal. The letter-offer of did not commit grave abuse of discretion in considering
Lourdes did not form part of the Lease Contract because it the appeal submitted for decision. The proper remedy in
was made more than six months after the commencement case of denial of the motion to dismiss is to file the
of the lease. appellee’s brief and proceed with the appeal. Instead,
petitioner opted to file a motion for reconsideration which,
It is also very clear that in Equatorial, the property was unfortunately, was pro forma. All the grounds raised
sold within the lease period. In this case, the subject therein have been discussed in the first resolution of the
property was sold not only after the expiration of the respondent Court of Appeals. There is no new ground
period provided in the letter-offer of Lourdes but also after raised that might warrant reversal of the resolution. A
the effectivity of the Contract of Lease. cursory perusal of the motion would readily show that it
was a near verbatim repetition of the grounds stated in the
Moreover, even if the offer of Lourdes was accepted by motion to dismiss; hence, the filing of the motion for
Roberto, still the former is not bound thereby because of reconsideration did not suspend the period for filing the
the absence of a consideration distinct and separate from appellee’s brief. Petitioner was therefore properly deemed
the price. The argument of Roberto that the separate to have waived his right to file appellee’s brief. (Emphasis
consideration was the liberality on the part of Lourdes supplied.)lawphi1
cannot stand. A perusal of the letter-offer of Lourdes
82
Sales – Chapter 3-12 Cases
HEIRS OF FAUSTO C. IGNACIO, namely MARFEL D. considered petitioner's offer to repurchase, there was no sale transactions. The properties covered by TCT Nos. T-
IGNACIO-MANALO, MILFA D. IGNACIO-MANALO AND repurchase contract executed. The present controversy 117774 to 117776 are still registered in the name of
FAUSTINO D. IGNACIO, Petitioners, was fuelled by petitioner's stance that a verbal respondent bank.6
vs. repurchase/compromise agreement was actually reached
HOME BANKERS SAVINGS AND TRUST COMPANY, and implemented by the parties. In a letter addressed to respondent bank dated July 25,
SPOUSES PHILLIP AND THELMA RODRIGUEZ, 1989, petitioner expressed his willingness to pay the
CATHERINE, REYNOLD & JEANETTE, all surnamed In the meantime, respondent bank made the following amount of ₱600,000.00 in full, as balance of the
ZUNIGA, Respondents. dispositions of the foreclosed properties already titled in repurchase price, and requested respondent bank to
its name: release to him the remaining parcels of land covered by
Before the Court is a Petition for Review on Certiorari TCT Nos. 111058 and T-154658 ("subject properties").7
under Rule 45 assailing the Decision1 dated July 18, 2006 TCT No. 111059 (Subdivided into six lots with individual Respondent bank however, turned down his request. This
and Resolution2 dated May 2, 2007 of the Court of titles - TCT Nos. 117771, 117772, 117773, 117774, prompted petitioner to cause the annotation of an adverse
Appeals (CA) in CA-G.R. CV No. 73551. The CA reversed 117775 and 117776) claim on the said titles on September 18, 1989.8
the Decision3 dated June 15, 1999 of the Regional Trial
Court (RTC) of Pasig City, Branch 151 in Civil Case No. A. TCT No. 117771 (16,350 sq.ms.) - Sold to Fermin Prior to the annotation of the adverse claim, on August 24,
58980. Salvador and Bella Salvador under Deed of Absolute Sale 1989, the property covered by TCT No. 154658 was sold
dated May 23, 1984 for the price of ₱150,000.00 by respondent bank to respondent spouses Phillip and
The factual antecedents: Thelma Rodriguez, without informing the petitioner. On
B. TCT No. 11772 (82,569 sq.ms. subdivided into 2 October 6, 1989, again without petitioner's knowledge,
In August 1981, petitioner Fausto C. Ignacio mortgaged portions respondent bank sold the property covered by TCT No T-
two parcels of land to Home Savings Bank and Trust 111058 to respondents Phillip and Thelma Rodriguez,
Company, the predecessor of respondent Home Bankers 1) Lot 3-B-1 (35,447 sq.ms.) - Sold to Dr. Oscar Remulla Catherine M. Zuñiga, Reynold M. Zuñiga and Jeannette
Savings and Trust Company, as security for the and Natividad Pagtakhan, Dr. Edilberto Torres and Dra. M. Zuñiga.9
₱500,000.00 loan extended to him by said bank. These Rebecca Amores under Deed of Absolute Sale dated April
properties which are located in Cabuyao, Laguna are 17, 1985 for the price of ₱150,000.00 On December 27, 1989, petitioner filed an action for
covered by Transfer Certificate of Title Nos. (T-40380) T- specific performance and damages in the RTC against the
8595 and (T-45804) T-8350 containing an area of 83,303 2) Lot 3-B-2 covered by separate title TCT No. 124660 respondent bank. As principal relief, petitioner sought in
square meters and 120,110 square meters, respectively.4 (Subdivided into 3 portions - his original complaint the reconveyance of the subject
properties after his payment of ₱600,000.00.10
When petitioner defaulted in the payment of his loan Lot 3-B-2-A (15,000 sq.ms.) - Sold to Dr. Myrna del Respondent bank filed its Answer denying the allegations
obligation, respondent bank proceeded to foreclose the Carmen Reyes under Deed of Absolute Sale dated March of petitioner and asserting that it was merely exercising its
real estate mortgage. At the foreclosure sale held on 23, 1987 for the price of ₱150,000.00 right as owner of the subject properties when the same
January 26, 1983, respondent bank was the highest were sold to third parties.
bidder for the sum of ₱764,984.67. On February 8, 1983, Lot 3-B-2-B (15,000 sq.ms.) - Sold to Dr. Rodito Boquiren
the Certificate of Sale issued to respondent bank was under Deed of Absolute Sale dated March 23, 1987 for For failure of respondent bank to appear during the pre-
registered with the Registry of Deeds of Calamba, the price of ₱150,000.00 trial conference, it was declared as in default and
Laguna. With the failure of petitioner to redeem the petitioner was allowed to present his evidence ex parte on
foreclosed properties within one year from such Lot 3-B-2-C (17,122 sq.ms.) covered by TCT No. T- the same date (September 3, 1990). Petitioner
registration, title to the properties were consolidated in 154568 - simultaneously filed an "Ex-Parte Consignation" tendering
favor of respondent bank. Consequently, TCT Nos. T- the amount of ₱235,000.00 as balance of the repurchase
8595 and T-8350 were cancelled and TCT Nos. 111058 C. TCT No.117773 (17,232 sq.ms.) - Sold to Rizalina price.11 On September 7, 1990, the trial court rendered
and 111059 were issued in the name of respondent Pedrosa under Deed of Absolute Sale dated June 4, 1984 judgment in favor of petitioner. Said decision, as well as
bank.5 for the price of ₱150,000.00 the order of default, were subsequently set aside by the
trial court upon the filing of a motion for reconsideration by
Despite the lapse of the redemption period and The expenses for the subdivision of lots covered by TCT the respondent bank.12
consolidation of title in respondent bank, petitioner offered No. 111059 and TCT No. 117772 were shouldered by
to repurchase the properties. While the respondent bank petitioner who likewise negotiated the above-mentioned
83
Sales – Chapter 3-12 Cases
In its Order dated November 19, 1990, the trial court CA said that these were not payments of the repurchase
granted the motion for intervention filed by respondents 5. Dismissing the counterclaim of the defendant and price but were actually remittances of the payments made
Phillip and Thelma Rodriguez, Catherine Zuñiga, Reynold intervenors against the plaintiff. by petitioner's buyers for the purchase of the foreclosed
Zuñiga and Jeannette Zuñiga. Said intervenors asserted properties already titled in the name of respondent bank. It
their status as innocent purchasers for value who had no Costs against the defendant. was noted that two of these receipts (Exhibits "K" and "K-
notice or knowledge of the claim or interest of petitioner 1")18 were issued to Fermin Salvador and Rizalina
when they bought the properties already registered in the SO ORDERED.15 Pedrosa, the vendees of two subdivided lots under
name of respondent bank. Aside from a counterclaim for separate Deeds of Absolute Sale executed in their favor
damages against the petitioner, intervenors also prayed The trial court found that respondent bank deliberately by the respondent bank. In view of the attendant
that in the event respondent bank is ordered to reconvey disregarded petitioner's substantial payments on the total circumstances, the CA concluded that petitioner acted
the properties, respondent bank should be adjudged liable repurchase consideration. Reference was made to the merely as a broker or middleman in the sales transactions
to the intervenors and return all amounts paid to it.13 letter dated March 22, 1984 (Exhibit "I")16 as the authority involving the foreclosed properties. Lastly, the
for petitioner in making the installment payments directly respondents-intervenors were found to be purchasers who
On July 8, 1991, petitioner amended his complaint to to the Universal Properties, Inc. (UPI), respondent bank's bought the properties in good faith without notice of
include as alternative relief under the prayer for collecting agent. Said court concluded that the petitioner's interest or claim. Nonetheless, since there was
reconveyance the payment by respondent bank of the compromise agreement amounts to a valid contract of no repurchase contract perfected, the sale of the subject
prevailing market value of the subject properties "less sale between petitioner, as Buyer, and respondent bank, properties to respondents-intervenors remains valid and
whatever remaining obligation due the bank by reason of as Seller. Hence, in entertaining other buyers for the same binding, and the issue of whether the latter were innocent
the mortgage under the terms of the compromise properties already sold to petitioner with intention to purchasers for value would be of no consequence.
agreement.14 increase its revenues, respondent bank acted in bad faith
and is thus liable for damages to the petitioner. Petitioner's motion for reconsideration was likewise
On June 15, 1999, the trial court rendered its Decision, Intervenors were likewise found liable for damages as denied by the appellate court.
the dispositive portion of which reads: they failed to exercise due diligence before buying the
subject properties. Hence, this petition alleging that:
WHEREFORE, findings [sic] the facts aver[r]ed in the
complaint supported by preponderance of evidences Respondent bank appealed to the CA which reversed the A.
adduced, judgment is hereby rendered in favor of the trial court's ruling, as follows:
plaintiff and against the defendant and intervenors by: THE HONORABLE COURT OF APPEALS COMMITTED
WHEREFORE, the foregoing premises considered, the GRAVE ABUSE OF DISCRETION IN REVERSING THE
1. Declaring the two Deeds of Sale executed by the instant appeal is hereby GRANTED. Accordingly, the FINDING OF THE TRIAL COURT THAT THERE WAS A
defendant in favor of the intervenors as null and void and assailed decision is hereby REVERSED and SET ASIDE. PERFECTED CONTRACT TO REPURCHASE
the Register of Deeds in Calamba, Laguna is ordered to BETWEEN PETITIONER AND RESPONDENT-BANK.
cancel and/or annul the two Transfer Certificate of Titles SO ORDERED.17
No. T-154658 and TCT No. T-111058 issued to the B.
intervenors. The CA held that by modifying the terms of the offer
contained in the March 22, 1984 letter of respondent THE HONORABLE COURT OF APPEALS COMMITTED
2. Ordering the defendant to refund the amount of bank, petitioner effectively rejected the original offer with GRAVE ABUSE OF DISCRETION IN REVERSING THE
₱1,004,250.00 to the intervenors as the consideration of his counter-offer. There was also no written conformity by FINDING OF THE TRIAL COURT THAT PETITIONER
the sale of the two properties. respondent bank's officers to the amended conditions for DID NOT ACT AS BROKER IN THE SALE OF THE
repurchase which were unilaterally inserted by petitioner. FORECLOSED PROPERTIES AND THUS FAILED TO
3. Ordering the defendant to execute the appropriate Consequently, no contract of repurchase was perfected CONSIDER THE EXISTENCE OF OFFICIAL RECEIPTS
Deed of Reconveyance of the two (2) properties in favor and respondent bank acted well within its rights when it ISSUED IN THE NAME OF THE PETITIONER THAT
of the plaintiff after the plaintiff pays in full the amount of sold the subject properties to herein respondents- ARE DULY NOTED FOR HIS ACCOUNT.
₱600,000.00 as balance of the repurchase price. intervenors.
C.
4. Ordering the defendant bank to pay plaintiff the sum of As to the receipts presented by petitioner allegedly
₱50,000.00 as attorney's fees. proving the installment payments he had completed, the
84
Sales – Chapter 3-12 Cases
THE HONORABLE COURT OF APPEALS COMMITTED terms of payment, a contract is produced. The offer must Metro Manila
GRAVE ABUSE OF DISCRETION IN REVERSING THE be certain. To convert the offer into a contract, the
FINDING OF THE TRIAL COURT THAT RESPONDENT- acceptance must be absolute and must not qualify the Dear Judge Ignacio:
BANK DID NOT HAVE THE RIGHT TO DISPOSE THE terms of the offer; it must be plain, unequivocal,
SUBJECT PROPERTIES. unconditional, and without variance of any sort from the Your proposal to repurchase your foreclosed properties
proposal. A qualified acceptance, or one that involves a located at Cabuyao, Laguna consisting of a total area of
D. new proposal, constitutes a counter-offer and is a 203,413 square meters has been favorably considered
rejection of the original offer. Consequently, when subject to the following terms and conditions:
THE HONORABLE COURT OF APPEALS COMMITTED something is desired which is not exactly what is
GRAVE ABUSE OF DISCRETION IN REVERSING THE proposed in the offer, such acceptance is not sufficient to 1) Total Selling Price shall be ₱950,000.00
FINDING OF THE TRIAL COURT THAT generate consent because any modification or variation
RESPONDENTS-INTERVENORS ARE NOT INNOCENT from the terms of the offer annuls the offer.22 2) Downpayment of ₱150,00000 with the balance
PURCHASERS FOR VALUE IN GOOD FAITH.19 Payable in Three (3) equal installments
The acceptance must be identical in all respects with that as follows:
It is to be noted that the above issues raised by petitioner of the offer so as to produce consent or meeting of the
alleged grave abuse of discretion committed by the CA, minds.23 Where a party sets a different purchase price 1st Installment - P 266,667 - on or before May 31, '84
which is proper in a petition for certiorari under Rule 65 of than the amount of the offer, such acceptance was
the 1997 Rules of Civil Procedure, as amended, but not in qualified which can be at most considered as a counter- 2nd Installment - P 266,667 - on or before Sept. 31, '84
the present petition for review on certiorari under Rule 45. offer; a perfected contract would have arisen only if the
other party had accepted this counter-offer.24 In 3rd Installment - P 266,666 - on or before Jan. 30, '85
The core issue for resolution is whether a contract for the Villanueva v. Philippine National Bank25 this Court further
repurchase of the foreclosed properties was perfected elucidated on the meaning of unqualified acceptance, as TOTAL - P 800,000.00
between petitioner and respondent bank. follows:
3) All expenses pertinent to the subdivision of the parcel
The Court sustains the decision of the CA. …While it is impossible to expect the acceptance to echo of land consisting of 120,110 square meters shall be for
every nuance of the offer, it is imperative that it assents to your account.
Contracts are perfected by mere consent, which is those points in the offer which, under the operative facts
manifested by the meeting of the offer and the acceptance of each contract, are not only material but motivating as Thank you,
upon the thing and the cause which are to constitute the well. Anything short of that level of mutuality produces not
contract.20 The requisite acceptance of the offer is a contract but a mere counter-offer awaiting acceptance. Very truly yours,
expressed in Article 1319 of the Civil Code which states: More particularly on the matter of the consideration of the
contract, the offer and its acceptance must be unanimous RITA B. MANUEL
ART. 1319. Consent is manifested by the meeting of the both on the rate of the payment and on its term. An President
offer and the acceptance upon the thing and the cause acceptance of an offer which agrees to the rate but varies
which are to constitute the contract. The offer must be the term is ineffective.26 (Emphasis supplied) According to petitioner, he wrote the notations in the
certain and the acceptance absolute. A qualified presence of a certain Mr. Lazaro, the representative of
acceptance constitutes a counter-offer. Petitioner submitted as evidence of a perfected contract of Mrs. Manuel (President), and a certain Mr. Fajardo, which
repurchase the March 22, 1984 letter (Exhibit "I")27 from notations supposedly represent their "compromise
In Palattao v. Court of Appeals,21 this Court held that if Rita B. Manuel, then President of UPI, a corporation agreement."28 These notations indicate that the
the acceptance of the offer was not absolute, such formed by respondent bank to dispose of its acquired repurchase price would be ₱900,000.00 which shall be
acceptance is insufficient to generate consent that would assets, with notations handwritten by petitioner himself. paid as follows: ₱150,000 - end of May '84; ₱150,000 -
perfect a contract. Thus: Said letter reads: end of June '84; Balance - "Depending on financial
position". Petitioner further alleged the following
Contracts that are consensual in nature, like a contract of March 22, 1984 conditions of the verbal agreement: (1) respondent bank
sale, are perfected upon mere meeting of the minds. Once shall release the equivalent land area for payments made
there is concurrence between the offer and the Honorable Judge Fausto Ignacio by petitioner who shall shoulder the expenses for
acceptance upon the subject matter, consideration, and 412 Bagumbayan Street, Pateros subdivision of the land; (2) in case any portion of the
85
Sales – Chapter 3-12 Cases
subdivided land is sold by petitioner, a separate document some acts or conduct communicated to the offeror, it may or negotiations between petitioner and Mr. Lazaro and Mr.
of sale would be executed directly to the buyer; (3) the be made either in a formal or an informal manner, and Fajardo. Petitioner therefore had no legal right to compel
remaining portion of the properties shall not be subject of may be shown by acts, conduct, or words of the accepting respondent bank to accept the ₱600,000 being tendered
respondent bank's transaction without the consent and party that clearly manifest a present intention or by him as payment for the supposed balance of
authority of petitioner; (4) the petitioner shall continue in determination to accept the offer to buy or sell. Thus, repurchase price.
possession of the properties and whatever portion still acceptance may be shown by the acts, conduct, or words
remaining, and attending to the needs of its tenants; and of a party recognizing the existence of the contract of A contract of sale is consensual in nature and is perfected
(5) payments shall be made directly to UPI.29 sale.31 upon mere meeting of the minds. When there is merely an
offer by one party without acceptance of the other, there is
The foregoing clearly shows that petitioner's acceptance Even assuming that the bank officer or employee whom no contract.35 When the contract of sale is not perfected,
of the respondent bank's terms and conditions for the petitioner claimed he had talked to regarding the March it cannot, as an independent source of obligation, serve
repurchase of the foreclosed properties was not absolute. 22, 1984 letter had acceded to his own modified terms for as a binding juridical relation between the parties.36
Petitioner set a different repurchase price and also the repurchase, their supposed verbal exchange did not
modified the terms of payment, which even contained a bind respondent bank in view of its corporate nature. In sum, we find the ruling of the CA more in accord with
unilateral condition for payment of the balance There was no evidence that said Mr. Lazaro or Mr. the established facts and applicable law and
(₱600,000), that is, depending on petitioner's "financial Fajardo was authorized by respondent bank's Board of jurisprudence. Petitioner's claim of utmost accommodation
position." The CA thus considered the qualified Directors to accept petitioner's counter-proposal to by respondent bank of his own terms for the repurchase of
acceptance by petitioner as a counter-proposal which repurchase the foreclosed properties at the price and his foreclosed properties are simply contrary to normal
must be accepted by respondent bank. However, there terms other than those communicated in the March 22, business practice. As aptly observed by the appellate
was no evidence of any document or writing showing the 1984 letter. As this Court ruled in AF Realty & court:
conformity of respondent bank's officers to this counter- Development, Inc. v. Dieselman Freight Services, Co.32
proposal. The submission of the plaintiff-appellee is unimpressive.
Section 23 of the Corporation Code expressly provides
Petitioner contends that the receipts issued by UPI on his that the corporate powers of all corporations shall be First, if the counter-proposal was mutually agreed upon by
installment payments are concrete proof -- despite denials exercised by the board of directors. Just as a natural both the plaintiff-appellee and defendant-appellant, how
to the contrary by respondent bank -- that there was an person may authorize another to do certain acts in his come not a single signature of the representative of the
implied acceptance of his counter-proposal and that he behalf, so may the board of directors of a corporation defendant-appellant was affixed thereto. Second, it is
did not merely act as a broker for the sale of the validly inconceivable that an agreement of such great
subdivided portions of the foreclosed properties to third importance, involving two personalities who are both
parties. Since all these receipts, except for two receipts delegate some of its functions to individual officers or aware and familiar of the practical and legal necessity of
issued in the name of Fermin Salvador and Rizalina agents appointed by it.1âwphi1 Thus, contracts or acts of reducing agreements into writing, the plaintiff-appellee,
Pedrosa, were issued in the name of petitioner instead of a corporation must be made either by the board of being a lawyer and the defendant-appellant, a banking
the buyers themselves, petitioner emphasizes that the directors or by a corporate agent duly authorized by the institution, not to formalize their repurchase agreement.
payments were made for his account. Moreover, petitioner board. Absent such valid delegation/authorization, the rule Third, it is quite absurd and unusual that the defendant-
asserts that the execution of the separate deeds of sale is that the declarations of an individual director relating to appellant could have acceded to the condition that the
directly to the buyers was in pursuance of the perfected the affairs of the corporation, but not in the course of, or balance of the payment of the repurchase price would
repurchase agreement with respondent bank, such an connected with, the performance of authorized duties of depend upon the financial position of the plaintiff-appellee.
arrangement being "an accepted practice to save on taxes such director, are held not binding on the corporation.33 Such open[-]ended and indefinite period for payment is
and shortcut paper works." hardly acceptable to a banking institution like the
Thus, a corporation can only execute its powers and defendant-appellant whose core existence fundamentally
The Court is unconvinced. transact its business through its Board of Directors and depends upon its financial arrangements and transactions
through its officers and agents when authorized by a which, most, if not all the times are intended to bear
In Adelfa Properties, Inc. v. CA,30 the Court ruled that: board resolution or its by-laws.34 favorable outcome to its business. Last, had there been a
repurchase agreement, then, there should have been
x x x The rule is that except where a formal acceptance is In the absence of conformity or acceptance by properly titles or deeds of conveyance issued in favor of the
so required, although the acceptance must be authorized bank officers of petitioner's counter-proposal, plaintiff-appellee. But as it turned out, the plaintiff-appellee
affirmatively and clearly made and must be evidenced by no perfected repurchase contract was born out of the talks never had any land deeded or titled in his name as a
86
Sales – Chapter 3-12 Cases
result of the alleged repurchase agreement. All these,
reinforce the conclusion that the counter-proposal was
unilaterally made and inserted by the plaintiff-appellee in
Exhibit "I" and could not have been accepted by the
defendant-appellant, and that a different agreement other
than a repurchase agreement was perfected between
them.37

Petitioner Fausto C. Ignacio passed away on November


11, 2008 and was substituted by his heirs, namely: Marfel
D. Ignacio-Manalo, Milfa D. Ignacio-Manalo and Faustino
D. Ignacio.

WHEREFORE, the petition for review on certiorari is


DENIED. The Decision dated July 18, 2006 and
Resolution dated May 2, 2007 of the Court of Appeals in
CA-G.R. CV No. 73551 are hereby AFFIRMED.

With costs against the petitioners.

SO ORDERED.

87
Sales – Chapter 3-12 Cases
VILLONCO REALTY COMPANY, plaintiff-appellee and In the course of the negotiations, the brothers Romeo
EDITH PEREZ DE TAGLE, intervenor-appellee, Villonco and Teofilo Villonco conferred with Cervantes in (4) That if my negotiations with said property will not
vs. his office to discuss the price and terms of the sale. Later, be consummated by reason beyond my control, I will
BORMAHECO, INC., FRANCISCO N. CERVANTES and Cervantes "went to see Villonco for the same reason until return to you your deposit of P100,000 and the sale of my
ROSARIO N. CERVANTES, defendants-appellants. some agreement" was arrived at. On a subsequent property to you will not also be consummated; and
Meer, Meer & Meer for plaintiff-appellee. occasion, Cervantes, accompanied by Edith Perez de
Tagle, discussed again the terms of the sale with Villonco. (5) That final negotiations on both properties can be
This action was instituted by Villonco Realty Company definitely known after 45 days.
against Bormaheco, Inc. and the spouses Francisco N. During the negotiations, Villonco Realty Company
Cervantes and Rosario N. Cervantes for the specific assumed that the lots belonged to Bormaheco, Inc. and If the above terms is (are) acceptable to your Board,
performance of a supposed contract for the sale of land that Cervantes was duly authorized to sell the same. please issue out the said earnest money in favor of
and the improvements thereon for one million four Cervantes did not disclose to the broker and to Villonco Bormaheco, Inc., and deliver the same thru the bearer,
hundred thousand pesos. Edith Perez de Tagle, as agent, Realty Company that the lots were conjugal properties of Miss Edith Perez de Tagle.
intervened in order to recover her commission. The lower himself and his wife and that they were mortgaged to the
court enforced the sale. Bormaheco, Inc. and the DBP. Very truly yours,
Cervantes spouses, as supposed vendors, appealed.
Bormaheco, Inc., through Cervantes, made a written offer SGD. FRANCISCO N. CERVANTES
This Court took cognizance of the appeal because the dated February 12, 1964, to Romeo Villonco for the sale President
amount involved is more than P200,000 and the appeal of the property. The offer reads (Exh. B):
was perfected before Republic Act No. 5440 took effect on The property mentioned in Bormaheco's letter was the
September 9, 1968. The facts are as follows: BORMAHECO, INC. land of the National Shipyards & Steel Corporation
(Nassco), with an area of twenty thousand square meters,
Francisco N. Cervantes and his wife, Rosario P. Navarra- February 12,1964 located at Punta, Sta. Ana, Manila. At the bidding held on
Cervantes, are the owners of lots 3, 15 and 16 located at January 17, 1964 that land was awarded to Bormaheco,
245 Buendia Avenue, Makati, Rizal with a total area of Mr. Romeo Inc., the highest bidder, for the price of P552,000. The
three thousand five hundred square meters (TCT Nos. Villonco Villonco Building Nassco Board of Directors in its resolution of February 18,
43530, 43531 and 43532, Exh. A, A-1 and A-2). The lots Buendia Avenue 1964 authorized the General Manager to sign the
were mortgaged to the Development Bank of the Phil Makati, Rizal. necessary contract (Exh. H).
(DBP) on April 21, 1959 as security for a loan of
P441,000. The mortgage debt was fully paid on July 10, Dear Mr. Villonco: On February 28, 1964, the Nassco Acting General
1969. Manager wrote a letter to the Economic Coordinator,
This is with reference to our telephone conversation this requesting approval of that resolution. The Acting
Cervantes is the president of Bormaheco, Inc., a dealer noon on the matter of the sale of our property located at Economic Coordinator approved the resolution on March
and importer of industrial and agricultural machinery. The Buendia Avenue, with a total area of 3,500 sq. m., under 24, 1964 (Exh. 1).
entire lots are occupied by the building, machinery and the following conditions:
equipment of Bormaheco, Inc. and are adjacent to the In the meanwhile, Bormaheco, Inc. and Villonco Realty
property of Villonco Realty Company situated at 219 (1) That we are offering to sell to you the above Company continued their negotiations for the sale of the
Buendia Avenue. property at the price of P400.00 per square meter; Buendia Avenue property. Cervantes and Teofilo Villonco
had a final conference on February 27, 1964. As a result
In the early part of February, 1964 there were negotiations (2) That a deposit of P100,000.00 must be placed as of that conference Villonco Realty Company, through
for the sale of the said lots and the improvements thereon earnest money on the purchase of the above property Teofilo Villonco, in its letter of March 4, 1964 made a
between Romeo Villonco of Villonco Realty Company which will become part payment of the property in the revised counter- offer (Romeo Villonco's first counter-offer
"and Bormaheco, Inc., represented by its president, event that the sale is consummated; was dated February 24, 1964, Exh. C) for the purchase of
Francisco N. Cervantes, through the intervention of Edith the property. The counter-offer was accepted by
Perez de Tagle, a real estate broker". (3) That this sale is to be consummated only after I Cervantes as shown in Exhibit D, which is quoted below:
shall have also consummated my purchase of another
property located at Sta. Ana, Manila; VILLONCO REALTY COMPANY
88
Sales – Chapter 3-12 Cases
V. R. C. Building As regards to the other conditions which we have Punta lot had been awarded to Bormaheco, Inc. (25-26
219 Buendia Avenue, Makati, discussed during our last conference on February 27, tsn).
Rizal, Philippines 1964, the same shall be finalized upon preparation of the
contract to sell.* Edith Perez de Tagle, the broker, in a letter to Cervantes
March 4, 1964 dated March 31, 1964 articulated her shock and surprise
If the above terms and conditions are acceptable to you, at Bormaheco's turnabout. She reviewed the history of the
Mr. Francisco Cervantes. kindly sign your conformity hereunder. Enclosed is our deal and explained why Romeo Villonco could not agree
Bormaheco, Inc. check for ONE HUNDRED THOUSAND (P100,000.00) to the rescission of the sale (Exh. G).**
245 Buendia Avenue PESOS, MBTC Check No. 448314, as earnest money.
Makati, Rizal Cervantes in his letter of April 6, 1964, a reply to Miss
Very truly yours, Tagle's letter, alleged that the forty-five day period had
Dear Mr. Cervantes: already expired and the sale to Bormaheco, Inc. of the
VILLONCO REALTY COMPANY Punta property had not been consummated. Cervantes
In reference to the letter of Miss E. Perez de Tagle dated (Sgd.) TEOFILO VILLONCO said that his letter was a "manifestation that we are no
February 12th and 26, 1964 in respect to the terms and longer interested to sell" the Buendia Avenue property to
conditions on the purchase of your property located at CONFORME: Villonco Realty Company (Annex I of Stipulation of Facts).
Buendia Ave., Makati, Rizal, with a total area of 3,500 sq. The latter was furnished with a copy of that letter.
meters., we hereby revise our offer, as follows: BORMAHECO, INC.
(Sgd.) FRANCISCO CERVANTES In a letter dated April 7, 1964 Villonco Realty Company
1. That the price of the property shall be P400.00 returned the two checks to Bormaheco, Inc., stating that
per sq. m., including the improvements thereon; That this sale shall be subject to favorable consummation the condition for the cancellation of the contract had not
of a property in Sta. Ana we are negotiating. arisen and at the same time announcing that an action for
2. That a deposit of P100,000.00 shall be given to breach of contract would be filed against Bormaheco, Inc.
you as earnest money which will become as part payment (Sgd.) FRANCISCO CERVANTES (Annex G of Stipulation of Facts).1äwphï1.ñët
in the event the sale is consummated;
The check for P100,000 (Exh. E) mentioned in the On that same date, April 7, 1964 Villonco Realty Company
3. This sale shall be cancelled, only if your deal with foregoing letter-contract was delivered by Edith Perez de filed the complaint (dated April 6) for specific performance
another property in Sta. Ana shall not be consummated Tagle to Bormaheco, Inc. on March 4, 1964 and was against Bormaheco, Inc. Also on that same date, April 7,
and in such case, the P100,000-00 earnest money will be received by Cervantes. In the voucher-receipt evidencing at eight-forty-five in the morning, a notice of lis pendens
returned to us with a 10% interest p.a. However, if our the delivery the broker indicated in her handwriting that was annotated on the titles of the said lots.
deal with you is finalized, said P100,000.00 will become the earnest money was "subject to the terms and
as part payment for the purchase of your property without conditions embodied in Bormaheco's letter" of February Bormaheco, Inc. in its answers dated May 5 and 25, 1964
interest: 12 and Villonco Realty Company's letter of March 4, 1964 pleaded the defense that the perfection of the contract of
(Exh. E-1; 14 tsn). sale was subject to the conditions (a) "that final
4. The manner of payment shall be as follows: acceptance or not shall be made after 45 days" (sic) and
Then, unexpectedly, in a letter dated March 30, 1964, or (b) that Bormaheco, Inc. "acquires the Sta. Ana property".
a. P100,000.00 earnest money and twenty-six days after the signing of the contract of sale,
650,000.00 as part of the down payment, or Exhibit D, Cervantes returned the earnest money, with On June 2, 1964 or during the pendency of this case, the
P750,000.00 as total down payment interest amounting to P694.24 (at ten percent per annum). Nassco Acting General Manager wrote to Bormaheco,
Cervantes cited as an excuse the circumstance that Inc., advising it that the Board of Directors and the
b. The balance is payable as follows: "despite the lapse of 45 days from February 12, 1964 Economic Coordinator had approved the sale of the Punta
P100,000.00 after 3 months there is no certainty yet" for the acquisition of the Punta lot to Bormaheco, Inc. and requesting the latter to send its
125,000.00 -do- property (Exh. F; F-I and F-2). Villonco Realty Company duly authorized representative to the Nassco for the
212,500.00 -do- refused to accept the letter and the checks of Bormaheco, signing of the deed of sale (Exh. 1).
P650,000.00 Total Inc. Cervantes sent them by registered mail. When he
rescinded the contract, he was already aware that the The deed of sale for the Punta land was executed on
June 26, 1964. Bormaheco, Inc. was represented by
89
Sales – Chapter 3-12 Cases
Cervantes (Exh. J. See Bormaheco, Inc. vs. Abanes, L- Bormaheco, Inc. and the Cervantes spouses appealed. indubitably proves that there was a meeting of minds upon
28087, July 31, 1973, 52 SCRA 73). Their principal contentions are (a) that no contract of sale the subject matter and consideration of the sale.
was perfected because Cervantes made a supposedly Therefore, on that date the sale was perfected. (Compare
In view of the disclosure in Bormaheco's amended answer qualified acceptance of the revised offer contained in with McCullough vs. Aenlle & Co., 3 Phil. 285; Goyena vs.
that the three lots were registered in the names of the Exhibit D, which acceptance amounted to a counter-offer, Tambunting, 1 Phil. 490). Not only that Bormaheco's
Cervantes spouses and not in the name of Bormaheco, and because the condition that Bormaheco, inc. would acceptance of the part payment of one hundred ,thousand
Inc., Villonco Realty Company on July 21, 1964 filed an acquire the Punta land within the forty-five-day period was pesos shows that the sale was conditionally
amended complaint impleading the said spouses as not fulfilled; (2) that Bormaheco, Inc. cannot be compelled consummated or partly executed subject to the purchase
defendants. Bormaheco, Inc. and the Cervantes spouses to sell the land which belongs to the Cervantes spouses by Bormaheco, Inc. of the Punta property. The
filed separate answers. and (3) that Francisco N. Cervantes did not bind the nonconsummation of that purchase would be a negative
conjugal partnership and his wife when, as president of resolutory condition (Taylor vs. Uy Tieng Piao, 43 Phil.
As of January 15, 1965 Villonco Realty Company had paid Bormaheco, Inc., he entered into negotiations with 873).
to the Manufacturers' Bank & Trust Company the sum of Villonco Realty Company regarding the said land.
P8,712.25 as interests on the overdraft line of P100,000 On February 18, 1964 Bormaheco's bid for the Punta
and the sum of P27.39 as interests daily on the same loan We hold that the appeal, except as to the issue of property was already accepted by the Nassco which had
since January 16, 1965. (That overdraft line was later damages, is devoid of merit. authorized its General Manager to sign the corresponding
settled by Villonco Realty Company on a date not deed of sale. What was necessary only was the approval
mentioned in its manifestation of February 19, 1975). "By the contract of sale one of the contracting parties of the sale by the Economic Coordinator and a request for
obligates himself to transfer the ownership of and to that approval was already pending in the office of that
Villonco Realty Company had obligated itself to pay the deliver a determining thing, and the other to pay therefor a functionary on March 4, 1964.
sum of P20,000 as attorney's fees to its lawyers. It price certain in money or its equivalent. A contract of sale
claimed that it was damaged in the sum of P10,000 a may be absolute or conditional" (Art. 1458, Civil Code). Bormaheco, Inc. and the Cervantes spouses contend that
month from March 24, 1964 when the award of the Punta the sale was not perfected because Cervantes allegedly
lot to Bormaheco, Inc. was approved. On the other hand, "The contract of sale is perfected at the moment there is a qualified his acceptance of Villonco's revised offer and,
Bormaheco, Inc. claimed that it had sustained damages of meeting of minds upon the thing which is the object of the therefore, his acceptance amounted to a counter-offer
P200,000 annually due to the notice of lis pendens which contract and upon the price. From that moment, the which Villonco Realty Company should accept but no
had prevented it from constructing a multi-story building parties may reciprocally demand performance, subject to such acceptance was ever transmitted to Bormaheco, Inc.
on the three lots. (Pars. 18 and 19, Stipulation of the provisions of the law governing the form of contracts" which, therefore, could withdraw its offer.
Facts).1äwphï1.ñët (Art. 1475, Ibid.).
That contention is not well-taken. It should be stressed
Miss Tagle testified that for her services Bormaheco, Inc., "Contracts are perfected by mere consent, and from that that there is no evidence as to what changes were made
through Cervantes, obligated itself to pay her a three moment the parties are bound not only to the fulfillment of by Cervantes in Villonco's revised offer. And there is no
percent commission on the price of P1,400,000 or the what has been expressly stipulated but also to all the evidence that Villonco Realty Company did not assent to
amount of forty-two thousand pesos (14 tsn). consequences which, according to their nature, may be in the supposed changes and that such assent was never
keeping with good faith, usage and law" (Art. 1315, Civil made known to Cervantes.
After trial, the lower court rendered a decision ordering the Code).
Cervantes spouses to execute in favor of Bormaheco, Inc. What the record reveals is that the broker, Miss Tagle,
a deed of conveyance for the three lots in question and "Consent is manifested by the meeting of the offer and the acted as intermediary between the parties. It is safe to
directing Bormaheco, Inc. (a) to convey the same lots to acceptance upon the thing and the cause which are to assume that the alleged changes or qualifications made
Villonco Realty Company, (b) to pay the latter, as constitute the contract. The offer must be certain and the by Cervantes were approved by Villonco Realty Company
consequential damages, the sum of P10,000 monthly from acceptance absolute. A qualified acceptance constitutes a and that such approval was duly communicated to
March 24, 1964 up to the consummation of the sale, (c) to counter-offer" (Art. 1319, Civil Code). "An acceptance may Cervantes or Bormaheco, Inc. by the broker as shown by
pay Edith Perez de Tagle the sum of P42,000 as broker's be express or implied" (Art. 1320, Civil Code). the fact that Villonco Realty Company paid, and
commission and (d) pay P20,000 as to attorney's fees Bormaheco, Inc. accepted, the sum of P100,000 as
(Civil Case No. 8109). Bormaheco's acceptance of Villonco Realty Company's earnest money or down payment. That crucial fact implies
offer to purchase the Buendia Avenue property, as shown that Cervantes was aware that Villonco Realty Company
in Teofilo Villonco's letter dated March 4, 1964 (Exh. D), had accepted the modifications which he had made in
90
Sales – Chapter 3-12 Cases
Villonco's counter-offer. Had Villonco Realty Company not not be categorized as a major alteration of that counter-
assented to those insertions and annotations, then it offer that prevented a meeting of the minds of the parties. Thus, in the Zayco case, Salvador Serra offered to sell to
would have stopped payment on its check for P100,000. It was understood that the parties had contemplated a rate Lorenzo Zayco his sugar central for P1,000,000 on
The fact that Villonco Realty Company allowed its check of ten percent per annum since ten percent a month or condition that the price be paid in cash, or, if not paid in
to be cashed by Bormaheco, Inc. signifies that the semi-annually would be usurious. cash, the price would be payable within three years
company was in conformity with the changes made by provided security is given for the payment of the balance
Cervantes and that Bormaheco, Inc. was aware of that Appellants Bormaheco, Inc. and Cervantes further within three years with interest. Zayco, instead of
conformity. Had those insertions not been binding, then contend that Cervantes, in clarifying in the voucher for the unconditionally accepting those terms, countered that he
Bormaheco, Inc. would not have paid interest at the rate earnest money of P100,000 that Bormaheco's acceptance was going to make a down payment of P100,000, that
of ten percent per annum, on the earnest money of thereof was subject to the terms and conditions embodied Serra's mortgage obligation to the Philippine National
P100,000. in Bormaheco's letter of February 12, 1964 and your Bank of P600,000 could be transferred to Zayco's account
(Villonco's) letter of March 4, 1964" made Bormaheco's and that he (plaintiff) would give a bond to secure the
The truth is that the alleged changes or qualifications in acceptance "qualified and conditional". payment of the balance of the price. It was held that the
the revised counter — offer (Exh. D) are not material or acceptance was conditional or was a counter-offer which
are mere clarifications of what the parties had previously That contention is not correct. There is no incompatibility had to be accepted by Serra. There was no such
agreed upon. between Bormaheco's offer of February 12, 1964 (Exh. B) acceptance. Serra revoked his offer. Hence, there was no
and Villonco's counter-offer of March 4, 1964 (Exh. D). perfected contract.
Thus, Cervantes' alleged insertion in his handwriting of The revised counter-offer merely amplified Bormaheco's
the figure and the words "12th and" in Villonco's counter- original offer. In the Beaumont case, Benito Valdes offered to sell to W
offer is the same as the statement found in the voucher- Borck the Nagtahan Hacienda owned by Benito Legarda,
receipt for the earnest money, which reads: "subject to the The controlling fact is that there was agreement between who had empowered Valdes to sell it. Borck was given
terms and conditions embodied in Bormaheco's letter of the parties on the subject matter, the price and the mode three months from December 4, 1911 to buy the hacienda
Feb. 12, 1964 and your letter of March 4, 1964" (Exh. E- of payment and that part of the price was paid. "Whenever for P307,000. On January 17, 1912 Borck wrote to
1). earnest money is given in a contract of sale, it shall be Valdes, offering to purchase the hacienda for P307,000
considered as part of the price and as proof of the payable on May 1, 1912. No reply was made to that letter.
Cervantes allegedly crossed out the word "Nassco" in perfection of the contract" (Art. 1482, Civil Code). Borck wrote other letters modifying his proposal. Legarda
paragraph 3 of Villonco's revised counter-offer and refused to convey the property.
substituted for it the word "another" so that the original "It is true that an acceptance may contain a request for
phrase, "Nassco's property in Sta. Ana", was made to certain changes in the terms of the offer and yet be a It was held that Borck's January 17th letter plainly
read as "another property in Sta. Ana". That change is binding acceptance. 'So long as it is clear that the departed from the terms of the offer as to the time of
trivial. What Cervantes did was merely to adhere to the meaning of the acceptance is positively and unequivocally payment and was a counter-offer which amounted to a
wording of paragraph 3 of Bormaheco's original offer to accept the offer, whether such request is granted or rejection of Valdes' original offer. A subsequent
(Exh. B) which mentions "another property located at Sta. not, a contract is formed.' " (Stuart vs. Franklin Life Ins. unconditional acceptance could not revive that offer.
Ana." His obvious purpose was to avoid jeopardizing his Co., 165 Fed. 2nd 965, citing Sec. 79, Williston on
negotiation with the Nassco for the purchase of its Sta. Contracts). The instant case is different from Laudico and Harden vs.
Ana property by unduly publicizing it. Arias Rodriguez, 43 Phil. 270 where the written offer to
Thus, it was held that the vendor's change in a phrase of sell was revoked by the offer or before the offeree's
It is noteworthy that Cervantes, in his letter to the broker the offer to purchase, which change does not essentially acceptance came to the offeror's knowledge.
dated April 6, 1964 (Annex 1) or after the Nassco property change the terms of the offer, does not amount to a
had been awarded to Bormaheco, Inc., alluded to the rejection of the offer and the tender of a counter-offer Appellants' next contention is that the contract was not
"Nassco property". At that time, there was no more need (Stuart vs. Franklin Life Ins. Co., supra). perfected because the condition that Bormaheco, Inc.
of concealing from the public that Bormaheco, Inc. was would acquire the Nassco land within forty-five days from
interested in the Nassco property. The instant case is not governed by the rulings laid down February 12, 1964 or on or before March 28, 1964 was
in Beaumont vs. Prieto, 41 Phil. 670, 985, 63 L. Ed. 770, not fulfilled. This contention is tied up with the following
Similarly, Cervantes' alleged insertion of the letters "PA" ( and Zayco vs. Serra, 44 Phil. 326. In those two cases the letter of Bormaheco, Inc. (Exh. F):
per annum) after the word "interest" in that same acceptance radically altered the offer and, consequently,
paragraph 3 of the revised counter-offer (Exh. D) could there was no meeting of the minds of the parties. BORMAHECO, INC.
91
Sales – Chapter 3-12 Cases
consummated after he had consummated the purchase of subject to the condition that "the final negotiations
March 30, 1964 the Nassco property. Then, in paragraph 5 of the same (acceptance) will have to be made by defendant within 45
offer he stated "that final negotiations on both properties days from said acceptance" (31 Record on Appeal). If that
Villonco Realty Company can be definitely known after forty-five days" (See Exh. B). were so, then the consummation of Bormaheco's
V.R.C. Building purchase of the Nassco property would be made within
219 Buendia Ave., It is deducible from the tenor of those statements that the forty-five days from March 4, 1964.
Makati, Rizal consummation of the sale of the Buendia lots to Villonco
Realty Company was conditioned on Bormaheco's What makes Bormaheco's stand more confusing and
Gentlemen: acquisition of the Nassco land. But it was not spelled out untenable is that in its three answers it invariably
that such acquisition should be effected within forty-five articulated the incoherent and vague affirmative defense
We are returning herewith your earnest money together days from February 12, 1964. Had it been Cervantes' that its acceptance of Villonco's revised counter-offer was
with interest thereon at 10% per annum. Please be intention that the forty-five days would be the period within conditioned on the circumstance "that final acceptance or
informed that despite the lapse of the 45 days from which the Nassco land should be acquired by Bormaheco, not shall be made after 45 days" whatever that means.
February 12, 1964 there is no certainty yet for us to then he would have specified that period in paragraph 3 of That affirmative defense is inconsistent with the other
acquire a substitute property, hence the return of the his offer so that paragraph would read in this wise: "That aforequoted incoherent statement in its third answer that
earnest money as agreed upon. this sale is to be consummated only after I shall have "the final negotiations (acceptance) will have to be made
consummated my purchase of another property located at by defendant within 45 days from said acceptance" (31
Very truly yours, Sta. Ana, Manila within forty-five days from the date Record on Appeal).1äwphï1.ñët
hereof ." He could have also specified that period in his
SGD. FRANCISCO N. CERVANTES "conforme" to Villonco's counter-offer of March 4, 1964 Thus, Bormaheco's three answers and paragraph 5 of his
President (Exh. D) so that instead of merely stating "that this sale offer of February 12, 1964 do not sustain at all its theory
shall be subject to favorable consummation of a property that the Nassco property should be acquired on or before
Encl.: P.N.B. Check No. 112994 J in Sta. Ana we are negotiating" he could have said: "That March 28, 1964. Its rescission or revocation of its
P.N.B. Check No. 112996J this sale shall be subject to favorable consummation acceptance cannot be anchored on that theory which, as
within forty-five days from February 12, 1964 of a property articulated in its pleadings, is quite equivocal and unclear.
That contention is predicated on the erroneous in Sta. Ana we are negotiating".
assumption that Bormaheco, Inc. was to acquire the It should be underscored that the condition that
Nassco land within forty-five days or on or before March No such specification was made. The term of forty-five Bormaheco, Inc. should acquire the Nassco property was
28, 1964. days was not a part of the condition that the Nassco fulfilled. As admitted by the appellants, the Nassco
property should be acquired. It is clear that the statement property was conveyed to Bormaheco, Inc. on June 26,
The trial court ruled that the forty-five-day period was "that final negotiations on both property can be definitely 1964. As early as January 17, 1964 the property was
merely an estimate or a forecast of how long it would take known after 45 days" does not and cannot mean that awarded to Bormaheco, Inc. as the highest bidder. On
Bormaheco, Inc. to acquire the Nassco property and it Bormaheco, Inc. should acquire the Nassco property February 18, 1964 the Nassco Board authorized its
was not "a condition or a deadline set for the defendant within forty-five days from February 12, 1964 as General Manager to sell the property to Bormaheco, Inc.
corporation to decide whether or not to go through with pretended by Cervantes. It is simply a surmise that after (Exh. H). The Economic Coordinator approved the award
the sale of its Buendia property". forty-five days (in fact when the forty-five day period on March 24, 1964. It is reasonable to assume that had
should be computed is not clear) it would be known Cervantes been more assiduous in following up the
The record does not support the theory of Bormaheco, whether Bormaheco, Inc. would be able to acquire the transaction, the Nassco property could have been
Inc. and the Cervantes spouses that the forty-five-day Nassco property and whether it would be able to sell the transferred to Bormaheco, Inc. on or before March 28,
period was the time within which (a) the Nassco property Buendia property. That aforementioned paragraph 5 does 1964, the supposed last day of the forty-five-day period.
and two Pasong Tamo lots should be acquired, (b) when not even specify how long after the forty-five days the
Cervantes would secure his wife's consent to the sale of outcome of the final negotiations would be known. The appellants, in their fifth assignment of error, argue
the three lots and (c) when Bormaheco, Inc. had to decide that Bormaheco, Inc. cannot be required to sell the three
what to do with the DBP encumbrance. It is interesting to note that in paragraph 6 of Bormaheco's lots in question because they are conjugal properties of
answer to the amended complaint, which answer was the Cervantes spouses. They aver that Cervantes in
Cervantes in paragraph 3 of his offer of February 12, 1964 verified by Cervantes, it was alleged that Cervantes dealing with the Villonco brothers acted as president of
stated that the sale of the Buendia lots would be accepted Villonco's revised counter-offer of March 4, 1964 Bormaheco, Inc. and not in his individual capacity and,
92
Sales – Chapter 3-12 Cases
therefore, he did not bind the conjugal partnership nor prior negotiations, the defendant (Bormaheco, Inc.) made Bormaheco's three answers that were verified by
Mrs. Cervantes who was allegedly opposed to the sale. a formal offer to sell to the plaintiff the property of the said Cervantes.
defendant situated at the abovenamed address along
Those arguments are not sustainable. It should be Buendia Avenue, Makati, Rizal, under the terms of the It is significant to note that Bormaheco, Inc. in its three
remembered that Cervantes, in rescinding the contract of letter-offer, a copy of which is hereto attached as Annex A answers, which were verified by Cervantes, never pleaded
sale and in returning the earnest money, cited as an hereof", now Exhibit B (2 Record on Appeal). as an affirmative defense that Mrs. Cervantes opposed
excuse the circumstance that there was no certainty in the sale of the three lots or that she did not authorize her
Bormaheco's acquisition of the Nassco property (Exh. F That paragraph 2 was not, repeat, was not denied by husband to sell those lots. Likewise, it should be noted
and Annex 1). He did not say that Mrs. Cervantes was Bormaheco, Inc. in its answer dated May 5, 1964. It did that in their separate answer the Cervantes spouses
opposed to the sale of the three lots. He did not tell not traverse that paragraph 2. Hence, it was deemed never pleaded as a defense that Mrs. Cervantes was
Villonco Realty Company that he could not bind the admitted. However, it filed an amended answer dated May opposed to the sale of three lots or that Cervantes could
conjugal partnership. In truth, he concealed the fact that 25, 1964 wherein it denied that it was the owner of the not bind the conjugal partnership. The appellants were at
the three lots were registered "in the name of three lots. It revealed that the three lots "belong and are first hesitant to make it appear that Cervantes had
FRANCISCO CERVANTES, Filipino, of legal age, married registered in the names of the spouses Francisco N. committed the skullduggery of trying to sell property which
to Rosario P. Navarro, as owner thereof in fee simple". He Cervantes and Rosario N. Cervantes." he had no authority to alienate.
certainly led the Villonco brothers to believe that as
president of Bormaheco, Inc. he could dispose of the said The three answers of Bormaheco, Inc. contain the It was only during the trial on May 17, 1965 that Cervantes
lots. He inveigled the Villoncos into believing that he had following affirmative defense: declared on the witness stand that his wife was opposed
untrammelled control of Bormaheco, Inc., that to the sale of the three lots, a defense which, as already
Bormaheco, Inc. owned the lots and that he was invested 13. That defendant's insistence to finally decide on stated, was never interposed in the three answers of
with adequate authority to sell the same. the proposed sale of the land in question after 45 days Bormaheco, Inc. and in the separate answer of the
had not only for its purpose the determination of its Cervantes spouses. That same viewpoint was adopted in
Thus, in Bormaheco's offer of February 12, 1964, acquisition of the said Sta. Ana (Nassco) property during defendants' motion for reconsideration dated November
Cervantes first identified the three lots as "our property" the said period, but also to negotiate with the actual and 20, 1965.
which "we are offering to sell ..." (Opening paragraph and registered owner of the parcels of land covered by T.C.T.
par. 1 of Exh. B). Whether the prounoun "we" refers to Nos. 43530, 43531 and 43532 in question which plaintiff But that defense must have been an afterthought or was
himself and his wife or to Bormaheco, Inc. is not clear. was fully aware that the same were not in the name of the evolved post litem motam since it was never disclosed in
Then, in paragraphs 3 and 4 of the offer, he used the first defendant (sic; Par. 18 of Answer to Amended Complaint, Cervantes' letter of rescission and in his letter to Miss
person and said: "I shall have consummated my 10, 18 and 34, Record on Appeal). Tagle (Exh. F and Annex 1). Moreover, Mrs. Cervantes
purchase" of the Nassco property; "... my negotiations did not testify at the trial to fortify that defense which had
with said property" and "I will return to you your deposit". In that affirmative defense, Bormaheco, Inc. pretended already been waived for not having been pleaded (See
Those expressions conveyed the impression and that it needed forty- five days within which to acquire the sec. 2, Rule 9, Rules of Court).
generated the belief that the Villoncos did not have to deal Nassco property and "to negotiate" with the registered
with Mrs. Cervantes nor with any other official of owner of the three lots. The absurdity of that pretension Taking into account the situation of Cervantes vis-a-vis
Bormaheco, Inc. stands out in bold relief when it is borne in mind that the Bormaheco, Inc. and his wife and the fact that the three
answers of Bormaheco, Inc. were verified by Cervantes lots were entirely occupied by Bormaheco's building,
The pleadings disclose that Bormaheco, Inc. and and that the registered owner of the three lots is machinery and equipment and were mortgaged to the
Cervantes deliberately and studiously avoided making the Cervantes himself. That affirmative defense means that DBP as security for its obligation, and considering that
allegation that Cervantes was not authorized by his wife to Cervantes as president of Bormaheco, Inc. needed forty- appellants' vague affirmative defenses do not include Mrs.
sell the three lots or that he acted merely as president of five days in order to "negotiate" with himself (Cervantes). Cervantes' alleged opposition to the sale, the plea that
Bormaheco, Inc. That defense was not interposed so as Cervantes had no authority to sell the lots strains the
not to place Cervantes in the ridiculous position of having The incongruous stance of the Cervantes spouses is also rivets of credibility (Cf. Papa and Delgado vs. Montenegro,
acted under false pretenses when he negotiated with the patent in their answer to the amended complaint. In that 54 Phil. 331; Riobo vs. Hontiveros, 21 Phil. 31).
Villoncos for the sale of the three lots. answer they disclaimed knowledge or information of
certain allegations which were well-known to Cervantes as "Obligations arising from contracts have the force of law
Villonco Realty Company, in paragraph 2 of its original president of Bormaheco, Inc. and which were admitted in between the contracting parties and should be complied
complaint, alleged that "on February 12, 1964, after some with in good faith" (Art. 1159, Civil Code). Inasmuch as
93
Sales – Chapter 3-12 Cases
the sale was perfected and even partly executed, 4. Bormaheco, Inc. is ordered (a) to pay Villonco
Bormaheco, Inc., and the Cervantes spouses, as a matter The appellants in their eighth assignment of error impugn Realty Company twenty thousand pesos (P20,000) as
of justice and good faith, are bound to comply with their the trial court's adjudication of forty-two thousand pesos attorney's fees and (b) to pay Edith Perez de Tagle the
contractual commitments. as three percent broker's commission to Miss Tagle. They sum of forty-two thousand pesos (P42,000) as
allege that there is no evidence that Bormaheco, Inc. commission. Costs against the defendants-appellants.
Parenthetically, it may be observed that much engaged her services as a broker in the projected sale of
misunderstanding could have been avoided had the the three lots and the improvements thereon. That SO ORDERED.
broker and the buyer taken the trouble of making some allegation is refuted by paragraph 3 of the stipulation of
research in the Registry of Deeds and availing themselves facts and by the documentary evidence. It was stipulated
of the services of a competent lawyer in drafting the that Miss Tagle intervened in the negotiations for the sale
contract to sell. of the three lots. Cervantes in his original offer of February
12, 1964 apprised Villonco Realty Company that the
Bormaheco, Inc. and the Cervantes spouses in their sixth earnest money should be delivered to Miss Tagle, the
assignment of error assail the trial court's award to bearer of the letter-offer. See also Exhibit G and Annex I
Villonco Realty Company of consequential damage of the stipulation of facts.
amounting to ten thousand pesos monthly from March 24,
1964 (when the Economic Coordinator approved the We hold that the trial court did not err in adjudging that
award of the Nassco property to Bormaheco, Inc.) up to Bormaheco, Inc. should pay Miss Tagle her three percent
the consummation of the sale. The award was based on commission.
paragraph 18 of the stipulation of facts wherein Villonco
Realty Company "submits that the delay in the WHEREFORE, the trial court's decision is modified as
consummation of the sale" has caused it to suffer the follows:
aforementioned damages.
1. Within ten (10) days from the date the
The appellants contend that statement in the stipulation of defendants-appellants receive notice from the clerk of the
facts simply means that Villonco Realty Company lower court that the records of this case have been
speculates that it has suffered damages but it does not received from this Court, the spouses Francisco N.
mean that the parties have agreed that Villonco Realty Cervantes and Rosario P. Navarra-Cervantes should
Company is entitled to those damages. execute a deed conveying to Bormaheco, Inc. their three
lots covered by Transfer Certificate of Title Nos. 43530,
Appellants' contention is correct. As rightly observed by 43531 and 43532 of the Registry of Deeds of Rizal.
their counsel, the damages in question were not
specifically pleaded and proven and were "clearly 2. Within five (5) days from the execution of such
conjectural and speculative". deed of conveyance, Bormaheco, Inc. should execute in
favor of Villonco Realty Company, V. R. C. Building, 219
However, appellants' view in their seventh assignment of Buendia Avenue, Makati, Rizal a registerable deed of sale
error that the trial court erred in ordering Bormaheco, Inc. for the said three lots and all the improvements thereon,
to pay Villonco Realty Company the sum of twenty free from all lien and encumbrances, at the price of four
thousand pesos as attorney's fees is not tenable. Under hundred pesos per square meter, deducting from the total
the facts of the case, it is evident that Bormaheco, Inc. purchase price the sum of P100,000 previously paid by
acted in gross and evident bad faith in refusing to satisfy Villonco Realty Company to Bormaheco, Inc.
the valid and just demand of Villonco Realty Company for
specific performance. It compelled Villonco Realty 3. Upon the execution of such deed of sale,
Company to incure expenses to protect its interest. Villonco Realty Company is obligated to pay Bormaheco,
Moreover, this is a case where it is just and equitable that Inc. the balance of the price in the sum of one million
the plaintiff should recover attorney's fees (Art. 2208, Civil three hundred thousand pesos (P1,300,000).
Code).
94
Sales – Chapter 3-12 Cases
MANILA METAL CONTAINER CORPORATION, Petitioner sent a letter dated August 25, 1983 to respondent would return the deposit should petitioner desire to withdraw
Petitioner, PNB, requesting that it be granted an extension of time to its offer to purchase the property.[17] On February 25, 1985,
- versus - redeem/repurchase the property.[8] In its reply dated August petitioner, through counsel, requested that PNB reconsider its
PHILIPPINE NATIONAL BANK, 30, 1983, respondent PNB informed petitioner that the letter dated December 28, 1984. Petitioner declared that it
Respondent, request had been referred to its Pasay City Branch for had already agreed to the SAMDs offer to purchase the
DMCI-PROJECT DEVELOPERS, INC., appropriate action and recommendation.[9] property for P1,574,560.47, and that was why it had paid
Intervenor. P725,000.00. Petitioner warned respondent PNB that it would
In a letter[10] dated February 10, 1984, petitioner reiterated seek judicial recourse should PNB insist on the position.[18]
Before us is a petition for review on certiorari of the its request for a one year extension from February 17, 1984
Decision[1] of the Court of Appeals (CA) in CA-G.R. No. within which to redeem/repurchase the property on On June 4, 1985, respondent PNB informed petitioner that
46153 which affirmed the decision[2] of the Regional Trial installment basis. It reiterated its request to repurchase the the PNB Board of Directors had accepted petitioners offer to
Court (RTC), Branch 71, Pasig City, in Civil Case No. 58551, property on installment.[11] Meanwhile, some PNB Pasay purchase the property, but for P1,931,389.53 in cash less the
and its Resolution[3] denying the motion for reconsideration City Branch personnel informed petitioner that as a matter of P725,000.00 already deposited with it.[19] On page two of the
filed by petitioner Manila Metal Container Corporation policy, the bank does not accept partial redemption.[12] letter was a space above the typewritten name of petitioners
(MMCC). President, Pablo Gabriel, where he was to affix his signature.
Since petitioner failed to redeem the property, the Register of However, Pablo Gabriel did not conform to the letter but
The Antecedents Deeds cancelled TCT No. 32098 on June 1, 1984, and issued merely indicated therein that he had received it.[20] Petitioner
a new title in favor of respondent PNB.[13] Petitioners offers did not respond, so PNB requested petitioner in a letter dated
Petitioner was the owner of a 8,015 square meter parcel of had not yet been acted upon by respondent PNB. June 30, 1988 to submit an amended offer to repurchase.
land located in Mandaluyong (now a City), Metro Manila. The
property was covered by Transfer Certificate of Title (TCT) Meanwhile, the Special Assets Management Department Petitioner rejected respondents proposal in a letter dated July
No. 332098 of the Registry of Deeds of Rizal. To secure a (SAMD) had prepared a statement of account, and as of June 14, 1988. It maintained that respondent PNB had agreed to
P900,000.00 loan it had obtained from respondent Philippine 25, 1984 petitioners obligation amounted to P1,574,560.47. sell the property for P1,574,560.47, and that since its
National Bank (PNB), petitioner executed a real estate This included the bid price of P1,056,924.50, interest, P725,000.00 downpayment had been accepted, respondent
mortgage over the lot. Respondent PNB later granted advances of insurance premiums, advances on realty taxes, PNB was proscribed from increasing the purchase price of
petitioner a new credit accommodation of P1,000,000.00; registration expenses, miscellaneous expenses and the property.[21] Petitioner averred that it had a net balance
and, on November 16, 1973, petitioner executed an publication cost.[14] When apprised of the statement of payable in the amount of P643,452.34. Respondent PNB,
Amendment[4] of Real Estate Mortgage over its property. On account, petitioner remitted P725,000.00 to respondent PNB however, rejected petitioners offer to pay the balance of
March 31, 1981, petitioner secured another loan of as deposit to repurchase, and Official Receipt No. 978191 P643,452.34 in a letter dated August 1, 1989.[22]
P653,000.00 from respondent PNB, payable in quarterly was issued to it.[15]
installments of P32,650.00, plus interests and other On August 28, 1989, petitioner filed a complaint against
charges.[5] In the meantime, the SAMD recommended to the respondent PNB for Annulment of Mortgage and Mortgage
management of respondent PNB that petitioner be allowed to Foreclosure, Delivery of Title, or Specific Performance with
On August 5, 1982, respondent PNB filed a petition for repurchase the property for P1,574,560.00. In a letter dated Damages. To support its cause of action for specific
extrajudicial foreclosure of the real estate mortgage and November 14, 1984, the PNB management informed performance, it alleged the following:
sought to have the property sold at public auction for petitioner that it was rejecting the offer and the
P911,532.21, petitioners outstanding obligation to respondent recommendation of 34. As early as June 25, 1984, PNB had accepted the down
PNB as of June 30, 1982,[6] plus interests and attorneys the SAMD. It was suggested that petitioner purchase the payment from Manila Metal in the substantial amount of
fees. property for P2,660,000.00, its minimum market value. P725,000.00 for the redemption/repurchase price of
Respondent PNB gave petitioner until December 15, 1984 to P1,574,560.47 as approved by its SMAD and considering the
After due notice and publication, the property was sold at act on the proposal; otherwise, its P725,000.00 deposit would reliance made by Manila Metal and the long time that has
public auction on September 28, 1982 where respondent be returned and the property would be sold to other elapsed, the approval of the higher management of the Bank
PNB was declared the winning bidder for P1,000,000.00. The interested buyers.[16] to confirm the agreement of its SMAD is clearly a potestative
Certificate of Sale[7] issued in its favor was registered with condition which cannot legally prejudice Manila Metal which
the Office of the Register of Deeds of Rizal, and was Petitioner, however, did not agree to respondent PNBs has acted and relied on the approval of SMAD. The Bank
annotated at the dorsal portion of the title on February 17, proposal. Instead, it wrote another letter dated December 12, cannot take advantage of a condition which is entirely
1983. Thus, the period to redeem the property was to expire 1984 requesting for a reconsideration. Respondent PNB dependent upon its own will after accepting and benefiting
on February 17, 1984. replied in a letter dated December 28, 1984, wherein it from the substantial payment made by Manila Metal.
reiterated its proposal that petitioner purchase the property
for P2,660,000.00. PNB again informed petitioner that it
95
Sales – Chapter 3-12 Cases
35. PNB approved the repurchase price of P1,574,560.47 for e) Ordering the defendant PNB to pay the plaintiff Manila counterclaim. It ordered respondent PNB to refund the
which it accepted P725,000.00 from Manila Metal. PNB Metals actual damages, moral and exemplary damages in the P725,000.00 deposit petitioner had made.[32] The trial court
cannot take advantage of its own delay and long inaction in aggregate amount of not less than P80,000.00 as may be ruled that there was no perfected contract of sale between
demanding a higher amount based on unilateral computation warranted by the evidence and fixed by this Honorable Court the parties; hence, petitioner had no cause of action for
of interest rate without the consent of Manila Metal. in the exercise of its sound discretion, and attorneys fees of specific performance against respondent. The trial court
P50,000.00 and litigation expenses of at least P30,000.00 as declared that respondent had rejected petitioners offer to
Petitioner later filed an amended complaint and supported its may be proved during the trial, and costs of suit. repurchase the property. Petitioner, in turn, rejected the terms
claim for damages with the following arguments: and conditions contained in the June 4, 1985 letter of the
Plaintiff likewise prays for such further reliefs which may be SAMD. While petitioner had offered to repurchase the
36. That in order to protect itself against the wrongful and deemed just and equitable in the premises.[24] property per its letter of
malicious acts of the defendant Bank, plaintiff is constrained July 14, 1988, the amount of P643,422.34 was way below the
to engage the services of counsel at an agreed fee of In its Answer to the complaint, respondent PNB averred, as a P1,206,389.53 which respondent PNB had demanded. It
P50,000.00 and to incur litigation expenses of at least special and affirmative defense, that it had acquired further declared that the P725,000.00 remitted by petitioner to
P30,000.00, which the defendant PNB should be condemned ownership over the property after the period to redeem had respondent PNB on June 4, 1985 was a deposit, and not a
to pay the plaintiff Manila Metal. elapsed. It claimed that no contract of sale was perfected downpayment or earnest money.
between it and petitioner after the period to redeem the
37. That by reason of the wrongful and malicious actuations property had expired. On appeal to the CA, petitioner made the following
of defendant PNB, plaintiff Manila Metal suffered besmirched allegations:
reputation for which defendant PNB is liable for moral During pre-trial, the parties agreed to submit the case for
damages of at least P50,000.00. decision, based on their stipulation of facts.[25] The parties I
agreed to limit the issues to the following: THE LOWER COURT ERRED IN RULING THAT
38. That for the wrongful and malicious act of defendant PNB DEFENDANT-APPELLEES LETTER DATED 4 JUNE 1985
which are highly reprehensible, exemplary damages should 1. Whether or not the June 4, 1985 letter of the defendant APPROVING/ACCEPTING PLAINTIFF-APPELLANTS
be awarded in favor of the plaintiff by way of example or approving/accepting plaintiffs offer to purchase the property is OFFER TO PURCHASE THE SUBJECT PROPERTY IS
correction for the public good of at least P30,000.00.[23] still valid and legally enforceable. NOT VALID AND ENFORCEABLE.

2. Whether or not the plaintiff has waived its right to purchase II


Petitioner prayed that, after due proceedings, judgment be the property when it failed to conform with the conditions set THE LOWER COURT ERRED IN RULING THAT THERE
rendered in its favor, thus: forth by the defendant in its letter dated June 4, 1985. WAS NO PERFECTED CONTRACT OF SALE BETWEEN
PLAINTIFF-APPELLANT AND DEFENDANT-APPELLEE.
a) Declaring the Amended Real Estate Mortgage (Annex A) 3. Whether or not there is a perfected contract of sale
null and void and without any legal force and effect. between the parties.[26] III
THE LOWER COURT ERRED IN RULING THAT
b) Declaring defendants acts of extra-judicially foreclosing PLAINTIFF-APPELLLANT WAIVED ITS RIGHT TO
the mortgage over plaintiffs property and setting it for auction While the case was pending, respondent PNB demanded, on PURCHASE THE SUBJECT PROPERTY WHEN IT FAILED
sale null and void. September 20, 1989, that petitioner vacate the property within TO CONFORM WITH CONDITIONS SET FORTH BY
15 days from notice,[27] but petitioners refused to do so. DEFENDANT-APPELLEE IN ITS LETTER DATED 4 JUNE
c) Ordering the defendant Register of Deeds to cancel the 1985.
new title issued in the name of PNB (TCT NO. 43792) On March 18, 1993, petitioner offered to repurchase the
covering the property described in paragraph 4 of the property for P3,500,000.00.[28] The offer was however IV
Complaint, to reinstate TCT No. 37025 in the name of Manila rejected by respondent PNB, in a letter dated April 13, 1993. THE LOWER COURT ERRED IN DISREGARDING THE
Metal and to cancel the annotation of the mortgage in According to it, the prevailing market value of the property FACT THAT IT WAS THE DEFENDANT-APPELLEE WHICH
question at the back of the TCT No. 37025 described in was approximately P30,000,000.00, and as a matter of policy, RENDERED IT DIFFICULT IF NOT IMPOSSIBLE FOR
paragraph 4 of this Complaint. it could not sell the property for less than its market value.[29] PLAINTIFF-APPELLANT TO COMPLETE THE BALANCE
On June 21, 1993, petitioner offered to purchase the property OF THEIR PURCHASE PRICE.
d) Ordering the defendant PNB to return and/or deliver for P4,250,000.00 in cash.[30] The offer was again rejected
physical possession of the TCT No. 37025 described in by respondent PNB on September 13, 1993.[31] V
paragraph 4 of this Complaint to the plaintiff Manila Metal. THE LOWER COURT ERRED IN DISREGARDING THE
On May 31, 1994, the trial court rendered judgment FACT THAT THERE WAS NO VALID RESCISSION OR
dismissing the amended complaint and respondent PNBs
96
Sales – Chapter 3-12 Cases
CANCELLATION OF SUBJECT CONTRACT OF According to the appellate court, the claim for damages and
REPURCHASE. the counterclaim were correctly dismissed by the court a quo The threshold issue is whether or not petitioner and
for no evidence was presented to support it. Respondent respondent PNB had entered into a perfected contract for
VI PNBs letter dated June 30, 1988 cannot revive the failed petitioner to repurchase the property from respondent.
THE LOWER COURT ERRED IN DECLARING THAT negotiations between the parties. Respondent PNB merely
PLAINTIFF FAILED AND REFUSED TO SUBMIT THE asked petitioner to submit an amended offer to repurchase. Petitioner maintains that it had accepted respondents offer
AMENDED REPURCHASE OFFER. While petitioner reiterated its request for a lower selling price made through the SAMD, to sell the property for
and that the balance of the repurchase be reduced, however, P1,574,560.00. When the acceptance was made in its letter
VII respondent rejected the proposal in a letter dated August 1, dated June 25, 1984; it then deposited P725,000.00 with the
THE LOWER COURT ERRED IN DISMISSING THE 1989. SAMD as partial payment, evidenced by Receipt No. 978194
AMENDED COMPLAINT OF PLAINTIFF-APPELLANT. which respondent had issued. Petitioner avers that the
Petitioner filed a motion for reconsideration, which the CA SAMDs acceptance of the deposit amounted to an
VIII likewise denied. acceptance of its offer to repurchase. Moreover, as gleaned
THE LOWER COURT ERRED IN NOT AWARDING from the letter of SAMD dated June 4, 1985, the PNB Board
PLAINTIFF-APPELLANT ACTUAL, MORAL AND Thus, petitioner filed the instant petition for review on of Directors had approved petitioners offer to purchase the
EXEMPLARY DAMAGES, ATTOTRNEYS FEES AND certiorari, alleging that: property. It claims that this was the suspensive condition, the
LITIGATION EXPENSES.[33] fulfillment of which gave rise to the contract. Respondent
Meanwhile, on June 17, 1993, petitioners Board of Directors I. THE COURT OF APPEALS ERRED ON A QUESTION OF could no longer unilaterally withdraw its offer to sell the
approved Resolution No. 3-004, where it waived, assigned LAW WHEN IT RULED THAT THERE IS NO PERFECTED property for P1,574,560.47, since the acceptance of the offer
and transferred its rights over the property covered by TCT CONTRACT OF SALE BETWEEN THE PETITIONER AND resulted in a perfected contract of sale; it was obliged to remit
No. 33099 and TCT No. 37025 in favor of Bayani Gabriel, RESPONDENT. to respondent the balance of the original purchase price of
one of its Directors.[34] Thereafter, Bayani Gabriel executed P1,574,560.47, while respondent was obliged to transfer
a Deed of Assignment over 51% of the ownership and II. THE COURT OF APPEALS ERRED ON A QUESTION OF ownership and deliver the property to petitioner, conformably
management of the property in favor of Reynaldo Tolentino, LAW WHEN IT RULED THAT THE AMOUNT OF with Article 1159 of the New Civil Code.
who later moved for leave to intervene as plaintiff-appellant. PHP725,000.00 PAID BY THE PETITIONER IS NOT AN
On July 14, 1993, the CA issued a resolution granting the EARNEST MONEY. Petitioner posits that respondent was proscribed from
motion,[35] and likewise granted the motion of Reynaldo increasing the interest rate after it had accepted respondents
Tolentino substituting petitioner MMCC, as plaintiff-appellant, III. THE COURT OF APPEALS ERRED ON A QUESTION offer to sell the property for P1,574,560.00. Consequently,
and his motion to withdraw as intervenor.[36] OF LAW WHEN IT RULED THAT THE FAILURE OF THE respondent could no longer validly make a counter-offer of
PETITIONER-APPELLANT TO SIGNIFY ITS CONFORMITY P1,931,789.88 for the purchase of the property. It likewise
The CA rendered judgment on May 11, 2000 affirming the TO THE TERMS CONTAINED IN PNBS JUNE 4, 1985 maintains that, although the P725,000.00 was considered as
decision of the RTC.[37] It declared that petitioner obviously LETTER MEANS THAT THERE WAS NO VALID AND deposit for the repurchase of the property in the receipt
never agreed to the selling price proposed by respondent LEGALLY ENFORCEABLE CONTRACT OF SALE issued by the SAMD, the amount constitutes earnest money
PNB (P1,931,389.53) since petitioner had kept on insisting BETWEEN THE PARTIES. as contemplated in Article 1482 of the New Civil Code.
that the selling price should be lowered to P1,574,560.47. Petitioner cites the rulings of this Court in Villonco v.
Clearly therefore, there was no meeting of the minds between IV. THE COURT OF APPEALS ERRED ON A QUESTION Bormaheco[39] and Topacio v. Court of Appeals.[40]
the parties as to the price or consideration of the sale. OF LAW THAT NON-PAYMENT OF THE PETITIONER-
APPELLANT OF THE BALANCE OF THE OFFERED PRICE Petitioner avers that its failure to append its conformity to the
The CA ratiocinated that petitioners original offer to purchase IN THE LETTER OF PNB DATED JUNE 4, 1985, WITHIN June 4, 1984 letter of respondent and its failure to pay the
the subject property had not been accepted by respondent SIXTY (60) DAYS FROM NOTICE OF APPROVAL balance of the price as fixed by respondent within the 60-day
PNB. In fact, it made a counter-offer through its June 4, 1985 CONSTITUTES NO VALID AND LEGALLY ENFORCEABLE period from notice was to protest respondents breach of its
letter specifically on the selling price; petitioner did not agree CONTRACT OF SALE BETWEEN THE PARTIES. obligation to petitioner. It did not amount to a rejection of
to the counter-offer; and the negotiations did not prosper. respondents offer to sell the property since respondent was
Moreover, petitioner did not pay the balance of the purchase V. THE COURT OF APPEALS SERIOUSLY ERRED WHEN merely seeking to enforce its right to pay the balance of
price within the sixty-day period set in the June 4, 1985 letter IT HELD THAT THE LETTERS OF PETITIONER- P1,570,564.47. In any event, respondent had the option
of respondent PNB. Consequently, there was no perfected APPELLANT DATED MARCH 18, 1993 AND JUNE 21, 1993, either to accept the balance of the offered price or to cause
contract of sale, and as such, there was no contract to OFFERING TO BUY THE SUBJECT PROPERTY AT the rescission of the contract.
rescind. DIFFERENT AMOUNT WERE PROOF THAT THERE IS NO
PERFECTED CONTRACT OF SALE.[38] Petitioners letters dated March 18, 1993 and June 21, 1993
to respondent during the pendency of the case in the RTC
97
Sales – Chapter 3-12 Cases
were merely to compromise the pending lawsuit, they did not Board of Directors. The SAMD was not authorized by according to their nature, may be in keeping with good faith,
constitute separate offers to repurchase the property. Such respondents Board to enter into contracts of sale with third usage and law.[43]
offer to compromise should not be taken against it, in persons involving corporate assets. There is absolutely
accordance with Section 27, Rule 130 of the Revised Rules nothing on record that respondent authorized the SAMD, or By the contract of sale, one of the contracting parties
of Court. made it appear to petitioner that it represented itself as obligates himself to transfer the ownership of and deliver a
having such authority. determinate thing, and the other to pay therefor a price
For its part, respondent contends that the parties never certain in money or its equivalent.[44] The absence of any of
graduated from the negotiation stage as they could not agree Respondent reiterates that SAMD had informed petitioner the essential elements will negate the existence of a
on the amount of the repurchase price of the property. All that that its offer to repurchase had been approved by the Board perfected contract of sale. As the Court ruled in Boston Bank
transpired was an exchange of proposals and counter- subject to the condition, among others, that the selling price of the Philippines v. Manalo:[45]
proposals, nothing more. It insists that a definite agreement shall be the total banks claim as of documentation date x x x
on the amount and manner of payment of the price are payable in cash (P725,000.00 already deposited) A definite agreement as to the price is an essential element of
essential elements in the formation of a binding and a binding agreement to sell personal or real property because
enforceable contract of sale. There was no such agreement within 60 days from notice of approval. A new Statement of it seriously affects the rights and obligations of the parties.
in this case. Primarily, the concept of suspensive condition Account was attached therein indicating the total banks claim Price is an essential element in the formation of a binding and
signifies a future and uncertain event upon the fulfillment of to be P1,931,389.53 less deposit of P725,000.00, or enforceable contract of sale. The fixing of the price can never
which the obligation becomes effective. It clearly P1,206,389.00. Furthermore, while respondents Board of be left to the decision of one of the contracting parties. But a
presupposes the existence of a valid and binding agreement, Directors accepted petitioners offer to repurchase the price fixed by one of the contracting parties, if accepted by
the effectivity of which is subordinated to its fulfillment. Since property, the acceptance was qualified, in that it required a the other, gives rise to a perfected sale.[46]
there is no perfected contract in the first place, there is no higher sale price and subject to specified terms and
basis for the application of the principles governing conditions enumerated therein. This qualified acceptance was A contract of sale is consensual in nature and is perfected
suspensive conditions. in effect a counter-offer, necessitating petitioners acceptance upon mere meeting of the minds. When there is merely an
in return. offer by one party without acceptance of the other, there is no
According to respondent, the Statement of Account prepared contract.[47] When the contract of sale is not perfected, it
by SAMD as of June 25, 1984 cannot be classified as a The Ruling of the Court cannot, as an independent source of obligation, serve as a
counter-offer; it is simply a recital of its total monetary claims binding juridical relation between the parties.[48]
against petitioner. Moreover, the amount stated therein could The ruling of the appellate court that there was no perfected
not likewise be considered as the counter-offer since as contract of sale between the parties on June 4, 1985 is In San Miguel Properties Philippines, Inc. v. Huang,[49] the
admitted by petitioner, it was only recommendation which correct. Court ruled that the stages of a contract of sale are as
was subject to approval of the PNB Board of Directors. follows: (1) negotiation, covering the period from the time the
A contract is a meeting of minds between two persons prospective contracting parties indicate interest in the
Neither can the receipt by the SAMD of P725,000.00 be whereby one binds himself, with respect to the other, to give contract to the time the contract is perfected; (2) perfection,
regarded as evidence of a perfected sale contract. As something or to render some service.[41] Under Article 1318 which takes place upon the concurrence of the essential
gleaned from the parties Stipulation of Facts during the of the New Civil Code, there is no contract unless the elements of the sale which are the meeting of the minds of
proceedings in the court a quo, the amount is merely an following requisites concur: the parties as to the object of the contract and upon the price;
acknowledgment of the receipt of P725,000.00 as deposit to and (3) consummation, which begins when the parties
repurchase the property. The deposit of P725,000.00 was (1) Consent of the contracting parties; perform their respective undertakings under the contract of
accepted by respondent on the condition that the purchase sale, culminating in the extinguishment thereof.
price would still be approved by its Board of Directors. (2) Object certain which is the subject matter of the contract;
Respondent maintains that its acceptance of the amount was A negotiation is formally initiated by an offer, which, however,
qualified by that condition, thus not absolute. Pending such (3) Cause of the obligation which is established. must be certain.[50] At any time prior to the perfection of the
approval, it cannot be legally claimed that respondent is contract, either negotiating party may stop the negotiation. At
already bound by any contract of sale with petitioner. Contracts are perfected by mere consent which is manifested this stage, the offer may be withdrawn; the withdrawal is
by the meeting of the offer and the acceptance upon the thing effective immediately after its manifestation. To convert the
According to respondent, petitioner knew that the SAMD has and the cause which are to constitute the contract.[42] Once offer into a contract, the acceptance must be absolute and
no capacity to bind respondent and that its authority is limited perfected, they bind other contracting parties and the must not qualify the terms of the offer; it must be plain,
to administering, managing and preserving the properties and obligations arising therefrom have the form of law between unequivocal, unconditional and without variance of any sort
other special assets of PNB. The SAMD does not have the the parties and should be complied with in good faith. The from the proposal. In Adelfa Properties, Inc. v. Court of
power to sell, encumber, dispose of, or otherwise alienate the parties are bound not only to the fulfillment of what has been Appeals,[51] the Court ruled that:
assets, since the power to do so must emanate from its expressly stipulated but also to the consequences which,
98
Sales – Chapter 3-12 Cases
x x x The rule is that except where a formal acceptance is so petitioner had accepted this counter-offer, a perfected
required, although the acceptance must be affirmatively and The statement of account prepared by the SAMD stating that contract of sale would have arisen; as it turns out, however,
clearly made and must be evidenced by some acts or the net claim of respondent as of June 25, 1984 was petitioner merely sought to have the counter-offer
conduct communicated to the offeror, it may be shown by P1,574,560.47 cannot be considered an unqualified reconsidered. This request for reconsideration would later be
acts, conduct, or words of the accepting party that clearly acceptance to petitioners offer to purchase the property. The rejected by respondent.
manifest a present intention or determination to accept the statement is but a computation of the amount which petitioner
offer to buy or sell. Thus, acceptance may be shown by the was obliged to pay in case respondent would later agree to We do not agree with petitioners contention that the
acts, conduct, or words of a party recognizing the existence sell the property, including interests, advances on insurance P725,000.00 it had remitted to respondent was earnest
of the contract of sale.[52] premium, advances on realty taxes, publication cost, money which could be considered as proof of the perfection
registration expenses and miscellaneous expenses. of a contract of sale under Article 1482 of the New Civil Code.
A qualified acceptance or one that involves a new proposal The provision reads:
constitutes a counter-offer and a rejection of the original offer. There is no evidence that the SAMD was authorized by
A counter-offer is considered in law, a rejection of the original respondents Board of Directors to accept petitioners offer and ART. 1482. Whenever earnest money is given in a contract of
offer and an attempt to end the negotiation between the sell the property for P1,574,560.47. Any acceptance by the sale, it shall be considered as part of the price and as proof of
parties on a different basis.[53] Consequently, when SAMD of petitioners offer would not bind respondent. As this the perfection of the contract.
something is desired which is not exactly what is proposed in Court ruled in AF Realty Development, Inc. vs. Diesehuan
the offer, such acceptance is not sufficient to guarantee Freight Services, Inc.:[60]
consent because any modification or variation from the terms This contention is likewise negated by the stipulation of facts
of the offer annuls the offer.[54] The acceptance must be which the parties entered into in the trial court:
identical in all respects with that of the offer so as to produce Section 23 of the Corporation Code expressly provides that
consent or meeting of the minds. the corporate powers of all corporations shall be exercised by 8. On June 8, 1984, the Special Assets Management
the board of directors. Just as a natural person may authorize Department (SAMD) of PNB prepared an updated Statement
In this case, petitioner had until February 17, 1984 within another to do certain acts in his behalf, so may the board of of Account showing MMCCs total liability to PNB as of June
which to redeem the property. However, since it lacked the directors of a corporation validly delegate some of its 25, 1984 to be P1,574,560.47 and recommended this amount
resources, it requested for more time to redeem/repurchase functions to individual officers or agents appointed by it. Thus, as the repurchase price of the subject property.
the property under such terms and conditions agreed upon by contracts or acts of a corporation must be made either by the
the parties.[55] The request, which was made through a letter board of directors or by a corporate agent duly authorized by 9. On June 25, 1984, MMCC paid P725,000.00 to PNB as
dated August 25, 1983, was referred to the respondents main the board. Absent such valid delegation/authorization, the deposit to repurchase the property. The deposit of P725,000
branch for appropriate action.[56] Before respondent could rule is that the declarations of an individual director relating to was accepted by PNB on the condition that the purchase
act on the request, petitioner again wrote respondent as the affairs of the corporation, but not in the course of, or price is still subject to the approval of the PNB Board.[62]
follows: connected with the performance of authorized duties of such
director, are held not binding on the corporation.
1. Upon approval of our request, we will pay your goodselves Thus, the P725,000.00 was merely a deposit to be applied as
ONE HUNDRED & FIFTY THOUSAND PESOS part of the purchase price of the property, in the event that
(P150,000.00); Thus, a corporation can only execute its powers and transact respondent would approve the recommendation of SAMD for
its business through its Board of Directors and through its respondent to accept petitioners offer to purchase the
2. Within six months from date of approval of our request, we officers and agents when authorized by a board resolution or property for P1,574,560.47. Unless and until the respondent
will pay another FOUR HUNDRED FIFTY THOUSAND its by-laws.[61] accepted the offer on these terms, no perfected contract of
PESOS (P450,000.00); and sale would arise. Absent proof of the concurrence of all the
It appears that the SAMD had prepared a recommendation essential elements of a contract of sale, the giving of earnest
3. The remaining balance together with the interest and other for respondent to accept petitioners offer to repurchase the money cannot establish the existence of a perfected contract
expenses that will be incurred will be paid within the last six property even beyond the one-year period; it recommended of sale.[63]
months of the one year grave period requested for.[57] that petitioner be allowed to redeem the property and pay
P1,574,560.00 as the purchase price. Respondent later It appears that, per its letter to petitioner dated June 4, 1985,
approved the recommendation that the property be sold to the respondent had decided to accept the offer to purchase
When the petitioner was told that respondent did not allow petitioner. But instead of the P1,574,560.47 recommended by the property for P1,931,389.53. However, this amounted to
partial redemption,[58] it sent a letter to respondents the SAMD and to which petitioner had previously conformed, an amendment of respondents qualified acceptance, or an
President reiterating its offer to purchase the property.[59] respondent set the purchase price at P2,660,000.00. In fine, amended counter-offer, because while the respondent
There was no response to petitioners letters dated February respondents acceptance of petitioners offer was qualified, lowered the purchase price, it still declared that its
10 and 15, 1984. hence can be at most considered as a counter-offer. If acceptance was subject to the following terms and conditions:
99
Sales – Chapter 3-12 Cases

SO ORDERED.

1. That the selling price shall be the total Banks claim


as of documentation date (pls. see attached statement of
account as of 5-31-85), payable in cash (P725,000.00 already
deposited) within sixty (60) days from notice of approval;

2. The Bank sells only whatever rights, interests and


participation it may have in the property and you are charged
with full knowledge of the nature and extent of said rights,
interests and participation and waive your right to warranty
against eviction.

3. All taxes and other government imposts due or to


become due on the property, as well as expenses including
costs of documents and science stamps, transfer fees, etc.,
to be incurred in connection with the execution and
registration of all covering documents shall be borne by you;

4. That you shall undertake at your own expense and


account the ejectment of the occupants of the property
subject of the sale, if there are any;

5. That upon your failure to pay the balance of the


purchase price within sixty (60) days from receipt of advice
accepting your offer, your deposit shall be forfeited and the
Bank is thenceforth authorized to sell the property to other
interested parties.

6. That the sale shall be subject to such other terms


and conditions that the Legal Department may impose to
protect the interest of the Bank.[64]

It appears that although respondent requested petitioner to


conform to its amended counter-offer, petitioner refused and
instead requested respondent to reconsider its amended
counter-offer. Petitioners request was ultimately rejected and
respondent offered to refund its P725,000.00 deposit.

In sum, then, there was no perfected contract of sale between


petitioner and respondent over the subject property.

IN LIGHT OF ALL THE FOREGOING, the petition is


DENIED.
The assailed decision is AFFIRMED. Costs against petitioner
Manila Metal Container Corporation.
100
Sales – Chapter 3-12 Cases
FIRST OPTIMA REALTY CORPORATION, Petitioner, Sometime thereafter, Eleazar personally went to petitioner’s
vs. office offering to pay for the subject property in cash, which The check was eventually deposited with and credited to
SECURITRON SECURITY SERVICES, INC., Respondent. he already brought with him. However, Young declined to petitioner’s bank account.
accept payment, saying that she still needed to secure her
In a potential sale transaction, the prior payment of earnest sister’s advice on the matter.10 She likewise informed Thereafter, respondent through counsel demanded in writing
money even before the property owner can agree to sell his Eleazar that prior approval of petitioner’s Board of Directors that petitioner proceed with the sale of the property.16 In a
property is irregular, and cannot be used to bind the owner to was required for the transaction, to which remark Eleazar March 3, 2006 Letter17 addressed to respondent’s counsel,
the obligations of a seller under an otherwise perfected replied that respondent shall instead await such approval.11 petitioner wrote back:
contract of sale; to cite a well-worn cliche, the carriage cannot
be placed before the horse. The property owner-prospective On February 4, 2005, respondent sent a Letter12 of even Dear Atty. De Jesus:
seller may not be legally obliged to enter into a sale with a date to petitioner. It was accompanied by Philippine National
prospective buyer through the latter's employment of Bank Check No. 24677 (the subject check), issued for Anent your letter dated January 16, 2006 received on
questionable practices which prevent the owner from freely ₱100,000.00 and made payable to petitioner. The letter February 20, 2006, please be informed of the following:
giving his consent to the transaction; this constitutes a states thus:
palpable transgression of the prospective seller's rights of 1. It was your client SECURITRON SECURITY SERVICES,
ownership over his property, an anomaly which the Court will Gentlemen: INC. represented by Mr. Antonio Eleazar who offered to buy
certainly not condone. our property located at corner Layug and Lim-An St., Pasay
As agreed upon, we are making a deposit of ONE HUNDRED City;
This Petition for Review on Certiorari1 seeks to set aside: 1) THOUSAND PESOS (Php 100,000.00) as earnest money for
the September 30, 2011 Decision2 of the Court of Appeals your property at the corner of Layug St., & Lim-An St., Pasay 2. It tendered an earnest money despite the fact that we are
(CA) in CA-G.R. CV No. 93715 affirming the February 16, City as per TCT No. 125318 with an area of 256 sq. m. at still undecided to sell the said property;
2009 Decision' of the Regional Trial Court (RTC) of Pasay 6,000.00/ sq. m. for a total of ONE MILLION FIVE HUNDRED
City, Branch 115 in Civil Case No. 06-0492 CFM; and 2) the THIRTY SIX THOUSAND PESOS (Php 1,536,000.00). 3. Our Board of Directors failed to pass a resolution to date
CA’s December 9, 2011 Resolution4 denying the herein whether it agrees to sell the property;
petitioner’s Motion for Reconsideration5 of the assailed Full payment upon clearing of the tenants at said property
judgment. and signing of the Deed of Sale. 4. We have no Contract for the earnest money nor Contract
to Sell the said property with your client;
Factual Antecedents (signed)
ANTONIO S. ELEAZAR13 Considering therefore the above as well as due to haste and
Petitioner First Optima Realty Corporation is a domestic demands which we feel [are forms] of intimidation and
corporation engaged in the real estate business. It is the Despite the delicate nature of the matter and large amount harassment, we regret to inform you that we are now incline
registered owner of a 256-square meter parcel of land with involved, respondent did not deliver the letter and check (sic) not to accept your offer to buy our property. Please
improvements located in Pasay City, covered by Transfer directly to Young or her office; instead, they were coursed inform your client to coordinate with us for the refund of this
Certificate of Title No. 125318 (the subject property).6 through an ordinary receiving clerk/receptionist of the (sic) money.
Respondent Securitron Security Services, Inc., on the other petitioner, who thus received the same and therefor issued
hand, is a domestic corporation with offices located beside and signed Provisional Receipt No. 33430.14 The said Very truly yours,
the subject property. receipt reads:
(signed)
Looking to expand its business and add toits existing offices, Received from x x x Antonio Eleazar x x x the sum of Pesos CAROLINA T. YOUNG
respondent – through its General Manager, Antonio Eleazar One Hundred Thousand x x x Executive Vice[-]President18
(Eleazar) – sent a December 9, 2004 Letter7 addressed to
petitioner – through its Executive Vice-President, Carolina T. IN PAYMENT OF THE FOLLOWING x x x Ruling of the Regional Trial Court of Pasay City
Young (Young) – offering to purchase the subject property at
₱6,000.00 per square meter. A series of telephone calls Earnest money or Partial payment of On April 18, 2006, respondent filed with the Pasay RTC a civil
ensued, but only between Eleazar and Young’s secretary;8 case against petitioner for specific performance with
Eleazar likewise personally negotiated with a certain Maria Pasay Property Layug & Lim-an St. x x x. damages to compel the latter to consummate the supposed
Remoso (Remoso), who was an employee of petitioner.9 At sale of the subject property. Docketed as Civil Case No. 06-
this point, Eleazar was unable to personally negotiate with Note: This is issued to transactions not 0492 CFM and assigned to Branch 115 of the Pasay RTC,
Young or the petitioner’s board of directors. yet cleared but subsequently an OfficialReceipt will be issued. the Complaint19 is predicated on the claim that since a
x x x15 perfected contract of sale arose between the parties after
101
Sales – Chapter 3-12 Cases
negotiations were conducted and respondent paid the SO ORDERED.22 Upon a careful consideration of the arguments of the parties
₱100,000.00 supposed earnest money – which petitioner and the records of the case, we are more inclined to sustain
accepted, the latter should be compelled to sell the subject In ruling for the respondent, the trial court held that the arguments of the plaintiff-appellee and affirm the findings
property to the former. Thus, respondent prayed that petitioner’s acceptance of ₱100,000.00 earnest money of the trial court that there was indeed a perfected contract of
petitioner be ordered to comply with its obligation as seller, indicated the existence of a perfected contract of sale sale between the parties. The following instances militate
accept the balance of the purchase price, and execute the between the parties; that there is no showing that when against the claim of the defendant-appellant: First. The letter
corresponding deed of sale in respondent’s favor; and that respondent gave the February 4, 2005 letter and check to of the plaintiff-appellee dated February 4, 2005 reiterating
petitioner be made to pay ₱200,000.00 damages for its petitioner’s receiving clerk, the latter was harassed or forced their agreement as to the sale of the realty for the
breach and delay in the performance of its obligations, to accept the same; and that for the sale of the subject consideration of Php 1,536,000.00 was not disputed nor
₱200,000.00 by way of attorney's fees, and costs of suit. property, no resolution of petitioner’s board of directors was replied to by the defendant-appellant, the said letter also
required since Young was "free to represent" the corporation provides for the payment of the earnest money of Php
In its Answer with Compulsory Counterclaim,20 petitioner in negotiating with respondent for the sale thereof. Ruling of 100,000.00 and the full payment upon the clearing of the
argued that it never agreed to sell the subject property; that the Court of Appeals property of unwanted tenants, if the defendant-appellant did
its board of directors did not authorize the sale thereof to not really agree on the sale of the property it could have
respondent, as no corresponding board resolution to such Petitioner filed an appeal with the CA. Docketed as CA-G.R. easily replied to the said letter informing the plaintiff-appellee
effect was issued; that the respondent’s ₱100,000.00 check CV No. 93715, the appeal made out a case that no earnest that it is not selling the property or that the matter will be
payment cannot be considered as earnest money for the money can be considered to have been paid to petitioner as decided first by the board of directors, defendant-appellant’s
subject property, since said payment was merely coursed the supposed payment was received by a mere receiving silence or inaction on said letter shows its conformity or
through petitioner’s receiving clerk, who was forced to accept clerk, who was not authorized to accept the same; that the consent thereto; Second. In addition to the aforementioned
the same; and that respondent was simply motivated by a required board of directors resolution authorizing the sale of letter, defendant-appellant’s acceptance of the earnest
desire to acquire the subject property at any cost. Thus, corporate assets cannot be dispensed with in the case of money and the issuance of a provisional receipt clearly
petitioner prayed for the dismissal of the case and, by way of petitioner; that whatever negotiations were held between the shows that there was indeed an agreement between the
counterclaim, it sought the payment of moral damages in the parties only concerned the possible sale, not the sale itself, of parties and we do not subscribe to the argument of the
amount of ₱200,000.00; exemplary damages in the amount the subject property; that without the written authority of defendant-appellant that the check was merely forced upon
of ₱100,000.00; and attorney’s fees and costs of suit. petitioner’s board of directors, Young cannot enter into a sale its employee and the contents of the receipt was just dictated
of its corporate property; and finally, that there was no by the plaintiff-appellee’s employee because common sense
In a Reply,21 respondent countered that authorization by meeting of the minds between the parties in the first place. dictates that a person would not issue a receipt for a check
petitioner’s Board of Directors was not necessary since it is a with a huge amount if she does not know what that is for and
real estate corporation principally engaged in the buying and On September 30, 2011, the CA issued the assailed Decision similarly would not issue [a] receipt which would bind her
selling of real property; that respondent did not force nor affirming the trial court’s February 16, 2009Decision, employer if she does not have prior instructions to do [so]
intimidate petitioner’s receiving clerk into accepting the pronouncing thus: from her superiors; Third. The said check for earnest money
February 4, 2005 letter and check for ₱100,000.00; that was deposited in the bank by defendant-appellant and not
petitioner’s acceptance of the check and its failure – for more Article 1318 of the Civil Code declares that no contract exists until after one year did it offer to return the same. Defendant-
than a year – to return respondent’s payment amounts to unless the following requisites concur: (1) consent of the appellant cannot claim lack of knowledge of the payment of
estoppel and a ratification of the sale; and that petitioner is contracting parties; (2) object certain which is the subject the check since there was a letter for it, and it is just
not entitled to its counterclaim. matter of the contract; and (3) cause of the obligation incredible that a big amount of money was deposited in [its]
established. account [without knowing] about it [or] investigat[ing] what [it
After due proceedings were taken, the Pasay RTC issued its was] for. We are more inclined to believe that their inaction
Decision dated February 16, 2009, decreeing as follows: A careful perusal of the records of the case show[s] that there for more than one year on the earnest money paid was due to
was indeed a negotiation between the parties as regards the the fact that after the payment of earnest money the place
WHEREFORE, defendant First Optima Realty Corporation is sale of the subject property, their disagreement lies on should be cleared of unwanted tenants before the full amount
directed to comply with its obligation by accepting the whether they have arrived on an agreement regarding said of the purchase price will be paid as agreed upon as shown in
remaining balance of One Million Five Hundred Thirty-Six sale. Plaintiff-appellee avers that the parties have already the letter sent by the plaintiff-appellee.
Thousand Pesos and Ninety-Nine Centavos (₱1,536,000.99), agreed on the sale and the price for it and the payment of
and executing the corresponding deed of sale in favor of the earnest money and the remaining balance upon clearing of As stated above the presence of defendant-appellant’s
plaintiff Securitron Security Services, Inc. over the subject the property of unwanted tenants. Defendant-appellant on the consent and, corollarily, the existence of a perfected contract
parcel of land. other hand disputes the same and insists that there was no between the parties are evidenced by the payment and
concrete agreement between the parties. receipt of Php 100,000.00 as earnest money by the
No costs. contracting parties’ x x x. Under the law on sales, specifically
Article 1482 of the Civil Code, it provides that whenever
102
Sales – Chapter 3-12 Cases
earnest money is given in a contract of sale, it shall be Furthermore, assuming arguendo that a board resolution was
considered as part of the price and proof of the perfection of indeed needed for the sale of the subject property, the THE HONORABLE COURT OF APPEALS COMMITTED
the contract. Although the presumption is not conclusive, as defendant-appellant is estopped from raising it now since, [it] SERIOUS AND GRAVE ERROR WHEN IT IGNOREDTHE
the parties may treat the earnest money differently, there is did not inform the plaintiff-appellee of the same, and the latter RESERVATION IN THE PROVISIONAL RECEIPT – "Note:
nothing alleged in the present case that would give rise to a deal (sic) with them in good faith. Also it must be stressed This is issued to transactions not yet cleared but
contrary presumption. that the plaintiff-appellee negotiated with one of the top officer subsequently an Official Receipt will be issued."26
(sic) of the company thus, any requirement on the said sale
We also do not find merit in the contention of the defendant- must have been known to Ms. Young and she should have Petitioner’s Arguments
appellant that there is a need for a board resolution for them informed the plaintiff-appellee of the same.
to sell the subject property since it is a corporation, a juridical In its Petition and Reply27 seeking to reverse and set aside
entity which acts only thru the board of directors. While we In view of the foregoing we do not find any reason to deviate the assailed CA dispositions and in effect to dismiss Civil
agree that said rule is correct, we must also point out that from the findings of the trial court, the parties entered into the Case No. 06-0492 CFM, petitioner argues that respondent
said rule is the general rule for all corporations [but] a contract freely, thus they must perform their obligation failed to prove its case that a contract of sale was perfected
corporation [whose main business is buying and selling real faithfully. Defendant-appellant’s unjustified refusal to perform between the parties. It particularly notes that, contrary to the
estate] like herein defendant-appellant, is not required to its part of the agreement constitutes bad faith and the court CA’s ruling, respondent’s delivery of the February 4, 2005
have a board resolution for the sale of the realty in the will not tolerate the same. letter and check; petitioner’s failure to respond to said letter;
ordinary course of business, thus defendant-appellant’s claim petitioner’s supposed acceptance of the check by depositing
deserves scant consideration. WHEREFORE, premises considered, the Decision of the the same in its account; and its failure to return the same
Regional Trial Court of Pasay City Branch 115, in Civil Case after more than one year from its tender – these
Furthermore, the High Court has held that "a corporate officer No. 06-0492 CFM is hereby AFFIRMED. circumstances do not at all prove that a contract of sale was
or agent may represent and bind the corporation in perfected between the parties. It claims that there was never
transactions with third persons to the extent that the authority SO ORDERED.23 an agreement in the first place between them concerning the
to do so has been conferred upon him, and this includes sale of the subject property, much less the payment of
powers which have been intentionally conferred, and also Petitioner moved for reconsideration,24 but in a December 9, earnest money therefor; that during trial, Eleazar himself
such powers as, in the usual course of the particular 2011 Resolution, the CA held its ground. Hence, the present admitted that the check was merely a "deposit";28 that the
business, are incidental to, or may be implied from, the Petition. February 4, 2005 letter and check were delivered not to
powers intentionally conferred, powers added by custom and Young, but to a mere receiving clerk of petitioner who knew
usage, as usually pertaining to the particular officer or agent, Issues nothing about the supposed transaction and was simply
and such apparent powers as the corporation has caused obliged to accept the same without the prerogative to reject
persons dealing with the officer or agent to believe that it was In an October 9,2013 Resolution,25 this Court resolved to them; that the acceptance of respondent’s supposed payment
conferred." give due course to the Petition, which raises the following was not cleared and was subject to approval and issuance of
issues: the corresponding official receipt as noted in Provisional
In the case at bench, it is not disputed and in fact was Receipt No. 33430; that respondent intentionally delivered the
admitted by the defendant-appellant that Ms. Young, the I letter and check in the manner that it did in order to bind
Executive Vice-President was authorized to negotiate for the petitioner to the supposed sale with or without the latter’s
possible sale of the subject parcel of land. Therefore, Ms. THE HONORABLE COURT OF APPEALS ERRED ON A consent; that petitioner could not be faulted for receiving the
Young can represent and bind defendant-appellant in the QUESTION OF LAW WHEN IT RULED THAT THE MONEY check and for depositing the same as a matter of operational
transaction. RESPONDENT DELIVERED TO PETITIONER WAS procedure with respect to checks received in the course of its
EARNEST MONEY THEREBY PROVIDING A PERFECTED day-to-day business.
Moreover, plaintiff-appellee can assume that Ms. Young, by CONTRACT OF SALE.
virtue of her position, was authorized to sell the property of Petitioner argues that ultimately, it cannot be said that it gave
the corporation. Selling of realty is not foreign to [an] II its consent to any transaction with respondent or to the
executive vice[-]president’s function, and the real estate sale payment made by the latter. Respondent’s letter and check
was shown to be a normal business activity of defendant- THE HONORABLE COURT OF APPEALS ERRED ON A constitute merely an offer which required petitioner’s
appellant since its primary business is the buy and sell of real QUESTION OF LAW WHEN IT RULED THAT THE TIME acceptance in order to give rise to a perfected sale;
estate. Unmistakably, its Executive Vice-President is cloaked THAT LAPSED IN RETURNING THE MONEY AND IN "[o]therwise, a buyer can easily bind any unsuspecting seller
with actual or apparent authority to buy or sell real property, REPLYING TO THE LETTER IS PROOF OF ACCEPTANCE to a contract of sale by merely devising a way that prevents
an activity which falls within the scope of her general OF EARNEST MONEY. the latter from acting on the communicated offer."29
authority.
III
103
Sales – Chapter 3-12 Cases
Petitioner thus theorizes that since it had no perfected Instead, Young informed Eleazar during said meeting that under Art. 1482 of the Civil Code, "there must first be a
agreement with the respondent, the latter’s check should be she still had to confer with her sister and petitioner’s board of perfected contract of sale before we can speak of earnest
treated not as earnest money, but as mere guarantee, directors; in turn, Eleazar told Young that respondent shall money."35 "Where the parties merely exchanged offers and
deposit or option money to prevent the prospective seller await the necessary approval. counter-offers, no contract is perfected since they did not yet
from backing out from the sale,30 since the payment of any give their consent to such offers. Earnest money applies to a
consideration acquires the character of earnest money only Thus, the trial and appellate courts failed to appreciate that perfected sale."36
after a perfected sale between the parties has been arrived respondent’s offer to purchase the subject property was
at.31 never accepted by the petitioner at any instance, even after This Court is inclined to accept petitioner’s explanation that
negotiations were held between them. Thus, as between since the check was mixed up with all other checks and
Respondent’s Arguments them, there is no sale to speak of. "When there is merely an correspondence sent to and received by the corporation
offer by one party without acceptance of the other, there is no during the course of its daily operations, Young could not
In its Comment,32 respondent counters that petitioner’s case contract."33 To borrow a pronouncement in a previously have timely discovered respondent’s check payment;
typifies a situation where the seller has had an undue change decided case, petitioner’s failure to return the purported earnest money
of mind and desires to escape the legal consequences cannot mean that it agreed to respondent’s offer.
attendant to a perfected contract of sale. It reiterates the The stages of a contract of sale are: (1) negotiation, starting
appellate court’s pronouncements that petitioner’s failure to from the time the prospective contracting parties indicate Besides, respondent’s payment of supposed earnest money
reply to respondent’s February 4, 2005 letter indicates its interest in the contract to the time the contract is perfected; was made under dubious circumstances and in disregard of
consent to the sale; that its acceptance of the check as (2) perfection, which takes place upon the concurrence of the sound business practice and common sense. Indeed,
earnest money and the issuance of the provisional receipt essential elements of the sale; and (3) consummation, which respondent must be faulted for taking such a course of action
prove that there is a prior agreement between the parties; commences when the parties perform their respective that is irregular and extraordinary: common sense and logic
that the deposit of the check in petitioner’s account and undertakings under the contract of sale, culminating in the dictate that if any payment is made under the supposed sale
failure to timely return the money to respondent militates extinguishment of the contract. transaction, it should have been made directly to Young or
against petitioner’s claim of lack of knowledge and consent. coursed directly through her office, since she is the officer
Rather they indicate petitioner’s decision to sell subject In the present case, the parties never got past the negotiation directly responsible for negotiating the sale, as far as
property as agreed. Respondent adds that contrary to stage. Nothing shows that the parties had agreed on any final respondent is concerned and considering the amount of
petitioner’s claim, negotiations were in fact held between the arrangement containing the essential elements of a contract money involved; no other ranking officer of petitioner can be
parties after it sent its December 9, 2004 letter-offer, which of sale, namely, (1) consent or the meeting of the minds of expected to know of the ongoing talks covering the subject
negotiations precisely culminated in the preparation and the parties; (2) object or subject matter of the contract; and property. Respondent already knew, from Eleazar’s previous
issuance of the February4, 2005 letter; that petitioner’s failure (3) price or consideration of the sale.34 meeting with Young, that it could only effectively deal with
to reply to its February 4, 2005 letter meant that it was her; more than that, it should know that corporations work
amenable to respondent’s terms; that the issuance of a Respondent’s subsequent sending of the February 4, 2005 only through the proper channels. By acting the way it did –
provisional receipt does not prevent the perfection of the letter and check to petitioner – without awaiting the approval coursing the February 4, 2005 letter and check through
agreement between the parties, since earnest money was of petitioner’s board of directors and Young’s decision, or petitioner’s mere receiving clerk or receptionist instead of
already paid; and that petitioner cannot pretend to be ignorant without making a new offer – constitutes a mere reiteration of directly with Young’s office, respondent placed itself under
of respondent’s check payment, as it involved a large sum of its original offer which was already rejected previously; thus, grave suspicion of putting into effect a premeditated plan to
money that was deposited in the former’s bank account. petitioner was under no obligation to reply to the February 4, unduly bind petitioner to its rejected offer, in a manner which
2005 letter. It would be absurd to require a party to reject the it could not achieve through negotiation and employing
Our Ruling very same offer each and every time it is made; otherwise, a normal business practices. It impresses the Court that
perfected contract of sale could simply arise from the failure respondent attempted to secure the consent needed for the
The Court grants the Petition. The trial and appellate courts to reject the same offer made for the hundredth time.1âwphi1 sale by depositing part of the purchase price and under the
erred materially in deciding the case; they overlooked Thus, said letter cannot be considered as evidence of a false pretense that an agreement was already arrived at,
important facts that should change the complexion and perfected sale, which does not exist in the first place; no even though there was none. Respondent achieved the
outcome of the case. binding obligation on the part of the petitioner to sell its desired effect up to this point, but the Court will not be fooled.
property arose as a consequence. The letter made no new
It cannot be denied that there were negotiations between the offer replacing the first which was rejected. Thus, as between respondent’s irregular and improper
parties conducted after the respondent’s December 9, 2004 actions and petitioner’s failure to timely return the
letter-offer and prior to the February 4, 2005 letter. These Since there is no perfected sale between the parties, ₱100,000.00 purported earnest money, this Court sides with
negotiations culminated in a meeting between Eleazar and respondent had no obligation to make payment through the petitioner. In a manner of speaking, respondent cannot fault
Young whereby the latter declined to enter into an agreement check; nor did it possess the right to deliver earnest money to petitioner for not making a refund since it is equally to blame
and accept cash payment then being tendered by the former. petitioner in order to bind the latter to a sale. As contemplated for making such payment under false pretenses and irregular
104
Sales – Chapter 3-12 Cases
circumstances, and with improper motives. Parties must ordered to REFUND the amount of ₱100,000.00 to
come to court with clean hands, as it were. respondent Securitron Security Services, Inc. without interest,
unless petitioner has done so during the course of the
In a potential sale transaction, the prior payment of earnest proceedings.
money even before the property owner can agree to sell his
property is irregular, and cannot be used to bind the owner to SO ORDERED.
the obligations of a seller under an otherwise perfected
contract of sale; to cite a well-worn cliché, the carriage cannot
be placed before the horse. The property owner-prospective
seller may not be legally obliged to enter into a sale with a
prospective buyer through the latter’s employment of
questionable practices which prevent the owner from freely
giving his consent to the transaction; this constitutes a
palpable transgression of the prospective seller’s rights of
ownership over his property, an anomaly which the Court will
certainly not condone. An agreement where the prior free
consent of one party thereto is withheld or suppressed will be
struck down, and the Court shall always endeavor to protect a
property owner’s rights against devious practices that put his
property in danger of being lost or unduly disposed without
his prior knowledge or consent. As this ponente has held
before, "[t]his Court cannot presume the existence of a sale of
land, absent any direct proof of it."37

Nor will respondent's supposed payment be 'treated as a


deposit or guarantee; its actions will not be dignified and must
be called for what they are: they were done irregularly and
with a view to acquiring the subject property against
petitioner's consent.

Finally, since there is nothing in legal contemplation which


petitioner must perform particularly for the respondent, it
should follow that Civil Case No. 06-0492 CFM for specific
performance with damages is left with no leg. to stand on; it
must be dismissed.

With the foregoing view, there is no need to resolve the other


specific issues and arguments raised by the petitioner, as
they do not materially affect the rights and obligations of the
parties - the Court having declared that no agreement exists
between them; nor do they have the effect of altering the
outcome of the case.

WHEREFORE, the Petition is GRANTED. The September 30,


2011 Decision and December 9, 2011 Resolution of the Court
of Appeals in CA-G.R. CV No. 93715, as well as the February
16, 2009 Decision of the Regional Trial Court of Pasay City,
Branch 115 in Civil Case No. 06-0492 CFM are REVERSED
and SET ASIDE. Civil Case No. 06-0492 CFM is ordered
DISMISSED. , Petitioner First Optima Realty Corporation is
105
Sales – Chapter 3-12 Cases
RIZALINO, substituted by his heirs, JOSEFINA, died, petitioners, together with Adolfo and Jesus, acquired Oesmer, and his children, Rolando O. Oesmer and
ROLANDO and FERNANDO, ERNESTO, LEONORA, the lots as heirs of the former by right of succession. Fernando O. Oesmer. However, the name of Rizalino was
BIBIANO, JR., LIBRADO and ENRIQUETA, all retained in the title of the case both in the RTC and the
surnamed OESMER, Respondent Paraiso Development Corporation is known Court of Appeals.
Petitioners, to be engaged in the real estate business.
- versus - After trial on the merits, the lower court rendered a
PARAISO DEVELOPMENT CORPORATION, Sometime in March 1989, Rogelio Paular, a resident and Decision[9] dated 27 March 1996 in favor of the
Respondent. former Municipal Secretary of Carmona, Cavite, brought respondent, the dispositive portion of which reads:
along petitioner Ernesto to meet with a certain Sotero Lee,
Before this Court is a Petition for Review on Certiorari President of respondent Paraiso Development WHEREFORE, premises considered, judgment is hereby
under Rule 45 of the 1997 Revised Rules of Civil Corporation, at Otani Hotel in Manila. The said meeting rendered in favor of herein [respondent] Paraiso
Procedure seeking to reverse and set aside the Court of was for the purpose of brokering the sale of petitioners Development Corporation. The assailed Contract to Sell is
Appeals Decision[1] dated 26 April 2002 in CA-G.R. CV properties to respondent corporation. valid and binding only to the undivided proportionate
No. 53130 entitled, Rizalino, Ernesto, Leonora, Bibiano, share of the signatory of this document and recipient of
Jr., Librado, Enriqueta, Adolfo, and Jesus, all surnamed Pursuant to the said meeting, a Contract to Sell[5] was the check, [herein petitioner] co-owner Ernesto Durumpili
Oesmer vs. Paraiso Development Corporation, as drafted by the Executive Assistant of Sotero Lee, Oesmer. The latter is hereby ordered to execute the
modified by its Resolution[2] dated 4 March 2003, Inocencia Almo. On 1 April 1989, petitioners Ernesto and Contract of Absolute Sale concerning his 1/8 share over
declaring the Contract to Sell valid and binding with Enriqueta signed the aforesaid Contract to Sell. A check in the subject two parcels of land in favor of herein
respect to the undivided proportionate shares of the six the amount of P100,000.00, payable to Ernesto, was [respondent] corporation, and to pay the latter the
signatories of the said document, herein petitioners, given as option money. Sometime thereafter, Rizalino, attorneys fees in the sum of Ten Thousand (P10,000.00)
namely: Ernesto, Enriqueta, Librado, Rizalino, Bibiano, Leonora, Bibiano, Jr., and Librado also signed the said Pesos plus costs of suit.
Jr., and Leonora (all surnamed Oesmer); and ordering Contract to Sell. However, two of the brothers, Adolfo and
them to execute the Deed of Absolute Sale concerning Jesus, did not sign the document. The counterclaim of [respondent] corporation is hereby
their 6/8 share over the subject parcels of land in favor of Dismissed for lack of merit.[10]
herein respondent Paraiso Development Corporation, and On 5 April 1989, a duplicate copy of the instrument was
to pay the latter the attorneys fees plus costs of the suit. returned to respondent corporation. On 21 April 1989,
The assailed Decision, as modified, likewise ordered the respondent brought the same to a notary public for Unsatisfied, respondent appealed the said Decision
respondent to tender payment to the petitioners in the notarization. before the Court of Appeals. On 26 April 2002, the
amount of P3,216,560.00 representing the balance of the appellate court rendered a Decision modifying the
purchase price of the subject parcels of land. In a letter[6] dated 1 November 1989, addressed to Decision of the court a quo by declaring that the Contract
The facts of the case are as follows: respondent corporation, petitioners informed the former of to Sell is valid and binding with respect to the undivided
their intention to rescind the Contract to Sell and to return proportionate shares of the six signatories of the said
Petitioners Rizalino, Ernesto, Leonora, Bibiano, Jr., the amount of P100,000.00 given by respondent as option document, herein petitioners, namely: Ernesto, Enriqueta,
Librado, and Enriqueta, all surnamed Oesmer, together money. Librado, Rizalino, Bibiano, Jr., and Leonora (all surnamed
with Adolfo Oesmer (Adolfo) and Jesus Oesmer (Jesus), Oesmer). The decretal portion of the said Decision states
are brothers and sisters, and the co-owners of undivided Respondent did not respond to the aforesaid letter. On 30 that:
shares of two parcels of agricultural and tenanted land May 1991, herein petitioners, together with Adolfo and
situated in Barangay Ulong Tubig, Carmona, Cavite, Jesus, filed a Complaint[7] for Declaration of Nullity or for WHEREFORE, premises considered, the Decision of the
identified as Lot 720 with an area of 40,507 square meters Annulment of Option Agreement or Contract to Sell with court a quo is hereby MODIFIED. Judgment is hereby
(sq. m.) and Lot 834 containing an area of 14,769 sq. m., Damages before the Regional Trial Court (RTC) of rendered in favor of herein [respondent] Paraiso
or a total land area of 55,276 sq. m. Both lots are Bacoor, Cavite. The said case was docketed as Civil Case Development Corporation. The assailed Contract to Sell is
unregistered and originally owned by their parents, No. BCV-91-49. valid and binding with respect to the undivided
Bibiano Oesmer and Encarnacion Durumpili, who proportionate share of the six (6) signatories of this
declared the lots for taxation purposes under Tax During trial, petitioner Rizalino died. Upon motion of document, [herein petitioners], namely, Ernesto,
Declaration No. 3438[3] (cancelled by I.D. No. 6064-A) for petitioners, the trial court issued an Order,[8] dated 16 Enriqueta, Librado, Rizalino, Bibiano, Jr., and Leonora (all
Lot 720 and Tax Declaration No. 3437[4] (cancelled by September 1992, to the effect that the deceased petitioner surnamed Oesmer). The said [petitioners] are hereby
I.D. No. 5629) for Lot 834. When the spouses Oesmer be substituted by his surviving spouse, Josefina O. ordered to execute the Deed of Absolute Sale concerning
106
Sales – Chapter 3-12 Cases
their 6/8 share over the subject two parcels of land and in II. On a question of law in not Art. 1874. When a sale of a piece of land or any interest
favor of herein [respondent] corporation, and to pay the holding that, the supposed Contract to Sell (Exhibit D) is therein is through an agent, the authority of the latter shall
latter the attorneys fees in the sum of Ten Thousand void altogether considering that respondent itself did not be in writing; otherwise, the sale shall be void.
Pesos (P10,000.00) plus costs of suit.[11] sign it as to indicate its consent to be bound by its terms.
Moreover, Exhibit D is really a unilateral promise to sell
without consideration distinct from the price, and hence, The law itself explicitly requires a written authority before
Aggrieved by the above-mentioned Decision, petitioners void. an agent can sell an immovable. The conferment of such
filed a Motion for Reconsideration of the same on 2 July an authority should be in writing, in as clear and precise
2002. Acting on petitioners Motion for Reconsideration, terms as possible. It is worth noting that petitioners
the Court of Appeals issued a Resolution dated 4 March Petitioners assert that the signatures of five of them signatures are found in the Contract to Sell. The Contract
2003, maintaining its Decision dated 26 April 2002, with namely: Enriqueta, Librado, Rizalino, Bibiano, Jr., and is absolutely silent on the establishment of any principal-
the modification that respondent tender payment to Leonora, on the margins of the supposed Contract to Sell agent relationship between the five petitioners and their
petitioners in the amount of P3,216,560.00, representing did not confer authority on petitioner Ernesto as agent to brother and co-petitioner Ernesto as to the sale of the
the balance of the purchase price of the subject parcels of sell their respective shares in the questioned properties, subject parcels of land. Thus, the Contract to Sell,
land. The dispositive portion of the said Resolution reads: and hence, for lack of written authority from the above- although signed on the margin by the five petitioners, is
named petitioners to sell their respective shares in the not sufficient to confer authority on petitioner Ernesto to
WHEREFORE, premises considered, the assailed subject parcels of land, the supposed Contract to Sell is act as their agent in selling their shares in the properties in
Decision is hereby modified. Judgment is hereby rendered void as to them. Neither do their signatures signify their question.
in favor of herein [respondent] Paraiso Development consent to directly sell their shares in the questioned
Corporation. The assailed Contract to Sell is valid and properties. Assuming that the signatures indicate consent, However, despite petitioner Ernestos lack of written
binding with respect to the undivided proportionate shares such consent was merely conditional. The effectivity of the authority from the five petitioners to sell their shares in the
of the six (6) signatories of this document, [herein alleged Contract to Sell was subject to a suspensive subject parcels of land, the supposed Contract to Sell
petitioners], namely, Ernesto, Enriqueta, Librado, Rizalino, condition, which is the approval of the sale by all the co- remains valid and binding upon the latter.
Bibiano, Jr., and Leonora (all surnamed Oesmer). The owners.
said [petitioners] are hereby ordered to execute the Deed As can be clearly gleaned from the contract itself, it is not
of Absolute Sale concerning their 6/8 share over the Petitioners also assert that the supposed Contract to Sell only petitioner Ernesto who signed the said Contract to
subject two parcels of land in favor of herein [respondent] (Exhibit D), contrary to the findings of the Court of Sell; the other five petitioners also personally affixed their
corporation, and to pay the latter attorneys fees in the sum Appeals, is not couched in simple language. signatures thereon. Therefore, a written authority is no
of Ten Thousand Pesos (P10,000.00) plus costs of suit. longer necessary in order to sell their shares in the subject
Respondent is likewise ordered to tender payment to the They further claim that the supposed Contract to Sell does parcels of land because, by affixing their signatures on the
above-named [petitioners] in the amount of Three Million not bind the respondent because the latter did not sign the Contract to Sell, they were not selling their shares through
Two Hundred Sixteen Thousand Five Hundred Sixty said contract as to indicate its consent to be bound by its an agent but, rather, they were selling the same directly
Pesos (P3,216,560.00) representing the balance of the terms. Furthermore, they maintain that the supposed and in their own right.
purchase price of the subject two parcels of land. [12] Contract to Sell is really a unilateral promise to sell and
the option money does not bind petitioners for lack of The Court also finds untenable the following arguments
cause or consideration distinct from the purchase price. raised by petitioners to the effect that the Contract to Sell
Hence, this Petition for Review on Certiorari. is not binding upon them, except to Ernesto, because: (1)
The Petition is bereft of merit. the signatures of five of the petitioners do not signify their
Petitioners come before this Court arguing that the Court consent to sell their shares in the questioned properties
of Appeals erred: It is true that the signatures of the five petitioners, namely: since petitioner Enriqueta merely signed as a witness to
Enriqueta, Librado, Rizalino, Bibiano, Jr., and Leonora, on the said Contract to Sell, and that the other petitioners,
I. On a question of law in not the Contract to Sell did not confer authority on petitioner namely: Librado, Rizalino, Leonora, and Bibiano, Jr., did
holding that, the supposed Contract to Sell (Exhibit D) is Ernesto as agent authorized to sell their respective shares not understand the importance and consequences of their
not binding upon petitioner Ernesto Oesmers co-owners in the questioned properties because of Article 1874 of action because of their low degree of education and the
(herein petitioners Enriqueta, Librado, Rizalino, Bibiano, the Civil Code, which expressly provides that: contents of the aforesaid contract were not read nor
Jr., and Leonora). explained to them; and (2) assuming that the signatures
indicate consent, such consent was merely conditional,
107
Sales – Chapter 3-12 Cases
thus, the effectivity of the alleged Contract to Sell was Enriqueta affixed her signature on the said contract when on the Contract to Sell and by taking receipt of the amount
subject to a suspensive condition, which is the approval the same was drafted. She even admitted that she of P100,000.00 which formed part of the purchase price.
by all the co-owners of the sale. understood the undertaking that she and petitioner
Ernesto made in connection with the contract. She xxxx
It is well-settled that contracts are perfected by mere likewise disclosed that pursuant to the terms embodied in
consent, upon the acceptance by the offeree of the offer the Contract to Sell, she updated the payment of the real As to [petitioner] Librado, the [appellate court] finds it
made by the offeror. From that moment, the parties are property taxes and transferred the Tax Declarations of the preposterous that he willingly affixed his signature on a
bound not only to the fulfillment of what has been questioned properties in her name.[15] Hence, it cannot document written in a language (English) that he
expressly stipulated but also to all the consequences be gainsaid that she merely signed the Contract to Sell as purportedly does not understand. He testified that the
which, according to their nature, may be in keeping with a witness because she did not only actively participate in document was just brought to him by an 18 year old niece
good faith, usage and law. To produce a contract, the the negotiation and execution of the same, but her named Baby and he was told that the document was for a
acceptance must not qualify the terms of the offer. subsequent actions also reveal an attempt to comply with check to be paid to him. He readily signed the Contract to
However, the acceptance may be express or implied. For the conditions in the said contract. Sell without consulting his other siblings. Thereafter, he
a contract to arise, the acceptance must be made known exerted no effort in communicating with his brothers and
to the offeror. Accordingly, the acceptance can be With respect to the other petitioners assertion that they did sisters regarding the document which he had signed, did
withdrawn or revoked before it is made known to the not understand the importance and consequences of their not inquire what the check was for and did not thereafter
offeror.[13] action because of their low degree of education and ask for the check which is purportedly due to him as a
because the contents of the aforesaid contract were not result of his signing the said Contract to Sell. (TSN, 28
In the case at bar, the Contract to Sell was perfected read nor explained to them, the same cannot be September 1993, pp. 22-23)
when the petitioners consented to the sale to the sustained.
respondent of their shares in the subject parcels of land The [appellate court] notes that Librado is a 43 year old
by affixing their signatures on the said contract. Such We only have to quote the pertinent portions of the Court family man (TSN, 28 September 1993, p. 19). As such, he
signatures show their acceptance of what has been of Appeals Decision, clear and concise, to dispose of this is expected to act with that ordinary degree of care and
stipulated in the Contract to Sell and such acceptance issue. Thus, prudence expected of a good father of a family. His
was made known to respondent corporation when the unwitting testimony is just divinely disbelieving.
duplicate copy of the Contract to Sell was returned to the First, the Contract to Sell is couched in such a simple
latter bearing petitioners signatures. language which is undoubtedly easy to read and The other [petitioners] (Rizalino, Leonora and Bibiano Jr.)
understand. The terms of the Contract, specifically the are likewise bound by the said Contract to Sell. The theory
As to petitioner Enriquetas claim that she merely signed amount of P100,000.00 representing the option money adopted by the [petitioners] that because of their low
as a witness to the said contract, the contract itself does paid by [respondent] corporation, the purchase price of degree of education, they did not understand the contents
not say so. There was no single indication in the said P60.00 per square meter or the total amount of of the said Contract to Sell is devoid of merit. The
contract that she signed the same merely as a witness. P3,316,560.00 and a brief description of the subject [appellate court] also notes that Adolfo (one of the co-
The fact that her signature appears on the right-hand properties are well-indicated thereon that any prudent and heirs who did not sign) also possess the same degree of
margin of the Contract to Sell is insignificant. The contract mature man would have known the nature and extent of education as that of the signing co-heirs (TSN, 15 October
indisputably referred to the Heirs of Bibiano and the transaction encapsulated in the document that he was 1991, p. 19). He, however, is employed at the Provincial
Encarnacion Oesmer, and since there is no showing that signing. Treasury Office at Trece Martirez, Cavite and has even
Enriqueta signed the document in some other capacity, it accompanied Rogelio Paular to the Assessors Office to
can be safely assumed that she did so as one of the Second, the following circumstances, as testified by the locate certain missing documents which were needed to
parties to the sale. witnesses and as can be gleaned from the records of the transfer the titles of the subject properties. (TSN, 28
case clearly indicate the [petitioners] intention to be bound January 1994, pp. 26 & 35) Similarly, the other co-heirs
Emphasis should also be given to the fact that petitioners by the stipulations chronicled in the said Contract to Sell. [petitioners], like Adolfo, are far from ignorant, more so,
Ernesto and Enriqueta concurrently signed the Contract to illiterate that they can be extricated from their obligations
Sell. As the Court of Appeals mentioned in its As to [petitioner] Ernesto, there is no dispute as to his under the Contract to Sell which they voluntarily and
Decision,[14] the records of the case speak of the fact that intention to effect the alienation of the subject property as knowingly entered into with the [respondent] corporation.
petitioner Ernesto, together with petitioner Enriqueta, met he in fact was the one who initiated the negotiation
with the representatives of the respondent in order to process and culminated the same by affixing his signature The Supreme Court in the case of Cecilia Mata v. Court of
finalize the terms and conditions of the Contract to Sell. Appeals (207 SCRA 753 [1992]), citing the case of Tan
108
Sales – Chapter 3-12 Cases
Sua Sia v. Yu Baio Sontua (56 Phil. 711), instructively WITNESS: No sir.
ruled as follows: ATTY. GAMO: What you are saying is that when your Therefore, this Court finds no error in the findings of the
brothers and sisters except Jesus and you did not sign Court of Appeals that all the petitioners who were
The Court does not accept the petitioners claim that she that agreement which had been marked as [Exhibit] D, signatories in the Contract to Sell are bound thereby.
did not understand the terms and conditions of the your brothers and sisters were grossly violating your
transactions because she only reached Grade Three and agreement. The final arguments of petitioners state that the Contract
was already 63 years of age when she signed the to Sell is void altogether considering that respondent itself
documents. She was literate, to begin with, and her age WITNESS: Yes, sir, they violated what we have agreed did not sign it as to indicate its consent to be bound by its
did not make her senile or incompetent. x x x. upon.[17] terms; and moreover, the Contract to Sell is really a
unilateral promise to sell without consideration distinct
At any rate, Metrobank had no obligation to explain the We also cannot sustain the allegation of the petitioners from the price, and hence, again, void. Said arguments
documents to the petitioner as nowhere has it been that assuming the signatures indicate consent, such must necessarily fail.
proven that she is unable to read or that the contracts consent was merely conditional, and that, the effectivity of
were written in a language not known to her. It was her the alleged Contract to Sell was subject to the suspensive The Contract to Sell is not void merely because it does not
responsibility to inform herself of the meaning and condition that the sale be approved by all the co-owners. bear the signature of the respondent corporation.
consequence of the contracts she was signing and, if she The Contract to Sell is clear enough. It is a cardinal rule in Respondent corporations consent to be bound by the
found them difficult to comprehend, to consult other the interpretation of contracts that if the terms of a terms of the contract is shown in the uncontroverted facts
persons, preferably lawyers, to explain them to her. After contract are clear and leave no doubt upon the intention which established that there was partial performance by
all, the transactions involved not only a few hundred or of the contracting parties, the literal meaning of its respondent of its obligation in the said Contract to Sell
thousand pesos but, indeed, hundreds of thousands of stipulation shall control.[18] The terms of the Contract to when it tendered the amount of P100,000.00 to form part
pesos. Sell made no mention of the condition that before it can of the purchase price, which was accepted and
become valid and binding, a unanimous consent of all the acknowledged expressly by petitioners. Therefore, by
As the Court has held: heirs is necessary. Thus, when the language of the force of law, respondent is required to complete the
contract is explicit, as in the present case, leaving no payment to enforce the terms of the contract. Accordingly,
x x x The rule that one who signs a contract is presumed doubt as to the intention of the parties thereto, the literal despite the absence of respondents signature in the
to know its contents has been applied even to contracts of meaning of its stipulation is controlling. Contract to Sell, the former cannot evade its obligation to
illiterate persons on the ground that if such persons are pay the balance of the purchase price.
unable to read, they are negligent if they fail to have the In addition, the petitioners, being owners of their
contract read to them. If a person cannot read the respective undivided shares in the subject properties, can As a final point, the Contract to Sell entered into by the
instrument, it is as much his duty to procure some reliable dispose of their shares even without the consent of all the parties is not a unilateral promise to sell merely because it
persons to read and explain it to him, before he signs it, co-heirs. Article 493 of the Civil Code expressly provides: used the word option money when it referred to the
as it would be to read it before he signed it if he were able amount of P100,000.00, which also form part of the
to do and his failure to obtain a reading and explanation of Article 493. Each co-owner shall have the full ownership purchase price.
it is such gross negligence as will estop from avoiding it of his part and of the fruits and benefits pertaining thereto,
on the ground that he was ignorant of its contents.[16] and he may therefore alienate, assign or mortgage it, and Settled is the rule that in the interpretation of contracts,
even substitute another person in its enjoyment, except the ascertainment of the intention of the contracting
That the petitioners really had the intention to dispose of when personal rights are involved. But the effect of the parties is to be discharged by looking to the words they
their shares in the subject parcels of land, irrespective of alienation or the mortgage, with respect to the co-owners, used to project that intention in their contract, all the
whether or not all of the heirs consented to the said shall be limited to the portion which may be allotted to him words, not just a particular word or two, and words in
Contract to Sell, was unveiled by Adolfos testimony as in the division upon the termination of the co-ownership. context, not words standing alone.[19]
follows: [Emphases supplied.]
In the instant case, the consideration of P100,000.00 paid
ATTY. GAMO: This alleged agreement between you and by respondent to petitioners was referred to as option
your other brothers and sisters that unless everybody will Consequently, even without the consent of the two co- money. However, a careful examination of the words used
agree, the properties would not be sold, was that heirs, Adolfo and Jesus, the Contract to Sell is still valid in the contract indicates that the money is not option
agreement in writing? and binding with respect to the 6/8 proportionate shares of money but earnest money. Earnest money and option
the petitioners, as properly held by the appellate court. money are not the same but distinguished thus: (a)
109
Sales – Chapter 3-12 Cases
earnest money is part of the purchase price, while option
money is the money given as a distinct consideration for
an option contract; (b) earnest money is given only where
there is already a sale, while option money applies to a
sale not yet perfected; and, (c) when earnest money is
given, the buyer is bound to pay the balance, while when
the would-be buyer gives option money, he is not required
to buy, but may even forfeit it depending on the terms of
the option.[20]

The sum of P100,000.00 was part of the purchase price.


Although the same was denominated as option money, it
is actually in the nature of earnest money or down
payment when considered with the other terms of the
contract. Doubtless, the agreement is not a mere
unilateral promise to sell, but, indeed, it is a Contract to
Sell as both the trial court and the appellate court declared
in their Decisions.
WHEREFORE, premises considered, the Petition is
DENIED, and the Decision and Resolution of the Court of
Appeals dated 26 April 2002 and 4 March 2003,
respectively, are AFFIRMED, thus, (a) the Contract to Sell
is DECLARED valid and binding with respect to the
undivided proportionate shares in the subject parcels of
land of the six signatories of the said document, herein
petitioners Ernesto, Enriqueta, Librado, Rizalino, Bibiano,
Jr., and Leonora (all surnamed Oesmer); (b) respondent is
ORDERED to tender payment to petitioners in the amount
of P3,216,560.00 representing the balance of the
purchase price for the latters shares in the subject parcels
of land; and (c) petitioners are further ORDERED to
execute in favor of respondent the Deed of Absolute Sale
covering their shares in the subject parcels of land after
receipt of the balance of the purchase price, and to pay
respondent attorneys fees plus costs of the suit. Costs
against petitioners.

SO ORDERED.

110
Sales – Chapter 3-12 Cases
SERAFIN, RAUL, NENITA, NAZARETO, NEOLANDA, influenza prior to his death, Roque had been physically
all surnamed NARANJA, AMELIA NARANJA- sound.[5] Belardos only source of income was her store and coffee
RUBINOS, NILDA NARANJA-LIMANA, and NAIDA shop. Sometimes, her children would give her money to
NARANJA-GICANO, Roque had no other source of income except for the help with the household expenses, including the expenses
Petitioners, P200.00 monthly rental of his two properties. To show his incurred for Roques support. At times, she would also
- versus - gratitude to Belardo, Roque sold Lot No. 4 and his one- borrow money from Margarita Dema-ala, a neighbor.[8]
COURT OF APPEALS, LUCILIA P. BELARDO, third share in Lot No. 2 to Belardo on August 21, 1981, When the amount of her loan reached P15,000.00, Dema-
represented by her Attorney-in-Fact, REBECCA through a Deed of Sale of Real Property which was duly ala required a security. On November 19, 1983, Roque
CORDERO, and THE LOCAL REGISTER OF DEEDS, notarized by Atty. Eugenio Sanicas. The Deed of Sale executed a deed of sale in favor of Dema-ala, covering his
BACOLOD CITY, reads: two properties in consideration of the P15,000.00
Respondents. outstanding loan and an additional P15,000.00, for a total
I, ROQUE NARANJA, of legal age, single, Filipino and a of P30,000.00. Dema-ala explained that she wanted
This petition seeks a review of the Court of Appeals (CA) resident of Bacolod City, do hereby declare that I am the Roque to execute the deed of sale himself since the
Decision[1] dated September 13, 2002 and Resolution[2] registered owner of Lot No. 4 of the Cadastral Survey of properties were still in his name. Belardo merely acted as
dated September 24, 2003 which upheld the contract of the City of Bacolod, consisting of 136 square meters, a witness. The titles to the properties were given to Dema-
sale executed by petitioners predecessor, Roque Naranja, more or less, covered by Transfer Certificate of Title No. ala for safekeeping.[9]
during his lifetime, over two real properties. T-18764 and a co-owner of Lot No. 2, situated at the City
of Bacolod, consisting of 151 square meters, more or less, Three days later, or on December 2, 1983, Roque died of
Roque Naranja was the registered owner of a parcel of covered by Transfer Certificate of Title No. T-18762 and influenza. The proceeds of the loan were used for his
land, denominated as Lot No. 4 in Consolidation- my share in the aforesaid Lot No. 2 is one-third share. treatment while the rest was spent for his burial.[10]
Subdivision Plan (LRC) Pcs-886, Bacolod Cadastre, with
an area of 136 square meters and covered by Transfer That for and in consideration of the sum of TEN In 1985, Belardo fully paid the loan secured by the second
Certificate of Title (TCT) No. T-18764. Roque was also a THOUSAND PESOS (P10,000.00), Philippine Currency, deed of sale. Dema-ala returned the certificates of title to
co-owner of an adjacent lot, Lot No. 2, of the same and other valuable consideration, receipt of which in full I Belardo, who, in turn, gave them back to Atty.
subdivision plan, which he co-owned with his brothers, hereby acknowledge to my entire satisfaction, by these Sanicas.[11]
Gabino and Placido Naranja. When Placido died, his one- presents, I hereby transfer and convey by way of absolute
third share was inherited by his children, Nenita, sale the above-mentioned Lot No. 4 consisting of 136 Unknown to Belardo, petitioners, the children of Placido
Nazareto, Nilda, Naida and Neolanda, all surnamed square meters covered by Transfer Certificate of Title No. and Gabino Naranja, executed an Extrajudicial Settlement
Naranja, herein petitioners. Lot No. 2 is covered by TCT T-18764 and my one-third share in Lot No. 2, covered by Among Heirs[12] on October 11, 1985, adjudicating
No. T-18762 in the names of Roque, Gabino and the said Transfer Certificate of Title No. T-18762, in favor of my among themselves Lot No. 4. On February 19, 1986,
children of Placido. TCT No. T-18762 remained even after sister LUCILIA P. BELARDO, of legal age, Filipino citizen, petitioner Amelia Naranja-Rubinos, accompanied by
Gabino died. The other petitioners Serafin Naranja, Raul married to Alfonso D. Belardo, and a resident of Belardo, borrowed the two TCTs, together with the lease
Naranja, and Amelia Naranja-Rubinos are the children of Pontevedra, Negros Occidental, her heirs, successors and agreement with Esso Standard Eastern, Inc., from Atty.
Gabino.[3] assigns. Sanicas on account of the loan being proposed by
Belardo to her. Thereafter, petitioners had the
The two lots were being leased by Esso Standard IN WITNESS WHEREOF, I have hereunto set my hand Extrajudicial Settlement Among Heirs notarized on
Eastern, Inc. for 30 years from 1962-1992. For his this 21st day of August, 1981 at Bacolod City, Philippines. February 25, 1986. With Roques copy of TCT No. T-
properties, Roque was being paid P200.00 per month by 18764 in their possession, they succeeded in having it
the company.[4] (SGD.) cancelled and a new certificate of title, TCT No. T-140184,
ROQUE NARANJA[6] issued in their names.[13]
In 1976, Roque, who was single and had no children,
lived with his half sister, Lucilia P. Belardo (Belardo), in In 1987, Belardo decided to register the Deed of Sale
Pontevedra, Negros Occidental. At that time, a catheter Roques copies of TCT No. T-18764 and TCT No. T-18762 dated August 21, 1981. With no title in hand, she was
was attached to Roques body to help him urinate. But the were entrusted to Atty. Sanicas for registration of the deed compelled to file a petition with the RTC to direct the
catheter was subsequently removed when Roque was of sale and transfer of the titles to Belardo. But the deed of Register of Deeds to annotate the deed of sale even
already able to urinate normally. Other than this and the sale could not be registered because Belardo did not have without a copy of the TCTs. In an Order dated June 18,
the money to pay for the registration fees.[7] 1987, the RTC granted the petition. But she only
111
Sales – Chapter 3-12 Cases
succeeded in registering the deed of sale in TCT No. T- who are now the registered owners of the parcels of land. are not purchasers of the said properties. Hence, it is not
18762 because TCT No. T-18764 had already been The RTC disposed, thus: significant that private respondent failed to register the
cancelled.[14] deed of sale before the extrajudicial settlement among the
IN VIEW OF ALL THE FOREGOING, judgment is hereby heirs. The dispositive portion of the CA Decision reads:
On December 11, 1989, Atty. Sanicas prepared a rendered:
certificate of authorization, giving Belardos daughter, WHEREFORE, the decision dated March 5, 1997 in Civil
Jennelyn P. Vargas, the authority to collect the payments 1. Dismissing Civil Case No. 7144. Cases Nos. 7144 and 7214 is hereby REVERSED and
from Esso Standard Eastern, Inc. But it appeared from the SET ASIDE. In lieu thereof, judgment is hereby rendered
companys Advice of Fixed Payment that payment of the 2. Civil Case No. 7214. as follows:
lease rental had already been transferred from Belardo to
Amelia Naranja-Rubinos because of the Extrajudicial a) Declaring the Deed of Sale dated August 21, 1. Civil Case No. 7214 is hereby ordered DISMISSED for
Settlement Among Heirs. 1981, executed by Roque Naranja, covering his one-third lack of cause of action.
(1/3) share of Lot 2 of the consolidation-subdivision plan
On June 23, 1992, Belardo,[15] through her daughter and (LRC) Pcs-886, being a portion of the consolidation of 2. In Civil Case No. 7144, the extrajudicial settlement
attorney-in-fact, Rebecca Cordero, instituted a suit for Lots 240-A, 240-B, 240-C and 240-D, described on plan, executed by the heirs of Roque Naranja adjudicating
reconveyance with damages. The complaint prayed that Psd-33443 (LRC) GLRO Cad. Rec. No. 55 in favor of among themselves Lot No. 4 of the consolidation-
judgment be rendered declaring Belardo as the sole legal Lucilia Belardo, and entered as Doc. No. 80, Page 17, subdivision plan (LRC) Pcs 886 of the Bacolod Cadastre
owner of Lot No. 4, declaring null and void the Book No. XXXVI, Series of 1981 of Notary Public Eugenio is hereby declared null and void for want of factual and
Extrajudicial Settlement Among Heirs, and TCT No. T- Sanicas of Bacolod City, as null and void and of no force legal basis. The certificate of title issued to the heirs of
140184, and ordering petitioners to reconvey to her the and effect; Roque Naranja (Transfer Certificate of [T]i[t]le No. T-
subject property and to pay damages. The case was 140184) as a consequence of the void extra-judicial
docketed as Civil Case No. 7144. b) Ordering the Register of Deeds of Bacolod settlement is hereby ordered cancelled and the previous
City to cancel Entry No. 148123 annotate at the back of title to Lot No. 4, Transfer Certificate of Title No. T-18764,
Subsequently, petitioners also filed a case against Transfer Certificate of Title No. T-18762; is hereby ordered reinstated. Lucilia Belardo is hereby
respondent for annulment of sale and quieting of title with declared the sole and legal owner of said Lot No. 4, and
damages, praying, among others, that judgment be c) Ordering Lucilia Belardo or her successors- one-third of Lot No. 2 of the same consolidation-
rendered nullifying the Deed of Sale, and ordering the in-interest to pay plaintiffs the sum of P20,000.00 as subdivision plan, Bacolod Cadastre, by virtue of the deed
Register of Deeds of Bacolod City to cancel the attorneys fees, the amount of P500.00 as appearance of sale thereof in her favor dated August 21, 1981.
annotation of the Deed of Sale on TCT No. T-18762. This fees.
case was docketed as Civil Case No. 7214. SO ORDERED.[17]
Counterclaims in both Civil Cases Nos. 7144 and 7214
On March 5, 1997, the RTC rendered a Decision in the are hereby DISMISSED.
consolidated cases in favor of petitioners. The trial court The CA denied petitioners motion for reconsideration on
noted that the Deed of Sale was defective in form since it SO ORDERED.[16] September 24, 2003.[18] Petitioners filed this petition for
did not contain a technical description of the subject review, raising the following issues:
properties but merely indicated that they were Lot No. 4,
covered by TCT No. T-18764 consisting of 136 square On September 13, 2002, the CA reversed the RTC 1. WHETHER OR NOT THE HONORABLE
meters, and one-third portion of Lot No. 2 covered by TCT Decision. The CA held that the unregisterability of a deed RESPONDENT COURT OF APPEALS IS CORRECT IN
No. T-18762. The trial court held that, being defective in of sale will not undermine its validity and efficacy in IGNORING THE POINT RAISED BY [PETITIONERS]
form, the Deed of Sale did not vest title in private transferring ownership of the properties to private THAT THE DEED OF SALE WHICH DOES NOT
respondent. Full and absolute ownership did not pass to respondent. The CA noted that the records were devoid of COMPL[Y] WITH THE PROVISIONS OF ACT NO. 496 IS
private respondent because she failed to register the any proof evidencing the alleged vitiation of Roques [NOT] VALID.
Deed of Sale. She was not a purchaser in good faith since consent to the sale; hence, there is no reason to
she acted as a witness to the second sale of the property invalidate the sale. Registration is only necessary to bind 2. WHETHER OR NOT THE ALLEGED DEED OF SALE
knowing that she had already purchased the property from third parties, which petitioners, being the heirs of Roque [OF REAL PROPERTIES] IS VALID CONSIDERING
Roque. Whatever rights private respondent had over the Naranja, are not. The trial court erred in applying Article THAT THE CONSENT OF THE LATE ROQUE NARANJA
properties could not be superior to the rights of petitioners, 1544 of the Civil Code to the case at bar since petitioners HAD BEEN VITIATED; x x x THERE [IS] NO
112
Sales – Chapter 3-12 Cases
CONCLUSIVE SHOWING THAT THERE WAS In the instant case, the deed of sale clearly identifies the contrary, Atty. Sanicas, the notary public, attested that
CONSIDERATION AND THERE [ARE] SERIOUS subject properties by indicating their respective lot Roque was very healthy and mentally sound and sharp at
IRREGULARITIES IN THE NOTARIZATION OF THE numbers, lot areas, and the certificate of title covering the time of the execution of the deed of sale. Atty. Sanicas
SAID DOCUMENTS.[19] them. Resort can always be made to the technical said that Roque also told him that he was a Law
description as stated in the certificates of title covering the graduate.[28]
two properties.
In her Comment, private respondent questioned the Neither was the contract simulated. The late registration of
Verification and Certification of Non-Forum Shopping On the alleged nullity of the deed of sale, we hold that the Deed of Sale and Roques execution of the second
attached to the Petition for Review, which was signed by a petitioners failed to submit sufficient proof to show that deed of sale in favor of Dema-ala did not mean that the
certain Ernesto Villadelgado without a special power of Roque executed the deed of sale under the undue contract was simulated. We are convinced with the
attorney. In their reply, petitioners remedied the defect by influence of Belardo or that the deed of sale was explanation given by respondents witnesses that the deed
attaching a Special Power of Attorney signed by them. simulated or without consideration. of sale was not immediately registered because Belardo
did not have the money to pay for the fees. This
Pursuant to its policy to encourage full adjudication of the A notarized document carries the evidentiary weight explanation is, in fact, plausible considering that Belardo
merits of an appeal, the Court had previously excused the conferred upon it with respect to its due execution, and could barely support herself and her brother, Roque. As
late submission of a special power of attorney to sign a documents acknowledged before a notary public have in for the second deed of sale, Dema-ala, herself, attested
certification against forum-shopping.[20] But even if we their favor the presumption of regularity. It must be before the trial court that she let Roque sign the second
excuse this defect, the petition nonetheless fails on the sustained in full force and effect so long as he who deed of sale because the title to the properties were still in
merits. impugns it does not present strong, complete, and his name.
conclusive proof of its falsity or nullity on account of some
The Court does not agree with petitioners contention that flaws or defects provided by law.[23] Finally, petitioners argue that the Deed of Sale was not
a deed of sale must contain a technical description of the supported by a consideration since no receipt was shown,
subject property in order to be valid. Petitioners anchor Petitioners allege that Belardo unduly influenced Roque, and it is incredulous that Roque, who was already weak,
their theory on Section 127 of Act No. 496,[21] which who was already physically weak and senile at that time, would travel to Bacolod City just to be able to execute the
provides a sample form of a deed of sale that includes, in into executing the deed of sale. Belardo allegedly took Deed of Sale.
particular, a technical description of the subject property. advantage of the fact that Roque was living in her house
and was dependent on her for support. The Deed of Sale which states receipt of which in full I
To be valid, a contract of sale need not contain a technical hereby acknowledge to my entire satisfaction is an
description of the subject property. Contracts of sale of There is undue influence when a person takes improper acknowledgment receipt in itself. Moreover, the
real property have no prescribed form for their validity; advantage of his power over the will of another, depriving presumption that a contract has sufficient consideration
they follow the general rule on contracts that they may be the latter of a reasonable freedom of choice.[24] One who cannot be overthrown by a mere assertion that it has no
entered into in whatever form, provided all the essential alleges any defect, or the lack of consent to a contract by consideration.[29]
requisites for their validity are present.[22] The requisites reason of fraud or undue influence, must establish by full,
of a valid contract of sale under Article 1458 of the Civil clear and convincing evidence, such specific acts that Heirs are bound by contracts entered into by their
Code are: (1) consent or meeting of the minds; (2) vitiated the partys consent; otherwise, the latters predecessors-in-interest.[30] As heirs of Roque,
determinate subject matter; and (3) price certain in money presumed consent to the contract prevails.[25] For undue petitioners are bound by the contract of sale that Roque
or its equivalent. influence to be present, the influence exerted must have executed in favor of Belardo. Having been sold already to
so overpowered or subjugated the mind of a contracting Belardo, the two properties no longer formed part of
The failure of the parties to specify with absolute clarity party as to destroy his free agency, making him express Roques estate which petitioners could have inherited. The
the object of a contract by including its technical the will of another rather than his own.[26] deed of extrajudicial settlement that petitioners executed
description is of no moment. What is important is that over Lot No. 4 is, therefore, void, since the property
there is, in fact, an object that is determinate or at least Petitioners adduced no proof that Roque had lost control subject thereof did not become part of Roques estate.
determinable, as subject of the contract of sale. The form of his mental faculties at the time of the sale. Undue
of a deed of sale provided in Section 127 of Act No. 496 is influence is not to be inferred from age, sickness, or WHEREFORE, premises considered, the petition is
only a suggested form. It is not a mandatory form that debility of body, if sufficient intelligence remains.[27] The DENIED. The Court of Appeals Decision dated September
must be strictly followed by the parties to a contract. evidence presented pertained more to Roques physical 13, 2002 and Resolution dated September 24, 2003 are
condition rather than his mental condition. On the AFFIRMED.
113
Sales – Chapter 3-12 Cases

SO ORDERED.

114
Sales – Chapter 3-12 Cases
SPS. SEGUNDO DALION AND EPIFANIA SABESAJE- On May 28, 1973, Sabesaje sued to recover ownership of On the West-Catalino Espina.
DALION, petitioners, a parcel of land, based on a private document of absolute
vs. sale, dated July 1, 1965 (Exhibit "A"), allegedly executed (pp. 41-42, Rollo)
THE HONORABLE COURT OF APPEALS AND by Dalion, who, however denied the fact of sale,
RUPERTO SABESAJE, JR., respondents. contending that the document sued upon is fictitious, his The issues in this case may thus be limited to: a) the
signature thereon, a forgery, and that subject land is validity of the contract of sale of a parcel of land and b)
This is a petition to annul and set aside the decision of the conjugal property, which he and his wife acquired in 1960 the necessity of a public document for transfer of
Court of Appeals rendered on May 26, 1987, upholding from Saturnina Sabesaje as evidenced by the "Escritura ownership thereto.
the validity of the sale of a parcel of land by petitioner de Venta Absoluta" (Exhibit "B"). The spouses denied
Segundo Dalion (hereafter, "Dalion") in favor of private claims of Sabesaje that after executing a deed of sale The appellate court upheld the validity of the sale on the
respondent Ruperto Sabesaje, Jr. (hereafter, "Sabesaje"), over the parcel of land, they had pleaded with Sabesaje, basis of Secs. 21 and 23 of Rule 132 of the Revised Rules
described thus: their relative, to be allowed to administer the land because of Court.
Dalion did not have any means of livelihood. They
A parcel of land located at Panyawan, Sogod, Southern admitted, however, administering since 1958, five (5) SEC. 21. Private writing, its execution and
Leyte, declared in the name of Segundo Dalion, under parcels of land in Sogod, Southern Leyte, which belonged authenticity, how proved.-Before any private writing may
Tax Declaration No. 11148, with an area of 8947 to Leonardo Sabesaje, grandfather of Sabesaje, who died be received in evidence, its due execution and
hectares, assessed at P 180.00, and bounded on the in 1956. They never received their agreed 10% and 15% authenticity must be proved either:
North, by Sergio Destriza and Titon Veloso, East, by commission on the sales of copra and abaca,
Feliciano Destriza, by Barbara Bonesa (sic); and West, by respectively. Sabesaje's suit, they countered, was (a) By anyone who saw the writing executed;
Catalino Espina. (pp. 36-37, Rollo) intended merely to harass, preempt and forestall Dalion's
threat to sue for these unpaid commissions. (b) By evidence of the genuineness of the
The decision affirms in toto the ruling of the trial court 1 handwriting of the maker; or
issued on January 17, 1984, the dispositive portion of From the adverse decision of the trial court, Dalion
which provides as follows: appealed, assigning errors some of which, however, were (c) By a subscribing witness
disregarded by the appellate court, not having been raised
WHEREFORE, IN VIEW OF THE FOREGOING, the in the court below. While the Court of Appeals duly xxx xxx xxx
Court hereby renders judgment. recognizes Our authority to review matters even if not
assigned as errors in the appeal, We are not inclined to do SEC. 23. Handwriting, how proved. — The
(a) Ordering the defendants to deliver to the plaintiff so since a review of the case at bar reveals that the lower handwriting of a person may be proved by any witness
the parcel of land subject of this case, declared in the court has judicially decided the case on its merits. who believes it to be the handwriting of such person, and
name of Segundo Dalion previously under Tax has seen the person write, or has seen writing purporting
Declaration No. 11148 and lately under Tax Declaration As to the controversy regarding the identity of the land, to be his upon which the witness has acted or been
No. 2297 (1974) and to execute the corresponding formal We have no reason to dispute the Court of Appeals' charged, and has thus acquired knowledge of the
deed of conveyance in a public document in favor of the findings as follows: handwriting of such person. Evidence respecting the
plaintiff of the said property subject of this case, handwriting may also be given by a comparison, made by
otherwise, should defendants for any reason fail to do so, To be sure, the parcel of land described in Exhibit "A" is the witness or the court, with writings admitted or treated
the deed shall be executed in their behalf by the the same property deeded out in Exhibit "B". The as genuine by the party against whom the evidence is
Provincial Sheriff or his Deputy; boundaries delineating it from adjacent lots are identical. offered, or proved to be genuine to the satisfaction of the
Both documents detail out the following boundaries, to wit: judge. (Rule 132, Revised Rules of Court)
(b) Ordering the defendants to pay plaintiff the
amount of P2,000.00 as attorney's fees and P 500.00 as On the North-property of Sergio Destriza and Titon And on the basis of the findings of fact of the trial court as
litigation expenses, and to pay the costs; and Veloso; follows:

(c) Dismissing the counter-claim. (p. 38, Rollo) On the East-property of Feliciano Destriza; Here, people who witnessed the execution of subject
deed positively testified on the authenticity thereof. They
The facts of the case are as follows: On the South-property of Barbara Boniza and categorically stated that it had been executed and signed
by the signatories thereto. In fact, one of such witnesses,
115
Sales – Chapter 3-12 Cases
Gerardo M. Ogsoc, declared on the witness stand that he It may be noted that two signatures of Segundo D. Dalion the vendor may require the vendee to pay the thing sold
was the one who prepared said deed of sale and had appear on the face of the questioned document (Exh. A), (Art. 1458, NCC).
copied parts thereof from the "Escritura De Venta one at the right corner bottom of the document (Exh. A-2)
Absoluta" (Exhibit B) by which one Saturnina Sabesaje and the other at the left hand margin thereof (Exh. A-3). The trial court thus rightly and legally ordered Dalion to
sold the same parcel of land to appellant Segundo Dalion. The second signature is already a surplusage. A forger deliver to Sabesaje the parcel of land and to execute
Ogsoc copied the bounderies thereof and the name of would not attempt to forge another signature, an corresponding formal deed of conveyance in a public
appellant Segundo Dalion's wife, erroneously written as unnecessary one, for fear he may commit a revealing document. Under Art. 1498, NCC, when the sale is made
"Esmenia" in Exhibit "A" and "Esmenia" in Exhibit "B". (p. error or an erroneous stroke. (Decision, p. 10) (pp. 42-43, through a public instrument, the execution thereof is
41, Rollo) Rollo) equivalent to the delivery of the thing. Delivery may either
be actual (real) or constructive. Thus delivery of a parcel
xxx xxx xxx We see no reason for deviating from the appellate court's of land may be done by placing the vendee in control and
ruling (p. 44, Rollo) as we reiterate that possession of the land (real) or by embodying the sale in
Against defendant's mere denial that he signed the a public instrument (constructive).
document, the positive testimonies of the instrumental Appellate courts have consistently subscribed to the
Witnesses Ogsoc and Espina, aside from the testimony of principle that conclusions and findings of fact by the trial As regards petitioners' contention that the proper action
the plaintiff, must prevail. Defendant has affirmatively courts are entitled to great weight on appeal and should should have been one for specific performance, We
alleged forgery, but he never presented any witness or not be disturbed unless for strong and cogent reasons, believe that the suit for recovery of ownership is proper.
evidence to prove his claim of forgery. Each party must since it is undeniable that the trial court is in a more As earlier stated, Art. 1475 of the Civil Code gives the
prove his own affirmative allegations (Section 1, Rule 131, advantageous position to examine real evidence, as well parties to a perfected contract of sale the right to
Rules of Court). Furthermore, it is presumed that a person as to observe the demeanor of the witnesses while reciprocally demand performance, and to observe a
is innocent of a crime or wrong (Section 5 (a), Idem), and testifying in the case (Chase v. Buencamino, Sr., G.R. No. particular form, if warranted, (Art. 1357). The trial court,
defense should have come forward with clear and L-20395, May 13, 1985, 136 SCRA 365; Pring v. Court of aptly observed that Sabesaje's complaint sufficiently
convincing evidence to show that plaintiff committed Appeals, G.R. No. L-41605, August 19, 1985, 138 SCRA alleged a cause of action to compel Dalion to execute a
forgery or caused said forgery to be committed, to 185) formal deed of sale, and the suit for recovery of
overcome the presumption of innocence. Mere denial of ownership, which is premised on the binding effect and
having signed, does not suffice to show forgery. Assuming authenticity of his signature and the validity inter partes of the contract of sale, merely seeks
genuineness of the document, Dalion nonetheless still consummation of said contract.
In addition, a comparison of the questioned signatories or impugns the validity of the sale on the ground that the
specimens (Exhs. A-2 and A-3) with the admitted same is embodied in a private document, and did not thus ... . A sale of a real property may be in a private
signatures or specimens (Exhs. X and Y or 3-C) convey title or right to the lot in question since "acts and instrument but that contract is valid and binding between
convinces the court that Exhs. A-2 or Z and A-3 were contracts which have for their object the creation, the parties upon its perfection. And a party may compel
written by defendant Segundo Dalion who admitted that transmission, modification or extinction of real rights over the other party to execute a public instrument embodying
Exhs. X and Y or 3-C are his signatures. The questioned immovable property must appear in a public instrument" their contract affecting real rights once the contract
signatures and the specimens are very similar to each (Art. 1358, par 1, NCC). appearing in a private instrument hag been perfected
other and appear to be written by one person. (See Art. 1357).
This argument is misplaced. The provision of Art. 1358 on
Further comparison of the questioned signatures and the the necessity of a public document is only for ... . (p. 12, Decision, p. 272, Records)
specimens with the signatures Segundo D. Dalion convenience, not for validity or enforceability. It is not a
appeared at the back of the summons (p. 9, Record); on requirement for the validity of a contract of sale of a parcel ACCORDINGLY, the petition is DENIED and the decision
the return card (p. 25, Ibid.); back of the Court Orders of land that this be embodied in a public instrument. of the Court of Appeals upholding the ruling of the trial
dated December 17, 1973 and July 30, 1974 and for court is hereby AFFIRMED. No costs.
October 7, 1974 (p. 54 & p. 56, respectively, Ibid.), and on A contract of sale is a consensual contract, which means
the open court notice of April 13, 1983 (p. 235, Ibid.) that the sale is perfected by mere consent. No particular SO ORDERED.
readily reveal that the questioned signatures are the form is required for its validity. Upon perfection of the
signatures of defendant Segundo Dalion. contract, the parties may reciprocally demand
performance (Art. 1475, NCC), i.e., the vendee may
compel transfer of ownership of the object of the sale, and
116
Sales – Chapter 3-12 Cases
HEIRS OF ERNESTO BIONA, NAMELY: EDITHA B. own eight-hectare parcel of land in Surallah to help
BLANCAFLOR, MARIANITA D. DE JESUS, VILMA B. On July 3, 1962, defendant-appellant paid the sum of Soledad Biona; that to evidence the transaction, a deed of
BLANCAFLOR, ELSIE B. RAMOS and PERLITA B. P1,400.00 to the Development Bank of the Philippines to sale was handwritten by Soledad Biona and signed by her
CARMEN, petitioners, vs. THE COURT OF APPEALS cancel the mortgage previously constituted by the Biona and the defendant; that at the time of the sale, half of the
and LEOPOLDO HILAJOS, respondents. spouses on June 3, 1953 (Exhs. 4 and 6). portion of the property was already submerged in water
and from the years 1969 to 1984, two and one-half
Before us is a petition for review on certiorari under Rule Thereafter, and for a period of not less than twenty-five hectares thereof were eroded by the Allah River; that by
45 of the Decision of the Court of Appeals dated March years, defendant-appellant continued his peaceful and virtue of his continuous and peaceful occupation of the
31, 1992, reversing the decision of the Regional Trial public occupation of the property, declaring it in his name property from the time of its sale and for more than
Court, 11th Judicial region, Branch 26, Surallah, South for taxation purposes (Exhs. 10 and 11), paying real twenty- five years thereafter, defendant possesses a
Cotabato and the Resolution dated May 26, 1992, estate property taxes thereon (Exhs. 12, 13, 13-a to 13-e, better right thereto subject only to the rights of the tenants
denying the subsequent motion for reconsideration. F, G, H and I), and causing the same to be tenanted whom he had allowed to cultivate the land under the Land
(Exhs. 7, 8, 9). Reform Program of the government; that the complaint
Quoting from the decision of the Court of Appeals, the states no cause of action; that plaintiffs alleged right, if
antecedent facts are as follows: On June 19, 1985, plaintiffs-appellees, filed a complaint any, is barred by the statutes of fraud. As counterclaim,
for recovery of ownership, possession, accounting and defendant-appellant prayed that plaintiffs-appellees be
On October 23, 1953, the late Ernesto Biona, married to damages, with a prayer for a writ of preliminary mandatory ordered to execute a formal deed of sale over the subject
plaintiff-appellee Soledad Biona, was awarded injunction and/ or restraining order against defendant- property and to pay him actual, moral and exemplary
Homestead Patent No. V-840 over the property subject of appellant alleging, among others, that the latter had damages as the trial court may deem proper. He likewise
this suit, a parcel of agricultural land denominated as lot unlawfully been depriving them of the use, possession prayed for the award of attorney's fees in the sum of
177 of PLS-285-D, located in Bo. 3, Banga, Cotabato, and enjoyment of the subject property; that the entire P10,000.00.
containing an area of ten (10) hectares, forty-three (43) parcel of land, which was devoted and highly suited to
acres and sixty-eight (68) centares, Original Certificate of palay and corn, was yielding three harvests annually, with During the hearing of the case, plaintiffs-appellees
Title No. (V-2323) P-3831 was issued in his name by the an average of one hundred twenty (120) sacks of corn presented in evidence the testimonies of Editha Biona
Register of Deeds of Cotabato (Exh. C). On June 3, 1954, and eighty cavans of rice per hectare; that plaintiffs- Blancaflor and Vilma Biona Blancaflor, and documentary
Ernesto and Soledad Biona obtained a loan from the then appellees were deprived of its total produce amounting to exhibits A to G and their submarkings.
Rehabilitation Finance Corporation (now the Development P150,000.00. Plaintiffs-appellees prayed for the award of
Bank of the Philippines) and put up as collateral the moral damages in the sum of P50,000.00, exemplary Defendant-appellant, for his part, presented the
subject property (Exh. 4). On June 12, 1956, Ernesto damages in the amount of P20,000,00 and litigation testimonies of himself and Mamerto Famular, including
Biona died (Exh. B) leaving as his heirs herein plaintiffs- expenses in the amount of P2,000.00. documentary exhibits 1 to 13, F, G, H, I, and their
appellees, namely, his wife, Soledad Estrobillo Vda. De submarkings.[1]
Biona, and five daughters, Editha B. Blancaflor, Marianita On September 19, 1986, defendant-appellant filed his
B. de Jesus, Vilma B. Blancaflor, Elsie B. Ramos and answer with counterclaim traversing the material On January 31, 1990, the RTC rendered a decision with
Perlita B. Carmen. allegations in the complaint and alleging, by way of the following dispositive portion:
affirmative and special defenses, that: on September 11,
On March 1, 1960, plaintiff-appellee Soledad Biona 1961, Soledad Biona, after obtaining the loan of I (SIC) VIEW OF THE FOREGOING, decision is hereby
obtained a loan from defendant-appellant in the amount of P1,000.00 from defendant-appellant, approached and rendered:
P1,000 and as security therefore, the subject property was begged the latter to buy the whole of Lot No. 177 since it
mortgaged. It was further agreed upon by the contracting was then at the brink of foreclosure by the Development 1. ordering the defendant to vacate possession of the lot
parties that for a period of two years until the debt is paid, Bank of the Philippines and she had no money to redeem in question to the extent of six-tenths (6/10) of the total
defendant-appellant shall occupy the land in dispute and the same nor the resources to support herself and her five area thereof and to deliver the same to the plaintiff
enjoy the usufruct thereof. small children; that defendant-appellant agreed to buy the Soledad Estrobillo Biona upon the latter's payment of the
property for the amount of P4,300.00, which consideration sum of P1,000.00 TO THE FORMER IN REDEMPTION
The two-year period elapsed but Soledad Biona was not was to include the redemption price to be paid to the OF ITS MORTGAGE CONSTITUTED UNDER exh. "1" of
able to pay her indebtedness. Defendant-appellant Development Bank of the Philippines; that the purchase defendant;
continued occupying and cultivating the subject property price paid by defendant far exceeded the then current
without protest from plaintiffs-appellees. market value of the property and defendant had to sell his
117
Sales – Chapter 3-12 Cases
2. ordering the defendant to vacate the possession of the PROPERTY BY VIRTUE OF THE EQUITABLE to agree with the findings and conclusions of the
remaining four-tenths (4/10) of the area of the lot in PRINCIPLE OF LACHES. respondent court as they are more in accord with the law
question, representing the shares of the children of the and evidence on record.
late Ernesto Biona and deliver the same to said plaintiffs; IV- RESPONDENT COURT OF APPEALS ERRED IN
the defendant shall render an accounting of the net NOT HOLDING THAT PRIVATE RESPONDENT'S As to the authenticity of the deed of sale, we subscribe to
produce of the area ordered returned to the co-plaintiffs of RIGHT OF ACTION UNDER THE DEED OF SALE the Court of Appeals' appreciation of evidence that private
Soledad Biona commencing from the date of the filing of (EXHIBIT "2") HAD PRESCRIBED.[4] respondent has substantially proven that Soledad Biona
the complaint until possession thereto has been delivered indeed signed the deed of sale of the subject property in
to said co-plaintiffs and to deliver or pay 25% of said net As correctly pointed out by the Court of Appeals, the his favor. His categorical statement in the trial court that
produce to said co-plaintiffs; pivotal issue in the instant case is whether or not the deed he himself saw Soledad Estrobillo affix her signature on
of sale is valid and if it effectively conveyed to the private the deed of sale lends credence. This was corroborated
3. ordering the defendant to pay the costs of this suit. respondents the subject property. by another witness, Mamerto Famular. Although the
petitioners consider such testimony as self-serving and
The defendant's counter-claim are dismissed for lack of In ruling in favor of the petitioners, the trial court refused to biased,[6] it can not, however, be denied that private
merit. give weight to the evidence of private respondent which respondent has shown by competent proof that a contract
consisted of (1) the handwritten and unnotarized deed of of sale where all the essential elements are present for its
SO ORDERED.[2] sale executed by Soledad Biona in favor of the private validity was executed between the parties.[7] The burden
respondent; and (2) the corresponding acknowledgment is on the petitioners to prove the contrary which they have
Dissatisfied, herein private respondent appealed to the receipt of the amount of P3,500.00 as partial payment for dismally failed to do. As aptly stated by the Court of
Court of Appeals which reversed the trial court's ruling. the land in dispute. To the mind of the trial court, the Appeals:
The dispositive portion reads as follows: signature of Soledad Biona on the deed of sale was not
genuine. There was no direct evidence to prove that Having established the due execution of the subject deed
WHEREFORE, premises considered, the judgment Soledad Biona herself signed the document. Moreover, of sale and the receipt evidencing payment of the
appealed from is set aside and a new one entered the deed of sale was not notarized and therefore, did not consideration, the burden now shifted to plaintiffs-
dismissing the complaint, and the plaintiffs-appellees are convey any rights to the vendee. The trial court also ruled appellees to prove by contrary evidence that the property
ordered to execute a registrable deed of conveyance of that petitioners' rights over the land have not allegedly was not so transferred. They were not able to do this
the subject property in favor of the defendant-appellant prescribed. since the very person who could deny the due execution
within ten (10) days from the finality of this decision. With of the document, Soledad Biona, did not testify. She
costs against plaintiffs-appellees.[3] On the other hand, the respondent Court of Appeals similarly failed to take the witness stand in order to deny
accepted as genuine the deed of sale (Exh. 2) which "sets her signatures on Exhs. 2 and 3. Admitting as true that
Hence, the instant petition where the following assignment forth in unmistakable terms that Soledad Biona agreed for she was under medication in Manila while the hearing of
of errors were made: the consideration of P4,500.00, to transfer to defendant- the case was underway, it was easy enough to get her
appellant Lot 177. The fact that payment was made is deposition. Her non-presentation gives rise to the
I.- RESPONDENT COURT OF APPEALS ERRED IN evidenced by the acknowledgment receipt for P3,500.00 presumption that if her testimony was taken, the same
CONCLUDING THAT THE SIGNATURE OF SOLEDAD (Exh. 3) signed by Soledad Biona, and private respondent would be adverse to the claim by plaintiffs-appellees.
ESTROBILLO IN THE DEED OF SALE (EXHIBIT "2"), A previous delivery of P1,000.00 to her pursuant to the
PRIVATE DOCUMENT, IS GENUINE. Mutual Agreement (Exh. 1). It must also be noted that under Sec. 22 Rule 132 of our
procedural law, evidence respecting handwriting may also
II - RESPONDENT COURT OF APPEALS ERRED IN The contract of sale between the contracting parties was be given by a comparison, made by the witness or the
HOLDING THAT THE DEED OF SALE (EXHIBIT 2) IS consummated by the delivery of the subject land to private court, with writings admitted or treated as genuine by the
VALID AND COULD LEGALLY CONVEY TO PRIVATE respondent who since then had occupied and cultivated party against whom the evidence is offered. Our own
RESPONDENT OWNERSHIP AND TITLE OVER THE the same continuously and peacefully until the institution close scrutiny of the signature of Soledad Biona
SUBJECT PROPERTY. of this suit."[5] appearing on Exh. 1, the document admitted by the
contending parties, reveals that it is the same as the
III - RESPONDENT COURT OF APPEALS ERRED IN Given the contrary findings of the trial court and the signatures appearing on Exhs. 2 and 3, the documents in
HOLDING THAT HEREIN PETITIONERS HAD LOST respondent court, there is a need to re-examine the dispute. Admittedly, as was pointed out by the trial court,
THEIR RIGHT TO RECOVER THE SUBJECT evidence altogether. After a careful study, we are inclined the "S" in Exhs. 2 and 3 were written in printed type while
118
Sales – Chapter 3-12 Cases
that in Exh. 1 is in handwriting type. But a careful look at otherwise provides for a prescribed form, which is not so alienate that portion of Lot 177 belonging to her, which is
the text of Exh. 2 would reveal that Soledad Biona in this case. The deed of sale so executed, although a seven-twelfths of the entire property. She had no power or
alternately wrote the letter "S" in longhand and printed private document, is effective as between the parties authority to dispose of the shares of her co-owners, the
form. Thus, the words "Sum" and "Sept.," found in the themselves and also as the third persons having no better five daughters of the deceased Ernesto Biona, who were
penultimate and last paragraphs of the document, title, and should be admitted in evidence for the purpose entitled to an indivisible five-twelfths portion of the whole
respectively, were both written in longhand, while her of showing the rights and relations of the contracting property. It is not disputed, however, that as early as
name appearing on first part of the document, as well as parties (Carbonell v. Court of Appeals, 69 SCRA 99; 1960, when Soledad Biona borrowed money from
the erased word "Sept." in the last paragraph thereof were Elumbaring v. Elumbaring, 12 Phil. 384). Under Art. 1356 defendant-appellant (Exh. L), the latter entered,
written in printed form. Moreover, all doubts about the of the Civil Code, contracts shall be obligatory in whatever possessed and started occupying the same in the concept
genuineness of Soledad Biona's signatures on Exhs. 2 form they may have been entered into provided all the of an owner. He caused its cultivation through various
and 3 are removed upon their comparison to her signature essential requisites for their necessary elements for a tenants under Certificates of Land Transfer (Exhs. 7-9),
appearing on the special power of attorney (Exh. A) valid contract of sale were met when Soledad Biona declared the property in his name, religiously paid taxes
presented in evidence by plaintiffs-appellees during trial. agreed to sell and actually conveyed Lot 177 to thereon, reaped benefits therefrom, and executed other
In said document, Soledad Biona signed her name using defendant-appellant who paid the amount of P4,500.00 acts of dominion without any protest or interference from
the same fact that Soledad Estrobillo Biona wrote her therefore. The deed of sale (Exh. 2) is not made plaintiffs-appellees for more than twenty-five years. Even
entire name on Exh. 2 while she merely affixed her ineffective merely because it is not notarized or does not when the five daughters of the deceased Ernesto Biona
maiden name on the other two documents may have been appear in a public document. The contract is binding upon were way past the age of majority, when they could have
due to the lesser options left to her when the lawyers who the contracting parties, defendant-appellant and Soledad already asserted their right to their share, no sale in
drafted the two documents (Exhs. 2 and 3) already had Biona, including her successors-in-interest. Pursuant to defendant-appellant's favor was ever brought or any other
typewritten the names "SOLEDAD ESTROBILLO" thereon Art. 1357, plaintiffs-appellees may be compelled by action was taken by them to recover their share. Instead,
whereas in Exh. 2, it was Soledad Biona herself who defendant-appellant to execute a public document to they allowed defendant-appellant to peacefully occupy the
printed and signed her own name. Thus, in the special embody their valid and enforceable contract and for the property without protest. Although it is true that no title to
power of attorney (Exh. A), Soledad Biona signed her purpose of registering the property in the latter's name registered land in derogation of that of the registered
name in the same manner it was typewritten on the (Clarin v. Rulona, 127 SCRA 512; Heirs of Amparo v. owner shall be acquired by prescription or adverse
document.[8] Santos, 108 SCRA 43; Araneta v. Montelibano, 14 Phil. possession as the right to recover possession of
117).[12] registered land is imprescriptible, jurisprudence has laid
We agree with the private respondent that all the down the rule that a person and his heirs may lose their
requisites for a valid contract of sale are present in the Finally, we find no merit in petitioners' contention that their right to recover back the possession of such property and
instant case. For a valuable consideration of P4,500.00, right over the land has not prescribed. The principle of title thereto by reason of laches. (Victoriano v. Court of
Soledad Biona agreed to sell and actually conveyed the laches was properly applied against petitioner. Laches Appeals, 194 SCRA 19; Lola v. CA, 145 SCRA 439, 449).
subject property to private respondent. The fact that the has been defined as the failure or neglect, for an Indeed, it has been ruled in the case of Miguel v. Catalino,
deed of sale was not notarized does not render the unreasonable and unexplained length of time, to do that 26 SCRA 234, 239, that:
agreement null and void and without any effect. The which by exercising due diligence could or should have
provision of Article 1358 of the Civil Code[9] on the been done earlier, it is negligence or omission to assert a 'Courts can not look with favor at parties who, by their
necessity of a public document is only for convenience, right within a reasonable time, warranting a presumption silence, delay and inaction, knowingly induce another to
and not for validity or enforceability.[10] The observance that the party entitled to assert it has either abandoned it spend time, effort and expense in cultivating the land,
of which is only necessary to insure its efficacy, so that or declined to assert it.[13] In the instant case, the Court paying taxes and making improvements thereof for 30
after the existence of said contract had been admitted, the of Appeals point to the circumstances that warrant the long years, only to spring from ambush and claim title
party bound may be compelled to execute the proper principle to come into play: when the possessor's efforts and the rise of land values
document.[11] Undeniably, a contract has been entered offer an opportunity to make easy profit at his expense.'
into by Soledad Biona and the private respondent. Laches had been defined to be such neglect or omission
Regardless of its form, it was valid, binding and to assert a right taken in conjunction with the lapse of time Thus, notwithstanding the invalidity of the sale with
enforceable between the parties. We quote with favor the and other circumstances causing prejudice to an adverse respect to the share of plaintiffs-appellees, the daughters
respondent court's ratiocination on the matter: party, as will bar him in equity (Heirs of Batiog Lacamen v. of the late Ernesto Biona, they [allowed] the vendee,
Heirs of Laruan, 65 SCRA 605, 609-610). In the instant defendant-appellant herein, to enter, occupy and possess
xxx The trial court cannot dictate the manner in which the suit, Soledad Biona, at the time of the execution of the the property in the concept of an owner without demurrer
parties may execute their agreement, unless the law deed of sale (Exh. 2) on September 11, 1961, could only and molestation for a long period of time, never claiming
119
Sales – Chapter 3-12 Cases
the land as their own until 1985 when the property has
greatly appreciated in value. Vigilantibus non dormientibus
sequitas subvenit.[14]

WHEREFORE, the Petition is DENIED and the assailed


Decision of the Court of Appeals is AFFIRMED.

SO ORDERED.

120
Sales – Chapter 3-12 Cases
HEIRS OF CECILIO (also known as BASILIO) herein private respondents (hereinafter referred to as is in generic term (Section 2, Rule 3). The Court has
CLAUDEL, namely, MODESTA CLAUDEL, LORETA SIBLINGS OF CECILIO). In 1972, the HEIRS OF noticed also that with the exception of plaintiff Lampitoc
HERRERA, JOSE CLAUDEL, BENJAMIN CLAUDEL, CECILIO partitioned this lot among themselves and and (sic) the heirs of Raymunda Claudel are no longer
PACITA CLAUDEL, CARMELITA CLAUDEL, MARIO obtained the corresponding Transfer Certificates of Title residing in the property as they have (sic) left the same in
CLAUDEL, ROBERTO CLAUDEL, LEONARDO on their shares, as follows: 1967. But most important of all the plaintiffs failed to
CLAUDEL, ARSENIA VILLALON, PERPETUA present any document evidencing the alleged sale of the
CLAUDEL and FELISA CLAUDEL, petitioners, TCT No. 395391 1,997 sq. m. –– Jose Claudel property to their predecessors in interest by the father of
vs. the defendants. Considering that the subject matter of the
HON. COURT OF APPEALS, HEIRS OF MACARIO, TCT No. 395392 1,997 sq. m. –– Modesta supposed sale is a real property the absence of any
ESPERIDIONA, RAYMUNDA and CELESTINA, all Claudel and children document evidencing the sale would preclude the
surnamed CLAUDEL, respondents. admission of oral testimony (Statute of Frauds). Moreover,
TCT No. 395393 1,997 sq. m. –– Armenia C. considering also that the alleged sale took place in 1930,
This petition for review on certiorari seeks the reversal of Villalon the action filed by the plaintiffs herein for the recovery of
the decision rendered by the Court of Appeals in CA-G.R. the same more than thirty years after the cause of action
CV No. 044291 and the reinstatement of the decision of TCT No. 395394 1,997 sq. m. –– Felisa has accrued has already prescribed.
the then Court of First Instance (CFI) of Rizal, Branch CXI, Claudel4
in Civil Case No. M-5276-P, entitled. "Heirs of Macario WHEREFORE, the Court renders judgment dismissing the
Claudel, et al. v. Heirs of Cecilio Claudel, et al.," which Four years later, on December 7, 1976, private complaint, without pronouncement as to costs.
dismissed the complaint of the private respondents respondents SIBLINGS OF CECILIO, filed Civil Case No.
against the petitioners for cancellation of titles and 5276-P as already adverted to at the outset, with the then SO ORDERED.5
reconveyance with damages.2 Court of First Instance of Rizal, a "Complaint for
Cancellation of Titles and Reconveyance with Damages," On appeal, the following errors6 were assigned by the
As early as December 28, 1922, Basilio also known as alleging that 46 years earlier, or sometime in 1930, their SIBLINGS OF CECILIO:
"Cecilio" Claudel, acquired from the Bureau of Lands, Lot parents had purchased from the late Cecilio Claudel
No. 1230 of the Muntinlupa Estate Subdivision, located in several portions of Lot No. 1230 for the sum of P30.00. 1. THE TRIAL COURT ERRED IN DISMISSING
the poblacion of Muntinlupa, Rizal, with an area of 10,107 They admitted that the transaction was verbal. However, PLAINTIFFS' COMPLAINT DESPITE CONCLUSIVE
square meters; he secured Transfer Certificate of Title as proof of the sale, the SIBLINGS OF CECILIO EVIDENCE SHOWING THE PORTION SOLD TO EACH
(TCT) No. 7471 issued by the Registry of Deeds for the presented a subdivision plan of the said land, dated OF PLAINTIFFS' PREDECESSORS.
Province of Rizal in 1923; he also declared the lot in his March 25, 1930, indicating the portions allegedly sold to
name, the latest Tax Declaration being No. 5795. He the SIBLINGS OF CECILIO. 2. THE TRIAL COURT ERRED IN HOLDING THAT
dutifully paid the real estate taxes thereon until his death PLAINTIFFS FAILED TO PROVE ANY DOCUMENT
in 1937.3 Thereafter, his widow "Basilia" and later, her As already mentioned, the then Court of First Instance of EVIDENCING THE ALLEGED SALE.
son Jose, one of the herein petitioners, paid the taxes. Rizal, Branch CXI, dismissed the complaint, disregarding
the above sole evidence (subdivision plan) presented by 3. THE TRIAL COURT ERRED IN NOT GIVING
The same piece of land purchased by Cecilio would, the SIBLINGS OF CECILIO, thus: CREDIT TO THE PLAN, EXHIBIT A, SHOWING THE
however, become the subject of protracted litigation thirty- PORTIONS SOLD TO EACH OF THE PLAINTIFFS'
nine years after his death. Examining the pleadings as well as the evidence PREDECESSORS-IN-INTEREST.
presented in this case by the parties, the Court can not
Two branches of Cecilio's family contested the ownership but notice that the present complaint was filed in the name 4. THE TRIAL COURT ERRED IN NOT
over the land-on one hand the children of Cecilio, namely, of the Heirs of Macario, Espiridiona, Raymunda and DECLARING PLAINTIFFS AS OWNERS OF THE
Modesto, Loreta, Jose, Benjamin, Pacita, Carmelita, Celestina, all surnamed Claudel, without naming the PORTION COVERED BY THE PLAN, EXHIBIT A.
Roberto, Mario, Leonardo, Nenita, Arsenia Villalon, and different heirs particularly involved, and who wish to
Felisa Claudel, and their children and descendants, now recover the lots from the defendants. The Court tried to 5. THE TRIAL COURT ERRED IN NOT
the herein petitioners (hereinafter referred to as HEIRS find this out from the evidence presented by the plaintiffs DECLARING TRANSFER CERTIFICATES OF TITLE
OF CECILIO), and on the other, the brother and sisters of but to no avail. On this point alone, the Court would not be NOS. 395391, 395392, 395393 AND 395394 OF THE
Cecilio, namely, Macario, Esperidiona, Raymunda, and able to apportion the property to the real party in interest if REGISTER OF DEEDS OF RIZAL AS NULL AND VOID.
Celestina and their children and descendants, now the ever they are entitled to it as the persons indicated therein
121
Sales – Chapter 3-12 Cases
The Court of Appeals reversed the decision of the trial issued in the names of the HEIRS OF CECILIO and The rule of thumb is that a sale of land, once
court on the following grounds: corollarily ordered the execution of the following deeds of consummated, is valid regardless of the form it may have
reconveyance: been entered into.11 For nowhere does law or
1. The failure to bring and prosecute the action in jurisprudence prescribe that the contract of sale be put in
the name of the real party in interest, namely the parties To Celestina Claudel, Lot 1230-A with an area of 705 sq. writing before such contract can validly cede or transmit
themselves, was not a fatal omission since the court a quo m. rights over a certain real property between the parties
could have adjudicated the lots to the SIBLINGS OF themselves.
CECILIO, the parents of the herein respondents, leaving it To Raymunda Claudel, Lot 1230-B with an area of 599 sq.
to them to adjudicate the property among themselves. m. However, in the event that a third party, as in this case,
disputes the ownership of the property, the person against
2. The fact of residence in the disputed properties To Esperidiona Claudel, Lot 1230-C with an area of 597 whom that claim is brought can not present any proof of
by the herein respondents had been made possible by the sq. m. such sale and hence has no means to enforce the
toleration of the deceased Cecilio. contract. Thus the Statute of Frauds was precisely
To Macario Claudel, Lot 1230-D, with an area of 596 sq. devised to protect the parties in a contract of sale of real
3. The Statute of Frauds applies only to executory m.10 property so that no such contract is enforceable unless
contracts and not to consummated sales as in the case at certain requisites, for purposes of proof, are met.
bar where oral evidence may be admitted as cited in Iñigo The respondent court also enjoined that this disposition is
v. Estate of Magtoto7 and Diana, et al. v. Macalibo.8 without prejudice to the private respondents, as heirs of The provisions of the Statute of Frauds pertinent to the
their deceased parents, the SIBLINGS OF CECILIO, present controversy, state:
In addition, partitioning among themselves in accordance with law the
respective portions sold to and herein adjudicated to their Art. 1403 (Civil Code). The following contracts are
. . . Given the nature of their relationship with one another parents. unenforceable, unless they are ratified:
it is not unusual that no document to evidence the sale
was executed, . . ., in their blind faith in friends and The rest of the land, lots 1230-E and 1230-F, with an area xxx xxx xxx
relatives, in their lack of experience and foresight, and in of 598 and 6,927 square meters, respectively would go to
their ignorance, men, in spite of laws, will make and Cecilio or his heirs, the herein petitioners. Beyond these 2) Those that do not comply with the Statute of
continue to make verbal contracts. . . .9 apportionments, the HEIRS OF CECILIO would not Frauds as set forth in this number. In the following cases,
receive anything else. an agreement hereafter made shall be unenforceable by
4. The defense of prescription cannot be set up action unless the same, or some note or memorandum
against the herein petitioners despite the lapse of over The crux of the entire litigation is whether or not the Court thereof, be in writing, and subscribed by the party
forty years from the time of the alleged sale in 1930 up to of Appeals committed a reversible error in disposing the charged, or by his agent; evidence, therefore, of the
the filing of the "Complaint for Cancellation of Titles and question of the true ownership of the lots. agreement cannot be received without the writing, or a
Reconveyance . . ." in 1976. secondary evidence of its contents:
And the real issues are:
According to the Court of Appeals, the action was not for xxx xxx xxx
the recovery of possession of real property but for the 1. Whether or not a contract of sale of land may be
cancellation of titles issued to the HEIRS OF CECILIO in proven orally: e) An agreement for the leasing for a longer period
1973. Since the SIBLINGS OF CECILIO commenced their than one year, or for the sale of real property or of an
complaint for cancellation of titles and reconveyance with 2. Whether or not the prescriptive period for filing interest therein;
damages on December 7, 1976, only four years after the an action for cancellation of titles and reconveyance with
HEIRS OF CECILIO partitioned this lot among themselves damages (the action filed by the SIBLINGS OF CECILIO) xxx xxx xxx
and obtained the corresponding Transfer Certificates of should be counted from the alleged sale upon which they
Titles, then there is no prescription of action yet. claim their ownership (1930) or from the date of the (Emphasis supplied.)
issuance of the titles sought to be cancelled in favor of the
Thus the respondent court ordered the cancellation of the HEIRS OF CECILIO (1976). The purpose of the Statute of Frauds is to prevent fraud
Transfer Certificates of Title Nos. 395391, 395392, and perjury in the enforcement of obligations depending
395393, and 395394 of the Register of Deeds of Rizal for their evidence upon the unassisted memory of
122
Sales – Chapter 3-12 Cases
witnesses by requiring certain enumerated contracts and registered in another person's name. An implied trust was
transactions to be evidenced in Writing.12 The Court has invariably upheld the indefeasibility of the therefore created. This trustee was compelled by law to
torrens title. No possession by any person of any portion reconvey property fraudulently acquired notwithstanding
The provisions of the Statute of Frauds originally of the land could defeat the title of the registered owners the irrevocability of the torrens title.20
appeared under the old Rules of Evidence. However when thereof.14
the Civil Code was re-written in 1949 (to take effect in In the present case, however, the facts belie the claim of
1950), the provisions of the Statute of Frauds were taken A torrens title, once registered, cannot be defeated, even ownership.
out of the Rules of Evidence in order to be included under by adverse, open and notorious possession. A registered
the title on Unenforceable Contracts in the Civil Code. The title under the torrens system cannot be defeated by For several years, when the SIBLINGS OF CECILIO,
transfer was not only a matter of style but to show that the prescription.1âwphi1 The title, once registered, is notice to namely, Macario, Esperidiona Raymunda, and Celestina
Statute of Frauds is also a substantive law. the world. All persons must take notice. No one can plead were living on the contested premises, they regularly paid
ignorance of the registration.15 a sum of money, designated as "taxes" at first, to the
Therefore, except under the conditions provided by the widow of Cecilio, and later, to his heirs.21 Why their
Statute of Frauds, the existence of the contract of sale xxx xxx xxx payments were never directly made to the Municipal
made by Cecilio with his siblings13 can not be proved. Government of Muntinlupa when they were intended as
Furthermore, a private individual may not bring an action payments for "taxes" is difficult to square with their claim
On the second issue, the belated claim of the SIBLINGS for reversion or any action which would have the effect of of ownership. We are rather inclined to consider this fact
OF CECILIO who filed a complaint in court only in 1976 to cancelling a free patent and the corresponding certificate as an admission of non-ownership. And when we consider
enforce a light acquired allegedly as early as 1930, is of title issued on the basis thereof, with the result that the also that the petitioners HEIRS OF CECILIO had
difficult to comprehend. land covered thereby will again form part of the public individually paid to the municipal treasury the taxes
domain, as only the Solicitor General or the officer acting corresponding to the particular portions they were
The Civil Code states: in his stead may do so.16 occupying,22 we can readily see the superiority of the
petitioners' position.
Art. 1145. The following actions must be It is true that in some instances, the Court did away with
commenced within six years: the irrevocability of the torrens title, but the circumstances Renato Solema and Decimina Calvez, two of the
in the case at bar varied significantly from these cases. respondents who derive their right from the SIBLINGS OF
(1) Upon an oral contract . . . (Emphasis supplied). CLAUDEL, bought a portion of the lot from Felisa Claudel,
In Bornales v. IAC, 17 the defense of indefeasibility of a one of the HEIRS OF CLAUDEL.23 The Calvezes should
If the parties SIBLINGS OF CECILIO had allegedly certificate of title was disregarded when the transferee not be paying for a lot that they already owned and if they
derived their right of action from the oral purchase made who took it had notice of the flaws in the transferor's title. did not acknowledge Felisa as its owner.
by their parents in 1930, then the action filed in 1976 No right passed to a transferee from a vendor who did not
would have clearly prescribed. More than six years had have any in the first place. The transferees bought the In addition, before any of the SIBLINGS OF CECILIO
lapsed. land registered under the torrens system from vendors could stay on any of the portions of the property, they had
who procured title thereto by means of fraud. With this to ask first the permission of Jose Claudel again, one of
We do not agree with the parties SIBLINGS OF CECILIO knowledge, they can not invoke the indefeasibility of a the HEIRS OF CECILIO.24 In fact the only reason why
when they reason that an implied trust in favor of the certificate of title against the private respondent to the any of the heirs of SIBLINGS OF CECILIO could stay on
SIBLINGS OF CECILIO was established in 1972, when extent of her interest. This is because the torrens system the lot was because they were allowed to do so by the
the HEIRS OF CECILIO executed a contract of partition of land registration, though indefeasible, should not be HEIRS OF CECILIO.25
over the said properties. used as a means to perpetrate fraud against the rightful
owner of real property. In view of the foregoing, we find that the appellate court
But as we had pointed out, the law recognizes the committed a reversible error in denigrating the transfer
superiority of the torrens title. Mere registration of the sale is not good enough, good certificates of title of the petitioners to the survey or
faith must concur with registration. Otherwise registration subdivision plan proffered by the private respondents. The
Above all, the torrens title in the possession of the HEIRS becomes an exercise in futility.18 Court generally recognizes the profundity of conclusions
OF CECILIO carries more weight as proof of ownership and findings of facts reached by the trial court and hence
than the survey or subdivision plan of a parcel of land in In Amerol v. Bagumbaran,19 we reversed the decision of sustains them on appeal except for strong and cogent
the name of SIBLINGS OF CECILIO. the trial court. In this case, the title was wrongfully reasons inasmuch as the trial court is in a better position
123
Sales – Chapter 3-12 Cases
to examine real evidence and observe the demeanor of
witnesses in a case.

No clear specific contrary evidence was cited by the


respondent appellate court to justify the reversal of the
lower court's findings. Thus, in this case, between the
factual findings of the trial court and the appellate court,
those of the trial court must prevail over that of the
latter.26

WHEREFORE, the petition is GRANTED We REVERSE


and SET ASIDE the decision rendered in CA-G.R. CV No.
04429, and we hereby REINSTATE the decision of the
then Court of First Instance of Rizal (Branch 28, Pasay
City) in Civil Case No. M-5276-P which ruled for the
dismissal of the Complaint for Cancellation of Titles and
Reconveyance with Damages filed by the Heirs of
Macario, Esperidiona Raymunda, and Celestina, all
surnamed CLAUDEL. Costs against the private
respondents.

SO ORDERED.

124
Sales – Chapter 3-12 Cases
SPOUSES GODOFREDO ALFREDO and CARMEN No. 284 to Godofredo and Carmen. Armando and Adelia evidencing the alleged contract of sale over the Subject
LIMON ALFREDO, SPOUSES ARNULFO SAVELLANO subsequently paid the balance of the purchase price of Land in favor of Armando and Adelia. Petitioners objected
and EDITHA B. SAVELLANO, DANTON D. the Subject Land for which Carmen issued a receipt dated to whatever parole evidence Armando and Adelia
MATAWARAN, SPOUSES DELFIN F. ESPIRITU, JR. 11 March 1970. Godofredo and Carmen then delivered to introduced or offered on the alleged sale unless the same
and ESTELA S. ESPIRITU and ELIZABETH TUAZON, Adelia the owners duplicate copy of OCT No. 284, with was in writing and subscribed by Godofredo. Petitioners
petitioners, vs. SPOUSES ARMANDO BORRAS and the document of cancellation of mortgage, official receipts asserted that the Subsequent Buyers were buyers in good
ADELIA LOBATON BORRAS, respondents. of realty tax payments, and tax declaration in the name of faith and for value. As counterclaim, petitioners sought
Godofredo. Godofredo and Carmen introduced Armando payment of attorneys fees and incidental expenses.
The Case and Adelia, as the new owners of the Subject Land, to the
Natanawans, the old tenants of the Subject Land. Trial then followed. Armando and Adelia presented the
Before us is a petition for review assailing the Decision[1] Armando and Adelia then took possession of the Subject following witnesses: Adelia, Jesus Lobaton, Roberto
of the Court of Appeals dated 26 November 1999 Land. Lopez, Apolinario Natanawan, Rolando Natanawan,
affirming the decision[2] of the Regional Trial Court of Tomas Natanawan, and Mildred Lobaton. Petitioners
Bataan, Branch 4, in Civil Case No. DH-256-94. In January 1994, Armando and Adelia learned that hired presented two witnesses, Godofredo and Constancia
Petitioners also question the Resolution of the Court of persons had entered the Subject Land and were cutting Calonso.
Appeals dated 26 July 2000 denying petitioners motion for trees under instructions of allegedly new owners of the
reconsideration. Subject Land. Subsequently, Armando and Adelia On 7 June 1996, the trial court rendered its decision in
discovered that Godofredo and Carmen had re-sold favor of Armando and Adelia. The dispositive portion of
The Antecedent Facts portions of the Subject Land to several persons. the decision reads:

A parcel of land measuring 81,524 square meters (Subject On 8 February 1994, Armando and Adelia filed an WHEREFORE, premises considered, judgment is hereby
Land) in Barrio Culis, Mabiga, Hermosa, Bataan is the adverse claim with the Register of Deeds of Bataan. rendered in favor of plaintiffs, the spouses Adelia Lobaton
subject of controversy in this case. The registered owners Armando and Adelia discovered that Godofredo and Borras and Armando F. Borras, and against the
of the Subject Land were petitioner spouses, Godofredo Carmen had secured an owners duplicate copy of OCT defendant-spouses Godofredo Alfredo and Carmen Limon
Alfredo (Godofredo) and Carmen Limon Alfredo (Carmen). No. 284 after filing a petition in court for the issuance of a Alfredo, spouses Arnulfo Sabellano and Editha B.
The Subject Land is covered by Original Certificate of Title new copy. Godofredo and Carmen claimed in their petition Sabellano, spouses Delfin F. Espiritu, Jr. and Estela S.
No. 284 (OCT No. 284) issued to Godofredo and Carmen that they lost their owners duplicate copy. Armando and Espiritu, Danton D. Matawaran and Elizabeth Tuazon, as
under Homestead Patent No. V-69196. Adelia wrote Godofredo and Carmen complaining about follows:
their acts, but the latter did not reply. Thus, Armando and
On 7 March 1994, the private respondents, spouses Adelia filed a complaint for specific performance. 1. Declaring the Deeds of Absolute Sale of the disputed
Armando Borras (Armando) and Adelia Lobaton Borras parcel of land (covered by OCT No. 284) executed by the
(Adelia), filed a complaint for specific performance against On 28 March 1994, Armando and Adelia amended their spouses Godofredo Alfredo and Camen Limon Alfredo in
Godofredo and Carmen before the Regional Trial Court of complaint to include the following persons as additional favor of spouses Arnulfo Sabellano and Editha B.
Bataan, Branch 4. The case was docketed as Civil Case defendants: the spouses Arnulfo Savellano and Editha B. Sabellano, spouses Delfin F. Espiritu, Danton D.
No. DH-256-94. Savellano, Danton D. Matawaran, the spouses Delfin F. Matawaran and Elizabeth Tuazon, as null and void;
Espiritu, Jr. and Estela S. Espiritu, and Elizabeth Tuazon
Armando and Adelia alleged in their complaint that (Subsequent Buyers). The Subsequent Buyers, who are 2. Declaring the Transfer Certificates of Title Nos. T-
Godofredo and Carmen mortgaged the Subject Land for also petitioners in this case, purchased from Godofredo 163266 and T-163267 in the names of spouses Arnulfo
P7,000.00 with the Development Bank of the Philippines and Carmen the subdivided portions of the Subject Land. Sabellano and Editha B. Sabellano; Transfer Certificates
(DBP). To pay the debt, Carmen and Godofredo sold the The Register of Deeds of Bataan issued to the of Title Nos. T-163268 and 163272 in the names of
Subject Land to Armando and Adelia for P15,000.00, the Subsequent Buyers transfer certificates of title to the lots spouses Delfin F. Espiritu, Jr. and Estela S. Espiritu;
buyers to pay the DBP loan and its accumulated interest, they purchased. Transfer Certificates of Title Nos. T-163269 and T-163271
and the balance to be paid in cash to the sellers. in the name of Danton D. Matawaran; and Transfer
In their answer, Godofredo and Carmen and the Certificate of Title No. T-163270 in the name of Elizabeth
Armando and Adelia gave Godofredo and Carmen the Subsequent Buyers (collectively petitioners) argued that Tuazon, as null and void and that the Register of Deeds of
money to pay the loan to DBP which signed the release of the action is unenforceable under the Statute of Frauds. Bataan is hereby ordered to cancel said titles;
mortgage and returned the owners duplicate copy of OCT Petitioners pointed out that there is no written instrument
125
Sales – Chapter 3-12 Cases
3. Ordering the defendant-spouses Godofredo Alfredo negotiated with Godofredo and Carmen the sale of the
and Carmen Limon Alfredo to execute and deliver a good SO ORDERED.[4] Subject Land which Godofredo and Carmen subdivided so
and valid Deed of Absolute Sale of the disputed parcel of they could sell anew portions to the Subsequent Buyers.
land (covered by OCT No. 284) in favor of the spouses On 26 July 2000, the Court of Appeals denied petitioners
Adelia Lobaton Borras and Armando F. Borras within a motion for reconsideration. Calonso admitted that the Subject Land was adjacent to
period of ten (10) days from the finality of this decision; her own lot. The trial court pointed out that Calonso did
The Ruling of the Trial Court not inquire on the nature of the tenancy of the
4. Ordering defendant-spouses Godofredo Alfredo and Natanawans and on who owned the Subject Land.
Carmen Limon Alfredo to surrender their owners duplicate The trial court ruled that there was a perfected contract of Instead, she bought out the tenants for P150,000.00. The
copy of OCT No. 284 issued to them by virtue of the Order sale between the spouses Godofredo and Carmen and buy out was embodied in a Kasunduan. Apolinario
dated May 20, 1992 of the Regional Trial Court of Bataan, the spouses Armando and Adelia. The trial court found Natanawan (Apolinario) testified that he and his wife
Dinalupihan Branch, to the Registry of Deeds of Bataan that all the elements of a contract of sale were present in accepted the money and signed the Kasunduan because
within ten (10) days from the finality of this decision, who, this case. The object of the sale was specifically identified Calonso and the Subsequent Buyers threatened them
in turn, is directed to cancel the same as there exists in as the 81,524-square meter lot in Barrio Culis, Mabigas, with forcible ejectment. Calonso brought Apolinario to the
the possession of herein plaintiffs of the owners duplicate Hermosa, Bataan, covered by OCT No. 284 issued by the Agrarian Reform Office where he was asked to produce
copy of said OCT No. 284 and, to restore and/or reinstate Registry of Deeds of Bataan. The purchase price was the documents showing that Adelia is the owner of the
OCT No. 284 of the Register of Deeds of Bataan to its full fixed at P15,000.00, with the buyers assuming to pay the Subject Land. Since Apolinario could not produce the
force and effect; sellers P7,000.00 DBP mortgage loan including its documents, the agrarian officer told him that he would
accumulated interest. The balance of the purchase price lose the case. Thus, Apolinario was constrained to sign
5. Ordering the defendant-spouses Godofredo Alfredo was to be paid in cash to the sellers. The last payment of the Kasunduan and accept the P150,000.00.
and Carmen Limon Alfredo to restitute and/or return the P2,524.00 constituted the full settlement of the purchase
amount of the respective purchase prices and/or price and this was paid on 11 March 1970 as evidenced Another indication of Calonsos bad faith was her own
consideration of sale of the disputed parcels of land they by the receipt issued by Carmen. admission that she saw an adverse claim on the title of
sold to their co-defendants within ten (10) days from the the Subject Land when she registered the deeds of sale in
finality of this decision with legal interest thereon from The trial court found the following facts as proof of a the names of the Subsequent Buyers. Calonso ignored
date of the sale; perfected contract of sale: (1) Godofredo and Carmen the adverse claim and proceeded with the registration of
delivered to Armando and Adelia the Subject Land; (2) the deeds of sale.
6. Ordering the defendants, jointly and severally, to pay Armando and Adelia treated as their own tenants the
plaintiff-spouses the sum of P20,000.00 as and for tenants of Godofredo and Carmen; (3) Godofredo and The trial court awarded P20,000.00 as attorneys fees to
attorneys fees and litigation expenses; and Carmen turned over to Armando and Adelia documents Armando and Adelia. In justifying the award of attorneys
such as the owners duplicate copy of the title of the fees, the trial court invoked Article 2208 (2) of the Civil
7. Ordering defendants to pay the costs of suit. Subject Land, tax declaration, and the receipts of realty Code which allows a court to award attorneys fees,
tax payments in the name of Godofredo; and (4) the DBP including litigation expenses, when it is just and equitable
Defendants counterclaims are hereby dismissed for lack cancelled the mortgage on the Subject Property upon to award the same. The trial court ruled that Armando and
of merit. payment of the loan of Godofredo and Carmen. Moreover, Adelia are entitled to attorneys fees since they were
the receipt of payment issued by Carmen served as an compelled to file this case due to petitioners refusal to
SO ORDERED.[3] acknowledgment, if not a ratification, of the verbal sale heed their just and valid demand.
between the sellers and the buyers. The trial court ruled
Petitioners appealed to the Court of Appeals. that the Statute of Frauds is not applicable because in this The Ruling of the Court of Appeals
case the sale was perfected.
On 26 November 1999, the Court of Appeals issued its The Court of Appeals found the factual findings of the trial
Decision affirming the decision of the trial court, thus: The trial court concluded that the Subsequent Buyers court well supported by the evidence. Based on these
were not innocent purchasers. Not one of the Subsequent findings, the Court of Appeals also concluded that there
WHEREFORE, premises considered, the appealed Buyers testified in court on how they purchased their was a perfected contract of sale and the Subsequent
decision in Civil Case No. DH-256-94 is hereby respective lots. The Subsequent Buyers totally depended Buyers were not innocent purchasers.
AFFIRMED in its entirety. Treble costs against the on the testimony of Constancia Calonso (Calonso) to
defendants-appellants. explain the subsequent sale. Calonso, a broker,
126
Sales – Chapter 3-12 Cases
The Court of Appeals ruled that the handwritten receipt second sale was tainted with bad faith from the very good faith and for value whose individual titles to their
dated 11 March 1970 is sufficient proof that Godofredo beginning. respective lots are absolute and indefeasible, are valid.
and Carmen sold the Subject Land to Armando and
Adelia upon payment of the balance of the purchase Finally, the Court of Appeals noted that the issue of IV
price. The Court of Appeals found the recitals in the prescription was not raised in the Answer. Nonetheless,
receipt as sufficient to serve as the memorandum or note the appellate court explained that since this action is Whether petitioners are liable to pay Armando and Adelia
as a writing under the Statute of Frauds.[5] The Court of actually based on fraud, the prescriptive period is four P20,0000.00 as attorneys fees and litigation expenses
Appeals then reiterated the ruling of the trial court that the years, with the period starting to run only from the date of and the treble costs, where the claim of Armando and
Statute of Frauds does not apply in this case. the discovery of the fraud. Armando and Adelia Adelia is clearly unfounded and baseless.
discovered the fraudulent sale of the Subject Land only in
The Court of Appeals gave credence to the testimony of a January 1994. Armando and Adelia lost no time in writing V
witness of Armando and Adelia, Mildred Lobaton, who a letter to Godofredo and Carmen on 2 February 1994
explained why the title to the Subject Land was not in the and filed this case on 7 March 1994. Plainly, Armando and Whether petitioners are entitled to the counterclaim for
name of Armando and Adelia. Lobaton testified that Adelia did not sleep on their rights or lose their rights by attorneys fees and litigation expenses, where they have
Godofredo was then busy preparing to leave for Davao. prescription. sustained such expenses by reason of institution of a
Godofredo promised that he would sign all the papers clearly malicious and unfounded action by Armando and
once they were ready. Since Armando and Adelia were The Court of Appeals sustained the award of attorneys Adelia.[8]
close to the family of Carmen, they trusted Godofredo and fees and imposed treble costs on petitioners.
Carmen to honor their commitment. Armando and Adelia The Courts Ruling
had no reason to believe that their contract of sale was The Issues
not perfected or validly executed considering that they had The petition is without merit.
received the duplicate copy of OCT No. 284 and other Petitioners raise the following issues:
relevant documents. Moreover, they had taken physical In a petition for review on certiorari under Rule 45, this
possession of the Subject Land. I Court reviews only errors of law and not errors of facts.[9]
The factual findings of the appellate court are generally
The Court of Appeals held that the contract of sale is not Whether the alleged sale of the Subject Land in favor of binding on this Court.[10] This applies with greater force
void even if only Carmen signed the receipt dated 11 Armando and Adelia is valid and enforceable, where (1) it when both the trial court and the Court of Appeals are in
March 1970. Citing Felipe v. Heirs of Maximo Aldon,[6] the was orally entered into and not in writing; (2) Carmen did complete agreement on their factual findings.[11] In this
appellate court ruled that a contract of sale made by the not obtain the consent and authority of her husband, case, there is no reason to deviate from the findings of the
wife without the husbands consent is not void but merely Godofredo, who was the sole owner of the Subject Land lower courts. The facts relied upon by the trial and
voidable. The Court of Appeals further declared that the in whose name the title thereto (OCT No. 284) was appellate courts are borne out by the record. We agree
sale in this case binds the conjugal partnership even if issued; and (3) it was entered into during the 25-year with the conclusions drawn by the lower courts from these
only the wife signed the receipt because the proceeds of prohibitive period for alienating the Subject Land without facts.
the sale were used for the benefit of the conjugal the approval of the Secretary of Agriculture and Natural
partnership. The appellate court based this conclusion on Resources. Validity and Enforceability of the Sale
Article 161[7] of the Civil Code.
II The contract of sale between the spouses Godofredo and
The Subsequent Buyers of the Subject Land cannot claim Carmen and the spouses Armando and Adelia was a
that they are buyers in good faith because they had Whether the action to enforce the alleged oral contract of perfected contract. A contract is perfected once there is
constructive notice of the adverse claim of Armando and sale brought after 24 years from its alleged perfection had consent of the contracting parties on the object certain
Adelia. Calonso, who brokered the subsequent sale, been barred by prescription and by laches. and on the cause of the obligation.[12] In the instant case,
testified that when she registered the subsequent deeds the object of the sale is the Subject Land, and the price
of sale, the adverse claim of Armando and Adelia was III certain is P15,000.00. The trial and appellate courts found
already annotated on the title of the Subject Land. The that there was a meeting of the minds on the sale of the
Court of Appeals believed that the act of Calonso and the Whether the deeds of absolute sale and the transfer Subject Land and on the purchase price of P15,000.00.
Subsequent Buyers in forcibly ejecting the Natanawans certificates of title over the portions of the Subject Land This is a finding of fact that is binding on this Court. We
from the Subject Land buttresses the conclusion that the issued to the Subsequent Buyers, innocent purchasers in
127
Sales – Chapter 3-12 Cases
find no reason to disturb this finding since it is supported performed.[17] Thus, where one party has performed The wife may, during the marriage, and within ten years
by substantial evidence. ones obligation, oral evidence will be admitted to prove from the transaction questioned, ask the courts for the
the agreement.[18] In the instant case, the parties have annulment of any contract of the husband entered into
The contract of sale of the Subject Land has also been consummated the sale of the Subject Land, with both without her consent, when such consent is required, or
consummated because the sellers and buyers have sellers and buyers performing their respective obligations any act or contract of the husband which tends to defraud
performed their respective obligations under the contract. under the contract of sale. In addition, a contract that her or impair her interest in the conjugal partnership
In a contract of sale, the seller obligates himself to transfer violates the Statute of Frauds is ratified by the acceptance property. Should the wife fail to exercise this right, she or
the ownership of the determinate thing sold, and to deliver of benefits under the contract.[19] Godofredo and Carmen her heirs, after the dissolution of the marriage, may
the same, to the buyer who obligates himself to pay a benefited from the contract because they paid their DBP demand the value of property fraudulently alienated by the
price certain to the seller.[13] In the instant case, loan and secured the cancellation of their mortgage using husband.
Godofredo and Carmen delivered the Subject Land to the money given by Armando and Adelia. Godofredo and
Armando and Adelia, placing the latter in actual physical Carmen also accepted payment of the balance of the In Felipe v. Aldon,[23] we applied Article 173 in a case
possession of the Subject Land. This physical delivery of purchase price. where the wife sold some parcels of land belonging to the
the Subject Land also constituted a transfer of ownership conjugal partnership without the consent of the husband.
of the Subject Land to Armando and Adelia.[14] Godofredo and Carmen cannot invoke the Statute of We ruled that the contract of sale was voidable subject to
Ownership of the thing sold is transferred to the vendee Frauds to deny the existence of the verbal contract of sale annulment by the husband. Following petitioners
upon its actual or constructive delivery.[15] Godofredo and because they have performed their obligations, and have argument that Carmen sold the land to Armando and
Carmen also turned over to Armando and Adelia the accepted benefits, under the verbal contract. [20] Adelia without the consent of Carmens husband, the sale
documents of ownership to the Subject Land, namely the Armando and Adelia have also performed their obligations would only be voidable and not void.
owners duplicate copy of OCT No. 284, the tax under the verbal contract. Clearly, both the sellers and the
declaration and the receipts of realty tax payments. buyers have consummated the verbal contract of sale of However, Godofredo can no longer question the sale.
the Subject Land. The Statute of Frauds was enacted to Voidable contracts are susceptible of ratification.[24]
On the other hand, Armando and Adelia paid the full prevent fraud.[21] This law cannot be used to advance the Godofredo ratified the sale when he introduced Armando
purchase price as evidenced by the receipt dated 11 very evil the law seeks to prevent. and Adelia to his tenants as the new owners of the
March 1970 issued by Carmen. Armando and Adelia Subject Land. The trial court noted that Godofredo failed
fulfilled their obligation to provide the P7,000.00 to pay the Godofredo and Carmen also claim that the sale of the to deny categorically on the witness stand the claim of the
DBP loan of Godofredo and Carmen, and to pay the latter Subject Land to Armando and Adelia is void on two complainants witnesses that Godofredo introduced
the balance of P8,000.00 in cash. The P2,524.00 paid grounds. First, Carmen sold the Subject Land without the Armando and Adelia as the new landlords of the
under the receipt dated 11 March 1970 was the last marital consent of Godofredo. Second, the sale was made tenants.[25] That Godofredo and Carmen allowed
installment to settle fully the purchase price. Indeed, upon during the 25-year period that the law prohibits the Armando and Adelia to enjoy possession of the Subject
payment to DBP of the P7,000.00 and the accumulated alienation of land grants without the approval of the Land for 24 years is formidable proof of Godofredos
interests, the DBP cancelled the mortgage on the Subject Secretary of Agriculture and Natural Resources. acquiescence to the sale. If the sale was truly
Land and returned the owners duplicate copy of OCT No. unauthorized, then Godofredo should have filed an action
284 to Godofredo and Carmen. These arguments are without basis. to annul the sale. He did not. The prescriptive period to
annul the sale has long lapsed. Godofredos conduct
The trial and appellate courts correctly refused to apply The Family Code, which took effect on 3 August 1988, belies his claim that his wife sold the Subject Land without
the Statute of Frauds to this case. The Statute of provides that any alienation or encumbrance made by the his consent.
Frauds[16] provides that a contract for the sale of real husband of the conjugal partnership property without the
property shall be unenforceable unless the contract or consent of the wife is void. However, when the sale is Moreover, Godofredo and Carmen used most of the
some note or memorandum of the sale is in writing and made before the effectivity of the Family Code, the proceeds of the sale to pay their debt with the DBP. We
subscribed by the party charged or his agent. The applicable law is the Civil Code.[22] agree with the Court of Appeals that the sale redounded
existence of the receipt dated 11 March 1970, which is a to the benefit of the conjugal partnership. Article 161 of
memorandum of the sale, removes the transaction from Article 173 of the Civil Code provides that the disposition the Civil Code provides that the conjugal partnership shall
the provisions of the Statute of Frauds. of conjugal property without the wifes consent is not void be liable for debts and obligations contracted by the wife
but merely voidable. Article 173 reads: for the benefit of the conjugal partnership. Hence, even if
The Statute of Frauds applies only to executory contracts Carmen sold the land without the consent of her husband,
and not to contracts either partially or totally the sale still binds the conjugal partnership.
128
Sales – Chapter 3-12 Cases
contravene the basic rules of fair play, justice and due approval may be secured later, producing the effect of
Petitioners contend that Godofredo and Carmen did not process.[28] However, we will address this new issue to ratifying and adopting the transaction as if the sale had
deliver the title of the Subject Land to Armando and Adelia finally put an end to this case. been previously authorized.[35] As held in Evangelista v.
as shown by this portion of Adelias testimony on cross- Montano:[36]
examination: The sale of the Subject Land cannot be annulled on the
ground that the Secretary did not approve the sale, which Section 118 of Commonwealth Act No. 141, as amended,
Q -- No title was delivered to you by Godofredo Alfredo? was made within 25 years from the issuance of the specifically enjoins that the approval by the Department
homestead title. Section 118 of the Public Land Act Secretary "shall not be denied except on constitutional
A -- I got the title from Julie Limon because my sister told (Commonwealth Act No. 141) reads as follows: and legal grounds." There being no allegation that there
me.[26] were constitutional or legal impediments to the sales, and
SEC. 118. Except in favor of the Government or any of its no pretense that if the sales had been submitted to the
Petitioners raise this factual issue for the first time. The branches, units, or institutions or legally constituted Secretary concerned they would have been disapproved,
Court of Appeals could have passed upon this issue had banking corporation, lands acquired under free patent or approval was a ministerial duty, to be had as a matter of
petitioners raised this earlier. At any rate, the cited homestead provisions shall not be subject to course and demandable if refused. For this reason, and if
testimony of Adelia does not convincingly prove that encumbrance or alienation from the date of the approval necessary, approval may now be applied for and its effect
Godofredo and Carmen did not deliver the Subject Land of the application and for a term of five years from and will be to ratify and adopt the transactions as if they had
to Armando and Adelia. Adelias cited testimony must be after the date of the issuance of the patent or grant. been previously authorized. (Emphasis supplied)
examined in context not only with her entire testimony but
also with the other circumstances. xxx Action Not Barred by Prescription and Laches

Adelia stated during cross-examination that she obtained No alienation, transfer, or conveyance of any homestead Petitioners insist that prescription and laches have set in.
the title of the Subject Land from Julie Limon (Julie), her after 5 years and before twenty-five years after the We disagree.
classmate in college and the sister of Carmen. Earlier, issuance of title shall be valid without the approval of the
Adelias own sister had secured the title from the father of Secretary of Agriculture and Commerce, which approval The Amended Complaint filed by Armando and Adelia
Carmen. However, Adelias sister, who was about to leave shall not be denied except on constitutional and legal with the trial court is captioned as one for Specific
for the United States, gave the title to Julie because of the grounds. Performance. In reality, the ultimate relief sought by
absence of the other documents. Adelias sister told Adelia Armando and Adelia is the reconveyance to them of the
to secure the title from Julie, and this was how Adelia A grantee or homesteader is prohibited from alienating to Subject Land. An action for reconveyance is one that
obtained the title from Julie. a private individual a land grant within five years from the seeks to transfer property, wrongfully registered by
time that the patent or grant is issued.[29] A violation of another, to its rightful and legal owner.[37] The body of the
It is not necessary that the seller himself deliver the title of this prohibition renders a sale void.[30] This prohibition, pleading or complaint determines the nature of an action,
the property to the buyer because the thing sold is however, expires on the fifth year. From then on until the not its title or heading.[38] Thus, the present action should
understood as delivered when it is placed in the control next 20 years[31] the land grant may be alienated be treated as one for reconveyance.[39]
and possession of the vendee.[27] To repeat, Godofredo provided the Secretary of Agriculture and Natural
and Carmen themselves introduced the Natanawans, their Resources approves the alienation. The Secretary is Article 1456 of the Civil Code provides that a person
tenants, to Armando and Adelia as the new owners of the required to approve the alienation unless there are acquiring property through fraud becomes by operation of
Subject Land. From then on, Armando and Adelia acted constitutional and legal grounds to deny the approval. In law a trustee of an implied trust for the benefit of the real
as the landlords of the Natanawans. Obviously, this case, there are no apparent constitutional or legal owner of the property. The presence of fraud in this case
Godofredo and Carmen themselves placed control and grounds for the Secretary to disapprove the sale of the created an implied trust in favor of Armando and Adelia.
possession of the Subject Land in the hands of Armando Subject Land. This gives Armando and Adelia the right to seek
and Adelia. reconveyance of the property from the Subsequent
The failure to secure the approval of the Secretary does Buyers.[40]
Petitioners invoke the absence of approval of the sale by not ipso facto make a sale void.[32] The absence of
the Secretary of Agriculture and Natural Resources to approval by the Secretary does not nullify a sale made To determine when the prescriptive period commenced in
nullify the sale. Petitioners never raised this issue before after the expiration of the 5-year period, for in such event an action for reconveyance, plaintiffs possession of the
the trial court or the Court of Appeals. Litigants cannot the requirement of Section 118 of the Public Land Act disputed property is material. An action for reconveyance
raise an issue for the first time on appeal, as this would becomes merely directory[33] or a formality.[34] The based on an implied trust prescribes in ten years.[41] The
129
Sales – Chapter 3-12 Cases
ten-year prescriptive period applies only if there is an law (Art. 1456, Civil Code), so is the corresponding the rights of any innocent holder of the decree of
actual need to reconvey the property as when the plaintiff obligation to reconvey the property and the title thereto in registration on the original petition or application, xxx
is not in possession of the property.[42] However, if the favor of the true owner. In this context, and vis-a-vis
plaintiff, as the real owner of the property also remains in prescription, Article 1144 of the Civil Code is applicable. This provision should be read in conjunction with Article
possession of the property, the prescriptive period to 1456 of the Civil Code, which provides:
recover title and possession of the property does not run Article 1144. The following actions must be brought within
against him.[43] In such a case, an action for ten years from the time the right of action accrues: Article 1456. If property is acquired through mistake or
reconveyance, if nonetheless filed, would be in the nature fraud, the person obtaining it is, by force of law,
of a suit for quieting of title, an action that is (1) Upon a written contract; considered a trustee of an implied trust for the benefit of
imprescriptible.[44] the person from whom the property comes.
(2) Upon an obligation created by law;
In this case, the appellate court resolved the issue of The law thereby creates the obligation of the trustee to
prescription by ruling that the action should prescribe four (3) Upon a judgment. reconvey the property and the title thereto in favor of the
years from discovery of the fraud. We must correct this true owner. Correlating Section 53, paragraph 3 of
erroneous application of the four-year prescriptive period. xxxxxxxxx Presidential Decree No. 1529 and Article 1456 of the Civil
In Caro v. Court of Appeals,[45] we explained why an Code with Article 1144(2) of the Civil Code, supra, the
action for reconveyance based on an implied trust should (Emphasis supplied). prescriptive period for the reconveyance of fraudulently
prescribe in ten years. In that case, the appellate court registered real property is ten (10) years reckoned from
also erroneously applied the four-year prescriptive period. An action for reconveyance based on an implied or the date of the issuance of the certificate of title xxx
We declared in Caro: constructive trust must perforce prescribe in ten years and (Emphasis supplied)[46]
not otherwise. A long line of decisions of this Court, and of
We disagree. The case of Liwalug Amerol, et al. v. Molok very recent vintage at that, illustrates this rule. Following Caro, we have consistently held that an action
Bagumbaran, G.R. No. L-33261, September 30, 1987,154 Undoubtedly, it is now well-settled that an action for for reconveyance based on an implied trust prescribes in
SCRA 396 illuminated what used to be a gray area on the reconveyance based on an implied or constructive trust ten years.[47] We went further by specifying the reference
prescriptive period for an action to reconvey the title to prescribes in ten years from the issuance of the Torrens point of the ten-year prescriptive period as the date of the
real property and, corollarily, its point of reference: title over the property. The only discordant note, it seems, registration of the deed or the issuance of the title.[48]
is Balbin vs. Medalla which states that the prescriptive
xxx It must be remembered that before August 30, 1950, period for a reconveyance action is four years. However, Had Armando and Adelia remained in possession of the
the date of the effectivity of the new Civil Code, the old this variance can be explained by the erroneous reliance Subject Land, their action for reconveyance, in effect an
Code of Civil Procedure (Act No. 190) governed on Gerona vs. de Guzman. But in Gerona, the fraud was action to quiet title to property, would not be subject to
prescription. It provided: discovered on June 25,1948, hence Section 43(3) of Act prescription. Prescription does not run against the plaintiff
No. 190, was applied, the new Civil Code not coming into in actual possession of the disputed land because such
SEC. 43. Other civil actions; how limited.- Civil actions effect until August 30, 1950 as mentioned earlier. It must plaintiff has a right to wait until his possession is disturbed
other than for the recovery of real property can only be be stressed, at this juncture, that article 1144 and article or his title is questioned before initiating an action to
brought within the following periods after the right of action 1456, are new provisions. They have no counterparts in vindicate his right.[49] His undisturbed possession gives
accrues: the old Civil Code or in the old Code of Civil Procedure, him the continuing right to seek the aid of a court of equity
the latter being then resorted to as legal basis of the four- to determine the nature of the adverse claim of a third
xxx xxx xxx year prescriptive period for an action for reconveyance of party and its effect on his title.[50]
title of real property acquired under false pretenses.
3. Within four years: xxx An action for relief on the ground Armando and Adelia lost possession of the Subject Land
of fraud, but the right of action in such case shall not be An action for reconveyance has its basis in Section 53, when the Subsequent Buyers forcibly drove away from the
deemed to have accrued until the discovery of the fraud; paragraph 3 of Presidential Decree No. 1529, which Subject Land the Natanawans, the tenants of Armando
provides: and Adelia.[51] This created an actual need for Armando
xxx xxx xxx and Adelia to seek reconveyance of the Subject Land.
In all cases of registration procured by fraud, the owner The statute of limitation becomes relevant in this case.
In contrast, under the present Civil Code, we find that just may pursue all his legal and equitable remedies against The ten-year prescriptive period started to run from the
as an implied or constructive trust is an offspring of the the parties to such fraud without prejudice, however, to
130
Sales – Chapter 3-12 Cases
date the Subsequent Buyers registered their deeds of sale knowledge of the prior sale cannot be a registrant in good We sustain the award of attorneys fees. The decision of
with the Register of Deeds. faith.[55] Such second buyer cannot defeat the first buyers the court must state the grounds for the award of
title.[56] In case a title is issued to the second buyer, the attorneys fees. The trial court complied with this
The Subsequent Buyers bought the subdivided portions of first buyer may seek reconveyance of the property subject requirement.[65] We agree with the trial court that if it
the Subject Land on 22 February 1994, the date of of the sale.[57] were not for petitioners unjustified refusal to heed the just
execution of their deeds of sale. The Register of Deeds and valid demands of Armando and Adelia, the latter
issued the transfer certificates of title to the Subsequent Thus, to merit protection under the second paragraph of would not have been compelled to file this action.
Buyers on 24 February 1994. Armando and Adelia filed Article 1544[58] of the Civil Code, the second buyer must
the Complaint on 7 March 1994. Clearly, prescription act in good faith in registering the deed.[59] In this case, The Court of Appeals echoed the trial courts
could not have set in since the case was filed at the early the Subsequent Buyers good faith hinges on whether they condemnation of petitioners fraudulent maneuverings in
stage of the ten-year prescriptive period. had knowledge of the previous sale. Petitioners do not securing the second sale of the Subject Land to the
dispute that Armando and Adelia registered their adverse Subsequent Buyers. We will also not turn a blind eye on
Neither is the action barred by laches. We have defined claim with the Registry of Deeds of Bataan on 8 February petitioners brazen tactics. Thus, we uphold the treble
laches as the failure or neglect, for an unreasonable time, 1994. The Subsequent Buyers purchased their respective costs imposed by the Court of Appeals on petitioners.
to do that which, by the exercise of due diligence, could or lots only on 22 February 1994 as shown by the date of
should have been done earlier.[52] It is negligence or their deeds of sale. Consequently, the adverse claim WHEREFORE, the petition is DENIED and the appealed
omission to assert a right within a reasonable time, registered prior to the second sale charged the decision is AFFIRMED. Treble costs against petitioners.
warranting a presumption that the party entitled to assert it Subsequent Buyers with constructive notice of the defect
either has abandoned it or declined to assert it.[53] in the title of the sellers,[60] Godofredo and Carmen. SO ORDERED.
Armando and Adelia discovered in January 1994 the
subsequent sale of the Subject Land and they filed this It is immaterial whether Calonso, the broker of the second
case on 7 March 1994. Plainly, Armando and Adelia did sale, communicated to the Subsequent Buyers the
not sleep on their rights. existence of the adverse claim. The registration of the
adverse claim on 8 February 1994 constituted, by
Validity of Subsequent Sale of Portions of the Subject operation of law, notice to the whole world.[61] From that
Land date onwards, the Subsequent Buyers were deemed to
have constructive notice of the adverse claim of Armando
Petitioners maintain that the subsequent sale must be and Adelia. When the Subsequent Buyers purchased
upheld because the Subsequent Buyers, the co- portions of the Subject Land on 22 February 1994, they
petitioners of Godofredo and Carmen, purchased and already had constructive notice of the adverse claim
registered the Subject Land in good faith. Petitioners registered earlier.[62] Thus, the Subsequent Buyers were
argue that the testimony of Calonso, the person who not buyers in good faith when they purchased their lots on
brokered the second sale, should not prejudice the 22 February 1994. They were also not registrants in good
Subsequent Buyers. There is no evidence that Calonso faith when they registered their deeds of sale with the
was the agent of the Subsequent Buyers and that she Registry of Deeds on 24 February 1994.
communicated to them what she knew about the adverse
claim and the prior sale. Petitioners assert that the The Subsequent Buyers individual titles to their respective
adverse claim registered by Armando and Adelia has no lots are not absolutely indefeasible. The defense of
legal basis to render defective the transfer of title to the indefeasibility of the Torrens Title does not extend to a
Subsequent Buyers. transferee who takes the certificate of title with notice of a
flaw in his title.[63] The principle of indefeasibility of title
We are not persuaded. Godofredo and Carmen had does not apply where fraud attended the issuance of the
already sold the Subject Land to Armando and Adelia. titles as in this case.[64]
The settled rule is when ownership or title passes to the
buyer, the seller ceases to have any title to transfer to any Attorneys Fees and Costs
third person.[54] If the seller sells the same land to
another, the second buyer who has actual or constructive
131
Sales – Chapter 3-12 Cases
SPS. FORTUNATO SANTOS and ROSALINDA R Sometime in 1984, Rosalinda Santos met Carmen The Casedas gave an initial payment of P54,100.00 and
SANTOS, petitioners, Caseda, a fellow market vendor of hers in Pasay City and immediately took possession of the property, which they
vs. soon became very good friends with her. The duo even then leased out. They also paid in installments,
COURT OF APPEALS, SPS. MARIANO R. CASEDA became kumadres when Carmen stood as a wedding P81,696.84 of the mortgage loan. The Casedas, however,
and CARMEN CASEDA, respondents. sponsor of Rosalinda's nephew. failed to pay the remaining balance of the loan because
they suffered bankruptcy in 1987. Notwithstanding the
For review on certiorari is the decision of the Court of On June 16, 1984, the bank sent Rosalinda Santos a state of their finances, Carmen nonetheless paid in March
Appeals, dated March 28, 1995, in CA-G.R. CV No. letter demanding payment of P16,915.84 in unpaid 1990, the real estate taxes on the property for 1981-1984.
30955, which reversed and set aside the judgment of the interest and other charges. Since the Santos couple had She also settled the electric bills from December 12, 1988
Regional Trial Court of Makati, Branch 133, in Civil Case no funds, Rosalinda offered to sell the house and lot to to July 12, 1989. All these payments were made in the
No. 89-4759. Petitioners (the Santoses) were the owners Carmen. After inspecting the real property, Carmen and name of Rosalinda Santos.
of a house and lot informally sold, with conditions, to her husband agreed.
herein private respondents (the Casedas). In the trial In January 1989, the Santoses, seeing that the Casedas
court, the Casedas had complained that the Santoses Sometime that month of June, Carmen and Rosalinda lacked the means to pay the remaining installments and/or
refused to deliver said house and lot despite repeated signed a document, which reads: amortization of the loan, repossessed the property. The
demands. The trial court dismissed the complaint for Santoses then collected the rentals from the tenants.
specific performance and damages, but in the Court of "Received the amount of P54,100.00 as a partial payment
Appeals, the dismissal was reversed, as follows: of Mrs. Carmen Caseda to the (total) amount of In February 1989, Carmen Caseda sold her fishpond in
350,000.00 (house and lot) that is own (sic) by Mrs. Batangas. She then approached petitioners and offered to
"WHEREFORE, in view of the foregoing, the decision Rosalinda R. Santos. pay the balance of the purchase price for the house and
appealed from is hereby REVERSED and SET ASIDE lot. The parties, however, could not agree, and the deal
and a new one entered: (Sgd.) Carmen H. Caseda could not push through because the Santoses wanted a
higher price. For understandably, the real estate boom in
"1. GRANTING plaintiffs-appellants a period of direct buyer Metro Manila at this time, had considerably jacked up
NINETY (90) DAYS from the date of the finality of realty values. On August 11, 1989, the Casedas filed Civil
judgment within which to pay the balance of the obligation Mrs. Carmen Caseda Case No. 89-4759, with the RTC of Makati, to have the
in accordance with their agreement; Santoses execute the final deed of conveyance over the
"(Sgd.) Rosalinda Del R. Santos property, or in default thereof, to reimburse the amount of
"2. Ordering appellees to restore possession of the P180,000.00 paid in cash and P249,900.00 paid to the
subject house and lot to the appellants upon receipt of the Owner rural bank, plus interest, as well as rentals for eight
full amount of the balance due on the purchase price; and months amounting to P32,000.00, plus damages and
Mrs. Rosalinda R. Santos costs of suit.1âwphi1.nêt
"3. No pronouncement as to costs.
House and Lot After trial on the merits, the lower court disposed of the
"SO ORDERED."1 case as follows:
Better Living Subd. Parañaque, Metro Manila
The undisputed facts of this case are as follows: "WHEREFORE, judgment is hereby ordered:
Section V Don Bosco St."2
The spouses Fortunato and Rosalinda Santos owned the (a) dismissing plaintiff's (Casedas') complaint; and
house and lot consisting of 350 square meters located at The other terms and conditions that the parties agreed
Lot 7, Block 8, Better Living Subdivision, Parañaque, upon were for the Caseda spouses to pay: (1) the balance (b) declaring the agreement; marked as Annex "C"
Metro Manila, as evidenced by TCT (S-11029) 28005 of of the mortgage loan with the Rural bank amounting to of the complaint rescinded. Costs against plaintiffs.
the Register of Deeds of Parañaque. The land together P135,385.18; (2) the real estate taxes; (3) the electric and
with the house, was mortgaged with the Rural Bank of water bills; and (4) the balance of the cash price to be "SO ORDERED."4
Salinas, Inc., to secure a loan of P150,000.00 maturing on paid not later than June 16, 1987, which was the maturity
June 16, 1987. date of the loan.3 Said judgment of dismissal is mainly based on the trial
court's finding that:
132
Sales – Chapter 3-12 Cases
On appeal, the appellate court, as earlier noted, reversed decide, CA-G.R. CV No. 30955, without running afoul of
"Admittedly, the purchase price of the house and lot was the lower court. The appellate court held that rescission Supreme Court Circular No. 290 (4) [c].10
P485,385.18, i.e. P350,000.00 as cash payment and was not justified under the circumstances and allowed the
P135,385.18, assumption of mortgage. Of it plaintiffs Caseda spouses a period of ninety days within which to There is a question of law in a given case when the doubt
[Casedas] paid the following: (1) P54,100.00 down pay the balance of the agreed purchase price. or difference arises as to how the law is on a certain set of
payment; and (2) P81,694.64 installment payments to the facts, and there is a question of fact when the doubt or
bank on the loan (Exhs. E to E-19) or a total of Hence, this instant petition for review on certiorari filed by difference arises as to the truth or falsehood of the alleged
P135,794.64. Thus, plaintiffs were short of the purchase the Santoses. facts.11 But we note that the first assignment of error
price. They cannot, therefore, demand specific submitted by respondents for consideration by the
performance."5 Petitioners now submit the following issues for our appellate court dealt with the trial court's finding that
consideration: herein petitioners got back the property in question
The trial court further held that the Casedas were not because respondents did not have the means to pay the
entitled to reimbursement of payments already made, WHETHER OR NOT THE COURT OF APPEALS, HAS installments and/or amortization of the loan.12 The
reasoning that: JURISDICTION TO DECIDE PRIVATE RESPONDENT'S resolution of this question involved an evaluation of proof,
APPEAL INTERPOSING PURELY QUESTIONS OF and not only a consideration of the applicable statutory
"As earlier mentioned, plaintiffs made a total payment of LAW. and case laws. Clearly, C.A.-G.R. CV No. 30955 did not
P135,794.64 out of the purchase price of P485,385.18. involve pure questions of law, hence the Court of Appeals
The property was in plaintiffs' possession from June 1984 WHETHER THE SUBJECT TRANSACTION IS NOT A had jurisdiction and there was no violation of our Circular
to January 1989 or a period of fifty-five months. During CONTRACT OF ABSOLUTE SALE BUT A MERE ORAL No. 2-90.
that time, plaintiffs leased the property. Carmen said the CONTRACT TO SELL IN WHICH CASE JUDICIAL
property was rented for P25.00 a day or P750.00 a month DEMAND FOR RESCISSION (ART. 1592,7 CIVIL CODE) Moreover, we find that petitioners took an active part in
at the start and in 1987 it was increased to P2,000.00 and IS NOT APPLICABLE. the proceedings before the Court of Appeals, yet they did
P4,000 a month. But the evidence is not precise when the not raise there the issue of jurisdiction. They should have
different amounts of rental took place. Be that as it may, ASSUMING ARGUENDO THAT A JUDICIAL DEMAND raised this issue at the earliest opportunity before the
fairness demands that plaintiffs must pay defendants for FOR RESCISSION IS REQUIRED, WHETHER Court of Appeals. A party taking part in the proceedings
the exercise of dominical rights over the property by PETITIONERS' DEMAND AND PRAYER FOR before the appellate court and submitting his case for its
renting it to others. The amount of P2,000.00 a month RESCISSION CONTAINED IN THEIR ANSWER FILED decision ought not to later on attack the court's decision
would be reasonable based on the average of P750.00, BEFORE THE TRIAL SATISFIED THE SAID for want of jurisdiction because the decision turns out to
P2,000.00, P4,000.00 lease-rentals charged. Multiply REQUIREMENT. be adverse to him.13
P2,000 by 55 months, the plaintiffs must pay defendants
P110,000 for the use of the property. Deducting this WHETHER OR NOT THE NON-PAYMENT OF MORE The second and third issues deal with the question: Did
amount from the P135,794.64 payment of the plaintiffs on THAN HALF OF THE ENTIRE PURCHASE PRICE the Court of Appeals err in holding that a judicial
the property the difference is P25,794.64. Should the INCLUDING THE NON-COMPLIANCE WITH THE rescission of the agreement was necessary? In resolving
plaintiffs be entitled to a reimbursement of this amount? STIPULATION TO LIQUIDATE THE MORTGAGE LOAN both issues, we must first make a preliminary
The answer is in the negative. Because of failure of ON TIME WHICH CAUSED GRAVE DAMAGE AND determination of the nature of the contract in question:
plaintiffs to liquidated the mortgage loan on time, it had PREJUDICE TO PETITIONERS, CONSTITUTE Was it a contract of sale, as insisted by the respondents
ballooned from its original figure of P135,384.18 as of SUBSTANTIAL BREACH TO JUSTIFY RESCISSION OF or a mere contract to sell, as contended by petitioners?
June 1984 to P337,280.78 as of December 31, 1988. A CONTRACT TO SELL UNDER ARTICLE 1191 8 (CIVIL
Defendants [Santoses] had to pay the last amount to the CODE). Petitioners argue that the transaction between them and
bank to save the property from foreclosure. Logically, respondents was a mere contract to sell, and not a
plaintiffs must share in the burden arising from their failure On the first issue, petitioners argue that, since both the contract of sale, since the sole documentary evidence
to liquidate the loan per their contractual commitment. parties and the apellate court adopted the findings of trial (Exh. D, receipt) referring to their agreement clearly
Hence, the amount of P25,794.64 as their share in the court,9 no questions of fact were raised before the Court showed that they did not transfer ownership of the
defendants' damages in the form of increased loan- of Appeals. According to petitioners, CA-G.R. CV No. property in question simultaneous with its delivery and
amount, is reasonable."6 30955, involved only pure questions of law. They aver that hence remained its owners, pending fulfillment of the
the court a quo had no jurisdiction to hear, much less other suspensive conditions, i.e. full payment of the
balance of the purchase price and the loan amortizations.
133
Sales – Chapter 3-12 Cases
Petitioners point to Manuel v. Rodriguez, 109 Phil. 1 between Rosalinda Santos and Carmen Caseda is a As to the last issue, we need not tarry to make a
(1960) and Luzon Brokerage Co., Inc. v. Maritime Building contract to sell. In contracts to sell, ownership is reserved determination of whether the breach of contract by private
Co., Inc., 43 SCRA 93 (1972), where he held that article the by the vendor and is not to pass until full payment of respondents is so substantial as to defeat the purpose of
1592 of the Civil Code is inapplicable to a contract to sell. the purchase price. This we find fully applicable and the parties in entering into the agreement and thus entitle
They charge the court a quo with reversible error in understandable in this case, given that the property petitioners to rescission. Having ruled that there is no
holding that petitioners should have judicially rescinded involved is a titled realty under mortgage to a bank and rescission to speak of in this case, the question is moot.
the agreement with respondents when the latter failed to would require notarial and other formalities of law before
pay the amortizations on the bank loan. transfer thereof could be validly effected. WHEREFORE, the instant petition is GRANTED and the
assailed decision of the Court of Appeals in CA-G.R. CV
Respondents insist that there was a perfected contract of In view of our finding in the present case that the No. 30955 is REVERSED and SET ASIDE. The judgment
sale, since upon their partial payment of the purchase agreement between the parties is a contract to sell, it of the Regional Trial Court of Makati, Branch 133, with
price, they immediately took possession of the property as follows that the appellate court erred when it decreed that respect to the DISMISSAL of the complaint in Civil Case
vendees, and subsequently leased it, thus exercising all a judicial rescission of said agreement was necessary. No. 89-4759, is hereby REINSTATED. No pronouncement
the rights of ownership over the property. This showed This is because there was no rescission to speak of in the as to costs.1âwphi1.nêt
that transfer of ownership was simultaneous with the first place. As we earlier pointed, in a contract to sell, title
delivery of the realty sold, according to respondents. remains with the vendor and does not pass on to the SO ORDERED.
vendee until the purchase price is paid in full, Thus, in
It must be emphasized from the outset that a contract is contract to sell, the payment of the purchase price is a
what the law defines it to be, taking into consideration its positive suspensive condition. Failure to pay the price
essential elements, and not what the contracting parties agreed upon is not a mere breach, casual or serious, but
call it.14 Article 145815 of the Civil Code defines a a situation that prevents the obligation of the vendor to
contract of sale. Note that the said article expressly convey title from acquiring an obligatory force.20 This is
obliges the vendor to transfer the ownership of the thing entirely different from the situation in a contract of sale,
sold as an essential element of a contract of sale.16 We where non-payment of the price is a negative resolutory
have carefully examined the contents of the unofficial condition. The effects in law are not identical. In a contract
receipt, Exh. D, with the terms and conditions informally of sale, the vendor has lost ownership of the thing sold
agreed upon by the parties, as well as the proofs and cannot recover it, unless the contract of sale is
submitted to support their respective contentions. We are rescinded and set aside.21 In a contract to sell, however,
far from persuaded that there was a transfer of ownership the vendor remains the owner for as long as the vendee
simultaneously with the delivery of the property has not complied fully with the condition of paying the
purportedly sold. The records clearly show that, purchase. If the vendor should eject the vendee for failure
notwithstanding the fact that the Casedas first took then to meet the condition precedent, he is enforcing the
lost possession of the disputed house and lot, the title to contract and not rescinding it. When the petitioners in the
the property, TCT No. 28005 (S-11029) issued by the instant case repossessed the disputed house and lot for
Register of Deeds of Parañaque, has remained always in failure of private respondents to pay the purchase price in
the name of Rosalinda Santos.17 Note further that full, they were merely enforcing the contract and not
although the parties agreed that the Casedas would rescinding it. As petitioners correctly point out the Court of
assume the mortgage, all amortization payments made by Appeals erred when it ruled that petitioners should have
Carmen Caseda to the bank were in the name of judicially rescinded the contract pursuant to Articles 1592
Rosalinda Santos.18 We likewise find that the bank's and 1191 of the Civil Code. Article 1592 speaks of non-
cancellation and discharge of mortgage dated January 20, payment of the purchase price as a resolutory condition. It
1990, was made in favor of Rosalinda Santos.19 The does not apply to a contract to sell.22 As to Article 1191, it
foregoing circumstances categorically and clearly show is subordinated to the provisions of Article 1592 when
that no valid transfer of ownership was made by the applied to sales of immovable property.23 Neither
Santoses to the Casedas. Absent this essential element, provision is applicable in the present case.
their agreement cannot be deemed a contract of sale. We
agree with petitioner's averment that the agreement
134
Sales – Chapter 3-12 Cases
ZENAIDA M. SANTOS, petitioner, vs. CALIXTO SANTOS, that Salvador was the registered owner of the property, which was not financially capable to purchase it. The deeds of sale
ALBERTO SANTOS, ROSA SANTOS-CARREON and could only be subjected to encumbrances or liens annotated were therefore fictitious. Hence, the action to assail the same
ANTONIO SANTOS, respondents. on the title; that the respondents right to reconveyance was does not prescribe.[4]
already barred by prescription and laches; and that the
This petition for review[1] seeks to annul and set aside the complaint stated no cause of action. Upon appeal, the Court of Appeals affirmed the trial courts
decision dated March 10, 1998 of the Court of Appeals that decision dated March 10, 1998. It held that in order for the
affirmed the decision of the Regional Trial Court of Manila, On March 17, 1993, the trial court decided in private execution of a public instrument to effect tradition, as
Branch 48, dated March 17, 1993. Petitioner also seeks to respondents favor, thus: provided in Article 1498 of the Civil Code,[5] the vendor shall
annul the resolution that denied her motion for have had control over the thing sold, at the moment of sale. It
reconsideration. WHEREFORE, viewed from all the foregoing considerations, was not enough to confer upon the purchaser the ownership
judgment is hereby made in favor of the plaintiffs and against and the right of possession. The thing sold must be placed in
Petitioner Zenaida M. Santos is the widow of Salvador the defendants: his control. The subject deeds of sale did not confer upon
Santos, a brother of private respondents Calixto, Alberto, Salvador the ownership over the subject property, because
Antonio, all surnamed Santos and Rosa Santos-Carreon. a) Declaring Exh. B, the deed of sale executed by Rosalia even after the sale, the original vendors remained in
Santos and Jesus Santos on January 19, 1959, as entirely dominion, control, and possession thereof. The appellate
The spouses Jesus and Rosalia Santos owned a parcel of null and void for being fictitious or simulated and inexistent court further said that if the reason for Salvadors failure to
land registered under TCT No. 27571 with an area of 154 and without any legal force and effect; control and possess the property was due to his
square meters, located at Sta. Cruz Manila. On it was a four- acquiescence to his mother, in deference to Filipino custom,
door apartment administered by Rosalia who rented them b) Declaring Exh. D, the deed of sale executed by Rosa petitioner, at least, should have shown evidence to prove that
out. The spouses had five children, Salvador, Calixto, Alberto, Santos in favor of Salvador Santos on November 20, 1973, her husband declared the property for tax purposes in his
Antonio and Rosa. also as entirely null and void for being likewise fictitious or name or paid the land taxes, acts which strongly indicate
simulated and inexistent and without any legal force and control and possession. The appellate court disposed:
On January 19, 1959, Jesus and Rosalia executed a deed of effect;
sale of the properties in favor of their children Salvador and WHEREFORE, finding no reversible error in the decision
Rosa. TCT No. 27571 became TCT No. 60819. Rosa in turn c) Directing the Register of Deeds of Manila to cancel appealed from, the same is hereby AFFIRMED. No
sold her share to Salvador on November 20, 1973 which Transfer Certificate of Title No. T-113221 registered in the pronouncement as to costs.
resulted in the issuance of a new TCT No. 113221. Despite name of Salvador Santos, as well as, Transfer Certificate of
the transfer of the property to Salvador, Rosalia continued to Title No. 60819 in the names of Salvador Santos, Rosa SO ORDERED.[6]
lease and receive rentals from the apartment units. Santos, and consequently thereafter, reinstating with the
same legal force and effect as if the same was not cancelled, Hence, this petition where petitioner avers that the Court of
On November 1, 1979, Jesus died. Six years after or on and which shall in all respects be entitled to like faith and Appeals erred in:
January 9, 1985, Salvador died, followed by Rosalia who died credit; Transfer Certificate of Title No. T-27571 registered in
the following month. Shortly after, petitioner Zenaida, claiming the name of Rosalia A. Santos, married to Jesus Santos, the I.
to be Salvadors heir, demanded the rent from Antonio same to be partitioned by the heirs of the said registered
Hombrebueno,[2] a tenant of Rosalia. When the latter refused owners in accordance with law; and ...HOLDING THAT THE OWNERSHIP OVER THE
to pay, Zenaida filed an ejectment suit against him with the LITIGATED PROPERTY BY THE LATE HUSBAND OF
Metropolitan Trial Court of Manila, Branch 24, which d) Making the injunction issued in this case permanent. DEFENDANT-APPELLANT WAS AFFECTED BY HIS
eventually decided in Zenaidas favor. FAILURE TO EXERCISE CERTAIN ATTRIBUTES OF
Without pronouncement as to costs. OWNERSHIP.
On January 5, 1989, private respondents instituted an action
for reconveyance of property with preliminary injunction SO ORDERED.[3] II
against petitioner in the Regional Trial Court of Manila, where
they alleged that the two deeds of sale executed on January The trial court reasoned that notwithstanding the deeds of ...HOLDING THAT DUE EXECUTION OF A PUBLIC
19, 1959 and November 20, 1973 were simulated for lack of sale transferring the property to Salvador, the spouses INSTRUMENT IS NOT EQUIVALENT TO DELIVERY OF
consideration. They were executed to accommodate Rosalia and Jesus continued to possess the property and to THE LAND IN DISPUTE.
Salvador in generating funds for his business ventures and exercise rights of ownership not only by receiving the monthly
providing him with greater business flexibility. rentals, but also by paying the realty taxes. Also, Rosalia kept III
the owners duplicate copy of the title even after it was already
In her Answer, Zenaida denied the material allegations in the in the name of Salvador. Further, the spouses had no
complaint and as special and affirmative defenses, argued compelling reason in 1959 to sell the property and Salvador
135
Sales – Chapter 3-12 Cases
...NOT FINDING THAT THE CAUSE OF ACTION OF of rentals from the tenants by the seller of realty after
ROSALIA SANTOS HAD PRESCRIBED AND/OR BARRED execution of alleged deed of sale is contrary to the notion of Moreover, in Norkis Distributors, Inc. vs. CA, 193 SCRA 694,
BY LACHES. ownership. 698-699 (1991), citing the land case of Abuan vs. Garcia, 14
SCRA 759 (1965), we held that the critical factor in the
IV Petitioner argues that Salvador, in allowing her mother to use different modes of effecting delivery, which gives legal effect
the property even after the sale, did so out of respect for her to the act is the actual intention of the vendor to deliver, and
...IGNORING PETITIONERS ALLEGATION TO THE and out of generosity, a factual matter beyond the province of its acceptance by the vendee. Without that intention, there is
EFFECT THAT PLAINTIFF DR. ROSA [S.] CARREON IS this Court.[14] Significantly, in Alcos vs. IAC, 162 SCRA 823, no tradition. In the instant case, although the spouses Jesus
NOT DISQUALIFIED TO TESTIFY AS TO THE 837 (1988), we noted that the buyers immediate possession and Rosalia executed a deed of sale, they did not deliver the
QUESTIONED DEEDS OF SALE CONSIDERING THAT and occupation of the property corroborated the truthfulness possession and ownership of the property to Salvador and
SALVADOR SANTOS HAS LONG BEEN DEAD.[7] and authenticity of the deed of sale. Conversely, the vendors Rosa. They agreed to execute a deed of sale merely to
continued possession of the property makes dubious the accommodate Salvador to enable him to generate funds for
In this petition, we are asked to resolve the following: contract of sale between the parties. his business venture.

1. Are payments of realty taxes and retention of possession On the second issue, is a sale through a public instrument On the third issue, petitioner argues that from the date of the
indications of continued ownership by the original owners? tantamount to delivery of the thing sold? Petitioner in her sale from Rosa to Salvador on November 20, 1973, up to his
memorandum invokes Article 1477[15] of the Civil Code death on January 9, 1985, more or less twelve years had
2. Is a sale through a public instrument tantamount to delivery which provides that ownership of the thing sold is transferred lapsed, and from his death up to the filing of the case for
of the thing sold? to the vendee upon its actual or constructive delivery. Article reconveyance in the court a quo on January 5, 1989, four
1498, in turn, provides that when the sale is made through a years had lapsed. In other words, it took respondents about
3. Did the cause of action of Rosalia Santos and her heirs public instrument, its execution is equivalent to the delivery of sixteen years to file the case below. Petitioner argues that an
prescribe? the thing subject of the contract. Petitioner avers that applying action to annul a contract for lack of consideration prescribes
said provisions to the case, Salvador became the owner of in ten years and even assuming that the cause of action has
4. Can petitioner invoke the Dead Mans Statute?[8] the subject property by virtue of the two deeds of sale not prescribed, respondents are guilty of laches for their
executed in his favor. inaction for a long period of time.
On the first issue, petitioner contends that the Court of
Appeals erred in holding that despite the deeds of sale in Nowhere in the Civil Code, however, does it provide that Has respondents cause of action prescribed? In Lacsamana
Salvadors favor, Jesus and Rosalia still owned the property execution of a deed of sale is a conclusive presumption of vs. CA, 288 SCRA 287, 292 (1998), we held that the right to
because the spouses continued to pay the realty taxes and delivery of possession. The Code merely said that the file an action for reconveyance on the ground that the
possess the property. She argues that tax declarations are execution shall be equivalent to delivery. The presumption certificate of title was obtained by means of a fictitious deed
not conclusive evidence of ownership when not supported by can be rebutted by clear and convincing evidence.[16] of sale is virtually an action for the declaration of its nullity,
evidence. She avers that Salvador allowed his mother to Presumptive delivery can be negated by the failure of the which does not prescribe. This applies squarely to the
possess the property out of respect to her in accordance with vendee to take actual possession of the land sold.[17] present case. The complaint filed by respondents in the court
Filipino values. a quo was for the reconveyance of the subject property to the
In Danguilan vs. IAC, 168 SCRA 22, 32 (1988), we held that estate of Rosalia since the deeds of sale were simulated and
It is true that neither tax receipts nor declarations of for the execution of a public instrument to effect tradition, the fictitious. The complaint amounts to a declaration of nullity of
ownership for taxation purposes constitute sufficient proof of purchaser must be placed in control of the thing sold. When a void contract, which is imprescriptible. Hence, respondents
ownership. They must be supported by other effective there is no impediment to prevent the thing sold from cause of action has not prescribed.
proofs.[9] These requisite proofs we find present in this case. converting to tenancy of the purchaser by the sole will of the
As admitted by petitioner, despite the sale, Jesus and Rosalia vendor, symbolic delivery through the execution of a public Neither is their action barred by laches. The elements of
continued to possess and administer the property and enjoy instrument is sufficient. But if, notwithstanding the execution laches are: 1) conduct on the part of the defendant, or of one
its fruits by leasing it to third persons.[10] Both Rosa and of the instrument, the purchaser cannot have the enjoyment under whom he claims, giving rise to the situation of which
Salvador did not exercise any right of ownership over it.[11] and material tenancy nor make use of it himself or through the complaint seeks a remedy; 2) delay in asserting the
Before the second deed of sale to transfer her 1/2 share over another in his name, then delivery has not been effected. complainants rights, the complainant having had knowledge
the property was executed by Rosa, Salvador still sought the or notice of the defendants conduct as having been afforded
permission of his mother.[12] Further, after Salvador As found by both the trial and appellate courts and amply an opportunity to institute a suit; 3) lack of knowledge or
registered the property in his name, he surrendered the title supported by the evidence on record, Salvador was never notice on the part of the defendant that the complainant
to his mother.[13] These are clear indications that ownership placed in control of the property. The original sellers retained would assert the right in which he bases his suit; and 4) injury
still remained with the original owners. In Serrano vs. CA, 139 their control and possession. Therefore, there was no real or prejudice to the defendant in the event relief is accorded to
SCRA 179, 189 (1985), we held that the continued collection transfer of ownership. the complainant, or the suit is not held barred.[18] These
136
Sales – Chapter 3-12 Cases
elements must all be proved positively. The conduct which
caused the complaint in the court a quo was petitioners
assertion of right of ownership as heir of Salvador. This
started in December 1985 when petitioner demanded
payment of the lease rentals from Antonio Hombrebueno, the
tenant of the apartment units. From December 1985 up to the
filing of the complaint for reconveyance on January 5, 1989,
only less than four years had lapsed which we do not think is
unreasonable delay sufficient to bar respondents cause of
action. We likewise find the fourth element lacking. Neither
petitioner nor her husband made considerable investments
on the property from the time it was allegedly transferred to
the latter. They also did not enter into transactions involving
the property since they did not claim ownership of it until
December 1985. Petitioner stood to lose nothing. As we held
in the same case of Lacsamana vs. CA, cited above, the
concept of laches is not concerned with the lapse of time but
only with the effect of unreasonable lapse. In this case, the
alleged 16 years of respondents inaction has no adverse
effect on the petitioner to make respondents guilty of laches.

Lastly, petitioner in her memorandum seeks to expunge the


testimony of Rosa Santos-Carreon before the trial court in
view of Sec. 23, Rule 130 of the Revised Rules of Court,
otherwise known as the Dead Mans Statute.[19] It is too late
for petitioner, however, to invoke said rule. The trial court in
its order dated February 5, 1990, denied petitioners motion to
disqualify respondent Rosa as a witness. Petitioner did not
appeal therefrom. Trial ensued and Rosa testified as a
witness for respondents and was cross-examined by
petitioners counsel. By her failure to appeal from the order
allowing Rosa to testify, she waived her right to invoke the
dead mans statute. Further, her counsel cross-examined
Rosa on matters that occurred during Salvadors lifetime. In
Goi vs. CA, 144 SCRA 222, 231 (1986), we held that
protection under the dead mans statute is effectively waived
when a counsel for a petitioner cross-examines a private
respondent on matters occurring during the deceaseds
lifetime. The Court of Appeals cannot be faulted in ignoring
petitioner on Rosas disqualification.

WHEREFORE, the instant petition is DENIED. The assailed


decision dated March 10, 1998 of the Court of Appeals, which
sustained the judgment of the Regional Trial Court dated
March 17, 1993, in favor of herein private respondents, is
AFFIRMED. Costs against petitioner.
SO ORDERED.

137
Sales – Chapter 3-12 Cases
FRANCISCO N. DY, JR., Substituted by his Estate corporation, so it was declared in default again and the With regard to the validity of the proceedings before the
Rep. by ROSARIO PEREZ-DY, Administratrix, plaintiff was allowed to present its evidence ex parte Branch Clerk of Court, we agree with the observations of
petitioner, before the Branch Clerk of Court. However, in that same the Court of Appeals that:
vs. pre-trial conference the parties agreed that the evidence
COURT OF APPEALS and FERTILIZER MARKETING previously presented by the plaintiff shall remain on record Appellant is now estopped from questioning the retention
COMPANY OF THE PHILIPPINES, respondents. for purposes of the continuation of the trial, subject to of the proceedings held on August 19, 1983 before the
cross-examination in open court, and, that the Branch Clerk of Court since her husband agreed to the
This is a petition for review of the Court of Appeals' presentation of the affidavits in question and answer form same during the pre-trial conference held on November
decision dated December 11, 1990, which affirmed in toto will constitute the direct testimony of the defendant's 12, 1987. Agreements reached at the pre-trial conference
the decision of the Regional Trial Court of Makati dated witnesses likewise subject to cross-examination of the and embodied in the pre-trial order shall control the
July 18, 1988, which ordered the petitioner to pay the adverse counsel. subsequent course of the trial and should not be disturbed
private respondent the sum of P337,120.00 plus interest unless there could be manifest injustice.
of 12% per annum, attorney's fees and costs. On motion for reconsideration, the order of default against
the corporation was lifted. A second motion for The agreement is not unjust to appellant. Aside from
Private respondent Fertilizer Marketing Company of the reconsideration was filed by the defendants on January appellant having the right to adduce evidence on her
Philippines filed an action to collect from Francisco Dy, Jr. 22, 1988 to set aside the agreement for trial by affidavits behalf, the parties agreed that the evidence presented by
(now deceased) and the Francisco Dy, Jr. Trading but it was denied by the court. appellee before the Branch Clerk of Court would be
Corporation the sum of P337,120.00 as unpaid balance retained, with appellant having the right to cross-examine
on their purchase of fertilizers on credit from the private On the date of the hearing set on April 25, 1988, the appellee's witnesses.
respondent. defendants failed to appear to present their evidence
despite due notice, hence, they were deemed to have xxx xxx xxx
The defendants were declared in default on August 15, waived the presentation of their evidence. The case was
1983 for failure to answer the complaint within the submitted for decision upon the plaintiffs evidence. The agreement of the parties as contained in the pre-trial
reglementary period. Private respondent was thereafter order is not invalid. The parties are authorized by the
allowed to present its evidence ex parte before the Branch On July 18, 1988, the trial court rendered a decision Rules of Court to consider "[s]uch other matters as may
Clerk of Court. (mentioned earlier) for the plaintiff and against the aid in the prompt disposition of the action." An authority
defendants. The latter appealed to the Court of Appeals believes this includes "agreement on certain matters so
Subsequently, the defendants filed a motion to admit their (CA-G.R. CV No. 23540) alleging that the court a quo that witnesses need not and will not be called."
answer, but it was denied by the court. They filed a motion erred (1) in reinstating the nullified proceedings on August Undoubtedly, the procedure agreed upon by the parties in
for reconsideration; it was granted; the order of default 19, 1983 before the Branch Clerk of Court; (2) in denying this case would have greatly accelerated the trial and the
was set aside; their answer was admitted; and they were her procedural due process; and (3) in awarding damages decision therein, which, at the, time of the pre-trial
allowed to present their evidence without retaking the against her. conference, had been pending for three years and had
plaintiff s evidence. already gone up on appeal to this Court. (pp. 27-28,
During the pendency of the appeal, Francisco Dy, Jr. Rollo.)
On the date set for the reception of their evidence, the passed away on June 20, 1989. His wife, Rosario Perez-
defendants failed to appear despite due notice, so, Dy, as judicial administratrix of his estate, prosecuted the The presentation of the plaintiff's evidence before the
judgment was rendered by the trial court against them on appeal (Azarraga vs. Cortes, 9 Phil. 698). Branch Clerk of Court was not void. The Supreme Court,
January 4, 1984. in the case of Continental Bank vs. Tiangco, et al. (94
On December 11, 1990, the Court of Appeals dismissed SCRA 715) departing from its contrary statement in the
On appeal to the Court of Appeals, the judgment by the appeal (CA-G.R. CV No. 23540) for lack of merit. Lim Tan Hu case (66 SCRA 425), declared that a decision
default was set aside and the case was remanded to the based on evidence heard by a deputy clerk of court as
lower court for pre-trial and trial on the merits (AC-G.R. In this petition for review of that decision, the petitioner commissioner is valid and enforceable because it was
CV No. 03747, p. 46, Rollo). reiterates the same issues that she raised in the Court of rendered by a court of competent jurisdiction, was not
Appeals. impaired by extrinsic fraud, nor by lack of due process,
At the pre-trial conference on November 12, 1987, the and there was no showing that the private respondents
plaintiff and defendant Francisco Dy, Jr. appeared, but were prejudiced by such a procedure, or that the
there was no appearance for the defendant trading commissioner committed any mistake or abuse of
138
Sales – Chapter 3-12 Cases
discretion, or that the proceedings were vitiated by
collusion and collateral fraud. That ruling applies four
square to this case.

The practice of designating the clerk of court as a


commissioner to receive evidence in the event of the non-
appearance of the defendant and its counsel, is not
irregular and is sanctioned by Rule 33 of the Rules of
Court on trial by commissioner (J.M. Tuazon, Inc. vs. Dela
Rosa, 18 SCRA 591; Wassmer vs. Velez, 12 SCRA 648).

The petitioner was not denied due process. As pointed out


by the appellate court:

. . . Appellant retained her right to present evidence on her


behalf and the opportunity to cross-examine the witnesses
already presented by appellee. At any rate, if appellant
believes that her right to procedural due process had
been curtailed, the same was due to a voluntary waiver by
her husband. (p. 28, Rollo)

WHEREFORE, the petition for review is denied for lack of


merit. Costs against the petitioners.
SO ORDERED.

139
Sales – Chapter 3-12 Cases
SPOUSES EROSTO SANTIAGO and NELSIE P5,500.00, P7,000.00 and P24,500.00 on November 4, spouses Villamor, Sr. sold the land to the petitioners for
SANTIAGO, Petitioners, 1991, November 23, 1992, April 26, 1993 and June 8, P150,000.00.13
vs. 1994, respectively.7
MANCER VILLAMOR, CARLOS VILLAMOR, JOHN After the respondents and Catalina refused the petitioners’
VILLAMOR and DOMINGO VILLAMOR, JR., When the San Jacinto Bank refused to issue a deed of demand to vacate the land, the petitioners filed on
Respondents. conveyance in their favor despite full payment, the October 20, 1994 a complaint for quieting of title and
respondents and Catalina filed a complaint against the recovery of possession against the respondents.14 This is
We resolve the petition for review on certiorari1 tiled by San Jacinto Bank (docketed as Civil Case No. 200) with the case that is now before us.
spouses Eros to Santiago and Nelsie Santiago the RTC on October 11, 1994. The complaint was for
(petitioners) to challenge the August 10, 2004 decision2 specific performance with damages. The respondents and Catalina assailed the San Jacinto
and the June 8, 2005 resolution3 of the Court of Appeals Bank’s execution of the deed of sale in favor of Domingo,
(CA) in CA-G.R. CV No. 59112. The CA decision set The San Jacinto Bank claimed that it already issued a Sr., claiming that the respondents and Catalina made the
aside the May 28, 1997 decision4 of the Regional Trial deed of repurchase in favor of the spouses Villamor, Sr.; installment payments on their own behalf.15
Court (RTC) of San Jacinto, Masbate, Branch 50, in Civil the payments made by the respondents and Catalina
Case No. 201. The CA resolution denied the petitioners' were credited to the account of Domingo, Sr. since the In its May 28, 1997 decision,16 the RTC declared the
subsequent motion for reconsideration. real buyers of the land were the spouses Villamor, Sr.8 petitioners as the legal and absolute owners of the land,
finding that the petitioners were purchasers in good faith;
THE FACTUAL ANTECEDENTS In a February 10, 2004 decision, the RTC dismissed the the spouses Villamor, Sr.’s execution of the July 21, 1994
specific performance case. It found that the San Jacinto notarized deed of sale in favor of the petitioners resulted
In January 1982,5 the spouses Domingo Villamor, Sr. and Bank acted in good faith when it executed a deed of in the constructive delivery of the land. Thus, it ordered
Trinidad Gutierrez Villamor (spouses Villamor, Sr.), the "repurchase" in the spouses Villamor, Sr.’s names since the respondents to vacate and to transfer possession of
parents of Mancer Villamor, Carlos Villamor and Domingo Domingo, Sr., along with the respondents and Catalina, the land to the petitioners, and to pay P10,000.00 as
Villamor, Jr. (respondents) and the grandparents of was the one who transacted with the San Jacinto Bank to moral damages.17
respondent John Villamor, mortgaged their 4.5-hectare redeem the land.9
coconut land in Sta. Rosa, San Jacinto, Masbate, known On appeal, the CA, in its August 10, 2004 decision, found
as Lot No. 1814, to the Rural Bank of San Jacinto The CA, on appeal, set aside the RTC’s decision.10 The that the petitioners’ action to quiet title could not prosper
(Masbate), Inc. (San Jacinto Bank) as security for a CA found that the respondents and Catalina made the because the petitioners failed to prove their legal or
P10,000.00 loan. installment payments on their own behalf and not as equitable title to the land. It noted that there was no real
representatives of the spouses Villamor, Sr. The San transfer of ownership since neither the spouses Villamor,
For non-payment of the loan, the San Jacinto Bank Jacinto Bank mistakenly referred to the transaction as a Sr. nor the petitioners were placed in actual possession
extrajudicially foreclosed the mortgage, and, as the "repurchase" when the redemption period had already and control of the land after the execution of the deeds of
highest bidder at the public auction, bought the land. lapsed and the title had been transferred to its name; the sale. It also found that the petitioners failed to show that
When the spouses Villamor, Sr. failed to redeem the transaction of the respondents and Catalina was the respondents and Catalina’s title or claim to the land
property within the prescribed period, the San Jacinto altogether alien to the spouses Villamor, Sr.’s loan with was invalid or inoperative, noting the pendency of the
Bank obtained a final deed of sale in its favor sometime in mortgage. Thus, it ordered the San Jacinto Bank to specific performance case, at that time on appeal with the
1991. The San Jacinto Bank then offered the land for sale execute the necessary deed of sale in favor of the CA. Thus, it set aside the RTC decision and ordered the
to any interested buyer.6 respondents and Catalina, and to pay P30,000.00 as dismissal of the complaint, without prejudice to the
attorney’s fees.11 No appeal appears to have been taken outcome of the specific performance case.18
a. The Specific Performance Case from this decision.
When the CA denied19 the motion for reconsideration20
Since the respondents had been in possession and b. The Present Quieting of Title Case that followed, the petitioners filed the present Rule 45
cultivation of the land, they decided, together with their petition.
sister Catalina Villamor Ranchez, to acquire the land from On July 19, 1994 (or prior to the filing of the respondents
the San Jacinto Bank. The San Jacinto Bank agreed with and Catalina’s complaint for specific performance, as THE PETITION
the respondents and Catalina to a P65,000.00 sale, narrated above), the San Jacinto Bank issued a deed of
payable in installments. The respondents and Catalina sale in favor of Domingo, Sr.12 On July 21, 1994, the The petitioners argue that the spouses Villamor, Sr.’s
made four (4) installment payments of P28,000.00, execution of the July 21, 1994 deed of sale in the
140
Sales – Chapter 3-12 Cases
petitioners’ favor was equivalent to delivery of the land November 4, 1991 to June 8, 1994 and their actual "A purchaser in good faith is one who buys property
under Article 1498 of the Civil Code; the petitioners are possession of the disputed land. without notice that some other person has a right to or
purchasers in good faith since they had no knowledge of interest in such property and pays its fair price before he
the supposed transaction between the San Jacinto Bank After considering the parties’ evidence and arguments, we has notice of the adverse claims and interest of another
and the respondents and Catalina; and the respondents agree with the CA that the petitioners failed to prove that person in the same property."26 However, where the land
and Catalina’s possession of the land should not be they have any legal or equitable title over the disputed sold is in the possession of a person other than the
construed against them (petitioners) since, by tradition land. vendor, the purchaser must be wary and must investigate
and practice in San Jacinto, Masbate, the children use the rights of the actual possessor; without such inquiry,
their parents’ property. Execution of the deed of sale only a the buyer cannot be said to be in good faith and cannot
have any right over the property.27
THE CASE FOR THE RESPONDENTS prima facie presumption of delivery.
In this case, the spouses Villamor, Sr. were not in
The respondents and respondent John submit that they Article 1477 of the Civil Code recognizes that the possession of the land.1âwphi1 The petitioners, as
hold legal title to the land since they perfected the sale "ownership of the thing sold shall be transferred to the prospective vendees, carried the burden of investigating
with the San Jacinto Bank as early as November 4, 1991, vendee upon the actual or constructive delivery thereof." the rights of the respondents and respondent John who
the first installment payment, and are in actual possession Related to this article is Article 1497 which provides that were then in actual possession of the land. The petitioners
of the land; the petitioners are not purchasers in good "the thing sold shall be understood as delivered, when it is cannot take refuge behind the allegation that, by custom
faith since they failed to ascertain why the respondents placed in the control and possession of the vendee." and tradition in San Jacinto, Masbate, the children use
were in possession of the land. their parents' property, since they offered no proof
With respect to incorporeal property, Article 1498 of the supporting their bare allegation. The burden of proving the
THE ISSUE Civil Code lays down the general rule: the execution of a status of a purchaser in good faith lies upon the party
public instrument "shall be equivalent to the delivery of the asserting that status and cannot be discharged by reliance
The case presents to us the issue of whether the CA thing which is the object of the contract, if from the deed on the legal presumption of good faith.28 The petitioners
committed a reversible error when it set aside the RTC the contrary does not appear or cannot clearly be failed to discharge this burden.
decision and dismissed the petitioners’ complaint for inferred." However, the execution of a public instrument
quieting of title and recovery of possession. gives rise only to a prima facie presumption of delivery, Lastly, since the specific performance case already settled
which is negated by the failure of the vendee to take the respondents and respondent John's claim over the
OUR RULING actual possession of the land sold.24 "A person who does disputed land, the dispositive portion of the CA decision
not have actual possession of the thing sold cannot (dismissing the complaint without prejudice to the
The petition lacks merit. transfer constructive possession by the execution and outcome of the specific performance case29 ) is modified
delivery of a public instrument."25 to reflect this fact; we thus dismiss for lack of merit the
Quieting of title is a common law remedy for the removal complaint for quieting of title and recovery of possession.
of any cloud, doubt or uncertainty affecting title to real In this case, no constructive delivery of the land transpired
property. The plaintiffs must show not only that there is a upon the execution of the deed of sale since it was not the WHEREFORE, we hereby DENY the petition and ORDER
cloud or contrary interest over the subject real property,21 spouses Villamor, Sr. but the respondents who had actual the DISMISSAL of Civil Case No. 201 before the Regional
but that they have a valid title to it.22 Worth stressing, in possession of the land. The presumption of constructive Trial Court of San Jacinto, Masbate, Branch 50.
civil cases, the plaintiff must establish his cause of action delivery is inapplicable and must yield to the reality that
by preponderance of evidence; otherwise, his suit will not the petitioners were not placed in possession and control Costs against the petitioners.
prosper.23 of the land.
SO ORDERED.
The petitioners anchor their claim over the disputed land The petitioners are not purchasers in
on the July 21, 1994 notarized deed of sale executed in good faith.
their favor by the spouses Villamor, Sr. who in turn
obtained a July 19, 1994 notarized deed of sale from the The petitioners can hardly claim to be purchasers in good
San Jacinto Bank. On the other hand, the respondents faith.
and respondent John claim title by virtue of their
installment payments to the San Jacinto Bank from
141
Sales – Chapter 3-12 Cases
LA FUERZA, INC., petitioner, He caused, however, to be added to this offer at the foot a After the last trial run made in the month of July and after
vs. note which reads: "All specifications shall be in strict the plaintiff's technical manager had been advised several
THE HON. COURT OF APPEALS and ASSOCIATED accordance with the approved plan made part of this times to make the necessary and proper adjustments or
ENGINEERING CO., INC., respondents. agreement hereof." A few days later, Antonio Co made corrections in order to improve the efficiency of the
the demand for the down payment of P5,000.00 which conveyor system, it seems that the defects indicated by
Ordinary action for the recovery of a sum of money. In due was readily delivered by the defendant in the form of a the said president and general manager of the defendant
course, the Court of First Instance of Manila rendered check for the said amount. After that agreement, the had not been remedied so that they came to the parting of
judgment for defendant, La Fuerza, Inc. — hereinafter plaintiff started to prepare the premises for the the ways with the result that when the plaintiff billed the
referred to as La Fuerza — which was at first affirmed by installations of the conveyor system by digging holes in defendant for the balance of the contract price, the latter
the Court of Appeals. On motion for reconsideration, the the cement floor of the plant and on April 18, 1960, they refused to pay for the reason that according to the
latter, however, set aside its original decision and delivered one unit of 110' 26" wide flat belt conveyor, defendant the conveyor system installed by the plaintiff
sentenced La Fuerza to pay to the plaintiff, Associated valued at P3,750.00, and another unit measuring 190' and did not serve the purpose for which the same was
Engineering Co., — hereinafter referred to as the Plaintiff 4" wide flat conveyor, valued at P4,500.00, or a total of manufactured and installed at such a heavy expense. The
— the sum of P8,250.00, with interest at the rate of 1% P13,250.00. Deducting the down payment of P5,000.00 flat belt conveyors installed in the factory of the defendant
per month, from July, 1960 until fully paid, plus P500 as from this value, there is a balance, of P8,250.00 to be paid are still there....
attorney's fees and the costs. Hence, this Petition for by the defendant upon the completion of the installation,
review on certiorari. Exhibit B. xxx xxx xxx

The facts, as found by the Court of First Instance and The work went under way during the months of March and On March 22, 1961, the contractor commenced the
adopted by the Court of Appeals, are: April, during which time the president and general present action to recover the sums of P8,250, balance of
manager of the defendant corporation was duly apprised the stipulated price of the aforementioned conveyors, and
The plaintiff (Associated Engineering, Co., Inc.) is a of the progress of the same because his plant mechanic, P2,000, as attorney's fees, in addition to the costs.
corporation engaged in the manufacture and installation of one Mr. Santos, had kept him informed of the installation
flat belt conveyors. The defendant (La Fuerza, Inc.) is also for which he gave the go signal. It seems that the work In its answer to the complaint, La Fuerza alleged that the
a corporation engaged in the manufacture of wines. was completed during the month of May, 1960. Trial runs "conveyors furnished and installed by the plaintiff do not
Sometime in the month of January, 1960, Antonio Co, the were made in the presence of the president and general meet the conditions and warrantings" (warranties?) of the
manager of the plaintiff corporation, who is an engineer, manager of the defendant corporation, Antonio Co, the latter, and set up a counterclaim for the P5,000 advanced
called the office of the defendant located at 399 Muelle de technical manager of the plaintiff, and some other people. by La Fuerza, which prayed that the complaint be
Binondo, Manila and told Mariano Lim, the President and Several trial runs were made then totalling about five. dismissed; that its contract with the plaintiff be rescinded;
general manager of the defendant that he had just visited These runs were continued during the month of June and that plaintiff be sentenced to refund said sum of
the defendant's plant at Pasong Tamo, Makati, Rizal and where about three trial runs were made and, lastly, during P5,000 to La Fuerza, as well as to pay thereto P1,000 as
was impressed by its size and beauty but he believed it the month of July, 1960. attorney's fees, apart from the costs.
needed a conveyor system to convey empty bottles from
the storage room in the plant to the bottle washers in the As a result of this trial or experimental runs, it was After appropriate proceedings, the Court of First Instance
production room thereof. He therefore offered his services discovered, according to the defendant's general of Manila rendered a decision the dispositive part of which
to manufacture and install a conveyor system which, manager, that the conveyor system did not function to reads:
according to him, would increase production and their satisfaction as represented by the technical manager
efficiency of his business. The president of the defendant of the plaintiff Antonio Co for the reason that, when WHEREFORE, judgment is hereby rendered rescinding
corporation did not make up his mind then but suggested operated several bottles collided with each other, some the contract entered into by the parties in this case,
to Antonio Co to put down his offer in writing. Effectively, jumping off the conveyor belt and were broken, causing marked as Exhibit A, and ordering the plaintiff to refund or
on February 4, 1960, marked as Exhibit A in this case. considerable damage. It was further observed that the return to the defendant the amount of P5,000.00 which
Mariano Lim did not act on the said offer until February flow of the system was so sluggish that in the opinion of they had received as down payment, and the costs of this
11, 1960, when Antonio Co returned to inquire about the the said general manager of the defendant their old action. On the other hand, defendant is ordered to permit
action of the defendant on his said offer. The defendants system of carrying the bottles from the storage room to the plaintiff to remove the flat belt conveyors installed in
president and general manager then expressed his the washers by hand carrying them was even more their premises.
conformity to the offer made in Exhibit A by writing at the efficient and faster.
foot thereof under the word "confirmation" his signature.
142
Sales – Chapter 3-12 Cases
As above indicated, this decision was affirmed by the contract of sale" shall be applicable to a contract for a was not aware thereof. ."This provision shall not apply if
Court of Appeals, which, on motion for reconsideration of piece of work. Considering that Article 1571 is a provision the contrary has been stipulated, and the vendor was not
the plaintiff, later set aside its original decision and on sales, the delivery mentioned therein should be aware of the hidden faults or defects in the thing sold.
rendered another in plaintiff's favor, as stated in the construed in the light of the provisions on sales. Article
opening paragraph hereof. 1497 provides that the thing sold shall be understood as Art. 1567. In the cases of articles 1561, 1562, 1564, 1565
delivered when it is placed in the control and possession and 1566, the vendee may elect between withdrawing
The appealed resolution of the Court of Appeals was, in of the vendee. Therefore, when the thing subject of the from the contract and demanding a proportionate
effect, based upon the theory of prescription of La sale is placed in the control and possession of the reduction of the price, with damages in either case.
Fuerza's right of action for rescission of its contract with vendee, delivery is complete. Delivery is an act of the
the plaintiff, for — in the language of said resolution — vendor. Thus, one of the obligations of the vendor is the xxx xxx xxx
"Article 1571 of the Civil Code provides that an action to delivery of the thing sold (Art. 1495). The vendee has
rescind 'shall be barred after six months from delivery of nothing to do with the act of delivery by the vendor. On the Pursuant to these two (2) articles, if the thing sold has
the thing sold'", and, in the case at bar, La Fuerza did not other hand, acceptance is an obligation on the part of the hidden faults or defects — as the conveyors are claimed
avail of the right to demand rescission until the filing of its vendee (Art. 1582). Delivery and acceptance are two to have — the vendor — in the case at bar, the plaintiff —
answer in the Court of First Instance, on April 17, 1961, or distinct and separate acts of different parties. shall be responsible therefor and the vendee — or La
over ten (10) months after the installation of the conveyors Consequently, acceptance cannot be regarded as a Fuerza, in the present case — "may elect between
in question had been completed on May 30, 1960. condition to complete delivery. withdrawing from the contract and demanding a
proportional reduction of the price, with damages in either
La Fuerza assails the view taken by the Court of Appeals, xxx xxx xxx case." In the exercise of this right of election, La Fuerza
upon the ground: 1) that there has been, in contemplation had chosen to withdraw from the contract, by praying for
of law, no delivery of the conveyors by the plaintiff; and 2) We find no plausible reason to disagree with this view. its rescission; but the action therefor — in the language of
that, assuming that there has been such delivery, the Upon the completion of the installation of the conveyors, Art. 1571 — "shall be barred after six months, from the
period of six (6) months prescribed in said Art. 1571 refers in May, 1960, particularly after the last trial run, in July delivery of the thing sold." The period of four (4) years,
to the "period within which" La Fuerza may "bring an 1960, La Fuerza was in a position to decide whether or provided in Art. 1389 of said Code, for "the action to claim
action to demand compliance of the warranty against not it was satisfied with said conveyors, and, hence, to rescission," applies to contracts, in general, and must
hidden defects", not the action for rescission of the state whether the same were a accepted or rejected. The yields, in the instant case, to said Art. 1571, which refers
contract. Both grounds are untenable. failure of La Fuerza to express categorically whether they to sales in particular.
accepted or rejected the conveyors does not detract from
With respect to the first point, La Fuerza maintains that the fact that the same were actually in its possession and Indeed, in contracts of the latter type, especially when
plaintiff is deemed not to have delivered the conveyors, control; that, accordingly, the conveyors had already been goods, merchandise, machinery or parts or equipment
within the purview of Art. 1571, until it shall have complied delivered by the plaintiff; and that, the period prescribed in thereof are involved, it is obviously wise to require the
with the conditions or requirements of the contract said Art. 1571 had begun to run. parties to define their position, in relation thereto, within
between them — that is to say, until the conveyors shall the shortest possible time. Public interest demands that
meet La Fuerza's "need of a conveyor system that would With respect to the second point raised by La Fuerza, Art. the status of the relations between the vendor and the
mechanically transport empty bottles from the storage 1571 of the Civil Code provides: vendee be not left in a condition of uncertainty for an
room to the bottle workers in the production room thus unreasonable length of time, which would be the case, if
increasing the production and efficiency" of its business- Actions arising from the provisions of the preceding ten the lifetime of the vendee's right of rescission were four
and La Fuerza had accepted said conveyors. articles shall be barred after six months, from the delivery (4) years.
of the thing sold.
On this point, the Court of Appeals had the following to WHEREFORE, the appealed resolution of the Court of
say: xxx xxx xxx Appeals is hereby affirmed, with costs against appellant,
La Fuerza, Inc. It is so ordered.
Article 1571 of the Civil Code provides that an action to Among the "ten articles" referred to in this provision, are
rescind 'shall be barred after six months, from delivery of Articles 1566 and 1567, reading:
the thing sold". This article is made applicable to the case
at bar by Article 1714 which provides that "the pertinent Art. 1566. The vendor is responsible to the vendee for any
provisions on warranty of title against hidden defect in a hidden faults or defects in the thing sold, even though he
143
Sales – Chapter 3-12 Cases
FABIO CAHAYAG and CONRADO RIVERA, On 23 November 1981, a Certificate of Sale covering the On 8 March 1988, the MTC issued a Writ of Execution to
Petitioners, properties, together with all the buildings and enforce its Decision dated 20 October 1986 in Civil Case
vs. improvements existing thereon, was issued in favor of No. 2257 against Dulos Realty "and all persons claiming
COMMERCIAL CREDIT CORPORATION, represented CCC.9 The Certificate of Sale was annotated on the right under defendant."21 The subject of the writ of
by its President, LEONARDO B. ALEJANDRO; corresponding titles to the properties on 8 March 1982.10 execution was Lot 11 Block II,22 which was the lot sold by
TERESITA T. QUA, assisted by her husband Dulos Realty to petitioner Baldoza.
ALFONSO MA. QUA; and the REGISTER OF DEEDS Thereafter, or on 13 January 1983, Dulos Realty entered
OF LAS PINAS, METRO MANILA, DISTRICT IV, into a Contract to Sell with petitioner Escalona over the COMPLAINT FOR ANNULMENT
Respondents. house and lot covered by TCT No. S-29776.11 OF SHERIFF'S SALE AND OTHER DOCUMENTS

Before us are consolidated Rule 45 Petitions1 seeking to On 10 November 1983, an Affidavit of Consolidation in On 5 December 1988, petitioners filed a Complaint
nullify the Court of Appeals (CA) Decision dated 2 favor of respondent CCC dated 26 August 1983 was against respondents for the "Annulment of Sherifffs] Sale
November 20042 and Resolution dated 10 May 20053 in annotated on the corresponding titles to the properties.12 and Other Documents with Preliminary Injunction and/or
CA-G.R. CV No. 47421. The CA Decision reversed and By virtue of the affidavit, TCT Nos. S-39775, S-28335, S- Temporary Restraining Order" before the RTC of Makati
set aside the Decision dated 6 July 1992 issued by the 39778 and S-29776 - all in the name of Dulos Realty - City, where it was docketed as Civil Case No. 88-2599.23
Regional Trial Court (RTC), Branch 65 of Makati.4 were cancelled and TCT Nos. 74531, 74532, 74533 and
74534 were issued in the name of respondent CCC on the The Complaint24 alleged that petitioners Cahayag,
FACTUAL ANTECEDENTS same day.13 Rivera, Escalona and Baldoza were owners of the
properties in question by virtue of Contracts of Sale
Petitioner Dulos Realty was the registered owner of On 10 December 1983, Dulos Realty entered into a Deed individually executed in their favor, and that the Real
certain residential lots covered by Transfer Certificate of of Absolute Sale with petitioner Baldoza over the property Estate Mortgage between Dulos Realty and defendant-
Title (TCT) Nos. S-39767, S-39775, S-28335, S-39778 covered by TCT No. S-39778, together with the appellant CCC did not include the houses, but merely
and S-29776, located at Airmen's Village Subdivision, improvements existing thereon.14 referred to the lands themselves.25 Thus, the inclusion of
Pulang Lupa II, Las Pinas, Metro Manila. the housing units in the Deed of Sale executed by
On 21 December 1983, respondent CCC, through a Deed respondent CCC in favor of respondent Qua was allegedly
On 20 December 1980, Dulos Realty obtained a loan from of Absolute Sale, sold to respondent Qua the same illegal.26
respondent CCC in the amount of P300,000. To secure subject properties, now covered by TCT Nos. 74531,
the loan, the realty executed a Real Estate Mortgage over 74532, 74533 and 74534, which were in the name of Respondents failed to file an answer within the
the subject properties in favor of respondent. The respondent CCC. The sale was duly annotated on the reglementary period. Subsequently, they were declared in
mortgage was duly annotated on the certificates of title on corresponding titles to the properties on 5 January default. They appealed the order of default but their
3 February 1981.5 1984.15 appeal was dismissed on 8 February 1990.27

On 29 March 1981, Dulos Realty entered into a Contract Accordingly, TCT Nos. 74531, 74532, 74533 and 74534 On 6 July 1992, the RTC rendered a Decision,28 which
to Sell with petitioner Cahayag over the lot covered by were cancelled; and TCT Nos. 77012, 77013, 77014 and ruled that the houses were not included in the Real Estate
TCT No. S-39775.6 770015 were issued to respondent Qua on 5 January Mortgage; and that the foreclosure of the mortgage over
1984.16 the subject lots, as well as the housing units, was not
On 12 August 1981, Dulos Realty entered into another valid.29 The trial court held that this conclusion was
Contract to Sell, this time with petitioner Rivera over the Subsequently, respondent Qua filed ejectment suits established by the plaintiffs' evidence, which went
lot covered by TCT No. S-28335.7 individually against petitioners Du1os Realty,17 unrefuted when defendants were declared in default.30
Cahayag,18 Esca1ona,19 and Rivera20 before the
Dulos Realty defaulted in the payment of the mortgage Metropolitan Trial Court (MTC) of Las Piñas, Metro THE CA DECISION
loan, prompting respondent CCC to initiate extrajudicial Manila.
foreclosure proceedings. On 17 November 1981, the Respondents proceeded to the CA, where they secured a
auction sale was held, with respondent CCC emerging as The MTC rendered Decisions in favor of respondent Qua. favorable ruling. In its Decision rendered on 2 November
the highest bidder.8 It ordered Dulos Realty, Escalona, Cahayag, and Rivera 2004,31 the appellate court held that the extrajudicial
to vacate the properties. foreclosure was valid, since the Real Estate Mortgage

144
Sales – Chapter 3-12 Cases
clearly included the buildings and improvements on the executed on 10 January 1979, which preceded the the mortgage contract, as they were sold under existing
lands, subject of the mortgage. execution of the Deed of Real Estate Mortgage and the Contracts to Sell and Deed of Absolute Sale.39
registration of the mortgage on 3 February 1981.34 After
After establishing the inclusion of the housing units in the full payment of the price under the Contract to Sell, Dulos Petitioners Cahayag, Rivera and Escalona lean on the
Real Estate Mortgage, the CA determined the rights of the Realty executed the Deed of Absolute Sale. In other unregistered Contracts to Sell they had individually
buyers in the Contracts to Sell/Contract of Sale vis-a-vis words, Baldoza is arguing that she has a better title to the executed with Dulos Realty as vendor. For his part,
those of the mortgagee and its successor-in-interest. property than respondent Qua since the unregistered petitioner Baldoza points to the Deed of Absolute Sale
contract to sell in her favor was executed before the executed by Dulos Realty in his favor.
In the cases of petitioners Cahayag, Rivera and Escalona, registration of the mortgage. But the CA ignored Exhibit
the CA pointed to lack of evidence establishing full "L" and merely stated that there was only a Deed of Better Right over the Properties
payment of the price. As supporting reason, it stated that Absolute Sale in favor of Baldoza.
even if there were full payment of the purchase price, the Petitioners claim that respondent CCC cannot claim to be
mortgagee and the latter's successor-in-interest had a THE ARGUMENTS a mortgagee in good faith, since it is a financial
better right over the properties. The CA anchored this institution.40 As such, respondent CCC knew that it was
conclusion on the fact that the Real Estate Mortgage was The arguments of petitioners, as stated in their respective dealing with a subdivision developer, which was in the
annotated at the back of the titles to the subject properties Memoranda, are summarized as follows: business of selling subdivision lots.41 Dela Merced v.
before the execution of the Contracts to Sell. It said that GSIS42 which states that the general rule that a
the annotation constituted sufficient notice to third parties Coverage of the Mortgage mortgagee need not look beyond the title cannot benefit
that the property was subject to an encumbrance. With the banks and other financial institutions, as a higher due
notice, Cahayag, Rivera and Escalona should have Initially, petitioners attempt to stave off the effects of the diligence requirement is imposed on them.
redeemed the properties within the one-year redemption extra judicial foreclosure by attacking the coverage of the
period, but they failed to do so. Consequently, the right of Real Estate Mortgage with respect to its subject-matter.35 They also raise the contention that lack of full payment of
respondent CCC over the properties became absolute, They draw attention to the fact that the List of Properties the purchase price under the Contracts to Sell on the part
and the transfer to respondent Qua was valid. attached to the Deed of Real Estate Mortgage refers of Cahayag, Rivera and Escalona was due to respondent
merely to the lands themselves and does not include the Qua's "harassment and unlawful actuations.43
As regards Baldoza, though the case involved a Contract housing units found thereon.36 Petitioners also contend
of Sale, and not a mere Contract to Sell, the CA declared that doubts should be resolved against the drafter Petitioners further state that respondent Qua is a mere
the transaction null and void on the purported ground that inasmuch as the agreement is a contract of adhesion, transferee of respondent CCC and that, like a stream, she
Dulos was no longer the owner at the time of the sale. having been prepared by the mortgagee.37 cannot rise higher than her source. They also argue that
Qua is not an innocent purchaser for value, since she is a
The CA accordingly reversed and set aside the RTC As backup argument for the theory that the houses are former investor of respondent CCC and one of its principal
Decision, dismissed the case for lack of merit, and outside the coverage of the mortgage agreement, stockholders.44
ordered petitioners to surrender possession of the petitioners argue that the improvements were not owned
properties to respondent Qua. by Dulos Realty, the mortgagor, but by its buyers under No Prior Written HLURB Approval of
the Contracts to Sell and Contracts of Sale; hence, those the Mortgage
THE RULE 45 PETITIONS improvements are excluded from the coverage of the real
estate mortgage. Finally, petitioners allege that the mortgage contract in this
On 30 May 2005, petitioners Cahayag and Rivera filed case was not approved by the BLURB, which violates
their Rule 45 Petition with this Court.32 For their part, Validity of the Mortgage Section 18 of P.D. 95745 and results in the nullity of the
petitioners Dulos Realty, Baldoza and Escalona filed their mortgage.46
Rule 45 Petition on 19 July 2005.33 Petitioners next challenge the validity of the foreclosure
sale on the ground that the mortgage executed by the Exhibit "L" as Evidence of a Prior
In the Petition under G.R. No. 168357, it is argued, among mortgagor (petitioner Dulos Realty) and the mortgagee Contract to Sell
others, that the Deed of Absolute Sale in favor of (respondent CCC) was null and void.38 Petitioners claim
petitioner Baldoza was the culmination of a Contract to that Dulos Realty was no longer the owner of the The matter of CA ignoring Exhibit "L" as evidence of a
Sell between her and Dulos Realty. She claims that the properties it had mortgaged at the time of the execution of prior unregistered Contract to Sell was not included in the
Contract to Sell, marked as Exhibit "L" during the trial, was Memoranda of petitioners.
145
Sales – Chapter 3-12 Cases
and made and integral part hereof, together with all the to Sell in its Decision. Respondent Qua has raised this
THE ISSUES buildings and/or other improvements now existing or point in her Memorandum filed with us. This Court cannot
which may hereafter be place[d] or constructed thereon, be bound by the factual finding of the CA with regard to
Based on the foregoing facts and arguments raised by all of which the MORTGAGOR hereby warrants that he is the date of the Contract to Sell in favor of Cahayag. The
petitioners, the threshold issues to be resolved are the the absolute owner and exclusive possessor thereof, free general rule that the Court is bound by the factual findings
following: from all liens and encumbrances of whatever kind and of the CA must yield in this case, as it falls under one of
nature. xxx.47 (Emphasis Ours) the exceptions: when the findings of the CA are
1. Whether the real mortgage covers the lands only, as contradicted by the evidence on record.53 In this case,
enumerated in the Deed of Real Estate Mortgage or the Thus, the housing units would fall under the catch-all there is nothing in the records to support the CA's
housing units as well; phrase "together with all the buildings and/or other conclusion that the Contract to Sell was executed on 29
improvements now existing or which may hereafter be September 1980. The evidence on record, however,
2. Whether Dulos Realty was the owner of the properties it placed or constructed thereon." reveals that the correct date is 29 March 1981.
had mortgaged at the time of its execution in view of the
various Contracts to Sell and Deed of Absolute Sale The contra proferentem rule finds no application to this In the case of petitioner Rivera, the corresponding
respectively executed in favor of petitioners Cahayag, case. The doctrine provides that in the interpretation of Contract to Sell in his favor was executed only on 12
Rivera, Escalona and Cahayag; documents, ambiguities are to be construed against the August 1981, or almost eight months after the perfection
drafter.48 By its very nature, the precept assumes the of the mortgage contract on 20 December 1980.
3. Who, as between petitioners-buyers and respondent existence of an ambiguity in the contract, which is why
Qua, has a better right over the properties? contra proferentem is also called the ambiguity Lastly, Dulos Realty executed the Deed of Absolute Sale
doctrine.49 In this case, the Deed of Real Estate in favor of petitioner Baldoza on 10 December 1983,
4. Whether the Deed of Absolute Sale in favor of Baldoza Mortgage clearly establishes that the improvements found which was almost three years from the time the mortgage
was not preceded by a Contract to Sell and full payment on the real properties listed therein are included as contract was executed on 20 December 1980.
of the purchase price; and subject-matter of the contract. It covers not only the real
properties, but the buildings and improvements thereon as There was neither a contract to sell nor a deed of absolute
5. Whether the mortgage is void on the ground that it well. sale to speak of when the mortgage was executed.
lacked the prior written approval of the HLURB.
2. Challenge to the Foreclosure Petitioners equate a contract to sell to a contract of sale,
OUR RULING Sale with Regard to the in which the vendor loses ownership over the property
Ownership of the Mortgaged upon its delivery.54 But a contract to sell, standing alone,
We deny the Petition for reasons as follows. Properties does not transfer ownership.55 At the point of perfection,
the seller under a contract to sell does not even have the
1. Attack on the Subject-matter of To begin with, the Contracts to Sell and Deed of Absolute obligation to transfer ownership to the buyer.56 The
the Real Estate Mortgage Sale could not have posed an impediment at all to the obligation arises only when the buyer fulfills the condition:
mortgage, given that these contracts had yet to full payment of the purchase price.57 In other words, the
It is true that the List of Properties attached to the Deed of materialize when the mortgage was constituted. They seller retains ownership at the time of the execution of the
Real Estate Mortgage refers merely to the lands were all executed after the constitution of the Real Estate contract to sell.58
themselves and does not include the housing units found Mortgage on 20 December 1980.
thereon. A plain reading of the Real Estate Mortgage, There is no evidence to show that any of petitioners
however, reveals that it covers the housing units as well. As regards Cahayag, the Contract to Sell in his favor was Cahayag, Rivera and Escalona were able to effect full
We quote the pertinent provision of the agreement: executed on 29 March 1981, more than three months after payment of the purchase price, which could have at least
the execution of the mortgage contract.50 This is taken given rise to the obligation to transfer ownership.
[T]he MORTGAGOR has transferred and conveyed and, from the Contract to Sell itself, which forms part of the Petitioners Cahayag and Rivera even admit that they
by these presents, do hereby transfer and convey by way records of this case.51 defaulted on their obligations under their respective
of FIRST MORTGAGE unto the MORTGAGEE, its Contracts to Sell, although they attribute the default to
successors and assigns the real properties described in At this juncture, we note that the CA, for reasons respondent Qua's "harassment and unlawful
the list appearing at the back of this document and/or in a unknown, specified 29 September 1980,52 and not 29 actuations."59 The statement, though, was a mere
supplemental document attached hereto as Annex "A" March 1981, as the date of the execution of the Contract
146
Sales – Chapter 3-12 Cases
allegation that was left unsubstantiated and, as such, registered mortgage contract. The contract to sell between to the mortgage. Consequently, the determination of good
could not qualify as proof of anything.60 the mortgagors (Spouses Zulueta) and the buyer faith does not come into play.
(Francisco Dela Merced) was executed before the
3. Who Has a Better Right over the Properties former's constitution of the mortgage in favor of GSIS. Dela Merced materially differs from this case on another
Because the Zuluetas defaulted on their loans, the point. The Contract to Sell in favor of Dela Merced was
Registration of the mortgage hound the buyers under the mortgage was foreclosed; the properties were sold at followed by full payment of the price and execution of the
Contracts to Sell public auction to GSIS as the highest bidder; and the titles Deed of Absolute Sale. In this case, the Contract to Sell in
were consolidated after the spouses' failure to redeem the favor of each of petitioners Cahayag, Rivera and
Registration of the mortgage establishes a real right or properties within the one-year redemption period. GSIS Escalona, is not coupled with full payment and execution
lien in favor of the mortgagee, as provided by Articles later sold the contested lot to Elizabeth D. Manlongat and of a deed of absolute sale.
131261 and 212662 of the Civil Code.63 Corollary to the Ma. Therese D. Manlongat. However, Dela Merced was
rule, the lien has been treated as "inseparable from the able to fully pay the purchase price to Spouses Zulueta, This case also needs to be distinguished from Luzon
property inasmuch as it is a right in rem."64 In other who executed a Deed of Absolute Sale in his favor prior to Development Bank v. Enriquez.67 In that case, the
words, it binds third persons to the mortgage. the foreclosure sale. unregistered Contract to Sell was executed after the
execution of the mortgage. Instead of resorting to
The purpose of registration is to notify persons other than This Court stated therein the general rule that the foreclosure, the owner/developer and the bank entered
the parties to the contract that a transaction concerning purchaser is not required to go beyond the Torrens title if into a dacion en pago. The Court declared that the bank
the property was entered into.65 Ultimately, registration, there is nothing therein to indicate any cloud or vice in the was bound by the Contract to Sell despite the non-
because it provides constructive notice to the whole world, ownership of the property or any encumbrance thereon. registration of the contract. It reasoned that the bank
makes the certificate of title reliable, such that third The case nonetheless provided an exception to the impliedly assumed the risk that some of the units might
persons dealing with registered land need only look at the general rule. The exception arises when the purchaser or have been covered by contracts to sell. On the other
certificate to determine the status of the property.66 mortgagee has knowledge of a defect in the vendor's title hand, the Court pronounced the mortgage to be void, as it
or lack thereof, or is aware of sufficient facts to induce a was without the approval of the Housing and Land Use
In this case, the Real Estate Mortgage over the property reasonably prudent person to inquire into the status of the Regulatory Board (HLURB). The Court consequently
was registered on 3 February 1981. On the other hand, property under litigation. The Court applied the exception, ordered the unit buyer in that case to pay the balance to
the Contracts to Sell were all executed after the taking into consideration the fact that GSIS, the the bank, after which the buyer was obliged to deliver a
registration of the mortgage. The Contract to Sell in favor mortgagee, was a financing institution. clean title to the property.
of petitioner Cahayag was executed on 29 March 1981, or
almost two months after the registration of the mortgage. But Dela Merced is not relevant here. Dela Merced There are points of distinction between the case at bar
The corresponding Contract to Sell in favor of Rivera was involved a Contract to Sell that was executed prior to the and Luzon Development Bank. First, there is a definite
executed only on 12 August 1981, roughly six months mortgage, while the Contracts to Sell in this case were all finding in Luzon Development Bank that the mortgage
after the registration of the mortgage contract. Lastly, the executed after the constitution and registration of the was without prior HLURB approval, rendering the
Contract to Sell in favor of Escalona was executed on 13 mortgage. mortgage void. In the present case, as will be discussed
January 1983, or nearly two years after the registration of later, there is no proof from the records on whether the
the mortgage on 3 February 1981. In Dela Merced, since GSIS had knowledge of the HLURB did or did not approve the mortgage. Second,
contract to sell, this knowledge was equivalent to the Luzon Development Bank did not even reach the
Consequently, petitioners Cahayag, Rivera and Escalona, registration of the Contract to Sell. Effectively, this foreclosure stage of the mortgage. This case, however,
were bound to the mortgage executed between mortgagor constitutes registration canceled out the subsequent not only reached the foreclosure stage; it even went past
Dulos Realty and mortgagee CCC, by virtue of its registration of the mortgage. In other words, the buyer the redemption period, consolidation of the title in the
registration. Definitely, the buyers each had constructive under the Contract to Sell became the- first to register. owner, and sale of the property by the highest bidder to a
knowledge of the existence of the mortgage contract when Following the priority in time rule in civil law, the lot buyer third person.
they individually executed the Contracts to Sell. was accorded preference or priority in right in Dela
Merced. The first distinction deserves elaboration. The absence of
Dela Merced v. GSIS not applicable prior written approval of the mortgage by the HLURB
In this case, the registration of the mortgage, which rendered it void. This effectively wiped out any discussion
Petitioner invokes the above case. Dela Merced involved predated the Contracts to Sell, already bound the buyers on whether registration bound the installment buyer. In
a clash between an unrecorded contract to sell and a fact, Luzon Development Bank did not even bother to
147
Sales – Chapter 3-12 Cases
state whether the mortgage was registered or not. More of the subject lots, became absolute. As a matter of right, contract of sale at its consummation stage, and not at the
important, the tables were turned when Luzon it was entitled to the consolidation of the titles in its name perfection stage.74
Development Bank held that the bank was bound to the and to the possession of those lots. Further, the right of
Contract to Sell in view of the latter's constructive notice of respondent CCC over the lots was transferred to Cavite Development Bank v. Spouses Syrus Lim75 puts
the Contract to Sell. Stated differently, the actually respondent Qua by virtue of the Deed of Sale executed nemo dat quad non habet in its proper place.1âwphi1
unregistered Contract to Sell became fictionally between them. Initially, the Court rules out ownership as a requirement
registered, making it binding on the bank. for the perfection of a contract of sale. For all that is
Given the foregoing considerations, respondent Qua, who required is a meeting of the minds upon the object of the
In this case, on account of its registration, and the fact that now has title to the properties subject of the various contract and the price. The case then proceeds to give
the contracts were entered into after it, the mortgage is Contracts to Sell, is the lawful owner thereof. examples of the rule. It cites Article 1434 of the Civil
valid even as to petitioners. Code, which provides that in case the seller does not own
Foreclosure Sale vs. Contract of Sale the subject matter of the contract at the time of the sale,
No Redemption within One Year from the Foreclosure but later acquires title to the thing sold, ownership shall
Sale When Dulos Realty executed a Deed of Absolute Sale pass to the buyer. The Court also refers to the rule as the
covering the real property registered under TCT No. S- rationale behind Article 1462, which deals with sale of
When it comes to extrajudicial foreclosures, the law68 39778 in favor of petitioner Baldoza on 10 December "future goods."
grants mortgagors or their successors-in-interest an 1983, it was no longer the owner of the property. Titles to
opportunity to redeem the property within one year from the subject properties, including the one sold to Baldoza, Cavite Development Bank thereafter turns to Article 1459,
the date of the sale. The one-year period has been had already been consolidated in favor of respondent which requires ownership by the seller of the thing sold at
jurisprudentially held to be counted from the registration of CCC as early as 10 November 1983. In fact, on the same the time of delivery or consummation stage of the sale.
the foreclosure sale with the Register of Deeds.69 An date, the titles to the subject lots in the name of Dulos The Court explains that if the rule were otherwise, the
exception to this rule has been carved out by Congress for Realty had already been cancelled and new ones issued seller would not be able to comply with the latter's
juridical mortgagors. Section 47 of the General Banking to respondent CCC. obligation to transfer ownership to the buyer under a
Law of 2000 shortens the redemption period to within perfected contract of sale. The Court ends the discourse
three months after the foreclosure sale or until the The fact that Dulos Realty was no longer the owner of the with the conclusion that "[i]t is at the consummation stage
registration of the certificate of sale, whichever comes real property at the time of the sale led the CA to declare where the principle of nemo dat quad non habet
first.70 The General Banking Law of 2000 came into law that the Contract of Sale was null and void. On this applies.76
on 13 June 2000. premise, the appellate court concluded that respondent
Qua had a better title to the property over petitioner Case law also provides that the fact th,at the seller is not
If the redemption period expires and the mortgagors or Baldoza. the owner of the subject matter of the sale at the time of
their successors-in-interest fail to redeem the foreclosed perfection does not make the sale void.77
property, the title thereto is consolidated in the We find no error in the conclusion of the CA that
purchaser.71 The consolidation confirms the purchaser as respondent Qua has a better right to the property. The Hence, the lesson: for title to pass to the buyer, the seller
the owner of the property; concurrently, the mortgagor-for problem lies with its reasoning. We therefore take a must be the owner of the thing sold at the consummation
failure to exercise the right of redemption within the different route to reach the same conclusion. stage or at the time of delivery of the item sold. The seller
period-loses all interest in the property.72 need not be the owner at the perfection stage of the
Proper place of nemo dat quod non habet in the Law on contract, whether it is of a contract to sell or a contract of
We now apply the rules to this case. Sales sale. Ownership is not a requirement for a valid contract of
sale; it is a requirement for a valid transfer of ownership'.
As the foreclosure sale took place prior to the advent of Undeniably, there is an established rule under the law on
the General Banking Law of 2000, the applicable sales that one cannot give what one does not have (Nemo Consequently, it was not correct for the CA to consider the
redemption period is one year. In this case, because the dat quad non ha bet).73 The CA, however, confuses the contract of sale void. The CA erroneously considered lack
Certificate of Sale in favor of respondent CCC was application of this rule with respect to time. It makes the of ownership on the part of the seller as having an effect
registered on 8 March 1982, the redemption period was nemo dat quad non habet rule a requirement for the on the validity of the sale. The sale was very much valid
until 8 March 1983. It lapsed without any right of perfection of a contract of sale, such that a violation when the Deed of Absolute Sale between the parties was
redemption having been exercised by Dulos Realty. thereof goes into the validity of the sale. But the Latin executed on 10 December 1983, even though title to the
Consequently, the right of respondent CCC, as purchaser precept has been jurisprudentially held to apply to a property had earlier been consolidated in favor of
148
Sales – Chapter 3-12 Cases
respondent CCC as early as 10 November 1983. The fact no need to go into a discussion of whether Qua is an The exception does not apply to the case of Baldoza.
that Dulos Realty was no longer the owner of the property innocent purchaser for value. While she duly identified the Contract to Sell during her
in question at the time of the sale did not affect the validity direct examination, which was duly recorded, Exhibit "L"
of the contract. 4. Dispute as to the Factual Finding of the CA that the was not incorporated into the records.
Deed of Absolute Sale in Favor of Baldoza was not
On the contrary, lack of title goes into the performance of Preceded by a Contract to Sell and Full Payment of the Exhibit "L" not relevant
a contract of sale. It is therefore crucial to determine in Purchase Price
this case if the seller was the owner at the time of delivery Be that as it may, the contention that a Contract to Sell in
of the object of the sale. For this purpose, it should be We absolutely discard the argument. We can think of at favor of Baldoza preceded the sale in her favor is
noted that execution of a public instrument evidencing a least four reasons why. First, Exhibit "L" was not formally irrelevant. It must be stressed that the sale to Baldoza
sale translates to delivery.78 It transfers ownership of the offered in evidence. Second, it was not even incorporated made by Dulos Realty took place after the lapse of the
item sold to the buyer.79 into the records. Third, the argument is irrelevant. Fourth, redemption period and after consolidation of title in the
it was even abandoned in the Memoranda filed by name of respondent CCC on 10 November 1983, one
In this case, the delivery coincided with the perfection of petitioners with us. Last, we are not a trier of facts and month prior to the sale to Baldoza on 10 December 1983.
the contract -The Deed of Absolute Sale covering the real thus we yield to the finding of the CA. Dulos Realty still would have lost all interest over the
property in favor of petitioner Baldoza was executed on 10 property mortgaged.
December 1983. As already mentioned, Dulos Realty was Exhibit "L" not formally offered
no longer the owner of the property on that date. The fact that Dulos Realty ceased to be the owner of the
Accordingly, it could not have validly transferred A perusal of the records shows that the Contract to Sell property and therefore it could no longer effect delivery of
ownership of the real property it had sold to petitioner. that Baldoza referred to had in fact been marked as the property at the time the Deed of Absolute Sale in favor
Exhibit "L" during her direct examination in court.81 Even of Baldoza was executed is the very reason why the case
Thus, the correct conclusion that should be made is that so, Exhibit "L" was never formally offered as evidence. For of Baldoza cannot be compared with Dela Merced. In the
while there was a valid sale, there was no valid transfer of this reason, we reject her contention. Courts do not case, the buyer in the Contract to Sell was able to effect
title to Baldoza, since Dulos Realty was no longer the consider evidence that has not been formally offered.82 full payment of the purchase price and to execute a Deed
owner at the time of the execution of the Deed of Absolute This explains why the CA never mentioned the alleged of Absolute Sale in his favor before the foreclosure sale.
Sale. Contract to Sell in favor of Baldoza. In this case, the full payment of the purchase price and
the execution of a Deed of Absolute Sale in favor of
No Bad Faith on Qua The rationale behind the rule rests on the need for judges Baldoza was done after the foreclosure sale.
to confine their factual findings and ultimately their
The contention that Qua is a stockholder and former judgment solely and strictly to the evidence offered by the Issue over Exhibit "L" not included in the Memorandum
member of the Board of Directors of respondent CCC and parties to a suit.83 The rule has a threefold purpose. It
therefore she is not exactly a stranger to the affairs of allows the trial judge to know the purpose of the evidence Equally important is the fact that petitioners failed to
CCC is not even relevant. presented; affords opposing parties the opportunity to include the issue over Exhibit "L" in any of the Memoranda
examine the evidence and object to its admissibility when they filed with us. The omission is fatal. Issues raised in
An innocent purchaser for value is one who "buys the necessary; and facilitates review, given that an appellate previous pleadings but not included in the memorandum
property of another without notice that some other person court does not have to review documents that have not are deemed waived or abandoned (A.M. No. 99-2-04-SC).
has a right to or interest in it, and who pays a full and fair been subjected to scrutiny by the trial court.84 As they are "a summation of the parties' previous
price at the time of the purchase or before receiving any pleadings, the memoranda alone may be considered by
notice of another person's claim."80 The concept thus Exhibit "L" not incorporated into the records the Court in deciding or resolving the petition."85 Thus,
presupposes that there must be an adverse claim or even as the issue was raised in the Petition, the Court
defect in the title to the property to be purchased by the The rule, of course, admits an exception. Evidence not may not consider it in resolving the case on the ground of
innocent purchaser for value. formally offered may be admitted and considered by the failure of petitioners to include the issue in the
trial court so long as the following requirements obtain: (1) Memorandum. They have either waived or abandoned it.
Respondent Qua traces her title to respondent CCC, the evidence is duly identified by testimony duly recorded;
whose acquisition over the property proceeded from a and (2) the evidence is incorporated into the records of 5. Issue of HLURB's Non-Approval of the Mortgage
foreclosure sale that was valid. As there is no defect in the the case.
title of respondent CCC to speak of in this case, there is
149
Sales – Chapter 3-12 Cases
Petitioners allege before the Court that the mortgage any way related to jurisdiction, and so the first exception is
contract in this case was not approved by the HLURB. not applicable. There is nothing in the record to allow us to
They claim that this violates Section 18 of P.D. 95786 and make any conclusion with respect to this new allegation.
results in the nullity of the mortgage. Respondents have
disputed the claim and counter-argue that the allegation of Neither will the case fall under the second exception.
the petitioners is not supported by evidence. Respondents Evidence would be required of the respondents to
likewise aver that the argument was raised for the first disprove the new allegation that the mortgage did not
time on appeal.87 have the requisite prior HLURB approval. Besides, to the
mind of this court, to allow petitioners to change their
It is rather too late in the day for petitioners to raise this theory at this stage of the proceedings will be exceedingly
argument. Parties are not permitted to change their theory inappropriate.
of a case at the appellate stage.88 Thus, theories and
issues not raised at the trial level will not be considered by Petitioners raised the issue only after obtaining an
a reviewing court on the ground that they cannot be raised unfavorable judgment from the CA. Undoubtedly, if we
for the first time on appeal.89 Overriding considerations of allow a change of theory late in the game, so to speak, we
fair play, justice and due process dictate this recognized will unjustifiably close our eyes to the fundamental right of
rule.90 This Court cannot even receive evidence on this petitioners to procedural due process. They will lose the
matter. opportunity to meet the challenge, because trial has
already ended. Ultimately, we will be throwing the
Petitioners' original theory of the case is the nullity of the Constitutional rulebook out the window.
mortgage on the grounds previously discussed. If
petitioners are allowed to introduce their new theory, WHEREFORE, premises considered, the Petitions are
respondents would have no more opportunity to rebut the DENIED, and the Court of Appeals Decision dated 2
new claim with contrary evidence, as the trial stage has November 2004 and Resolution dated 10 May 2005 in
already been terminated. In the interest of fair play and CA-G.R. CV No. 47421 are hereby AFFIRMED.
justice, the introduction of the new argument must be
barred.91 SO ORDERED.

Exceptions Not Applicable

The Court is aware that the foregoing is merely a general


rule. Exceptions are written in case law: first, an issue of
jurisdiction may be raised at any time, even on appeal, for
as long as the exercise thereof will not result in a mockery
of the demands of fair play;92 second, in the interest of
justice and at the sound discretion of the appellate court,
a party may be allowed to change its legal theory on
appeal, but only when the factual bases thereof would not
require further presentation of evidence by the adverse
party for the purpose of addressing the issue raised in the
new theory;93 and last, which is actually a bogus
exception, is when the question falls within the issues
raised at the trial court.94

The exceptions do not apply to the instant case. The new


argument offered in this case concerns a factual matter -
prior approval by the HLURB. This prerequisite is not in
150
Sales – Chapter 3-12 Cases
BEHN, MEYER & CO., plaintiff, direction of the said Alien Property Custodian, one W.D. assets and estate of the said firm, upon giving a bond in
vs. Pemberton was appointed receiver and placed in full the sum of P1,000.
J.S. STANLEY, ET AL., defendants. charge of the business and assets of the firm.
On the 4th of September, 1923, the said Lazarus G.
And During the month of January, 1919, the business of the Joseph, as such receiver, commenced an action in the
Philippine branch of Behn, Meyer & Co., Ltd., was Court of First Instance of Manila against the Bank of the
LAZARUS G. JOSEPH and A.N. JUREIDINI & BROS., liquidated and the property and assets of the corporation Philippine Islands and J.M. Menzi, being civil case No.
appellants, in the Philippine Islands, including the goodwill, trade- 24892 of said court, to annul the aforesaid sale of the
vs. marks, accounts receivable, together with all vouchers, business, property, and assets, etc., of the said Behn, &
JOHN BORDMAN, J.M. MENZI, and THE BANK OF entries, and other proofs of the indebtedness, such as the Co., Ltd., to John Bordman and to recover back the
THE PHILIPPINE ISLANDS, intervenors-appellees. books of account, etc., were sold to one of the intervenors property sold as property of the said Behn, Meyer & Co.,
herein, John Bordman, by the direction and under the Ltd., and for an accounting and other relief.
There is particularly no dispute as to the facts in this case. supervision of the said Alien Property Custodian, in
On January 23, 1917, Behn, Meyer & Co., Ltd., a foreign accordance with the provisions of the Alien Enemy Act, for On the 5th of September, 1923, the said Lazarus G.
corporation with a branch in the Philippine Islands, the sum of P660,000, as shown by the letters and bills of Joseph, in his capacity of receiver, appeared in the
brought an action against the Collector of Customs to sale, Exhibits B, C, D, and E. present case in the Court of First Instance and obtained
recover the possession of certain merchandise imported an order directed to the said J.M. Menzi citing him to
into the Islands and then in the hands of the Collector. The intervenor herein the Bank of the Philippine Islands, appear before the court on a certain date to show cause
A.N. Jureidini & Bros. intervened in the case and claimed advanced to Bordman the sum of P660,000 with which to why he should not turn over to the said receiver the books
title to the merchandise under a sale of the same ordered purchase the said business, property, and assets of the of account of the said Behn, Meyer & Co., Ltd.
by the British Admiralty Court of Alexandria, Egypt, in said Behn, Meyer & Co., Ltd., which sum was turned over
prize court proceedings. to W.D. Pemberton, the receiver appointed by the Alien On September 14, 1923, John Bordman, J.M. Menzi, and
Property Custodian. the Bank of the Philippine Islands filed in the same case a
The Court of First Instance on February 28, 1918, motion for permission to intervene in the receivership
rendered judgment in favor of Behn, Meyer & Co., on the On the 21st of February, 1919, Behn, Meyer & Co., Ltd., proceedings solely for the purpose of vacating the order of
ground that the title to the merchandise originally rested in was declared by the Alien Property Custodian to be an August 10, 1922, appointing a receiver for the property,
Behn, Meyer & Co., Ltd., and that no record on the prize enemy not holding a license granted by the President, and assets, and estate of the said Behn, Meyer & Co., Ltd.,
court proceedings showing that Behn, Meyer & Co., Ltd., on the same date demand was made on the receiver to and alleging in support thereof that they had a legal
had been divested of the title had been presented in convey, transfer, assign, deliver, and pay over to the Alien interest in the subject-matter of said receivership and an
evidence. On appeal to the Supreme Court the judgment Property Custodian the bet proceeds of the sale and interest against that of the parties to said
was reversed and the case remanded to the court below liquidation of the business, property, and assets aforesaid, proceedings.lawphi1.net
with instructions to allow Jureidini & Bros. a reasonable and by virtue of that demand, the said net proceeds in the
time within which to obtain a duly certified copy of the sum of P392,674.96 was on February 28, 1919, delivered At the same time the intervenors filed a verified motion
decision of the Admiralty Court of Alexandria, in which the to the managing director of the office of the Alien Property setting forth the facts hereinabove stated asking that the
court declared that the merchandise constituted lawful Custodian in the Philippine Islands, as shown by Exhibits said order of August 10, 1922, appointing the said
prize. 1 A new trial was held on February 24, 1922, after F and G, which sum as far as the record shows, is still in Lazarus G. Joseph, receiver of the said Behn, Meyer &
which a judgment was entered in favor of A.N. Jureidini & possession of the Alien Property Custodian. Co. Ltd., be vacated and set aside on the ground that
Bros. and against Behn, Meyer & Co., Ltd., for the sum of Jureidini & Bros., under the facts and circumstances
P1,988 in damages for the further sum of P1,988 for the Execution of the judgment of February 24, 1922, in favor stated, had no legal right to such receivership and that the
value of the merchandise in default of delivery to Jureidini of A.N. Jureidini & Bros, having been issued and returned court had no jurisdiction to make such appointment, and
& Bros. unsatisfied, Jureidini & Bros. on August 8, 1922, filed an that consequently its order to that effect was null and void.
ex-parte petition in the same case praying that a receiver
In the meantime, on the 16th day of February, 1918, all be appointed by the court to take charge of the estate and Upon hearing, the Court of First Instance, under date of
the business, property, and assets of every nature of the effects of Behn, Meyer & Co., Ltd., and on August 10, September 26, 1923, entered an order, the dispositive
firm of Behn, Meyer & Co., Ltd., were taken over by the 1922, the Court of First Instance issued an order part of which reads as follows:
Alien Property Custodian of the United States under the appointing Lazarus G. Joseph receiver of the property,
provisions of the Trading with the Enemy Act and by
151
Sales – Chapter 3-12 Cases
For the foregoing and the interests of J.M. Menzi, John which still were an open issue and did not attempt to money of other property so requested and held, may give
Bordman and the Bank of the Philippine Islands in this interfere in the part of the case which was covered by the notice of his claim and institute a suit in equity against the
proceeding having, in the opinion of the court, been final judgment. They claimed no interest in the controversy Custodian or the Treasurer, as the case may be, to
shown, that of Bordman consisting in his having in his between Jureidini & Bros., and Behn, Meyer & Co., Ltd., establish and enforce his claim, and where suit is brought,
acquired through purchase for the sum of P660,000 all the but that Bordman and the Bank of the Philippine Islands the money or property is to be retained by the Custodian
interests, rights, choses in action, books, vouchers of the had a vital interest in the subsequent receivership is or in the Treasury, to abide the final decree. The same
herein plaintiff; that of J.M. Menzi in his having been clearly shown by the fact that one of the first actions of the section further provides:
designated by said Bordman to take charge of said receiver appears to have been the institution of an action
properties and books in his name; and that of the Bank of against them to annul the sale made by the Alien Property Except as herein provided, the money or other property
the Philippine Islands in its having furnished the sum of Custodian to Bordman, thus disturbing the latter in his ' conveyed, transferred, assigned, delivered, or paid to the
money with which said Bordman made the purchase, it is property rights and threatening the lien held by the bank Alien Property Custodian shall not be liable to lien,
hereby adjudged to permit said parties, as they are hereby upon the property sold. As to the appellee Menzi, it is attachment, garnishment, trustee, process, or execution,
permitted and authorized, to intervene in this case; and sufficient to say that he was brought into the present case or subject to any order to decree of any court.
the court having reached the conclusion that it has not, by the receiver himself on the order to show cause why he
and did not have, any jurisdiction to appoint a receiver in did not turn over and deliver to said receiver the books of Section 17 of the same Act provides:
view of the fact that all of the properties of the said plaintiff account of Behn, Meyer & Co., Ltd. We fail to find any
had been sold by the Alien Property Custodian in error or abuse of discretion on the part of the court below That the district courts of the United States are hereby
accordance with the Act of Congress hereinbefore in permitting the intervention. given jurisdiction to make and enter all such rules as to
mentioned; it is hereby adjudged that the order of this notice and otherwise, and all such orders and decrees,
court of August 10, 1922, appointing Lazarus G. Joseph, Appellants further maintain that the court erred in holding and to issue such process as may be necessary and
receiver, should be, as it hereby is, set aside. Let the bond that the appointment of the receiver was in excess of its proper in the premises to enforce the provisions of this
given by said receiver to secure the faithful performance jurisdiction. This contention is also untenable. As soon as Act, with a right of appeal from the final order or decree of
of his duties be cancelled, and J.M. Menzi is held to be Behn, Meyer & Co., Ltd., was an "enemy not holding a such court as provided in sections one hundred and
under no obligation to deliver to the aforesaid Lazarus G. license granted by the President of the United States," it twenty-eight and two hundred and thirty-eight of the Act of
Joseph, the books under said Menzi's charge which became the duty of the Alien Property Custodian to take March third, nineteen hundred and eleven, entitled "An
formerly belonged to the plaintiff Behn, Meyer & Co., Ltd. possession of its business and all its assets within United Act to codify, revise, and amend the laws relating to the
States territory, and we must presume that this duty was judiciary."
No exception was taken to this order neither by the duly performed and that all such assets are now either
receiver nor by Jureidini Bros., but on October 1, 1923, actually or constructively in the possession of the Alien The only jurisdiction given to the Courts of First Instance
their counsel filed the following motion for reconsideration: Property Custodian and under his control. If so, they are of the Philippine Islands is in regard to criminal offenses
beyond the jurisdiction and control of the Philippine under said Act, as shown by section 18 thereof. Had it
Come now the Receiver and A.N. Jureidini & Bros. in the Courts. Section 7 of the Trading with Enemy Act as been the intention of Congress to give the Philippine
above entitled case and move this court that the court amended provides as follows: courts jurisdiction over civil litigation in regard to property
reconsider the resolution of this court dated September under the control of the Alien Property Custodian, the Act
26, 1923, and, thereafter order the delivery of the books to "The sole relief and remedy of any person having any would, of course, have so stated.
the said receiver. claim to any money or other property heretofore or
hereafter conveyed, transferred, assigned, delivered, or The orders appealed from are affirmed, with the costs
On December 3, 1923, the motion for reconsideration was paid over to the Alien Property Custodian, or required so against the appellants. So ordered.
denied, exception duly taken and the case is now before to be, or seized by him shall be that provided by the terms
us upon appeal from the two orders last mentioned. of this Act, and in the event of sale or other disposition of
such property by the Alien Property Custodian, shall be
The appellants contend that the court below erred in limited to and enforced against the net proceeds received
permitting the appellees to intervene inasmuch as (a) a therefrom and held by the Alien Property Custodian or by
final judgment had been entered in the case and (b) the the Treasurer of the United States."lawphi1.net
appellees had no legal interest in the matter in litigation.
Neither of these points is, in our opinion, well taken. The Section 9 of the Act provides that anyone "not an enemy
appellees intervene only in the receivership proceedings or ally of enemy claiming any interest, right, or title in any
152
Sales – Chapter 3-12 Cases
VIRGILIO S. DAVID, Petitioner, customs duties, and incidental expenses were for the account unenforceable under the Statute of Frauds. MOELCI argued
vs. of the buyer. that the quotation letter could not be considered a binding
MISAMIS OCCIDENTAL II ELECTRIC COOPERATIVE, contract because there was nothing in the said document
INC., Respondent. The Board Resolution, on the other hand, stated that the from which consent, on its part, to the terms and conditions
purchase of the said transformer was to be financed through proposed by David could be inferred. David knew that
Before this Court is a petition for review under Rule 45 of the a loan from the National Electrification Administration (NEA). MOELCI’s assent could only be obtained upon the issuance
Rules of Court assailing the July 8, 2010 Decision1 of the As there was no immediate action on the loan application, of a purchase order in favor of the bidder chosen by the
Court of Appeals (CA), in CA-G.R. CR No. 91839, which Engr. Rada returned to Manila in early December 1992 and Canvass and Awards Committee.
affirmed the July 17, 2008 Decision2 of the Regional Trial requested David to deliver the transformer to them even
Court, Branch VIII, Manila (RTC) in Civil Case No. 94-69402, without the required downpayment. David granted the request Eventually, pursuant to Rule 16, Section 5 of the Rules of
an action for specific performance and damages. provided that MOELCI would pay interest at 24% per annum. Court, MOELCI filed its Motion for Preliminary Hearing of
Engr. Rada acquiesced to the condition. On December 17, Affirmative Defenses and Deferment of the Pre-Trial
The Facts: 1992, the goods were shipped to Ozamiz City via William Conference which was denied by the RTC to abbreviate
Lines. In the Bill of Lading, a sales invoice was included proceedings and for the parties to proceed to trial and avoid
Petitioner Virgilio S. David (David) was the owner or which stated the agreed interest rate of 24% per annum. piecemeal resolution of issues. The order denying its motion
proprietor of VSD Electric Sales, a company engaged in the was raised with the CA, and then with this Court. Both courts
business of supplying electrical hardware including When nothing was heard from MOELCI for sometime after sustained the RTC ruling.
transformers for rural electric cooperatives like respondent the shipment, Emanuel Medina (Medina), David’s Marketing
Misamis Occidental II Electric Cooperative, Inc. (MOELCI), Manager, went to Ozamiz City to check on the shipment. Trial ensued. By reason of MOELCI’s continued failure to
with principal office located in Ozamis City. Medina was able to confer with Engr. Rada who told him that appear despite notice, David was allowed to present his
the loan was not yet released and asked if it was possible to testimonial and documentary evidence ex parte, pursuant to
To solve its problem of power shortage affecting some areas withdraw the shipped items. Medina agreed. Rule 18, Section 5 of the Rules. A Very Urgent Motion to
within its coverage, MOELCI expressed its intention to Allow Defendant to Present Evidence was filed by MOELCI,
purchase a 10 MVA power transformer from David. For this When no payment was made after several months, Medina but was denied.
reason, its General Manager, Engr. Reynaldo Rada (Engr. was constrained to send a demand letter, dated September
Rada), went to meet David in the latter’s office in Quezon 15, 1993, which MOELCI duly received. Engr. Rada replied in In its July 17, 2008 Decision, the RTC dismissed the
City. David agreed to supply the power transformer provided writing that the goods were still in the warehouse of William complaint. It found that although a contract of sale was
that MOELCI would secure a board resolution because the Lines again reiterating that the loan had not been approved perfected, it was not consummated because David failed to
item would still have to be imported. by NEA. This prompted Medina to head back to Ozamiz City prove that there was indeed a delivery of the subject item and
where he found out that the goods had already been released that MOELCI received it.3
On June 8, 1992, Engr. Rada and Director Jose Jimenez to MOELCI evidenced by the shipping company’s copy of the
(Jimenez), who was in-charge of procurement, returned to Bill of Lading which was stamped "Released," and with the Aggrieved, David appealed his case to the CA.
Manila and presented to David the requested board notation that the arrastre charges in the amount of P5,095.60
resolution which authorized the purchase of one 10 MVA had been paid. This was supported by a receipt of payment On July 8, 2010, the CA affirmed the ruling of the RTC. In the
power transformer. In turn, David presented his proposal for with the corresponding cargo delivery receipt issued by the assailed decision, the CA reasoned out that although David
the acquisition of said transformer. This proposal was the Integrated Port Services of Ozamiz, Inc. was correct in saying that MOELCI was deemed to have
same proposal that he would usually give to his clients. admitted the genuineness and due execution of the
Subsequently, demand letters were sent to MOELCI "quotation letter" (Exhibit A), wherein the signatures of the
After the reading of the proposal and the discussion of terms, demanding the payment of the whole amount plus the Chairman and the General Manager of MOELCI appeared,
David instructed his then secretary and bookkeeper, Ellen M. balance of previous purchases of other electrical hardware. he failed to offer any textual support to his stand that it was a
Wong, to type the names of Engr. Rada and Jimenez at the Aside from the formal demand letters, David added that contract of sale instead of a mere price quotation agreed to
end of the proposal. Both signed the document under the several statements of accounts were regularly sent through by MOELCI representatives. On this score, the RTC erred in
word "conforme." The board resolution was thereafter the mails by the company and these were never disputed by stating that a contract of sale was perfected between the
attached to the proposal. MOELCI. parties despite the irregularities that tainted their transaction.
Further, the fact that MOELCI’s representatives agreed to the
As stated in the proposal, the subject transformer, together On February 17, 1994, David filed a complaint for specific terms embodied in the agreement would not preclude the
with the basic accessories, was valued at P5,200,000.00. It performance with damages with the RTC. In response, finding that said contract was at best a mere contract to sell.
was also stipulated therein that 50% of the purchase price MOECLI moved for its dismissal on the ground that there was
should be paid as downpayment and the remaining balance lack of cause of action as there was no contract of sale, to A motion for reconsideration was filed by David but it was
to be paid upon delivery. Freight handling, insurance, begin with, or in the alternative, the said contract was denied.4
153
Sales – Chapter 3-12 Cases
(1) When the conclusion is a finding grounded entirely on stipulated costs quoted should it ultimately ripen into one of
Hence, this petition. speculation, surmises and conjectures; sale. The conditions upon which that development may occur
may even be obvious from statements in the agreement itself,
Before this Court, David presents the following issues for (2) When the inference made is manifestly mistaken, absurd that go beyond just "captions." Thus, the appellant opens
consideration: or impossible; with, "WE are pleased to submit our quotation xxx." The
purported contract also ends with. "Thank you for giving us
I. (3) Where there is a grave abuse of discretion: the opportunity to quote on your requirements and we hope to
receive your order soon" apparently referring to a purchase
WHETHER OR NOT THERE WAS A PERFECTED (4) When the judgment is based on a misapprehension of order which MOELCI contends to be a formal requirement for
CONTRACT OF SALE. facts; the entire transaction.8

II. (5) When the findings of fact are conflicting; In other words, the CA was of the position that Exhibit A was
at best a contract to sell.
WHETHER OR NOT THERE WAS A DELIVERY THAT (6) When the Court of Appeals, in making its findings, went
CONSUMMATED THE CONTRACT. beyond the issues of the case and the same is contrary to the A perusal of the records persuades the Court to hold
admissions of both appellant and appellee; otherwise.
The Court finds merit in the petition.
(7) When the findings are contrary to those of the trial court; The elements of a contract of sale are, to wit: a) Consent or
I. meeting of the minds, that is, consent to transfer ownership in
(8) When the findings of fact are without citation of specific exchange for the price; b) Determinate subject matter; and c)
On the issue as to whether or not there was a perfected evidence on which the conclusions are based; Price certain in money or its equivalent.9 It is the absence of
contract of sale, this Court is required to delve into the the first element which distinguishes a contract of sale from
evidence of the case. In a petition for review on certiorari (9) When the facts set forth in the petition as well as in the that of a contract to sell.
under Rule 45 of the Rules of Court, the issues to be petitioner’s main and reply briefs are not disputed by the
threshed out are generally questions of law only, and not of respondents; and In a contract to sell, the prospective seller explicitly reserves
fact. the transfer of title to the prospective buyer, meaning, the
(10) When the findings of fact of the Court of Appeals are prospective seller does not as yet agree or consent to
This was reiterated in the case of Buenaventura v. Pascual,5 premised on the supposed absence of evidence and transfer ownership of the property subject of the contract to
where it was written: contradicted by the evidence on record. 6 [Emphasis sell until the happening of an event, such as, in most cases,
supplied] the full payment of the purchase price. What the seller agrees
Time and again, this Court has stressed that its jurisdiction in or obliges himself to do is to fulfill his promise to sell the
a petition for review on certiorari under Rule 45 of the Rules In this case, the CA and the RTC reached different subject property when the entire amount of the purchase
of Court is limited to reviewing only errors of law, not of fact, conclusions on the question of whether or not there was a price is delivered to him. In other words, the full payment of
unless the findings of fact complained of are devoid of perfected contract of sale. The RTC ruled that a contract of the purchase price partakes of a suspensive condition, the
support by the evidence on record, or the assailed judgment sale was perfected although the same was not consummated non-fulfillment of which prevents the obligation to sell from
is based on the misapprehension of facts. The trial court, because David failed to show proof of delivery.7 arising and, thus, ownership is retained by the prospective
having heard the witnesses and observed their demeanor seller without further remedies by the prospective buyer.10
and manner of testifying, is in a better position to decide the The CA was of the opposite view. The CA wrote:
question of their credibility. Hence, the findings of the trial In a contract of sale, on the other hand, the title to the
court must be accorded the highest respect, even finality, by Be that as it may, it must be emphasized that the appellant property passes to the vendee upon the delivery of the thing
this Court. failed to offer any textual support to his insistence that Exhibit sold. Unlike in a contract to sell, the first element of consent is
"A" is a contract of sale instead of a mere price quotation present, although it is conditioned upon the happening of a
That being said, the Court is not unmindful, however, of the conformed to by MOELCI representatives. To that extent, the contingent event which may or may not occur. If the
recognized exceptions well-entrenched in jurisprudence. It trial court erred in laying down the premise that "indeed a suspensive condition is not fulfilled, the perfection of the
has always been stressed that when supported by substantial contract of sale is perfected between the parties despite the contract of sale is completely abated. However, if the
evidence, the findings of fact of the CA are conclusive and irregularities attending the transaction." x x x suspensive condition is fulfilled, the contract of sale is thereby
binding on the parties and are not reviewable by this Court, perfected, such that if there had already been previous
unless the case falls under any of the following recognized That representatives of MOELCI conformed to the terms delivery of the property subject of the sale to the buyer,
exceptions: embodied in the agreement does not preclude the finding that ownership thereto automatically transfers to the buyer by
such contract is, at best, a mere contract to sell with operation of law without any further act having to be
154
Sales – Chapter 3-12 Cases
performed by the seller. The vendor loses ownership over the thereby complying with the first element. Thus, the said to MOELCI. David was authorized to send the power
property and cannot recover it until and unless the contract is document cannot just be considered a contract to sell but transformer to the buyer pursuant to their agreement. When
resolved or rescinded.11 rather a perfected contract of sale. David sent the item through the carrier, it amounted to a
delivery to MOELCI.
An examination of the alleged contract to sell, "Exhibit A," II.
despite its unconventional form, would show that said Furthermore, in the case of Behn, Meyer & Co. (Ltd.) v.
document, with all the stipulations therein and with the Now, the next question is, was there a delivery? Yangco,14 it was pointed out that a specification in a contract
attendant circumstances surrounding it, was actually a relative to the payment of freight can be taken to indicate the
Contract of Sale. The rule is that it is not the title of the MOELCI, in denying that the power transformer was delivered intention of the parties with regard to the place of delivery. So
contract, but its express terms or stipulations that determine to it, argued that the Bill of Lading which David was relying that, if the buyer is to pay the freight, as in this case, it is
the kind of contract entered into by the parties.12 First, there upon was not conclusive. It argued that although the bill of reasonable to suppose that the subject of the sale is
was meeting of minds as to the transfer of ownership of the lading was stamped "Released," there was nothing in it that transferred to the buyer at the point of shipment. In other
subject matter. The letter (Exhibit A), though appearing to be indicated that said power transformer was indeed released to words, the title to the goods transfers to the buyer upon
a mere price quotation/proposal, was not what it seemed. It it or delivered to its possession. For this reason, it is its shipment or delivery to the carrier.
contained terms and conditions, so that, by the fact that position that it is not liable to pay the purchase price of the 10
Jimenez, Chairman of the Committee on Management, and MVA power transformer. Of course, Article 1523 provides a mere presumption and in
Engr. Rada, General Manager of MOELCI, had signed their order to overcome said presumption, MOELCI should have
names under the word "CONFORME," they, in effect, agreed This Court is unable to agree with the CA that there was no presented evidence to the contrary. The burden of proof was
with the terms and conditions with respect to the purchase of delivery of the items. On the contrary, there was delivery and shifted to MOELCI, who had to show that the rule under
the subject 10 MVA Power Transformer. As correctly argued release. Article 1523 was not applicable. In this regard, however,
by David, if their purpose was merely to acknowledge the MOELCI failed.
receipt of the proposal, they would not have signed their To begin with, among the terms and conditions of the
name under the word "CONFORME." proposal to which MOELCI agreed stated: There being delivery and release, said fact constitutes partial
performance which takes the case out of the protection of the
Besides, the uncontroverted attending circumstances bolster 2. Delivery – Ninety (90) working days upon receipt of your Statute of Frauds. It is elementary that the partial execution of
the fact that there was consent or meeting of minds in the purchase order and downpayment. a contract of sale takes the transaction out of the provisions
transfer of ownership. To begin with, a board resolution was of the Statute of Frauds so long as the essential requisites of
issued authorizing the purchase of the subject power C&F Manila, freight, handling, insurance, custom duties and consent of the contracting parties, object and cause of the
transformer. Next, armed with the said resolution, top officials incidental expenses shall be for the account of MOELCI II. 13 obligation concur and are clearly established to be present.15
of MOELCI visited David’s office in Quezon City three times (Emphasis supplied)
to discuss the terms of the purchase. Then, when the loan That being said, the Court now comes to David’s prayer that
that MOELCI was relying upon to finance the purchase was On this score, it is clear that MOELCI agreed that the power MOELCI be made to pay the total sum of ₱ 5,472,722.27 plus
not forthcoming, MOELCI, through Engr. Rada, convinced transformer would be delivered and that the freight, handling, the stipulated interest at 24% per annum from the filing of the
David to do away with the 50% downpayment and deliver the insurance, custom duties, and incidental expenses shall be complaint. Although the Court agrees that MOELCI should
unit so that it could already address its acute power shortage shouldered by it. pay interest, the stipulated rate is, however, unconscionable
predicament, to which David acceded when it made the and should be equitably reduced. While there is no question
delivery, through the carrier William On the basis of this express agreement, Article 1523 of the that parties to a loan agreement have wide latitude to
Civil Code becomes applicable.1âwphi1 It provides: stipulate on any interest rate in view of the Central Bank
Lines, as evidenced by a bill of lading. Circular No. 905 s. 1982 which suspended the Usury Law
Where, in pursuance of a contract of sale, the seller is ceiling on interest effective January 1, 1983, it is also worth
Second, the document specified a determinate subject matter authorized or required to send the goods to the buyer delivery stressing that interest rates whenever unconscionable may
which was one (1) Unit of 10 MVA Power Transformer with of the goods to a carrier, whether named by the buyer or not, still be reduced to a reasonable and fair level. There is
corresponding KV Line Accessories. And third, the document for the purpose of transmission to the buyer is deemed to be nothing in the said circular which grants lenders carte blanche
stated categorically the price certain in money which was a delivery of the goods to the buyer, except in the cases authority to raise interest rates to levels which will either
P5,200,000.00 for one (1) unit of 10 MVA Power Transformer provided for in Article 1503, first, second and third enslave their borrowers or lead to a hemorrhaging of their
and P2,169,500.00 for the KV Line Accessories. paragraphs, or unless a contrary intent appears. (Emphasis assets.16 Accordingly, the excessive interest of 24% per
supplied) annum stipulated in the sales invoice should be reduced to
In sum, since there was a meeting of the minds, there was 12% per annum.
consent on the part of David to transfer ownership of the Thus, the delivery made by David to William Lines, Inc., as
power transformer to MOELCI in exchange for the price, evidenced by the Bill of Lading, was deemed to be a delivery
155
Sales – Chapter 3-12 Cases
Indeed, David was compelled to file an action against
MOELCI but this reason alone will not warrant an award of
attorney’s fees. It is settled that the award of attorney's fees is
the exception rather than the rule. Counsel's fees are not
awarded every time a party prevails in a suit because of the
policy that no premium should be placed on the right to
litigate. Attorney's fees, as part of damages, are not
necessarily equated to the amount paid by a litigant to a
lawyer. In the ordinary sense, attorney's fees represent the
reasonable compensation paid to a lawyer by his client for the
legal services he has rendered to the latter; while in its
extraordinary concept, they may be awarded by the court as
indemnity for damages to be paid by the losing party to the
prevailing party. Attorney's fees as part of damages are
awarded only in the instances specified in Article 2208 of the
Civil Code 17 which demands factual, legal, and equitable
justification. Its basis cannot be left to speculation or
conjecture. In this regard, none was proven.

Moreover, in the absence of stipulation, a winning party may


be awarded attorney's fees only in case plaintiffs action or
defendant's stand is so untenable as to amount to gross and
evident bad faith.18 is MOELCI's case cannot be similarly
classified.

Also, David's claim for the balance of P73,059.76 plus the


stipulated interest is denied for being unsubstantiated.

WHEREFORE, the petition Is GRANTED. The July 8, 2010


Decision of the Court of Appeals Is REVERSED and SET
ASIDE. Respondent Misamis Occidental II Electric
Cooperative, Inc. is ordered to pay petitioner Virgilio S. David
the total sum of P5,472,722.27 with interest at the rate of
12o/o per annum reckoned from the filing of the complaint
until fully paid.

SO ORDERED.

156
Sales – Chapter 3-12 Cases
SPOUSES JOSE C. ROQUE AND BEATRIZ DELA well as the segregation and the concomitant issuance of a
CRUZ ROQUE, with deceased Jose C. Roque separate title over the subject portion in their names. After In defense, NCCP and Sabug, Jr. denied any knowledge
represented by his substitute heir JOVETTE ROQUE- the deed’s execution, Sps. Roque took possession and of the 1977 Deed of Conditional Sale through which the
LIBREA, Petitioners, introduced improvements on the subject portion which subject portion had been purportedly conveyed to Sps.
vs. they utilized as a balut factory.7 Roque.16
MA. PAMELA P. AGUADO, FRUCTUOSO C. SABUG,
JR., NATIONAL COUNCIL OF CHURCHES IN THE On August 12, 1991, Fructuoso Sabug, Jr. (Sabug, Jr.), For her part, Aguado raised the defense of an innocent
PHILIPPINES (NCCP), represented by its Secretary former Treasurer of the National Council of Churches in purchaser for value as she allegedly derived her title
General SHARON ROSE JOY RUIZ-DUREMDES, LAND the Philippines (NCCP), applied for a free patent over the (through the 1999 Deed of Absolute Sale) from Sabug, Jr.,
BANK OF THE PHILIPPINES (LBP), represented by entire Lot 18089 and was eventually issued Original the registered owner in OCT No. M-5955, covering Lot
Branch Manager EVELYN M. MONTERO, ATTY. Certificate of Title (OCT) No. M-59558 in his name on 18089, which certificate of title at the time of sale was free
MARIO S.P. DIAZ, in his Official Capacity as Register October 21, 1991. On June 24, 1993, Sabug, Jr. and from any lien and/or encumbrances. She also claimed that
of Deeds for Rizal, Morong Branch, and CECILIO U. Rivero, in her personal capacity and in representation of Sps. Roque’s cause of action had already prescribed
PULAN, in his Official Capacity as Sheriff, Office of Rivero, et al., executed a Joint Affidavit9 (1993 Joint because their adverse claim was made only on April 21,
the Clerk of Court, Regional Trial Court, Binangonan, Affidavit), acknowledging that the subject portion belongs 2003, or four (4) years from the date OCT No. M-5955
Rizal, Respondents. to Sps. Roque and expressed their willingness to was issued in Sabug, Jr.’s name on December 17,
segregate the same from the entire area of Lot 18089. 1999.17
Assailed in this petition for review on certiorari1 are the
Decision2 dated May 12, 2010 and the Resolution3 dated On December 8, 1999, however, Sabug, Jr., through a On the other hand, Land Bank averred that it had no
September 15, 2010 of the Court of Appeals (CA) in CA Deed of Absolute Sale10 (1999 Deed of Absolute Sale), knowledge of Sps. Roque’s claim relative to the subject
G.R. CV No. 92113 which affirmed the Decision4 dated sold Lot 18089 to one Ma. Pamela P. Aguado (Aguado) portion, considering that at the time the loan was taken
July 8, 2008 of the Regional Trial Court of Binangonan, for ₱2,500,000.00, who, in turn, caused the cancellation of out, Lot 18089 in its entirety was registered in Aguado’s
Rizal, Branch 69 (RTC) that dismissed Civil Case Nos. 03- OCT No. M-5955 and the issuance of Transfer Certificate name and no lien and/or encumbrance was annotated on
022 and 05-003 for reconveyance, annulment of sale, of Title (TCT) No. M-96692 dated December 17, 199911 her certificate of title.18
deed of real estate mortgage, foreclosure and certificate in her name.
of sale, and damages. Meanwhile, on January 18, 2005, NCCP filed a separate
Thereafter, Aguado obtained an ₱8,000,000.00 loan from complaint19 also for declaration of nullity of documents
The Facts the Land Bank of the Philippines (Land Bank) secured by and certificates of title and damages, docketed as Civil
a mortgage over Lot 18089.12 When she failed to pay her Case No. 05-003. It claimed to be the real owner of Lot
The property subject of this case is a parcel of land with loan obligation, Land Bank commenced extra-judicial 18089 which it supposedly acquired from Sabug, Jr.
an area of 20,862 square meters (sq. m.), located in Sitio foreclosure proceedings and eventually tendered the through an oral contract of sale20 in the early part of
Tagpos, Barangay Tayuman, Binangonan, Rizal, known highest bid in the auction sale. Upon Aguado’s failure to 1998, followed by the execution of a Deed of Absolute
as Lot 18089.5 redeem the subject property, Land Bank consolidated its Sale on December 2, 1998 (1998 Deed of Absolute
ownership, and TCT No. M-11589513 was issued in its Sale).21 NCCP also alleged that in October of the same
On July 21, 1977, petitioners-spouses Jose C. Roque and name on July 21, 2003.14 year, it entered into a Joint Venture Agreement (JVA) with
Beatriz dela Cruz Roque (Sps. Roque) and the original Pilipinas Norin Construction Development Corporation
owners of the then unregistered Lot 18089 – namely, On June 16, 2003, Sps. Roque filed a complaint15 for (PNCDC), a company owned by Aguado’s parents, for the
Velia R. Rivero (Rivero), Magdalena Aguilar, Angela reconveyance, annulment of sale, deed of real estate development of its real properties, including Lot 18089,
Gonzales, Herminia R. Bernardo, Antonio Rivero, Araceli mortgage, foreclosure, and certificate of sale, and into a subdivision project, and as such, turned over its
R. Victa, Leonor R. Topacio, and Augusto Rivero (Rivero, damages before the RTC, docketed as Civil Case No. 03- copy of OCT No. M-5955 to PNCDC.22 Upon knowledge
et al.) – executed a Deed of Conditional Sale of Real 022, against Aguado, Sabug, Jr., NCCP, Land Bank, the of the purported sale of Lot 18089 to Aguado, Sabug, Jr.
Property6 (1977 Deed of Conditional Sale) over a 1,231- Register of Deeds of Morong, Rizal, and Sheriff Cecilio U. denied the transaction and alleged forgery. Claiming that
sq. m. portion of Lot 18089 (subject portion) for a Pulan, seeking to be declared as the true owners of the the Aguados23 and PNCDC conspired to defraud NCCP,
consideration of ₱30,775.00. The parties agreed that Sps. subject portion which had been erroneously included in it prayed that PNCDC’s corporate veil be pierced and that
Roque shall make an initial payment of ₱15,387.50 upon the sale between Aguado and Sabug, Jr., and, the Aguados be ordered to pay the amount of
signing, while the remaining balance of the purchase price subsequently, the mortgage to Land Bank, both covering ₱38,092,002.00 representing the unrealized profit from
shall be payable upon the registration of Lot 18089, as Lot 18089 in its entirety. the JVA.24 Moreover, NCCP averred that Land Bank
157
Sales – Chapter 3-12 Cases
failed to exercise the diligence required to ascertain the On the other hand, regarding NCCP’s complaint, the RTC Resolution40 dated September 15, 2010, prompting them
true owners of Lot 18089. Hence, it further prayed that: (a) observed that while it anchored its claim of ownership to seek further recourse before the Court.
all acts of ownership and dominion over Lot 18089 that over Lot 18089 on the 1998 Deed of Absolute Sale, the
the bank might have done or caused to be done be said deed was not annotated on OCT No. M-5955. Neither The Issue Before the Court
declared null and void; (b) it be declared the true and real was any certificate of title issued in its name nor did it take
owners of Lot 18089; and (c) the Register of Deeds of possession of Lot 18089 or paid the real property taxes The central issue in this case is whether or not the CA
Morong, Rizal be ordered to cancel any and all certificates therefor. Hence, NCCP’s claim cannot prevail against erred in not ordering the reconveyance of the subject
of title covering the lot, and a new one be issued in its Land Bank’s title, which was adjudged by the RTC as an portion in Sps. Roque’s favor.
name.25 In its answer, Land Bank reiterated its stance innocent purchaser for value. Also, the RTC disregarded
that Lot 18089 was used as collateral for the NCCP’s allegation that the signature of Sabug, Jr. on the Sps. Roque maintain that the CA erred in not declaring
₱8,000,000.00 loan obtained by the Countryside Rural 1999 Deed of Absolute Sale in favor of Aguado was them as the lawful owners of the subject portion despite
Bank, Aguado, and one Bella Palasaga. There being no forged because his signatures on both instruments bear having possessed the same since the execution of the
lien and/ or encumbrance annotated on its certificate of semblances of similarity and appear genuine. Besides, the 1977 Deed of Conditional Sale, sufficient for acquisitive
title, i.e., TCT No. M-115895, it cannot be held liable for examiner from the National Bureau of Investigation, who prescription to set in in their favor.41 To bolster their
NCCP’s claims. Thus, it prayed for the dismissal of purportedly found that Sabug, Jr.’s signature thereon was claim, they also point to the 1993 Joint Affidavit whereby
NCCP’s complaint.26 spurious leading to the dismissal of a criminal case Sabug, Jr. and Rivero acknowledged their ownership
against him, was not presented as a witness in the civil thereof.42 Being the first purchasers and in actual
On September 7, 2005, Civil Case Nos. 02-022 and 05- action.32 possession of the disputed portion, they assert that they
003 were ordered consolidated.27 have a better right over the 1,231- sq. m. portion of Lot
Finally, the RTC denied the parties’ respective claims for 18089 and, hence, cannot be ousted therefrom by Land
The RTC Ruling damages.33 Bank, which was adjudged as a ortgagee/purchaser in
bad faith, pursuant to Article 1544 of the Civil Code.43
After due proceedings, the RTC rendered a Decision28 The CA Ruling
dated July 8, 2008, dismissing the complaints of Sps. In opposition, Land Bank espouses that the instant
Roque and NCCP. On appeal, the Court of Appeals (CA) affirmed the petition should be dismissed for raising questions of fact,
foregoing RTC findings in a Decision34 dated May 12, in violation of the proscription under Rule 45 of the Rules
With respect to Sps. Roque’s complaint, the RTC found 2010. While Land Bank was not regarded as a of Court which allows only pure questions of law to be
that the latter failed to establish their ownership over the mortgagee/purchaser in good faith with respect to the raised.44 Moreover, it denied that ownership over the
subject portion, considering the following: (a) the subject portion considering Sps. Roque’s possession subject portion had been acquired by Sps. Roque who
supposed owners-vendors, i.e., Rivero, et al., who thereof,35 the CA did not order its reconveyance or admittedly failed to pay the remaining balance of the
executed the 1977 Deed of Conditional Sale, had no proof segregation in the latter’s favor because of Sps. Roque’s purchase price.45 Besides, Land Bank points out that
of their title over Lot 18089; (b) the 1977 Deed of failure to pay the remaining balance of the purchase price. Sps. Roque’s action for reconveyance had already
Conditional Sale was not registered with the Office of the Hence, it only directed Land Bank to respect Sps. Roque’s prescribed.46
Register of Deeds;29 (c) the 1977 Deed of Conditional possession with the option to appropriate the
Sale is neither a deed of conveyance nor a transfer improvements introduced thereon upon payment of Instead of traversing the arguments of Sps. Roque,
document, as it only gives the holder the right to compel compensation.36 NCCP, in its Comment47 dated December 19, 2011,
the supposed vendors to execute a deed of absolute sale advanced its own case, arguing that the CA erred in
upon full payment of the consideration; (d) neither Sps. As regards NCCP, the CA found that it failed to establish holding that it failed to establish its claimed ownership
Roque nor the alleged owners-vendors, i.e., Rivero, et al., its right over Lot 18089 for the following reasons: (a) the over Lot 18089 in its entirety. Incidentally, NCCP’s appeal
have paid real property taxes in relation to Lot 18089; and sale to it of the lot by Sabug, Jr. was never registered; and from the CA Decision dated May 12, 2010 was already
(e) Sps. Roque’s occupation of the subject portion did not (b) there is no showing that it was in possession of Lot denied by the Court,48 and hence, will no longer be dealt
ripen into ownership that can be considered superior to 18089 or any portion thereof from 1998. Thus, as far as with in this case.
the ownership of Land Bank.30 Moreover, the RTC ruled NCCP is concerned, Land Bank is a mortgagee/purchaser
that Sps. Roque’s action for reconveyance had already in good faith.37 The Court’s Ruling
prescribed, having been filed ten (10) years after the
issuance of OCT No. M-5955.31 Aggrieved, both Sps. Roque38 and NCCP39 moved for The petition lacks merit.
reconsideration but were denied by the CA in a
158
Sales – Chapter 3-12 Cases
The essence of an action for reconveyance is to seek the sell and not one of sale contrary to Sps. Roque’s belief.52 portion despite their knowledge of the fact that, as early as
transfer of the property which was wrongfully or In this relation, it has been consistently ruled that where 1993, the entire Lot 18089 was registered in Sabug, Jr.’s
erroneously registered in another person’s name to its the seller promises to execute a deed of absolute sale name under OCT No. M-5955; and (c) while they signified
rightful owner or to one with a better right.49 Thus, it is upon the completion by the buyer of the payment of the their willingness to pay the balance of the purchase
incumbent upon the aggrieved party to show that he has a purchase price, the contract is only a contract to sell even price,59 Sps. Roque neither compelled Rivero et al.,
legal claim on the property superior to that of the if their agreement is denominated as a Deed of and/or Sabug, Jr. to accept the same nor did they consign
registered owner and that the property has not yet passed Conditional Sale,53 as in this case. This treatment stems any amount to the court, the proper application of which
to the hands of an innocent purchaser for value.50 from the legal characterization of a contract to sell, that is, would have effectively fulfilled their obligation to pay the
a bilateral contract whereby the prospective seller, while purchase price.60 Instead, Sps. Roque waited 26 years,
Sps. Roque claim that the subject portion covered by the expressly reserving the ownership of the subject property reckoned from the execution of the 1977 Deed of
1977 Deed of Conditional Sale between them and Rivero, despite delivery thereof to the prospective buyer, binds Conditional Sale, to institute an action for reconveyance
et al. was wrongfully included in the certificates of title himself to sell the subject property exclusively to the (in 2003), and only after Lot 18089 was sold to Land Bank
covering Lot 18089, and, hence, must be segregated prospective buyer upon fulfillment of the condition agreed in the foreclosure sale and title thereto was consolidated
therefrom and their ownership thereof be confirmed. The upon, such as, the full payment of the purchase price.54 in its name. Thus, in view of the foregoing, Sabug, Jr. – as
salient portions of the said deed state: Elsewise stated, in a contract to sell, ownership is retained the registered owner of Lot 18089 borne by the grant of
by the vendor and is not to pass to the vendee until full his free patent application – could validly convey said
DEED OF CONDITIONAL SALE OF REAL PROPERTY payment of the purchase price.55 Explaining the subject property in its entirety to Aguado who, in turn, mortgaged
matter further, the Court, in Ursal v. CA,56 held that: the same to Land Bank. Besides, as aptly observed by the
KNOW ALL MEN BY THESE PRESENTS: RTC, Sps. Roque failed to establish that the parties who
[I]n contracts to sell the obligation of the seller to sell sold the property to them, i.e., Rivero, et al., were indeed
xxxx becomes demandable only upon the happening of the its true and lawful owners.61 In fine, Sps. Roque failed to
suspensive condition, that is, the full payment of the establish any superior right over the subject portion as
That for and in consideration of the sum of THIRTY purchase price by the buyer. It is only upon the existence against the registered owner of Lot 18089, i.e., Land
THOUSAND SEVEN HUNDRED SEVENTY FIVE PESOS of the contract of sale that the seller becomes obligated to Bank, thereby warranting the dismissal of their
(₱30,775.00), Philippine Currency, payable in the manner transfer the ownership of the thing sold to the buyer. Prior reconveyance action, without prejudice to their right to
hereinbelow specified, the VENDORS do hereby sell, to the existence of the contract of sale, the seller is not seek damages against the vendors, i.e., Rivero et al.62 As
transfer and convey unto the VENDEE, or their heirs, obligated to transfer the ownership to the buyer, even if applied in the case of Coronel v. CA:63
executors, administrators, or assignors, that unsegregated there is a contract to sell between them.
portion of the above lot, x x x. It is essential to distinguish between a contract to sell and
Here, it is undisputed that Sps. Roque have not paid the a conditional contract of sale specially in cases where the
That the aforesaid amount shall be paid in two final installment of the purchase price.57 As such, the subject property is sold by the owner not to the party the
installments, the first installment which is in the amount of condition which would have triggered the parties’ seller contracted with, but to a third person, as in the case
__________ (₱15,387.50) and the balance in the amount obligation to enter into and thereby perfect a contract of at bench. In a contract to sell, there being no previous
of __________ (₱15,387.50), shall be paid as soon as the sale in order to effectively transfer the ownership of the sale of the property, a third person buying such property
described portion of the property shall have been subject portion from the sellers (i.e., Rivero et al.) to the despite the fulfilment of the suspensive condition such as
registered under the Land Registration Act and a buyers (Sps. Roque) cannot be deemed to have been the full payment of the purchase price, for instance,
Certificate of Title issued accordingly; fulfilled. Consequently, the latter cannot validly claim cannot be deemed a buyer in bad faith and the
ownership over the subject portion even if they had made prospective buyer cannot seek the relief of reconveyance
That as soon as the total amount of the property has been an initial payment and even took possession of the of the property.
paid and the Certificate of Title has been issued, an same.58
absolute deed of sale shall be executed accordingly; There is no double sale in such case.1âwphi1 Title to the
The Court further notes that Sps. Roque did not even take property will transfer to the buyer after registration
x x x x51 any active steps to protect their claim over the disputed because there is no defect in the owner-seller’s title per
portion. This remains evident from the following se, but the latter, of course, may be sued for damages by
Examining its provisions, the Court finds that the circumstances appearing on record: (a) the 1977 Deed of the intending buyer. (Emphasis supplied)
stipulation above-highlighted shows that the 1977 Deed of Conditional Sale was never registered; (b) they did not
Conditional Sale is actually in the nature of a contract to seek the actual/physical segregation of the disputed
159
Sales – Chapter 3-12 Cases
On the matter of double sales, suffice it to state that Sps.
Roque’s reliance64 on Article 154465 of the Civil Code
has been misplaced since the contract they base their
claim of ownership on is, as earlier stated, a contract to
sell, and not one of sale. In Cheng v. Genato,66 the Court
stated the circumstances which must concur in order to
determine the applicability of Article 1544, none of which
are obtaining in this case, viz.:

(a) The two (or more) sales transactions in issue must


pertain to exactly the same subject matter, and must be
valid sales transactions;

(b) The two (or more) buyers at odds over the rightful
ownership of the subject matter must each represent
conflicting interests; and

(c) The two (or more) buyers at odds over the rightful
ownership of the subject matter must each have bought
from the same seller.

Finally, regarding Sps. Roque’s claims of acquisitive


prescription and reimbursement for the value of the
improvements they have introduced on the subject
property,67 it is keenly observed that none of the
arguments therefor were raised before the trial court or
the CA.68 Accordingly, the Court applies the well-settled
rule that litigants cannot raise an issue for the first time on
appeal as this would contravene the basic rules of fair
play and justice. In any event, such claims appear to
involve questions of fact which are generally prohibited
under a Rule 45 petition.69

With the conclusions herein reached, the Court need not


belabor on the other points raised by the parties, and
ultimately finds it proper to proceed with the denial of the
petition.

WHEREFORE, the petition is DENIED. The Decision


dated May 12, 2010 and the Resolution dated September
15, 2010 of the Court of Appeals in CAG.R. CV No. 92113
are hereby AFFIRMED.

SO ORDERED.

160
Sales – Chapter 3-12 Cases
ROMULO A. CORONEL, ALARICO A. CORONEL, certificate of title immediately upon receipt of the down payment paid by Concepcion in the bank in trust for
ANNETTE A. CORONEL, ANNABELLE C. GONZALES payment above-stated. Ramona Patricia Alcaraz.
(for herself and on behalf of Florida C. Tupper, as
attorney-in-fact), CIELITO A. CORONEL, FLORAIDA A. On our presentation of the TCT already in or name, We On February 22, 1985, Concepcion, et al., filed a
ALMONTE, and CATALINA BALAIS MABANAG, will immediately execute the deed of absolute sale of said complaint for specific performance against the Coronels
petitioners, property and Miss Ramona Patricia Alcaraz shall and caused the annotation of a notice of lis pendens at
vs. immediately pay the balance of the P1,190,000.00. the back of TCT No. 327403 (Exh. "E"; Exh. "5").
THE COURT OF APPEALS, CONCEPCION D.
ALCARAZ, and RAMONA PATRICIA ALCARAZ, Clearly, the conditions appurtenant to the sale are the On April 2, 1985, Catalina caused the annotation of a
assisted by GLORIA F. NOEL as attorney-in-fact, following: notice of adverse claim covering the same property with
respondents. the Registry of Deeds of Quezon City (Exh. "F"; Exh. "6").
1. Ramona will make a down payment of Fifty
The petition before us has its roots in a complaint for Thousand (P50,000.00) Pesos upon execution of the On April 25, 1985, the Coronels executed a Deed of
specific performance to compel herein petitioners (except document aforestated; Absolute Sale over the subject property in favor of
the last named, Catalina Balais Mabanag) to consummate Catalina (Exh. "G"; Exh. "7").
the sale of a parcel of land with its improvements located 2. The Coronels will cause the transfer in their
along Roosevelt Avenue in Quezon City entered into by names of the title of the property registered in the name of On June 5, 1985, a new title over the subject property was
the parties sometime in January 1985 for the price of their deceased father upon receipt of the Fifty Thousand issued in the name of Catalina under TCT No. 351582
P1,240,000.00. (P50,000.00) Pesos down payment; (Exh. "H"; Exh. "8").

The undisputed facts of the case were summarized by 3. Upon the transfer in their names of the subject (Rollo, pp. 134-136)
respondent court in this wise: property, the Coronels will execute the deed of absolute
sale in favor of Ramona and the latter will pay the former In the course of the proceedings before the trial court
On January 19, 1985, defendants-appellants Romulo the whole balance of One Million One Hundred Ninety (Branch 83, RTC, Quezon City) the parties agreed to
Coronel, et al. (hereinafter referred to as Coronels) Thousand (P1,190,000.00) Pesos. submit the case for decision solely on the basis of
executed a document entitled "Receipt of Down Payment" documentary exhibits. Thus, plaintiffs therein (now private
(Exh. "A") in favor of plaintiff Ramona Patricia Alcaraz On the same date (January 15, 1985), plaintiff-appellee respondents) proffered their documentary evidence
(hereinafter referred to as Ramona) which is reproduced Concepcion D. Alcaraz (hereinafter referred to as accordingly marked as Exhibits "A" through "J", inclusive
hereunder: Concepcion), mother of Ramona, paid the down payment of their corresponding submarkings. Adopting these same
of Fifty Thousand (P50,000.00) Pesos (Exh. "B", Exh. "2"). exhibits as their own, then defendants (now petitioners)
RECEIPT OF DOWN PAYMENT accordingly offered and marked them as Exhibits "1"
On February 6, 1985, the property originally registered in through "10", likewise inclusive of their corresponding
P1,240,000.00 — Total amount the name of the Coronels' father was transferred in their submarkings. Upon motion of the parties, the trial court
names under TCT gave them thirty (30) days within which to simultaneously
50,000 — Down payment No. 327043 (Exh. "D"; Exh. "4") submit their respective memoranda, and an additional 15
——————————— days within which to submit their corresponding comment
P1,190,000.00 — Balance On February 18, 1985, the Coronels sold the property or reply thereof, after which, the case would be deemed
covered by TCT No. 327043 to intervenor-appellant submitted for resolution.
Received from Miss Ramona Patricia Alcaraz of 146 Catalina B. Mabanag (hereinafter referred to as Catalina)
Timog, Quezon City, the sum of Fifty Thousand Pesos for One Million Five Hundred Eighty Thousand On April 14, 1988, the case was submitted for resolution
purchase price of our inherited house and lot, covered by (P1,580,000.00) Pesos after the latter has paid Three before Judge Reynaldo Roura, who was then temporarily
TCT No. 119627 of the Registry of Deeds of Quezon City, Hundred Thousand (P300,000.00) Pesos (Exhs. "F-3"; detailed to preside over Branch 82 of the RTC of Quezon
in the total amount of P1,240,000.00. Exh. "6-C") City. On March 1, 1989, judgment was handed down by
Judge Roura from his regular bench at Macabebe,
We bind ourselves to effect the transfer in our names from For this reason, Coronels canceled and rescinded the Pampanga for the Quezon City branch, disposing as
our deceased father, Constancio P. Coronel, the transfer contract (Exh. "A") with Ramona by depositing the down follows:

161
Sales – Chapter 3-12 Cases
WHEREFORE, judgment for specific performance is defendants and intervenor did not object to the authority
hereby rendered ordering defendant to execute in favor of of Judge Reynaldo Roura to decide the case prior to the Hence, the instant petition which was filed on March 5,
plaintiffs a deed of absolute sale covering that parcel of rendition of the decision, when they met for the first time 1992. The last pleading, private respondents' Reply
land embraced in and covered by Transfer Certificate of before the undersigned Presiding Judge at the hearing of Memorandum, was filed on September 15, 1993. The
Title No. 327403 (now TCT No. 331582) of the Registry of a pending incident in Civil Case No. Q-46145 on case was, however, re-raffled to undersigned ponente
Deeds for Quezon City, together with all the November 11, 1988, they were deemed to have only on August 28, 1996, due to the voluntary inhibition of
improvements existing thereon free from all liens and acquiesced thereto and they are now estopped from the Justice to whom the case was last assigned.
encumbrances, and once accomplished, to immediately questioning said authority of Judge Roura after they
deliver the said document of sale to plaintiffs and upon received the decision in question which happens to be While we deem it necessary to introduce certain
receipt thereof, the said document of sale to plaintiffs and adverse to them; (3) While it is true that Judge Reynaldo refinements in the disquisition of respondent court in the
upon receipt thereof, the plaintiffs are ordered to pay Roura was merely a Judge-on-detail at this Branch of the affirmance of the trial court's decision, we definitely find
defendants the whole balance of the purchase price Court, he was in all respects the Presiding Judge with full the instant petition bereft of merit.
amounting to P1,190,000.00 in cash. Transfer Certificate authority to act on any pending incident submitted before
of Title No. 331582 of the Registry of Deeds for Quezon this Court during his incumbency. When he returned to his The heart of the controversy which is the ultimate key in
City in the name of intervenor is hereby canceled and Official Station at Macabebe, Pampanga, he did not lose the resolution of the other issues in the case at bar is the
declared to be without force and effect. Defendants and his authority to decide or resolve such cases submitted to precise determination of the legal significance of the
intervenor and all other persons claiming under them are him for decision or resolution because he continued as document entitled "Receipt of Down Payment" which was
hereby ordered to vacate the subject property and deliver Judge of the Regional Trial Court and is of co-equal rank offered in evidence by both parties. There is no dispute as
possession thereof to plaintiffs. Plaintiffs' claim for with the undersigned Presiding Judge. The standing rule to the fact that said document embodied the binding
damages and attorney's fees, as well as the and supported by jurisprudence is that a Judge to whom a contract between Ramona Patricia Alcaraz on the one
counterclaims of defendants and intervenors are hereby case is submitted for decision has the authority to decide hand, and the heirs of Constancio P. Coronel on the
dismissed. the case notwithstanding his transfer to another branch or other, pertaining to a particular house and lot covered by
region of the same court (Sec. 9, Rule 135, Rule of Court). TCT No. 119627, as defined in Article 1305 of the Civil
No pronouncement as to costs. Code of the Philippines which reads as follows:
Coming now to the twin prayer for reconsideration of the
So Ordered. Decision dated March 1, 1989 rendered in the instant Art. 1305. A contract is a meeting of minds
case, resolution of which now pertains to the undersigned between two persons whereby one binds himself, with
Macabebe, Pampanga for Quezon City, March 1, 1989. Presiding Judge, after a meticulous examination of the respect to the other, to give something or to render some
documentary evidence presented by the parties, she is service.
(Rollo, p. 106) convinced that the Decision of March 1, 1989 is supported
by evidence and, therefore, should not be disturbed. While, it is the position of private respondents that the
A motion for reconsideration was filed by petitioner before "Receipt of Down Payment" embodied a perfected
the new presiding judge of the Quezon City RTC but the IN VIEW OF THE FOREGOING, the "Motion for contract of sale, which perforce, they seek to enforce by
same was denied by Judge Estrella T. Estrada, thusly: Reconsideration and/or to Annul Decision and Render means of an action for specific performance, petitioners
Anew Decision by the Incumbent Presiding Judge" dated on their part insist that what the document signified was a
The prayer contained in the instant motion, i.e., to annul March 20, 1989 is hereby DENIED. mere executory contract to sell, subject to certain
the decision and to render anew decision by the suspensive conditions, and because of the absence of
undersigned Presiding Judge should be denied for the SO ORDERED. Ramona P. Alcaraz, who left for the United States of
following reasons: (1) The instant case became submitted America, said contract could not possibly ripen into a
for decision as of April 14, 1988 when the parties Quezon City, Philippines, July 12, 1989. contract absolute sale.
terminated the presentation of their respective
documentary evidence and when the Presiding Judge at (Rollo, pp. 108-109) Plainly, such variance in the contending parties'
that time was Judge Reynaldo Roura. The fact that they contentions is brought about by the way each interprets
were allowed to file memoranda at some future date did Petitioners thereupon interposed an appeal, but on the terms and/or conditions set forth in said private
not change the fact that the hearing of the case was December 16, 1991, the Court of Appeals (Buena, instrument. Withal, based on whatever relevant and
terminated before Judge Roura and therefore the same Gonzaga-Reyes, Abad Santos (P), JJ.) rendered its admissible evidence may be available on record, this,
should be submitted to him for decision; (2) When the decision fully agreeing with the trial court. Court, as were the courts below, is now called upon to
162
Sales – Chapter 3-12 Cases
adjudge what the real intent of the parties was at the time prevented the obligation of the vendor to convey title from although the property may have been previously delivered
the said document was executed. acquiring binding force. to him. The prospective seller still has to convey title to the
prospective buyer by entering into a contract of absolute
The Civil Code defines a contract of sale, thus: Stated positively, upon the fulfillment of the suspensive sale.
condition which is the full payment of the purchase price,
Art. 1458. By the contract of sale one of the the prospective seller's obligation to sell the subject It is essential to distinguish between a contract to sell and
contracting parties obligates himself to transfer the property by entering into a contract of sale with the a conditional contract of sale specially in cases where the
ownership of and to deliver a determinate thing, and the prospective buyer becomes demandable as provided in subject property is sold by the owner not to the party the
other to pay therefor a price certain in money or its Article 1479 of the Civil Code which states: seller contracted with, but to a third person, as in the case
equivalent. at bench. In a contract to sell, there being no previous
Art. 1479. A promise to buy and sell a determinate sale of the property, a third person buying such property
Sale, by its very nature, is a consensual contract because thing for a price certain is reciprocally demandable. despite the fulfillment of the suspensive condition such as
it is perfected by mere consent. The essential elements of the full payment of the purchase price, for instance,
a contract of sale are the following: An accepted unilateral promise to buy or to sell a cannot be deemed a buyer in bad faith and the
determinate thing for a price certain is binding upon the prospective buyer cannot seek the relief of reconveyance
a) Consent or meeting of the minds, that is, consent promissor if the promise is supported by a consideration of the property. There is no double sale in such case. Title
to transfer ownership in exchange for the price; distinct from the price. to the property will transfer to the buyer after registration
because there is no defect in the owner-seller's title per
b) Determinate subject matter; and A contract to sell may thus be defined as a bilateral se, but the latter, of course, may be used for damages by
contract whereby the prospective seller, while expressly the intending buyer.
c) Price certain in money or its equivalent. reserving the ownership of the subject property despite
delivery thereof to the prospective buyer, binds himself to In a conditional contract of sale, however, upon the
Under this definition, a Contract to Sell may not be sell the said property exclusively to the prospective buyer fulfillment of the suspensive condition, the sale becomes
considered as a Contract of Sale because the first upon fulfillment of the condition agreed upon, that is, full absolute and this will definitely affect the seller's title
essential element is lacking. In a contract to sell, the payment of the purchase price. thereto. In fact, if there had been previous delivery of the
prospective seller explicity reserves the transfer of title to subject property, the seller's ownership or title to the
the prospective buyer, meaning, the prospective seller A contract to sell as defined hereinabove, may not even property is automatically transferred to the buyer such
does not as yet agree or consent to transfer ownership of be considered as a conditional contract of sale where the that, the seller will no longer have any title to transfer to
the property subject of the contract to sell until the seller may likewise reserve title to the property subject of any third person. Applying Article 1544 of the Civil Code,
happening of an event, which for present purposes we the sale until the fulfillment of a suspensive condition, such second buyer of the property who may have had
shall take as the full payment of the purchase price. What because in a conditional contract of sale, the first element actual or constructive knowledge of such defect in the
the seller agrees or obliges himself to do is to fulfill is of consent is present, although it is conditioned upon the seller's title, or at least was charged with the obligation to
promise to sell the subject property when the entire happening of a contingent event which may or may not discover such defect, cannot be a registrant in good faith.
amount of the purchase price is delivered to him. In other occur. If the suspensive condition is not fulfilled, the Such second buyer cannot defeat the first buyer's title. In
words the full payment of the purchase price partakes of a perfection of the contract of sale is completely abated (cf. case a title is issued to the second buyer, the first buyer
suspensive condition, the non-fulfillment of which prevents Homesite and housing Corp. vs. Court of Appeals, 133 may seek reconveyance of the property subject of the
the obligation to sell from arising and thus, ownership is SCRA 777 [1984]). However, if the suspensive condition sale.
retained by the prospective seller without further remedies is fulfilled, the contract of sale is thereby perfected, such
by the prospective buyer. In Roque vs. Lapuz (96 SCRA that if there had already been previous delivery of the With the above postulates as guidelines, we now proceed
741 [1980]), this Court had occasion to rule: property subject of the sale to the buyer, ownership to the task of deciphering the real nature of the contract
thereto automatically transfers to the buyer by operation of entered into by petitioners and private respondents.
Hence, We hold that the contract between the petitioner law without any further act having to be performed by the
and the respondent was a contract to sell where the seller. It is a canon in the interpretation of contracts that the
ownership or title is retained by the seller and is not to words used therein should be given their natural and
pass until the full payment of the price, such payment In a contract to sell, upon the fulfillment of the suspensive ordinary meaning unless a technical meaning was
being a positive suspensive condition and failure of which condition which is the full payment of the purchase price, intended (Tan vs. Court of Appeals, 212 SCRA 586
is not a breach, casual or serious, but simply an event that ownership will not automatically transfer to the buyer
163
Sales – Chapter 3-12 Cases
[1992]). Thus, when petitioners declared in the said of title changed to their names and immediately thereafter, The Court significantly notes this suspensive condition
"Receipt of Down Payment" that they — to execute the written deed of absolute sale. was, in fact, fulfilled on February 6, 1985 (Exh. "D"; Exh.
"4"). Thus, on said date, the conditional contract of sale
Received from Miss Ramona Patricia Alcaraz of 146 Thus, the parties did not merely enter into a contract to between petitioners and private respondent Ramona P.
Timog, Quezon City, the sum of Fifty Thousand Pesos sell where the sellers, after compliance by the buyer with Alcaraz became obligatory, the only act required for the
purchase price of our inherited house and lot, covered by certain terms and conditions, promised to sell the property consummation thereof being the delivery of the property
TCT No. 1199627 of the Registry of Deeds of Quezon to the latter. What may be perceived from the respective by means of the execution of the deed of absolute sale in
City, in the total amount of P1,240,000.00. undertakings of the parties to the contract is that a public instrument, which petitioners unequivocally
petitioners had already agreed to sell the house and lot committed themselves to do as evidenced by the "Receipt
without any reservation of title until full payment of the they inherited from their father, completely willing to of Down Payment."
entire purchase price, the natural and ordinary idea transfer full ownership of the subject house and lot to the
conveyed is that they sold their property. buyer if the documents were then in order. It just Article 1475, in correlation with Article 1181, both of the
happened, however, that the transfer certificate of title Civil Code, plainly applies to the case at bench. Thus,
When the "Receipt of Down Payment" is considered in its was then still in the name of their father. It was more
entirety, it becomes more manifest that there was a clear expedient to first effect the change in the certificate of title Art. 1475. The contract of sale is perfected at the
intent on the part of petitioners to transfer title to the so as to bear their names. That is why they undertook to moment there is a meeting of minds upon the thing which
buyer, but since the transfer certificate of title was still in cause the issuance of a new transfer of the certificate of is the object of the contract and upon the price.
the name of petitioner's father, they could not fully effect title in their names upon receipt of the down payment in
such transfer although the buyer was then willing and able the amount of P50,000.00. As soon as the new certificate From the moment, the parties may reciprocally demand
to immediately pay the purchase price. Therefore, of title is issued in their names, petitioners were performance, subject to the provisions of the law
petitioners-sellers undertook upon receipt of the down committed to immediately execute the deed of absolute governing the form of contracts.
payment from private respondent Ramona P. Alcaraz, to sale. Only then will the obligation of the buyer to pay the
cause the issuance of a new certificate of title in their remainder of the purchase price arise. Art. 1181. In conditional obligations, the
names from that of their father, after which, they promised acquisition of rights, as well as the extinguishment or loss
to present said title, now in their names, to the latter and There is no doubt that unlike in a contract to sell which is of those already acquired, shall depend upon the
to execute the deed of absolute sale whereupon, the latter most commonly entered into so as to protect the seller happening of the event which constitutes the condition.
shall, in turn, pay the entire balance of the purchase price. against a buyer who intends to buy the property in
installment by withholding ownership over the property Since the condition contemplated by the parties which is
The agreement could not have been a contract to sell until the buyer effects full payment therefor, in the contract the issuance of a certificate of title in petitioners' names
because the sellers herein made no express reservation entered into in the case at bar, the sellers were the one was fulfilled on February 6, 1985, the respective
of ownership or title to the subject parcel of land. who were unable to enter into a contract of absolute sale obligations of the parties under the contract of sale
Furthermore, the circumstance which prevented the by reason of the fact that the certificate of title to the became mutually demandable, that is, petitioners, as
parties from entering into an absolute contract of sale property was still in the name of their father. It was the sellers, were obliged to present the transfer certificate of
pertained to the sellers themselves (the certificate of title sellers in this case who, as it were, had the impediment title already in their names to private respondent Ramona
was not in their names) and not the full payment of the which prevented, so to speak, the execution of an contract P. Alcaraz, the buyer, and to immediately execute the
purchase price. Under the established facts and of absolute sale. deed of absolute sale, while the buyer on her part, was
circumstances of the case, the Court may safely presume obliged to forthwith pay the balance of the purchase price
that, had the certificate of title been in the names of What is clearly established by the plain language of the amounting to P1,190,000.00.
petitioners-sellers at that time, there would have been no subject document is that when the said "Receipt of Down
reason why an absolute contract of sale could not have Payment" was prepared and signed by petitioners Romeo It is also significant to note that in the first paragraph in
been executed and consummated right there and then. A. Coronel, et al., the parties had agreed to a conditional page 9 of their petition, petitioners conclusively admitted
contract of sale, consummation of which is subject only to that:
Moreover, unlike in a contract to sell, petitioners in the the successful transfer of the certificate of title from the
case at bar did not merely promise to sell the properly to name of petitioners' father, Constancio P. Coronel, to their 3. The petitioners-sellers Coronel bound
private respondent upon the fulfillment of the suspensive names. themselves "to effect the transfer in our names from our
condition. On the contrary, having already agreed to sell deceased father Constancio P. Coronel, the transfer
the subject property, they undertook to have the certificate certificate of title immediately upon receipt of the
164
Sales – Chapter 3-12 Cases
downpayment above-stated". The sale was still subject to In obligation to do or not to do, the courts shall determine, posture contrary to that which they took when they
this suspensive condition. (Emphasis supplied.) in each case, the retroactive effect of the condition that entered into the agreement with private respondent
has been complied with. Ramona P. Alcaraz. The Civil Code expressly states that:
(Rollo, p. 16)
the rights and obligations of the parties with respect to the Art. 1431. Through estoppel an admission or
Petitioners themselves recognized that they entered into a perfected contract of sale became mutually due and representation is rendered conclusive upon the person
contract of sale subject to a suspensive condition. Only, demandable as of the time of fulfillment or occurrence of making it, and cannot be denied or disproved as against
they contend, continuing in the same paragraph, that: the suspensive condition on February 6, 1985. As of that the person relying thereon.
point in time, reciprocal obligations of both seller and
. . . Had petitioners-sellers not complied with this condition buyer arose. Having represented themselves as the true owners of the
of first transferring the title to the property under their subject property at the time of sale, petitioners cannot
names, there could be no perfected contract of sale. Petitioners also argue there could been no perfected claim now that they were not yet the absolute owners
(Emphasis supplied.) contract on January 19, 1985 because they were then not thereof at that time.
yet the absolute owners of the inherited property.
(Ibid.) Petitioners also contend that although there was in fact a
We cannot sustain this argument. perfected contract of sale between them and Ramona P.
not aware that they set their own trap for themselves, for Alcaraz, the latter breached her reciprocal obligation when
Article 1186 of the Civil Code expressly provides that: Article 774 of the Civil Code defines Succession as a she rendered impossible the consummation thereof by
mode of transferring ownership as follows: going to the United States of America, without leaving her
Art. 1186. The condition shall be deemed fulfilled address, telephone number, and Special Power of
when the obligor voluntarily prevents its fulfillment. Art. 774. Succession is a mode of acquisition by virtue of Attorney (Paragraphs 14 and 15, Answer with Compulsory
which the property, rights and obligations to be extent and Counterclaim to the Amended Complaint, p. 2; Rollo, p.
Besides, it should be stressed and emphasized that what value of the inheritance of a person are transmitted 43), for which reason, so petitioners conclude, they were
is more controlling than these mere hypothetical through his death to another or others by his will or by correct in unilaterally rescinding rescinding the contract of
arguments is the fact that the condition herein referred to operation of law. sale.
was actually and indisputably fulfilled on February 6,
1985, when a new title was issued in the names of Petitioners-sellers in the case at bar being the sons and We do not agree with petitioners that there was a valid
petitioners as evidenced by TCT No. 327403 (Exh. "D"; daughters of the decedent Constancio P. Coronel are rescission of the contract of sale in the instant case. We
Exh. "4"). compulsory heirs who were called to succession by note that these supposed grounds for petitioners'
operation of law. Thus, at the point their father drew his rescission, are mere allegations found only in their
The inevitable conclusion is that on January 19, 1985, as last breath, petitioners stepped into his shoes insofar as responsive pleadings, which by express provision of the
evidenced by the document denominated as "Receipt of the subject property is concerned, such that any rights or rules, are deemed controverted even if no reply is filed by
Down Payment" (Exh. "A"; Exh. "1"), the parties entered obligations pertaining thereto became binding and the plaintiffs (Sec. 11, Rule 6, Revised Rules of Court).
into a contract of sale subject only to the suspensive enforceable upon them. It is expressly provided that rights The records are absolutely bereft of any supporting
condition that the sellers shall effect the issuance of new to the succession are transmitted from the moment of evidence to substantiate petitioners' allegations. We have
certificate title from that of their father's name to their death of the decedent (Article 777, Civil Code; Cuison vs. stressed time and again that allegations must be proven
names and that, on February 6, 1985, this condition was Villanueva, 90 Phil. 850 [1952]). by sufficient evidence (Ng Cho Cio vs. Ng Diong, 110 Phil.
fulfilled (Exh. "D"; Exh. "4"). 882 [1961]; Recaro vs. Embisan, 2 SCRA 598 [1961].
Be it also noted that petitioners' claim that succession Mere allegation is not an evidence (Lagasca vs. De Vera,
We, therefore, hold that, in accordance with Article 1187 may not be declared unless the creditors have been paid 79 Phil. 376 [1947]).
which pertinently provides — is rendered moot by the fact that they were able to effect
the transfer of the title to the property from the decedent's Even assuming arguendo that Ramona P. Alcaraz was in
Art. 1187. The effects of conditional obligation to name to their names on February 6, 1985. the United States of America on February 6, 1985, we
give, once the condition has been fulfilled, shall retroact to cannot justify petitioner-sellers' act of unilaterally and
the day of the constitution of the obligation . . . Aside from this, petitioners are precluded from raising extradicially rescinding the contract of sale, there being no
their supposed lack of capacity to enter into an agreement express stipulation authorizing the sellers to extarjudicially
at that time and they cannot be allowed to now take a rescind the contract of sale. (cf. Dignos vs. CA, 158 SCRA
165
Sales – Chapter 3-12 Cases
375 [1988]; Taguba vs. Vda. de Leon, 132 SCRA 722 In reciprocal obligations, neither party incurs in delay if the In his commentaries on the Civil Code, an accepted
[1984]) other does not comply or is not ready to comply in a authority on the subject, now a distinguished member of
proper manner with what is incumbent upon him. From the the Court, Justice Jose C. Vitug, explains:
Moreover, petitioners are estopped from raising the moment one of the parties fulfill his obligation, delay by
alleged absence of Ramona P. Alcaraz because although the other begins. (Emphasis supplied.) The governing principle is prius tempore, potior jure (first
the evidence on record shows that the sale was in the in time, stronger in right). Knowledge by the first buyer of
name of Ramona P. Alcaraz as the buyer, the sellers had There is thus neither factual nor legal basis to rescind the the second sale cannot defeat the first buyer's rights
been dealing with Concepcion D. Alcaraz, Ramona's contract of sale between petitioners and respondents. except when the second buyer first registers in good faith
mother, who had acted for and in behalf of her daughter, if the second sale (Olivares vs. Gonzales, 159 SCRA 33).
not also in her own behalf. Indeed, the down payment was With the foregoing conclusions, the sale to the other Conversely, knowledge gained by the second buyer of the
made by Concepcion D. Alcaraz with her own personal petitioner, Catalina B. Mabanag, gave rise to a case of first sale defeats his rights even if he is first to register,
check (Exh. "B"; Exh. "2") for and in behalf of Ramona P. double sale where Article 1544 of the Civil Code will since knowledge taints his registration with bad faith (see
Alcaraz. There is no evidence showing that petitioners apply, to wit: also Astorga vs. Court of Appeals, G.R. No. 58530, 26
ever questioned Concepcion's authority to represent December 1984). In Cruz vs. Cabana (G.R. No. 56232, 22
Ramona P. Alcaraz when they accepted her personal Art. 1544. If the same thing should have been sold June 1984, 129 SCRA 656), it has held that it is essential,
check. Neither did they raise any objection as regards to different vendees, the ownership shall be transferred to to merit the protection of Art. 1544, second paragraph,
payment being effected by a third person. Accordingly, as the person who may have first taken possession thereof in that the second realty buyer must act in good faith in
far as petitioners are concerned, the physical absence of good faith, if it should be movable property. registering his deed of sale (citing Carbonell vs. Court of
Ramona P. Alcaraz is not a ground to rescind the contract Appeals, 69 SCRA 99, Crisostomo vs. CA, G.R. No.
of sale. Should if be immovable property, the ownership shall 95843, 02 September 1992).
belong to the person acquiring it who in good faith first (J. Vitug Compendium of Civil Law and Jurisprudence,
Corollarily, Ramona P. Alcaraz cannot even be deemed to recorded it in Registry of Property. 1993 Edition, p. 604).
be in default, insofar as her obligation to pay the full
purchase price is concerned. Petitioners who are Should there be no inscription, the ownership shall pertain Petitioner point out that the notice of lis pendens in the
precluded from setting up the defense of the physical to the person who in good faith was first in the case at bar was annoted on the title of the subject
absence of Ramona P. Alcaraz as above-explained possession; and, in the absence thereof to the person property only on February 22, 1985, whereas, the second
offered no proof whatsoever to show that they actually who presents the oldest title, provided there is good faith. sale between petitioners Coronels and petitioner
presented the new transfer certificate of title in their Mabanag was supposedly perfected prior thereto or on
names and signified their willingness and readiness to The record of the case shows that the Deed of Absolute February 18, 1985. The idea conveyed is that at the time
execute the deed of absolute sale in accordance with their Sale dated April 25, 1985 as proof of the second contract petitioner Mabanag, the second buyer, bought the
agreement. Ramona's corresponding obligation to pay the of sale was registered with the Registry of Deeds of property under a clean title, she was unaware of any
balance of the purchase price in the amount of Quezon City giving rise to the issuance of a new adverse claim or previous sale, for which reason she is
P1,190,000.00 (as buyer) never became due and certificate of title in the name of Catalina B. Mabanag on buyer in good faith.
demandable and, therefore, she cannot be deemed to June 5, 1985. Thus, the second paragraph of Article 1544
have been in default. shall apply. We are not persuaded by such argument.

Article 1169 of the Civil Code defines when a party in a The above-cited provision on double sale presumes title In a case of double sale, what finds relevance and
contract involving reciprocal obligations may be or ownership to pass to the first buyer, the exceptions materiality is not whether or not the second buyer was a
considered in default, to wit: being: (a) when the second buyer, in good faith, registers buyer in good faith but whether or not said second buyer
the sale ahead of the first buyer, and (b) should there be registers such second sale in good faith, that is, without
Art. 1169. Those obliged to deliver or to do no inscription by either of the two buyers, when the knowledge of any defect in the title of the property sold.
something, incur in delay from the time the obligee second buyer, in good faith, acquires possession of the
judicially or extrajudicially demands from them the property ahead of the first buyer. Unless, the second As clearly borne out by the evidence in this case,
fulfillment of their obligation. buyer satisfies these requirements, title or ownership will petitioner Mabanag could not have in good faith,
not transfer to him to the prejudice of the first buyer. registered the sale entered into on February 18, 1985
xxx xxx xxx because as early as February 22, 1985, a notice of lis
pendens had been annotated on the transfer certificate of
166
Sales – Chapter 3-12 Cases
title in the names of petitioners, whereas petitioner
Mabanag registered the said sale sometime in April, 1985.
At the time of registration, therefore, petitioner Mabanag
knew that the same property had already been previously
sold to private respondents, or, at least, she was charged
with knowledge that a previous buyer is claiming title to
the same property. Petitioner Mabanag cannot close her
eyes to the defect in petitioners' title to the property at the
time of the registration of the property.

This Court had occasions to rule that:

If a vendee in a double sale registers that sale after he


has acquired knowledge that there was a previous sale of
the same property to a third party or that another person
claims said property in a pervious sale, the registration will
constitute a registration in bad faith and will not confer
upon him any right. (Salvoro vs. Tanega, 87 SCRA 349
[1978]; citing Palarca vs. Director of Land, 43 Phil. 146;
Cagaoan vs. Cagaoan, 43 Phil. 554; Fernandez vs.
Mercader, 43 Phil. 581.)

Thus, the sale of the subject parcel of land between


petitioners and Ramona P. Alcaraz, perfected on February
6, 1985, prior to that between petitioners and Catalina B.
Mabanag on February 18, 1985, was correctly upheld by
both the courts below.

Although there may be ample indications that there was in


fact an agency between Ramona as principal and
Concepcion, her mother, as agent insofar as the subject
contract of sale is concerned, the issue of whether or not
Concepcion was also acting in her own behalf as a co-
buyer is not squarely raised in the instant petition, nor in
such assumption disputed between mother and daughter.
Thus, We will not touch this issue and no longer disturb
the lower courts' ruling on this point.

WHEREFORE, premises considered, the instant petition


is hereby DISMISSED and the appealed judgment
AFFIRMED.

SO ORDERED.

167
Sales – Chapter 3-12 Cases
ANGEL M. PAGADUAN, et al. square meters to Agaton Pagaduan for P500.00. Later, on
- versus - June 5, 1962, Eugenia executed another deed of sale, SO ORDERED.[5]
SPOUSES ESTANISLAO & FE POSADAS OCUMA, this time conveying the entire parcel of land, including the
Respondents. southern portion, in respondents favor. Thus, TCT No. T-
1221 was cancelled and in lieu thereof TCT No. T-5425
In this Petition for Review,[1] petitioners assail the was issued in the name of respondents. On June 27, Dissatisfied with the decision, respondents appealed it to
Decision[2] of the Court of Appeals dated September 18, 1989, respondents subdivided the land into two lots. The the Court of Appeals. The Court of Appeals reversed and
2006 which ruled that petitioners action for reconveyance subdivision resulted in the cancellation of TCT No. T-5425 set aside the decision of the trial court; with the dispositive
is barred by prescription and consequently reversed the and the issuance of TCT Nos. T-37165 covering a portion portion of the decision reading, thus:
decision[3] dated June 25, 2002 of the Regional Trial with 31,418 square meters and T-37166 covering the
Court (RTC) of Olongapo City. remaining portion with 9,661 square meters.

Petitioners Angel N. Pagaduan, Amelia P. Tucci, Teresita On July 26, 1989, petitioners instituted a complaint for WHEREFORE, premises considered, the appeal is
P. del Monte, Orlita P. Gadin, Perla P. Espiritu, Elisa P. reconveyance of the southern portion with an area of granted. Accordingly, prescription having set in, the
Dunn, Lorna P. Kimble, Edito N. Pagaduan and Leo N. 8,754 square meters, with damages, against respondents assailed June 25, 2002 Decision of the RTC is reversed
Pagaduan are all heirs of the late Agaton Pagaduan. before the RTC of Olongapo City. and set aside, and the Complaint for reconveyance is
Respondents are the spouses Estanislao Ocuma and Fe hereby DISMISSED.
Posadas Ocuma.
SO ORDERED.[6]
The facts are as follows: On June 25, 2002, the trial court rendered a decision in
petitioners favor. Ruling that a constructive trust over the
property was created in petitioners favor, the court below The Court of Appeals ruled that while the registration of
The subject lot used to be part of a big parcel of land that ordered respondents to reconvey the disputed southern the southern portion in the name of respondents had
originally belonged to Nicolas Cleto as evidenced by portion and to pay attorneys fees as well as litigation created an implied trust in favor of Agaton Pagaduan,
Certificate of Title (C.T.) No. 14. The big parcel of land expenses to petitioners. The dispositive portion of the petitioners, however, failed to show that they had taken
was the subject of two separate lines of dispositions. The decision reads: possession of the said portion. Hence, the appellate court
first line of dispositions began with the sale by Cleto to concluded that prescription had set in, thereby precluding
Antonio Cereso on May 11, 1925. Cereso in turn sold the petitioners recovery of the disputed portion.
land to the siblings with the surname Antipolo on
September 23, 1943. The Antipolos sold the property to
Agaton Pagaduan, father of petitioners, on March 24,
1961. All the dispositions in this line were not registered Unperturbed by the reversal of the trial courts decision,
and did not result in the issuance of new certificates of title the petitioners come to this Court via a petition for review
in the name of the purchasers. on certiorari.[7] They assert that the Civil Code provision
on double sale is controlling. They submit further that
The second line of dispositions started on January 30, WHEREFORE, foregoing premises considered, judgment since the incontrovertible evidence on record is that they
1954, after Cletos death, when his widow Ruperta is hereby rendered: are in possession of the southern portion, the ten (10)-
Asuncion as his sole heir and new owner of the entire year prescriptive period for actions for reconveyance
tract, sold the same to Eugenia Reyes. This resulted in 1. Ordering the defendants to reconvey to the plaintiffs, a should not apply to them.[8] Respondents, on the other
the issuance of Transfer Certificate of Title (TCT) No. T- portion of their property originally covered by Certificate of hand, aver that the action for reconveyance has
1221 in the name of Eugenia Reyes in lieu of TCT No. T- Title No. T-54216[4] now TCT Nos. 37165 and 37166 an prescribed since the ten (10)-year period, which according
1220 in the name of Ruperta Asuncion. area equivalent to 8,754 square meters. to them has to be reckoned from the issuance of the title
in their name in 1962, has elapsed long ago.[9]
On November 26, 1961, Eugenia Reyes executed a 2. Ordering the defendant to pay plaintiffs P15,000.00 as
unilateral deed of sale where she sold the northern portion attorneys fees and P5,000.00 for litigation expenses. The Court of Appeals decision must be reversed and set
with an area of 32,325 square meters to respondents for aside, hence the petition succeeds.
P1,500.00 and the southern portion consisting of 8,754 3. Defendants counterclaims are dismissed.
168
Sales – Chapter 3-12 Cases
An action for reconveyance respects the decree of the same. Respondents sorely failed to meet this
registration as incontrovertible but seeks the transfer of This lack of a trust relationship does not inure to the requirement of good faith since they had actual
property, which has been wrongfully or erroneously benefit of the respondents. Despite a host of knowledge of Eugenias prior sale of the southern portion
registered in other persons' names, to its rightful and legal jurisprudence that states a certificate of title is property to the petitioners, a fact antithetical to good faith.
owners, or to those who claim to have a better right. indefeasible, unassailable and binding against the whole This cannot be denied by respondents since in the same
However, contrary to the positions of both the appellate world, it merely confirms or records title already existing deed of sale that Eugenia sold them the northern portion
and trial courts, no trust was created under Article 1456 of and vested, and it cannot be used to protect a usurper to the respondents for P1,500.00, Eugenia also sold the
the new Civil Code which provides: from the true owner, nor can it be used for the southern portion of the land to Agaton Pagaduan for
perpetration of fraud; neither does it permit one to enrich P500.00.[14]
himself at the expense of others.[11]
Art. 1456. If property is acquired through mistake or fraud,
the person obtaining it is, by force of law, considered a
trustee of an implied trust for the benefit of the person It is to be emphasized that the Agaton Pagaduan never
from whom the property comes. (Emphasis supplied) Rather, after a thorough scrutiny of the records of the parted with the ownership and possession of that portion
instant case, the Court finds that this is a case of double of Lot No. 785 which he had purchased from Eugenia
sale under article 1544 of the Civil Code which reads: Santos. Hence, the registration of the deed of sale by
respondents was ineffectual and vested upon them no
The property in question did not come from the preferential rights to the property in derogation of the
petitioners. In fact that property came from Eugenia ART. 1544. If the same thing should have been sold to rights of the petitioners.
Reyes. The title of the Ocumas can be traced back from different vendees, the ownership shall be transferred to
Eugenia Reyes to Ruperta Asuncion to the original owner the person who may have first possession thereof in good
Nicolas Cleto. Thus, if the respondents are holding the faith, if it should be movable property.
property in trust for anyone, it would be Eugenia Reyes Should it be immovable property, the ownership shall Respondents had prior knowledge of the sale of the
and not the petitioners. belong to the person acquiring it who in good faith first questioned portion to Agaton Pagaduan as the same
recorded it in the Registry of Property. deed of sale that conveyed the northern portion to them,
Moreover, as stated in Berico v. Court of Appeals,[10] Should there be no inscription, the ownership shall pertain conveyed the southern portion to Agaton Pagaduan.[15]
Article 1456 refers to actual or constructive fraud. Actual to the person who in good faith was first in possession; Thus the subsequent issuance of TCT No. T-5425, to the
fraud consists in deception, intentionally practiced to and, in the absence thereof; to the person who presents extent that it affects the Pagaduans portion, conferred no
induce another to part with property or to surrender some the oldest title, provided there is good faith. better right than the registration which was the source of
legal right, and which accomplishes the end designed. the authority to issue the said title. Knowledge gained by
Constructive fraud, on the other hand, is a breach of legal respondents of the first sale defeats their rights even if
or equitable duty which the law declares fraudulent Otherwise stated, where it is an immovable property that they were first to register the second sale. Knowledge of
irrespective of the moral guilt of the actor due to the is the subject of a double sale, ownership shall be the first sale blackens this prior registration with bad
tendency to deceive others, to violate public or private transferred: (1) to the person acquiring it who in good faith faith.[16] Good faith must concur with the registration.[17]
confidence, or to injure public interests. The latter first recorded it in the Registry of Property; (2) in default Therefore, because the registration by the respondents
proceeds from a breach of duty arising out of a fiduciary or thereof, to the person who in good faith was first in was in bad faith, it amounted to no registration at all.
confidential relationship. In the instant case, none of the possession; and (3) in default thereof, to the person who
elements of actual or constructive fraud exists. The presents the oldest title, provided there is good faith. The As the respondents gained no rights over the land, it is
respondents did not deceive Agaton Pagaduan to induce requirement of the law then is two-fold: acquisition in good petitioners who are the rightful owners, having established
the latter to part with the ownership or deliver the faith and registration in good faith.[12] that their successor-in-interest Agaton Pagaduan had
possession of the property to them. Moreover, no fiduciary purchased the property from Eugenia Reyes on
relations existed between the two parties. In this case there was a first sale by Eugenia Reyes to November 26, 1961 and in fact took possession of the
Agaton Pagaduan and a second sale by Eugenia Reyes said property. The action to recover the immovable is not
to the respondents.[13] For a second buyer like the barred by prescription, as it was filed a little over 27 years
respondents to successfully invoke the second paragraph, after the title was registered in bad faith by the Ocumas as
Article 1544 of the Civil Code, it must possess good faith per Article 1141 of the Civil Code.[18]
from the time of the sale in its favor until the registration of
169
Sales – Chapter 3-12 Cases
WHEREFORE, the petition is GRANTED. The Decision of
the Court of Appeals dated January 25, 2006 and its
Resolution dated May 5, 2006 are hereby REVERSED
and SET ASIDE. The Decision of the Regional Trial Court
is hereby REINSTATED.

SO ORDERED.

170
Sales – Chapter 3-12 Cases
ROSARIO CARBONELL, petitioner, the consent of his wife and parents, accepted the price had to pay in addition to her assuming the mortgaged
vs. proposed by petitioner, on the condition that from the obligation to Republic Savings Bank.
HONORABLE COURT OF APPEALS, JOSE PONCIO, purchase price would come the money to be paid to the
EMMA INFANTE and RAMON INFANTE, respondents. bank. Upon arriving at respondent Jose Poncio's house,
however, the latter told petitioner that he could not
Petitioner seeks a review of the resolution of the Court of Petitioner and respondent Jose Poncio then went to the proceed any more with the sale, because he had already
Appeals (Special Division of Five) dated October 30, Republic Savings Bank and secured the consent of the given the lot to respondent Emma Infants; and that he
1968, reversing its decision of November 2, 1967 (Fifth President thereof for her to pay the arrears on the could not withdraw from his deal with respondent Mrs.
Division), and its resolution of December 6, 1968 denying mortgage and to continue the payment of the installments Infante, even if he were to go to jail. Petitioner then sought
petitioner's motion for reconsideration. as they fall due. The amount in arrears reached a total to contact respondent Mrs. Infante but the latter refused to
sum of P247.26. But because respondent Poncio had see her.
The dispositive part of the challenged resolution reads: previously told her that the money, needed was only
P200.00, only the latter amount was brought by petitioner On February 5, 1955, petitioner saw Emma Infante
Wherefore, the motion for reconsideration filed on behalf constraining respondent Jose Poncio to withdraw the sum erecting a all around the lot with a gate.
of appellee Emma Infante, is hereby granted and the of P47.00 from his bank deposit with Republic Savings
decision of November 2, 1967, is hereby annulled and set Bank. But the next day, petitioner refunded to Poncio the Petitioner then consulted Atty. Jose Garcia, who advised
aside. Another judgement shall be entered affirming in sum of P47.00. her to present an adverse claim over the land in question
toto that of the court a quo, dated January 20, 1965, with the Office of the Register of Deeds of Rizal. Atty.
which dismisses the plaintiff's complaint and defendant's On January 27, 1955, petitioner and respondent Poncio, Garcia actually sent a letter of inquiry to the Register of
counterclaim. in the presence of a witness, made and executed a Deeds and demand letters to private respondents Jose
document in the Batanes dialect, which, translated into Poncio and Emma Infante.
Without costs. English, reads:
In his answer to the complaint Poncio admitted "that on
The facts of the case as follows: CONTRACT FOR ONE HALF LOT WHICH I BOUGHT January 30, 1955, Mrs. Infante improved her offer and he
FROM agreed to sell the land and its improvements to her for
Prior to January 27, 1955, respondent Jose Poncio, a P3,535.00" (pp. 38-40, ROA).
native of the Batanes Islands, was the owner of the parcel JOSE PONCIO
of land herein involve with improvements situated at 179 In a private memorandum agreement dated January 31,
V. Agan St., San Juan, Rizal, having an area of some one Beginning today January 27, 1955, Jose Poncio can start 1955, respondent Poncio indeed bound himself to sell to
hundred ninety-five (195) square meters, more or less, living on the lot sold by him to me, Rosario Carbonell, until his corespondent Emma Infante, the property for the sum
covered by TCT No. 5040 and subject to mortgage in after one year during which time he will not pa anything. of P2,357.52, with respondent Emma Infante still
favor of the Republic Savings Bank for the sum of Then if after said one can he could not find an place assuming the existing mortgage debt in favor of Republic
P1,500.00. Petitioner Rosario Carbonell, a cousin and where to move his house, he could still continue Savings Bank in the amount of P1,177.48. Emma Infante
adjacent neighbor of respondent Poncio, and also from occupying the site but he should pay a rent that man, be lives just behind the houses of Poncio and Rosario
the Batanes Islands, lived in the adjoining lot at 177 V. agreed. Carbonell.
Agan Street.
(Sgd) JOSE PONCIO On February 2, 1955, respondent Jose Poncio executed
Both petitioners Rosario Carbonell and respondent Emma (Sgd.) ROSARIO CARBONELL the formal deed of sale in favor of respondent Mrs. Infante
Infante offered to buy the said lot from Poncio (Poncio's (Sgd) CONSTANCIO MEONADA in the total sum of P3,554.00 and on the same date, the
Answer, p. 38, rec. on appeal). Witness latter paid Republic Savings Bank the mortgage
indebtedness of P1,500.00. The mortgage on the lot was
Respondent Poncio, unable to keep up with the (Pp. 6-7 rec. on appeal). eventually discharged.
installments due on the mortgage, approached petitioner
one day and offered to sell to the latter the said lot, Thereafter, petitioner asked Atty. Salvador Reyes, also Informed that the sale in favor of respondent Emma
excluding the house wherein respondent lived. Petitioner from the Batanes Islands, to prepare the formal deed of Infante had not yet been registered, Atty. Garcia prepared
accepted the offer and proposed the price of P9.50 per sale, which she brought to respondent Poncio together an adverse claim for petitioner, who signed and swore to
square meter. Respondent Poncio, after having secured with the amount of some P400.00, the balance she still an registered the same on February 8, 1955.
171
Sales – Chapter 3-12 Cases
dismissed the complaint on the ground that the After the re-hearing, the trial court rendered a decision,
The deed of sale in favor of respondent Mrs. Infante was memorandum presented by petitioner to prove said sale reversing its decision of December 5, 1962 on the ground
registered only on February 12, 1955. As a consequence does not satisfy the requirements of the law (pp. 31-35, that the claim of the respondents was superior to the claim
thereof, a Transfer Certificate of Title was issued to her ROA in the C.A.). of petitioner, and dismissing the complaint (pp. 91-95,
but with the annotation of the adverse claim of petitioner ROA in the C.A.), From this decision, petitioner Rosario
Rosario Carbonell. From the above order of dismissal, petitioner appealed to Carbonell appealed to the respondent Court of Appeals
the Supreme Court (G.R. No. L-11231) which ruled in a (p. 96, ROA in the C.A.).
Respondent Emma Infante took immediate possession of decision dated May 12, 1958, that the Statute of Frauds,
the lot involved, covered the same with 500 cubic meters being applicable only to executory contracts, does not On November 2, 1967, the Court of Appeals (Fifth Division
of garden soil and built therein a wall and gate, spending apply to the alleged sale between petitioner and composed of Justices Magno Gatmaitan, Salvador V.
the sum of P1,500.00. She further contracted the services respondent Poncio, which petitioner claimed to have been Esguerra and Angle H. Mojica, speaking through Justice
of an architect to build a house; but the construction of the partially performed, so that petitioner is entitled to Magno Gatmaitan), rendered judgment reversing the
same started only in 1959 — years after the litigation establish by parole evidence "the truth of this allegation, decision of the trial court, declaring petitioner therein, to
actually began and during its pendency. Respondent Mrs. as well as the contract itself." The order appealed from have a superior right to the land in question, and
Infante spent for the house the total amount of was thus reversed, and the case remanded to the court a condemning the defendant Infantes to reconvey to
P11,929.00. quo for further proceedings (pp. 26-49, ROA in the C.A.). petitioner after her reimbursement to them of the sum of
P3,000.00 plus legal interest, the land in question and all
On June 1, 1955, petitioner Rosario Carbonell, thru After trial in the court a quo; a decision was, rendered on its improvements (Appendix "A" of Petition).
counsel, filed a second amended complaint against December 5, 1962, declaring the second sale by
private respondents, praying that she be declared the respondent Jose Poncio to his co-respondents Ramon Respondent Infantes sought reconsideration of said
lawful owner of the questioned parcel of land; that the Infante and Emma Infante of the land in question null and decision and acting on the motion for reconsideration, the
subsequent sale to respondents Ramon R. Infante and void and ordering respondent Poncio to execute the Appellate Court, three Justices (Villamor, Esguerra and
Emma L. Infante be declared null and void, and that proper deed of conveyance of said land in favor of Nolasco) of Special Division of Five, granted said motion,
respondent Jose Poncio be ordered to execute the petitioner after compliance by the latter of her covenants annulled and set aside its decision of November 2, 1967,
corresponding deed of conveyance of said land in her under her agreement with respondent Poncio (pp. 5056, and entered another judgment affirming in toto the
favor and for damages and attorney's fees (pp. 1-7, rec. ROA in the C.A.). decision of the court a quo, with Justices Gatmaitan and
on appeal in the C.A.). Rodriguez dissenting (Appendix "B" of Petition).
On January 23, 1963, respondent Infantes, through
Respondents first moved to dismiss the complaint on the another counsel, filed a motion for re-trial to adduce Petitioner Rosario Carbonell moved to reconsider the
ground, among others, that petitioner's claim is evidence for the proper implementation of the court's Resolution of the Special Division of Five, which motion
unenforceable under the Statute of Frauds, the alleged decision in case it would be affirmed on appeal (pp. 56-60, was denied by Minute Resolution of December 6, 1968
sale in her favor not being evidenced by a written ROA in the C.A.), which motion was opposed by petitioner (but with Justices Rodriguez and Gatmaitan voting for
document (pp. 7-13, rec. on appeal in the C.A.); and when for being premature (pp. 61-64, ROA in the C.A.). Before reconsideration) [Appendix "C" of Petition].
said motion was denied without prejudice to passing on their motion for re-trial could be resolved, respondent
the question raised therein when the case would be tried Infantes, this time through their former counsel, filed Hence, this appeal by certiorari.
on the merits (p. 17, ROA in the C.A.), respondents filed another motion for new trial, claiming that the decision of
separate answers, reiterating the grounds of their motion the trial court is contrary to the evidence and the law (pp. Article 1544, New Civil Code, which is decisive of this
to dismiss (pp. 18-23, ROA in the C.A.). 64-78, ROA in the C.A.), which motion was also opposed case, recites:
by petitioner (pp. 78-89, ROA in the C.A.).
During the trial, when petitioner started presenting If the same thing should have been sold to different
evidence of the sale of the land in question to her by The trial court granted a new trial (pp. 89-90, ROA in the vendees, the ownership shall be transferred to the person
respondent Poncio, part of which evidence was the C.A.), at which re-hearing only the respondents introduced who may have first taken possession thereof in good faith,
agreement written in the Batanes dialect aforementioned, additional evidence consisting principally of the cost of if it should movable property.
respondent Infantes objected to the presentation by improvements they introduced on the land in question (p.
petitioner of parole evidence to prove the alleged sale 9, ROA in the C.A.). Should it be immovable property, the ownership shall
between her and respondent Poncio. In its order of April belong to the person acquiring it who in good faith first
26, 1966, the trial court sustained the objection and recorded it in the Registry of Property.
172
Sales – Chapter 3-12 Cases
done in good faith and should emphasize Infante's bad during the trial, in much the same way that the Infantes
Should there be no inscription, the ownership shall pertain faith when she registered her deed of sale four (4) days were able to present as evidence Exhibit "1" — Infantes,
to the person who in good faith was first in the later on February 12, 1955. Poncio's savings deposit passbook, of which Poncio
possession; and, in the absence thereof, to the person necessarily remained in possession as the said deposit
who presents the oldest title, provided there is good faith Bad faith arising from previous knowledge by Infante of passbook was never involved in the contract of sale with
(emphasis supplied). the prior sale to Carbonell is shown by the following facts, assumption of mortgage. Said savings deposit passbook
the vital significance and evidenciary effect of which the merely proves that Poncio had to withdraw P47.26, which
It is essential that the buyer of realty must act in good faith respondent Court of Appeals either overlooked of failed to amount was tided to the sum of P200.00 paid by
in registering his deed of sale to merit the protection of the appreciate: Carbonell for Poncio's amortization arrearages in favor of
second paragraph of said Article 1544. the bank on January 27, 1955; because Carbonell on that
(1) Mrs. Infante refused to see Carbonell, who day brought with her only P200.00, as Poncio told her that
Unlike the first and third paragraphs of said Article 1544, wanted to see Infante after she was informed by Poncio was the amount of his arrearages to the bank. But the
which accord preference to the one who first takes that he sold the lot to Infante but several days before next day Carbonell refunded to Poncio the sum of P47.26.
possession in good faith of personal or real property, the Infante registered her deed of sale. This indicates that
second paragraph directs that ownership of immovable Infante knew — from Poncio and from the bank — of the (3) The fact that Poncio was no longer in possession
property should be recognized in favor of one "who in prior sale of the lot by Poncio to Carbonell. Ordinarily, one of his mortgage passbook and that the said mortgage
good faith first recorded" his right. Under the first and third will not refuse to see a neighbor. Infante lives just behind passbook was already in possession of Carbonell, should
paragraph, good faith must characterize the act of anterior the house of Carbonell. Her refusal to talk to Carbonell have compelled Infante to inquire from Poncio why he was
registration (DBP vs. Mangawang, et al., 11 SCRA 405; could only mean that she did not want to listen to no longer in possession of the mortgage passbook and
Soriano, et al. vs. Magale, et al., 8 SCRA 489). Carbonell's story that she (Carbonell) had previously from Carbonell why she was in possession of the same
bought the lot from Poncio. (Paglago, et. al vs. Jara et al 22 SCRA 1247, 1252-1253).
If there is no inscription, what is decisive is prior The only plausible and logical reason why Infante did not
possession in good faith. If there is inscription, as in the (2) Carbonell was already in possession of the bother anymore to make such injury , w because in the
case at bar, prior registration in good faith is a pre- mortgage passbook [not Poncio's saving deposit ordinary course of business the bank must have told her
condition to superior title. passbook — Exhibit "1" — Infantes] and Poncio's copy of that Poncio already sold the lot to Carbonell who thereby
the mortgage contract, when Poncio sold the lot Carbonell assumed the mortgage indebtedness of Poncio and to
When Carbonell bought the lot from Poncio on January who, after paying the arrearages of Poncio, assumed the whom Poncio delivered his mortgage passbook. Hoping to
27, 1955, she was the only buyer thereof and the title of balance of his mortgaged indebtedness to the bank, which give a semblance of truth to her pretended good faith,
Poncio was still in his name solely encumbered by bank in the normal course of business must have necessarily Infante snubbed Carbonell's request to talk to her about
mortgage duly annotated thereon. Carbonell was not informed Infante about the said assumption by Carbonell the prior sale to her b Poncio of the lot. As aforestated,
aware — and she could not have been aware — of any of the mortgage indebtedness of Poncio. Before or upon this is not the attitude expected of a good neighbor
sale of Infante as there was no such sale to Infante then. paying in full the mortgage indebtedness of Poncio to the imbued with Christian charity and good will as well as a
Hence, Carbonell's prior purchase of the land was made Bank. Infante naturally must have demanded from Poncio clear conscience.
in good faith. Her good faith subsisted and continued to the delivery to her of his mortgage passbook as well as
exist when she recorded her adverse claim four (4) days Poncio's mortgage contract so that the fact of full payment (4) Carbonell registered on February 8, 1955 her
prior to the registration of Infantes's deed of sale. of his bank mortgage will be entered therein; and Poncio, adverse claim, which was accordingly annotated on
Carbonell's good faith did not cease after Poncio told her as well as the bank, must have inevitably informed her Poncio's title, four [4] days before Infante registered on
on January 31, 1955 of his second sale of the same lot to that said mortgage passbook could not be given to her February 12, 1955 her deed of sale executed on February
Infante. Because of that information, Carbonell wanted an because it was already delivered to Carbonell. 2, 1955. Here she was again on notice of the prior sale to
audience with Infante, which desire underscores Carbonell. Such registration of adverse claim is valid and
Carbonell's good faith. With an aristocratic disdain If Poncio was still in possession of the mortgage passbook effective (Jovellanos vs. Dimalanta, L-11736-37, Jan. 30,
unworthy of the good breeding of a good Christian and and his copy of the mortgage contract at the time he 1959, 105 Phil. 1250-51).
good neighbor, Infante snubbed Carbonell like a leper and executed a deed of sale in favor of the Infantes and when
refused to see her. So Carbonell did the next best thing to the Infantes redeemed his mortgage indebtedness from (5) In his answer to the complaint filed by Poncio, as
protect her right — she registered her adversed claim on the bank, Poncio would have surrendered his mortgage defendant in the Court of First Instance, he alleged that
February 8, 1955. Under the circumstances, this recording passbook and his copy of the mortgage contract to the both Mrs. Infante and Mrs. Carbonell offered to buy the lot
of her adverse claim should be deemed to have been Infantes, who could have presented the same as exhibits at P15.00 per square meter, which offers he rejected as
173
Sales – Chapter 3-12 Cases
he believed that his lot is worth at least P20.00 per square memorandum in writing within the purview of the Statute Poncio, he being a native of said region. Exhibit A states
meter. It is therefore logical to presume that Infante was of Frauds, the trial judge himself recognized the fact of the that Poncio would stay in the land sold by him to plaintiff
told by Poncio and consequently knew of the offer of prior sale to Carbonell when he stated that "the for one year, from January 27, 1955, free of charge, and
Carbonell which fact likewise should have put her on her memorandum in question merely states that Poncio is that, if he cannot find a place where to transfer his house
guard and should have compelled her to inquire from allowed to stay in the property which he had sold to the thereon, he may remain upon. Incidentally, the allegation
Poncio whether or not he had already sold the property to plaintiff. There is no mention of the reconsideration, a in Poncio's answer to the effect that he signed Exhibit A
Carbonell. description of the property and such other essential under the belief that it "was a permit for him to remain in
elements of the contract of sale. There is nothing in the the premises in the" that "he decided to sell the property"
As recounted by Chief Justice Roberto Concepcion, then memorandum which would tend to show even in the to the plaintiff at P20 a sq. m." is, on its face, somewhat
Associate Justice, in the preceding case of Rosario slightest manner that it was intended to be an evidence of difficult to believe. Indeed, if he had not decided as yet to
Carbonell vs. Jose Poncio, Ramon Infante and Emma contract sale. On the contrary, from the terms of the sell the land to plaintiff, who had never increased her offer
Infante (1-11231, May 12, 1958), Poncio alleged in his memorandum, it tends to show that the sale of the of P15 a square meter, there was no reason for Poncio to
answer: property in favor of the plaintiff is already an accomplished get said permit from her. Upon the other hand, if plaintiff
act. By the very contents of the memorandum itself, it intended to mislead Poncio, she would have caused
... that he had consistently turned down several offers, cannot therefore, be considered to be the memorandum Exhibit A to be drafted, probably, in English , instead of
made by plaintiff, to buy the land in question, at P15 a which would show that a sale has been made by Poncio in taking the trouble of seeing to it that it was written
square meter, for he believes that it is worth not less than favor of the plaintiff" (p. 33, ROA, emphasis supplied). As precisely in his native dialect, the Batanes. Moreover,
P20 a square meter; that Mrs. Infante, likewise, tried to found by the trial court, to repeat the said memorandum Poncio's signature on Exhibit A suggests that he is neither
buy the land at P15 a square meter; that, on or about states "that Poncio is allowed to stay in the property which illiterate nor so ignorant as to sign document without
January 27, 1955, Poncio was advised by plaintiff that he had sold to the plaintiff ..., it tends to show that the sale reading its contents, apart from the fact that Meonada had
should she decide to buy the property at P20 a square of the property in favor of the plaintiff is already an read Exhibit A to him and given him a copy thereof, before
meter, she would allow him to remain in the property for accomplished act..." he signed thereon, according to Meonada's
one year; that plaintiff then induced Poncio to sign a uncontradicted testimony.
document, copy of which if probably the one appended to (2) When the said order was appealed to the
the second amended complaint; that Poncio signed it Supreme Court by Carbonell in the previous case of Then, also, defendants say in their brief:
'relying upon the statement of the plaintiff that the Rosario Carbonell vs. Jose Poncio, Ramon Infante and
document was a permit for him to remain in the premises Emma Infante The only allegation in plaintiff's complaint that bears any
in the event defendant decided to sell the property to the (L-11231, supra), Chief Justice Roberto Concepcion, then relation to her claim that there has been partial
plaintiff at P20.00 a square meter'; that on January 30, Associate Justice, speaking for a unanimous Court, performance of the supposed contract of sale, is the
1955, Mrs. Infante improved her offer and agreed to sell reversed the aforesaid order of the trial court dismissing notation of the sum of P247.26 in the bank book of
the land and its improvement to her for P3,535.00; that the complaint, holding that because the complaint alleges defendant Jose Poncio. The noting or jotting down of the
Poncio has not lost 'his mind,' to sell his property, worth at and the plaintiff claims that the contract of sale was partly sum of P247.26 in the bank book of Jose Poncio does not
least P4,000, for the paltry sum P1,177.48, the amount of performed, the same is removed from the application of prove the fact that the said amount was the purchase
his obligation to the Republic Saving s Bank; and that the Statute of Frauds and Carbonell should be allowed to price of the property in question. For all we knew, the sum
plaintiff's action is barred by the Statute of Frauds. ... (pp. establish by parol evidence the truth of her allegation of of P247.26 which plaintiff claims to have paid to the
38-40, ROA, emphasis supplied). partial performance of the contract of sale, and further Republic Savings Bank for the account of the defendant,
stated: assuming that the money paid to the Republic Savings
II Bank came from the plaintiff, was the result of some
Apart from the foregoing, there are in the case at bar usurious loan or accomodation, rather than earnest
EXISTENCE OF THE PRIOR SALE TO CARBONELL several circumstances indicating that plaintiff's claim might money or part payment of the land. Neither is it competent
DULY ESTABLISHED not be entirely devoid of factual basis. Thus, for instance, or satisfactory evidence to prove the conveyance of the
Poncio admitted in his answer that plaintiff had offered land in question the fact that the bank book account of
(1) In his order dated April 26, 1956 dismissing the several times to purchase his land. Jose Poncio happens to be in the possession of the
complaint on the ground that the private document Exhibit plaintiff. (Defendants-Appellees' brief, pp. 25-26).
"A" executed by Poncio and Carbonell and witnessed by Again, there is Exhibit A, a document signed by the
Constancio Meonada captioned "Contract for One-half Lot defendant. It is in the Batanes dialect, which, according to How shall We know why Poncio's bank deposit book is in
which I Bought from Jose Poncio," was not such a plaintiff's uncontradicted evidence, is the one spoken by plaintiffs possession, or whether there is any relation
174
Sales – Chapter 3-12 Cases
between the P247.26 entry therein and the partial description of the lot referred to in the note, especially Jose there had been celebrated a sale of the property
payment of P247.26 allegedly made by plaintiff to Poncio when the note refers to only one half lot. With respect to excluding the house for the price of P9.50 per square
on account of the price of his land, if we do not allow the the latter argument of the Exhibit 'A', the court has arrived meter, so much so that on faith of that, Rosario had
plaintiff to explain it on the witness stand? Without at the conclusion that there is a sufficient description of advanced the sum of P247.26 and binding herself to pay
expressing any opinion on the merits of plaintiff's claim, it the lot referred to in Exh. 'A' as none other than the parcel unto Jose the balance of the purchase price after
is clear, therefore, that she is entitled , legally as well as of land occupied by the defendant Poncio and where he deducting the indebtedness to the Bank and since the
from the viewpoint of equity, to an opportunity to introduce has his improvements erected. The Identity of the parcel wording of Exhibit A, the private document goes so far as
parol evidence in support of the allegations of her second of land involved herein is sufficiently established by the to describe their transaction as one of sale, already
amended complaint. (pp. 46-49, ROA, emphasis contents of the note Exh. "A". For a while, this court had consummated between them, note the part tense used in
supplied). that similar impression but after a more and thorough the phrase, "the lot sold by him to me" and going so far
consideration of the context in Exh. 'A' and for the reasons even as to state that from that day onwards, vendor would
(3) In his first decision of December 5, 1962 declaring null stated above, the Court has arrived at the conclusion continue to live therein, for one year, 'during which time he
and void the sale in favor of the Infantes and ordering stated earlier (pp. 52-54, ROA, emphasis supplied). will not pay anything' this can only mean that between
Poncio to execute a deed of conveyance in favor of Rosario and Jose, there had been a true contract of sale,
Carbonell, the trial judge found: (4) After re-trial on motion of the Infantes, the trial consummated by delivery constitutum possession, Art.
Judge rendered on January 20, 1965 another decision 1500, New Civil Code; vendor's possession having
... A careful consideration of the contents of Exh. 'A' show dismissing the complaint, although he found become converted from then on, as a mere tenant of
to the satisfaction of the court that the sale of the parcel of vendee, with the special privilege of not paying rental for
land in question by the defendant Poncio in favor of the 1. That on January 27, 1955, the plaintiff purchased one year, — it is true that the sale by Jose Poncio to
plaintiff was covered therein and that the said Exh. "a' was from the defendant Poncio a parcel of land with an area of Rosario Carbonell corroborated documentarily only by
also executed to allow the defendant to continue staying 195 square meters, more or less, covered by TCT No. Exhibit A could not have been registered at all, but it was
in the premises for the stated period. It will be noted that 5040 of the Province of Rizal, located at San Juan del a valid contract nonetheless, since under our law, a
Exh. 'A' refers to a lot 'sold by him to me' and having been Monte, Rizal, for the price of P6.50 per square meter; contract sale is consensual, perfected by mere consent,
written originally in a dialect well understood by the Couto v. Cortes, 8 Phil 459, so much so that under the
defendant Poncio, he signed the said Exh. 'A' with a full 2. That the purchase made by the plaintiff was not New Civil Code, while a sale of an immovable is ordered
knowledge and consciousness of the terms and reduced to writing except for a short note or memorandum to be reduced to a public document, Art. 1358, that
consequences thereof. This therefore, corroborates the Exh. A, which also recited that the defendant Poncio mandate does not render an oral sale of realty invalid, but
testimony of the plaintiff Carbonell that the sale of the land would be allowed to continue his stay in the premises, merely incapable of proof, where still executory and action
was made by Poncio. It is further pointed out that there among other things, ... (pp. 91-92, ROA, emphasis is brought and resisted for its performance, 1403, par. 2,
was a partial performance of the verbal sale executed by supplied). 3; but where already wholly or partly executed or where
Poncio in favor of the plaintiff, when the latter paid even if not yet, it is evidenced by a memorandum, in any
P247.26 to the Republic Savings Bank on account of From such factual findings, the trial Judge confirms the case where evidence to further demonstrate is presented
Poncio's mortgage indebtedness. Finally, the possession due execution of Exhibit "A", only that his legal conclusion and admitted as the case was here, then the oral sale
by the plaintiff of the defendant Poncio's passbook of the is that it is not sufficient to transfer ownership (pp. 93-94, becomes perfectly good, and becomes a good cause of
Republic Savings Bank also adds credibility to her ROA). action not only to reduce it to the form of a public
testimony. The defendant contends on the other hand that document, but even to enforce the contract in its entirety,
the testimony of the plaintiff, as well as her witnesses, (5) In the first decision of November 2, 1967 of the Art. 1357; and thus it is that what we now have is a case
regarding the sale of the land made by Poncio in favor of Fifth Division of the Court of Appeals composed of wherein on the one hand Rosario Carbonell has proved
the plaintiff is inadmissible under the provision of the Justices Esguerra (now Associate Justice of the Supreme that she had an anterior sale, celebrated in her favor on
Statute of Fraud based on the argument that the note Court), Gatmaitan and Mojica, penned by Justice 27 January, 1955, Exhibit A, annotated as an adverse
Exh. "A" is not the note or memorandum referred to in the Gatmaitan, the Court of Appeals found that: claim on 8 February, 1955, and on other, a sale is due
to in the Statute of Fraud. The defendants argue that Exh. form in favor of Emma L. Infante on 2 February, 1955,
"A" fails to comply with the requirements of the Statute of ... the testimony of Rosario Carbonell not having at all Exhibit 3-Infante, and registered in due form with title unto
Fraud to qualify it as the note or memorandum referred to been attempted to be disproved by defendants, her issued on 12 February, 1955; the vital question must
therein and open the way for the presentation of parole particularly Jose Poncio, and corroborated as it is by the now come on which of these two sales should prevail; ...
evidence to prove the fact contained in the note or private document in Batanes dialect, Exhibit A, the (pp. 74-76, rec., emphasis supplied).
memorandum. The defendant argues that there is even no testimony being to the effect that between herself and
175
Sales – Chapter 3-12 Cases
(6) In the resolution dated October 30, 1968 penned which amount was refunded to him by Carbonell the the observation of the Supreme Court through Mr. Chief
by then Court of Appeals Justice Esguerra (now a following day. This conveyance was confirmed that same Justice Concepcion in G.R. No. L-11231, supra, bears
member of this Court), concurred in by Justices Villamor day, January 27, 1955, by the private document, Exhibit repeating:
and Nolasco, constituting the majority of a Special "A", which was prepared in the Batanes dialect by the
Division of Five, the Court of Appeals, upon motion of the witness Constancio Meonada, who is also from Batanes ... Incidentally, the allegation in Poncio's answer to the
Infantes, while reversing the decision of November 2, like Poncio and Carbonell. effect that he signed Exhibit A under the belief that it 'was
1967 and affirming the decision of the trial court of a permit for him to remain in the premises in the event that
January 20, 1965 dismissing plaintiff's complaint, admitted The sale did not include Poncio's house on the lot. And 'he decided to sell the property' to the plaintiff at P20.00 a
the existence and genuineness of Exhibit "A", the private Poncio was given the right to continue staying on the land sq. m is, on its face, somewhat difficult to believe. Indeed,
memorandum dated January 27, 1955, although it did not without paying any rental for one year, after which he if he had not decided as yet to sell that land to plaintiff,
consider the same as satisfying "the essential elements of should pay rent if he could not still find a place to transfer who had never increased her offer of P15 a square meter,
a contract of sale," because it "neither specifically his house. All these terms are part of the consideration of there as no reason for Poncio to get said permit from her.
describes the property and its boundaries, nor mention its the sale to Carbonell. Upon the they if plaintiff intended to mislead Poncio, she
certificate of title number, nor states the price certain to be would have Exhibit A to be drafted, probably, in English,
paid, or contrary to the express mandate of Articles 1458 It is evident therefore that there was ample consideration, instead of taking the trouble of seeing to it that it was
and 1475 of the Civil Code. and not merely the sum of P200.00, for the sale of Poncio written precisely in his native dialect, the Batanes.
to Carbonell of the lot in question. Moreover, Poncio's signature on Exhibit A suggests that
(7) In his dissent concurred in by Justice Rodriguez, he is neither illiterate nor so ignorant as to sign a
Justice Gatmaitan maintains his decision of November 2, But Poncio, induced by the higher price offered to him by document without reading its contents, apart from the fact
1967 as well as his findings of facts therein, and reiterated Infante, reneged on his commitment to Carbonell and told that Meonada had read Exhibit A to him-and given him a
that the private memorandum Exhibit "A", is a perfected Carbonell, who confronted him about it, that he would not copy thereof, before he signed thereon, according to
sale, as a sale is consensual and consummated by mere withdraw from his deal with Infante even if he is sent to jail Meonada's uncontradicted testimony. (pp. 46-47, ROA).
consent, and is binding on and effective between the The victim, therefore, "of injustice and outrage is the
parties. This statement of the principle is correct [pp. 89- widow Carbonell and not the Infantes, who without moral As stressed by Justice Gatmaitan in his first decision of
92, rec.]. compunction exploited the greed and treacherous nature November 2, 1965, which he reiterated in his dissent from
of Poncio, who, for love of money and without remorse of the resolution of the majority of the Special Division. of
III conscience, dishonored his own plighted word to Five on October 30, 1968, Exhibit A, the private document
Carbonell, his own cousin. in the Batanes dialect, is a valid contract of sale between
ADEQUATE CONSIDERATION OR PRICE FOR THE the parties, since sale is a consensual contract and is
SALE Inevitably evident therefore from the foregoing discussion, perfected by mere consent (Couto vs. Cortes, 8 Phil. 459).
IN FAVOR OF CARBONELL is the bad faith of Emma Infante from the time she enticed Even an oral contract of realty is all between the parties
Poncio to dishonor his contract with Carbonell, and and accords to the vendee the right to compel the vendor
It should be emphasized that the mortgage on the lot was instead to sell the lot to her (Infante) by offering Poncio a to execute the proper public document As a matter of fact,
about to be foreclosed by the bank for failure on the part much higher price than the price for which he sold the Exhibit A, while merely a private document, can be fully or
of Poncio to pay the amortizations thereon. To forestall same to Carbonell. Being guilty of bad faith, both in taking partially performed, to it from the operation of the statute
the foreclosure and at the same time to realize some physical possession of the lot and in recording their deed of frauds. Being a all consensual contract, Exhibit A
money from his mortgaged lot, Poncio agreed to sell the of sale, the Infantes cannot recover the value of the effectively transferred the possession of the lot to the
same to Carbonell at P9.50 per square meter, on improvements they introduced in the lot. And after the vendee Carbonell by constitutum possessorium (Article
condition that Carbonell [1] should pay (a) the amount of filing by Carbonell of the complaint in June, 1955, the 1500, New Civil Code); because thereunder the vendor
P400.00 to Poncio and 9b) the arrears in the amount of Infantes had less justification to erect a building thereon Poncio continued to retain physical possession of the lot
P247.26 to the bank; and [2] should assume his mortgage since their title to said lot is seriously disputed by as tenant of the vendee and no longer as knew thereof.
indebtedness. The bank president agreed to the said sale Carbonell on the basis of a prior sale to her. More than just the signing of Exhibit A by Poncio and
with assumption of mortgage in favor of Carbonell an Carbonell with Constancio Meonada as witness to fact the
Carbonell accordingly paid the arrears of P247.26. On With respect to the claim of Poncio that he signed the contract of sale, the transition was further confirmed when
January 27, 1955, she paid the amount of P200.00 to the document Exhibit "A" under the belief that it was a permit Poncio agreed to the actual payment by at Carbonell of
bank because that was the amount that Poncio told her as for him to remain in the premises in ease he decides to his mortgage arrearages to the bank on January 27, 1955
his arrearages and Poncio advanced the sum of P47.26, sell the property to Carbonell at P20.00 per square meter, and by his consequent delivery of his own mortgage
176
Sales – Chapter 3-12 Cases
passbook to Carbonell. If he remained owner and amount of P1,500.00, which the Infantes paid to the WHEREFORE, THE DECISION OF THE SPECIAL
mortgagor, Poncio would not have surrendered his Republic Savings Bank to redeem the mortgage. DIVISION OF FIVE OF THE COURT OF APPEALS OF
mortgage passbook to' Carbonell. OCTOBER 30, 1968 IS HEREBY REVERSED;
It appearing that the Infantes are possessors in bad faith, PETITIONER ROSARIO CARBONELL IS HEREBY
IV their rights to the improvements they introduced op the DECLARED TO HAVE THE SUPERIOR RIGHT TO THE
disputed lot are governed by Articles 546 and 547 of the LAND IN QUESTION AND IS HEREBY DIRECTED TO
IDENTIFICATION AND DESCRIPTION OF THE New Civil Code. Their expenses consisting of P1,500.00 REIMBURSE TO PRIVATE RESPONDENTS INFANTES
DISPUTED LOT IN THE MEMORANDUM EXHIBIT "A" for draining the property, filling it with 500 cubic meters of THE SUM OF ONE THOUSAND FIVE HUNDRED
garden soil, building a wall around it and installing a gate PESOS (P1,500.00) WITHIN THREE (3) MONTHS FROM
The claim that the memorandum Exhibit "A" does not and P11,929.00 for erecting a b ' bungalow thereon, are THE FINALITY OF THIS DECISION; AND THE
sufficiently describe the disputed lot as the subject matter useful expenditures, for they add to the value of the REGISTER OF DEEDS OF RIZAL IS HEREBY
of the sale, was correctly disposed of in the first decision property (Aringo vs. Arenas, 14 Phil. 263; Alburo vs. DIRECTED TO CANCEL TRANSFER CERTIFICATE OF
of the trial court of December 5, 1962, thus: "The Villanueva, 7 Phil. 277; Valencia vs. Ayala de Roxas, 13 TITLE NO. 37842 ISSUED IN FAVOR OF PRIVATE
defendant argues that there is even no description of the Phil. 45). RESPONDENTS INFANTES COVERING THE
lot referred to in the note (or memorandum), especially DISPUTED LOT, WHICH CANCELLED TRANSFER
when the note refers to only one-half lot. With respect to Under the second paragraph of Article 546, the possessor CERTIFICATE OF TITLE NO. 5040 IN THE NAME OF
the latter argument of the defendant, plaintiff points out in good faith can retain the useful improvements unless JOSE PONCIO, AND TO ISSUE A NEW TRANSFER
that one- half lot was mentioned in Exhibit 'A' because the the person who defeated him in his possession refunds CERTIFICATE OF TITLE IN FAVOR OF PETITIONER
original description carried in the title states that it was him the amount of such useful expenses or pay him the ROSARIO CARBONELL UPON PRESENTATION OF
formerly part of a bigger lot and only segregated later. The increased value the land may have acquired by reason PROOF OF PAYMENT BY HER TO THE INFANTES OF
explanation is tenable, in (sic) considering the time value thereof. Under Article 547, the possessor in good faith has THE AFORESAID AMOUNT OF ONE THOUSAND FIVE
of the contents of Exh. 'A', the court has arrived at the also the right to remove the useful improvements if such HUNDRED PESOS (P1,500.00).
conclusion that there is sufficient description of the lot removal can be done without damage to the land, unless
referred to in Exh. As none other than the parcel of lot the person with the superior right elects to pay for the PRIVATE RESPONDENTS INFANTES MAY REMOVE
occupied by the defendant Poncio and where he has his useful improvements or reimburse the expenses therefor THEIR AFOREMENTIONED USEFUL IMPROVEMENTS
improvements erected. The Identity of the parcel of land under paragraph 2 of Article 546. These provisions seem FROM THE LOT WITHIN THREE (3) MONTHS FROM
involved herein is sufficiently established by the contents to imply that the possessor in bad faith has neither the THE FINALITY OF THIS DECISION, UNLESS THE
of the note Exh. 'A'. For a while, this court had that similar right of retention of useful improvements nor the right to a PETITIONER ROSARIO CARBONELL ELECTS TO
impression but after a more and through consideration of refund for useful expenses. ACQUIRE THE SAME AND PAYS THE INFANTES THE
the context in Exh. 'A' and for the reasons stated above, AMOUNT OF THIRTEEN THOUSAND FOUR HUNDRED
the court has arrived to (sic) the conclusion stated earlier" But, if the lawful possessor can retain the improvements TWENTY-NINE PESOS (P13,429.00) WITHIN THREE (3)
(pp. 53-54, ROA). introduced by the possessor in bad faith for pure luxury or MONTHS FROM THE FINALITY OF THIS DECISION.
mere pleasure only by paying the value thereof at the time SHOULD PETITIONER CARBONELL FAIL TO PAY THE
Moreover, it is not shown that Poncio owns another parcel he enters into possession (Article 549 NCC), as a matter SAID AMOUNT WITHIN THE AFORESTATED PERIOD
with the same area, adjacent to the lot of his cousin of equity, the Infantes, although possessors in bad faith, OF THREE (3) MONTHS FROM THE FINALITY OF THIS
Carbonell and likewise mortgaged by him to the Republic should be allowed to remove the aforesaid improvements, DECISION, THE PERIOD OF THREE (3) MONTHS
Savings Bank. The transaction therefore between Poncio unless petitioner Carbonell chooses to pay for their value WITHIN WHICH THE RESPONDENTS INFANTES MAY
and Carbonell can only refer and does refer to the lot at the time the Infantes introduced said useful REMOVE THEIR AFOREMENTIONED USEFUL
involved herein. If Poncio had another lot to remove his improvements in 1955 and 1959. The Infantes cannot IMPROVEMENTS SHALL COMMENCE FROM THE
house, Exhibit A would not have stipulated to allow him to claim reimbursement for the current value of the said EXPIRATION OF THE THREE (3) MONTHS GIVEN
stay in the sold lot without paying any rent for one year useful improvements; because they have been enjoying PETITIONER CARBONELL TO PAY FOR THE SAID
and thereafter to pay rental in case he cannot find another such improvements for about two decades without paying USEFUL IMPROVEMENTS.
place to transfer his house. any rent on the land and during which period herein
petitioner Carbonell was deprived of its possession and WITH COSTS AGAINST PRIVATE RESPONDENTS.
While petitioner Carbonell has the superior title to the lot, use.
she must however refund to respondents Infantes the

177
Sales – Chapter 3-12 Cases
Republic of the Philippines Nos. 5665 and 7967, all located at Daanbantayan, Cebu, where a big part of it was used to foot his hospital and
*HOSPICIO D. ROSAROSO, ANTONIO D. ROSAROSO, in their favor.5 medical expenses.11
MANUEL D. ROSAROSO, ALGERICA D. ROSAROSO,
and CLEOFE R. LABINDAO, Petitioners, They also alleged that, despite the fact that the said Respondent Meridian, in its Answer with Compulsory
vs. properties had already been sold to them, respondent Counterclaim, averred that Luis was fully aware of the
LUCILA LABORTE SORIA, SPOUSES HAM SOLUTAN Laila, in conspiracy with her mother, Lucila, obtained the conveyances he made. In fact, Sophia Sanchez
and **LAILA SOLUTAN, and MERIDIAN REALTY Special Power of Attorney (SPA),6 dated April 3, 1993, (Sanchez), Vice-President of the corporation, personally
CORPORATION, Respondents. from Luis (First SPA); that Luis was then sick, infirm, blind, witnessed Luis affix his thumb mark on the deed of sale in
and of unsound mind; that Lucila and Laila accomplished its favor. As to petitioners’ contention that Meridian acted
This is a petition for review on certiorari under Rule 45 of this by affixing Luis’ thumb mark on the SPA which in bad faith when it did not endeavor to make some
the Rules of Court assailing the December 4, 2009 purportedly authorized Laila to sell and convey, among inquiries as to the status of the properties in question, it
Decision1 of the Court of Appeals (CA). in CA G.R. CV others, Lot Nos. 8, 22 and 23, which had already been countered that before purchasing the properties, it
No. 00351, which reversed and set aside the July 30, sold to them; and that on the strength of another SPA7 by checked the titles of the said lots with the Register of
2004 Decision2 of the Regional Trial Court, Branch 8, 7th Luis, dated July 21, 1993 (Second SPA), respondents Deeds of Cebu and discovered therein that the First Sale
Judicial Region, Cebu City (RTC), in Civil Case No. CEB- Laila and Ham mortgaged Lot No. 19 to Vital Lending purportedly executed in favor of the plaintiffs was not
16957, an action for declaration of nullity of documents. Investors, Inc. for and in consideration of the amount of registered with the said Register of Deeds. Finally, it
₱150,000.00 with the concurrence of Lourdes.8 argued that the suit against it was filed in bad faith.12
The Facts
Petitioners further averred that a second sale took place On her part, Lourdes posited that her signature as well as
Spouses Luis Rosaroso (Luis) and Honorata Duazo on August 23, 1994, when the respondents made Luis that of Luis appearing on the deed of sale in favor of
(Honorata) acquired several real properties in Daan sign the Deed of Absolute Sale9 conveying to Meridian petitioners, was obtained through fraud, deceit and
Bantayan, Cebu City, including the subject properties. The three (3) parcels of residential land for ₱960,500.00 trickery. She explained that they signed the prepared
couple had nine (9) children namely: Hospicio, Arturo, (Second Sale); that Meridian was in bad faith when it did deed out of pity because petitioners told them that it was
Florita, Lucila, Eduardo, Manuel, Cleofe, Antonio, and not make any inquiry as to who were the occupants and necessary for a loan application. In fact, there was no
Angelica. On April 25, 1952, Honorata died. Later on, Luis owners of said lots; and that if Meridian had only consideration involved in the First Sale. With respect to
married Lourdes Pastor Rosaroso (Lourdes). investigated, it would have been informed as to the true the Second Sale, she never encouraged the same and
status of the subject properties and would have desisted neither did she participate in it. It was purely her
On January 16, 1995, a complaint for Declaration of in pursuing their acquisition. husband’s own volition that the Second Sale materialized.
Nullity of Documents with Damages was filed by Luis, as She, however, affirmed that she received Meridian’s
one of the plaintiffs, against his daughter, Lucila R. Soria Petitioners, thus, prayed that they be awarded moral payment on behalf of her husband who was then
(Lucila); Lucila’s daughter, Laila S. Solutan (Laila); and damages, exemplary damages, attorney’s fees, actual bedridden.13
Meridian Realty Corporation (Meridian). Due to Luis’ damages, and litigation expenses and that the two SPAs
untimely death, however, an amended complaint was filed and the deed of sale in favor of Meridian be declared null RTC Ruling
on January 6, 1996, with the spouse of Laila, Ham and void ab initio.10
Solutan (Ham); and Luis’ second wife, Lourdes, included After the case was submitted for decision, the RTC ruled
as defendants.3 On their part, respondents Lucila and Laila contested the in favor of petitioners. It held that when Luis executed the
First Sale in favor of petitioners. They submitted that even second deed of sale in favor of Meridian, he was no
In the Amended Complaint, it was alleged by petitioners assuming that it was valid, petitioners were estopped from longer the owner of Lot Nos. 19, 22 and 23 as he had
Hospicio D. Rosaroso, Antonio D. Rosaroso (Antonio), questioning the Second Sale in favor of Meridian because already sold them to his children by his first marriage. In
Angelica D. Rosaroso (Angelica), and Cleofe R. Labindao they failed not only in effecting the necessary transfer of fact, the subject properties had already been delivered to
(petitioners) that on November 4, 1991, Luis, with the full the title, but also in annotating their interests on the titles the vendees who had been living there since birth and so
knowledge and consent of his second wife, Lourdes, of the questioned properties. With respect to the assailed had been in actual possession of the said properties. The
executed the Deed of Absolute Sale4 (First Sale) covering SPAs and the deed of absolute sale executed by Luis, trial court stated that although the deed of sale was not
the properties with Transfer Certificate of Title (TCT) No. they claimed that the documents were valid because he registered, this fact was not prejudicial to their interest. It
31852 (Lot No. 8); TCT. No. 11155 (Lot 19); TCT No. was conscious and of sound mind and body when he was of the view that the actual registration of the deed of
10885 (Lot No. 22); TCT No. 10886 (Lot No. 23); and Lot executed them. In fact, it was Luis together with his wife sale was not necessary to render a contract valid and
who received the check payment issued by Meridian effective because where the vendor delivered the
178
Sales – Chapter 3-12 Cases
possession of the parcel of land to the vendee and no Second Sale, he should have revoked not only the First
superior rights of third persons had intervened, the On appeal, the CA reversed and set aside the RTC SPA but also the Second SPA. The latter being valid, all
efficacy of said deed was not destroyed. In other words, decision. The CA ruled that the first deed of sale in favor transactions emanating from it, particularly the mortgage
Luis lost his right to dispose of the said properties to of petitioners was void because they failed to prove that of Lot 19, its subsequent redemption and its second sale,
Meridian from the time he executed the first deed of sale they indeed tendered a consideration for the four (4) were valid.20 Thus, the CA disposed in this wise:
in favor of petitioners. The same held true with his alleged parcels of land. It relied on the testimony of Lourdes that
sale of Lot 8 to Lucila Soria.14 Specifically, the dispositive petitioners did not pay her husband. The price or WHEREFORE, the appeal is hereby GRANTED. The
portion of the RTC decision reads: consideration for the sale was simulated to make it appear Decision dated 30 July 2004 is hereby REVERSED AND
that payment had been tendered when in fact no payment SET ASIDE, and in its stead a new decision is hereby
IN VIEW OF THE FOREGOING, the Court finds that a was made at all.16 rendered:
preponderance of evidence exists in favor of the plaintiffs
and against the defendants. Judgment is hereby With respect to the validity of the Second Sale, the CA 1. DECLARING the Special Power of Attorney, dated 21
rendered: stated that it was valid because the documents were July 1993, as valid;
notarized and, as such, they enjoyed the presumption of
a. Declaring that the Special Power of Attorney, Exhibit regularity. Although petitioners alleged that Luis was 2. DECLARING the Special Power of Attorney, dated 03
"K," for the plaintiffs and Exhibit "3" for the defendants null manipulated into signing the SPAs, the CA opined that April 1993, as valid up to the time of its revocation on 24
and void including all transactions subsequent thereto and evidence was wanting in this regard. Dr. Arlene Letigio November 1994;
all proceedings arising therefrom; Pesquira, the attending physician of Luis, testified that
while the latter was physically infirmed, he was of sound 3. DECLARING the Deed of Absolute sale, dated 04
b. Declaring the Deed of Sale marked as Exhibit "E" valid mind when he executed the first SPA.17 November 1991, as ineffective and without any force and
and binding; effect;
With regard to petitioners’ assertion that the First SPA
c. Declaring the Deed of Absolute Sale of Three (3) was revoked by Luis when he executed the affidavit, 4. DECLARING the Deed of Absolute Sale of Three (3)
Parcels of Residential Land marked as Exhibit "F" null and dated November 24, 1994, the CA ruled that the Second Parcels of Residential Land, dated 23 August 1994, valid
void from the beginning; Sale remained valid. The Second Sale was transacted on and binding from the very beginning;
August 23, 1994, before the First SPA was revoked. In
d. Declaring the Deed of Sale, Exhibit "16" (Solutan) or other words, when the Second Sale was consummated, 5. DECLARING the Deed of Absolute Sale, dated 27
Exhibit "FF," null and void from the beginning; the First SPA was still valid and subsisting. Thus, September 1994, also valid and binding from the very
"Meridian had all the reasons to rely on the said SPA beginning;
e. Declaring the vendees named in the Deed of Sale during the time of its validity until the time of its actual
marked as Exhibit "E" to be the lawful, exclusive and filing with the Register of Deeds considering that 6. ORDERING the substituted plaintiffs to pay jointly and
absolute owners and possessors of Lots Nos. 8, 19, 22, constructive notice of the revocation of the SPA only came severally the defendant-appellant Meridian Realty
and 23; into effect upon the filing of the Adverse Claim and the Corporation the sum of Php100,000.00 as moral
aforementioned Letters addressed to the Register of damages, Php100,000.00 as attorney’s fee and
f. Ordering the defendants to pay jointly and severally Deeds on 17 December 1994 and 25 November 1994, Php100,000.00 as litigation expenses; and
each plaintiff ₱50,000.00 as moral damages; and respectively, informing the Register of Deeds of the
revocation of the first SPA."18 Moreover, the CA observed 7. ORDERING the substituted plaintiffs to pay jointly and
g. Ordering the defendants to pay plaintiffs ₱50,000.00 as that the affidavit revoking the first SPA was also revoked severally the defendant-appellants Leila Solutan et al., the
attorney’s fees; and ₱20,000.00 as litigation expenses. by Luis on December 12, 1994.19 sum of Php50,000.00 as moral damages.

The crossclaim made by defendant Meridian Realty Furthermore, although Luis revoked the First SPA, he did SO ORDERED.21
Corporation against defendants Soria and Solutan is not revoke the Second SPA which authorized respondent
ordered dismissed for lack of sufficient evidentiary basis. Laila to sell, convey and mortgage, among others, the Petitioners filed a motion for reconsideration, but it was
property covered by TCT T-11155 (Lot No. 19). The CA denied in the CA Resolution,22 dated November 18,
SO ORDERED."15 opined that had it been the intention of Luis to discredit 2010. Consequently, they filed the present petition with
the the following ASSIGNMENT OF ERRORS
Ruling of the Court of Appeals
179
Sales – Chapter 3-12 Cases
I. order to overthrow a certificate of a notary public to the was never contested by the respondents. What they
effect that a grantor executed a certain document and vehemently insist, though, is that the said sale was
THE HONORABLE COURT OF APPEALS (19TH acknowledged the fact of its execution before him, mere simulated because the purported sale was made without a
DIVISION) GRAVELY ERRED WHEN IT DECLARED AS preponderance of evidence will not suffice. Rather, the valid consideration.
VOID THE FIRST SALE EXECUTED BY THE LATE LUIS evidence must (be) so clear, strong and convincing as to
ROSAROSO IN FAVOR OF HIS CHILDREN OF HIS exclude all reasonable dispute as to the falsity of the Under Section 3, Rule 131 of the Rules of Court, the
FIRST MARRIAGE. certificate. When the evidence is conflicting, the certificate following are disputable presumptions: (1) private
will be upheld x x x . transactions have been fair and regular; (2) the ordinary
II. course of business has been followed; and (3) there was
A notarial document is by law entitled to full faith and sufficient consideration for a contract.29 These
THE HONORABLE COURT OF APPEALS GRAVELY credit upon its face. (Ramirez vs. Ner, 21 SCRA 207). As presumptions operate against an adversary who has not
ERRED IN NOT SUSTAINING AND AFFIRMING THE such it … must be sustained in full force and effect so long introduced proof to rebut them. They create the necessity
RULING OF THE TRIAL COURT DECLARING THE as he who impugns it shall not have presented strong, of presenting evidence to rebut the prima facie case they
MERIDIAN REALTY CORPORATION A BUYER IN BAD complete and conclusive proof of its falsity or nullity on created, and which, if no proof to the contrary is presented
FAITH, DESPITE THE TRIAL COURT’S FINDINGS THAT account of some flaw or defect provided against by law and offered, will prevail. The burden of proof remains
THE DEED OF SALE (First Sale), IS GENUINE AND HAD (Robinson vs. Villafuerte, 18 Phil. 171, 189-190).25 where it is but, by the presumption, the one who has that
FULLY COMPLIED WITH ALL THE LEGAL burden is relieved for the time being from introducing
FORMALITIES. Furthermore, petitioners aver that it was erroneous for the evidence in support of the averment, because the
CA to say that the records of the case were bereft of presumption stands in the place of evidence unless
III. evidence that they paid the price of the lots sold to them. rebutted.30
In fact, a perusal of the records would reveal that during
THE HONORABLE COURT OF APPEALS FURTHER the cross-examination of Antonio Rosaroso, when asked if In this case, the respondents failed to trounce the said
ERRED IN NOT HOLDING THE SALE (DATED 27 there was a monetary consideration, he testified that they presumption. Aside from their bare allegation that the sale
SEPTEMBER 1994), NULL AND VOID FROM THE VERY indeed paid their father and their payment helped him was made without a consideration, they failed to supply
BEGINNING SINCE LUIS ROSAROSO ON NOVEMBER sustain his daily needs.26 clear and convincing evidence to back up this claim. It is
4, 1991 WAS NO LONGER THE OWNER OF LOTS 8, elementary in procedural law that bare allegations,
19, 22 AND 23 AS HE HAD EARLIER DISPOSED SAID Petitioners also assert that Meridian was a buyer in bad unsubstantiated by evidence, are not equivalent to proof
LOTS IN FAVOR OF THE CHILDREN OF HIS (LUIS faith because when its representative visited the site, she under the Rules of Court.31
ROSAROSO) FIRST MARRIAGE.23 did not make the necessary inquiries. The fact that there
were already houses on the said lots should have put The CA decision ran counter to this established rule
Petitioners argue that the second deed of sale was null Meridian on its guard and, for said reason, should have regarding disputable presumption. It relied heavily on the
and void because Luis could not have validly transferred made inquiries as to who owned those houses and what account of Lourdes who testified that the children of Luis
the ownership of the subject properties to Meridian, he their rights were over the same.27 approached him and convinced him to sign the deed of
being no longer the owner after selling them to his sale, explaining that it was necessary for a loan
children. No less than Atty. William Boco, the lawyer who Meridian’s assertion that the Second Sale was registered application, but they did not pay the purchase price for the
notarized the first deed of sale, appeared and testified in in the Register of Deeds was a falsity. The subject titles, subject properties.32 This testimony, however, is self-
court that the said deed was the one he notarized and that namely: TCT No. 11155 for Lot 19, TCT No. 10885 for Lot serving and would not amount to a clear and convincing
Luis and his second wife, Lourdes, signed the same 22, and TCT No. 10886 for Lot 23 were free from any evidence required by law to dispute the said presumption.
before him. He also identified the signatures of the annotation of the alleged sale.28 As such, the presumption that there was sufficient
subscribing witnesses.24 Thus, they invoke the finding of consideration will not be disturbed.
the RTC which wrote: After an assiduous assessment of the records, the Court
finds for the petitioners. Granting that there was no delivery of the consideration,
In the case of Heirs of Joaquin Teves, Ricardo Teves the seller would have no right to sell again what he no
versus Court of Appeals, et al., G.R. No. 109963, October The First Deed Of Sale Was Valid longer owned. His remedy would be to rescind the sale for
13, 1999, the Supreme Court held that a public document failure on the part of the buyer to perform his part of their
executed [with] all the legal formalities is entitled to a The fact that the first deed of sale was executed, obligation pursuant to Article 1191 of the New Civil Code.
presumption of truth as to the recitals contained therein. In conveying the subject properties in favor of petitioners, In the case of Clara M. Balatbat v. Court Of Appeals and
180
Sales – Chapter 3-12 Cases
Spouses Jose Repuyan and Aurora Repuyan,33 it was requirement of the law then is two-fold: acquisition in good an otherwise reasonable man is not an innocent
written: faith and registration in good faith. Good faith must concur purchaser for value. A purchaser cannot close his eyes to
with the registration. If it would be shown that a buyer was facts which should put a reasonable man upon his guard,
The failure of the buyer to make good the price does not, in bad faith, the alleged registration they have made and then claim that he acted in good faith under the belief
in law, cause the ownership to revest to the seller unless amounted to no registration at all. that there was no defect in the title of the vendor. As we
the bilateral contract of sale is first rescinded or resolved have held:
pursuant to Article 1191 of the New Civil Code. Non- The principle of primus tempore, potior jure (first in time,
payment only creates a right to demand the fulfillment of stronger in right) gains greater significance in case of a The failure of appellees to take the ordinary precautions
the obligation or to rescind the contract. [Emphases double sale of immovable property. When the thing sold which a prudent man would have taken under the
supplied] twice is an immovable, the one who acquires it and first circumstances, specially in buying a piece of land in the
records it in the Registry of Property, both made in good actual, visible and public possession of another person,
Meridian is Not a faith, shall be deemed the owner. Verily, the act of other than the vendor, constitutes gross negligence
Buyer in Good Faith registration must be coupled with good faith— that is, the amounting to bad faith.
registrant must have no knowledge of the defect or lack of
Respondents Meridian and Lucila argue that, granting that title of his vendor or must not have been aware of facts In this connection, it has been held that where, as in this
the First Sale was valid, the properties belong to them as which should have put him upon such inquiry and case, the land sold is in the possession of a person other
they acquired these in good faith and had them first investigation as might be necessary to acquaint him with than the vendor, the purchaser is required to go beyond
recorded in the Registry of Property, as they were the defects in the title of his vendor.)35 [Emphases and the certificate of title to make inquiries concerning the
unaware of the First Sale.34 underlining supplied] rights of the actual possessor. Failure to do so would
make him a purchaser in bad faith. (Citations omitted).
Again, the Court is not persuaded. When a piece of land is in the actual possession of
persons other than the seller, the buyer must be wary and One who purchases real property which is in the actual
The fact that Meridian had them first registered will not should investigate the rights of those in possession. possession of another should, at least make some inquiry
help its cause. In case of double sale, Article 1544 of the Without making such inquiry, one cannot claim that he is a concerning the right of those in possession. The actual
Civil Code provides: buyer in good faith. When a man proposes to buy or deal possession by other than the vendor should, at least put
with realty, his duty is to read the public manuscript, that the purchaser upon inquiry. He can scarely, in the
ART. 1544. If the same thing should have been sold to is, to look and see who is there upon it and what his rights absence of such inquiry, be regarded as a bona fide
different vendees, the ownership shall be transferred to are. A want of caution and diligence, which an honest man purchaser as against such possessors. (Emphases
the person who may have first possession thereof in good of ordinary prudence is accustomed to exercise in making supplied)
faith, if it should be movable property. purchases, is in contemplation of law, a want of good
faith. The buyer who has failed to know or discover that Prescinding from the foregoing, the fact that private
Should it be immovable property, the ownership shall the land sold to him is in adverse possession of another is respondent RRC did not investigate the Sarmiento
belong to the person acquiring it who in good faith first a buyer in bad faith.36 In the case of Spouses Sarmiento spouses' claim over the subject land despite its
recorded it in the Registry of Property. v. Court of Appeals,37 it was written: knowledge that Pedro Ogsiner, as their overseer, was in
actual possession thereof means that it was not an
Should there be no inscription, the ownership shall pertain Verily, every person dealing with registered land may innocent purchaser for value upon said land. Article 524 of
to the person who in good faith was first in possession; safely rely on the correctness of the certificate of title the Civil Code directs that possession may be exercised in
and, in the absence thereof; to the person who presents issued therefor and the law will in no way oblige him to go one's name or in that of another. In herein case, Pedro
the oldest title, provided there is good faith. behind the certificate to determine the condition of the Ogsiner had informed RRC that he was occupying the
property. Thus, the general rule is that a purchaser may subject land on behalf of the Sarmiento spouses. Being a
Otherwise stated, ownership of an immovable property be considered a purchaser in good faith when he has corporation engaged in the business of buying and selling
which is the subject of a double sale shall be transferred: examined the latest certificate of title. An exception to this real estate, it was gross negligence on its part to merely
(1) to the person acquiring it who in good faith first rule is when there exist important facts that would create rely on Mr. Puzon's assurance that the occupants of the
recorded it in the Registry of Property; (2) in default suspicion in an otherwise reasonable man to go beyond property were mere squatters considering the invaluable
thereof, to the person who in good faith was first in the present title and to investigate those that preceded it. information it acquired from Pedro Ogsiner and
possession; and (3) in default thereof, to the person who Thus, it has been said that a person who deliberately considering further that it had the means and the
presents the oldest title, provided there is good faith. The ignores a significant fact which would create suspicion in
181
Sales – Chapter 3-12 Cases
opportunity to investigate for itself the accuracy of such that Luis still owned them. Simply, Meridian Realty failed Lorenzo Sumulong, R. B. Hilao and B. S. Felipe for
information. [Emphases supplied] to exercise the due diligence required by law of intervenor-appellee.
purchasers in acquiring a piece of land in the possession REGALA, J.:
In another case, it was held that if a vendee in a double of person or persons other than the seller. This is an appeal, on purely legal questions, from a
sale registers the sale after he has acquired knowledge of decision of the Court of First Instance of Quezon City,
a previous sale, the registration constitutes a registration In this regard, great weight is accorded to the findings of Branch IV, declaring the intervenor-appellee, Teodoro
in bad faith and does not confer upon him any right. If the fact of the RTC. Basic is the rule that the trial court is in a Santos, entitled to the possession of the car in dispute.
registration is done in bad faith, it is as if there is no better position to examine real evidence as well as to The records before this Court disclose that sometime in
registration at all, and the buyer who has first taken observe the demeanor of witnesses who testify in the May, 1959, Teodoro Santos advertised in two
possession of the property in good faith shall be case.40 metropolitan papers the sale of his FORD FAIRLANE 500.
preferred.38 In the afternoon of May 28, 1959, a certain L. De Dios,
WHEREFORE, the petition is GRANTED. The December claiming to be a nephew of Vicente Marella, went to the
In the case at bench, the fact that the subject properties 4, 2009 Decision and the November 18, 201 0 Resolution Santos residence to answer the ad. However, Teodoro
were already in the possession of persons other than Luis of the Court of Appeals, in CA-G.R. CV No. 00351, are Santos was out during this call and only the latter's son,
was never disputed. Sanchez, representative and witness REVERSED and SET ASIDE. The July 30, 2004 Decision Irineo Santos, received and talked with De Dios. The latter
for Meridian, even testified as follows: of the Regional Trial Court, Branch 8, 7th Judicial Region, told the young Santos that he had come in behalf of his
Cebu City, in Civil Case No. CEB-16957, is hereby uncle, Vicente Marella, who was interested to buy the
x x x; that she together with the two agents, defendant REINSTATED. advertised car.
Laila Solutan and Corazon Lua, the president of Meridian On being informed of the above, Teodoro Santos
Realty Corporation, went immediately to site of the lots; SO ORDERED. instructed his son to see the said Vicente Marella the
that the agents brought with them the three titles of the following day at his given address: 1642 Crisostomo
lots and Laila Solutan brought with her a special power of Street, Sampaloc, Manila. And so, in the morning of May
attorney executed by Luis B. Rosaroso in her favor but 29, 1959, Irineo Santos went to the above address. At this
she went instead directly to Luis Rosaroso to be sure; that meeting, Marella agreed to buy the car for P14,700.00 on
the lots were pointed to them and she saw that there were the understanding that the price would be paid only after
houses on it but she did not have any interest of the the car had been registered in his name.
houses because her interest was on the lots; that Luis Irineo Santos then fetched his father who, together with L.
Rosaroso said that the houses belonged to him; that he De Dios, went to the office of a certain Atty. Jose Padolina
owns the property and that he will sell the same because where the deed of the sale for the car was executed in
he is very sickly and he wanted to buy medicines; that she Marella's favor. The parties to the contract thereafter
requested someone to check the records of the lots in the proceeded to the Motor Vehicles Office in Quezon City
Register of Deeds; that one of the titles was mortgaged where the registration of the car in Marella's name was
and she told them to redeem the mortgage because the effected. Up to this stage of the transaction, the
corporation will buy the property; that the registered owner purchased price had not been paid.
of the lots was Luis Rosaroso; that in more or less three From the Motor Vehicles Office, Teodoro Santos returned
months, the encumbrance was cancelled and she told the to his house. He gave the registration papers and a copy
prospective sellers to prepare the deed of sale; that there of the deed of sale to his son, Irineo, and instructed him
were no encumbrances or liens in the title; that when the not to part with them until Marella shall have given the full
deed of absolute sale was prepared it was signed by the payment for the car. Irineo Santos and L. De Dios then
vendor Luis Rosaroso in their house in Opra x x x.39 G.R. No. L-18536 March 31, 1965 proceeded to 1642 Crisostomo Street, Sampaloc, Manila
(Underscoring supplied) JOSE B. AZNAR, plaintiff-appellant, where the former demanded the payment from Vicente
vs. Marella. Marella said that the amount he had on hand
From the above testimony, it is clear that Meridian, RAFAEL YAPDIANGCO, defendant-appellee; then was short by some P2,000.00 and begged off to be
through its agent, knew that the subject properties were in TEODORO SANTOS, intervenor-appellee. allowed to secure the shortage from a sister supposedly
possession of persons other than the seller. Instead of Florentino M. Guanlao for plaintiff-appellant. living somewhere on Azcarraga Street, also in Manila.
investigating the rights and interests of the persons Rafael Yapdiangco in his own behalf as defendant- Thereafter, he ordered L. De Dios to go to the said sister
occupying the said lots, however, it chose to just believe appellee. and suggested that Irineo Santos go with him. At the
182
Sales – Chapter 3-12 Cases
same time, he requested the registration papers and the property by Vicente Marella, from whom the plaintiff- in consequence of certain contracts, by tradition." As
deed of sale from Irineo Santos on the pretext that he appellant traced his right. Consequently, although the interpreted by this Court in a host of cases, by this
would like to show them to his lawyer. Trusting the good plaintiff-appellant acquired the car in good faith and for a provision, ownership is not transferred by contract merely
faith of Marella, Irineo handed over the same to the latter valuable consideration from Vicente Marella, the said but by tradition or delivery. Contracts only constitute titles
and thereupon, in the company of L. De Dios and another decision concluded, still the intervenor-appellee was or rights to the transfer or acquisition of ownership, while
unidentified person, proceeded to the alleged house of entitled to its recovery on the mandate of Article 559 of the delivery or tradition is the mode of accomplishing the
Marella's sister. New Civil Code which provides: same (Gonzales v. Rojas, 16 Phil. 51; Ocejo, Perez and
At a place on Azcarraga, Irineo Santos and L. De Dios ART. 559. The possession of movable property acquired Co. v. International Bank, 37 Phil. 631, Fidelity and
alighted from the car and entered a house while their in good faith is equivalent to title. Nevertheless, one who Deposit Co. v. Wilson, 8 Phil. 51; Kuenzle & Streiff v.
unidentified companion remained in the car. Once inside, lost any movable or has been unlawfully deprived thereof, Wacke & Chandler, 14 Phil. 610; Easton v. Diaz Co., 32
L. De Dios asked Irineo Santos to wait at the sala while he may recover it from the person in possession of the same. Phil. 180).
went inside a room. That was the last that Irineo saw of If the possessor of a movable lost or of which the owner For the legal acquisition and transfer of ownership and
him. For, after a considerable length of time waiting in vain has been unlawfully deprived, has acquired it in good faith other property rights, the thing transferred must be
for De Dios to return, Irineo went down to discover that at a public sale, the owner cannot obtain its return without delivered, inasmuch as, according to settled
neither the car nor their unidentified companion was there reimbursing the price paid therefor. jurisprudence, the tradition of the thing is a necessary and
anymore. Going back to the house, he inquired from a From this decision, Jose B. Aznar appeals. indispensable requisite in the acquisition of said
woman he saw for L. De Dios and he was told that no The issue at bar is one and simple, to wit: Between ownership by virtue of contract. (Walter Laston v. E. Diaz
such name lived or was even known therein. Whereupon, Teodoro Santos and the plaintiff-appellant, Jose B. Aznar, & Co. & the Provincial Sheriff of Albay, supra.)
Irineo Santos rushed to 1642 Crisostomo to see Marella. who has a better right to the possession of the disputed So long as property is not delivered, the ownership over it
He found the house closed and Marella gone. Finally, he automobile? is not transferred by contract merely but by delivery.
reported the matter to his father who promptly advised the We find for the intervenor-appellee, Teodoro Santos. Contracts only constitute titles or rights to the transfer or
police authorities. The plaintiff-appellant accepts that the car in question acquisition of ownership, while delivery or tradition is the
That very same day, or on the afternoon of May 29, 1959 originally belonged to and was owned by the intervenor- method of accomplishing the same, the title and the
Vicente Marella was able to sell the car in question to the appellee, Teodoro Santos, and that the latter was method of acquiring it being different in our law. (Gonzales
plaintiff-appellant herein, Jose B. Aznar, for P15,000.00. unlawfully deprived of the same by Vicente Marella. v. Roxas, 16 Phil. 51)
Insofar as the above incidents are concerned, we are However, the appellant contends that upon the facts of In the case on hand, the car in question was never
bound by the factual finding of the trial court that Jose B. this case, the applicable provision of the Civil Code is delivered to the vendee by the vendor as to complete or
Aznar acquired the said car from Vicente Marella in good Article 1506 and not Article 559 as was held by the consummate the transfer of ownership by virtue of the
faith, for a valuable consideration and without notice of the decision under review. Article 1506 provides: contract. It should be recalled that while there was indeed
defect appertaining to the vendor's title. ART. 1506. Where the seller of goods has a voidable title a contract of sale between Vicente Marella and Teodoro
While the car in question was thus in the possession of thereto, but his, title has not been voided at the time of the Santos, the former, as vendee, took possession of the
Jose B. Aznar and while he was attending to its sale, the buyer acquires a good title to the goods, subject matter thereof by stealing the same while it was in
registration in his name, agents of the Philippine provided he buys them in good faith, for value, and the custody of the latter's son.
Constabulary seized and confiscated the same in without notice of the seller's defect of title. There is no adequate evidence on record as to whether
consequence of the report to them by Teodoro Santos The contention is clearly unmeritorious. Under the Irineo Santos voluntarily delivered the key to the car to the
that the said car was unlawfully taken from him. aforequoted provision, it is essential that the seller should unidentified person who went with him and L. De Dios to
In due time, Jose B. Aznar filed a complaint for replevin have a voidable title at least. It is very clearly inapplicable the place on Azcarraga where a sister of Marella allegedly
against Captain Rafael Yapdiangco, the head of the where, as in this case, the seller had no title at all. lived. But even if Irineo Santos did, it was not the delivery
Philippine Constabulary unit which seized the car in Vicente Marella did not have any title to the property contemplated by Article 712 of the Civil Code. For then, it
question Claiming ownership of the vehicle, he prayed for under litigation because the same was never delivered to would be indisputable that he turned it over to the
its delivery to him. In the course of the litigation, however, him. He sought ownership or acquisition of it by virtue of unidentified companion only so that he may drive Irineo
Teodoro Santos moved and was allowed to intervene by the contract. Vicente Marella could have acquired Santos and De Dios to the said place on Azcarraga and
the lower court. ownership or title to the subject matter thereof only by the not to vest the title to the said vehicle to him as agent of
At the end of the trial, the lower court rendered a decision delivery or tradition of the car to him. Vicente Marella. Article 712 above contemplates that the
awarding the disputed motor vehicle to the intervenor- Under Article 712 of the Civil Code, "ownership and other act be coupled with the intent of delivering the thing. (10
appellee, Teodoro Santos. In brief, it ruled that Teodoro real rights over property are acquired and transmitted by Manresa 132)
Santos had been unlawfully deprived of his personal law, by donation, by testate and intestate succession, and
183
Sales – Chapter 3-12 Cases
The lower court was correct in applying Article 559 of the committed, cannot be applied in a case which is covered
Civil Code to the case at bar, for under it, the rule is to the by an express provision of the new Civil Code, specifically
effect that if the owner has lost a thing, or if he has been Article 559. Between a common law principle and a
unlawfully deprived of it, he has a right to recover it, not statutory provision, the latter must prevail in this
only from the finder, thief or robber, but also from third jurisdiction. (Cruz v. Pahati, supra)
persons who may have acquired it in good faith from such UPON ALL THE FOREGOING, the instant appeal is
finder, thief or robber. The said article establishes two hereby dismissed and the decision of the lower court
exceptions to the general rule of irrevindicability, to wit, affirmed in full. Costs against the appellant.
when the owner (1) has lost the thing, or (2) has been Bengzon, C.J., Bautista Angelo, Concepcion, Reyes,
unlawfully deprived thereof. In these cases, the possessor J.B.L., Barrera, Paredes, Dizon, M
cannot retain the thing as against the owner, who may
recover it without paying any indemnity, except when the
possessor acquired it in a public sale. (Del Rosario v.
Lucena, 8 Phil. 535; Varela v. Finnick, 9 Phil. 482; Varela
v. Matute, 9 Phil. 479; Arenas v. Raymundo, 19 Phil.
46. Tolentino, id., Vol. II, p. 261.)
In the case of Cruz v. Pahati, et al., 52 O.G. 3053 this
Court has already ruled
that —
Under Article 559 of the new Civil Code, a person illegally
deprived of any movable may recover it from the person in
possession of the same and the only defense the latter
may have is if he has acquired it in good faith at a public
sale, in which case, the owner cannot obtain its return
without reimbursing the price paid therefor. In the present
case, plaintiff has been illegally deprived of his car
through the ingenious scheme of defendant B to enable
the latter to dispose of it as if he were the owner thereof.
Plaintiff, therefore, can still recover possession of the car
even if it is in the possession of a third party who had
acquired it in good faith from defendant B. The maxim that Tagatac v. Jimenez
"no man can transfer to another a better title than he had TRINIDAD TAGATAC v. LIBERATO JIMENEZ
himself" obtains in the civil as well as in the common law. 1957 / Ocampo / Appeal from CFI judgment
(U.S. v. Sotelo, 28 Phil. 147) Trinidad Tagatac bought a car for $4,500 in the US, and
Finally, the plaintiff-appellant here contends that inasmuch seven months later, she brought the car to the Philippines.
as it was the intervenor-appellee who had caused the When her friend Joseph Lee came to see her, he was with
fraud to be perpetrated by his misplaced confidence on one Warner Feist who posed as a wealthy man. Seeing
Vicente Marella, he, the intervenor-appellee, should be that Tagatac seemed to believe him, he offered to buy her
made to suffer the consequences arising therefrom, car for P15,000, and Tagatac was amenable to the idea.
following the equitable principle to that effect. Suffice it to The deed of sale was made, Feist paid by means of a
say in this regard that the right of the owner to recover postdated check, and the car was delivered to Feist.
personal property acquired in good faith by another, is When Tagatac tried to encash the check, PNB refused to
based on his being dispossessed without his consent. The honor it and told her that Feist had no account in said
common law principle that where one of two innocent bank. Tagatac notified the law enforcement agencies of
persons must suffer by a fraud perpetrated by another, the the estafa committed on her by Feist, but he was not
law imposes the loss upon the party who, by his apprehended and the car disappeared.
misplaced confidence, has enabled the fraud to be
184
Sales – Chapter 3-12 Cases
Meanwhile, Feist managed to have the private deed of fall under the scope of NCC 599. 2In this case, there is a
sale notarized, so he succeeded in having the car’s valid transmission of ownership from true owner [Tagatac]
registration certificate [RC] transferred in his name. He to the swindler [Feist], considering that they had a contract
sold the car to Sanchez, who was able to transfer the RC of sale.
to his name. He offered to sell the car to defendant As long as no action is taken by the entitled party
Liberato Jimenez, who bought the car for P10,000 after [annulment / ratification], the contract of sale remains valid
investigating in the Motor Vehicles Office. Jimenez and binding. Feist acquired defective and voidable title,
delivered the car to the California Car Exchange so that it but when he sold it to Sanchez, he conferred a good title
may be displayed for sale. Masalonga offered to sell the on the latter. Jimenez bought the car from Sanchez in
car for Jimenez, so the car was transferred to the former, good faith, for value, and without notice of any defect in
but when Masalonga failed to sell it right away, he Sanchez’ title, so he acquired a good title to the car. Good
transferred it to Villanueva so he could sell it for Jimenez. title means an indefeasible title to the car, even as against
Tagatac discovered that the car was in California Car original owner Tagatac. As between two innocent parties,
Exchange’s possession, so she demanded from the the one whose acts made possible the injury must
manager for the delivery of the car, but the latter refused. shoulder the consequences thereof.
The RC was retransferred to Jimenez. ————————-
Tagatac filed a suit for the recovery of the car’s 1 Civil liability of person who is criminally liable includes

possession, and the sheriff, pursuant to a warrant of restitution of thing even though it is with a third person
seizure that Tagatac obtained, seized and impounded the who acquired it legally
2 Although possession of movable property acquired in
car, but it was delivered back to Jimenez upon his filing of
a counter-bond. The lower court held that Jimenez had good faith is equivalent to a title, one who has lost any
the right of ownership and possession over the car. movable or has been unlawfully deprived thereof may
JIMENEZ IS A PURCHASER IN GOOD FAITH; recover it from the person who possesses it.
TAGATAC NOT ENTITLED TO POSSESSION
RATIO
The disputable presumption that a person found in
possession of a thing taken in the doing of a recent
wrongful act is the taker and the doer of the whole
act does NOT apply in this case because the car was not
stolen from Tagatac, and Jimenez came into possession
of the car two months after Feist swindled Tagatac. In
addition, when Jimenez acquired the car, he had no
knowledge of any flaw in the title of the person from whom
he acquired it. It was only later that he became fully aware
that there were some questions regarding the car, when
he filed a petition to dissolve Tagatac’s search warrant
which had as its subject the car in question.
Re: Tagatac’s allegation that the lower court ignored the
judgment
convicting Feist of estafa, and that it erred in not declaring
that restitution of the swindled property must follow, SHE
IS WRONG! The lower court noted that Feist was accused
of estafa because of the check and NOT because of the
delivery of the car.
Her legal basis for the restitution of thing is RPC 104-51 .
Now the question is WON she has beenunlawfully G.R. No. 80298 April 26, 1990
deprived of her car. It seems like though, but it does not
185
Sales – Chapter 3-12 Cases
EDCA PUBLISHING & DISTRIBUTING Meanwhile, EDCA having become suspicious over a The argument that the private respondents did not acquire
CORP., petitioner, second order placed by Cruz even before clearing of his the books in good faith has been dismissed by the lower
vs. first check, made inquiries with the De la Salle College courts, and we agree. Leonor Santos first ascertained the
THE SPOUSES LEONOR and GERARDO SANTOS, where he had claimed to be a dean and was informed that ownership of the books from the EDCA invoice showing
doing business under the name and style of "SANTOS there was no such person in its employ. Further that they had been sold to Cruz, who said he was selling
BOOKSTORE," and THE COURT OF verification revealed that Cruz had no more account or them for a discount because he was in financial need.
APPEALS, respondents. deposit with the Philippine Amanah Bank, against which Private respondents are in the business of buying and
Emiliano S. Samson, R. Balderrama-Samson, Mary Anne he had drawn the payment check. 7 EDCA then went to selling books and often deal with hard-up sellers who
B. Samson for petitioner. the police, which set a trap and arrested Cruz on October urgently have to part with their books at reduced prices.
Cendana Santos, Delmundo & Cendana for private 7, 1981. Investigation disclosed his real name as Tomas To Leonor Santos, Cruz must have been only one of the
respondents. de la Peña and his sale of 120 of the books he had many such sellers she was accustomed to dealing with. It
ordered from EDCA to the private respondents. 8 is hardly bad faith for any one in the business of buying
On the night of the same date, EDCA sought the and selling books to buy them at a discount and resell
CRUZ, J.: assistance of the police in Precinct 5 at the UN Avenue, them for a profit.
The case before us calls for the interpretation of Article which forced their way into the store of the private But the real issue here is whether the petitioner has been
559 of the Civil Code and raises the particular question of respondents and threatened Leonor Santos with unlawfully deprived of the books because the check
when a person may be deemed to have been "unlawfully prosecution for buying stolen property. They seized the issued by the impostor in payment therefor was
deprived" of movable property in the hands of another. 120 books without warrant, loading them in a van dishonored.
The article runs in full as follows: belonging to EDCA, and thereafter turned them over to In its extended memorandum, EDCA cites numerous
Art. 559. The possession of movable property acquired in the petitioner. 9 cases holding that the owner who has been unlawfully
good faith is equivalent to a title. Nevertheless, one who Protesting this high-handed action, the private deprived of personal property is entitled to its recovery
has lost any movable or has been unlawfully deprived respondents sued for recovery of the books after demand except only where the property was purchased at a public
thereof, may recover it from the person in possession of for their return was rejected by EDCA. A writ of sale, in which event its return is subject to reimbursement
the same. preliminary attachment was issued and the petitioner, of the purchase price. The petitioner is begging the
If the possessor of a movable lost or of which the owner after initial refusal, finally surrendered the books to the question. It is putting the cart before the horse. Unlike in
has been unlawfully deprived has acquired it in good faith private respondents. 10 As previously stated, the petitioner the cases invoked, it has yet to be established in the case
at a public sale, the owner cannot obtain its return without was successively rebuffed in the three courts below and at bar that EDCA has been unlawfully deprived of the
reimbursing the price paid therefor. now hopes to secure relief from us. books.
The movable property in this case consists of books, To begin with, the Court expresses its disapproval of the The petitioner argues that it was, because the impostor
which were bought from the petitioner by an impostor who arbitrary action of the petitioner in taking the law into its acquired no title to the books that he could have validly
sold it to the private respondents. Ownership of the books own hands and forcibly recovering the disputed books transferred to the private respondents. Its reason is that
was recognized in the private respondents by the from the private respondents. The circumstance that it did as the payment check bounced for lack of funds, there
Municipal Trial Court, 1 which was sustained by the so with the assistance of the police, which should have was a failure of consideration that nullified the contract of
Regional Trial Court, 2 which was in turn sustained by the been the first to uphold legal and peaceful processes, has sale between it and Cruz.
Court of Appeals. 3 The petitioner asks us to declare that compounded the wrong even more deplorably. Questions The contract of sale is consensual and is perfected once
all these courts have erred and should be reversed. like the one at bar are decided not by policemen but by agreement is reached between the parties on the subject
This case arose when on October 5, 1981, a person judges and with the use not of brute force but of lawful matter and the consideration. According to the Civil Code:
identifying himself as Professor Jose Cruz placed an order writs. Art. 1475. The contract of sale is perfected at the moment
by telephone with the petitioner company for 406 books, Now to the merits there is a meeting of minds upon the thing which is the
payable on delivery. 4 EDCA prepared the corresponding It is the contention of the petitioner that the private object of the contract and upon the price.
invoice and delivered the books as ordered, for which respondents have not established their ownership of the From that moment, the parties may reciprocally demand
Cruz issued a personal check covering the purchase price disputed books because they have not even produced a performance, subject to the provisions of the law
of P8,995.65. 5 On October 7, 1981, Cruz sold 120 of the receipt to prove they had bought the stock. This is governing the form of contracts.
books to private respondent Leonor Santos who, after unacceptable. Precisely, the first sentence of Article 559 xxx xxx xxx
verifying the seller's ownership from the invoice he provides that "the possession of movable property Art. 1477. The ownership of the thing sold shall be
showed her, paid him P1,700.00. 6 acquired in good faith is equivalent to a title," thus transferred to the vendee upon the actual or constructive
dispensing with further proof. delivery thereof.
186
Sales – Chapter 3-12 Cases
Art. 1478. The parties may stipulate that ownership in the method of deprivation of property. In a manner of would be left holding the bag, so to speak, and would be
thing shall not pass to the purchaser until he has fully paid speaking, plaintiff-appellant was "illegally deprived" of her compelled to return the thing bought by him in good faith
the price. car, for the way by which Warner L. Feist induced her to without even the right to reimbursement of the amount he
It is clear from the above provisions, particularly the last part with it is illegal and is punished by law. But does this had paid for it.
one quoted, that ownership in the thing sold shall not pass "unlawful deprivation" come within the scope of Article 559 It bears repeating that in the case before us, Leonor
to the buyer until full payment of the purchase only if there of the New Civil Code? Santos took care to ascertain first that the books belonged
is a stipulation to that effect. Otherwise, the rule is that xxx xxx xxx to Cruz before she agreed to purchase them. The EDCA
such ownership shall pass from the vendor to the vendee . . . The fraud and deceit practiced by Warner L. Feist invoice Cruz showed her assured her that the books had
upon the actual or constructive delivery of the thing earmarks this sale as a voidable contract (Article 1390 been paid for on delivery. By contrast, EDCA was less
sold even if the purchase price has not yet been paid. N.C.C.). Being a voidable contract, it is susceptible of than cautious — in fact, too trusting in dealing with the
Non-payment only creates a right to demand payment or either ratification or annulment. If the contract is ratified, impostor. Although it had never transacted with him
to rescind the contract, or to criminal prosecution in the the action to annul it is extinguished (Article 1392, N.C.C.) before, it readily delivered the books he had ordered (by
case of bouncing checks. But absent the stipulation above and the contract is cleansed from all its defects (Article telephone) and as readily accepted his personal check in
noted, delivery of the thing sold will effectively transfer 1396, N.C.C.); if the contract is annulled, the contracting payment. It did not verify his identity although it was easy
ownership to the buyer who can in turn transfer it to parties are restored to their respective situations before enough to do this. It did not wait to clear the check of this
another. the contract and mutual restitution follows as a unknown drawer. Worse, it indicated in the sales invoice
In Asiatic Commercial Corporation v. Ang,11 the plaintiff consequence (Article 1398, N.C.C.). issued to him, by the printed terms thereon, that the books
sold some cosmetics to Francisco Ang, who in turn sold However, as long as no action is taken by the party had been paid for on delivery, thereby vesting ownership
them to Tan Sit Bin. Asiatic not having been paid by Ang, entitled, either that of annulment or of ratification, the in the buyer.
it sued for the recovery of the articles from Tan, who contract of sale remains valid and binding. When plaintiff- Surely, the private respondent did not have to go beyond
claimed he had validly bought them from Ang, paying for appellant Trinidad C. Tagatac delivered the car to Feist by that invoice to satisfy herself that the books being offered
the same in cash. Finding that there was no conspiracy virtue of said voidable contract of sale, the title to the car for sale by Cruz belonged to him; yet she did. Although
between Tan and Ang to deceive Asiatic the Court of passed to Feist. Of course, the title that Feist acquired the title of Cruz was presumed under Article 559 by his
Appeals declared: was defective and voidable. Nevertheless, at the time he mere possession of the books, these being movable
Yet the defendant invoked Article 464 12 of the Civil Code sold the car to Felix Sanchez, his title thereto had not property, Leonor Santos nevertheless demanded more
providing, among other things that "one who has been been avoided and he therefore conferred a good title on proof before deciding to buy them.
unlawfully deprived of personal property may recover it the latter, provided he bought the car in good faith, for It would certainly be unfair now to make the private
from any person possessing it." We do not believe that the value and without notice of the defect in Feist's title respondents bear the prejudice sustained by EDCA as a
plaintiff has been unlawfully deprived of the cartons of (Article 1506, N.C.C.). There being no proof on record that result of its own negligence.1âwphi1 We cannot see the
Gloco Tonic within the scope of this legal provision. It has Felix Sanchez acted in bad faith, it is safe to assume that justice in transferring EDCA's loss to the Santoses who
voluntarily parted with them pursuant to a contract of he acted in good faith. had acted in good faith, and with proper care, when they
purchase and sale. The circumstance that the price was The above rulings are sound doctrine and reflect our own bought the books from Cruz.
not subsequently paid did not render illegal a transaction interpretation of Article 559 as applied to the case before While we sympathize with the petitioner for its plight, it is
which was valid and legal at the beginning. us. clear that its remedy is not against the private
In Tagatac v. Jimenez,13 the plaintiff sold her car to Feist, Actual delivery of the books having been made, Cruz respondents but against Tomas de la Peña, who has
who sold it to Sanchez, who sold it to Jimenez. When the acquired ownership over the books which he could then apparently caused all this trouble. The private
payment check issued to Tagatac by Feist was validly transfer to the private respondents. The fact that he respondents have themselves been unduly
dishonored, the plaintiff sued to recover the vehicle from had not yet paid for them to EDCA was a matter between inconvenienced, and for merely transacting a customary
Jimenez on the ground that she had been unlawfully him and EDCA and did not impair the title acquired by the deal not really unusual in their kind of business. It is they
deprived of it by reason of Feist's deception. In ruling for private respondents to the books. and not EDCA who have a right to complain.
Jimenez, the Court of Appeals held: One may well imagine the adverse consequences if the WHEREFORE, the challenged decision is AFFIRMED and
The point of inquiry is whether plaintiff-appellant Trinidad phrase "unlawfully deprived" were to be interpreted in the the petition is DENIED, with costs against the petitioner.
C. Tagatac has been unlawfully deprived of her car. At manner suggested by the petitioner. A person relying on Narvasa, Gancayco, Griño-Aquino and Medialdea, JJ.,
first blush, it would seem that she was unlawfully deprived the seller's title who buys a movable property from him concur.
thereof, considering that she was induced to part with it by would have to surrender it to another person claiming to
reason of the chicanery practiced on her by Warner L. be the original owner who had not yet been paid the
Feist. Certainly, swindling, like robbery, is an illegal purchase price therefor. The buyer in the second sale
187
Sales – Chapter 3-12 Cases
CHRYSLER PHILIPPINES CORPORATION, petitioner, automotive products referred to in the Complaint, and
vs. professed no knowledge of having ordered from petitioner
THE HONORABLE COURT OF APPEALS and said articles.
SAMBOK MOTORS CO. (BACOLOD), respondents, Upon a Joint Motion to Dismiss filed by petitioner and
Reyes, Santayana, Tayao & Picazo Law Office for Allied Brokerage, the Trial Court. on October 23, 1975,
petitioner. dismissed the case with prejudice against Allied
Alampay, Alvero & Alampay Law Office for private Brokerage for lack of cause of action, and also dismissed
respondent. the latter's counterclaim against petitioner.
On July 31, 1978, the Trial Court rendered its Decision
MELENCIO-HERRERA, J: dismissing the Complaint against Negros Navigation for
Subject of this Petition for Review is the Decision of the lack of cause of action, but finding Sambok, Bacolod,
then Court of Appeals in CA-G.R. No. 65328-R reversing liable for the claim of petitioner, thus:
the judgment of the then Court of First Instance of Rizal, PREMISES CONSIDERED, the Court renders judgment
Branch XX, in Civil Case No. 16624, and dismissing as follows:
petitioner Chrysler Philippines Corporation's suit for (1) The complaint against defendant Negros Navigation is
Damages against private respondent Sambok Motors dismissed for lack of cause of action.
Company (Bacolod) arising from breach of contract. (2) Defendant Sambok Motors Co. (Bacolod) is ordered to
Petitioner is a domestic corporation engaged in the pay plaintiff Chrysler Philippines Corporation:
assembling and sale of motor vehicles and other (a) The sum of Thirty-One Thousand Thirty Seven Pesos
automotive products. Respondent Sambok Motors Co., a and Fifty Six Centavos (P31,037.56) with interest at the
general partnership, during the period relevant to these rate of twelve percent (12) per annum from January 1,
proceedings, was its dealer for automotive products with 1971 until fully paid;
offices at Bacolod (Sambok, Bacolod) and Iloilo (Sambok, (b) The sum of Five Thousand Pesos as and for attorney's
Iloilo). The two offices were run by relatives. Miguel Ng fees and expenses of litigation;
was Assistant Manager for Sambok, Bacolod, while an (c) The costs of the suit.
elder brother, Pepito Ng, was the President. 1 (3) The counterclaim of defendant Negros Navigation and
On September 7, 1972, petitioner filed with the Court of Sambok Motors Co. (Bacolod) are dismissed for lack of
First Instance of Rizal, Branch XX, Pasig, Rizal, a merit.
Complaint for Damages against Allied Brokerage The case against Negros Navigation was dismissed for
Corporation, Negros Navigation Company and Sambok, failure of petitioner and Sambok, Bacolod, to file the
Bacolod, alleging that on October 2, 1970, Sambok, necessary notices and claims as conditions precedent for
Bacolod, ordered from petitioner various automotive a judicial action. 2
products worth P30,909.61, payable in 45 days; that on On the other hand, the Trial Court found that the act of
November 25, 1970, petitioner delivered said products to Sambok, Bacolod, "in refusing to take delivery of the
its forwarding agent, Allied Brokerage Corporation, for shipment for no justifiable reason from Negros Navigation
shipment; that Allied Brokerage loaded the goods on despite having received the Bill of Lading constituted
board the M/S Doña Florentina, a vessel owned and wrongful neglect or refusal to accept and pay for the
operated by Negros Navigation Company, for delivery to subject shipment, by reason of which defendant Sambok
Sambok, Bacolod; that when petitioner tried to collect from Motors may be held liable for damages."
the latter the amount of P31,037.56, representing the Sambok, Bacolod, appealed. On November 26, 1980,
price of the spare parts plus handling charges, Sambok, respondent Appellate Court set aside the appealed
Bacolod, refused to pay claiming that it had not received judgment and dismissed petitioner's Complaint, after
the merchandise; that petitioner also demanded the return finding that the latter had not performed its part of the
of the merchandise or their value from Allied Brokerage obligation under the contract by not delivering the goods
and Negros Navigation, but both denied any liability. at Sambok, Iloilo, the place designated in the Parts Order
In its Answer, Sambok, Bacolod, denied having received Form (Exhibits "A", "A-1" to "A-6"), and must, therefore,
from petitioner or from any of its co-defendants, the
188
Sales – Chapter 3-12 Cases
suffer the loss. In other words, respondent Appellate Court the order for spare parts was made by the President of
found. that there was misdelivery. Sambok, Pepito Ng, through its marketing consultant.
Hence, this Petition for Review on Certiorari, with the Notwithstanding, upon receipt of the Bill of Lading,
following errors assigned to respondent Court: Sambok, Bacolod, initiated, but did not pursue, steps to
I take delivery as they were advised by Negros Navigation
The Respondent Court of Appeals erred in finding that the that because some parts were missing. they would just be
issue of misshipment or misdelivery of the automotive informed as soon as the missing parts were located. 4
spare parts involved in the litigation was raised by the It was only four years later, however, or in 1974, when a
private respondent Sambok Motors Co. (Bacolod) in the warehouseman of Negros Navigation, Severino Aguarte,
Trial Court. found in their off-shore bodega, parts of the shipment.- in
II question, but already deteriorated and valueless. 5
The Respondent Court of Appeals erred in refusing to Under the circumstances, Sambok, Bacolod, cannot be
apply the provisions of Section 18, Rule 46 of the Revised faulted for not accepting or refusing to accept the
Rules of Court quoted below, that since the question of shipment from Negros Navigation four years after
misshipment or misdelivery was not raised by the private shipment. The evidence is clear that Negros Navigation
respondent in the Trial Court, this issue cannot for the first could not produce the merchandise nor ascertain its
time be raised on appeal. whereabouts at the time Sambok, Bacolod, was ready to
Section 18. Questions that may be raised on appeal. take delivery. Where the seller delivers to the buyer a
Whether or not the appellant has filed a motion for new quantity of goods less than he contracted to sell, the buyer
trial in the court below, he may include in his assignment may reject them. 6
of errors any question of law or fact that has been raised From the evidentiary record, Negros Navigation was the
in the court below and which is within the issues framed party negligent in failing to deliver the complete shipment
by the parties. either to Sambok, Bacolod, or to Sambok, Iloilo, but as the
III Trial Court found, petitioner failed to comply with the
The Respondent Court of Appeals erred in finding that the conditions precedent to the filing of a judicial action. Thus,
private respondent gave the alleged instruction to the in the last analysis, it is petitioner that must shoulder the
petitioner to ship the automotive spare parts to Iloilo City resulting loss. The general rule that before, delivery, the
and not to Bacolod City. risk of loss is home by the seller who is still the owner,
IV under the principle of "res petit domino", 7 is applicable in
The Respondent Court of Appeals erred in finding that the petitioner's case.
defendant Negros Navigation notified the private In sum, the judgment of respondent Appellate Court, will
respondent of the arrival of the shipment at Bacolod City. have to be sustained not on the basis of misdelivery but
V on non-delivery since the merchandise was never placed
The Respondent Court of Appeals erred in reversing the in the control and possession of Sambok, Bacolod, the
decision of the Trial Court that the act of the private vendee. 8
respondent in refusing to take delivery of the automotive WHEREFORE, we hereby affirm the Decision of the then
spare parts that it purchased from the petitioner after Court of Appeals in CA-G.R. No. 65328-R, without
having been notified of the shipment constitutes wrongful pronouncement as to costs.
neglect resulting in the loss of the cargo for which it
should be liable in damages to the petitioner.
To our minds, the matter of misdelivery is not the decisive
factor for relieving Sambok, Bacolod, of liability herein.
While it may be that the Parts Order Form (E exhibits "A",
"A-1" to "A-6") specifically indicated Iloilo as the
destination, as testified to by Ernesto Ordonez, Parts
Sales Representative of petitioner, 3 Sambok, Bacolod,
and Sambok, Iloilo, are actually one. In fact, admittedly,
189
Sales – Chapter 3-12 Cases
for by Tabora on May 15, 1955 in his law office Ignacio
Building, Naga City.
In the midnight of the same date, however, a big fire broke
out in that locality which destroyed and burned all the
buildings standing on one whole block including at the law
office and library of Tabora As a result, the books bought
from the company as above stated, together with Tabora's
important documents and papers, were burned during the
conflagration. This unfortunate event was immediately
reported by Tabora to the company in a letter he sent on
May 20, 1955. On May 23, the company replied and as a
token of goodwill it sent to Tabora free of charge volumes
75, 76, 77 and 78 of the Philippine Reports. As Tabora
failed to pay he monthly installments agreed upon on the
balance of the purchase price notwithstanding the long
time that had elapsed, the company demanded payment
of the installments due, and having failed, to pay the
same, it commenced the present action before the Court
of First Instance of Manila for the recovery of the balance
of the obligation. Plaintiff also prayed that defendant be
ordered to pay 25% of the amount due as liquidated
damages, and the cost of action.
Defendant, in his answer, pleaded force majeure as a
defense. He alleged that the books bought from the
plaintiff were burned during the fire that broke out in Naga
City on May 15, 1955, and since the loss was due to force
majeure he cannot be held responsible for the loss. He
prayed that the complaint be dismissed and that he be
awarded moral damages in the amount of P15,000.00.
After due hearing, the court a quo rendered judgment for
LAWYERS COOPERATIVE PUBLISHING the plaintiff. It ordered the defendant to pay the sum of
COMPANY, plaintiff-appellee, P1,382.40, with legal interest thereon from the filing of the
vs. complaint, plus a sum equivalent to 25% of the total
PERFECTO A. TABORA, defendant-appellant. amount due as liquidated damages, and the cost of
Paredes, Poblador, Cruz and Nazareno for plaintiff- action.
appellee. Defendant took the case to the Court of Appeals, but the
Tabora and Concon for defendant-appellant. same is now before us by virtue of a certification issued by
BAUTISTA ANGELO, J.: that Court that the case involves only questions of law.
On May 3, 1955, Perfecto A. Tabora bought from the Appellant bought from appellee one set of American
Lawyers Cooperative Publishing Company one complete Jurisprudence, including one set of general index, payable
set of American Jurisprudence consisting of 48 volumes on installment plan. It was provided in the contract that
with 1954 pocket parts, plus one set of American "title to and ownership of the books shall remain with the
Jurisprudence, General Index, consisting of 4 volumes, for seller until the purchase price shall have been fully paid.
a total price of P1,675.50 which, in addition to the cost of Loss or damage to the books after delivery to the buyer
freight of P6.90, makes a total of P1,682.40. Tabora made shall be borne by the buyer." The total price of the books,
a partial payment of P300.00, leaving a balance of including the cost of freight, amounts to P1,682.40.
P1,382.40. The books were duly delivered and receipted Appellant only made a down payment of P300.00 thereby
190
Sales – Chapter 3-12 Cases
leaving a balance of P1,382.40. This is now the import of pecuniary in nature, and the obligor bound himself to
the present action aside from liquidated damages. assume the loss after the delivery of the goods to him. In G.R. No. L-46306 October 27, 1939
Appellant now contends that since it was agreed that the other words, the obligor agreed to assume any risk LEVY HERMANOS, INC., plaintiff-appellant,
title to and the ownership of the books shall remain with concerning the goods from the time of their delivery, which vs.
the seller until the purchase price shall have been fully is an exception to the rule provided for in Article 1262 of LAZARO BLAS GERVACIO, defendant-appellee.
paid, and the books were burned or destroyed our Civil Code. Felipe Caniblas for appellant.
immediately after the transaction, appellee should be the Appellant likewise contends that the court a quo erred in Abreu, Lichaucco and Picazo for appellee.
one to bear the loss for, as a result, the loss is always sentencing him to pay attorney's fees. This is merely the
borne by the owner. Moreover, even assuming that the result of a misapprehension for what the court a
ownership of the books were transferred to the buyer after quo ordered appellant to pay is not 25% of the amount MORAN, J.:
the perfection of the contract the latter should not answer due as attorney's fees, but as liquidated damages, which On February 9-4, 1938, plaintiff filed a complaint in the
for the loss since the same occurred through force is in line with an express stipulation of the contract. We Court of First Instance of Manila, which substantially
majeure. Here, there is no evidence that appellant has believe, however, that the appellant should not be made recites the following facts:
contributed in any way to the occurrence of the to pay any damages because his denial to pay the On March 10, 1937, plaintiff Levy Hermanos, Inc., sold to
conflagration.1äwphï1.ñët balance of the account is not due to bad faith. defendant Lazaro Blas Gervacio, a Packard car.
This contention cannot be sustained. While as a rule the WHEREFORE, the decision appealed from is modified by Defendant, after making the initial payment, executed a
loss of the object of the contract of sale is borne by the eliminating that portion which refers to liquidated promissory note for the balance of P2,400, payable on or
owner or in case of force majeure the one under obligation damages. No costs. before June 15, 1937, with interest at 12 per cent per
to deliver the object is exempt from liability, the application annum, to secure the payment of the note, he mortgaged
of that rule does not here obtain because the law on the the car to the plaintiff. Defendant failed to pay the note it
contract entered into on the matter argues against it. It is its maturity. Wherefore, plaintiff foreclosed the mortgage
true that in the contract entered into between the parties and the car was sold at public auction, at which plaintiff
the seller agreed that the ownership of the books shall was the highest bidder for P1,800. The present action is
remain with it until the purchase price shall have been fully for the collection of the balance of P1,600 and interest.
paid, but such stipulation cannot make the seller liable in Defendant admitted the allegations of the complaint, and
case of loss not only because such was agreed merely to with this admission, the parties submitted the case for
secure the performance by the buyer of his obligation but decision. The lower court applied, the provisions of Act
in the very contract it was expressly agreed that the "loss No. 4122, inserted as articles 1454-A of the Civil Code,
or damage to the books after delivery to the buyer shall be and rendered judgment in favor of the defendant. Plaintiff
borne by the buyer." Any such stipulation is sanctioned by appealed.
Article 1504 of our Civil Code, which in part provides: Article 1454-A of the Civil Code reads as follows:
(1) Where delivery of the goods has been made to the In a contract for the sale of personal property payable in
buyer or to a bailee for the buyer, in pursuance of the installments shall confer upon the vendor the right to
contract and the ownership in the goods has been cancel the sale or foreclose the mortgage if one has been
retained by the seller merely to secure performance by the given on the property, without reimbursement to the
buyer of his obligations under the contract, the goods are purchaser of the installments already paid, if there be an
at the buyer's risk from the time of such delivery. agreement to this effect.
Neither can appellant find comfort in the claim that since However, if the vendor has chosen to foreclose the
the books were destroyed by fire without any fault on his mortgage he shall have no further action against the
part he should be relieved from the resultant obligation purchaser for the recovery of any unpaid balance owing
under the rule that an obligor should be held exempt from by the same and any agreement to the contrary shall be
liability when the loss occurs thru a fortuitous event. This null and void.
is because this rule only holds true when the obligation In Macondray and Co. vs. De Santos (33 Off. Gaz., 2170),
consists in the delivery of a determinate thing and there is we held that "in order to apply the provisions of article
no stipulation holding him liable even in case of fortuitous 1454-A of the Civil Code it must appear that there was a
event. Here these qualifications are not present. The contract for the sale of personal property payable in
obligation does not refer to a determinate thing, but is installments and that there has been a failure to pay two
191
Sales – Chapter 3-12 Cases
or more installments." The contract, in the instant case,
while a sale of personal property, is not, however, one on
installments, but on straight term, in which the balance,
after payment of the initial sum, should be paid in its
totality at the time specified in the promissory note. The
transaction is not is not, therefore, the one contemplated
in Act No. 4122 and accordingly the mortgagee is not
bound by the prohibition therein contained as to the right
to the recovery of the unpaid balance.
Undoubtedly, the law is aimed at those sales where the
price is payable in several installments, for, generally, it is
in these cases that partial payments consist in relatively
small amounts, constituting thus a great temptation for
improvident purchasers to buy beyond their means. There
is no such temptation where the price is to be paid in
cash, or, as in the instant case, partly in cash and partly in [G.R. No. 61043. September 2, 1992.]
one term, for, in the latter case, the partial payments are
not so small as to place purchasers off their guard and DELTA MOTOR SALES CORPORATION, Plaintiff-
delude them to a miscalculation of their ability to pay. The Appellee, v. NIU KIM DUAN and CHAN FUE
oretically, perhaps, there is no difference between paying ENG, Defendants-Appellants.
the price in tow installments, in so far as the size of each
partial payment is concerned; but in actual practice the Francisco C. Bonoan for Plaintiff-Appellee.
difference exists, for, according to the regular course of
business, in contracts providing for payment of the price in Agapito M. Joaquin, for Defendants-Appellants.
two installments, there is generally a provision for initial
payment. But all these considerations are immaterial, the
language of the law being so clear as to require no SYLLABUS
construction at all.lâwphi1.nêt
The suggestion that the cash payment made in this case
should be considered as an installment in order to bring 1. CIVIL LAW; SALES; TREATMENT OF THE
the contract sued upon under the operation of the law, is INSTALLMENT PAYMENTS AS RENTALS;
completely untenable. A cash payment cannot be STIPULATION IN A CONTRACT THAT THE
considered as a payment by installment, and even if it can INSTALLMENTS PAID SHALL NOT BE RETURNED TO
be so considered, still the law does not apply, for it THE VENDEE HELD VALID PROVIDED IT IS NOT
requires non-payment of two or more installments in order UNCONSCIONABLE. — Defendants-appellants cannot
that its provisions may be invoked. Here, only one complain that their downpayment of P774.00 and
installment was unpaid. installment payments of P5,655.92 were treated as rentals
Judgment is reversed, and the defendant-appellee is — even though the total amount of P6,429,92 which they
hereby sentenced to pay plaintiff-appellant the sum of had paid, approximates one-third (1/3) of the cost of the
P1,600 with interest at the rate of 12 per cent per annum three (3) air-conditioners. A stipulation in a contract that
from June 15, 1937, and the sum of P52.08 with interest the installments paid shall not be returned to the vendee is
at the rate of 6 per cent from the date of the filing of the valid insofar as the same may not be unconscionable
complaint, with costs in both instances against the under the circumstances is sanctioned by Article 1486 of
appellee. the New Civil Code. The monthly installment payable by
defendants-appellants was P774.00. The P5,655.92
installment payments correspond only to seven (7)
monthly installments. Since they admit having used the
192
Sales – Chapter 3-12 Cases
air-conditioners for twenty-two (22) months, this means The events which led to the filing of the case in the lower by the computation, Exhibit F, after deducting the interests
that they did not pay fifteen (15) monthly installments on court were summarized by the Court of Appeals, as in arrears, cover charges, replevin bond premiums, the
the said air-conditioners and were thus using the same follows:jgc:chanrobles.com.ph value of the units repossessed and the like; and, that in
FREE for said period — to the prejudice of plaintiff- view of the failure of the defendants to pay their
appellee. Under the circumstances, the treatment of the "‘On July 5, 1975, the defendants purchased from the obligations, the amount of P6,966.00 which had been paid
installment payments as rentals cannot be said to be plaintiff three (3) units of ‘DAIKIN’ air-conditioner all by way of installments were treated as rentals for the units
unconscionable. valued at P19,350.00 as evidenced by the Deed of in question for two (2) years pursuant to the provisions of
Conditional Sale, Exhibit A; that the aforesaid deed of sale paragraph 5 of the Deed of Conditional Sale, Exhibit A.’
2. REMEDIES OF THE VENDOR IN A SALE OF had the following terms and conditions:chanrob1es virtual (pp. 5-7, Record; pp. 4-6, Appellant’s Brief)." chanrobles
PERSONAL PROPERTY PAYABLE IN INSTALLMENTS; 1aw library law library
REMEDIES ARE ALTERNATIVE AND NOT
CUMULATIVE. — The vendor in a sale of personal ‘(a) the defendants shall pay a down payment of P774.00 As above-stated, the trial court ruled in favor of Plaintiff-
property payable in installments may exercise one of three and the balance of P18,576.00 shall [be] paid by them in Appellee.
remedies, namely, (1) exact the fulfillment of the twenty four (24) installments; (b) the title to the properties
obligation, should the vendee fail to pay; (2) cancel the purchased shall remain with the plaintiff until the purchase Defendants-appellants assail the Deed of Conditional
sale upon the vendee’s failure to pay two or more price thereof is fully paid; (c) if any two installments are Sale under which they purchased the three (3) Daikin air-
installments; (3) foreclose the chattel mortgage, if one has not paid by the defendants on their due dates, the whole conditioners from plaintiff-appellee as being contrary to
been constituted on the property sold, upon the vendee’s of the principal sum remaining unpaid shall become due, law, morals, good custom, public order or public policy. In
failure to pay two or more installments. The third option or with interest at the rate of 14% per annum: and (d) in case particular, they point to the contract’s paragraphs 5 and 7
remedy, however, is subject to the limitation that the of a suit, the defendants shall pay an amount equivalent as iniquitous, which paragraphs state
vendor cannot recover any unpaid balance of the price to 25% of the remaining unpaid obligation as damages, that:jgc:chanrobles.com.ph
and any agreement to the contrary is void (Art. 1484) The penalty and attorney’s fees; that to secure the payment of
three (3) remedies are alternative and NOT cumulative. If the balance of P18,576.00 the defendants jointly and "5. Should BUYER fail to pay any of the monthly
the creditor chooses one remedy, he cannot avail himself severally executed in favor of the plaintiff a promissory installments when due, or otherwise fail to comply with
of the other two. note, Exhibit C; that the three (3) air-conditioners were any of the terms and conditions herein stipulated, this
delivered to and received by the defendants as shown by contract shall automatically become null and void and all
the delivery receipt, Exhibit B; that after paying the sums so paid by BUYER by reason thereof shall be
DECISION amount of P6,966.00, the defendants failed to pay at least considered as rental and the SELLER shall then and there
two (2) monthly installments; that as of January 6, 1977, be free to take possession thereof without liability for
the remaining unpaid obligation of the defendants trespass or responsibility for any article left in or attached
NOCON, J.: amounted to P12,920.08; that statements of accounts to the PROPERTY:chanrob1es virtual 1aw library
were sent to the defendants and the plaintiff’s collectors x x x
personally went to the former to effect collections but they
Elevated to this Court by the Court of Appeals, in its failed to do so; that because of the unjustified refusal of
Resolution of May 20, 1982, on a pure question of law, 1 the defendants to pay their outstanding account and their "7. Should SELLER rescind this contract for any of the
is the appeal therein by defendants-appellants, Niu Kim wrongful detention of the properties in question, the reasons stipulated in the preceding paragraph, the
Duan and Chan Fue Eng assailing the trial court’s plaintiff tried to recover the said properties extra-judicially BUYER, by these presents obligates himself to peacefully
decision promulgated on October 11, 1977, 2 which but it failed to do so; that the matter was later referred by deliver the PROPERTY to the SELLER in case of
ordered them to pay plaintiff-appellee, Delta Motor Sales the plaintiff to its legal counsel for legal action; that in its rescission, and should a suit be brought in court by the
Corporation, the amount of P6,188.29 with a 14% per verified complaint dated January 28, 1977, the plaintiff SELLER to seek judicial declaration of rescission and take
annum interest which was due on the three (3) "Daikin" prayed for the issuance of a writ of replevin, which the possession of the PROPERTY, the BUYER hereby
air-conditioners defendants-appellants purchased from Court granted in its Order dated February 28, 1977, after obligates himself to pay all the expenses to be incurred by
plaintiff-appellee under a Deed of Conditional Sale, after the plaintiff posted the requisite bond; that on April 11, reason of such suit and in addition to pay the sum
the same was declared rescinded by the trial court. They 1977, the plaintiff, by virtue of the aforesaid writ, equivalent to 25% of the remaining unpaid obligation as
were likewise ordered to pay plaintiff-appellee P1,000.00 succeeded in retrieving the properties in question: that as damages, penalty and attorney’s fees;" 3
for and as attorney’s fees.chanrobles virtual lawlibrary of October 3, 1977, the outstanding account of the
defendants is only in the amount of P6,188.29 as shown Defendants-appellants claim that for the use of the
193
Sales – Chapter 3-12 Cases
plaintiff-appellee’s three air-conditioners, from July 5, chattel mortgage, if one has been constituted on the
1975 4 to April 11, 1977, 5 or for a period of about 22 property sold, upon the vendee’s failure to pay two or
months, they, in effect, paid rentals in the amount of more installments. The third option or remedy, however, is
P6,429,92, 6 or roughly one-third (1/3) of the entire price subject to the limitation that the vendor cannot recover
of said air-conditioners which was P19,350.00. They also any unpaid balance of the price and any agreement to the
complain that for the said period the trial court is ordering contrary is void (Art. 1484) 11
them to pay P6,188.29 as the balance due for the three
air-conditioners repossessed. Defendants-appellants were The three (3) remedies are alternative and NOT
likewise ordered to pay P1,000.00 as attorney’s fees when cumulative. If the creditor chooses one remedy, he cannot
plaintiff-appellee never sought for attorney’s fees in its avail himself of the other two.chanrobles lawlibrary :
complaint. They satirically pointed out that by putting "a rednad
few touches here and there, the same units can be sold
again to the next imprudent customer" 7 by plaintiff- It is not disputed that the plaintiff-appellee had taken
appellee. Thus, enforcement of the Deed of Conditional possession of the three air-conditioners, through a writ of
Sale will unjustly enrich plaintiff-appellee at the expense of replevin when defendants-appellants refused to extra-
defendants-appellants.chanrobles law library : red judicially surrender the same. This was done pursuant to
I paragraphs 5 and 7 of its Deed of Conditional Sale when
defendants-appellants failed to pay at least two (2)
monthly installments, so much so that as of January 6,
Defendants-appellants cannot complain that their 1977, the total amount they owed plaintiff-appellee,
downpayment of P774.00 and installment payments of inclusive of interest, was P12,920.08. 12 The case
P5,655.92 8 were treated as rentals — even though the plaintiff-appellee filed was to seek a judicial declaration
total amount of P6,429,92 which they had paid, that it had validly rescinded the Deed of Conditional Sale.
approximates one-third (1/3) of the cost of the three (3) 13
air-conditioners. A stipulation in a contract that the
installments paid shall not be returned to the vendee is Clearly, plaintiff-appellee chose the second remedy of
valid insofar as the same may not be unconscionable Article 1484 in seeking enforcement of its contract with
under the circumstances is sanctioned by Article 1486 of defendants-appellants. This is shown from the fact that its
the New Civil Code. 9 The monthly installment payable by Exhibit "F" which showed the computation of the
defendants-appellants was P774.00. 10 The P5,655.92 outstanding account of defendants-appellants as of
installment payments correspond only to seven (7) October 3, 1977 took into account "the value of the units
monthly installments. Since they admit having used the repossessed." 14 Having done so, it is barred from
air-conditioners for twenty-two (22) months, this means exacting payment from defendants-appellants of the
that they did not pay fifteen (15) monthly installments on balance of the price of the three air-conditioning units
the said air-conditioners and were thus using the same which it had already repossessed. It cannot have its cake
FREE for said period — to the prejudice of plaintiff- and eat it too. 15
appellee. Under the circumstances, the treatment of the
installment payments as rentals cannot be said to be WHEREFORE, the judgment of the trial court in Civil Case
unconscionable. No. 25578 is hereby SET ASIDE and the complaint filed
II by plaintiff-appellee Delta Motor Sales Corporation is
hereby DISMISSED. No costs.

The vendor in a sale of personal property payable in SO ORDERED.


installments may exercise one of three remedies, namely,
(1) exact the fulfillment of the obligation, should the Narvasa, C.J., Padilla, Regalado and Melo, JJ., concur.
vendee fail to pay; (2) cancel the sale upon the vendee’s
failure to pay two or more installments; (3) foreclose the
194
Sales – Chapter 3-12 Cases
To prevent such sale, the Tajanlangits instituted this "What is being sought in this present action" say
action in the Iloilo court of first instance for the purpose appellants "is to prohibit and forbid the appellee Sheriff of
among others, of annulling the alias writ of execution and Iloilo from attaching and selling at public auction sale the
all proceedings subsequent thereto. Their two main real properties of appellants because that is now
theories: (1) They had returned the machineries and farm forbidden by our law after the chattels that have been
G.R. No. L-10789 May 28, 1957 implements to the Southern Motors Inc., the latter purchased and duly mortgagee had already been
AMADOR TAJANLANGIT, ET AL., plaintiff-appellants, accepted them, and had thereby settled their accounts; for repossessed by the same vendor-mortgagee and later on
vs. that reason, said spouses did not contest the action in sold at public auction sale and purchased by the same at
SOUTHERN MOTORS, INC., ET AL., defendants- Civil Case No. 2942; and (2) as the Southern Motors Inc. such meager sum of P10,000."
appellees. had repossessed the machines purchased on installment "Our law" provides,
Almacen and Almacen for appellants. (and mortgaged) the buyers were thereby relieved from ART. 1484. In a contract of sale of personal property the
Diosdado Garingalao for appellees. further responsibility, in view of the Recto Law, now article price of which is payable in installments, the vendor may
BENGZON, J.: 1484 of the New Civil Code. exercise of the following remedies:
The case. Appellants seek to reverse the order of Hon. For answer, the company denied the alleged "settlement (1) Exact fulfillment of the obligation, should the vendee
Pantaleon Pelayo, Judge of the Iloilo court of first instance and understanding" during the pendency of civil case No. fail to pay;
refusing to interfere with the alias writ of execution issued 2949. It also denied having repossessed the machineries, (2) Cancel the sale, should the vendee's failure to pay
in Civil Case No. 2942 pending in another sala of the the truth being that they were attached by the sheriff and cover two or more installments;
same court. then deposited by the latter in its shop for safekeeping, (3) Foreclose the chattel mortgage on the thing sold, if
The facts. In April 1953 Amador Tajanlangit and his wife before the sale at public auction. one has been constituted, should the vendee's failure to
Angeles, residents of Iloilo, bought, from the Southern The case was submitted for decision mostly upon a pay cover two or more installments. In this case, he shall
Motors Inc. of Iloilo two tractors and a thresher. In stipulation of facts. Additional testimony was offered have no further action against the purchaser to recover
payment for the same, they executed the promissory note together with documentary evidence. Everything any unpaid balance of the price. Any agreement to the
Annex A whereby they undertook to satisfy the total considered the court entered judgment, saying in part; contrary shall be void. (New Civil Code.)
purchase price of P24,755.75 in several installments (with The proceedings in Civil Case No. 2942 above referred to, Appellants would invoke the last paragraph. But there has
interest) payable on stated dates from May 18, 1953 were had in the Court of First Instance (Branch 1) of the been no foreclosure of the chattel mortgage nor a
December 10, 1955. The note stipulated that if default be Province and of the City of Iloilo. While this court (Branch foreclosure sale. Therefore the prohibition against further
made in the payment of interest or of any installment, then IV) sympathizes with plaintiffs, it cannot grant, in this collection does not apply.
the total principal sum still unpaid with interest shall at action, the relief prayed for the complaint because courts At any rate it is the actual sale of the mortgaged chattel in
once become demandable etc. The spouse failed to meet of similar jurisdiction cannot invalidate the judgments and accordance with section 14 Act No. 1508 that would bar
any installment. Wherefore, they were sued, in the above orders of each other. Plaintiffs have not pursued the the creditor (who chooses to foreclose) from recovering
Civil Case No. 2942, for the amount of the promissory proper remedy. This court is without authority and any unpaid balance. (Pacific Com. Co. vs.De la Rama, 72
note.1 The spouses defaulted, and the court, after jurisdiction to declare null and void the order directing the Phil. 380.) (Manila Motor Co. vs. Fernandez, 99 Phil.,
listening to the Southern Motors' evidence entered issuance of alias writ of execution because it was made 782.).
Judgment for it in the total sum of P24,755.75 together by another court of equal rank and category (see Cabiao It is true that there was a chattel mortgage on the goods
with interest at 12 per cent, plus 10 per cent of the total and Izquierdo vs. Del Rosario and Lim, 44 Phil., 82-186). sold. But the Southern Motors elected to sue on the note
amount due as attorney's fees and costs of collection. WHEREFORE, judgement is hereby rendered dismissing exclusively, i.e. to exact fulfillment of the obligation to pay.
Carrying out the order of execution, the sheriff levied on the complaint with costs against plaintiffs costs against It had a right to select among the three remedies
the same machineries and farm implements which had plaintiffs. Let the writ of preliminiary injunction issued on established in Article 1484. In choosing to sue on the
been bought by the spouses; and later sold them at public August 26, 1954, be lifted. note, it was not thereby limited to the proceeds of the sale,
auction to the highest bidder — which turned out to be the The plaintiffs reasonably brought the matter to the Court on execution, of the mortgaged good.2
Southern Motors itself — for the total sum of P10,000. of Appeals, but the latter forwarded the expediente, being In Southern Motors Inc. vs. Magbanua, (100 Phil., 155) a
As its judgment called for much more, the Southern of the opinion that the appeal involved questions of similar situation arose in connection with the purchase on
Motors subsequently asked and obtained, an alias writ of jurisdiction and/or law installment of a Chevrolet truck by Magbanua. Upon the
execution; and pursuant thereto, the provincial sheriff Discussion. Appellants' brief elaborately explains in the latter's default, suit on the note was filed, and the truck
levied attachment on the Tajanlangits' rights and interests nine errors assigned, their original two theories although levied on together with other properties of the debtor.
in certain real properties — with a view to another sale on their "settlement" idea appears to be somewhat modified. Contending that the seller was limited to the truck, the
execution.
195
Sales – Chapter 3-12 Cases
debtor obtained a discharge of the other properties. This same court. As stated, Judge Pelayo refused to interfere
court said: on that ground. Appellants insist this was error on several
By praying that the defendant be ordered to pay the sum counts. We deem it unnecessary to deal with this
of P4,690 together with the stipulated interest at 12% per procedural aspect, inasmuch as we find that, on the
annum from 17 March 1954 until fully paid, plus 10 per merits, plaintiffs are not entitled to the relief demanded.
cent of the total amount due as attorney's fees and cost of Judgment. The decision dismissing the complaint, is
collection, the plaintiff acted to exact the fulfillment of the affirmed, with costs against appellants. So ordered.
obligation and not to foreclose the mortgage on the truck. .
..
As the plaintiff has chosen to exact the fulfillment of the
defendant's obligation, the former may enforce execution
of the judgement rendered in its favor on the personal and
real properties of the latter not exempt from execution
sufficient to satisfy the judgment. That part of the
judgement depriving the plaintiff of its right to enforce
judgment against the properties of the defendant except
the mortgaged truck and discharging the writ of
attachment on his other properties is erroneous.
(Emphasis ours.)
Concerning their second theory, — settlement or
cancellation — appellants allege that the very implements
sold "were duly returned" by them, and "were duly
received and accepted by the said vendor-mortgagee".
Therefore they argue, "upon the return of the same
chattels and due acceptance of the same by the vendor-
mortgagee, the conditional sale is ipso facto cancelled,
with the right of the vendor-mortgagee to appropriate
whatever downpayment and posterior monthly
installments made by the purchaser as it did happen in the
present case at bar."
The trouble with the argument is that it assumes that
acceptance of the goods by the Southern Motors Co, with
a view to "cancellation" of the sale. The company denies
such acceptance and cancellation, asserting the goods,
were deposited in its shop when the sheriff attached them
in pursuance of the execution. Its assertion is backed up
by the sheriff, of whose credibility there is no reason to
doubt. Anyway this cancellation or settlement theory may
not be heeded now, because it would contravene the
decision in Civil Case No. 2942 above-mentioned — it
would show the Tajanlangits owned nothing to Southern
Motors Inc. Such decision is binding upon them, unless
and until they manage to set it aside in a proper
proceeding — and this is not it.
There are other points involved in the case, such as the
authority of the judge of one branch of a court of first
instance to enjoin proceedings in another branch of the
196
Sales – Chapter 3-12 Cases
This is a petition for review of the decision 1 of the Court of Sur, Pasig, Metro Manila in good working and body
Appeals which affirmed in toto the decision of the condition.
Regional Trial Court of Pasig, Branch 51, declaring On the same day, January 9, 1980, private respondent
respondent spouses Rolando Lantan and Rina Lantan executed a promissory note reading as follows:3
owners of a 1979 model 2-door Colt Lancer car which PROMISSORY NOTE
they had acquired under a car plan for top employees of P60,639.00
the Elizalde group of companies. FOR VALUE RECEIVED, we promise to pay [to] the order
The facts are as follows: of ELISCO TOOL MFG. CORP. — SPECIAL PROJECT,
Private respondent Rolando Lantan was employed at the at its office at Napindan, Taguig, Metro Manila,
Elisco Tool Manufacturing Corporation as head of its cash Philippines, the sum of ONE THOUSAND TEN & 65/100
department. On January 9, 1980, he entered into an PESOS (P1,010.65), Philippine Currency, beginning
agreement with the company which provided as follows:2 January 9, 1980, without the necessity of notice or
That, EMPLOYER is the owner of a car Colt Lancer 2 demand in accordance with the schedule of payment
door, Model 1979, with Serial No. 3403 under LTC hereto attached as an integral part hereof.
Registration Certificate No. 0526558; In case of default, in the payment of any installment on the
That, for and in consideration of a monthly rental of ONE stipulated due date, we agree to pay as liquidated
THOUSAND TEN & 65/100 ONLY (P1,010.65) Philippine damages 2% of the amount due and unpaid for every
Currency, EMPLOYER desire to lease and EMPLOYEE thirty (30) days of default or fraction thereof. Where the
accept in lease the motor vehicle aforementioned for a default covers two successive installments, the entire
period of FIVE (5) years; unpaid balance shall automatically become due and
That, the EMPLOYEE agree as he hereby agreed to pay payable.
the lease rental thru salary deduction from his monthly It is further agreed that if upon such default attorney's
remuneration in the amount as above specified for a services are availed of, an additional sum equal to
period of FIVE (5) years; TWENTY (20%) percent of the total amount due thereon,
That, for the duration of the lease contract, all expenses but in no case be less than P1,000.00 shall be paid to
and costs of registration, insurance, repair and holder(s) hereof as attorney's fees in addition to the legal
maintenance, gasoline, oil, part replacement inclusive of costs provided for by law. We agree to submit to the
all expenses necessary to maintain the vehicle in top jurisdiction of the proper courts of Makati, Metro Manila or
condition shall be for the account of the EMPLOYEE; the Province of Rizal, at the option of the holder(s) waiving
That, at the end of FIVE (5) year period or upon payment for this purpose any other venue.1âwphi1.nêt
of the 60th monthly rental, EMPLOYEE may exercise the In case extraordinary inflation or deflation of the currency
option to purchase the motor vehicle from the stipulated should occur before this obligation is paid in full,
EMPLOYER and all monthly rentals shall be applied to the the value of the currency at the time of the establishment
payment of the full purchase price of the car and further, of the obligation will be the basis of payment.
should EMPLOYEE desire to exercise this option before Holder(s) may accept partial payment reserving his right
the 5-year period lapse, he may do so upon payment of of recourse against each and all endorsers who hereby
the remaining balance on the five year rental unto the waive DEMAND PRESENTMENT and NOTICE.
EMPLOYER, it being understood however that the option Acceptance by the holder(s) of payment or any part
G.R. No. 109966 May 31, 1999 is limited to the EMPLOYEE; thereof after due date shall not be considered as
ELISCO TOOL MANUFACTURING That, upon failure of the EMPLOYEE to pay THREE (3) extending the time for the payment of the aforesaid
CORPORATION, petitioner, accumulated monthly rentals will vest upon the obligation or as a modification of any of the condition
vs. EMPLOYER the full right to lease the vehicle to another hereof.
COURT OF APPEALS, ROLANDO LANTAN, and RINA EMPLOYEE; After taking possession of the car, private respondent
LANTAN, respondents. That, in the event of resignation and or dismissal from the installed accessories therein worth P15,000.00.
service, the EMPLOYEE shall return the subject motor In 1981, Elisco Tool ceased operations, as a result of
MENDOZA, J.: vehicle to the EMPLOYER in its compound at Kalawaan which private respondent Rolando Lantan was laid off.
Nonetheless, as of December 4, 1984, private respondent
197
Sales – Chapter 3-12 Cases
was able to make payments for the car in the total amount 1. Ordering the defendant Rolando Lantan to pay the purchase and that the promissory note was merely a
of P61,070.94. plaintiff an amount equivalent to twenty-five percent (25%) "nominal security" for the agreement. It contended that the
On June 6, 1986, petitioner filed a complaint, entitled of his outstanding obligation, for and as attorney's fees; mere acceptance of the amounts paid by private
"replevin plus sum of money," against private respondent 2. Ordering defendants to pay the cost or expenses of respondents and for indefinite periods of time was not
Rolando Lantan, his wife Rina, and two other persons, collection, repossession, bonding fees and other evidence that the parties' agreement was one of purchase
identified only as John and Susan Doe, before the incidental expenses to be proved during the trial; and and sale. Neither was it guilty of laches because, under
Regional Trial Court of Pasig, Metro Manila. Petitioner 3. Ordering defendants to pay the costs of suit. the law, an action based on a written contract can be
alleged that private respondents failed to pay the monthly Plaintiff also prays for such further reliefs as this brought within ten (10) years from the time the action
rentals which, as of May 1986, totalled P39,054.86; that Honorable Court may deem just and equitable under the accrues. On August 31, 1987, the trial court5 rendered its
despite demands, private respondents failed to settle their premises. decision.
obligation thereby entitling petitioner to the possession of Upon petitioner's posting a bond in the amount of The trial court sustained private respondents' claim that
the car; that petitioner was ready to post a bond in an P120,000, the sheriff took possession of the car in the agreement in question was one of sale and held that
amount double the value of the car, which was P60,000; question and after five (5) days turned it over to the latter had fully paid the price of the car having paid the
and that in case private respondents could not return the petitioner.4 total amount of P61,070.94 aside from installing
car, they should be held liable for the amount of P60,000 In due time, private respondents filed their answer. They accessories in the car worth P15,000.00. Said the trial
plus the accrued monthly rentals thereof, with interest at claimed that the agreement on which the complaint was court:
the rate of 14% per annum, until fully paid. Petitioner's based had not been signed by petitioner's representative, Plaintiff now comes claiming ownership of the car in
complaint contained the following prayer: Jose Ma. S. del Gallego, although it had been signed by question and has succeeded in repossessing the same by
WHEREFORE, plaintiffs prays that judgment be rendered private respondent Rolando Lantan; that their true virtue of the writ of seizure issued in this case on July 29,
as follows: agreement was "to buy and sell and not lease with option 1986. Not content with recovering possession of the said
ON THE FIRST CAUSE OF ACTION to buy" the car in question at a monthly amortization of car, plaintiff still asks that defendants should pay it the
Ordering defendant Rolando Lantan to pay the plaintiff the P1,000; and that petitioner accepted the installment sum of P39,054.86, allegedly representing the rentals due
sum of P39,054.86 plus legal interest from the date of payments made by them and, in January 1986, agreed on the car from the time of the last payment made by
demand until the whole obligation is fully paid; that the balance of the purchase price would be paid on or defendants to its repossession thereof. This is indeed a
ON THE SECOND CAUSE OF ACTION before December 31, 1986. Private respondents cited the classic case of one having his cake and eating it too!
To forthwith issue a Writ of Replevin ordering the seizure provision of the agreement making respondent Rolando Under the Recto law (Arts. 1484 & 1485, Civil Code), the
of the motor vehicle more particularly described in Lantan liable for the expenses for registration, insurance, vendor who repossesses the goods sold on installments,
paragraph 3 of the Complaint, from defendant Rolando repair and maintenance, gasoline, oil and part has no right to sue the vendee for the unpaid balance
Lantan and/or defendants Rina Lantan, John Doe, Susan replacements, inclusive of all necessary expenses, as thereof.
Doe and other person or persons in whose possession the evidence that the transaction was one of sale. Private The Court can take judicial notice of the practice wherein
said motor vehicle may be found, complete with respondents further alleged that, in any event, petitioner executives enjoy car plans in progressive companies. The
accessories and equipment, and direct deliver thereof to had waived its rights under the agreement because of the agreement of January 9, 1980 between the parties is one
plaintiff in accordance with law, and after due hearing to following circumstances: (a) while the parties agreed that such car plan. If defendant Rolando Lantan failed to keep
confirm said seizure and plaintiff's possession over the payment was to be made through salary deduction, up with his amortizations on the car in question, it was not
same; petitioner accepted payments in cash or checks; (b) because of his own liking but rather he was pushed to it
ON THE ALTERNATIVE CAUSE OF ACTION although they agreed that upon the employee's by circumstances when his employer folded up and sent
In the event that manual delivery of the subject motor resignation, the car should be returned to the employer, him to the streets. That plaintiff was giving all the chance
vehicle cannot be effected for any reason, to render private respondent Rolando Lantan was not required to do to defendants to pay the value of the car and acquire full
judgment in favor of plaintiff and against defendant so when he resigned in September 1982; (c) petitioner did ownership thereof is shown by the delay in instituting the
Rolando Lantan ordering the latter to pay the sum of not lease the vehicle to another employee after private instant case. . . .
SIXTY THOUSAND PESOS (P60,000.00) which is the respondent Rolando Lantan had allegedly failed to pay The court likewise found that the amount of P61,070.94
estimated actual value of the above-described motor three monthly "rentals"; and (d) petitioner failed to enforce included a 2% penalty for late payments for which there
vehicle, plus the accrued monthly rentals thereof with the manner of payment under the agreement by its was no stipulation in the agreement:
interests at the rate of fourteen percent (14%) per acceptance of payments in various amounts and on . . . The agreement and defendant Rolando Lantan's
annum until fully paid; different dates. promissory note of January 9, 1980 do not provide even
PRAYER COMMON TO ALL CAUSES OF ACTION In its reply, petitioner maintained that the contract for interest on the remaining balance of the purchase price
between the parties was one of lease with option to of the car. This privilege extended by corporations to their
198
Sales – Chapter 3-12 Cases
top executives is considered additional emolument to Court of Appeals granted private respondents' motion unless the price is fully paid. As this Court noted
them. And so the reason for the lack of provision for and, upon the filing of a bond, in the amount of in Vda. de Jose v. Barrueco:9
interest, much less penalty charges. Therefore, all P70,000.00, it issued a writ of execution, pursuant to Sellers desirous of malting conditional sales of their
payments made by defendant should be applied to the which the car was delivered to private respondents on goods, but who do not wish openly to make a bargain in
principal account. Since the principal was only April 16, 1989.6 that form, for one reason or another, have frequently
P60,639.00, the defendants have made an overpayment On August 26, 1992, the Court of Appeals rendered its resorted to the device of making contracts in the form of
of P431.94 which should be returned to defendant by decision, affirming in toto the decision of the trial court. leases either with options to the buyer to purchase for a
plaintiff. Hence, the instant petition for review on certiorari. small consideration at the end of term, provided the so-
For this reason, it ordered petitioner to pay private Petitioner contends that the Court of Appeals erred — called rent has been duly paid, or with stipulations that if
respondents the amount of P431.94 as excess payment, (a) in disregarding the admission in the pleadings as to the rent throughout the term is paid, title shall thereupon
as well as rentals at the rate of P1,000 a month for what documents contain the terms of the parties' vest in the lessee. It is obvious that such transactions are
depriving private respondents of the use of their car, and agreement. leases only in name. The so-called rent must necessarily
moral damages for the worry, embarrassment, and mental (b) in holding that the interest stipulation in respondents' be regarded as payment of the price in installments since
torture suffered by them on account of the repossession of Promissory Note was not valid and binding. the due payment of the agreed amount results, by the
the car. (c) in holding that respondents had fully paid their terms of the bargain, in the transfer of title to the lessee.
The dispositive portion of the trial court's decision reads obligations. In an earlier case, Manila Gas Corporation
as follows: It further argues that — v. Calupitan, 10 which involved a lease agreement of a
WHEREFORE, judgment is hereby rendered in favor of On the assumption that the Lease Agreement with option stove and a water heater, the Court said:
defendants and against plaintiff, dismissing plaintiff's to buy in this case may be treated as a sale on . . . [W]e are of the opinion, and so hold, that when in a
complaint; declaring defendants the lawful owners of that installments, the respondent Court of Appeals so-called contract of lease of personal property it is
Colt Lancer 2-door, Model 1979 with Serial No. 3403 nonetheless erred in not finding that the parties have stipulated that the alleged lessee shall pay a certain
under Registration Certificate No. 0526558; ordering validly agreed that the petitioner as seller may [i] cancel amount upon signing the contract, and on or before the
plaintiff to deliver to defendants the aforesaid motor the contract upon the respondent's default on three or 5th of every month, another specific amount, by way of
vehicle complete with all the accessories installed therein more installments, [ii] retake possession of the rental, giving the alleged lessee the right of option to buy
by defendants; should for any reason plaintiff is unable to personality, and [iii] keep the rents already paid. the said personal property before the expiration of the
deliver the said car to defendants, plaintiff is ordered to First. Petitioner does not deny that private respondent period of lease, which is the period necessary for the
pay to defendants the value of said car in the sum of Rolando Lantan acquired the vehicle in question under a payment of the said amount at the rate of so much a
P60,639.00 plus P15,000.00, the value of the car plan for executives of the Elizalde group of month, deducting the payments made by way of advance
accessories, plus interest of 12% on the said sums from companies. Under a typical car plan, the company and alleged monthly rentals, and the said alleged lessee
August 6, 1986; and sentencing plaintiff to pay defendants advances the purchase price of a car to be paid back by makes the advance payment and other monthly
the following sums: the employee through monthly deductions from his salary. installments, noting in his account and in the receipts
a) P12,431.94 as actual damages broken down as The company retains ownership of the motor vehicle until issued to him that said payments are on account of the
follows: it shall have been fully paid for.7 However, retention of price of the personal property allegedly leased, said
1) P431.94 overpayment made by defendants to plaintiff; registration of the car in the company's name is only a contract is one of sale on installment and not of lease. 11
and form of a lien on the vehicle in the event that the In U.S. Commercial v. Halili, 12 a lease agreement was
2) P12,000.00 rental on the car in question from August 6, employee would abscond before he has fully paid for it. declared to be in fact a sale of personal property by
1986 to August 5, 1987, plus the sum of P1,000.00 a There are also stipulations in car plan agreements to the installment. Said the Court: 13
month beginning August 6, 1987 until the car is returned effect that should the employment of the employee . . . There can hardly be any question that the so-called
by plaintiff to, and is received by, defendant; concerned be terminated before all installments are fully contracts of lease on which the present action is based
b) the sum of P20,000.00 as moral damages; paid, the vehicle will be taken by the employer and all were veritable leases of personal property with option to
c) the sum of P5,000.00 as exemplary damages; and installments paid shall be considered rentals per purchase, and as such come within the purview of the
d) the sum of P5,000.00 as attorney's fees. agreement. 8 above article [Art. 1454-A of the old Civil Code on sale of
Costs against the plaintiff. This Court has long been aware of the practice of vendors personal property by installment]. In fact the instruments
SO ORDERED. of personal property of denominating a contract of sale on (exhibits "A" and "B") embodying the contracts bear the
Petitioner appealed to the Court of Appeals. On the other installment as one of lease to prevent the ownership of the heading or title "Lease-Sale (Lease-Sale of Transportation
hand, private respondents filed a motion for execution object of the sale from passing to the vendee until and and/or Mechanical Equipment)." The contracts fix the
pending appeal. In its resolution of March 9, 1989, the value of the vehicles conveyed to the lessee and
199
Sales – Chapter 3-12 Cases
expressly refer to the remainder of said value after have no further action against the purchaser to recover liable for the amounts of P39,054.86 or P60,000.00
deduction of the down payment made by the lessee as any unpaid balance of the price. Any agreement to the because private respondents had fulfilled their part of the
"the unpaid balance of the purchase price of the leased contrary shall be void. obligation. The agreement does not provide for the
equipment." The contracts also provide that upon the full Art. 1485. The preceding article shall be applied to payment of interest on unpaid monthly "rentals" or
value (plus stipulated interest) being paid, the lease would contracts purporting to be leases of personal property with installments because it was entered into in pursuance of a
terminate and title to the leased property would be option to buy, when the lessor has deprived the lessee of car plan adopted by the company for the benefit of its
transferred to the lessee. Indeed, as the defendant- the possession or enjoyment of the thing. deserving employees. As the trial court correctly noted,
appellant points out, the inclusion of a clause waiving The remedies provided for in Art. 1484 are alternative, not the car plan was intended to give additional benefits to
benefit of article 1454-A of the old Civil Code is conclusive cumulative. The exercise of one bars the exercise of the executives of the Elizalde group of companies.
proof of the parties" understanding that they were entering others. 14 This limitation applies to contracts purporting to Petitioner contends that the promissory note provides for
into a lease contract with option to purchase which come be leases of personal property with option to buy by virtue such interest payment. However, as the Court of Appeals
within the purview of said article. of Art. 1485. 15 The condition that the lessor has deprived held:
Being leases of personal property with option to purchase the lessee of possession or enjoyment of the thing for the The promissory note in which the 2% monthly interest on
as contemplated in the above article, the contracts in purpose of applying Art. 1485 was fulfilled in this case by delayed payments appears does not form part of the
question are subject to the provision that when the lessor the filing by petitioner of the complaint for replevin to contract. There is no consideration for the promissory
in such case "has chosen to deprive the lessee of the recover possession of movable property. By virtue of the note. There is nothing to show that plaintiff advanced the
enjoyment of such personal property," "he shall have no writ of seizure issued by the trial court, the deputy sheriff purchase price of the vehicle for Lantan so as to make the
further action" against the lessee "for the recovery of any seized the vehicle on August 6, 1986 and thereby latter indebted to the former for the amount stated in the
unpaid balance" owing by the latter, "agreement to the deprived private respondents of its use. 16 The car was promissory note. Thus, as stated in the complaint: "That
contrary being null and void." not returned to private respondent until April 16, 1989, sometime in January, 1980, defendant Rolando Lantan
It was held that in choosing to deprive the defendant of after two (2) years and eight (8) months, upon issuance by entered into an agreement with the plaintiff for the lease of
possession of the leased vehicles, the plaintiff waived its the Court of Appeals of a writ of execution. 17 a motor vehicle supplied by the latter, with the option to
right to bring an action to recover unpaid rentals on the Petitioner prayed that private respondents be made to pay purchase at the end of the period of lease . . . ." In other
said vehicles. the sum of P39,054.86, the amount that they were words, plaintiff did not buy the vehicle for Rolando Lantan,
In the case at bar, although the agreement provides for supposed to pay as of May 1986, plus interest at the legal advancing the purchase price for that purpose. There is
the payment by private respondents of "monthly rentals," rate. 18 At the same time, it prayed for the issuance of a nothing in the complaint or in the evidence to show such
the fifth paragraph thereof gives them the option to writ of replevin or the delivery to it of the motor vehicle arrangement. Therefore, there was no indebtedness
purchase the motor vehicle at the end of the 5th year or "complete with accessories and equipment." 19 In the secured by a promissory note to speak of. There being no
upon payment of the 60th monthly rental when "all event the car could not be delivered to petitioner, it was consideration for the promissory note, the same, including
monthly rentals shall be applied to the payment of the full prayed that private respondent Rolando Lantan be made the penalty clause contained thereon, has no binding
purchase price of the car." It is clear that the transaction in to pay petitioner the amount of P60,000.00, the "estimated effect. 21
this case is a lease in name only. The so-called monthly actual value" of the car, "plus accrued monthly rentals There is no evidence that private respondents received
rentals are in truth monthly amortizations on the price of thereof with interests at the rate of fourteen percent the amount of P60,639.00 indicated in the promissory
the car. (14%) per annum until fully paid." 20 This prayer of course note as its value. What was proven below is the fact that
Second. The contract being one of sale on installment, the cannot be granted, even assuming that private private respondents received from petitioner the 2-door
Court of Appeals correctly applied to it the following respondents have defaulted in the payment of their Colt Lancer car which was valued at P60,000 and for
provisions of the Civil Code: obligation. This led the trial court to say that petitioner which private respondent Rolando Lantan paid monthly
Art. 1484. In a contract of sale of personal property the wanted to eat its cake and have it too. amortizations of P1,010.65 through salary deductions.
price of which is payable in installments, the vendor may Notwithstanding this impossibility in petitioner's choice of Indeed, as already stated, private respondents' default in
exercise any of the following remedies: remedy, this case should be considered as one for paying installments was due to the cessation of operations
(1) Exact fulfillment of the obligation, should the vendee specific performance, pursuant to Art. 1484(1), consistent of Elizalde Steel Corporation, petitioner's sister company.
fail to pay; with its prayer with respect to the unpaid installments as of Petitioner's acceptance of payments made by private
(2) Cancel the sale, should the vendee's failure to pay May 1986. In this view, the prayer for the issuance of a respondents through cash and checks could have been
cover two or more installments; writ of replevin is only for the purpose of insuring specific impelled solely by petitioner's inability to deduct the
(3) Foreclose the chattel mortgage on the thing sold, if performance by private respondents. amortizations from private respondent Rolando Lantan's
one has been constituted, should the vendee's failure to Both the trial court and the Court of Appeals correctly salary which he stopped receiving when his employment
pay cover two or more installments. In this case, he shall ruled that private respondents could no longer be held was terminated in September 1982. Apparently, to
200
Sales – Chapter 3-12 Cases
minimize the adverse consequences of the termination of
private respondent's employment, petitioner accepted
even late payments. That petitioner accepted payments
from private respondent Rolando Lantan more than two
(2) years after the latter's employment had been
terminated constitutes a waiver of petitioner's right to
collect interest upon the delayed payments. The 2%
surcharge is not provided for in the agreement. Its
collection by the company would in fact run counter to the
purpose of providing "added emoluments" to its deserving
employees. Consequently, the total amount of P61,070.94
already paid to petitioner should be considered payment
of the full purchase price of the car or the total
installments paid.
Third. Private respondents presented evidence that they
"felt bad, were worried, embarrassed and mentally
tortured" by the repossession of the car. 22 This has not
been rebutted by petitioner. There is thus a factual basis
for the award of moral damages. In addition, petitioner
acted in a wanton, fraudulent, reckless and oppressive
manner in filing the instant case, hence, the award of
exemplary damages is justified. 23 The award of attorney's
fees is likewise proper considering that private
respondents were compelled to incur expenses to protect
their rights. 24
WHEREFORE, the decision of the Court of Appeals is
AFFIRMED with costs against petitioner.1âwphi1.nêt
SO ORDERED.

G.R. No. 130347 March 3, 1999


ABELARDO VALARAO, GLORIOSA VALARAO and
CARLOS VALARAO, petitioners,
vs.
COURT OF APPEALS and MEDEN A.
ARELLANO, respondents.

PANGANIBAN, J.:
Art. 1592 of the Civil Code applies only to contracts of
sale, and not to contracts to sell or conditional sales
where title passes to the vendee only upon full payment of
the purchase price. Furthermore, in order to enforce the
automatic forfeiture clause in a deed of conditional sale,

201
Sales – Chapter 3-12 Cases
the vendors have the burden of proving a contractual On September 4, 1987, spouses Abelardo and Gloriosa unavailing as vendors-[petitioners] never appeared in the
breach on the part of the vendee. Valarao, thru their son Carlos Valarao as their attorney-in- meetings arranged by the barangay lupon.
The Case fact, sold to [Private Respondent] Meden Arellano under a [Private respondent] tried to get in touch with [petitioners]
Before us is a Petition for Review assailing the June 13, Deed of Conditional Sale a parcel of land situated in the over the phone and was able to talk with [Petitioner]
1997 Decision of the Court of Appeals (CA) 1 which District of Diliman, Q. C., covered by TCT No. 152879 with Gloriosa Valarao who told her that she [would] no longer
reversed and set aside the October 10, 1994 Decision 2 of an area of 1,504 square meters, for the sum of THREE accept the payments being offered and that [private
the Regional Trial Court (RTC) of Quezon City, Branch 82. MILLION TWO HUNDRED TWENTY FIVE THOUSAND respondent] should instead confer with her lawyer, a
The dispositive portion of the assailed CA Decision reads: PESOS (P3,225,000.00) payable under a schedule of certain Atty. Tuazon. When all her efforts to make
WHEREFORE, the decision appealed from is REVERSED payment stated therein. payment were unsuccessful, [private respondent] sought
and SET ASIDE, and a new one is entered (1) ordering In the same Deed of Conditional Sale, the [private judicial action. by filing this petition for consignation on
[herein private respondent] to pay the amount of [o]ne respondent] vendee obligated herself to encumber by way January 4, 1991.
[m]illion [o]ne [h]undred [n]inety [s]even [t]housand [p]esos of real estate mortgage in favor of [petitioners] vendors On the other hand, vendors-[petitioners], thru counsel,
(P1,197,000.00) in favor of [herein petitioners], with legal her separate piece of property with the condition that upon sent [private respondent] a letter dated 4 January 1991
interest thereon from December 31, 1992; (2) and full payment of the balance of P2,225.000.00, the said (Exh. "C") notifying her that they were enforcing the
directing [herein petitioners] to execute in favor of [herein mortgage shall become null and void and without further provision on automatic rescission as a consequence of
respondent], upon receipt of the aforesaid amount, the force and effect. (Item No. 3, pp. 2-3 of Deed of which the Deed of Conditional Sale [was deemed] null and
final and absolute deed of sale of the subject property with Conditional Sale). void, and . . . all payments made, as well as the
all the improvements. 3 It was further stipulated upon that should the vendee fail improvements introduced on the property, [were] thereby
Also assailed by petitioners is the August 21, 1997 CA to pay three (3) successive monthly installments or forfeited. The letter also made a formal demand on the
Resolution denying reconsideration. anyone year-end lump sum payment within the period [private respondent] to vacate the property should she not
The aforementioned RTC Decision, which was reversed stipulated, the sale shall be considered automatically heed the demand of [petitioners] to sign a contract of
and set aside by the CA, disposed as follows: rescinded without the necessity of judicial action and all lease for her continued stay in the property (p. 2 of Letter
WHEREFORE, premises considered, judgment is hereby payments made by the vendee shall be forfeited in favor dated Jan. 4, 1991; Exh. "C").
rendered declaring the aforesaid Deed of Conditional Sale of the vendors by way of rental for the use and occupancy In reply, [private respondent] sent a letter dated January
as automatically rescinded and all payments made of the property and as liquidated damages. All 14, 1991 (Exh. "D"), denying that she [had] refused to pay
thereunder by the [private respondent] to the [petitioners] improvements introduced by the vendee to the property the installments due [in] the months of October, November
as forfeited in favor of the latter, by way of rentals and as shall belong to the vendors without any right of and December, and countered that it was [petitioners] who
liquidated damages, as well as declaring all improvements reimbursement. (Par. (2), Item No. 3, p. 3 of Deed of refused to accept payment, thus constraining her to file a
introduced on the property subject to the said Deed of Conditional Sale). petition for consignation before the Regional Trial Court of
Condition[al] Sale to belong to the [petitioners] without any [Private respondent] appellant alleged that as of Quezon City docketed as Civil Case No. Q-91-7603.
right of reimbursement. Further, the [private respondent] September, 1990, she had already paid the amount of Notwithstanding their knowledge of the filing by [private
and all persons claiming right under her are hereby [t]wo [m]illion [t]wenty-[e]ight [t]housand (P2,028,000.00) respondent] of a consignation case against them in the
ordered to vacate the said property and to turnover [p]esos, although she admitted having failed to pay the Regional Trial Court of Quezon City docketed as Civil
possession thereof to the [petitioners]. FINALLY, the installments due in October and November, 1990. Case No. Q-91-7603, [petitioners], through counsel, sent
[private respondent] is hereby ordered to pay to the Petitioner, however, [had] tried to pay the installments due the [private respondent] another letter dated January 19,
[petitioners] the amount of P50,000.00 as attorney's fees [in] the said months, including the amount due [in] the 1991 (Exh. "F"), denying the allegations of her attempts to
and for expenses of litigation, as well as to pay the costs month of December, 1990 on December 30 and 31, 1990, tender payment on December 30 and 31, 1990, and
of the suit. The Writ of Preliminary Injunction previously but was turned down by the vendors-[petitioners] thru their demanding that [private respondent] vacate and turnover
issued is hereby ordered LIFTED and DISSOLVED, and maid, Mary Gonzales, who refused to accept the payment the property and pay a monthly compensation for her
the bond posted for its issuance held liable for the offered. [Private respondent] maintains that on previous continued occupation of the subject property at the rate of
satisfaction of the money judgment herein made in favor occasions, the same maid was the one who [had] P20,000.00, until she shall have vacated the same.
of the received payments tendered by her. It appears that Mary Ruling of the Court of Appeals
[petitioners]. 4 Gonzales refused to receive payment allegedly on orders In reversing the Regional Trial Court, the Court of Appeals
The Facts of her employers who were not at home. held that the refusal of herein petitioners "to accept the
The undisputed facts of the case as narrated by the Court [Private respondent] then reported the matter to, and tender of payment was unjustified." Notwithstanding the
of Appeals are as follows: sought the help of, the local barangay officials. Efforts to stipulation in the Deed of Conditional Sale that "the
settle the controversy before the barangay proved rescission of the contract shall of right take place" upon
202
Sales – Chapter 3-12 Cases
the failure of the vendee to pay three successive monthly We believe, however, that the issue of whether the claiming right under her therefrom with the use of
installments, the appellate court observed that a judicial requirement of a judicial demand or a notarial act has reasonable force if necessary.
demand or a notarial act was still required pursuant to been fulfilled is immaterial to the resolution of the present That upon full payment to the VENDORS of the total
Article 1592 of the Civil Code. Thus, petitioners' letter case. Article 1592 of the Civil Code. states: consideration of P3,225,000.00, the VENDORS shall
informing private respondent of the rescission of the Art. 1592. In the sale of immovable property, even though immediately and without delay execute in favor of the
contract did not suffice, for it was not notarized. The CA it may have been stipulated that upon failure to pay the VENDEE the final and absolute deed of sale of the
also observed that "the alleged breach of contract arising price at the time agreed upon the rescission of the property and all its improvements.
from the failure of the vendee to pay the monthly contract shall of right take place, the vendee may pay, Petitioners-vendors unmistakably reserved for themselves
installments for October and November 1990 within the even after the expiration of the period, as long as no the title to the property until full payment of the purchase
stipulated time is rather slight and not substantial, and to demand for rescission of the contract has been made price by the vendee. Clearly, the agreement was not a
authorize the automatic rescission on account thereof will upon him either judicially or by notarial act. After the deed of sale, but more in he nature of a contract to sell or
work injustice to the other party, who has paid a total of demand, the court may not grant him a new term. of a sale on installments. 13 Even after the execution of
P2,028,000.00 out of a total obligation of P3,225,000.00. It is well-settled that the above-quoted provision applies the Deed of Conditional Sale, the Torrens Certificate of
The rule is that rescission cannot be availed of as to only to a contract of sale, 8 and not to a sale on Title remained with and in the name of the vendors. In
unjustly enrich one party." installment 9or a contract to sell. 10 Thus, in Luzon rejecting the application of Article 1592 to a contract to
The Issues Brokerage v. Maritime Building, 11 this Court ruled that sell, the Court held in Luzon Brokerage 14 that "the full
In their Memorandum before us, petitioners raise the "Art. 1592 of the new Civil Code (Art. 1504 of the old Civil payment of the price (through the punctual performance of
following issues: 5 Code) requiring demand by suit or notarial act in case the the monthly payments) was a condition precedent to the
I Whether the Answer [— (a)] categorically indicating vendor of realty wants to rescind does not apply to a execution of the final sale and to the transfer of the
willingness to accept the amount already due if the contract to sell or promise to sell, where title remains with property from [the vendor] to the [vendee]; so that there
[private respondent] would update the account, [(b)] the vendor until" full payment of the price. The Court was to be no actual sale until and unless full payment was
praying that "if she fail[ed] to do so immediately, . . . the stresses the difference between these two types of to be no actual sale until and unless full payment was
Deed of Conditional Sale be declared rescinded, pursuant contract. In a contract to sell, "the title over the subject made."
to the second paragraph of Section 3 thereof, with costs property is transferred to the vendee only upon the full Main Issue: Enforcement of the
against the [private respondent], [(c)] ordering the latter to payment of the stipulated consideration. Unlike in a Automatic Forfeiture Clause
vacate and turnover possession of the premises to the contract of sale, the title does not pass to the vendee As a general rule, a contract is the law between the
[petitioners], and to pay the latter attorney's fees in the upon the execution of the agreement or the delivery of the parties. 15 Thus, "from the moment the contract is
amount of P50,000.00 and the expenses of litigation" [—] thing sold." 12 perfected, the parties are bound not only to the fulfillment
is tantamount to a judicial demand and notice of In the present case, the Deed of Conditional Sale is of the of what has been expressly stipulated but also to all
rescission under Art. 1592 of the Civil Code. same nature as a sale on installment or a contract to sell, consequences which, according to their nature, may be in
II Whether the automatic forfeiture clause is valid and which is not covered by Article 1592. The aforementioned keeping with good faith, usage and law." 16 Also, "the
binding between the parties. agreement provides: stipulations of the contract being the law between the
III Whether the action for consignation may prosper xxx xxx xxx parties, courts have no alternative but to enforce them as
without actual deposit [in court] of the amount due . . . [so Should the VENDEE fail to pay three (3) successive they were agreed [upon] and written, there being no law or
as] to produce the effect of payment. monthly installments or any one year-end lump sum public policy against the stipulated forfeiture of payments
The Court's Ruling payment within the period stipulated herein, this Deed of already made." 17However, it must be shown that private
The petition 6 is devoid of merit. Conditional Sale shall be considered . . . automatically respondent-vendee failed to perform her obligation,
Preliminary Matter: rescinded without the necessity of judicial action[,] and all thereby giving petitioners-vendors the right to demand the
Notarial or Judicial Demand payments made by the VENDEE shall be forfeited in favor enforcement of the contract.
Citing Article 1592 of the Civil Code, the Court of Appeals of the VENDORS by way of rental for the use and We concede the validity of the automatic forfeiture clause,
ruled that the petitioners' letter dated January 4, 1991, occupancy of the property and as liquidated damages. All which deems any previous payments forfeited and the
could not effect the rescission of the Deed of Conditional improvements introduced by the VENDEE to the property contract automatically rescinded upon the failure of the
Sale, because the said letter was not notarized. On the shall belong to the VENDORS without any right of vendee to pay three successive monthly installments or
other hand, petitioners argue that they made a judicial reimbursement. The VENDORS and/or their agents or any one yearend lump sum payment. However, petitioners
demand, which was embodied in their Manifestation filed representatives shall have the right to enter the premises failed to prove the conditions that would warrant the
on May 1, 1991, and Answer submitted on July 1,1991. 7 of the property and to eject the VENDEE and all persons implementation of this clause.

203
Sales – Chapter 3-12 Cases
Both the appellate and the trial courts agree on the power of attorney to do so. Clearly, they are estopped him, which is hereby fixed at the rate of one month grace
following: from denying that she had such authority. Under Article period for every year of installment payments
1. The Deed of Conditional Sale provided for automatic 1241 of the Civil Code, payment through a third person is made: Provided, That this right shall be exercised by the
rescission in case the vendee failed to pay three (3) valid "[i]f by the creditor's conduct, the debtor has been buyer only once in every five years of the life of the
successive monthly installments or any one yearend lump led to believe that the third person had authority to receive contract and its extensions, if any.
sum payment within the stipulated period therein. the payment." (b) If the contract is cancelled, the seller shall refund to
2. Each monthly installment was due at the end of the Failure to Consign the the buyer the cash surrender value on the payments on
month. Amount Due the property equivalent to fifty percent of the total
3. The installments for October and November 1990 were Petitioners also maintain that the consignation was not payments made and, after five years of installments, an
not paid. valid because the amount tendered was not deposited additional five percent every year but not to exceed ninety
4. The private respondent-vendee, Meden Arellano, went with the trial court. True, there is no showing that she percent of the total payments made: Provided, That the
to the house of the petitioners-vendors on December 30, deposited the money with the proper judicial authority actual cancellation of the contract shall take place after
1990. which, taken together with the other requisites for a valid thirty days from receipt by the buyer of the notice of
5. Arellano offered to pay P48,000 (total amount of consignation, 20 would have released her from her cancellation or the demand for rescission of the contract
installments due in October, November, and December obligation to pay. However, she does not deny her by a notarial act and upon full payment of the cash
1990) to Mary Gonzales, the petitioner's maid, but the obligation and, in fact, is willing to pay not only the three surrender value to the buyer.
latter refused to accept it upon instruction of petitioners. monthly installments due but also the entire residual Down payments, deposits or options on the contract shall
6. Arellano returned the next day, December 31, 1990, amount of the purchase price. Verily, she even filed a be included in the computation of the total number of
and insisted on paying, but again the maid refused to Motion to Deposit the said entire balance with the trial installments made.
accept it. court, which however denied said motion upon opposition Hence, the private respondent was entitled to a one-
7. Arellano proceeded to the barangay office around of the petitioners. 21 month grace period for every year of installments paid,
10:00 a.m. to file a case against petitioners for their Accordingly, we agree with the Court of Appeals that it which means that she had a total grace period of three
refusal to accept the payments. would be inequitable to allow the forfeiture of the amount months from December 31, 1990. Indeed, to rule in favor
8. Four (4) days later, on January 4, 1991, private of more than two million pesos already paid by private of petitioner would result in patent injustice and unjust
respondents filed a Petition for Consignation. respondent, a sum which constitutes two thirds of the total enrichment. This tribunal is not merely a court of law, but
9. Despite the said petition, the money was nevertheless consideration. Because she did make a tender of payment also a court of justice.
not deposited in court. which was unjustifiably refused, we hold that petitioners WHEREFORE, the Petition is DENIED and the dispositive
10. Negotiations between both parties went under way, cannot enforce the automatic forfeiture clause of the portion of the appealed Decision of the Court of Appeals is
culminating in the vendee's filing a Motion to Deposit the contract. hereby AFFIRMED. The CA's discussion on the need for
entire balance due, which was duly opposed by the Application of the Maceda Law judicial or notarial demand is MODIFIED in accordance
vendor, and hence was denied by the trial court. In any event, the rescission of the contract and the with this Decision. Costs against petitioners.
From the foregoing, it is clear that petitioners were not forfeiture of the payments already made could not be SO ORDERED.
justified in refusing to accept the tender of payment made effected, because the case falls squarely under Republic
by private respondent on December 30 and 31, 1990. Had Act No. 6552, 22 otherwise known as the "Maceda Law."
they accepted it on either of said dates, she would have Section 3 of said law provides:
paid all three monthly installments due. In other words, Sec. 3. In all transactions or contracts involving the sale or
there was no deliberate failure on her part to meet her financing of real estate on installment payments, including
responsibility to pay. 18 The Court takes note of her residential condominium apartments but excluding
willingness and persistence to do so, and, petitioners industrial lots, commercial buildings and sales to tenants
cannot now say otherwise. The fact is: they refused to under Republic Act. Numbered Thirty-eight hundred Forty-
accept her payment and thus have no reason to demand four as amended by Republic Act Numbered Sixty-three
the enforcement of the automatic forfeiture clause. They hundred eighty-nine, where the buyer has paid at least
cannot be rewarded for their own misdeed. two years of installments, the buyer is entitled to the
Because their maid had received monthly payments in the following rights in case he defaults in the payment of
past, 19 it is futile for petitioners to insist now that she succeeding installments:
could not have accepted the aforementioned tender of (a) To pay, without additional interest, the unpaid
payment, on the ground that she did not have a special installments due within the total grace period earned by
204
Sales – Chapter 3-12 Cases
As agreed upon, plaintiffs shall make a down payment of
Five Hundred Thousand (₱500,000.00) Pesos upon
signing of the contract. The balance of Two Million Six
Hundred Seventy Thousand Two Hundred Twenty
(₱2,670,220.00) Pesos shall be paid in three installments,
viz: Five Hundred Thousand (₱500,000.00) Pesos on
G.R. No. 172036 April 23, 2010 June 30, 1993; Five Hundred Thousand (₱500,000.00)
SPOUSES FAUSTINO AND JOSEFINA GARCIA, Pesos on August 30, 1993; One Million Six Hundred
SPOUSES MELITON GALVEZ AND HELEN GALVEZ, Seventy Thousand Two Hundred Twenty (₱1,670,220.00)
and CONSTANCIA ARCAIRA represented by their Pesos on December 31, 1993.
Attorney-in-Fact JULIANA O. MOTAS, Petitioners, On its due date, December 31, 1993, plaintiffs failed to
vs. pay the last installment in the amount of One Million Six
COURT OF APPEALS, EMERLITA DE LA CRUZ, and Hundred Seventy Thousand Two Hundred Twenty
DIOGENES G. BARTOLOME, Respondents. (₱1,670,220.00) Pesos. Sometime in July 1995, plaintiffs
DECISION offered to pay the unpaid balance, which had already
CARPIO, J.: been delayed by one and [a] half year, which defendant
G.R. No. 172036 is a petition for review1 assailing the refused to accept. On September 23, 1995, defendant
Decision2 promulgated on 25 January 2006 as well as the sold the same parcels of land to intervenor Diogenes G.
Resolution3 promulgated on 16 March 2006 of the Court Bartolome for Seven Million Seven Hundred Ninety Three
of Appeals (appellate court) in CA-G.R. CV No. 63651. Thousand (₱7,793,000.00) Pesos.
The appellate court reversed and set aside the decision of In order to compel defendant to accept plaintiffs’ payment
Branch 23 of the Regional Trial Court of Trece Martires in full satisfaction of the purchase price and, thereafter,
City, Cavite (trial court) in Civil Case No. TM-622. The execute the necessary document of transfer in their favor,
appellate court ordered Emerlita Dela Cruz (Dela Cruz) to plaintiffs filed before the RTC a complaint for specific
return to spouses Faustino and Josefina Garcia, spouses performance.
Meliton and Helen Galvez, and Constancia Arcaira In their complaint, plaintiffs alleged that they discovered
(collectively, petitioners) the amount in excess of one-half the infirmity of the Deed of Absolute Sale covering Lot
percent of ₱1,500,000. Dela Cruz’s co-defendant, Nos. 2776, 2767 and 2769, between their former owner
Diogenes Bartolome (Bartolome), did not incur any Angel Abelida and defendant, the same being spurious
liability. because the signature of Angel Abelida and his wife were
The appellate court narrated the facts as follows: falsified; that at the time of the execution of the said deed,
On May 28, 1993, plaintiffs spouses Faustino and said spouses were in the United States; that due to their
Josefina Garcia and spouses Meliton and Helen Galvez apprehension regarding the authenticity of the document,
(herein appellees) and defendant Emerlita dela Cruz they withheld payment of the last installment which was
(herein appellant) entered into a Contract to Sell wherein supposedly due on December 31, 1993; that they
the latter agreed to sell to the former, for Three Million tendered payment of the unpaid balance sometime in July
One Hundred Seventy Thousand Two Hundred Twenty 1995, after Angel Abelida ratified the sale made in favor
(₱3,170,220.00) Pesos, five (5) parcels of land situated at [of] defendant, but defendant refused to accept their
Tanza, Cavite particularly known as Lot Nos. 47, 2768, payment for no jusitifiable reason.
2776, 2767, 2769 and covered by Transfer Certificate of In her answer, defendant denied the allegation that the
Title Nos. T-340674, T-340673, T-29028, T-29026, T- Deed of Absolute Sale was spurious and argued that
29027, respectively. At the time of the execution of the plaintiffs failed to pay in full the agreed purchase price on
said contract, three of the subject lots, namely, Lot Nos. its due date despite repeated demands; that the Contract
2776, 2767, and 2769 were registered in the name of one to Sell contains a proviso that failure of plaintiffs to pay the
Angel Abelida from whom defendant allegedly acquired purchase price in full shall cause the rescission of the
said properties by virtue of a Deed of Absolute Sale dated contract and forfeiture of one-half (1/2%) percent of the
March 31, 1989. total amount paid to defendant; that a notarized letter
205
Sales – Chapter 3-12 Cases
stating the indended rescission of the contract to sell and Further, defendant is directed to pay plaintiff the sum of purchaser for value despite the presence of evidence as
forfeiture of payments was sent to plaintiffs at their last ₱100,000.00 as attorney’s fees. to his bad faith.8
known address but it was returned with a notation SO ORDERED.5 The Court’s Ruling
"insufficient address." Dela Cruz and Bartolome appealed from the judgment of The petition has no merit.
Intervenor Diogenes G. Bartolome filed a complaint in the trial court. Both parties admit the following: (1) the contract between
intervention alleging that the Contract to Sell dated May The Decision of the Appellate Court petitioners and Dela Cruz was a contract to sell; (2)
31, 1993 between plaintiffs and defendant was rescinded The appellate court reversed the trial court’s decision and petitioners failed to pay in full the agreed purchase price
and became ineffective due to unwarranted failure of the dismissed Civil Case No. TM-622. Dela Cruz’s obligation of the subject property on the stipulated date; and (3) Dela
plaintiffs to pay the unpaid balance of the purchase price under the Contract to Sell did not arise because of Cruz did not want to accept petitioners’ offer of payment
on or before the stipulated date; that he became petitioners’ undue failure to pay in full the agreed and did not want to execute a document of transfer in
interested in the subject parcels of land because of their purchase price on the stipulated date. Moreover, judicial petitioners’ favor.
clean titles; that he purchased the same from defendant action for the rescission of a contract is not necessary The pertinent provisions of the contract, denominated
by virtue of an Absolute Deed of Sale executed on where the contract provides that it may be revoked and Contract to Sell, between the parties read:
September 23, 1995 in consideration of the sum of Seven cancelled for violation of any of its terms and conditions. Failure on the part of the vendees to comply with the
Million Seven Hundred Ninety Three Thousand The dispositive portion of the appellate court’s decision herein stipulation as to the terms of payment shall cause
(₱7,793,000.00) Pesos.4 reads: the rescission of this contract and the payments made
The Decision of the Trial Court WHEREFORE, in view of all the foregoing, the appealed shall be returned to the vendees subject however, to
In its Decision dated 15 April 1999, the trial court ruled decision of the Regional Trial Court is hereby REVERSED forfeiture in favor of the Vendor equivalent to 1/2% of the
that Dela Cruz’s rescission of the contract was not valid. and SET ASIDE and Civil Case No. TM-622 is, total amount paid.
The trial court applied Republic Act No. 6552 (Maceda consequently, DISMISSED. Defendant is however xxx
Law) and stated that Dela Cruz is not allowed to ordered to return to plaintiffs the amount in excess of one- It is hereby agreed and covenanted that possession shall
unilaterally cancel the Contract to Sell. The trial court half (1/2%) percent of One Million Five Hundred Thousand be retained by the VENDOR until a Deed of Absolute Sale
found that petitioners are justified in withholding the (₱1,500,000.00) Pesos which was earlier paid by shall be executed by her in favor of the Vendees. Violation
payment of the balance of the consideration because of plaintiffs. of this provision shall authorize/empower the VENDOR
the alleged spurious sale between Angel Abelida and SO ORDERED.6 [to] demolish any construction/improvement without need
Emerlita Dela Cruz. Moreover, intervenor Diogenes The appellate court likewise resolved to deny petitioners’ of judicial action or court order.
Bartolome (Bartolome) is not a purchaser in good faith Motion for Reconsideration for lack of merit.7 That upon and after the full payment of the balance, a
because he was aware of petitioners’ interest in the Hence, this petition. Deed of Absolute Sale shall be executed by the Vendor in
subject parcels of land. Issues favor of the Vendees.
The dispositive portion of the trial court’s decision reads: Petitioners raised the following grounds for the grant of That the duplicate original of the owner’s copy of the
ACCORDINGLY, defendant Emerlita dela Cruz is ordered their petition: Transfer Certificate of Title of the above subject parcels of
to accept the balance of the purchase price in the amount I. The Honorable Court of Appeals erred when it failed to land shall remain in the possession of the Vendor until the
of ₱1,670,220.00 within ten (10) days after the judgment consider the provisions of Republic Act 6552, otherwise execution of the Deed of Absolute Sale.9
of this Court in the above-entitled case has become final known as the Maceda Law. Contracts are law between the parties, and they are
and executory and to execute immediately the final deed II. The Honorable Court of Appeals erred when it failed to bound by its stipulations. It is clear from the above-quoted
of sale in favor of plaintiffs. consider that Respondent Dela Cruz could not pass title provisions that the parties intended their agreement to be
Defendant is further directed to pay plaintiffs the amount over the three (3) properties at the time she entered to a a Contract to Sell: Dela Cruz retains ownership of the
of ₱400,000.00 as moral damages and ₱100,000.00 as Contract to Sell as her purported ownership was tainted subject lands and does not have the obligation to execute
exemplary damages. with fraud, thereby justifying Petitioners Spouses Garcia, a Deed of Absolute Sale until petitioners’ payment of the
The deed of sale executed by defendant Emerlita dela Spouses Galvez and Arcaira’s suspension of payment. full purchase price. Payment of the price is a positive
Cruz in favor of Atty. Diogenes Bartolome is declared null III. The Honorable Court of Appeals gravely erred when it suspensive condition, failure of which is not a breach but
and void and the amount of ₱7,793,000.00 which was failed to consider that Respondent Dela Cruz’s an event that prevents the obligation of the vendor to
paid by intervenor Bartolome to Emerlita dela Cruz as the "rescission" was done in evident bad faith and malice on convey title from becoming effective. Strictly speaking,
consideration of the sale of the five (5) parcels of land is account of a second sale she entered with Respondent there can be no rescission or resolution of an obligation
hereby directed to be returned by Emerlita dela Cruz to Bartolome for a much bigger amount. that is still non-existent due to the non-happening of the
Atty. Diogenes Bartolome within ten (10) days from the IV. The Honorable Court of Appeals erred when it failed to suspensive condition.10 Dela Cruz is thus not obliged to
finality of judgment. declare Respondent Bartolome is not an innocent execute a Deed of Absolute Sale in petitioners’ favor
206
Sales – Chapter 3-12 Cases
because of petitioners’ failure to make full payment on the the decision of the court will be merely declaratory of the notice to the address supplied by petitioners in the
stipulated date. revocation, but it is not in itself the revocatory act. Contract to Sell.
We ruled thus in Pangilinan v. Court of Appeals:11 Moreover, the vendor’s right in contracts to sell with It is undeniable that petitioners failed to pay the balance of
Article 1592 of the New Civil Code, requiring demand by reserved title to extrajudicially cancel the sale upon failure the purchase price on the stipulated date of the Contract
suit or by notarial act in case the vendor of realty wants to of the vendee to pay the stipulated installments and retain to Sell. Thus, Dela Cruz is within her rights to sell the
rescind does not apply to a contract to sell but only to the sums and installments already received has long been subject lands to Bartolome. Neither Dela Cruz nor
contract of sale. In contracts to sell, where ownership is recognized by the well-established doctrine of 39 years Bartolome can be said to be in bad faith.
retained by the seller and is not to pass until the full standing. The validity of the stipulation in the contract WHEREFORE, we DENY the petition. We AFFIRM in
payment, such payment, as we said, is a positive providing for automatic rescission upon non-payment toto the Court of Appeals’ Decision promulgated on 25
suspensive condition, the failure of which is not a breach, cannot be doubted. It is in the nature of an agreement January 2006 as well as the Resolution promulgated on
casual or serious, but simply an event that prevented the granting a party the right to rescind a contract unilaterally 16 March 2006 in CA-G.R. CV No. 63651.
obligation of the vendor to convey title from acquiring in case of breach without need of going to court. Thus, Costs against petitioners.
binding force. To argue that there was only a casual rescission under Article 1191 was inevitable due to SO ORDERED.
breach is to proceed from the assumption that the contract petitioners’ failure to pay the stipulated price within the
is one of absolute sale, where non-payment is a resolutory original period fixed in the agreement.
condition, which is not the case. Petitioners justify the delay in payment by stating that they
The applicable provision of law in instant case is Article had notice that Dela Cruz is not the owner of the subject
1191 of the New Civil Code which provides as follows: land, and that they took pains to rectify the alleged defect
Art. 1191. The power to rescind obligations is implied in in Dela Cruz’s title. Be that as it may, Angel Abelida’s
reciprocal ones, in case one of the obligors should not (Abelida) affidavit12 confirming the sale to Dela Cruz only
comply with what is incumbent upon him. serves to strengthen Dela Cruz’s claim that she is the
The injured party may choose between the fulfillment and absolute owner of the subject lands at the time the
the rescission of the obligation, with the payment of Contract to Sell between herself and petitioners was
damages in either case. He may also seek rescission, executed. Dela Cruz did not conceal from petitioners
even after he has chosen fulfillment, if the latter should that the title to Lot Nos. 2776, 2767 and 2769 still
become impossible. remained under Abelida’s name, and the Contract to
The Court shall decree the rescission claimed, unless Sell13 even provided that petitioners should shoulder
there be just cause authorizing the fixing of a period. the attendant expenses for the transfer of ownership
This is understood to be without prejudice to the rights of from Abelida to Dela Cruz.
third persons who have acquired the thing, in accordance The trial court erred in applying R.A. 6552,14 or the
with Articles 1385 and 1388 and the Mortgage Law. Maceda Law, to the present case. The Maceda Law
(1124) applies to contracts of sale of real estate on installment
Pursuant to the above, the law makes it available to the payments, including residential condominium apartments
injured party alternative remedies such as the power to but excluding industrial lots, commercial buildings and
rescind or enforce fulfillment of the contract, with damages sales to tenants. The subject lands, comprising five (5)
in either case if the obligor does not comply with what is parcels and aggregating 69,028 square meters, do not
incumbent upon him. There is nothing in this law which comprise residential real estate within the contemplation
prohibits the parties from entering into an agreement that of the Maceda Law.15 Moreover, even if we apply the
a violation of the terms of the contract would cause its Maceda Law to the present case, petitioners’ offer of
cancellation even without court intervention. The rationale payment to Dela Cruz was made a year and a half after
for the foregoing is that in contracts providing for the stipulated date. This is beyond the sixty-day grace
automatic revocation, judicial intervention is necessary not period under Section 4 of the Maceda Law.16 Petitioners
for purposes of obtaining a judicial declaration rescinding still cannot use the second sentence of Section 4 of the
a contract already deemed rescinded by virtue of an Maceda Law against Dela Cruz for Dela Cruz’s alleged
agreement providing for rescission even without judicial failure to give an effective notice of cancellation or
intervention, but in order to determine whether or not the demand for rescission because Dela Cruz merely sent the
rescission was proper. Where such propriety is sustained,
207
Sales – Chapter 3-12 Cases

ACTIVE REALTY & DEVELOPMENT


CORPORATION, petitioner,
vs.
NECITA G. DAROYA, represented by Attorney-In-Fact
Shirley Daroya-Quinones, respondents.
PUNO, J.:
This is a petition for review on certiorari under Rule 45 of
the Revised Rules of Court which seeks to reverse and
set aside the Resolution of the Court of Appeals, dated
August 3, 1999, denying due course to petitioner’s appeal
for insufficiency of form and substance.1âwphi1.nêt
Petitioner ACTIVE REALTY & DEVELOPMENT
CORPORATION is the owner and developer of Town &
Country Hills Executive Village in Antipolo, Rizal. On
January 2, 1985, it entered into a Contract to Sell 1 with
respondent NECITA DAROYA, a contract worker in the
Middle East, whereby the latter agreed to buy a 515 sq.
m. lot for P224,025.00 in petitioner’s subdivision.
The contract to sell stipulated that the respondent shall
pay the initial amount of P53,766.00 upon execution of the
contract and the balance of P170,259.00 in sixty (60)
monthly installments of P4,893.35. Adding the down
payment and installment payments, it would appear that
the total amount is P346,367.00, a figure higher than that
stated as the contract price.
On May 5, 1989, petitioner accepted respondent’s
amortization in the amount of P40,000.00. By August 8,
1989, respondent was in default of P15,282.85
representing three (3) monthly amortizations.
Petitioner sent respondent a notice of cancellation2 of
their contract to sell, to take effect thirty (30) days from
receipt of the letter. It does not appear from the records,
however, when respondent received the letter.
Nonetheless, when respondent offered to pay for the
balance of the contract price, petitioner refused as it has
allegedly sold the lot to another buyer.
On August 26, 1991, respondent filed a complaint for
specific performance and damages3 against petitioner
before the Arbitration Branch of the Housing and Land
208
Sales – Chapter 3-12 Cases
Use Regulatory Board (HLURB). It sought to compel the of the date of the contract, with interest at 12% per Decision of the Chief Presidential Legal Counsel and his
petitioner to execute a final Deed of Absolute Sale in annum computed from August 26, 1991 until fully Resolution denying petitioner’s motion for reconsideration,
respondent’s favor after she pays any balance that may paid, or to deliver a substitute lot at the choice of the Decision of the HLURB Board of Commissioners and
still be due from her. Respondent claimed that she is respondent.6 that of the HLURB arbiter. A perusal of these documents
entitled to the final deed of sale after she offered to pay Upon denial of its motion for reconsideration, petitioner will reveal that they contained all the relevant facts of the
the balance of P24,048.47, considering that she has assailed the Decision in the Court of Appeals. However, case from which the appellate body can form its own
already paid the total sum of P314,816.76, which its petition for review7 was denied due course for decision. Its failure to submit the other documents, like the
amount is P90,835.76 more than the total contract insufficiency in form and substance,8 because: 1) no Complaint, Answer, Position Papers and Appeal
price of P224,025.00. affidavit of service was attached to the petition; 2) except Memoranda of the parties before the HLURB, was due to
On June 14, 1993, HLURB Arbiter Alfredo M. Tan II found for certified true copies of the decision and resolution of the refusal of the Office of the President to give them a
for the respondent. He ruled that the cancellation of the the Office of the President, no other material portions of certified true copy of these documents which were
contract to sell was void as petitioner failed to pay the the record, as would support the allegations in the petition, submitted with said Office. Third, as to the lack of Board
cash surrender value to respondent as mandated by law. were attached; and, 3) the certification of forum-shopping Resolution by petitioner corporation authorizing Atty. Rene
However, as the subject lot was already sold to a third was signed by the head counsel and vice-president of the Katigbak, its Chief Legal Counsel and Vice-President for
party and the respondent had agreed to a full refund of petitioner corporation who was not authorized by a Board Legal Affairs, to represent it in the filing of the appeal,
her installment payments, petitioner was ordered Resolution to represent petitioner. petitioner admits that this was due to its honest belief that
to refund to respondent all her payments in the Petitioner moved for reconsideration. The Court of such authority is not required as it was not mentioned in
amount of P314,816.70, with 12% interest per annum Appeals denied it on an entirely new ground, i.e., for Section 6(c) of Rule 43.12 To make up for such omission,
from August 26, 1991 (the date of the filing of the untimely filing of the petition for review.9 petitioner submitted a Secretary’s Certificate 13 confirming
complaint) until fully paid and to pay P10,000.00 as Petitioner now impugns the decision of the Court of and ratifying the authority of Atty. Katigbak to represent
attorney’s fees.4 Appeals and raises the following procedural issues: petitioner. Finally, we find that the Court of Appeals erred
On appeal, the HLURB Board of Commissioners set I in denying petitioner’s motion for reconsideration due to
aside the Arbiter’s Decision. The Board refused to apply THE HONORABLE COURT OF APPEALS GROSSLY untimely filing as the records clearly show that it was filed
the remedies provided under the Maceda Law and instead ERRED IN RELYING TOO MUCH ON FORM RATHER on June 25, 1999, a day before the expiration of the
deemed it fit to formulate an "equitable" solution to the THAN ON THE MERITS OF THE PETITION THEREBY period to appeal granted by the Court of Appeals.14
case. It ruled that, as both parties were at DENYING PETITIONER OF ITS RIGHT TO DUE In denying due course to the petition, the appellate court
fault, i.e., respondent incurred in delay in her installment PROCESS. gave premium to form and failed to consider the important
payments and respondent failed to send a notarized II rights of the parties in the case at bar. 15 At the very least,
notice of cancellation, petitioner was ordered to refund THE HONORABLE COURT OF APPEALS ANCHORED petitioner substantially complied with the procedural
to the respondent one half of the total amount she has THE DENIAL OF PETITIONER’S MOTION FOR requirements for appeal, hence, it is best to give due
paid or P157,408.35, which was allegedly akin to the RECONSIDERATION ON INCONSISTENT AND course to the petition at bar to clarify the rights and duties
remedy provided under the Maceda Law.5 CONFLICTING RULINGS NOT BORNE BY THE FACTS of a buyer in contracts to sell real estate on installment
Respondent appealed to the Office of the President. AND THE RECORDS OF THE CASE. basis.
On June 2, 1998, then Chief Presidential Counsel Renato On the procedural points raised, we find for the petitioner. The issue to be resolved is whether or not the petitioner
C. Corona, acting by authority of the President, modified Our perusal of the record reveals that petitioner can be compelled to refund to the respondent the value of
the Decision of the HLURB as he found that it was not in substantially complied with the formal requirements of the lot or to deliver a substitute lot at respondent’s option.
accord with the provisions of the Maceda Law. He held Rule 43 of the Rules of Court.10 First, as to the non- We find for the respondent and rule in the
that as petitioner did not comply with the legal requisites attachment of the affidavit of service, the records bear affirmative.1âwphi1.nêt
for a valid cancellation of the contract, the contract to sell that the petition was accompanied by the original registry The contract to sell in the case at bar is governed by
between the parties subsisted and concluded that receipts issued by the post office, showing that the petition Republic Act No. 6552 -- "The Realty Installment Buyer
respondent was entitled to the lot after payment of her and its annexes were served upon the parties. Moreover, Protection Act," or more popularly known as the Maceda
outstanding balance. However, as the petitioner disclosed respondent’s counsel of record, Atty. Sergio Guadiz, Law -- which came into effect in September 1972. Its
that the lot was already sold to another person and that actually received a copy of the petition. 11 Second, declared public policy is to protect buyers of real estate on
the actual value of the lot as of the date of the contract petitioner likewise complied with Section 6 (c) of Rule 43 installment basis against onerous and oppressive
was P1,700.00 per square meter, petitioner was ordered requiring the submission of copies of the award, judgment, conditions.16 The law seeks to address the acute housing
to refund to the respondent the amount final order and resolution appealed from. Its petition was shortage problem in our country that has prompted
of P875,000.00, the true and actual value of the lot as accompanied by the duplicate original of the appealed thousands of middle and lower class buyers of houses,
209
Sales – Chapter 3-12 Cases
lots and condominium units to enter into all sorts of amortizations. Petitioner refused to accept respondent’s after she has paid for, not just the contract price, but more
contracts with private housing developers involving subsequent tender of payment of the outstanding balance than the consideration stated in the contract to sell.
installment schemes. Lot buyers, mostly low income alleging that it has already cancelled the contract and sold Thus, for failure to cancel the contract in accordance
earners eager to acquire a lot upon which to build their the subject lot to another buyer. However, the records with the procedure provided by law, we hold that the
homes, readily affix their signatures on these contracts, clearly show that the petitioner failed to comply with contract to sell between the parties remains valid and
without an opportunity to question the onerous provisions the mandatory twin requirements for a valid and subsisting. Following Section 3(a) of R.A. No. 6552,
therein as the contract is offered to them on a "take it or effective cancellation under the law,19 i.e., he failed to respondent has the right to offer to pay for the
leave it" basis.17 Most of these contracts of adhesion, send a notarized notice of cancellation and refund the balance of the purchase price, without interest, which
drawn exclusively by the developers, entrap innocent cash surrender value. At no time, from the date it gave a she did in this case. Ordinarily, petitioner would have
buyers by requiring cash deposits for reservation notice of cancellation up to the time immediately before had no other recourse but to accept payment. However,
agreements which oftentimes include, in fine print, the respondent filed the case against petitioner, did the respondent can no longer exercise this right as the subject
onerous default clauses where all the installment latter exert effort to pay the cash surrender value. In fact, lot was already sold by the petitioner to another buyer
payments made will be forfeited upon failure to pay any the records disclose that it was only during the preliminary which lot, as admitted by the petitioner, was valued
installment due even if the buyers had made payments for hearing of the case before the HLURB arbiter when at P1,700.00 per square meter. As respondent lost her
several years.18 Real estate developers thus enjoy an petitioner offered to pay the cash surrender value. chance to pay for the balance of the P875,000.00 lot, it is
unnecessary advantage over lot buyers who they often Petitioner justifies its inaction on the ground that the only just and equitable that the petitioner be ordered to
exploit with iniquitous results. They get to forfeit all the respondent was always out of the country. Even then, the refund to respondent the actual value of the lot
installment payments of defaulting buyers and resell the records are bereft of evidence to show that petitioner resold, i.e., P875,000.00, with 12% interest per annum
same lot to another buyer with the same exigent attempted to pay the cash surrender value to respondent computed from August 26, 1991 until fully paid or to
conditions. To help especially the low income lot buyers, through her last known address. The omission is deliver a substitute lot at the option of the respondent.
the legislature enacted R.A. No. 6552 delineating the surprising considering that even during the times On a final note, it would not be amiss to stress that the
rights and remedies of lot buyers and protect them from respondent was out of the country, petitioner has been HLURB Board Decision ordering petitioner to refund to
one-sided and pernicious contract stipulations. sending her written notices to remind her to pay her respondent one half of her total payments is not an
More specifically, Section 3 of R.A. No. 6552 provided for installment arrears through her last known address. equitable solution as it punished the respondent for her
the rights of the buyer in case of default in the payment of Clearly, had respondent not filed a case demanding a final delinquent payments but totally disregarded petitioner’s
succeeding installments, where he has already paid at deed of sale in her favor, petitioner would not have lifted a failure to comply with the mandatory requisites for a
least two (2) years of installments, thus: finger to give respondent what was due her – actual valid cancellation of the contract to sell. The Board failed
"(a) To pay, without additional interest, the unpaid payment of the cash surrender value, among others. In to consider that the Maceda law was enacted to remedy
installments due within the total grace period earned by disregard of basic equitable principles, petitioner’s stance the plight of low and middle-income lot buyers, save
him, which is hereby fixed at the rate of one month grace would enable it to resell the property, keep respondent’s them from the exacting default clauses in real estate sales
period for every one year of installment payments made; x installment payments, not to mention the cash surrender and assure them of a home they can call their own.
xx value which it was obligated to return. The Layug20 case Neither would the Decision of the HLURB Arbiter ordering
(b) If the contract is cancelled, the seller shall refund to cited by petitioner is inapropos. In Layug, the lot buyer did a full refund of the installment payments of respondent in
the buyer the cash surrender value of the payments on not pay for the outstanding balance of his account and the the amount of P314,816.70 be justified as, under the law,
the property equivalent to fifty per cent of the total Court found that notarial rescission or cancellation was no respondent is entitled to the lot she purchased after
payments made; provided, that the actual cancellation of longer necessary as the seller has already filed in court a payment of her outstanding balance which she was ready
the contract shall take place after thirty days from case for rescission of the contract to sell. In the case at and willing to do. Thus, to penalize the petitioner for failing
receipt by the buyer of the notice of cancellation or the bar, respondent offered to pay for her outstanding balance in its obligation to deliver the subject lot and to give the
demand for rescission of the contract by a notarial act of the contract price but respondent refused to accept it. respondent what is rightly hers, the petitioner was
and upon full payment of the cash surrender value to Neither did petitioner adduce proof that the respondent’s correctly ordered to refund to the respondent the actual
the buyer." offer to pay was made after the effectivity date stated in its value of the land (P875,000.00) she lost to another buyer,
In this case, respondent has already paid in four (4) notice of cancellation. Moreover, there was no formal plus interest at the rate of 12% per annum from August
years a total of P314,860.76 or P90,835.76 more than notice of cancellation or court action to rescind the 26, 1991 until fully paid or to deliver a substitute lot at the
the contract price of P224,035.00. In April 1989, contract. Given the circumstances, we find it illegal and choice of the respondent.1âwphi1.nêt
petitioner decided to cancel the contract when the iniquitous that petitioner, without complying with the IN VIEW WHEREOF, the Decision of then Chief
respondent incurred in delay in the payment mandatory legal requirements for canceling the contract, Presidential Legal Assistant Renato Corona, Office of the
of P15,282.85, representing three (3) monthly forfeited both respondent’s land and hard-earned money
210
Sales – Chapter 3-12 Cases
President, dated June 2, 1998, is AFFIRMED in respondent was obliged to peacefully vacate the premises letter from petitioner’s counsel dated February 24, 1997
toto. Costs against petitioner. and deliver the possession thereof to the vendor. demanding that she vacate the premises within five days
SO ORDERED. Petitioner claimed that respondent paid only P12,950. She on the ground that her possession had become unlawful.
allegedly stopped paying after December 1979 without Respondent ignored the demand. The Punong
any justification or explanation. Moreover, in a Barangay failed to settle the dispute amicably.
"Kasunduan"1 dated November 18, 1979, respondent On April 8, 1997, petitioner filed a Complaint for unlawful
borrowed P3,000 from Patricio payable in one year either detainer against respondent with the Municipal Trial Court
in one lump sum payment or by installments, failing which (MTC) of Guiguinto, Bulacan praying that, after hearing,
the balance of the loan would be added to the principal judgment be rendered ordering respondent to immediately
subject of the monthly amortizations on the land. vacate the subject property and surrender it to petitioner;
G.R. No. 147695 September 13, 2007 Lastly, petitioner asserted that when respondent ceased forfeiting the amount of P12,950 in favor of petitioner as
MANUEL C. PAGTALUNAN, petitioner, paying her installments, her status of buyer was rentals; ordering respondent to pay petitioner the amount
vs. automatically transformed to that of a lessee. Therefore, of P3,000 under the Kasunduan and the amount of P500
RUFINA DELA CRUZ VDA. DE MANZANO, respondent. she continued to possess the property by mere tolerance per month from January 1980 until she vacates the
DECISION of Patricio and, subsequently, of petitioner. property, and to pay petitioner attorney’s fees and the
AZCUNA, J.: On the other hand, respondent alleged that she paid her costs.
This is a petition for review on certiorari under Rule 45 of monthly installments religiously, until sometime in 1980 On December 22, 1998, the MTC rendered a decision in
the Rules of Court of the Court of Appeals’ (CA) Decision when Patricio changed his mind and offered to refund all favor of petitioner. It stated that although the Contract to
promulgated on October 30, 2000 and its Resolution her payments provided she would surrender the house. Sell provides for a rescission of the agreement upon
dated March 23, 2001 denying petitioner’s motion for She refused. Patricio then started harassing her and failure of the vendee to pay any installment, what the
reconsideration. The Decision of the CA affirmed the began demolishing the house portion by portion. contract actually allows is properly termed a resolution
Decision of the Regional Trial Court (RTC) of Malolos, Respondent admitted that she failed to pay some under Art. 1191 of the Civil Code.
Bulacan, dated June 25, 1999 dismissing the case of installments after December 1979, but that she resumed The MTC held that respondent’s failure to pay not a few
unlawful detainer for lack of merit. paying in 1980 until her balance dwindled to P5,650. She installments caused the resolution or termination of the
The facts are as follows: claimed that despite several months of delay in payment, Contract to Sell. The last payment made by respondent
On July 19, 1974, Patricio Pagtalunan (Patricio), Patricio never sued for ejectment and even accepted her was on January 9, 1980 (Exh. 71). Thereafter,
petitioner’s stepfather and predecessor-in-interest, late payments. respondent’s right of possession ipso facto ceased to be a
entered into a Contract to Sell with respondent, wife of Respondent also averred that on September 14, 1981, legal right, and became possession by mere tolerance of
Patricio’s former mechanic, Teodoro Manzano, whereby she and Patricio signed an agreement (Exh. 2) whereby Patricio and his successors-in-interest. Said tolerance
the former agreed to sell, and the latter to buy, a house he consented to the suspension of respondent’s monthly ceased upon demand on respondent to vacate the
and lot which formed half of a parcel of land, covered by payments until December 1981. However, even before the property.
Transfer Certificate of Title (TCT) No. T-10029 (now TCT lapse of said period, Patricio resumed demolishing The dispositive portion of the MTC Decision reads:
No. RT59929 [T-254773]), with an area of 236 square respondent’s house, prompting her to lodge a complaint Wherefore, all the foregoing considered, judgment is
meters. The consideration of P17,800 was agreed to be with the Barangay Captain who advised her that she could hereby rendered, ordering the defendant:
paid in the following manner: P1,500 as downpayment continue suspending payment even beyond December a. to vacate the property covered by Transfer Certificate of
upon execution of the Contract to Sell, and the balance to 31, 1981 until Patricio returned all the materials he took Title No. T-10029 of the Register of Deeds of Bulacan
be paid in equal monthly installments of P150 on or before from her house. This Patricio failed to do until his death. (now TCT No. RT-59929 of the Register of Deeds of
the last day of each month until fully paid. Respondent did not deny that she still owed Bulacan), and to surrender possession thereof to the
It was also stipulated in the contract that respondent could Patricio P5,650, but claimed that she did not resume plaintiff;
immediately occupy the house and lot; that in case of paying her monthly installment because of the unlawful b. to pay the plaintiff the amount of P113,500 representing
default in the payment of any of the installments for 90 acts committed by Patricio, as well as the filing of the rentals from January 1980 to the present;
days after its due date, the contract would be ejectment case against her. She denied having any c. to pay the plaintiff such amount of rentals,
automatically rescinded without need of judicial knowledge of the Kasunduan of November 18, 1979. at P500/month, that may become due after the date of
declaration, and that all payments made and all Patricio and his wife died on September 17, 1992 and on judgment, until she finally vacates the subject property;
improvements done on the premises by respondent would October 17, 1994, respectively. Petitioner became their d. to pay to the plaintiff the amount of P25,000 as
be considered as rentals for the use and occupation of the sole successor-in-interest pursuant to a waiver by the attorney’s fees.
property or payment for damages suffered, and other heirs. On March 5, 1997, respondent received a SO ORDERED.2
211
Sales – Chapter 3-12 Cases
On appeal, the RTC of Malolos, Bulacan, in a Decision B. The Court of Appeals erred in resolving the issue on applied and must be applied to rentals for her use of the
dated June 25, 1999, reversed the decision of the MTC the applicability of the Maceda Law, which issue was not house and lot after December, 1979 or after she stopped
and dismissed the case for lack of merit. According to the raised in the proceedings a quo. payment of her installments.
RTC, the agreement could not be automatically rescinded C. Assuming arguendo that the RTC was correct in ruling Petitioner argues that assuming Patricio accepted
since there was delivery to the buyer. A judicial that the MTC has no jurisdiction over a rescission case, respondent’s delayed installments in 1981, such act
determination of rescission must be secured by petitioner the Court of Appeals erred in not remanding the case to cannot prevent the cancellation of the Contract to Sell.
as a condition precedent to convert the possession de the RTC for trial.5 Installments after 1981 were still unpaid and the
facto of respondent from lawful to unlawful. Petitioner submits that the Maceda Law supports and applicable grace periods under the Maceda Law on the
The dispositive portion of the RTC Decision states: recognizes the right of vendors of real estate to cancel the unpaid installments have long lapsed. Respondent cannot
WHEREFORE, judgment is hereby rendered reversing the sale outside of court, without need for a judicial be allowed to hide behind the Maceda Law. She acted
decision of the Municipal Trial Court of Guiguinto, Bulacan declaration of rescission, citing Luzon Brokerage Co., Inc., with bad faith and must bear the consequences of her
and the ejectment case instead be dismissed for lack of v. Maritime Building Co., Inc.6 deliberate withholding of and refusal to make the monthly
merit.3 Petitioner contends that respondent also had more than payments.
The motion for reconsideration and motion for execution the grace periods provided under the Maceda Law within Petitioner also contends that the applicability of the
filed by petitioner were denied by the RTC for lack of merit which to pay. Under Sec. 37 of the said law, a buyer who Maceda Law was never raised in the proceedings below;
in an Order dated August 10, 1999. has paid at least two years of installments has a grace hence, it should not have been applied by the CA in
Thereafter, petitioner filed a petition for review with the period of one month for every year of installment paid. resolving the case.
CA. Based on the amount of P12,950 which respondent had The Court is not persuaded.
In a Decision promulgated on October 30, 2000, the CA already paid, she is entitled to a grace period of six The CA correctly ruled that R.A No. 6552, which governs
denied the petition and affirmed the Decision of the RTC. months within which to pay her unpaid installments after sales of real estate on installment, is applicable in the
The dispositive portion of the Decision reads: December, 1979. Respondent was given more than six resolution of this case.
WHEREFORE, the petition for review on certiorari is months from January 1980 within which to settle her This case originated as an action for unlawful detainer.
Denied. The assailed Decision of the Regional Trial Court unpaid installments, but she failed to do so. Petitioner’s Respondent is alleged to be illegally withholding
of Malolos, Bulacan dated 25 June 1999 and its Order demand to vacate was sent to respondent in February possession of the subject property after the termination of
dated 10 August 1999 are hereby AFFIRMED. 1997. the Contract to Sell between Patricio and respondent. It is,
SO ORDERED. 4 There is nothing in the Maceda Law, petitioner asserts, therefore, incumbent upon petitioner to prove that the
The CA found that the parties, as well as the MTC and which gives the buyer a right to pay arrearages after the Contract to Sell had been cancelled in accordance with
RTC failed to advert to and to apply Republic Act (R.A.) grace periods have lapsed, in the event of an invalid R.A. No. 6552.
No. 6552, more commonly referred to as the Maceda Law, demand for rescission. The Maceda Law only provides The pertinent provision of R.A. No. 6552 reads:
which is a special law enacted in 1972 to protect buyers of that actual cancellation shall take place after 30 days from Sec. 3. In all transactions or contracts involving the sale or
real estate on installment payments against onerous and receipt of the notice of cancellation or demand for financing of real estate on installment payments, including
oppressive conditions. rescission and upon full payment of the cash surrender residential condominium apartments but excluding
The CA held that the Contract to Sell was not validly value to the buyer. industrial lots, commercial buildings and sales to tenants
cancelled or rescinded under Sec. 3 (b) of R.A. No. 6552, Petitioner contends that his demand letter dated February under Republic Act Numbered Thirty-eight hundred forty-
and recognized respondent’s right to continue occupying 24, 1997 should be considered the notice of cancellation four as amended by Republic Act Numbered Sixty-three
unmolested the property subject of the contract to sell. since the demand letter informed respondent that she had hundred eighty-nine, where the buyer has paid at least
The CA denied petitioner’s motion for reconsideration in a "long ceased to have any right to possess the premises in two years of installments, the buyer is entitled to the
Resolution dated March 23, 2001. question due to [her] failure to pay without justifiable following rights in case he defaults in the payment of
Hence, this petition for review on certiorari. cause." In support of his contention, he cited Layug v. succeeding installments:
Petitioner contends that: Intermediate Appellate Court8 which held that "the (a) To pay, without additional interest, the unpaid
A. Respondent Dela Cruz must bear the consequences of additional formality of a demand on [the seller’s] part for installments due within the total grace period earned by
her deliberate withholding of, and refusal to pay, the rescission by notarial act would appear, in the premises, him, which is hereby fixed at the rate of one month grace
monthly payment. The Court of Appeals erred in allowing to be merely circuitous and consequently superfluous." He period for every one year of installment payments made:
Dela Cruz who acted in bad faith from benefiting under the stated that in Layug, the seller already made a written Provided, That this right shall be exercised by the buyer
Maceda Law. demand upon the buyer. only once in every five years of the life of the contract and
In addition, petitioner asserts that whatever cash its extensions, if any.
surrender value respondent is entitled to have been
212
Sales – Chapter 3-12 Cases
(b) If the contract is cancelled, the seller shall refund house and lot after respondent stopped payment after possession of the property by respondent who has paid
to the buyer the cash surrender value of the payments January 1980. the substantial amount of P12,300 out of the purchase
on the property equivalent to fifty percent of the total The Court, however, finds that the letter11 dated February price of P17,800, the Court agrees with the CA that it is
payments made and, after five years of installments, an 24, 1997, which was written by petitioner’s counsel, only right and just to allow respondent to pay her arrears
additional five percent every year but not to exceed ninety merely made formal demand upon respondent to vacate and settle the balance of the purchase price.
percent of the total payments made: Provided, That the the premises in question within five days from receipt For respondent’s delay in the payment of the installments,
actual cancellation of the contract shall take place thereof since she had "long ceased to have any right to the Court, in its discretion, and applying Article 220914 of
after thirty days from receipt by the buyer of possess the premises x x x due to [her] failure to pay the Civil Code, may award interest at the rate of 6% per
the notice of cancellation or the demand for without justifiable cause the installment payments x x x." annum15 on the unpaid balance considering that there is
rescission of the contract by a notarial act and upon Clearly, the demand letter is not the same as the notice of no stipulation in the Contract to Sell for such interest. For
full payment of the cash surrender value to the cancellation or demand for rescission by a notarial purposes of computing the legal interest, the reckoning
buyer.9 actrequired by R.A No. 6552. Petitioner cannot rely period should be the filing of the complaint for unlawful
R.A. No. 6552, otherwise known as the "Realty on Layug v. Intermediate Appellate Court12 to support his detainer on April 8, 1997.
Installment Buyer Protection Act," recognizes in contention that the demand letter was sufficient Based on respondent’s evidence16 of payments made, the
conditional sales of all kinds of real estate (industrial, compliance. Layug held that "the additional formality of a MTC found that respondent paid a total of P12,300 out of
commercial, residential) the right of the seller to cancel the demand on [the seller’s] part for rescission by notarial act the purchase price of P17,800. Hence, respondent still
contract upon non-payment of an installment by the buyer, would appear, in the premises, to be merely circuitous and has a balance of P5,500, plus legal interest at the rate of
which is simply an event that prevents the obligation of the consequently superfluous" since the seller therein filed an 6% per annum on the unpaid balance starting April 8,
vendor to convey title from acquiring binding force.10 The action for annulment of contract, which is a kindred 1997.
Court agrees with petitioner that the cancellation of the concept of rescission by notarial act.13 Evidently, the case The third issue is disregarded since petitioner assails an
Contract to Sell may be done outside the court particularly of unlawful detainer filed by petitioner does not exempt inexistent ruling of the RTC on the lack of jurisdiction of
when the buyer agrees to such cancellation. him from complying with the said requirement. the MTC over a rescission case when the instant case he
However, the cancellation of the contract by the seller In addition, Sec. 3 (b) of R.A. No. 6552 requires refund of filed is for unlawful detainer.
must be in accordance with Sec. 3 (b) of R.A. No. 6552, the cash surrender value of the payments on the property WHEREFORE, the Decision of the Court of Appeals dated
which requires a notarial act of rescission and the refund to the buyer before cancellation of the contract. The October 30, 2000 sustaining the dismissal of the unlawful
to the buyer of the full payment of the cash surrender provision does not provide a different requirement for detainer case by the RTC is AFFIRMED with the
value of the payments on the property. Actual cancellation contracts to sell which allow possession of the property by following MODIFICATIONS:
of the contract takes place after 30 days from receipt by the buyer upon execution of the contract like the instant 1. Respondent Rufina Dela Cruz Vda. de Manzano shall
the buyer of the notice of cancellation or the demand for case. Hence, petitioner cannot insist on compliance with pay petitioner Manuel C. Pagtalunan the balance of the
rescission of the contract by a notarial act and upon full the requirement by assuming that the cash surrender purchase price in the amount of Five Thousand Five
payment of the cash surrender value to the buyer. value payable to the buyer had been applied to rentals of Hundred Pesos (P5,500) plus interest at 6% per annum
Based on the records of the case, the Contract to Sell was the property after respondent failed to pay the installments from April 8, 1997 up to the finality of this judgment, and
not validly cancelled or rescinded under Sec. 3 (b) of R.A. due. thereafter, at the rate of 12% per annum;
No. 6552. There being no valid cancellation of the Contract to Sell, 2. Upon payment, petitioner Manuel C. Pagtalunan shall
First, Patricio, the vendor in the Contract to Sell, died on the CA correctly recognized respondent’s right to continue execute a Deed of Absolute Sale of the subject property
September 17, 1992 without canceling the Contract to occupying the property subject of the Contract to Sell and and deliver the certificate of title in favor of respondent
Sell. affirmed the dismissal of the unlawful detainer case by the Rufina Dela Cruz Vda. de Manzano; and
Second, petitioner also failed to cancel the Contract to RTC. 3. In case of failure to pay within 60 days from finality of
Sell in accordance with law. The Court notes that this case has been pending for more this Decision, respondent Rufina Dela Cruz Vda. de
Petitioner contends that he has complied with the than ten years. Both parties prayed for other reliefs that Manzano shall immediately vacate the premises without
requirements of cancellation under Sec. 3 (b) of R.A. No. are just and equitable under the premises. Hence, the need of further demand, and the downpayment and
6552. He asserts that his demand letter dated February rights of the parties over the subject property shall be installment payments of P12,300 paid by her shall
24, 1997 should be considered as the notice of resolved to finally dispose of that issue in this case. constitute rental for the subject property.
cancellation or demand for rescission by notarial act and Considering that the Contract to Sell was not cancelled by No costs.
that the cash surrender value of the payments on the the vendor, Patricio, during his lifetime or by petitioner in SO ORDERED.
property has been applied to rentals for the use of the accordance with R.A. No. 6552 when petitioner filed this
case of unlawful detainer after 22 years of continuous
213
Sales – Chapter 3-12 Cases
Inc. (BF Homes) and herein respondent Julie Chandumal
(Chandumal). The property subject of the contract is
located in Talon Dos, Las Piñas City and covered by
Transfer Certificate of Title No. T-10779. On February 12,
1993, BF Homes sold to PDB all its rights, participations
and interests over the contract.
Chandumal paid her monthly amortizations from
December 1990 until May 1994 when she began to
default in her payments. In a Notice of Delinquency and
Rescission of Contract with Demand to Vacate4 dated July
14, 1998, PDB gave Chandumal a period of thirty (30)
days from receipt within which to settle her installment
arrearages together with all its increments; otherwise, all
her rights under the contract shall be deemed
extinguished and terminated and the contract declared as
rescinded. Despite demand, Chandumal still failed to
settle her obligation.
On June 18, 1999, an action for judicial confirmation of
notarial rescission and delivery of possession was filed by
PDB against Chandumal, docketed as Civil Case No. LP-
99-0137. PDB alleged that despite demand, Chandumal
failed and/or refused to pay the amortizations as they fell
due; hence, it caused the rescission of the contract by
means of notarial act, as provided in Republic Act (R.A.)
No. 6552.5 According to PDB, it tried to deliver the cash
surrender value of the subject property, as required under
R.A. No. 6552, in the amount of ₱ 10,000.00; however,
the defendant was unavailable for such purpose.6
Consequently, summons was issued and served by
G.R. No. 195619 September 5, 2012 deputy sheriff Roberto T. Galing (Sheriff Galing).
PLANTERS DEVELOPMENT BANK, Petitioner, According to his return, Sheriff Galing attempted to
vs. personally serve the summons upon Chandumal on July
JULIE CHANDUMAL, Respondent. 15, 19 and 22, 1999 but it was unavailing as she was
DECISION always out of the house on said dates. Hence, the sheriff
REYES, J.: caused substituted service of summons on August 5,
In this petition for review under Rule 45 of the Rules of 1999 by serving the same through Chandumal’s mother
Court, Planters Development Bank (PDB) questions the who acknowledged receipt thereof.7
Decision1 dated July 27, 2010 of the Court of Appeals For her failure to file an answer within the prescribed
(CA), as well as its Resolution2 dated February 16, 2011, period, PDB filed on April 24, 2000 an ex parte motion to
denying the petitioner's motion for reconsideration in CA- declare Chandumal in default. On January 12, 2001, the
G.R. CV No. 82861. The assailed decision nullified the RTC issued an Order granting the motion of PDB.8
Decision3 dated May 31, 2004 of the Regional Trial Court On February 23, 2001, Chandumal filed an Urgent Motion
(RTC), Las Piñas City, Branch 255 in Civil Case No. LP- to Set Aside Order of Default and to Admit Attached
99-0137. Answer. She maintained that she did not receive the
Antecedent Facts summons and/or was not notified of the same. She further
The instant case stemmed from a contract to sell a parcel alleged that her failure to file an answer within the
of land, together with improvements, between BF Homes, reglementary period was due to fraud, mistake or
214
Sales – Chapter 3-12 Cases
excusable negligence. In her answer, Chandumal alleged dated May 31, 2004, in Civil Case No. LP-99-0137 is 2. Whether Chandumal voluntarily submitted to the
the following defenses: (a) contrary to the position of PDB, hereby NULLIFIED and VACATED. jurisdiction of the trial court; and
the latter did not make any demand for her to pay the SO ORDERED.15 3. Whether there was proper rescission by notarial act of
unpaid monthly amortization; and (b) PDB did not tender PDB filed a motion for reconsideration but it was denied the contract to sell.
or offer to give the cash surrender value of the property in by the CA in its Resolution dated February 16, 2011. Our Ruling
an amount equivalent to fifty percent (50%) of the actual Hence, this petition based on the following assignment of The fundamental rule is that jurisdiction over a defendant
total payment made, as provided for under Section 3(b) of errors: in a civil case is acquired either through service of
R.A. No. 6552. Moreover, Chandumal claimed that since I summons or through voluntary appearance in court and
the total payment she made amounts to ₱ 782,000.00, the The Honorable Court of Appeals erred in reversing the submission to its authority. If a defendant has not been
corresponding cash surrender value due her should be ₱ decision of the trial court on the ground of improper properly summoned, the court acquires no jurisdiction
391,000.00.9 service of summons; over its person, and a judgment rendered against it is null
Per Order10 dated August 2, 2001, the RTC denied II and void.17
Chandumal’s motion to set aside the order of default. Her The decision of the trial court is valid as it duly acquired Where the action is in personam18 and the defendant is in
motion for reconsideration was also denied for lack of jurisdiction over the person of respondent Chandumal the Philippines, service of summons may be made
merit.11 Conformably, the RTC allowed PDB to present its through voluntary appearance; and through personal service, that is, summons shall be
evidence ex parte.12 On May 31, 2004, the RTC rendered III served by handing to the defendant in person a copy
a The trial court did not err in confirming and ratifying the thereof, or if he refuses to receive and sign for it, by
Decision13 in favor of PDB, the dispositive portion of which notarial rescission of the subject contract to sell.16 tendering it to him.19 If the defendant cannot be personally
reads: PDB contends that the RTC properly acquired jurisdiction served with summons within a reasonable time, it is then
WHEREFORE, the foregoing considered, judgment is over the person of Chandumal.1âwphi1 According to that substituted service may be made.20 Personal service
hereby rendered in favor of the plaintiff Planters PDB, there was proper service of summons since the of summons should and always be the first option, and it
Development Bank and against defendant Julie sheriff complied with the proper procedure governing is only when the said summons cannot be served within a
Chandumal as follows, to wit: substituted service of summons as laid down in Section 7, reasonable time can the process server resort to
1. Declaring the notarial rescission of the Contract to Sell Rule 14 of the Rules of Court. PDB alleges that it is clear substituted service.21
dated 03 January 1990 made by the plaintiff per the from the sheriff’s return that there were several attempts No valid substituted service of
Notice of Delinquency and Rescission of Contract with on at least three (3) different dates to effect personal summons
Demand to Vacate dated 14 July 1998 as judicially service within a reasonable period of nearly a month, In this case, the sheriff resorted to substituted service of
confirmed and ratified; before he caused substituted service of summons. The summons due to his failure to serve it personally. In
2. Requiring the plaintiff to deposit in the name of the sheriff likewise stated the reason for his failure to effect Manotoc v. Court of Appeals,22 the Court detailed the
defendant the amount of ₱ 10,000.00 representing the personal service and that on his fourth attempt, he requisites for a valid substituted service of summons,
cash surrender value for the subject property with the effected the service of summons through Chandumal’s summed up as follows: (1) impossibility of prompt
Land Bank of the Philippines, Las Pi[ñ]as City Branch in mother who is unarguably, a person of legal age and with personal service – the party relying on substituted service
satisfaction of the provisions of R.A. No. 6552; and, sufficient discretion. PDB also argues that Chandumal or the sheriff must show that the defendant cannot be
3. Ordering the defendant to pay the plaintiff the amount voluntarily submitted herself to the jurisdiction of the court served promptly or there is impossibility of prompt service;
of ₱ 50,000.00 as and by way of attorney’s fees, including when she filed an Urgent Motion to Set Aside Order of (2) specific details in the return – the sheriff must describe
the costs of suit. Default and to Admit Attached Answer. in the Return of Summons the facts and circumstances
SO ORDERED.14 For her part, Chandumal asserts that she never received surrounding the attempted personal service; (3) a person
From the foregoing judgment, Chandumal appealed to the a copy of the summons or was ever notified of it and she of suitable age and discretion – the sheriff must determine
CA. only came to know of the case sometime in July or August if the person found in the alleged dwelling or residence of
On July 27, 2010, the CA, without ruling on the propriety 2000, but she was already in the United States of America defendant is of legal age, what the recipient’s relationship
of the judicial confirmation of the notarial rescission, by that time, and that the CA correctly ruled that there was with the defendant is, and whether said person
rendered the assailed decision nullifying the RTC decision no valid service of summons; hence, the RTC never comprehends the significance of the receipt of the
due to invalid and ineffective substituted service of acquired jurisdiction over her person. summons and his duty to immediately deliver it to the
summons. The dispositive portion of the CA decision Issues defendant or at least notify the defendant of said receipt of
provides: 1. Whether there was a valid substituted service of summons, which matters must be clearly and specifically
WHEREFORE, premises considered, the decision of summons; described in the Return of Summons; and (4) a competent
Branch 255 of the Regional Trial Court of Las Piñas City, person in charge, who must have sufficient knowledge to
215
Sales – Chapter 3-12 Cases
understand the obligation of the defendant in the person. Thus, it was ruled that the filing of motions to rights of the buyer in case of cancellation. Thus, Sec. 3 (b)
summons, its importance, and the prejudicial effects admit answer, for additional time to file answer, for of the law provides that:
arising from inaction on the summons.23 These were reconsideration of a default judgment, and to lift order of "If the contract is cancelled, the seller shall refund to the
reiterated and applied in Pascual v. Pascual,24 where the default with motion for reconsideration is considered buyer the cash surrender value of the payments on the
substituted service of summon made was invalidated due voluntary submission to the trial court’s jurisdiction.27 The property equivalent to fifty percent of the total payments
to the sheriff’s failure to specify in the return the necessary Court notes that aside from the allegation that she did not made and, after five years of installments, an additional
details of the failed attempts to effect personal service receive any summons, Chandumal’s motion to set aside five percent every year but not to exceed ninety percent of
which would justify resort to substituted service of order of default and to admit attached answer failed to the total payments made: Provided, That the actual
summons. positively assert the trial court’s lack of jurisdiction. In fact, cancellation of the contract shall take place after thirty
In applying the foregoing requisites in the instant case, the what was set forth therein was the substantial claim that days from receipt by the buyer of the notice of cancellation
CA correctly ruled that the sheriff’s return failed to justify a PDB failed to comply with the requirements of R.A. No. or the demand for rescission of the contract by a notarial
resort to substituted service of summons. According to the 6552 on payment of cash surrender value,28 which already act and upon full payment of the cash surrender value to
CA, the Return of Summons does not specifically show or delves into the merits of PDB’s cause of action. In the buyer."31 (Citation omitted and emphasis ours)
indicate in detail the actual exertion of efforts or any addition, Chandumal even appealed the RTC decision to R.A. No. 6552 recognizes the right of the seller to cancel
positive step taken by the officer or process server in the CA, an act which demonstrates her recognition of the the contract but any such cancellation must be done in
attempting to serve the summons personally to the trial court’s jurisdiction to render said judgment. conformity with the requirements therein prescribed. In
defendant. The return merely states the alleged Given Chandumal’s voluntary submission to the addition to the notarial act of rescission, the seller is
whereabouts of the defendant without indicating that such jurisdiction of the trial court, the RTC, Las Piñas City, required to refund to the buyer the cash surrender value of
information was verified from a person who had Branch 255, had all authority to render its Decision dated the payments on the property. The actual cancellation of
knowledge thereof.25Indeed, the sheriff’s return shows a May 31, 2004. The CA, therefore, erred in nullifying said the contract can only be deemed to take place upon the
mere perfunctory attempt to cause personal service of the RTC decision and dispensing with the resolution of the expiry of a thirty (30)-day period following the receipt by
summons on Chandumal. There was no indication if he substantial issue raised herein, i.e., validity of the notarial the buyer of the notice of cancellation or demand for
even asked Chandumal’s mother as to her specific rescission. Instead, however, of remanding this case to rescission by a notarial act and the full payment of the
whereabouts except that she was "out of the house", the CA, the Court will resolve the same considering that cash surrender value.32
where she can be reached or whether he even tried to the records of the case are already before us and in order In this case, it is an admitted fact that PDB failed to give
await her return. The "efforts" exerted by the sheriff clearly to avoid any further delay.29 Chandumal the full payment of the cash surrender value.
do not suffice to justify substituted service and his failure There is no valid rescission of the In its complaint,33 PDB admitted that it tried to deliver the
to comply with the requisites renders such service contract to sell by notarial act cash surrender value of the subject property as required
ineffective.26 pursuant to Section 3(b), R.A. No. 6552 under R.A. No. 6552 but Chandumal was "unavailable" for
Respondent voluntarily submitted That the RTC had jurisdiction to render the decision does such purpose. Thus, it prayed in its complaint that it be
to the jurisdiction of the trial court not necessarily mean, however, that its ruling on the ordered to "deposit with a banking institution in the
Despite that there was no valid substituted service of validity of the notarial rescission is in accord with the Philippines, for the account of Defendants (sic), the
summons, the Court, nevertheless, finds that Chandumal established facts of the case, the relevant law and amount of Ten Thousand Pesos (₱ 10,000.00), Philippine
voluntarily submitted to the jurisdiction of the trial court. jurisprudence.1âwphi1 Currency, representing the cash surrender value of the
Section 20, Rule 14 of the Rules of Court states: PDB claims that it has validly rescinded the contract by subject property; x x x."34 The allegation that Chandumal
Sec. 20. Voluntary appearance. – The defendant’s notarial act as provided under R.A. No. 6552. Basically, made herself unavailable for payment is not an excuse as
voluntary appearance in the action shall be equivalent to PDB instituted Civil Case No. LP-99-0137 in order to the twin requirements for a valid and effective cancellation
service of summons. The inclusion in a motion to dismiss secure judicial confirmation of the rescission and to under the law, i.e., notice of cancellation or demand for
of other grounds aside from lack of jurisdiction over the recover possession of the property subject of the contract. rescission by a notarial act and the full payment of the
person of the defendant shall not be deemed a voluntary In Leaño v. Court of Appeals,30 it was held that: cash surrender value, is mandatory.35 Consequently, there
appearance. R. A. No. 6552 recognizes in conditional sales of all kinds was no valid rescission of the contract to sell by notarial
When Chandumal filed an Urgent Motion to Set Aside of real estate (industrial, commercial, residential) the right act undertaken by PDB and the RTC should not have
Order of Default and to Admit Attached Answer, she of the seller to cancel the contract upon non-payment of given judicial confirmation over the same.
effectively submitted her person to the jurisdiction of the an installment by the buyer, which is simply an event that WHEREFORE, the petition is DENIED. The Decision
trial court as the filing of a pleading where one seeks an prevents the obligation of the vendor to convey title from dated July 27, 2010 of the Court of Appeals, as well as its
affirmative relief is equivalent to service of summons and acquiring binding force. The law also provides for the Resolution dated February 16, 2011, denying the Motion
vests the trial court with jurisdiction over the defendant’s for Reconsideration in CA-G.R. CV No. 82861
216
Sales – Chapter 3-12 Cases
are AFFIRMED in so far as there was no valid service of
summons. Further, the Court DECLARES that there was
no valid rescission of contract pursuant to R.A. No. 6552.
Accordingly, the Decision dated May 31, 2004 of the
Regional Trial Court, Las Piñas City, Branch 255 in Civil
Case No. LP-99-0 137 is REVERSED and SET ASIDE,
and is therefore, DISMISSED for lack of merit.
SO ORDERED.

G.R. No. 189145 December 4, 2013


OPTIMUM DEVELOPMENT BANK, Petitioner,
vs.
SPOUSES BENIGNO V. JOVELLANOS and LOURDES
R. JOVELLANOS, Respondents.
DECISION
PERLAS-BERNABE, J.:
Assailed in this petition for review on certiorari 1 are the
Decision2 dated May 29, 2009 and Resolution 3 dated
August 10, 2009 of the Court of Appeals (CA) in CA-G.R.
SP No. 104487 which reversed the Decision4 dated
217
Sales – Chapter 3-12 Cases
December 27, 2007 of the Regional Trial Court of Optimum reasonable compensation in the amount of parties thereunder as well as the governing law, among
Caloocan City, Branch 128 (RTC) in Civil Case No. C- ₱5,000.00 for its use and occupation until possession has others, Republic Act No. (RA) 6552.24
21867 that, in turn, affirmed the Decision5 dated June 8, been surrendered. It held that Sps. Jovellanos’s Accordingly, it concluded that the subject matter is one
2007 of the Metropolitan Trial Court, Branch 53 of that possession of the said property was by virtue of a which is incapable of pecuniary estimation and thus,
same city (MeTC) in Civil Case No. 06-28830 ordering Contract to Sell which had already been cancelled for within the jurisdiction of the RTC.25
respondents-spouses Benigno and Lourdes Jovellanos non-payment of the stipulated monthly installment Undaunted, Optimum moved for reconsideration which
(Sps. Jovellanos) to, inter alia, vacate the premises of the payments. As such, their "rights of possession over the was denied in a Resolution26 dated August 10, 2009.
property subject of this case. subject property necessarily terminated or expired and Hence, the instant petition, submitting that the case is one
The Facts hence, their continued possession thereof constitute[d] for unlawful detainer, which falls within the exclusive
On April 26, 2005, Sps. Jovellanos entered into a Contract unlawful detainer."16 original jurisdiction of the municipal trial courts, and not a
to Sell6 with Palmera Homes, Inc. (Palmera Homes) for Dissatisfied, Sps. Jovellanos appealed to the RTC, case incapable of pecuniary estimation cognizable solely
the purchase of a residential house and lot situated in claiming that Optimum counsel made them believe that a by the regional trial courts.
Block 3, Lot 14, Villa Alegria Subdivision, Caloocan City compromise agreement was being prepared, thus their The Court’s Ruling
(subject property) for a total consideration of decision not to engage the services of counsel and their The petition is meritorious. What is determinative of the
₱1,015,000.00. Pursuant to the contract, Sps. Jovellanos concomitant failure to file an answer.17 nature of the action and the court with jurisdiction over it
took possession of the subject property upon a down They also assailed the jurisdiction of the MeTC, claiming are the allegations in the complaint and the character of
payment of ₱91,500.00, undertaking to pay the remaining that the case did not merely involve the issue of physical the relief sought, not the defenses set up in an answer.27
balance of the contract price in equal monthly installments possession but rather, questions arising from their rights A complaint sufficiently alleges a cause of action for
of ₱13,107.00 for a period of 10 years starting June 12, under a contract to sell which is a matter that is incapable unlawful detainer if it recites that:
2005.7 of pecuniary estimation and, therefore, within the (a) initially, possession of the property by the defendant
On August 22, 2006, Palmera Homes assigned all its jurisdiction of the RTC.18 was by contract with or by tolerance of the plaintiff;
rights, title and interest in the Contract to Sell in favor of The RTC Ruling (b) eventually, such possession became illegal upon
petitioner Optimum Development Bank (Optimum) through In a Decision19 dated December 27, 2007, the RTC notice by plaintiff to defendant of the termination of the
a Deed of Assignment of even date.8 affirmed the MeTC’s judgment, holding that the latter did latter's right of possession;
On April 10, 2006, Optimum issued a Notice of not err in refusing to admit Sps. Jovellanos’ s belatedly (c) thereafter, defendant remained in possession of the
Delinquency and Cancellation of Contract to Sell 9 for Sps. filed answer considering the mandatory period for its filing. property and deprived plaintiff of the enjoyment thereof;
Jovellanos’s failure to pay their monthly installments It also affirmed the MeTC’s finding that the action does not and
despite several written and verbal notices.10 involve the rights of the respective parties under the (d) within one year from the last demand on defendant to
In a final Demand Letter dated May 25, 2006, 11 Optimum contract but merely the recovery of possession by vacate the property, plaintiff instituted the complaint for
required Sps. Jovellanos to vacate and deliver possession Optimum of the subject property after the spouses’ ejectment.28
of the subject property within seven (7) days which, default.20 Corollarily, the only issue to be resolved in an unlawful
however, remained unheeded. Hence, Optimum filed, on Aggrieved, Sps. Jovellanos moved for reconsideration detainer case is physical or material possession of the
November 3, 2006, a complaint for unlawful which was, however, denied in a Resolution21 dated June property involved, independent of any claim of ownership
detainer12 before the MeTC, docketed as Civil Case No. 27, 2008. Hence, the petition before the CA reiterating by any of the parties involved.29
06-28830. Despite having been served with summons, that the RTC erred in affirming the decision of the MeTC In its complaint, Optimum alleged that it was by virtue of
together with a copy of the complaint,13 Sps. Jovellanos with respect to: the April 26, 2005 Contract to Sell that Sps. Jovellanos
failed to file their answer within the prescribed (a) the non-admission of their answer to the complaint; were allowed to take possession of the subject property.
reglementary period, thus prompting Optimum to move for and However, since the latter failed to pay the stipulated
the rendition of judgment.14 (b) the jurisdiction of the MeTC over the complaint for monthly installments, notwithstanding several written and
Thereafter, Sps. Jovellanos filed their opposition with unlawful detainer.22 verbal notices made upon them, it cancelled the said
motion to admit answer, questioning the jurisdiction of the The CA Ruling contract as per the Notice of Delinquency and
court, among others. Further, they filed a Motion to In an Amended Decision23 dated May 29, 2009, the CA Cancellation dated April 10, 2006. When Sps. Jovellanos
Reopen and Set the Case for Preliminary Conference, reversed and set aside the RTC’s decision, ruling to refused to vacate the subject property despite repeated
which the MeTC denied. dismiss the complaint for lack of jurisdiction. It found that demands, Optimum instituted the present action for
The MeTC Ruling the controversy does not only involve the issue of unlawful detainer on November 3, 2006, or within one
In a Decision15 dated June 8, 2007, the MeTC ordered possession but also the validity of the cancellation of the year from the final demand made on May 25, 2006.
Sps. Jovellanos to vacate the subject property and pay Contract to Sell and the determination of the rights of the
218
Sales – Chapter 3-12 Cases
While the RTC upheld the MeTC’s ruling in favor of Interpretation of the contract between the plaintiff and the this case is one which has for its object real property to be
Optimum, the CA, on the other hand, declared that the defendant is inevitable because it is the contract that sold on an installment basis, the said contract is especially
MeTC had no jurisdiction over the complaint for unlawful initially granted the defendant the right to possess the governed by – and thus, must be examined under the
detainer, reasoning that the case involves a matter which property; it is this same contract that the plaintiff provisions of – RA 6552, or the "Realty Installment Buyer
is incapable of pecuniary estimation – i.e., the validity of subsequently claims was violated or extinguished, Protection Act", which provides for the rights of the buyer
the cancellation of the Contract to Sell and the terminating the defendant’s right to possess. We ruled in in case of his default in the payment of succeeding
determination of the rights of the parties under the Sps. Refugia v. CA that – where the resolution of the installments. Breaking down the provisions of the law, the
contract and law – and hence, within the jurisdiction of the issue of possession hinges on a determination of the Court, in the case of Rillo v. CA,40 explained the
RTC. The Court disagrees. Metropolitan Trial Courts are validity and interpretation of the document of title or any mechanics of cancellation under RA 6552 which are
conditionally vested with authority to resolve the question other contract on which the claim of possession is based mainly on the amount of installments already paid
of ownership raised as an incident in an ejectment case premised, the inferior court may likewise pass upon these by the buyer under the subject contract, to wit:41
where the determination is essential to a complete issues. Given the nature of the contract of the parties, the
adjudication of the issue of possession.30 Concomitant to The MeTC’s ruling on the rights of the parties based on its respondent court correctly applied Republic Act No. 6552.
the ejectment court’s authority to look into the claim of interpretation of their contract is, of course, not conclusive, Known as the Maceda Law, R.A. No. 6552 recognizes in
ownership for purposes of resolving the issue of but is merely provisional and is binding only with respect conditional sales of all kinds of real estate (industrial,
possession is its authority to interpret the contract or to the issue of possession. (Emphases supplied; citations commercial, residential) the right of the seller to cancel the
agreement upon which the claim is premised. Thus, in the omitted) contract upon non-payment of an installment by the buyer,
case of Oronce v. CA,31 wherein the litigants’ opposing In the case at bar, the unlawful detainer suit filed by which is simply an event that prevents the obligation of the
claims for possession was hinged on whether their written Optimum against Sps. Jovellanos for illegally withholding vendor to convey title from acquiring binding force. It also
agreement reflected the intention to enter into a sale or possession of the subject property is similarly premised provides the right of the buyer on installments in case he
merely an equitable mortgage, the Court affirmed the upon the cancellation or termination of the Contract to Sell defaults in the payment of succeeding installments, viz.:
propriety of the ejectment court’s examination of the terms between them. Indeed, it was well within the jurisdiction of (1) Where he has paid at least two years of installments,
of the agreement in question by holding that, "because the MeTC to consider the terms of the parties’ agreement (a) To pay, without additional interest, the unpaid
metropolitan trial courts are authorized to look into the in order to ultimately determine the factual bases of installments due within the total grace period earned by
ownership of the property in controversy in ejectment Optimum’s possessory claims over the subject property. him, which is hereby fixed at the rate of one month grace
cases, it behooved MTC Branch 41 to examine the bases Proceeding accordingly, the MeTC held that Sps. period for every one year of installment payments made:
for petitioners’ claim of ownership that entailed Jovellanos’s non-payment of the installments due had Provided, That this right shall be exercised by the buyer
interpretation of the Deed of Sale with Assumption of rendered the Contract to Sell without force and effect, thus only once in every five years of the life of the contract and
Mortgage."32 Also, in Union Bank of the Philippines v. depriving the latter of their right to possess the property its extensions, if any. (b) If the contract is cancelled, the
Maunlad Homes, Inc.33 (Union Bank), citing Sps. Refugia subject of said contract.36 The foregoing disposition aptly seller shall refund to the buyer the cash surrender value of
v. CA,34 the Court declared that MeTCs have authority to squares with existing jurisprudence. As the Court similarly the payments on the property equivalent to fifty per cent of
interpret contracts in unlawful detainer cases, viz.: 35 held in the Union Bank case, the seller’s cancellation of the total payments made and, after five years of
The authority granted to the MeTC to preliminarily resolve the contract to sell necessarily extinguished the buyer’s installments, an additional five per cent every year but not
the issue of ownership to determine the issue of right of possession over the property that was the subject to exceed ninety per cent of the total payments made:
possession ultimately allows it to interpret and enforce the of the terminated agreement.37 Provided, That the actual cancellation of the contract shall
contract or agreement between the plaintiff and the Verily, in a contract to sell, the prospective seller binds take place after cancellation or the demand for rescission
defendant. To deny the MeTC jurisdiction over a himself to sell the property subject of the agreement of the contract by a notarial act and upon full payment of
complaint merely because the issue of possession exclusively to the prospective buyer upon fulfillment of the the cash surrender value to the buyer.
requires the interpretation of a contract will effectively rule condition agreed upon which is the full payment of the Down payments, deposits or options on the contract shall
out unlawful detainer as a remedy. As stated, in an action purchase price but reserving to himself the ownership of be included in the computation of the total number of
for unlawful detainer, the defendant’s right to possess the the subject property despite delivery thereof to the installments made.
property may be by virtue of a contract, express or prospective buyer.38 (2) Where he has paid less than two years in installments,
implied; The full payment of the purchase price in a contract to sell Sec. 4. x x x the seller shall give the buyer a grace period
corollarily, the termination of the defendant’s right to is a suspensive condition, the non-fulfillment of which of not less than sixty days from the date the installment
possess would be governed by the terms of the same prevents the prospective seller’s obligation to convey title became due. If the buyer fails to pay the installments due
contract. from becoming effective,39 as in this case. Further, it is at the expiration of the grace period, the seller may cancel
significant to note that given that the Contract to Sell in the contract after thirty days from receipt by the buyer of
219
Sales – Chapter 3-12 Cases
the notice of cancellation or the demand for rescission of are SET ASIDE. The Decision dated June 8, 2007 of
the contract by a notarial act. (Emphasis and underscoring Metropolitan Trial Court, Branch 53, Caloocan City in Civil
supplied) Case No. 06-28830 is hereby REINSTATED.
Pertinently, since Sps. Jovellanos failed to pay their SO ORDERED.
stipulated monthly installments as found by the MeTC, the
Court examines Optimum’s compliance with Section 4 of
RA 6552, as above-quoted and highlighted, which is the
provision applicable to buyers who have paid less than
two (2) years-worth of installments. Essentially, the said
provision provides for three (3) requisites before the seller
may actually cancel the subject contract: first, the seller
shall give the buyer a 60-day grace period to be
reckoned from the date the installment became
due; second, the seller must give the buyer a notice of
cancellation/demand for rescission by notarial act if
the buyer fails to pay the installments due at the expiration
of the said grace period; and third, the seller may actually
cancel the contract only after thirty (30) days from the
buyer’s receipt of the said notice of cancellation/demand
for rescission by notarial act. In the present case, the 60-
day grace period automatically operated42 in favor of the
buyers, Sps. Jovellanos, and took effect from the time that
the maturity dates of the installment payments lapsed.
With the said grace period having expired bereft of any
installment payment on the part of Sps.
Jovellanos,43 Optimum then issued a notarized Notice of
Delinquency and Cancellation of Contract on April 10,
2006. Finally, in proceeding with the actual cancellation of
the contract to sell, Optimum gave Sps. Jovellanos an
additional thirty (30) days within which to settle their
arrears and reinstate the contract, or sell or assign their
rights to another.44
It was only after the expiration of the thirty day (30) period
did Optimum treat the contract to sell as effectively
cancelled – making as it did a final demand upon Sps.
Jovellanos to vacate the subject property only on May 25,
2006. Thus, based on the foregoing, the Court finds that
there was a valid and effective cancellation of the Contract
to Sell in accordance with Section 4 of RA 6552 and since
Sps. Jovellanos had already lost their right to retain
possession of the subject property as a consequence of
such cancellation, their refusal to vacate and turn over
possession to Optimum makes out a valid case for
unlawful detainer as properly adjudged by the MeTC.
WHEREFORE, the petition is GRANTED. The Decision
dated May 29, 2009 and Resolution dated August 10,
2009 of the Court of Appeals in CA-G.R. SP No. 104487
220
Sales – Chapter 3-12 Cases
vs. within which to pay the arrears and failure to do so would
CITIHOMES BUILDER AND DEVELOPMENT, authorize Citihomes to consider the contract as cancelled.
INC., Respondent. On June 15, 2009, Citihomes sent its final demand letter
DECISION asking Spouses Noynay to vacate the premises due to
MENDOZA, J.: their continued failure to pay the arrears. Spouses Noynay
In this petition for review on certiorari1 under Rule 45 of did not heed the demand, forcing Citihomes to file the
the Rules of Court, Spouses Noel and Michelle Noynay complaint for unlawful detainerbefore the MTCC on July
(Spouses Noynay) assail the July 16, 2012 Decision 2 of 29, 2009.
the Court of Appeals (CA) and October 15, 2012 In the said complaint, Citihomes alleged that as per
Resolution,3 which affirmed with modification the Statement of Account as of March 18, 2009,
September 17, 20 I 0 Decision4 of the Regional Trial SpousesNoynay had a total arrears in the amount of
CoUii, Branch 21, Malolos, Bulacan ( RTCJ. Earlier, the ₱272,477.00, inclusive of penalties. Thus, Citihomes
RTC reversed the March 26, 2010 Decision 5 of the prayed that Spouses Noynay be ordered to vacate the
Municipal Trial Court for Cities, San Jose Del Monte, subject property and pay the amount of ₱8,715.97 a
Bulacan (MTCC). which dismissed the complaint6 for month as a reasonable compensation for the use and
unlawful detainer filed by Citihomes Builder and occupancy to commence from January 8, 2007 until
Development, Inc. (Citihomes) against Spouses Noynay Spouses Noynay vacate the same.
for lack of cause of action. In its March 26, 2010 Decision,10 the MTCC dismissed the
The Facts: complaint. It considered the annotation in the certificate of
On December 29, 2004, Citihomes and Spouses Noynay title, which was dated prior to the filing of the complaint,
executed a contract to sell7 covering the sale of a house which showed that Citihomes had executed the
and lot located in San Jose Del Monte, Bulacan, and Assignment favor of UCPB, as having the legal effect of
covered by Transfer Certificate of Title (TCT) No. T- divesting Citihomes of its interest and right over the
43469. Under the terms of the contract, the price of the subject property. As far as the MTCC was concerned,
property was fixed at ₱915,895.00, with a downpayment Citihomes did not have a cause of action against Spouses
of ₱183,179.00, and the remaining balance to be paid in Noynay.
120 equal monthly installments with an annual interest The RTC, however, reversedthe ruling of the MTCC. In its
rate of 21% commencing on February 8, 2005 and every September 17, 2010 Decision,11 the RTC stated that the
8th day of the month thereafter. MTCC erred in interpreting the deed of assignment as
Subsequently, on May 12, 2005, Citihomes executed the having the effect of relinquishing all of Citihomes’ rights
Deed of Assignment of Claims and over the subject property. The RTC explained that the
Accounts8(Assignment)in favor of United Coconut assignment was limited only to the installment accounts
Planters Bank (UCPB) on May 12, 2005. Under the said receivables due from Spouses Noynay and did not include
agreement, UCPB purchased from Citihomes various the transfer of title or ownership over the property. It
accounts, including the account of Spouses Noynay, for a pointed out that Citihomes remained as the registered
consideration of ₱100,000,000.00. In turn, Citihomes owner of the subject property, and so it had the right to
assigned its rights, titles, interests, and participation in ask for the eviction of Spouses Noynay. As to the issue of
various contracts to sell with its buyers to UCPB. who had the better right of possession, the RTC ordered
In February of 2007, Spouses Noynay allegedly started to that the records be remanded to the MTCC for the proper
default in their payments. Months later, Citihomes decided determination.
to declare Spouses Noynay delinquent and to cancel the Spouses Noynay then went to the CA. On July 16, 2012,
contract considering that nine months of agreed the CA affirmed the conclusion of the RTC that Citihomes
amortizations were left unpaid. On December 8, 2007, the still had the right and interest over the property in its
notarized Notice of Delinquency and Cancellation of the capacity as the registered owner. Moreover, the issue on
G.R. No. 204160 September 22, 2014 Contract To Sell,9 dated November 21, 2007, was who, between the parties had a better possessory right
SPOUSES MICHELLE M. NOYNAY and NOEL S. received by Spouses Noynay. They were given 30 days over the property, was resolved infavor of Citihomes.
NOYNAY, Petitioners,
221
Sales – Chapter 3-12 Cases
In disposing the issue of possession, the CA primarily Position of Citihomes The determination of whether Citihomes has a right to ask
recognized the relevance of Republic Act (R.A.)No. 6552, Citihomes counters that it has the right to ask for the for the eviction of Spouses Noynay entirely depends on
otherwise known as the Realty Installment Buyer Act eviction of the petitioners in its capacity as the registered the review of the Assignment of Claims and Accounts it
(Maceda Law), in determining the limits of the right to owner despite the assignment of rights it made to UCPB. executed in favor of UCPB. If it turns out that what was
possess of Spouses Noynay in their capacity as defaulting It believes that because Spouses Noynay failed to pay at assigned merely covered the collectible amounts or
buyers in a realty installment scheme. Under the said law, least two (2) years of installments, the cancellation receivables due from Spouses Noynay, Citihomes would
the cancellation of a contract would only follow if the became effective upon the expiration of the 30-day necessarily have the right to demand the latter’s eviction
requirements set forth therein had been complied with, periodfollowing the receipt of the notice of delinquency asonly an aspect of the contract to sell passed on to
particularly the giving of a "notice of delinquency and and cancellation notice and without the need for the UCPB. Simply put, because an assignment covered only
cancellation of the contract" to the defaulting party and,in payment of the cash surrender value under Section 3(b) of credit dues, the relation between Citihomes as the seller
some cases, the payment to the buyer of the cash the Maceda Law. and Spouses Noynay as the buyer under their Contract to
surrender value if at least two years of installments had Ruling of the Court Sell remained. If on the other hand, it appears that the
been paid. The CA noted that Spouses Noynay failed to Cause of action has been defined as an act or omission assignment covered all of Citihomes’ rights, obligations
complete the minimum two (2) years of installment, by which a party violates a right of another.12 It requires and benefits in favor ofUCPB, the conclusion would
despite the allegation that three (3) years of amortizations the existence of a legal right on the part of the plaintiff, a certainly be different.
had already been paid. As an effect, the CA pronounced correlative obligation of the defendant to respect such Under the provisions of the Assignment, it was stipulated
that the termination of the contract was validly effected by right, and an act or omission of such defendant in violation that:
the expiration of the 30-day period from the time the of the plaintiff’s rights.13 A complaint should not be NOW, THEREFORE, for and in consideration of the
notice of cancellation was received by Spouses Noynay. dismissed for insufficiency of cause of action if it appears foregoing premises, the ASSIGNOR hereby agrees as
From thatmoment, the CA treated Spouses Noynay to clearly from the complaint and its attachments that the follows:
have lost the right to possess the property. In addition, the plaintiff is entitled to relief.14 The complaint, however, may 1. The ASSIGNOR hereby assigns, transfers and sets
CA made Spouses Noynay liable for the payment of be dismissed for lack of cause of action laterafter over unto the ASSIGNEE all its rights, titles and interest in
monthly rentals from the time their possession became questions of fact have been resolved on the basis of and to, excluding its obligations under the Contract/s to
illegal. stipulations, admissions or evidence presented.15 Sellenumerated and described in the List of Assigned
Spouses Noynay moved for reconsideration, but the CA Relative thereto, a plaintiff in an unlawful detainer case Receivables which is hereto attached and marked as
denied their motion. which seeks recovery of the property must prove one’s Annex "A" hereof, including any and all sum of money due
Hence, this petition. legal right to evict the defendant, a correlative obligation and payable to the ASSIGNOR, the properties pertaining
ISSUE on the part of such defendant to respect the plaintiff’s right thereto,all replacements, substitution, increases and
The lone issue presented for resolution is whether to evict, and the defendant’s act or omission in the form of accretion thereof and thereto which the ASSIGNOR has
Citihomes has a cause of action for ejectment against refusal to vacate upon demand when his possession executed with the Buyers, as defined in the Agreement,
Spouses Noynay. In effect, Spouses Noynay would have ultimately becomes illegal. and all moneysdue, or which may grow upon the sales
this Court determine whether Citihomes may rightfully At first glance, the main thrust of the discussion in the therein set forth.
evict them. lower courts is the issue on whether Citihomes had 2. For purposes of this ASSIGNMENT, the ASSIGNOR
Position of Spouses Noynay suchright to evict Spouses Noynay. At its core is the ruling hereby delivers to the ASSIGNEE, which hereby
Spouses Noynay insist that by virtue of the assignment of of the MTCC thatthe right to demand the eviction of acknowledges receipt of the following documents
rights which Citihomes executed in favor of UCPB, Spouses Noynay was already transferred to UCPB from evidencing the ASSIGNOR’s title, right, interest,
Citihomes did not have a cause of action against them the moment the Assignment was executed by Citihomes, participation and benefit in the assigned Installment
because it no longerhad an interest over the subject which was done prior to the institution of the unlawful Account Receivables listed in Annex "A" and made as
property. Contrary to the findings of the CA, the monthly detainer case. Thus, based on the evidence presented integral part hereof.
installments amounting to three years were already paid, during the trial, the MTCC held that Citihomes did not a) Original Contracts to Sell
by reason of which, Section 3(b) of the Maceda Law have a cause of action against Spouses Noynay.The RTC b) Transfer Certificates of Title
should apply. This means that for the cancellation to be held otherwise justifying that Citihomes may still be the 3. The ASSIGNOR, hereby irrevocably appoints the
effective, the cash surrender value should have been paid right party to evict Spouses Noynay in its capacity as the ASSIGNEE to be its true and lawful agent or
first to them by Citihomes; and that because no payment registered owner of the property. The CA affirmed the representativefor it and in its name and stead, but for such
was made, it follows that no valid cancellation could also RTC on this point. ASSIGNEE’s own benefit: (1) to sell, assign, transfer, set
be effected. Thisallegedly strengthened their right to the The Court, however, agrees with the MTCC. over, pledge, compromise or discharge the whole, or any
possession of the property even to this day. part, of said assignment; (2) to do all acts and things
222
Sales – Chapter 3-12 Cases
necessary, or proper, for any such purpose; (3) to ask, residential house and lot subject of this agreement in favor detainer case. Considering,however, that the right to
collect, receive and sue for the moneys due, or which may of other persons as if this agreement had never been cancel was already assigned prior to the commencement
grow due, upon the said Assignment;and (4) to substitute entered into. of this controversy with the execution of the Assignment,
one person, or more, with like powers; hereby ratifying WHERE the BUYER has paid atleast two (2) years of its legal consequences cannotbe avoided.
and confirming all that said agent or representative, or his installments and he defaults in the payment of three (3) Well-established is the rule that the assignee is deemed
substitute, or substitutes, shall lawfully do, by virtue consecutive monthly installments, the SELLER shall be subrogated to the rights as well as to the obligations of the
hereof.16 entitled: seller/assignor. By virtue of the deed of assignment, the
[Emphases supplied] a. To pay, without additional interest, the unpaid assignee is deemed subrogated to the rights and
Clearly, the conclusion of the MTCC had factual and legal installment due within the total grace period earned by the obligations of the assignor and is bound by exactly the
bases. Evident from the tenor of the agreement was the BUYER which is fixed at the rate of one (1) month grace same conditions as those which bound the
intent on the part of Citihomes, as assignor, to assign all period for every one (1) year of installment payment assignor.18 What can be inferred from here is the effect on
of its rights and benefits in favor of UCPB. Specifically, made; Provided, that this right shall be exercised by the the status of the assignor relative to the relations
what Citihomes did was an assignment or transfer of all BUYER only once for every five (5) years of the life of this established by a contract which has been subsequently
contractual rights arising from various contracts to sell, agreement. assigned; that is, the assignor becomes a complete
including the subject contract to sell, with all the rights, b. If this agreement is cancelled, the SELLER shall refund stranger to all the mattersthat have been conferred to the
obligations and benefits appurtenant thereto in favor of to the BUYER the cash surrender value of the payments assignee.
UCPB for a consideration of ₱100,000,000.00. Indeed, equivalent to fifty percent thereof and, after five years of In this case, the execution of the Assignment in favor of
the intent was more than just an assignment of credit. This installments, an additional five percent (5%) for every year UCPB relegated Citihomes to the status of a mere
intent to assign all rights under the contract to sell was but not to exceed ninety (90%) of the total payments stranger to the jural relations established under the
even fortified by the delivery of documents such as the made; Provided, that the actual cancellation of this contractto sell. With UCPB as the assignee, it is clear that
pertinent contracts to sell and the TCTs. Had it been the agreement shall take place after thirty (30) days from Citihomes has ceased tohave any right to cancel the
intent of Citihomes to assign merely its interest in the receipt by the BUYER of the notice of cancellation or contract to sell with Spouses Noynay. Without this
receivables due from Spouses Noynay, the tenor of the demand for rescission by a notarial act and upon full right,which has been vested in UCPB, Citihomes
deed of assignment would have been couched in very payment of the cash surrender value to the BUYER. undoubtedly had no cause ofaction against Spouses
specific terms. xxx xxx xxx Noynay.
Included in those matters which were handed over to The BUYER, at the termination of the contract, shall This is not to say that Citihomes lost all interest over the
UCPB were the provisions outlined in Section 6 of the promptly surrender the said property to the SELLER, and property. To be clear, what were assigned covered only
Contract to Sell. In the said provision, Citihomes, as the should the former fail to comply with the provision, on top the rights inthe Contract to Sell and not the property rights
seller has been given the right to cancel the contract to of the remedy provided for above, the BUYER hereby over the house and lot, which remained registered under
sell in cases of continuing default by Spouses Noynay, to expressly appoints the SELLER as their duly authorized Citihomes’ name. Considering, however, that the unlawful
wit: SECTION 6. If for any reason, whatsoever, the attorney-in-fact with power and authority to open, enter detainer case involves mere physical or
BUYER fails to pay three (3) consecutive monthly and take full possession of the property in the presence of materialpossession of the property and is independent of
installments, the provision of RA No. 6552 shall apply. any peace officer and to take an inventory of the any claim of ownership by any of the parties,19 the
Where the BUYER has paid less than two (2) years of equipment, furniture, merchandise and effect. In case the invocation of ownership by Citihomes is immaterial in the
installments and defaults in the payment of three (3) BUYER fails to claim the said equipment, furniture, just determination of the case.
consecutive monthly installment, he shall be given a grace merchandise and effects and/or liquidate their liabilities Granting that the MTCC erred in ruling that Citihomes had
period of not less than sixty (60) days from the date the with the SELLER within thirty (30) days from the date of no cause of action by reason of the Assignment it made in
installment payments became due and payable within transfer of possession of the property to the latter, the favor of UCPB, the Court still upholds the right of the
which to pay the installments and/or make payments in SELLER is hereby given the right to dispose of said Spouses Noynay to remain undisturbed in the possession
arrears together with the installments corresponding to the property in a private or public sale and to apply the of the subject property. The reason is simple – Citihomes
months of the grace period. In the event the BUYER proceeds to whatever expenses it may have incurred in failed to comply with the procedures for the proper
continues to default in the payment of the installments line with the warehousing of the equipment, furniture, cancellation of the contract to sell as prescribed by
within or at the expiration of the grace period herein merchandise and effects.17 Maceda Law.
provided, the SELLER shall have the right to cancel this The exercise of such right to cancel necessarily In Pagtalunan v. Manzano,20 the Court stressed the
agreement thirty (30) days from the BUYER’s receipt of determines the existence of the right to evict Spouses importance of complying with the provisions of the
the notice of cancellation or demand for rescission by a Noynay. The existence of the right to evict is the first Maceda Law as to the cancellation of contracts to sell
notarial act. Thereafter, the SELLER may dispose of the constitutive element ofthe cause of action in this unlawful involving realty installment schemes. There it was held
223
Sales – Chapter 3-12 Cases
that the cancellation of the contract by the seller must be full downpayment shall be paid upon signing of this such admission and are conclusive as to such party, and
in accordance with Section 3 (b) of the Maceda Law, contract. all proofs to the contrary or inconsistent therewith should
which requires the notarial act of rescission and the refund b. The balanceof the total purchase price in the amount of be ignored, whether objection is interposed by the party or
to the buyer of the full payment of the cash surrender SEVEN HUNDRED THIRTY TWO THOUSAND SEVEN not. The allegations, statements or admissions contained
value of the payments made on the property. The actual HUNDRED SIXTEEN PESOS ONLY, (₱732,716..00), in a pleading are conclusive as against the pleader. A
cancellation of the contract takes place after thirty (30) Philippine Currency shall be paid by the BUYER in 120 party cannot subsequently take a position contrary of or
days from receipt by the buyer of the notice of cancellation equal monthly installments in the amount of ₱14,649.31 inconsistent with what was pleaded. (Citations omitted)
or the demand for rescission of the contract by a notarial per month with an interest of 21% per annum to [Emphases supplied]
act and upon fullpayment of the cash surrender value to commenceon 02.08.05 and every 8th day of the month Here, Spouses Noynay proposed for stipulation the factual
the buyer, to wit: (b) If the contract is cancelled, the seller thereafter.21 allegation that they had been paying Citihomesthe
shall refund to the buyer the cash surrender value of the Citihomes claimed that the period ofthe payment of the monthly amortization of the property for more than three
payments on the propertyequivalent to fifty percent of the amortizations started from May 31, 2005.22 As can be (3) years and only stopped payment by January 8, 2008.
total payments made and, after five years of installments, gleaned from the contract to sell, however, it appears that In the Preliminary Conference Order,25 dated January 28,
an additional five percent every year but not to exceed the payment of the downpayment started from the signing 2010, the MTCC noted the said factas admitted, to wit:
ninety percent of the total payments made: Provided, That thereof on December 29, 2004. The defendants proposed the following matters for
the actual cancellation of the contract shall take place To this end, the factual admissions made by the parties stipulations:
after thirty days from receipt by the buyer of the notice of during the preliminary conference would shed light on the 1. That the defendants had already paid the plaintiff the
cancellation or the demand for rescission of the contract matter. It must be remembered that these judicial total amount of Php 633,000.00 – Not Admitted
bya notarial act and upon full payment of the cash admissions are legally binding on the party making the 2. That the defendants have beenpaying the plaintiff the
surrender value to the buyer. admissions. Similar to pre-trial admissions in a pre-trial monthly amortization of the property for more than three
[Emphases supplied] order in ordinary civil cases, the contents of the record of years and only stopped payment by January 8, 2008 –
According to the lower courts, Spouses Noynay failed to a preliminary conference control the subsequent course of Admitted.26
complete the two-year minimum period of paid the action, thereby, defining and limiting the issues to be xxx xxx x x x[Emphasis supplied]
amortizations, thus, the cancellation of the contract to sell tried. A contrary ruling would render useless the Moreover, based on the Statement of Account,27 dated
no longer required the payment of the cash surrender proceedings during the preliminary conference and would, March 18, 2009, Spouses Noynay started defaulting from
value. This conclusion rests on the allegation that the in fact, be antithetical to the very purpose of a preliminary January 8, 2008. This shows that prior to that date,
amortization payments commenced only on May 31, conference, which is, among others, to allow the parties to amortizations covering the 3-year period, which started
2005. If indeedit were true that the payments started only admit and stipulate on a given set of facts and to simplify with the downpayment, had been paid. This is consistent
on that date, Spouses Noynay would not have completed the issues involved.23 with the admission of Citihomes during the preliminary
the required two-year period to be entitled to the payment The fairly recent case of Oscar Constantino v. Heirs of conference. By its admission that Spouses Noynay had
of cash surrender value. Records, however, show Oscar Constantino,24 is most instructive: been paying the amortizations for three (3) years, there is
otherwise. The Contract to Sell, dated December 29, In Bayas, et al. v. Sandiganbayan, et al., this Court no reason to doubt Spouses Noynay's compliance with
2004, was very particularon the matter. It stipulated as emphasized that: the minimum requirement of two years payment of
follows: Once the stipulations are reduced into writing and signed amortization, entitling them to the payment of the cash
SECTION 1. NOW, THEREFORE, for and in by the parties and their counsels, they become binding on surrender value provided for by law and by the contract to
consideration of the sume of NINE HUNDRED FIFTEEN the parties who made them. They become judicial sell. To reiterate, Section 3(b) of the Maceda Law requires
THOUSAND EIGHT HUNDRED NINETY FIVE PESOS admissionsof the fact or facts stipulated. Even if placed at that for an actual cancellation to take place, the notice of
ONLY, (915,895.00) Philippine Currency, inclusive of a disadvantageous position, a party may not be allowed to cancellation by notarial act and the full payment of the
miscellaneous charges hereunder set forth, and of the rescind them unilaterally, it must assume the cash surrender value must be first received by the buyer.
foregoing premises, the SELLER hereby agrees to sell, consequences of the disadvantage.(citations omitted) Clearly, no payment of the cash surrender value was
cede and convey to the BUYER, their heirs, Moreover, in Alfelor v. Halasan,this Court declared that: made to Spouses Noynay. Necessarily, no cancellation of
administrators, and successors-in-interest, the A party who judicially admits a fact cannot later challenge the contract to selI could be considered as validly
aforedescribed residential house and lot or lot only under the fact as judicial admissions are a waiver of proof; effected.
the following terms and conditions: production of evidence is dispensed with. A judicial Without the valid cancellation of the contract, there is no
a. The amount of ONE HUNDRED EIGHTY THREE admissionalso removes an admitted fact from the field of basis to treat the possession of the property by Spouses
THOUSAND ONE HUNDRED SEVENTY NINE PESOS controversy. Consequently, an admission made in the Noynay as illegal.1âwphi1 In AMOSUP-PTGWO-ITF v.
ONLY (₱183,179.00), Philippine Currency, representing pleadings cannot be controverted by the party making Decena,28 the Court essentially held that such similar
224
Sales – Chapter 3-12 Cases
failure' to validly cancel the contract, meant that the
possessor therein, similar to Spouses Noynay in this case,
remained entitled to the possession of the property. In the
said case, the Court stated:
In the parallel case of Pagtalunan v. Dela Cruz Vda. De
Manzano, which likewise originated as an action for
unlawful detainer, we affirmed the finding of the appellate
court that, since the contract to sell was not validly
cancelled or rescinded under Section 3(b) of R.A. No.
6552, the respondent therein had the right to continue
occupying unmolested the property subject thereof.
WHEREFORE, the petition is GRANTED. The July 16,
2012 Decision and October 15, 2012 Resolution of the
Court of Appeals are hereby REVERSED and SET
ASIDE. The March 26, 2010 Decision of the Municipal
Trial Court for Cities is REINSTATED.
SO ORDERED.

G.R. No. 178584 October 8, 2012


ASSOCIATED MARINE OFFICERS AND SEAMEN'S
UNION OF THE PHILIPPINES PTGWO-ITF, Petitioner,
vs.
NORIEL DECENA, Respondent.
DECISION
225
Sales – Chapter 3-12 Cases
PERLAS-BERNABE, J.: case for unlawful detainer, docketed as Civil Case No. and effective cancellation of a contract to sell under
This Petition for Review on Certiorari seeks the reversal of 12108 before the Municipal Trial Court (MTC) of Section 3 (b) of R.A. No. 6552: (1) to send a notarized
the Decision1 of the Court of Appeals (CA) dated July 31, Dasmariñas, Cavite. notice of cancellation, and (2) to refund the cash
2006, as well as the Resolution2 dated June 20, 2007, The Ruling of the MTC surrender value of the payments on the property.
which dismissed the complaint for unlawful detainer filed On December 4, 2002, the MTC found petitioner's case Consequently, it held that the contract to sell still subsists,
by petitioner against respondent on the ground of meritorious and, thus, rendered judgment9 ordering at least until properly rescinded, and the action for
prematurity, as petitioner has not shown that it complied respondent to (1) vacate the premises; (2) pay monthly ejectment filed by petitioner is premature.19
with the mandatory requirements for a valid and effective rental in the amount of ₱ 8,109.00 from August 1999 with Aggrieved, petitioner filed a motion for reconsideration,
cancellation of the contract to sell a house and lot. legal interests thereon until he has actually and fully paid which was denied by the CA in its Resolution 20 dated June
The Factual Antecedents the same; and (3) pay attorney's fees in the amount of ₱ 20, 2007. Hence, petitioner is now before this Court
Associated Marine Officers and Seamen's Union of the 30,000.00, as well as the costs of suit. alleging that –
Philippines – The Ruling of the RTC The Issues
PTGWO-ITF (petitioner) is a duly registered labor On appeal (App. Civil Case No. 312-03), the Regional 1. The Honorable Court of Appeals erred in changing the
organization engaged in an on-going Shelter Program, Trial Court (RTC) of Imus, Cavite, affirmed10 in toto the main issue to be resolved in the instant unlawful detainer
which offers residential lots and fully-furnished houses to decision of the MTC after finding that the cancellation and case from who has the better right of possession to
its members-seafarers under a reimbursement scheme revocation of the contract for failure of respondent to remit whether or not the agreement between the parties is a
requiring no down payment and no interest on the 25 monthly reimbursement payments converted the contract of lease or a contract to sell, especially when the
principal sum advanced for the acquisition and latter's stay on the premises to one of "mere nature of the agreement between the parties was never
development of the land and the construction of the permission"11by petitioner, and that respondent's refusal questioned nor raised as an issue in the court a quo.
house. to heed the notice to vacate the premises rendered his 2. Even assuming that the Honorable Court of Appeals
On April 27, 1995, petitioner entered into a contract3 under continued possession thereof unlawful.12 was correct in changing the main issue to be resolved, it
the Shelter Program with one of its members, Noriel With respect to the issue raised by respondent that the nevertheless erred in determining that:
Decena (respondent), allowing the latter to take instant case is covered by Republic Act No. 6552 (R.A. a. The agreement between the parties is allegedly one of
possession of a house and lot described as 7 STOLT No. 6552),13 the Maceda Law, the RTC ruled in the contract to sell – when the Housing and Land Use
MODEL, Lot 16, Block 7, in the Seamen's Village, Sitio negative, ratiocinating that the Shelter Contract Award is Regulatory Board itself already made a pronouncement
Piela, Barangay Paliparan, Dasmariñas, Cavite, with the neither a contract of sale nor a contract to sell. Rather, it is that the Shelter Program and its contract award is not a
obligation to reimburse petitioner the cost "more akin to a contract of lease with the monthly sale of real estate.
(US$28,563)4 thereof in 180 equal monthly payments. It reimbursements as rentals."14 b. The action for unlawful detainer filed by petitioner
was stipulated in said contract that, in case respondent The Ruling of the Court of Appeals AMOSUP is allegedly premature – especially considering
fails to remit three (3) monthly reimbursement payments, On petition for review (CA-G.R. SP No. 81954) before the that Republic Act No. 6552, which requires notarial notice
he shall be given a 3-month grace period within which to CA, the appellate court set aside the decision of the RTC of rescission, is not applicable to the case at bar and,
remit his arrears, otherwise, the contract shall be and entered a new judgment15 dismissing the complaint thus, the written notice of termination previously served on
automatically revoked or cancelled and respondent shall for unlawful detainer and restoring respondent to the the respondent is already sufficient.21
voluntarily vacate the premises without need of demand or peaceful possession of the subject house and lot. The CA The Ruling of the Court
judicial action.5 held that the contract between the parties is not a contract It is basic that a contract is what the law defines it to be,
Subsequently, respondent failed to pay twenty-five (25) of lease, but a contract to sell, which stipulates that upon and not what it is called by the contracting parties. A
monthly reimbursement payments covering the period full payment of the value of the house and lot, respondent contract to sell is defined as a bilateral contract whereby
August 1999 to August 2001, despite demands. Hence, shall become the owner thereof.16 The issues, which the prospective seller, while expressly reserving the
petitioner cancelled the contract and treated all his involve "the propriety of terminating the relationship ownership of the subject property despite delivery thereof
reimbursement payments as rental payments for his contracted by the parties, as well as the demand upon to the prospective buyer, binds itself to sell the said
occupancy of the house and lot. [respondent] to deliver the premises and to pay unpaid property exclusively to the prospective buyer upon
On August 21, 2001, petitioner sent respondent a notice reimbursements,"17 extend beyond those commonly fulfillment of the condition agreed upon, that is, full
of final demand6 requiring him to fulfill his obligation within involved in unlawful detainer suits, thus, converting the payment of the purchase price.22
a 30-day grace period. Thereafter, on October 18, 2001, instant case into one incapable of pecuniary estimation The Shelter Contract Award granted to respondent
his wife received a notice to vacate7 the premises. For exclusively cognizable by the RTC.18 expressly stipulates that "upon completion of payment of
failure of respondent to heed said notices, petitioner filed Moreover, the appellate court faulted petitioner for failing the amount of US$28,563 representing the full value of
a complaint before the barangay lupon and, eventually, a to comply with the mandatory twin requirements for a valid the House and Lot subject of the Contract Award, the
226
Sales – Chapter 3-12 Cases
UNION shall execute a Deed of Transfer and shall cause property equivalent to fifty per cent of the total payments b.) Sale or Sell – "sale" or "sell" shall include every
the issuance of the corresponding Transfer Certificate of made, and, after five years of installments, an additional disposition, or attempt to dispose, for a valuable
Title in favor of and in the name of the AWARDEE." 23 It five per cent every year but not to exceed ninety per cent consideration, of a subdivision lot, including the building
cannot be denied, therefore, that the parties herein of the total payments made: Provided, That the actual and other improvements thereon, if any, in a subdivision
entered into a contract to sell in the guise of a cancellation of the contract shall take place after thirty project or a condominium unit in a condominium project.
reimbursement scheme requiring respondent to make days from receipt by the buyer of the notice of cancellation "Sale" or "sell" shall include a contract to sell, a contract of
monthly reimbursement payments which are, in actuality, or the demand for rescission of the contract by a notarial purchase and sale, an exchange, an attempt to sell, an
installment payments for the value of the subject house act and upon full payment of the cash surrender value to option of sale or purchase, a solicitation of a sale, or an
and lot. the buyer. (Emphasis supplied) offer to sell, directly or by an agent, or by a circular letter,
While respondent occupied the subject premises, title As we emphasized in Pagtalunan, "R.A. No. 6552, advertisement or otherwise.
nonetheless remained with petitioner. Considering, otherwise known as the Realty Installment Buyer A privilege given to a member of a cooperative,
therefore, that the basis for such occupation is a contract Protection Act, recognizes in conditional sales of all kinds corporation, partnership, or any association and/or the
to sell the premises on installment, the contractual of real estate (industrial, commercial, residential) the right issuance of a certificate or receipt evidencing or giving the
relations between the parties are more than that of a of the seller to cancel the contract upon non-payment of right of participation in, or right to any land in
lessor-lessee.24 The appellate court thus correctly ruled an installment by the buyer, which is simply an event that consideration of payment of the membership fee or dues,
that the Shelter Contract Award has not been converted prevents the obligation of the vendor to convey title from shall be deemed a sale within the meaning of this
into one of lease. acquiring binding force." While we agreed that the definition.
Petitioner tried, albeit in vain, to mislead the Court that the cancellation of a contract to sell may be done outside of A reading of the Decision in its entirety reveals a
nature of the agreement between the parties, and even court, however, "the cancellation by the seller must be in vacillation on the part of the HLURB in classifying the
the validity of the termination thereof, were never raised in accordance with Sec. 3(b) of R.A. No. 6552, which transaction between petitioner and its members. While the
the trial courts. In the pre-trial brief filed by respondent requires a notarial act of rescission and the refund to the HLURB held that there is no sale as contemplated under
before the MTC, the first issue he presented is "whether buyer of the full payment of the cash surrender value of the first paragraph of the aforequoted provision "for the
or not the present action is a simple case of or an action the payments on the property."27 In the present case, as reason that there is no valuable consideration involved in
for unlawful detainer or an action for rescission of the aptly pointed out by the appellate court, petitioner failed to the transaction,"31 yet it went on to opine that the second
Contract of Shelter Award which is outside of the prove that the Shelter Contract Award had been cancelled paragraph of the same provision "appears to have an
jurisdiction of the Honorable Court."25 in accordance with R.A. No. 6552, which would have been apparent application in the instant case although the
In the parallel case of Pagtalunan v. Dela Cruz Vda. De the basis for the illegality of respondent's possession of same is not clear."32 Then, in its final disposition,33 the
Manzano,26 which likewise originated as an action for the subject premises. Hence, the action for ejectment HLURB required petitioner to secure a Certificate of
unlawful detainer, we affirmed the finding of the appellate must necessarily fail. Registration and License to Sell for its subdivision project
court that, since the contract to sell was not validly Petitioner nonetheless insists on the inapplicability of R.A. thereby effectively bringing it under the jurisdiction of said
cancelled or rescinded under Section 3(b) of R.A. No. No. 6552 in this case, capitalizing on the Decision28 of the office. Clearly, the argument of petitioner that respondent
6552, the respondent therein had the right to continue Housing and Land Use Regulatory Board in HLURB is not a realty installment buyer that needs to be protected
occupying unmolested the property subject thereof. CASE No. IV6-090902-1842 entitled "Seamen's Village by the law has no leg to stand on.
Section 3(b) reads: Brotherhood Homeowners Association, Inc. v. Associated In the interest, however, of putting an end to the
SEC. 3. In all transactions or contracts involving the sale Marine Officers And Seamen's Union of the Philippines controversy between the parties herein that had lasted for
or financing of real estate on installment payments, (AMOSUP)" which held that the transaction between more than ten (10) years, as in the cited case of
including residential condominium apartments but petitioner and the residents of Seamen's Village cannot be Pagtalunan, the Court orders respondent to pay his
excluding industrial lots, commercial buildings and sales considered a sale within the purview of Presidential arrears and settle the balance of the full value of the
to tenants under Republic Act Numbered Thirty-eight Decree (P.D.) No. 957.29 It should be pointed out that the subject premises. He had enjoyed the use thereof since
hundred forty-four, as amended by Republic Act only issue resolved in that case is "whether or not the 1995. After defaulting in August 1999, respondent had not
Numbered Sixty-three hundred eighty-nine, where the respondent (petitioner herein) is engaged in the business made any subsequent reimbursement payments. Thus,
buyer has paid at least two years of installments, the of selling real estate subdivisions, so as to fall under the for the delay in his reimbursement payments, we award
buyer is entitled to the following rights in case he defaults ambit of P.D. 957, the resolution of which would determine interest at the rate of 6% per annum on the unpaid
in the payment of succeeding installments: whether or not respondent is required under the law to balance applying Article 220934 of the Civil Code, there
xxx register with (the) Office and procure a license to sell." 30 being no stipulation in the Shelter Contract Award for such
(b) If the contract is canceled, the seller shall refund to the Section 2(b) of P.D. 957 defines a sale as follows: interest.35 For purposes of computing the legal interest,
buyer the cash surrender value of the payments on the the reckoning period should be the notice of final demand,
227
Sales – Chapter 3-12 Cases
conformably with Articles 116936 and 158937 of the same reasonable rental in the amount determined by the trial
Code, which, as found by the MTC, was sent by petitioner court.
to respondent on August 21, 2001.38 SO ORDERED.
In his Comment to the instant Petition, respondent
claimed that he had made payments in the amount of ₱
318,167.70.39 The total amount for reimbursement for the
subject house and lot is US$28,563, which the Shelter
Contract Award requires to be paid in "180 equal monthly
periodic reimbursements of US$159 or in equivalent
Philippine Currency at the time the same falls due." 40 For
lack of pertinent data with which to determine how many
months respondent's alleged total payment of ₱
318,167.70 is equivalent to, we direct petitioner to submit
to the trial court an accounting of the payments made by
respondent particularly showing the number of months he
was able to make the required payments of US$159 or its
peso equivalent. The balance of the full value of the
subject premises shall then be computed on the basis of
the following formula: (180 months minus the number of
months that respondent had already paid) multiplied by
US$159 or its peso equivalent at the time of payment.
WHEREFORE, the Decision of the Court of Appeals dated
July 31, 2006 and the Resolution dated June 20, 2007 are
hereby AFFIRMED with the following MODIFICATIONS:
1. The Municipal Trial Court of Dasmariñas, Cavite is
directed to conduct a hearing, within a maximum period of
thirty (30) days from receipt of this Decision, to determine:
(a) the unpaid balance of the full value of the subject
house and Jot; and (b) the reasonable amount of rental
for the subject property at present times.
2. Within sixty (60) days from the determination of the trial
court of said balance, respondent shall pay the amount
thereof to petitioner, with interest at six percent ( 6%) per
annum from August 1, 2001 up to the date of actual
payment;
3. Upon payment, petitioner shall execute a Deed of
Absolute Sale of the subject property and deliver the
transfer certificate of title in favor of respondent;
4. In case of failure to pay within the mandated 60-day
period, respondent shall immediately vacate the premises
without need of further demand. Petitioner, on the other
hand, shall pay respondent the cash surrender value
equivalent to 50% of the total reimbursement payments
made. The Shelter Contract Award shall then be deemed
cancelled thirty (30) days after receipt by respondent of
the full payment of the cash surrender value. If respondent
fails to vacate the premises, he shall be charged
228
Sales – Chapter 3-12 Cases
G.R. No. 107207 November 23, 1995 -and-
VIRGILIO R. ROMERO, petitioner, VIRGILIO R. ROMERO, married to Severina L. Lat, of
vs. Legal age, Filipino, and residing at 110 San Miguel St.,
HON. COURT OF APPEALS and ENRIQUETA CHUA Plainview Subd., Mandaluyong Metro Manila, hereinafter
VDA. DE ONGSIONG, respondents. referred to as the VENDEE:
W I T N E S S E T H : That
VITUG, J.: WHEREAS, the VENDOR is the owner of One (1) parcel
The parties pose this question: May the vendor demand of land with a total area of ONE THOUSAND NINE
the rescission of a contract for the sale of a parcel of land HUNDRED FIFTY TWO (1,952) SQUARE METERS,
for a cause traceable to his own failure to have the more or less, located in Barrio San Dionisio, Municipality
squatters on the subject property evicted within the of Parañaque, Province of Rizal, covered by TCT No.
contractually-stipulated period? 361402 issued by the Registry of Deeds of Pasig and
Petitioner Virgilio R. Romero, a civil engineer, was more particularly described as follows:
engaged in the business of production, manufacture and xxx xxx xxx
exportation of perlite filter aids, permalite insulation and WHEREAS, the VENDEE, for (sic) has offered to buy a
processed perlite ore. In 1988, petitioner and his foreign parcel of land and the VENDOR has accepted the offer,
partners decided to put up a central warehouse in Metro subject to the terms and conditions hereinafter stipulated:
Manila on a land area of approximately 2,000 square NOW, THEREFORE, for and in consideration of the sum
meters. The project was made known to several freelance of ONE MILLION FIVE HUNDRED SIXTY ONE
real estate brokers. THOUSAND SIX HUNDRED PESOS (P1,561,600.00)
A day or so after the announcement, Alfonso Flores and ONLY, Philippine Currency, payable by VENDEE to in to
his wife, accompanied by a broker, offered a parcel of (sic) manner set forth, the VENDOR agrees to sell to the
land measuring 1,952 square meters. Located in VENDEE, their heirs, successors, administrators,
Barangay San Dionisio, Parañaque, Metro Manila, the lot executors, assign, all her rights, titles and interest in and
was covered by TCT No. 361402 in the name of private to the property mentioned in the FIRST WHEREAS
respondent Enriqueta Chua vda. de Ongsiong. Petitioner CLAUSE, subject to the following terms and conditions:
visited the property and, except for the presence of 1. That the sum of FIFTY THOUSAND PESOS
squatters in the area, he found the place suitable for a (P50,000.00) ONLY Philippine Currency, is to be paid
central warehouse. upon signing and execution of this instrument.
Later, the Flores spouses called on petitioner with a 2. The balance of the purchase price in the amount of
proposal that should he advance the amount of ONE MILLION FIVE HUNDRED ELEVEN THOUSAND
P50,000.00 which could be used in taking up an SIX HUNDRED PESOS (P1,511,600.00) ONLY shall be
ejectment case against the squatters, private respondent paid 45 days after the removal of all squatters from the
would agree to sell the property for only P800.00 per above described property.
square meter. Petitioner expressed his concurrence. On 3. Upon full payment of the overall purchase price as
09 June 1988, a contract, denominated "Deed of aforesaid, VENDOR without necessity of demand shall
Conditional Sale," was executed between petitioner and immediately sign, execute, acknowledged (sic) and deliver
private respondent. The simply-drawn contract read: the corresponding deed of absolute sale in favor of the
DEED OF CONDITIONAL SALE VENDEE free from all liens and encumbrances and all
KNOW ALL MEN BY THESE PRESENTS: Real Estate taxes are all paid and updated.
This Contract, made and executed in the Municipality of It is hereby agreed, covenanted and stipulated by and
Makati, Philippines this 9th day of June, 1988 by and between the parties hereto that if after 60 days from the
between: date of the signing of this contract the VENDOR shall not
ENRIQUETA CHUA VDA. DE ONGSIONG, of legal age, be able to remove the squatters from the property being
widow, Filipino and residing at 105 Simoun St., Quezon purchased, the downpayment made by the buyer shall be
City, Metro Manila, hereinafter referred to as the returned/reimbursed by the VENDOR to the VENDEE.
VENDOR;
229
Sales – Chapter 3-12 Cases
That in the event that the VENDEE shall not be able to premises of the subject property, for which reason, he premises. Precisely, we refer you to our letters addressed
pay the VENDOR the balance of the purchase price of proposes that he shall take it upon himself to eject the to your client dated April 17, 1989 and June 8, 1989.
ONE MILLION FIVE HUNDRED ELEVEN THOUSAND squatters, provided, that expenses which shall be incurred Moreover, it is basic under the law on contracts that the
SIX HUNDRED PESOS (P1,511,600.00) ONLY after 45 by reason thereof shall be chargeable to the purchase power to rescind is given to the injured party.
days from written notification to the VENDEE of the price of the land.4 Undoubtedly, under the circumstances, our client is the
removal of the squatters from the property being Meanwhile, the Presidential Commission for the Urban injured party.
purchased, the FIFTY THOUSAND PESOS (P50,000.00) Poor ("PCUD"), through its Regional Director for Luzon, Furthermore, your client has not complied with her
previously paid as downpayment shall be forfeited in favor Farley O. Viloria, asked the Metropolitan Trial Court of obligation under their contract in good faith. It is
of the VENDOR. Parañaque for a grace period of 45 days from 21 April undeniable that Ms. Ongsiong deliberately refused to
Expenses for the registration such as registration fees, 1989 within which to relocate and transfer the squatter exert efforts to eject the squatters from the premises of
documentary stamp, transfer fee, assurances and such families. Acting favorably on the request, the court the subject property and her decision to retain the
other fees and expenses as may be necessary to transfer suspended the enforcement of the writ of execution property was brought about by the sudden increase in the
the title to the name of the VENDEE shall be for the accordingly. value of realties in the surrounding areas.
account of the VENDEE while capital gains tax shall be On 08 June 1989, Atty. Apostol reminded private Please consider this letter as a tender of payment to your
paid by the VENDOR. respondent on the expiry of the 45-day grace period and client and a demand to execute the absolute Deed of
IN WITNESS WHEREOF, the parties hereunto signed his client's willingness to "underwrite the expenses for the Sale.7
those (sic) presents in the City of Makati MM, Philippines execution of the judgment and ejectment of the A few days later (or on 27 June 1989), private respondent,
on this 9th day of June, 1988. occupants."5 prompted by petitioner's continued refusal to accept the
(Sgd.) (Sgd.) In his letter of 19 June 1989, Atty. Joaquin Yuseco, Jr., return of the P50,000.00 advance payment, filed with the
VIRGILIO R. ROMERO ENRIQUETA CHUA VDA. counsel for private respondent, advised Atty. Apostol that Regional Trial Court of Makati, Branch 133, Civil Case No.
DE ONGSIONG the Deed of Conditional Sale had been 89-4394 for rescission of the deed of "conditional" sale,
Vendee Vendor rendered null and void by virtue of his client's failure to plus damages, and for the consignation of P50,000.00
SIGNED IN THE PRESENCE OF: evict the squatters from the premises within the agreed cash.
(Sgd.) (Sgd.) 60-day period. He added that private respondent had Meanwhile, on 25 August 1989, the Metropolitan Trial
Rowena C. Ongsiong Jack M. Cruz1 "decided to retain the property."6 Court issued an alias writ of execution in Civil Case No.
Alfonso Flores, in behalf of private respondent, forthwith On 23 June 1989, Atty. Apostol wrote back to explain: 7579 on motion of private respondent but the squatters
received and acknowledged a check for P50,000.00 2from The contract of sale between the parties was perfected apparently still stayed on.
petitioner.3 from the very moment that there was a meeting of the Back to Civil Case No. 89-4394, on 26 June 1990, the
Pursuant to the agreement, private respondent filed a minds of the parties upon the subject lot and the price in Regional Trial Court of Makati8 rendered decision holding
complaint for ejectment (Civil Case No. 7579) against the amount of P1,561,600.00. Moreover, the contract had that private respondent had no right to rescind the
Melchor Musa and 29 other squatter families with the already been partially fulfilled and executed upon receipt contract since it was she who "violated her obligation to
Metropolitan Trial Court of Parañaque. A few months later, of the downpayment of your client. Ms. Ongsiong is eject the squatters from the subject property" and that
or on 21 February 1989, judgment was rendered ordering precluded from rejecting its binding effects relying upon petitioner, being the injured party, was the party who
the defendants to vacate the premises. The decision was her inability to eject the squatters from the premises of could, under Article 1191 of the Civil Code, rescind the
handed down beyond the 60-day period (expiring 09 subject property during the agreed period. Suffice it to agreement. The court ruled that the provisions in the
August 1988) stipulated in the contract. The writ of state that, the provision of the Deed of Conditional Sale contract relating to (a) the return/reimbursement of the
execution of the judgment was issued, still later, on 30 do not grant her the option or prerogative to rescind the P50,000.00 if the vendor were to fail in her obligation to
March 1989. contract and to retain the property should she fail to free the property from squatters within the stipulated
In a letter, dated 07 April 1989, private respondent sought comply with the obligation she has assumed under the period or (b), upon the other hand, the sum's forfeiture by
to return the P50,000.00 she received from petitioner contract. In fact, a perusal of the terms and conditions of the vendor if the vendee were to fail in paying the agreed
since, she said, she could not "get rid of the squatters" on the contract clearly shows that the right to rescind the purchase price, amounted to "penalty clauses". The court
the lot. Atty. Sergio A.F. Apostol, counsel for petitioner, in contract and to demand the return/reimbursement of the added:
his reply of 17 April 1989, refused the tender and stated:. downpayment is granted to our client for his protection. This Court is not convinced of the ground relied upon by
Our client believes that with the exercise of reasonable Instead, however, of availing himself of the power to the plaintiff in seeking the rescission, namely: (1) he (sic)
diligence considering the favorable decision rendered by rescind the contract and demand the return, is afraid of the squatters; and (2) she has spent so much
the Court and the writ of execution issued pursuant reimbursement of the downpayment, our client had opted to eject them from the premises (p. 6, tsn, ses. Jan. 3,
thereto, it is now possible to eject the squatters from the to take it upon himself to eject the squatters from the 1990). Militating against her profession of good faith is
230
Sales – Chapter 3-12 Cases
plaintiffs conduct which is not in accord with the rules of breach of the condition will simply prevent the duty to which, according to their nature, may be in keeping with
fair play and justice. Notably, she caused the issuance of convey title from acquiring an obligatory force. If the good faith, usage and law. Under the agreement, private
an alias writ of execution on August 25, 1989 (Exh. 6) in condition is imposed on an obligationof a party which is respondent is obligated to evict the squatters on the
the ejectment suit which was almost two months after she not complied with, the other party may either refuse to property. The ejectment of the squatters is a condition the
filed the complaint before this Court on June 27, 1989. If proceed or waive said condition (Art. 1545, Civil Code). operative act of which sets into motion the period of
she were really afraid of the squatters, then she should Where, of course, the condition is imposed upon compliance by petitioner of his own obligation, i.e., to pay
not have pursued the issuance of an alias writ of the perfection of the contract itself, the failure of such the balance of the purchase price. Private respondent's
execution. Besides, she did not even report to the police condition would prevent the juridical relation itself from failure "to remove the squatters from the property" within
the alleged phone threats from the squatters. To the mind coming into existence.13 the stipulated period gives petitioner the right to either
of the Court, the so-called squatter factor is simply In determining the real character of the contract, the title refuse to proceed with the agreement or waive that
factuitous (sic).9 given to it by the parties is not as much significant as its condition in consonance with Article 1545 of the Civil
The lower court, accordingly, dismissed the complaint and substance. For example, a deed of sale, although Code.16 This option clearly belongs to petitioner and not to
ordered, instead, private respondent to eject or cause the denominated as a deed of conditional sale, may be private respondent.
ejectment of the squatters from the property and to treated as absolute in nature, if title to the property sold is We share the opinion of the appellate court that the
execute the absolute deed of conveyance upon payment not reserved in the vendor or if the vendor is not granted undertaking required of private respondent does not
of the full purchase price by petitioner. the right to unilaterally rescind the contract predicated constitute a "potestative condition dependent solely on his
Private respondent appealed to the Court of Appeals. On on the fulfillment or non-fulfillment, as the case may be, of will" that might, otherwise, be void in accordance with
29 May 1992, the appellate court rendered its the prescribed condition.14 Article 1182 of the Civil Code17 but a "mixed" condition
decision. 10It opined that the contract entered into by the The term "condition" in the context of a perfected contract "dependent not on the will of the vendor alone but also of
parties was subject to a resolutory condition, i.e., the of sale pertains, in reality, to the compliance by one party third persons like the squatters and government agencies
ejectment of the squatters from the land, the non- of an undertaking the fulfillment of which would beckon, in and personnel concerned."18 We must hasten to add,
occurrence of which resulted in the failure of the object of turn, the demandability of the reciprocal prestation of the however, that where the so-called "potestative condition"
the contract; that private respondent substantially other party. The reciprocal obligations referred to would is imposed not on the birth of the obligation but on its
complied with her obligation to evict the squatters; that it normally be, in the case of vendee, the payment of the fulfillment, only the obligation is avoided, leaving
was petitioner who was not ready to pay the purchase agreed purchase price and, in the case of the vendor, the unaffected the obligation itself.19
price and fulfill his part of the contract, and that the fulfillment of certain express warranties (which, in the case In contracts of sale particularly, Article 1545 of the Civil
provision requiring a mandatory return/reimbursement of at bench is the timely eviction of the squatters on the Code, aforementioned, allows the obligee to choose
the P50,000.00 in case private respondent would fail to property). between proceeding with the agreement or waiving the
eject the squatters within the 60-day period was not a It would be futile to challenge the agreement here in performance of the condition. It is this provision which is
penal clause. Thus, it concluded. question as not being a duly perfected contract. A sale is the pertinent rule in the case at bench. Here, evidently,
WHEREFORE, the decision appealed from is REVERSED at once perfected when a person (the seller) obligates petitioner has waived the performance of the condition
and SET ASIDE, and a new one entered declaring the himself, for a price certain, to deliver and to transfer imposed on private respondent to free the property from
contract of conditional sale dated June 9, 1988 cancelled ownership of a specified thing or right to another (the squatters.20
and ordering the defendant-appellee to accept the return buyer) over which the latter agrees.15 In any case, private respondent's action for rescission is
of the downpayment in the amount of P50,000.00 which The object of the sale, in the case before us, was not warranted. She is not the injured party.21 The right of
was deposited in the court below. No pronouncement as specifically identified to be a 1,952-square meter lot in resolution of a party to an obligation under Article 1191 of
to costs.11 San Dionisio, Parañaque, Rizal, covered by Transfer the Civil Code is predicated on a breach of faith by the
Failing to obtain a reconsideration, petitioner filed this Certificate of Title No. 361402 of the Registry of Deeds for other party that violates the reciprocity between them.22 It
petition for review on certiorari raising issues that, in fine, Pasig and therein technically described. The purchase is private respondent who has failed in her obligation
center on the nature of the contract adverted to and the price was fixed at P1,561,600.00, of which P50,000.00 under the contract. Petitioner did not breach the
P50,000.00 remittance made by petitioner. was to be paid upon the execution of the document of agreement. He has agreed, in fact, to shoulder the
A perfected contract of sale may either be absolute or sale and the balance of P1,511,600.00 payable "45 days expenses of the execution of the judgment in the
conditional12 depending on whether the agreement is after the removal of all squatters from the above described ejectment case and to make arrangements with the sheriff
devoid of, or subject to, any condition imposed on property." to effect such execution. In his letter of 23 June 1989,
the passing of title of the thing to be conveyed or on From the moment the contract is perfected, the parties are counsel for petitioner has tendered payment and
the obligation of a party thereto. When ownership is bound not only to the fulfillment of what has been demanded forthwith the execution of the deed of absolute
retained until the fulfillment of a positive condition the expressly stipulated but also to all the consequences sale. Parenthetically, this offer to pay, having been made
231
Sales – Chapter 3-12 Cases
prior to the demand for rescission, assuming for the sake
of argument that such a demand is proper under Article
159223 of the Civil Code, would likewise suffice to defeat
private respondent's prerogative to rescind thereunder.
There is no need to still belabor the question of whether
the P50,000.00 advance payment is reimbursable to
petitioner or forfeitable by private respondent, since, on
the basis of our foregoing conclusions, the matter has
ceased to be an issue. Suffice it to say that petitioner
having opted to proceed with the sale, neither may
petitioner demand its reimbursement from private
respondent nor may private respondent subject it to
forfeiture. G.R. No. 130403 July 30, 2007
WHEREFORE, the questioned decision of the Court of FRANCISCO GONZALES, Petitioner,
Appeals is hereby REVERSED AND SET ASIDE, and vs.
another is entered ordering petitioner to pay private SEVERINO C. LIM and TOYOTA SHAW,
respondent the balance of the purchase price and the INC., Respondents.
latter to execute the deed of absolute sale in favor of DECISION
petitioner. No costs. CORONA, J.:
SO ORDERED. At bar is an appeal by certiorari under Rule 45 of the
Rules of Court questioning the decision1 and resolution2 of
the Court of Appeals (CA) in CA-G.R. CV No. 41716
entitled Severino C. Lim and Toyota Shaw, Inc. v.
Francisco Gonzales and Carmen Gonzales.
The facts follow.
Petitioner Francisco Gonzales, Roque Ma. Gonzales and
Carmen Gonzales (Gonzaleses) were the owners of
Motown Vehicles, Inc. (Motown). Motown was the
licensed distributor of Ford vehicles in the country. Its
assets included two buildings standing on a 4,944 sq. m.
lot leased from Tanglaw Realty Inc. (Tanglaw).
In 1988, when Ford Philippines ceased operations, the
Gonzaleses sold Motown’s shares of stocks to
respondents Severino C. Lim and Toyota Shaw, Inc.
which was then putting up a Toyota car dealership. The
"Agreement" signed by the parties stated that the sale
included Motown’s two lease contracts with Tanglaw. It
read:
WHEREAS, Motown, which owns these fixed and
moveable improvements and equipments…does not own
the land on which these improvements and equipments
are located, but merely leases the bare land… from
Tanglaw Realty Corp. under 2 Lease Contracts both dated
June 17, 1978 both commencing Nov. 15, 1977 and
expiring Nov. 14, 2002.
xxx xxx xxx

232
Sales – Chapter 3-12 Cases
4. PAYMENT — The aforementioned price amounting to Petitioner added that his only undertaking was to help obligation to submit the required official communication
₱6,746,000.00 shall be paid by [respondents] to the respondents negotiate a new lease contract that would from Tanglaw Realty Corporation. Thus, [respondents]
[Gonzaleses] in two (2) installments payable simultaneous have similar terms as his. As a counterclaim, petitioner are…freed from their obligation to pay the final installment
to the occurrence of the following events: asked for the payment of respondents’ ₱500,000 balance. of ₱500,000.00.
1. ₱6,246,000 – [u]pon signing of this contract xxx After trial, the RTC dismissed respondents’ case but xxx xxx xxx
2. ₱500,000 – [u]pon receipt of official communication granted petitioner’s counterclaim of ₱500,000. The court a WHEREFORE, judgment is hereby rendered MODIFYING
from Tanglaw Realty Corporation to the effect that quo’s decision read: the lower court’s decision by deleting the portion ordering
Motown can have continuing and unhampered use of the ...[T]he court finds that [petitioner] did not warrant the [respondents] to pay [petitioner] ₱500,000 plus legal
pieces of [the leased] land…covered by the 2 Lease existence of the lease on one of the premises. The court interest. Instead, the Court hereby declares [petitioner’s]
Contracts…[I]t is understood that the continuation of the believes that even before the ["Agreement"] has been counterclaim DISMISSED.6
lease at a reasonable rate for the original term of the 2 executed[,] [respondents were] already aware that one of Petitioner filed a motion for reconsideration (MR),
lease agreements is a central, indivisible and very basic the leases has been terminated…[I]f [petitioner] warranted contending that the payment of the ₱500,000 balance was
part of this agreement, since the [bases] for the valuation anything at all, it was only that he will help [respondents] already due because respondents themselves had
of Motown by [respondents are] its location and the procure a new lease contract under the old term. prevented him from fulfilling his undertaking in the
improvements and equipments contained therein.3 xxx xxx xxx "Agreement." Petitioner insisted that since respondents
xxx xxx xxx …In view of the foregoing, the complaint is DISMISSED[.] negotiated directly with Tanglaw for a new lease contract,
After paying the initial installment of ₱6,246,000 to the On the counterclaim, [respondents] are ordered to pay petitioner’s obligation should be deemed fulfilled.
Gonzaleses, respondents claimed they discovered that [petitioner] ₱500,000, representing the outstanding The CA denied the MR.7 Hence, this petition.8
one of Motown’s lease contracts had already been balance for the sale of Motown shares of stocks plus legal In this petition, the lone issue for resolution is whether
terminated prior to the sale. As a result, they were interest from October 10, 1989, the date of the lease petitioner was still entitled to the payment of ₱500,000
allegedly constrained to negotiate with Tanglaw for a new between Tanglaw Realty and Toyota Shaw, Inc., when despite failure to comply with the provision in the
lease contract (with a higher rental). [petitioner] was deemed to have fulfilled his promise. "Agreement" requiring him to obtain an official
Subsequently, respondents filed a case in the Regional xxx xxx xxx communication from Tanglaw regarding the continuation
Trial Court (RTC), Branch 65 of Makati,4 for declaratory SO ORDERED.5 of Motown’s lease contract.
relief with damages against the Gonzaleses, seeking Respondents appealed to the CA which affirmed with At the outset, petitioner’s undertaking set forth in the
release from their obligation to pay the ₱500,000 balance. modification the trial court’s decision. It agreed with the "Agreement" may be deemed a "condition," a future and
During the trial, respondents (as then plaintiffs) accused RTC that respondents could not feign ignorance of uncertain event upon which the existence of an obligation
the Gonzaleses of falsely representing to them that the Motown’s terminated lease contract; however, it deleted is made to depend or that which subordinates the
latter’s two lease contracts were still subsisting at the time the order directing respondents to pay petitioner existence of a liability under a contract to a certain future
of the sale. They maintained that the Gonzaleses ₱500,000. The CA ruled that the payment was not due event.9 It was a condition that was imposed on an
guaranteed a "continuing and unhampered use" of the since petitioner failed to obtain the required official obligation after the consummation of the contract of sale,
premises but Tanglaw had nonetheless threatened to communication from Tanglaw. The CA decision read: not a condition on the perfection of the contract itself (non-
evict them from one of the leased portions. xxx xxx xxx fulfillment of which could have prevented the juridical
To support their claim, they presented in court a copy of xxx. The phrase "continuation of the lease contract at a relation from coming into existence).
the "Agreement" indicating the Gonzaleses’ alleged reasonable rate" proves that [respondents] did not Article 1545 of the Civil Code is pertinent:
warranty that the two lease contracts with Tanglaw were contemplate stepping into the shoes of Motown as lessee Art. 1545. Where the obligation of either party to a
still good. of the parcels of land because if what they truly expected contract of sale is subject to any condition which is
Petitioner (with his then co-defendants) countered that was to continue exactly the same lease agreement not performed, such party may refuse to proceed with
respondents were well aware of the termination of one of between [Tanglaw] and Motown, there would have been the contract or he may waive performance of the
the two lease contracts at the time of sale. He denied no need to include [said] phrase… Clearly, [respondents] condition. If the other party has promised that the
giving a warranty on both contracts and explained that he anticipated nay expected that if they continue the lease, it condition should happen or be performed, such first
only signed the "Agreement" (showing Motown’s two would not be under the same terms and conditions as the mentioned party may also treat the nonperformance of the
lease contracts with Tanglaw) on prodding by respondents original contract, but rather at a new, reasonable rate. condition as a breach of warranty. (emphasis supplied)
that they needed it to convince Toyota Philippines they Therefore, there was no warranty from [petitioner]… xxx xxx xxx
were ready with their dealership site. According to …[W]ith regard to the question of whether [respondents] These options were echoed in Romero v. CA,10 where we
petitioner, respondents told him it was only "for show" and are now obliged to pay [petitioner] the ₱500,000.00…, the declared that if the condition was imposed on an
amendments thereto would be made later on. Court finds that [petitioner had] not been able to fulfill [his] obligation of a party which was not complied with, the
233
Sales – Chapter 3-12 Cases
other party may either (1) refuse to proceed with the
agreement or (2) waive the fulfillment of the condition.
In the case at bar, respondents obviously did not choose
the first option as they proceeded with their contract with
petitioner despite the latter’s non-fulfillment of the
condition in the agreement. In fact, in their comment, they
stated that they "took possession of the properties and
caused extensive improvement and installed facilities and
equipment" thereon.11
Did respondents, however, waive fulfillment of the
condition? Yes.
The records reveal that respondents negotiated directly
with Tanglaw for a new lease contract even without the
required official communication that petitioner was
supposed to obtain for them, a condition in the
"Agreement" which they themselves imposed on the latter.
Although they had the right to require his compliance with
the condition or compel his performance of the
undertaking, they opted otherwise.
Respondents’ assertion that they were merely forced to
deal directly with Tanglaw because the latter had
threatened to evict them has no merit. As the RTC and the
CA both held, respondents, at the time of the sale, already
knew that one of Motown’s two lease contracts with
Tanglaw had been terminated. This being a finding of fact,
we shall not look into it, absent any compelling reason to
do so.12 Respondents therefore cannot invoke this
argument to justify their actions and evade their liability to
petitioner.1awphil
Moreover, respondents’ contention that the condition did
not preclude them from dealing with Tanglaw or that they
were "to refrain from negotiating directly"13 can only mean
that they did not really expect petitioner to comply strictly
and absolutely with it. Respondents’ conduct showed that
they did not only disregard the condition but also placed
petitioner in a position that his compliance was no longer
necessary. We are thus constrained to rule that they had
effectively waived compliance with the condition.
Finally, the condition was deemed waived when
respondents forged their new lease contract with
Tanglaw.14
WHEREFORE, the petition is hereby GRANTED. The
assailed decision and resolution of the Court of Appeals in
CA-G.R. CV No. 41716 are hereby SET ASIDE and the
decision of the Regional Trial Court REINSTATED.
SO ORDERED.

234
Sales – Chapter 3-12 Cases
G.R. No. 146839 March 23, 2011 of which sum is hereby acknowledged in full from the from agricultural land into residential land which he
ROLANDO T. CATUNGAL, JOSE T. CATUNGAL, JR., VENDEE. claimed substantially increased the property’s value. He
CAROLYN T. CATUNGAL and ERLINDA CATUNGAL- b. The balance of TWENTY[-]FOUR MILLION FIVE likewise alleged that he actively negotiated for the road
WESSEL, Petitioners, HUNDRED THOUSAND PESOS (₱24,500,000.00) shall right of way as stipulated in the contract.9
vs. be payable in five separate checks, made to the order of Rodriguez further claimed that on August 31, 1990 the
ANGEL S. RODRIGUEZ, Respondent. JOSE Ch. CATUNGAL, the first check shall be for FOUR spouses Catungal requested an advance of
DECISION MILLION FIVE HUNDRED THOUSAND PESOS ₱5,000,000.00 on the purchase price for personal
LEONARDO-DE CASTRO, J.: (₱4,500,000.00) and the remaining balance to be paid in reasons. Rodriquez allegedly refused on the ground that
Before the Court is a Petition for Review on Certiorari, four checks in the amounts of FIVE MILLION PESOS the amount was substantial and was not due under the
assailing the following issuances of the Court of Appeals (₱5,000,000.00) each after the VENDEE have (sic) terms of their agreement. Shortly after his refusal to pay
in CA-G.R. CV No. 40627 consolidated with CA-G.R. SP successfully negotiated, secured and provided a Road the advance, he purportedly learned that the Catungals
No. 27565: (a) the August 8, 2000 Decision,1 which Right of Way consisting of 12 meters in width cutting were offering the property for sale to third parties.10
affirmed the Decision2 dated May 30, 1992 of the across Lot 10884 up to the national road, either by Thereafter, Rodriguez received letters dated October 22,
Regional Trial Court (RTC), Branch 27 of Lapu-lapu City, widening the existing Road Right of Way or by securing a 1990,11 October 24, 199012 and October 29, 1990,13 all
Cebu in Civil Case No. 2365-L, and (b) the January 30, new Road Right of Way of 12 meters in width. If however signed by Jose Catungal who was a lawyer, essentially
2001 Resolution,3 denying herein petitioners’ motion for said Road Right of Way could not be negotiated, the demanding that the former make up his mind about buying
reconsideration of the August 8, 2000 Decision. VENDEE shall give notice to the VENDOR for them to the land or exercising his "option" to buy because the
The relevant factual and procedural antecedents of this reassess and solve the problem by taking other options spouses Catungal allegedly received other offers and they
case are as follows: and should the situation ultimately prove futile, he shall needed money to pay for personal obligations and for
This controversy arose from a Complaint for Damages take steps to rescind or cancel the herein Conditional investing in other properties/business ventures. Should
and Injunction with Preliminary Injunction/Restraining Deed of Sale. Rodriguez fail to exercise his option to buy the land, the
Order4filed on December 10, 1990 by herein respondent c. That the access road or Road Right of Way leading to Catungals warned that they would consider the contract
Angel S. Rodriguez (Rodriguez), with the RTC, Branch 27, Lot 10963 shall be the responsibility of the VENDEE to cancelled and that they were free to look for other buyers.
Lapu-lapu City, Cebu, docketed as Civil Case No. 2365-L secure and any or all cost relative to the acquisition In a letter dated November 4, 1990,14 Rodriguez
against the spouses Agapita and Jose Catungal (the thereof shall be borne solely by the VENDEE. He shall, registered his objections to what he termed the Catungals’
spouses Catungal), the parents of petitioners. however, be accorded with enough time necessary for the unwarranted demands in view of the terms of the
In the said Complaint, it was alleged that Agapita T. success of his endeavor, granting him a free hand in Conditional Deed of Sale which allowed him sufficient time
Catungal (Agapita) owned a parcel of land (Lot 10963) negotiating for the passage. to negotiate a road right of way and granted him, the
with an area of 65,246 square meters, covered by Original BY THESE PRESENTS, the VENDOR do hereby agree to vendee, the exclusive right to rescind the contract. Still, on
Certificate of Title (OCT) No. 1055 in her name situated in sell by way of herein CONDITIONAL DEED OF SALE to November 15, 1990, Rodriguez purportedly received a
the Barrio of Talamban, Cebu City. The said property was VENDEE, his heirs, successors and assigns, the real letter dated November 9, 199015 from Atty. Catungal,
allegedly the exclusive paraphernal property of Agapita. property described in the Original Certificate of Title No. stating that the contract had been cancelled and
On April 23, 1990, Agapita, with the consent of her 105 x x x. terminated.
husband Jose, entered into a Contract to Sell6 with xxxx Contending that the Catungals’ unilateral rescission of the
respondent Rodriguez. Subsequently, the Contract to Sell 5. That the VENDEE has the option to rescind the sale. In Conditional Deed of Sale was unjustified, arbitrary and
was purportedly "upgraded" into a Conditional Deed of the event the VENDEE exercises his option to rescind the unwarranted, Rodriquez prayed in his Complaint, that:
Sale7 dated July 26, 1990 between the same parties. Both herein Conditional Deed of Sale, the VENDEE shall notify 1. Upon the filing of this complaint, a restraining order be
the Contract to Sell and the Conditional Deed of Sale the VENDOR by way of a written notice relinquishing his issued enjoining defendants [the spouses Catungal], their
were annotated on the title. rights over the property. The VENDEE shall then be employees, agents, representatives or other persons
The provisions of the Conditional Deed of Sale pertinent reimbursed by the VENDOR the sum of FIVE HUNDRED acting in their behalf from offering the property subject of
to the present dispute are quoted below: THOUSAND PESOS (₱500,000.00) representing the this case for sale to third persons; from entertaining offers
1. The VENDOR for and in consideration of the sum of downpayment, interest free, payable but contingent upon or proposals by third persons to purchase the said
TWENTY[-]FIVE MILLION PESOS (₱25,000,000.00) the event that the VENDOR shall have been able to sell property; and, in general, from performing acts in
payable as follows: the property to another party.8 furtherance or implementation of defendants’ rescission of
a. FIVE HUNDRED THOUSAND PESOS (₱500,000.00) In accordance with the Conditional Deed of Sale, their Conditional Deed of Sale with plaintiff [Rodriguez].
downpayment upon the signing of this agreement, receipt Rodriguez purportedly secured the necessary surveys and 2. After hearing, a writ of preliminary injunction be issued
plans and through his efforts, the property was reclassified upon such reasonable bond as may be fixed by the court
235
Sales – Chapter 3-12 Cases
enjoining defendants and other persons acting in their (2) his refusal to pay the additional amount of the road lots for ₱2,500.00 per square meter, the adjacent
behalf from performing any of the acts mentioned in the ₱5,000,000.00 asked by the Catungals, which to them lot owners were no longer willing to sell the road lots to
next preceding paragraph. indicated his lack of funds to purchase the property. The Rodriguez at ₱550.00 per square meter but were asking
3. After trial, a Decision be rendered: Catungals likewise contended that Rodriguez did not have for a price of ₱3,500.00 per square meter. In other words,
a) Making the injunction permanent; an exclusive right to rescind the contract and that the instead of assisting Rodriguez in his efforts to negotiate
b) Condemning defendants to pay to plaintiff, jointly and contract, being reciprocal, meant both parties had the right the road right of way, the spouses Catungal allegedly
solidarily: to rescind.23 The spouses Catungal further claimed that it intentionally and maliciously defeated Rodriguez’s
Actual damages in the amount of ₱400,000.00 for their was Rodriguez who was in breach of their agreement and negotiations for a road right of way in order to justify
unlawful rescission of the Agreement and their guilty of bad faith which justified their rescission of the rescission of the said contract and enable them to offer
performance of acts in violation or disregard of the said contract.24 By way of counterclaim, the spouses Catungal the property to other buyers.
Agreement; prayed for actual and consequential damages in the form Despite requesting the trial court for an extension of time
Moral damages in the amount of ₱200,000.00; of unearned interests from the balance (of the purchase to file an amended Answer,29 the Catungals did not file an
Exemplary damages in the amount of ₱200,000.00; price in the amount) of ₱24,500,000.00, moral and amended Answer and instead filed an Urgent Motion to
Expenses of litigation and attorney’s fees in the amount of exemplary damages in the amount of ₱2,000,000.00, Dismiss30 again invoking the ground of improper venue. In
₱100,000.00; and attorney’s fees in the amount of ₱200,000.00 and costs of the meantime, for failure to file an amended Answer within
Costs of suit.16 suits and litigation expenses in the amount of the period allowed, the trial court set the case for pre-trial
On December 12, 1990, the trial court issued a temporary ₱10,000.00.25 The spouses Catungal prayed for the on December 20, 1991.
restraining order and set the application for a writ of dismissal of the complaint and the grant of their During the pre-trial held on December 20, 1991, the trial
preliminary injunction for hearing on December 21, 1990 counterclaim. court denied in open court the Catungals’ Urgent Motion
with a directive to the spouses Catungal to show cause The Catungals amended their Answer twice,26 retaining to Dismiss for violation of the rules and for being
within five days from notice why preliminary injunction their basic allegations but amplifying their charges of repetitious and having been previously
should not be granted. The trial court likewise ordered that contractual breach and bad faith on the part of Rodriguez denied.31 However, Atty. Catungal refused to enter into
summons be served on them.17 and adding the argument that in view of Article 1191 of pre-trial which prompted the trial court to declare the
Thereafter, the spouses Catungal filed their the Civil Code, the power to rescind reciprocal obligations defendants in default and to set the presentation of the
opposition18 to the issuance of a writ of preliminary is granted by the law itself to both parties and does not plaintiff’s evidence on February 14, 1992.32
injunction and later filed a motion to dismiss 19 on the need an express stipulation to grant the same to the On December 23, 1991, the Catungals filed a motion for
ground of improper venue. According to the Catungals, injured party. In the Second Amended Answer with reconsideration33 of the December 20, 1991 Order
the subject property was located in Cebu City and thus, Counterclaim, the spouses Catungal added a prayer for denying their Urgent Motion to Dismiss but the trial court
the complaint should have been filed in Cebu City, not the trial court to order the Register of Deeds to cancel the denied reconsideration in an Order dated February 3,
Lapu-lapu City. Rodriguez opposed the motion to dismiss annotations of the two contracts at the back of their 1992.34Undeterred, the Catungals subsequently filed a
on the ground that his action was a personal action as its OCT.27 Motion to Lift and to Set Aside Order of Default 35 but it
subject was breach of a contract, the Conditional Deed of On October 24, 1991, Rodriguez filed an Amended was likewise denied for being in violation of the rules and
Sale, and not title to, or possession of real property.20 Complaint,28 adding allegations to the effect that the for being not meritorious.36 On February 28, 1992, the
In an Order dated January 17, 1991,21 the trial court Catungals were guilty of several misrepresentations which Catungals filed a Petition for Certiorari and
denied the motion to dismiss and ruled that the complaint purportedly induced Rodriguez to buy the property at the Prohibition37 with the Court of Appeals, questioning the
involved a personal action, being merely for damages with price of ₱25,000,000.00. Among others, it was alleged denial of their motion to dismiss and the order of default.
a prayer for injunction. that the spouses Catungal misrepresented that their Lot This was docketed as CA-G.R. SP No. 27565.
Subsequently, on January 30, 1991, the trial court ordered 10963 includes a flat portion of land which later turned out Meanwhile, Rodriguez proceeded to present his evidence
the issuance of a writ of preliminary injunction upon to be a separate lot (Lot 10986) owned by Teodora before the trial court.
posting by Rodriguez of a bond in the amount of Tudtud who sold the same to one Antonio Pablo. The In a Decision dated May 30, 1992, the trial court ruled in
₱100,000.00 to answer for damages that the defendants Catungals also allegedly misrepresented that the road favor of Rodriguez, finding that: (a) under the contract it
may sustain by reason of the injunction. right of way will only traverse two lots owned by Anatolia was complainant (Rodriguez) that had the option to
On February 1, 1991, the spouses Catungal filed their Tudtud and her daughter Sally who were their relatives rescind the sale; (b) Rodriguez’s obligation to pay the
Answer with Counterclaim22 alleging that they had the and who had already agreed to sell a portion of the said balance of the purchase price arises only upon successful
right to rescind the contract in view of (1) Rodriguez’s lots for the road right of way at a price of ₱550.00 per negotiation of the road right of way; (c) he proved his
failure to negotiate the road right of way despite the lapse square meter. However, because of the Catungals’ acts of diligent efforts to negotiate the road right of way; (d) the
of several months since the signing of the contract, and offering the property to other buyers who offered to buy spouses Catungal were guilty of misrepresentation which
236
Sales – Chapter 3-12 Cases
defeated Rodriguez’s efforts to acquire the road right of THE COURT A QUO ERRED IN NOT RESTRAINING 40627 and CA-G.R. SP No. 27565,47 affirming the trial
way; and (e) the Catungals’ rescission of the contract had ITSELF MOTU PROP[R]IO FROM CONTINUING WITH court’s Decision.
no basis and was in bad faith. Thus, the trial court made THE PROCEEDINGS IN THE CASE AND IN In a Motion for Reconsideration dated August 21,
the injunction permanent, ordered the Catungals to RENDERING DECISION THEREIN IF ONLY FOR 2000,48 counsel for the Catungals, Atty. Borromeo, argued
reduce the purchase price by the amount of acquisition of REASON OF COURTESY AND FAIRNESS BEING for the first time that paragraphs 1(b) and 549 of the
Lot 10963 which they misrepresented was part of the MANDATED AS DISPENSER OF FAIR AND EQUAL Conditional Deed of Sale, whether taken separately or
property sold but was in fact owned by a third party and JUSTICE TO ALL AND SUNDRY WITHOUT FEAR OR jointly, violated the principle of mutuality of contracts
ordered them to pay ₱100,000.00 as damages, FAVOR IT HAVING BEEN SERVED EARLIER WITH A under Article 1308 of the Civil Code and thus, said
₱30,000.00 as attorney’s fees and costs. COPY OF THE PETITION FOR CERTIORARI contract was void ab initio. He adverted to the cases
The Catungals appealed the decision to the Court of QUESTIONING ITS VENUE AND JURISDICTION IN CA- mentioned in his various citations of authorities to support
Appeals, asserting the commission of the following errors G.R. NO. SP 27565 IN FACT NOTICES FOR THE FILING his argument of nullity of the contract and his position that
by the trial court in their appellants’ brief38 dated February OF COMMENT THERETO HAD ALREADY BEEN SENT this issue may be raised for the first time on appeal.
9, 1994: OUT BY THE HONORABLE COURT OF APPEALS, Meanwhile, a Second Motion for Substitution 50 was filed
I SECOND DIVISION, AND THE COURT A QUO WAS by Atty. Borromeo in view of the death of Jose Catungal.
THE COURT A QUO ERRED IN NOT DISMISSING OF FURNISHED WITH COPY OF SAID NOTICE. In a Resolution dated January 30, 2001, the Court of
(SIC) THE CASE ON THE GROUNDS OF IMPROPER VII Appeals allowed the substitution of the deceased Agapita
VENUE AND LACK OF JURISDICTION. THE COURT A QUO ERRED IN DECIDING THE CASE and Jose Catungal by their surviving heirs and denied the
II IN FAVOR OF THE PLAINTIFF AND AGAINST THE motion for reconsideration for lack of merit
THE COURT A QUO ERRED IN CONSIDERING THE DEFENDANTS ON THE BASIS OF EVIDENCE WHICH Hence, the heirs of Agapita and Jose Catungal filed on
CASE AS A PERSONAL AND NOT A REAL ACTION. ARE IMAGINARY, FABRICATED, AND DEVOID OF March 27, 2001 the present petition for review,51 which
III TRUTH, TO BE STATED IN DETAIL IN THE essentially argued that the Court of Appeals erred in not
GRANTING WITHOUT ADMITTING THAT VENUE WAS DISCUSSION OF THIS PARTICULAR ERROR, AND, finding that paragraphs 1(b) and/or 5 of the Conditional
PROPERLY LAID AND THE CASE IS A PERSONAL THEREFORE, THE DECISION IS REVERSIBLE.39 Deed of Sale, violated the principle of mutuality of
ACTION, THE COURT A QUO ERRED IN DECLARING On August 31, 1995, after being granted several contracts under Article 1308 of the Civil Code. Thus, said
THE DEFENDANTS IN DEFAULT DURING THE PRE- extensions, Rodriguez filed his appellee’s contract was supposedly void ab initio and the Catungals’
TRIAL WHEN AT THAT TIME THE DEFENDANTS HAD brief,40 essentially arguing the correctness of the trial rescission thereof was superfluous.
ALREADY FILED THEIR ANSWER TO THE court’s Decision regarding the foregoing issues raised by In his Comment,52 Rodriguez highlighted that (a)
COMPLAINT. the Catungals. Subsequently, the Catungals filed a Reply petitioners were raising new matters that cannot be
IV Brief41 dated October 16, 1995. passed upon on appeal; (b) the validity of the Conditional
THE COURT A QUO ERRED IN CONSIDERING THE From the filing of the appellants’ brief in 1994 up to the Deed of Sale was already admitted and petitioners cannot
DEFENDANTS AS HAVING LOST THEIR LEGAL filing of the Reply Brief, the spouses Catungal were be allowed to change theories on appeal; (c) the
STANDING IN COURT WHEN AT MOST THEY COULD represented by appellant Jose Catungal himself. questioned paragraphs of the Conditional Deed of Sale
ONLY BE CONSIDERED AS IN DEFAULT AND STILL However, a new counsel for the Catungals, Atty. Jesus N. were valid; and (d) petitioners were the ones who
ENTITLED TO NOTICES OF ALL FURTHER Borromeo (Atty. Borromeo), entered his appearance committed fraud and breach of contract and were not
PROCEEDINGS ESPECIALLY AFTER THEY HAD FILED before the Court of Appeals on September 2, 1997.42 On entitled to relief for not having come to court with clean
THE MOTION TO LIFT THE ORDER OF DEFAULT. the same date, Atty. Borromeo filed a Motion for Leave of hands.
V Court to File Citation of Authorities43 and a Citation of The Court gave due course to the Petition53 and the
THE COURT A QUO ERRED IN ISSUING THE WRIT Authorities.44 This would be followed by Atty. Borromeo’s parties filed their respective Memoranda.
[OF] PRELIMINARY INJUNCTION RESTRAINING THE filing of an Additional Citation of Authority and Second The issues to be resolved in the case at bar can be
EXERCISE OF ACTS OF OWNERSHIP AND OTHER Additional Citation of Authority both on November 17, summed into two questions:
RIGHTS OVER REAL PROPERTY OUTSIDE OF THE 1997.45 I. Are petitioners allowed to raise their theory of nullity of
COURT’S TERRITORIAL JURISDICTION AND During the pendency of the case with the Court of the Conditional Deed of Sale for the first time on appeal?
INCLUDING PERSONS WHO WERE NOT BROUGHT Appeals, Agapita Catungal passed away and thus, her II. Do paragraphs 1(b) and 5 of the Conditional Deed of
UNDER ITS JURISDICTION, THUS THE NULLITY OF husband, Jose, filed on February 17, 1999 a motion for Sale violate the principle of mutuality of contracts under
THE WRIT. Agapita’s substitution by her surviving children.46 Article 1308 of the Civil Code?
VI On August 8, 2000, the Court of Appeals rendered a On petitioners’ change of theory
Decision in the consolidated cases CA-G.R. CV No.
237
Sales – Chapter 3-12 Cases
Petitioners claimed that the Court of Appeals should have not hear the parties, is not only irregular but also Ruling on the questioned provisions of the Conditional
reversed the trial courts’ Decision on the ground of the extrajudicial and invalid. The rule rests on the fundamental Deed of Sale
alleged nullity of paragraphs 1(b) and 5 of the Conditional tenets of fair play."59 Even assuming for the sake of argument that this Court
Deed of Sale notwithstanding that the same was not During the proceedings before the trial court, the spouses may overlook the procedural misstep of petitioners, we
raised as an error in their appellants’ brief. Citing Catholic Catungal never claimed that the provisions in the still cannot uphold their belatedly proffered arguments.
Bishop of Balanga v. Court of Appeals,54 petitioners Conditional Deed of Sale, stipulating that the payment of At the outset, it should be noted that what the parties
argued in the Petition that this case falls under the the balance of the purchase price was contingent upon entered into is a Conditional Deed of Sale, whereby the
following exceptions: the successful negotiation of a road right of way spouses Catungal agreed to sell and Rodriguez agreed to
(3) Matters not assigned as errors on appeal but (paragraph 1[b]) and granting Rodriguez the option to buy Lot 10963 conditioned on the payment of a certain
consideration of which is necessary in arriving at a just rescind (paragraph 5), were void for allegedly making the price but the payment of the purchase price was
decision and complete resolution of the case or to serve fulfillment of the contract dependent solely on the will of additionally made contingent on the successful
the interest of justice or to avoid dispensing piecemeal Rodriguez. negotiation of a road right of way. It is elementary that "[i]n
justice; On the contrary, with respect to paragraph 1(b), the conditional obligations, the acquisition of rights, as well as
(4) Matters not specifically assigned as errors on appeal Catungals did not aver in the Answer (and its amended the extinguishment or loss of those already acquired, shall
but raised in the trial court and are matters of record versions) that the payment of the purchase price was depend upon the happening of the event which
having some bearing on the issue submitted which the subject to the will of Rodriguez but rather they claimed constitutes the condition."60
parties failed to raise or which the lower court ignored; that paragraph 1(b) in relation to 1(c) only presupposed a Petitioners rely on Article 1308 of the Civil Code to
(5) Matters not assigned as errors on appeal but closely reasonable time be given to Rodriguez to negotiate the support their conclusion regarding the claimed nullity of
related to an error assigned; and road right of way. However, it was petitioners’ theory that the aforementioned provisions. Article 1308 states that
(6) Matters not assigned as errors but upon which the more than sufficient time had already been given "[t]he contract must bind both contracting parties; its
determination of a question properly assigned is Rodriguez to negotiate the road right of way. validity or compliance cannot be left to the will of one of
dependent.55 Consequently, Rodriguez’s refusal/failure to pay the them."
We are not persuaded. balance of the purchase price, upon demand, was Article 1182 of the Civil Code, in turn, provides:
This is not an instance where a party merely failed to allegedly indicative of lack of funds and a breach of the Art. 1182. When the fulfillment of the condition depends
assign an issue as an error in the brief nor failed to argue contract on the part of Rodriguez. upon the sole will of the debtor, the conditional obligation
a material point on appeal that was raised in the trial court Anent paragraph 5 of the Conditional Deed of Sale, shall be void. If it depends upon chance or upon the will of
and supported by the record. Neither is this a case where regarding Rodriguez’s option to rescind, it was petitioners’ a third person, the obligation shall take effect in conformity
a party raised an error closely related to, nor dependent theory in the court a quo that notwithstanding such with the provisions of this Code.
on the resolution of, an error properly assigned in his brief. provision, they retained the right to rescind the contract for In the past, this Court has distinguished between a
This is a situation where a party completely changes his Rodriguez’s breach of the same under Article 1191 of the condition imposed on the perfection of a contract and a
theory of the case on appeal and abandons his previous Civil Code. condition imposed merely on the performance of an
assignment of errors in his brief, which plainly should not Verily, the first time petitioners raised their theory of the obligation. While failure to comply with the first condition
be allowed as anathema to due process. nullity of the Conditional Deed of Sale in view of the results in the failure of a contract, failure to comply with
Petitioners should be reminded that the object of questioned provisions was only in their Motion for the second merely gives the other party the option to
pleadings is to draw the lines of battle between the Reconsideration of the Court of Appeals’ Decision, either refuse to proceed with the sale or to waive the
litigants and to indicate fairly the nature of the claims or affirming the trial court’s judgment. The previous filing of condition.61 This principle is evident in Article 1545 of the
defenses of both parties.56 In Philippine National various citations of authorities by Atty. Borromeo and the Civil Code on sales, which provides in part:
Construction Corporation v. Court of Appeals,57 we held Court of Appeals’ resolutions noting such citations were of Art. 1545. Where the obligation of either party to a
that "[w]hen a party adopts a certain theory in the trial no moment. The citations of authorities merely listed contract of sale is subject to any condition which is not
court, he will not be permitted to change his theory on cases and their main rulings without even any mention of performed, such party may refuse to proceed with the
appeal, for to permit him to do so would not only be unfair their relevance to the present case or any prayer for the contract or he may waive performance of the condition x x
to the other party but it would also be offensive to the Court of Appeals to consider them.1âwphi1 In sum, the x.
basic rules of fair play, justice and due process." 58 Court of Appeals did not err in disregarding the citations of Paragraph 1(b) of the Conditional Deed of Sale, stating
We have also previously ruled that "courts of justice have authorities or in denying petitioners’ motion for that respondent shall pay the balance of the purchase
no jurisdiction or power to decide a question not in issue. reconsideration of the assailed August 8, 2000 Decision in price when he has successfully negotiated and secured a
Thus, a judgment that goes beyond the issues and view of the proscription against changing legal theories on road right of way, is not a condition on the perfection of
purports to adjudicate something on which the court did appeal. the contract nor on the validity of the entire contract or its
238
Sales – Chapter 3-12 Cases
compliance as contemplated in Article 1308. It is a however, that where the so-called "potestative condition" defendants is without any legal or factual basis. 64 x x x.
condition imposed only on respondent’s obligation to pay is imposed not on the birth of the obligation but on its (Emphases supplied.)
the remainder of the purchase price. In our view and fulfillment, only the condition is avoided, leaving In all, we see no cogent reason to disturb the foregoing
applying Article 1182, such a condition is not purely unaffected the obligation itself.63 (Emphases supplied.) factual findings of the trial court.
potestative as petitioners contend. It is not dependent on From the provisions of the Conditional Deed of Sale Furthermore, it is evident from the language of paragraph
the sole will of the debtor but also on the will of third subject matter of this case, it was the vendee (Rodriguez) 1(b) that the condition precedent (for respondent’s
persons who own the adjacent land and from whom the that had the obligation to successfully negotiate and obligation to pay the balance of the purchase price to
road right of way shall be negotiated. In a manner of secure the road right of way. However, in the decision of arise) in itself partly involves an obligation to do, i.e., the
speaking, such a condition is likewise dependent on the trial court, which was affirmed by the Court of Appeals, undertaking of respondent to negotiate and secure a road
chance as there is no guarantee that respondent and the it was found that respondent Rodriguez diligently exerted right of way at his own expense.65 It does not escape our
third party-landowners would come to an agreement efforts to secure the road right of way but the spouses notice as well, that far from disclaiming paragraph 1(b) as
regarding the road right of way. This type of mixed Catungal, in bad faith, contributed to the collapse of the void, it was the Catungals’ contention before the trial court
condition is expressly allowed under Article 1182 of the negotiations for said road right of way. To quote from the that said provision should be read in relation to paragraph
Civil Code. trial court’s decision: 1(c) which stated:
Analogous to the present case is Romero v. Court of It is therefore apparent that the vendee’s obligations (sic) c. That the access road or Road Right of Way leading to
Appeals,62 wherein the Court interpreted the legal effect of to pay the balance of the purchase price arises only when Lot 10963 shall be the responsibility of the VENDEE to
a condition in a deed of sale that the balance of the the road-right-of-way to the property shall have been secure and any or all cost relative to the acquisition
purchase price would be paid by the vendee when the successfully negotiated, secured and provided. In other thereof shall be borne solely by the VENDEE. He shall,
vendor has successfully ejected the informal settlers words, the obligation to pay the balance is conditioned however, be accorded with enough time necessary for the
occupying the property. In Romero, we found that such a upon the acquisition of the road-right-of-way, in success of his endeavor, granting him a free hand in
condition did not affect the perfection of the contract but accordance with paragraph 2 of Article 1181 of the New negotiating for the passage.66 (Emphasis supplied.)
only imposed a condition on the fulfillment of the Civil Code. Accordingly, "an obligation dependent upon a The Catungals’ interpretation of the foregoing stipulation
obligation to pay the balance of the purchase price, to wit: suspensive condition cannot be demanded until after the was that Rodriguez’s obligation to negotiate and secure a
From the moment the contract is perfected, the parties are condition takes place because it is only after the fulfillment road right of way was one with a period and that period,
bound not only to the fulfillment of what has been of the condition that the obligation arises." (Javier v[s] CA i.e., "enough time" to negotiate, had already lapsed by the
expressly stipulated but also to all the consequences 183 SCRA) Exhibits H, D, P, R, T, FF and JJ show that time they demanded the payment of ₱5,000,000.00 from
which, according to their nature, may be in keeping with plaintiff [Rodriguez] indeed was diligent in his efforts to respondent. Even assuming arguendo that the Catungals
good faith, usage and law. Under the agreement, private negotiate for a road-right-of-way to the property. The were correct that the respondent’s obligation to negotiate
respondent is obligated to evict the squatters on the written offers, proposals and follow-up of his proposals a road right of way was one with an uncertain period, their
property. The ejectment of the squatters is a condition the show that plaintiff [Rodriguez] went all out in his efforts to rescission of the Conditional Deed of Sale would still be
operative act of which sets into motion the period of immediately acquire an access road to the property, even unwarranted. Based on their own theory, the Catungals
compliance by petitioner of his own obligation, i.e., to pay going to the extent of offering ₱3,000.00 per square meter had a remedy under Article 1197 of the Civil Code, which
the balance of the purchase price. Private respondent's for the road lots (Exh. Q) from the original ₱550.00 per sq. mandates:
failure "to remove the squatters from the property" within meter. This Court also notes that defendant (sic) [the Art. 1197. If the obligation does not fix a period, but from
the stipulated period gives petitioner the right to either Catungals] made misrepresentation in the negotiation they its nature and the circumstances it can be inferred that a
refuse to proceed with the agreement or waive that have entered into with plaintiff [Rodriguez]. (Exhs. F and period was intended, the courts may fix the duration
condition in consonance with Article 1545 of the Civil G) The misrepresentation of defendant (sic) [the thereof.
Code. This option clearly belongs to petitioner and not to Catungals] as to the third lot (Lot 10986) to be part and The courts shall also fix the duration of the period when it
private respondent. parcel of the subject property [(]Lot 10963) contributed in depends upon the will of the debtor.
We share the opinion of the appellate court that the defeating the plaintiff’s [Rodriguez’s] effort in acquiring the In every case, the courts shall determine such period as
undertaking required of private respondent does not road-right-of-way to the property. Defendants [the may under the circumstances have been probably
constitute a "potestative condition dependent solely on his Catungals] cannot now invoke the non-fulfillment of the contemplated by the parties. Once fixed by the courts, the
will" that might, otherwise, be void in accordance with condition in the contract as a ground for rescission when period cannot be changed by them.
Article 1182 of the Civil Code but a "mixed" condition defendants [the Catungals] themselves are guilty of What the Catungals should have done was to first file an
"dependent not on the will of the vendor alone but also of preventing the fulfillment of such condition. action in court to fix the period within which Rodriguez
third persons like the squatters and government agencies From the foregoing, this Court is of the considered view should accomplish the successful negotiation of the road
and personnel concerned." We must hasten to add, that rescission of the conditional deed of sale by the right of way pursuant to the above quoted provision. Thus,
239
Sales – Chapter 3-12 Cases
the Catungals’ demand for Rodriguez to make an Bearing in mind the aforementioned interpretative rules, negotiation is successful), Rodriguez must pay the
additional payment of ₱5,000,000.00 was premature and we find that the first sentence of paragraph 5 must be balance of the purchase price. In the event the condition
Rodriguez’s failure to accede to such demand did not taken in relation with the rest of paragraph 5 and with the is not fulfilled (or the negotiation fails), Rodriguez has the
justify the rescission of the contract. other provisions of the Conditional Deed of Sale. choice either (a) to not proceed with the sale and demand
With respect to petitioners’ argument that paragraph 5 of Reading paragraph 5 in its entirety will show that return of his downpayment or (b) considering that the
the Conditional Deed of Sale likewise rendered the said Rodriguez’s option to rescind the contract is not absolute condition was imposed for his benefit, to waive the
contract void, we find no merit to this theory. Paragraph 5 as it is subject to the requirement that there should be condition and still pay the purchase price despite the lack
provides: written notice to the vendor and the vendor shall only of road access. This is the most just interpretation of the
5. That the VENDEE has the option to rescind the sale. In return Rodriguez’s downpayment of ₱500,000.00, without parties’ contract that gives effect to all its provisions.
the event the VENDEE exercises his option to rescind the interest, when the vendor shall have been able to sell the In any event, even if we assume for the sake of argument
herein Conditional Deed of Sale, the VENDEE shall notify property to another party. That what is stipulated to be that the grant to Rodriguez of an option to rescind, in the
the VENDOR by way of a written notice relinquishing his returned is only the downpayment of ₱500,000.00 in the manner provided for in the contract, is tantamount to a
rights over the property. The VENDEE shall then be event that Rodriguez exercises his option to rescind is potestative condition, not being a condition affecting the
reimbursed by the VENDOR the sum of FIVE HUNDRED significant. To recall, paragraph 1(b) of the contract clearly perfection of the contract, only the said condition would be
THOUSAND PESOS (₱500,000.00) representing the states that the installments on the balance of the purchase considered void and the rest of the contract will remain
downpayment, interest free, payable but contingent upon price shall only be paid upon successful negotiation and valid. In Romero, the Court observed that "where the so-
the event that the VENDOR shall have been able to sell procurement of a road right of way. It is clear from such called ‘potestative condition’ is imposed not on the birth of
the property to another party.67 provision that the existence of a road right of way is a the obligation but on its fulfillment, only the condition is
Petitioners posited that the above stipulation was the material consideration for Rodriguez to purchase the avoided, leaving unaffected the obligation itself." 71
"deadliest" provision in the Conditional Deed of Sale for property. Thus, prior to him being able to procure the road It cannot be gainsaid that "contracts have the force of law
violating the principle of mutuality of contracts since it right of way, by express stipulation in the contract, he is between the contracting parties and should be complied
purportedly rendered the contract subject to the will of not bound to make additional payments to the Catungals. with in good faith."72 We have also previously ruled that
respondent. It was further stipulated in paragraph 1(b) that: "[i]f "[b]eing the primary law between the parties, the contract
We do not agree. however said road right of way cannot be negotiated, the governs the adjudication of their rights and obligations. A
It is petitioners’ strategy to insist that the Court examine VENDEE shall give notice to the VENDOR for them to court has no alternative but to enforce the contractual
the first sentence of paragraph 5 alone and resist a reassess and solve the problem by taking other options stipulations in the manner they have been agreed upon
correlation of such sentence with other provisions of the and should the situation ultimately prove futile, he and written."73 We find no merit in petitioners’ contention
contract. Petitioners’ view, however, ignores a basic rule [Rodriguez] shall take steps to rescind or [cancel] the that their parents were merely "duped" into accepting the
in the interpretation of contracts – that the contract should herein Conditional Deed of Sale." The intention of the questioned provisions in the Conditional Deed of Sale. We
be taken as a whole. parties for providing subsequently in paragraph 5 that note that although the contract was between Agapita
Article 1374 of the Civil Code provides that "[t]he various Rodriguez has the option to rescind the sale is undeniably Catungal and Rodriguez, Jose Catungal nonetheless
stipulations of a contract shall be interpreted together, only limited to the contingency that Rodriguez shall not be signed thereon to signify his marital consent to the same.
attributing to the doubtful ones that sense which may able to secure the road right of way. Indeed, if the parties We concur with the trial court’s finding that the spouses
result from all of them taken jointly." The same Code intended to give Rodriguez the absolute option to rescind Catungals’ claim of being misled into signing the contract
further sets down the rule that "[i]f some stipulation of any the sale at any time, the contract would have provided for was contrary to human experience and conventional
contract should admit of several meanings, it shall be the return of all payments made by Rodriguez and not wisdom since it was Jose Catungal who was a practicing
understood as bearing that import which is most adequate only the downpayment. To our mind, the reason only the lawyer while Rodriquez was a non-lawyer.74 It can be
to render it effectual."68 downpayment was stipulated to be returned is that the reasonably presumed that Atty. Catungal and his wife
Similarly, under the Rules of Court it is prescribed that vendee’s option to rescind can only be exercised in the reviewed the provisions of the contract, understood and
"[i]n the construction of an instrument where there are event that no road right of way is secured and, thus, the accepted its provisions before they affixed their signatures
several provisions or particulars, such a construction is, if vendee has not made any additional payments, other than thereon.
possible, to be adopted as will give effect to all" 69 and "for his downpayment. After thorough review of the records of this case, we have
the proper construction of an instrument, the In sum, Rodriguez’s option to rescind the contract is not come to the conclusion that petitioners failed to
circumstances under which it was made, including the purely potestative but rather also subject to the same demonstrate that the Court of Appeals committed any
situation of the subject thereof and of the parties to it, may mixed condition as his obligation to pay the balance of the reversible error in deciding the present controversy.
be shown, so that the judge may be placed in the position purchase price – i.e., the negotiation of a road right of However, having made the observation that it was
of those whose language he is to interpret."70 way. In the event the condition is fulfilled (or the desirable for the Catungals to file a separate action to fix
240
Sales – Chapter 3-12 Cases
the period for respondent Rodriguez’s obligation to
negotiate a road right of way, the Court finds it necessary
to fix said period in these proceedings. It is but equitable
for us to make a determination of the issue here to obviate
further delay and in line with the judicial policy of avoiding
multiplicity of suits.
If still warranted, Rodriguez is given a period of thirty (30)
days from the finality of this decision to negotiate a road
right of way. In the event no road right of way is secured
by Rodriquez at the end of said period, the parties shall
reassess and discuss other options as stipulated in
paragraph 1(b) of the Conditional Deed of Sale and, for
this purpose, they are given a period of thirty (30) days to
agree on a course of action. Should the discussions of the
parties prove futile after the said thirty (30)-day period,
immediately upon the expiration of said period for
discussion, Rodriguez may (a) exercise his option to
rescind the contract, subject to the return of his
downpayment, in accordance with the provisions of
paragraphs 1(b) and 5 of the Conditional Deed of Sale or
(b) waive the road right of way and pay the balance of the
deducted purchase price as determined in the RTC
Decision dated May 30, 1992.
WHEREFORE, the Decision dated August 8, 2000 and
the Resolution dated January 30, 2001 of the Court of
Appeals in CA-G.R. CV No. 40627 consolidated with CA-
G.R. SP No. 27565 are AFFIRMED with the following
modification:
If still warranted, respondent Angel S. Rodriguez is given
a period of thirty (30) days from the finality of this Decision
to negotiate a road right of way. In the event no road right
of way is secured by respondent at the end of said period,
the parties shall reassess and discuss other options as
stipulated in paragraph 1(b) of the Conditional Deed of
Sale and, for this purpose, they are given a period of thirty
(30) days to agree on a course of action. Should the
discussions of the parties prove futile after the said thirty
(30)-day period, immediately upon the expiration of said
period for discussion, Rodriguez may (a) exercise his
option to rescind the contract, subject to the return of his
downpayment, in accordance with the provisions of
paragraphs 1(b) and 5 of the Conditional Deed of Sale or
(b) waive the road right of way and pay the balance of the
deducted purchase price as determined in the RTC
Decision dated May 30, 1992.
No pronouncement as to costs.
SO ORDERED.
241
Sales – Chapter 3-12 Cases
G.R. No. 132269 April 27, 2000 prove that all existing taxes and customs duties have The Court of Appeals subsequently sustained the lower
HARRISON MOTORS CORPORATION, petitioner, been paid. 7 court, hence this recourse of petitioner. 16
vs. In December of 1988 government agents seized and Petitioner argues that it was no longer obliged to pay for
RACHEL A. NAVARRO, respondent. detained the two (2) Elf trucks of respondent after the additional taxes and customs duties imposed on the
discovering that there were still unpaid BIR taxes and imported component parts by the Memorandum Orders
customs duties thereon. The BIR and the BOC ordered and the two (2) Memoranda of Agreement since such
BELLOSILLO, J.: private respondent to pay the proper assessments or her administrative regulations only took effect after the
This is a review on certiorari of the Decision of the Court trucks would be impounded. 8 Private respondent went to execution of its contract of sale with private respondent.
of Appeals affirming that of the Regional Trial Court of Claros to ask for the receipts evidencing payment of BIR Holding it liable for payment of the taxes specified in the
Makati which ordered petitioner to pay private respondent taxes and customs duties; however, Claros refused to administrative regulations, which have the force and effect
P32,943.00 as reimbursement for taxes paid, P7,500.00 comply. 9 Private respondent then demanded from Claros of laws, would not only violate the non-impairment clause
as attorney's fees and the costs of suit. 1 that he pay the assessed taxes and warned him that he of the Constitution but also the principle of non-
Sometime in June of 1987 Harrison Motors Corporation would have to reimburse her should she be forced to pay retroactivity of laws provided in Art. 4 of the Civil
through its president, Renato Claros, sold two (2) Isuzu Elf for the assessments herself. Her demands were again Code. 17 Furthermore, petitioner claims that it did pay the
trucks to private respondent Rachel Navarro, owner of RN ignored. assessed taxes and duties otherwise it would not have
Freight Lines, a franchise holder operating and But wanting to secure the immediate release of the trucks been able to secure the release of such spare parts from
maintaining a fleet of cargo trucks all over Luzon. to comply with her business commitments, private the customs and to register the vehicles with the LTO
Petitioner, a known importer, assembler and respondent paid the assessed BIR taxes and customs under its name.
manufacturer, assembled the two (2) trucks using duties amounting to P32,943.00. 10 Consequently, she The records however reveal that the Memorandum Orders
imported component parts. 2 Prior to the sale, Renato returned to petitioner's office to ask for reimbursement, and Memoranda of Agreement do not impose any
Claros represented to private respondent that all the BIR but petitioner again refused, prompting her to send a additional BIR taxes or customs duties.
taxes and customs duties for the parts used on the two (2) demand letter through her lawyer. 11 When petitioner still Customs Memorandum Order No. 44-87 is concerned
trucks had been paid for. 3 ignored her letter, she filed a complaint for a sum of with the Rules, Regulations and Procedures in the
On 10 September 1987 the Bureau of Internal Revenue money on 24 September 1990 with the Regional Trial Payment of Duties and Taxes on Imported Vehicles
(BIR) and the Land Transportation Office (LTO) entered Court of Makati. 12 Locally Assembled by Non-Assemblers. 18 It does not
into a Memorandum of Agreement (MOA) which provided On 24 May 1991 private respondent filed a Motion to charge any new tax. It simply provides the procedure on
that prior to registration in the LTO of any assembled or Declare Defendant in Default which was granted by the how owners/consignees or their purchasers could
re-assembled motor vehicle which used imported parts, a RTC on the same day. voluntarily initiate payment for any unpaid customs duties
Certificate of Payment should first be obtained from the On 18 November 1991 private respondent filed a on locally assembled vehicles using imported component
BIR to prove payment of all taxes required under existing Manifestation and Motion praying for the scheduling of the parts.
laws. 4 reception of her evidence ex-parte since petitioner had not Neither does BIR Revenue Memorandum Order No. 44-
On 12 October 1987 the Bureau of Customs (BOC) issued as yet filed a Motion to Lift Order of Default. Thus, on 22 87 19 exact any tax. It merely outlines the procedure which
Customs Memorandum Order No. 44-87 promulgating November 1991 the trial court ordered the reception of governs the processing and issuance of the Certificate of
rules, regulations and procedure for the voluntary private respondent's evidence ex-parte.13 Payment of internal revenue taxes for purposes of
payment of duties and taxes on imported motor vehicles It was only on 2 December 1991 when petitioner finally registering motor vehicles with the arms and telling her
assembled by non-assemblers. 5 filed a Motion to Lift Order of Default. However, on 20 she looked beautiful. The special treatment and sexual
Pursuant to the 10 September 1987 MOA between the January 1992 the trial court denied petitioner's motion for advances continued during her employment for four (4)
BIR and the LTO, the BIR issued on 18 December 1987 its failure to attach an affidavit of merit showing that it had years but she never reciprocated his flirtations, until
Revenue Memorandum Order No. 44-87 which provided a valid and meritorious defense. 14 finally, she noticed that his attitude towards her changed.
the procedure governing the processing and issuance of On 5 March 1992 the trial court rendered a decision He made her understand that if she would not give in to
the Certificate of Payment of internal revenue taxes for ordering petitioner to reimburse private respondent in the his sexual advances he would cause her termination from
purposes of registering motor vehicles. 6 amount of P32,943.00 for the customs duties and internal the service; and he made good his threat when he started
On 16 June 1988 the BIR, BOC and LTO entered into a revenue taxes the latter had to pay to discharge her two harassing her. She just found out one day that her table
tripartite MOA which provided that prior to the registration (2) Elf trucks from government custody. Petitioner was which was equipped with telephone and intercom units
in the LTO of any locally assembled motor vehicle using also required to pay P7,500.00 for attorney's fees plus the and containing her personal belongings was transferred
imported component parts, a Certificate of Payment costs. 15 without her knowledge to a place with neither telephone
should first be obtained from the BIR and the BOC to nor intercom, for which reason, an argument ensued when
242
Sales – Chapter 3-12 Cases
she confronted William Chua resulting in her being no longer owned the vehicles. Whatever payments private It is also quite obvious that as between petitioner, who is
charged with gross disrespect. 11 respondent made to the government after the sale were the importer-assembler/manufacturer, and private
Respondent Cortez explains, as regards the second solely her concern and such burden should not be passed respondent, who is merely the buyer, it is petitioner which
charge, that the money entrusted to her for transmittal on to petitioner. 22 Petitioner further argues that holding it has the obligation to pay taxes to the BIR and the BOC.
was not lost; instead, she gave it to the company liable for payment of BIR taxes and customs duties Petitioner would be unjustly enriched if private respondent
personnel in-charge for proper transmittal as evidenced by required under the administrative regulations violates the should be denied reimbursement. 24 It would inequitably
a receipt duly signed by the latter. 12 principle of non-retroactivity of laws under the Civil Code. amass profits from selling assembled trucks even if it did
With respect to the third imputation, private respondent Such contention deserves scant consideration. It is true not pay the taxes due on its imported spare parts.
admits that she asked someone to punch-in her time card that administrative rulings and regulations are generally Imposing the tax burden on private respondent would only
because at that time she was doing an errand for one of prospective in nature.23 An inspection of the two (2) encourage the proliferation of smugglers who scheme to
the company's officers, Richard Tan, and that was with Memoranda of Agreement however demonstrates that evade taxes by passing on their tax obligations to their
the permission of William Chua. She maintains that she their intent is to enforce payment of taxes on unsuspecting buyers.
did it in good faith believing that she was anyway only assemblers/manufacturers who import component parts In a last ditch effort to exempt itself from liability, petitioner
accommodating the request of a company executive and without paying the correct assessments. The WHEREAS claims that it had paid the taxes due on the imported parts
done for the benefit of the company with the clause of the 10 September 1987 MOA clearly illustrates otherwise it would not have been able to obtain their
acquiescence of her boss, William Chua. Besides, the this — release from the BOC and to register the vehicles with the
practice was apparently tolerated as the employees were WHEREAS, in order to avoid or evade the higher taxes on LTO.
not getting any the BIR which would then transmit the imported motor vehicles, certain persons import parts and Non-sequitur. The fact that petitioner was able to secure
Certificate to the LTO to prove that all the BIR taxes assemble and re-assemble them into complete motor the release of the parts from customs and to register the
required under existing laws have been paid. vehicles, or assemble or re-assemble motor vehicles assembled trucks with the LTO does not necessarily
The 16 June 1988 tripartite MOA among the BIR, LTO using imported parts; mean that all taxes and customs duties were legally
and the BOC virtually contained the same provisions. The While the WHEREAS clause of the 16 June 1988 MOA settled. As a matter of fact, the provisions of the two (2)
MOA provided that prior to registration with the LTO of provides — Memoranda of Agreement clearly establish that the
any assembled motor vehicle using imported component WHEREAS, in order to avoid or evade the higher taxes on government is aware of the widespread registration of
parts, a certificate of Payment should first be secured from imported motor vehicles, certain persons, firms or assembled motor vehicles with the LTO even if the taxes
the BIR or the BOC which should then be duly forwarded corporations who are non-BOI licensed assemblers of due on their imported component parts remain unpaid.
to LTO. The Certificate would serve as proof that all taxes imported motor vehicle component parts would assemble Paragraph 1 of the 10 September 1987 MOA states —
and customs duties required under existing laws, rules or re-assemble them into whole unit motor vehicles; The LTO shall make as one of the requirements for the
and regulations had already been settled. It is also apparent in Par. 9 of the 16 June 1988 MOA that registration of motor vehicles that were assembled or re-
Clearly, petitioner's contention is unmeritorious. What the taxes to be enforced are designated as assembled using imported parts, the payment to the BIR
Sec. 10, Art. III, of the Constitution prohibits is the assembler's/manufacturer's tax. It states — of the taxes required under existing laws.
passage of a law which enlarges, abridges or in any 9. The BIR shall collect the assembler's/manufacturer's The same requirement shall be imposed with respect to
manner changes the intention of the contracting tax, while the BOC shall collect the duties and taxes the renewal of the registration of such motor vehicles had
parties. 21 The Memorandum Orders and the two (2) and ad valorem tax. they been registered or their registration been renewed in
Memoranda of Agreement do not impose any additional Thus, although private respondent is the one required by the past without the payment of the required taxes.
taxes which would unduly impair the contract of sale the administrative regulations to secure the Certificate of While par. 1 of the 16 June 1988 MOA states —
between petitioner and private respondent. Instead, these Payment for the purpose of registration, petitioner as the The LTO shall make as one of the requirements for the
administrative regulations were passed to enforce importer and the assembler/manufacturer of the two (2) registration of motor vehicles that were assembled or re-
payment of existing BIR taxes and customs duties at the Elf trucks is still the one liable for payment of revenue assembled by non-BOI licensed assemblers using
time of importation. taxes and customs duties. Petitioner's obligation to pay imported component parts, the payment to the BIR and
But who should pay the BIR taxes and customs duties does not arise from the administrative regulations but from the BOC of taxes and duties required under existing laws,
which the administrative regulations sought to enforce? the tax laws existing at the time of importation. Hence, rules and regulations;
Petitioner contends that private respondent should be the even if private respondent already owned the two (2) The same requirement shall be imposed with respect to
one to pay the internal revenue taxes and customs duties. trucks when the Memorandum Orders and Memoranda of the renewal of the registration of such motor vehicles even
It claims that at the time the Memorandum Orders and the Agreement took effect, the fact remains that petitioner was if they were already registered or their registration had
two (2) Memoranda of Agreement took effect the two (2) still the one duty-bound to pay for the BIR taxes and been renewed in the past without the payment of the
Elf trucks were already sold to private respondent, thus, it customs duties. required taxes.
243
Sales – Chapter 3-12 Cases
Obviously, the two (2) Memoranda of Agreement were needed for her business and filed a complaint for
executed to prevent the anomalous circumstance, as in damages, particularly seeking the reimbursement of the
the case at bar, where assembled vehicles are registered amount she paid to secure the release of her vehicles.
with the LTO even if taxes and customs duties remain WHEREFORE, the Decision of the Court of Appeals dated
unpaid. 22 January 1998 ordering petitioner HARRISON
Besides, petitioner's allegation that it already paid the BIR MOTORS CORPORATION to reimburse private
taxes and customs duties is highly doubtful. This entire respondent RACHEL A. NAVARRO for the taxes and
controversy would have been avoided had petitioner duties she paid in the amount of P32,943.00 and to pay
simply furnished private respondent with the receipts her attorney's fees in the amount of P7,500.00 is
evidencing payment of BIR taxes and customs duties. If AFFIRMED. In addition, the amount of P32,943.00 shall
only private respondent had the receipts to prove payment earn interest at the legal rate from 24 September 1990
of such assessments then she would have easily secured when private respondent filed her complaint with the trial
the release of her two (2) Elf trucks. But petitioner court until fully paid. Costs against petitioner.
arbitrarily and unjustly denied private respondent's SO ORDERED.1âwphi1.nêt
demands. Instead, petitioner obstinately insisted that it
was no longer concerned with the problem involving the
two (2) trucks since it no longer owned the vehicles after
the consummation of the sale.
It is true that the ownership of the trucks shifted to private
respondent after the sale. But petitioner must remember
that prior to its consummation it expressly intimated to her
that it had already paid the taxes and customs
duties.25Such representation shall be considered as a
seller's express warranty under Art. 1546 of the Civil Code
which covers any affirmation of fact or any promise by the
seller which induces the buyer to purchase the thing and
actually purchases it relying on such affirmation or
promise. 26 It includes all warranties which are derived
from express language, whether the language is in the
form of a promise or representation. 27 Presumably,
therefore, private respondent would not have purchased
the two (2) Elf trucks were it not for petitioner's assertion
and assurance that all taxes on its imported parts were
already settled.
This express warranty was breached the moment
petitioner refused to furnish private respondent with the
corresponding receipts since such documents were the
best evidence she could present to the government to
prove that all BIR taxes and customs duties on the
imported component parts were fully paid. Without
evidence of payment, she was powerless to prevent the
trucks from being impounded.
Under Art. 1599 of the Civil Code, once an express
warranty is breached the buyer can accept or keep the
goods and maintain an action against the seller for
damages. This was what private respondent did. She
opted to keep the two (2) trucks which she apparently
244
Sales – Chapter 3-12 Cases
G.R. No. 73913 January 31, 1989 of the Philippines. (hereinafter, DBP) for the purchase d) P5,000.00- Insurance that Crispin will come back and
JERRY T. MOLES, petitioner, thereof. An agent of Smith, Bell and Co. who is a friend of repair the linotype machine at seller's account as provided
vs. petitioner introduced the latter to private respondent, in the contract; after Crispin has put everything in order
INTERMEDIATE APPELLATE COURT and MARIANO owner of the Diolosa Publishing House in Iloilo City, who when he goes home on Sunday he will return the check of
M. DIOLOSA, respondents. had two available machines. Thereafter, petitioner went to P15,000.00.
Zoilo V. De la Cruz, Jr., Kenneth Barredo, Romeo Sabig Iloilo City to inspect the two machines offered for sale and 2) Official receipt in the amount of P 50,000.00 as full
and Natalio V. Sitjao for petitioners. was informed that the same were secondhand but payment of the linotype machine.
Rolando N. Medalla and Jose G. Guinez, Jr., for private functional. These were immediately complied with by private
respondents. On his second visit to the Diolosa Publishing House, respondent and on the same day, September 30,1977, he
petitioner together with Rogelio Yusay, a letter press received the DBP check for P50,000.00. 9
REGALADO, J.: machine operator, decided to buy the linotype machine, It is to be noted that the aforesaid official receipt No. 0451,
This petition for review on certiorari assails the decision of Model 14. The transaction was basically verbal in nature dated September 30, 1977 and prepared and signed by
the then Intermediate Appellate Court 1 dismissing the but to facilitate the loan application with the DBP, a pro private respondent, expressly states that he received from
complaint filed by herein petitioner against the herein forma invoice, dated April 23, 1977 and reflecting the the petitioner the DBP check for P50,000.00 issued in our
private respondent in the former Court of First Instance of amount of P50,000.00 as the consideration of the sale, favor in full payment of one (1) Unit Model 14 Linotype
Negros Occidental in Civil Case No. 13821 thereof. 2 was signed by petitioner with an addendum that payment Machine as per Pro forma Invoice dated April 23, 1977. 10
The factual backdrop of this controversy, as culled from had not yet been made but that he promised to pay the On November 29, 1977, petitioner wrote private
the records, 3 shows that on May 17, 1978, petitioner full amount upon the release of his loan from the respondent that the machine was not functioning properly
Jerry T. Moles commenced a suit against private aforementioned bank on or before the end of the as it needed a new distributor bar. In the same letter,
respondent Mariano M. Diolosa in the aforesaid trial court, month. 5 Although the agreed selling price was only petitioner unburdened himself of his grievances and
Branch IV in Bacolod City, for rescission of contract with P40,000.00, the amount on the invoice was increased by sentiments in this wise.
damages. Private respondent moved to dismiss on the P10,000.00, said increase being intended for the We bought this machine in good faith because we trusted
ground of improper venue, invoking therefor Sales Invoice purchase of new matrices for said machine. you very much being our elder brother in printing and
No. 075A executed between petitioner and private Sometime between April and May, 1977, the machine was publishing business. We did not hire anybody to look over
respondent on April 23, 1977 which provides that all delivered to petitioner's publishing house at Tangub, the machine, much more ask for a rebate in your price of
judicial actions arising from this contract shall be instituted Bacolod City where it was installed by one Crispino P40,000.00 and believed what your trusted two men,
in the City of Iloilo. 4This was opposed by petitioner who Escurido, an employee of respondent Diolosa. Another Tomas and Crispin, said although they were hiding the
averred that there is no formal document evidencing the employee of the Diolosa Publishing House, Tomas real and actual condition of the machine for your business
sale which is substantially verbal in character. In an order Plondaya, stayed at petitioners house for almost a month protection.
dated June 23, 1978, the trial court denied the motion to to train the latter's cousin in operating the machine. 6 Until last week, we found out the worst ever to happen to
dismiss, holding that the question of venue could not be Under date of August 29, 1977, private respondent issued us. We have been cheated because the expert of the
resolved at said stage of the case. The subsequent a certification wherein he warranted that the machine sold Linotype machine from Manila says, that the most he will
motion for reconsideration was likewise denied. was in A-1 condition, together with other express buy your machine is at P5,000.00 only. ... 11
Consequently, private respondent, invoking the aforesaid warranties. 7 Private respondent made no reply to said letter, so
venue stipulation, preceeded to this Court on a petition for Prior to the release of the loan, a representative from the petitioner engaged the services of other technicians.
prohibition with preliminary injunction in G.R. No. 49078, DBP, Bacolod, supposedly inspected the machine but he Later, after several telephone calls regarding the defects
questioning the validity of the order denying his aforesaid merely looked at it to see that it was there .8 The in the machine, private respondent sent two technicians to
two motions and seeking to enjoin the trial court from inspector's recommendation was favorable and, make the necessary repairs but they failed to put the
further proceeding with the case. This petition was thereafter, petitioner's loan of P50,000.00 was granted machine in running condition. In fact, since then petitioner
dismissed for lack of merit in a resolution of the Court, and released. However, before payment was made to was never able to use the machine.12
dated February 7, 1979, and which became final on March private respondent, petitioner required the former, in a On February 18, 1978, not having received from private
15, 1979. Thereafter, private respondent filed his answer letter dated September 30, 1977, to accomplish the respondent the action requested in his preceding letter as
and proceeded to trial. following, with the explanations indicated by him: herein before stated, petitioner again wrote private
The aforecited records establish that sometime in 1977, 1.) Crossed check for P15,407.10 representing. respondent, this time with the warning that he would be
petitioner needed a linotype printing machine for his a) P 10,000.00-Overprice in the machine: forced to seek legal remedies to protect his interest. 13
printing business, The LM Press at Bacolod City, and b) P203.00-Freight and handling of the machine; Obviously in response to the foregoing letter, private
applied for an industrial loan with the Development Bank c) P203.00-Share in the electric repair; and respondent decided to purchase a new distributor bar
245
Sales – Chapter 3-12 Cases
and, on March 16, 1978, private respondent delivered this From this decision, private respondent appealed to the of its quality or fitness. It is generally held that in the sale
spare part to petitioner through one Pedro Candido. Intermediate Appellate Court which reversed the judgment of a designated and specific article sold as secondhand,
However, when thereafter petitioner asked private of the lower court and dismissed petitioner's complaint, there is no implied warranty as to its quality or fitness for
respondent to pay for the price of the distributor bar, the hence the present petition. the purpose intended, at least where it is subject to
latter asked petitioner to share the cost with him. We find merit in petitioner's cause. inspection at the time of the sale. On the other hand, there
Petitioner thus finally decided to indorse the matter to his On the matter of venue, private respondent relies on the is also authority to the effect that in a sale of a
lawyer. aforementioned Sales Invoice No. 076A which allegedly secondhand articles there may be, under some
An expert witness for the petitioner, one Gil Legaspina, requires that the proper venue should be Iloilo City and circumstances, an implied warranty of fitness for the
declared that he inspected the linotype machine involved not Bacolod City. We agree with petitioner that said ordinary purpose of the article sold or for the particular
in this case at the instance of petitioner. In his inspection document is not the contract evidencing the sale of the purpose of the buyer. 19
thereof, he found the following defects: (1) the vertical linotype machine, it being merely a preliminary In a line of decisions rendered by the United States
automatic stop lever in the casting division was worn out; memorandum of a proposal to buy one linotype machine, Supreme Court, it had theretofore been held that there is
(2) the justification lever had a slight breach (balana in the using for such purpose a printed form used for printing job no implied warranty as to the condition, adaptation,
dialect); (3) the distributor bar was worn out; (4) the orders in private respondent's printing business. As fitness, or suitability for the purpose for which made, or
partition at the entrance channel had a tear; (5) there was hereinbefore explained, this issue on venue was brought the quality, of an article sold as and for a secondhand
no "pie stacker" tube entrance; and (6) the slouch arm to Us by private respondent in a special civil action for article. 20
lever in the driving division was worn out. prohibition with preliminary injunction in G.R. No. 49078. Thus, in finding for private respondent, the respondent
It turned out that the said linotype machine was the same After considering the allegations contained, the issues court cited the ruling in Sison vs. Ago, et al. 21 to the effect
machine that witness Legaspina had previously inspected raised and the arguments adduced in said petition, as well that unless goods are sold as to raise an implied warranty,
for Sy Brothers, a firm which also wanted to buy a linotype as the comments thereto, the Court dismissed the petition as a general rule there is no implied warranty in the sale
machine for their printing establishment. Having found for lack of merit. Respondent court erred in reopening the of secondhand articles.22
defects in said machine, the witness informed Sy Brother same issue on appeal, with a contrary ruling. Said general rule, however, is not without exceptions.
about his findings, hence the purchase was aborted. In his Furthermore, it was error for the respondent court, after Article 1562 of our Civil Code, which was taken from the
opinion, major repairs were needed to put the machine adopting the factual findings of the lower court, to reverse Uniform Sales Act, provides:
back in good running condition.14 the latter's holding that the sales invoice is merely a pro Art. 1562. In a sale of goods, there is an implied warranty
After trial, the court a quo rendered a decision the forma memorandum. The records do not show that this or condition as to the quality or fitness of the goods, as
dispositive portion of which reads: finding is grounded entirely on speculation, surmises or follows:
IN VIEW OF THE FOREGOING CONSIDERATIONS, conjectures as to warrant a reversal thereof. 16 In fact, as (1) Where the buyer, expressly or by implication, makes
judgment is hereby rendered as follows: hereinbefore stated, private respondent expressly known to the seller the particular purpose for which the
(1) Decreeing the rescission of the contract of sale admitted in his official receipt No. 0451, dated September goods are acquired, and it appears that the buyer relies
involving one linotype machine No. 14 between the 30, 1977, that the said sales invoice was merely a pro on the seller's skill or judgment (whether he be the grower
defendant as seller and the plaintiff as buyer; forma invoice. Consequently, the printed provisions or manufacturer or not), there is an implied warranty that
(2) Ordering the plaintiff to return to the defendant at the therein, especially since the printed form used was for the goods shall be reasonably fit for such purpose;
latter's place of business in Iloilo City the linotype machine purposes of other types of transactions, could not have xxx
aforementioned together with all accessories that been intended by the parties to govern their transaction In Drumar Mining Co. vs. Morris Ravine Mining Co., 23 the
originally were delivered to the plaintiff; on the printing machine. It is obvious that a venue District Court of Appeals, 3rd District, California, in
(3) Ordering the defendant to return to the plaintiff the sum stipulation, in order to bind the parties, must have been applying a similar provision of law, ruled:
of Forty Thousand Pesos (P40,000.00) representing the intelligently and deliberately intended by them to exclude 'There is nothing in the Uniform Sales Act declaring there
price of the linotype machine, plus interest at the legal rate their case from the reglementary rules on venue. Yet, is no implied warranty in the sale of secondhand goods.
counted from May 17, 1978 when this action was even such intended variance may not necessarily be Section 1735 of the Civil Code declares there is no
instituted, until fully paid; given judicial approval, as, for instance, where there are implied warranty or condition as to the quality or fitness for
(4) Ordering the defendant to indemnify the plaintiff the no restrictive or qualifying words in the agreement any particular purpose, of goods supplied under a contract
sum of Four Thousand Five Hundred Pesos (P4,500.00) indicating that venue cannot be laid in any place other to sell or a sale, except (this general statement is followed
representing unearned income or actual damages; than that agreed upon by the parties, 17 and in contracts of by an enumeration of several exceptions). It would seem
(5) Ordering the defendant to pay the plaintiff the sum of adhesion. 18 that the legislature intended this section to apply to all
One Thousand Pesos (Pl,000.00) for attorney's fees. Now, when an article is sold as a secondhand item, a sales of goods, whether new or secondhand. In
Costs against the defendant.15 question arises as to whether there is an implied warranty subdivision 1 of this section, this language is used: where
246
Sales – Chapter 3-12 Cases
the buyer ... makes known to the seller the particular petitioner to purchase it but to confirm in writing for and the decision of the court a quo is hereby
purpose for which the goods are required, and it appears purposes of the financing aspect of the transaction his REINSTATED.
that the buyer relies on the seller's skill or judgment ... representations thereon. Ordinarily, what does not appear SO ORDERED.
there is an implied warranty that the goods shall be on the face of the written instrument should be regarded
reasonably fit for such purpose.' as dealer's or trader's talk; 25 conversely, what is
Furthermore, and of a more determinative role in this specifically represented as true in said document, as in
case, a perusal of past American decisions 24 likewise the instant case, cannot be considered as mere dealer's
reveals a uniform pattern of rulings to the effect that an talk.
express warranty can be made by and also be binding on On the question as to whether the hidden defects in the
the seller even in the sale of a secondhand article. machine is sufficient to warrant a rescission of the
In the aforecited case of Markman vs. Hallbeck, while contract between the parties, we have to consider the rule
holding that there was an express warranty in the sale of a on redhibitory defects contemplated in Article 1561 of the
secondhand engine, the court said that it was not error to Civil Code. A redhibitory defect must be an imperfection or
refuse an instruction that upon the sale of secondhand defect of such nature as to engender a certain degree of
goods no warranty was implied, since secondhand goods importance. An imperfection or defect of little
might be sold under such circumstances as to raise an consequence does not come within the category of being
implied warranty. redhibitory.26
To repeat, in the case before Us, a certification to the As already narrated, an expert witness for the petitioner
effect that the linotype machine bought by petitioner was categorically established that the machine required major
in A-1 condition was issued by private respondent in favor repairs before it could be used. This, plus the fact that
of the former. This cannot but be considered as an petitioner never made appropriate use of the machine
express warranty. However, it is private respondent's from the time of purchase until an action was filed, attest
submission, that the same is not binding on him, not being to the major defects in said machine, by reason of which
a part of the contract of sale between them. This the rescission of the contract of sale is sought. The factual
contention is bereft of substance. finding, therefore, of the trial court that the machine is not
It must be remembered that the certification was a reasonably fit for the particular purpose for which it was
condition sine qua non for the release of petitioner's loan intended must be upheld, there being ample evidence to
which was to be used as payment for the purchase price sustain the same.
of the machine. Private respondent failed to refute this At a belated stage of this appeal, private respondent
material fact. Neither does he explain why he made that came up for the first time with the contention that the
express warranty on the condition of the machine if he action for rescission is barred by prescription. While it is
had not intended to be bound by it. In fact, the respondent true that Article 1571 of the Civil Code provides for a
court, in declaring that petitioner should have availed of prescriptive period of six months for a redhibitory action a
the remedy of requiring repairs as provided for in said cursory reading of the ten preceding articles to which it
certification, thereby considered the same as part and refers will reveal that said rule may be applied only in case
parcel of the verbal contract between the parties. of implied warranties. The present case involves one with
On the basis of the foregoing circumstances, the and express warranty. Consequently, the general rule on
inescapable conclusion is that private respondent is rescission of contract, which is four years 27 shall apply.
indeed bound by the express warranty he executed in Considering that the original case for rescission was filed
favor of herein petitioner. only one year after the delivery of the subject machine,
We disagree with respondent court that private the same is well within the prescriptive period. This is
respondents express warranty as to the A-1 condition of aside from the doctrinal rule that the defense of
the machine was merely dealer's talk. Private respondent prescription is waived and cannot be considered on
was not a dealer of printing or linotype machines to whom appeal if not raised in the trial court, 28 and this case does
could be ascribed the supposed resort to the usual not have the features for an exception to said rule.
exaggerations of trade in said items. His certification as to WHEREFORE, the judgment of dismissal of the
the condition of the machine was not made to induce respondent court is hereby REVERSED and SET ASIDE,
247
Sales – Chapter 3-12 Cases
G.R. No. 119745 June 20, 1997 That for and in consideration of the sum of Two Hundred On the same date, Mrs. C.D. Constantino, then General
POWER COMMERCIAL AND INDUSTRIAL Ninety-Five Thousand Pesos (P295,000.00) Philippine Manager of petitioner-corporation, submitted to PNB said
CORPORATION, petitioner, Currency, to us in hand paid in cash, and which we deed with a formal application for assumption of
vs. hereby acknowledge to be payment in full and received to mortgage.4
COURT OF APPEALS, SPOUSES REYNALDO and our entire satisfaction, by POWER COMMERCIAL AND On February 15, 1980, PNB informed respondent spouses
ANGELITA R. QUIAMBAO and PHILIPPINE NATIONAL INDUSTRIAL DEVELOPMENT CORPORATION, a 100% that, for petitioner's failure to submit the papers necessary
BANK, respondents. Filipino Corporation, organized and existing under and by for approval pursuant to the former's letter dated January
virtue of Philippine Laws with offices located at 252-C Vito 15, 1980, the application for assumption of mortgage was
PANGANIBAN, J.: Cruz Extension, we hereby by these presents SELL, considered withdrawn; that the outstanding balance of
Is the seller's failure to eject the lessees from a lot that is TRANSFER and CONVEY by way of absolute sale the P145,000.00 was deemed fully due and demandable; and
the subject of a contract of sale with assumption of above described property with all the improvements that said loan was to be paid in full within fifteen (15) days
mortgage a ground (1) for rescission of such contract and existing thereon unto the said Power Commercial and from notice.5
(2) for a return by the mortgagee of the amortization Industrial Development Corporation, its successors and Petitioner paid PNB P41,880.45 on June 24, 1980 and
payments made by the buyer who assumed such assigns, free from all liens and encumbrances. P20,283.14 on December 23, 1980, payments which were
mortgage? We hereby certify that the aforesaid property is not subject to be applied to the outstanding loan. On December 23,
Petitioner posits an affirmative answer to such question in to nor covered by the provisions of the Land Reform Code 1980, PNB received a letter from petitioner which reads: 6
this petition for review on certiorari of the March 27, 1995 — the same having no agricultural lessee and/or tenant. With regard to the presence of the people who are
Decision1 of the Court of Appeals, Eighth Division, in CA- We hereby also warrant that we are the lawful and currently in physical occupancy of the (l)ot . . . it is our
G.R. CV Case No. 32298 upholding the validity of the absolute owners of the above described property, free desire as buyers and new owners of this lot to make use
contract of sale with assumption of mortgage and from any lien and/or encumbrance, and we hereby agree of this lot for our own purpose, which is why it is our desire
absolving the mortgagee from the liability of returning the and warrant to defend its title and peaceful possession and intention that all the people who are currently
mortgage payments already made.2 thereof in favor of the said Power Commercial and physically present and in occupation of said lot should be
The Facts Industrial Development Corporation, its successors and removed immediately.
Petitioner Power Commercial & Industrial Development assigns, against any claims whatsoever of any and all For this purpose we respectfully request that . . . our
Corporation, an industrial asbestos manufacturer, needed third persons; subject, however, to the provisions assumption of mortgage be given favorable consideration,
a bigger office space and warehouse for its products. For hereunder provided to wit: and that the mortgage and title be transferred to our name
this purpose, on January 31, 1979, it entered into a That the above described property is mortgaged to the so that we may undertake the necessary procedures to
contract of sale with the spouses Reynaldo and Angelita Philippine National Bank, Cubao, Branch, Quezon City for make use of this lot ourselves.
R. Quiambao, herein private respondents. The contract the amount of one hundred forty-five thousand pesos, It was our understanding that this lot was free and clear of
involved a 612-sq. m. parcel of land covered by Transfer Philippine, evidenced by document No. 163, found on problems of this nature, and that the previous owner
Certificate of Title No. S-6686 located at the corner of page No. 34 of Book No. XV, Series of 1979 of Notary would be responsible for the removal of the people who
Bagtican and St. Paul Streets, San Antonio Village, Makati Public Herita were there. Inasmuch as the previous owner has not been
City. The parties agreed that petitioner would pay private L. Altamirano registered with the Register of Deeds of able to keep his commitment, it will be necessary for us to
respondents P108,000.00 as down payment, and the Pasig (Makati), Rizal . . . ; take legal possession of this lot inorder (sic) to take
balance of P295,000.00 upon the execution of the deed of That the said Power Commercial and Industrial physical possession.
transfer of the title over the property. Further, petitioner Development Corporation assumes to pay in full the entire On February 19, 1982, PNB sent petitioner a letter as
assumed, as part of the purchase price, the existing amount of the said mortgage above described plus follows:7
mortgage on the land. In full satisfaction thereof, he paid interest and bank charges, to the said mortgagee bank, (T)his refers to the loan granted to Mr. Reynaldo
P79,145.77 to Respondent Philippine National Bank thus holding the herein vendor free from all claims by the Quiambao which was assumed by you on June 4, 1979
("PNB" for brevity). said bank; for P101,500.00. It was last renewed on December 24,
On June 1, 1979, respondent spouses mortgaged again That both parties herein agree to seek and secure the 1980 to mature on June 4, 1981.
said land to PNB to guarantee a loan of P145,000.00, agreement and approval of the said Philippine National A review of our records show that it has been past due
P80,000.00 of which was paid to respondent spouses. Bank to the herein sale of this property, hereby agreeing from last maturity with interest arrearages amounting to
Petitioner agreed to assume payment of the loan. to abide by any and all requirements of the said bank, P25,826.08 as of February 19, 1982. The last payment
On June 26, 1979, the parties executed a Deed of agreeing that failure to do so shall give to the bank first received by us was on December 24, 1980 for P20,283.
Absolute Sale With Assumption of Mortgage which lieu (sic) over the herein described property. 14. In order to place your account in current form, we
contained the following terms and conditions:3
248
Sales – Chapter 3-12 Cases
request you to remit payments to cover interest, charges, sale, nor was the occupation thereof by said lessees a assigns, against any claims whatsoever of any and all
and at least part of the principal. violation of the warranty against eviction. Hence, there third persons; subject, however, to the provisions
On March 17, 1982, petitioner filed Civil Case No. 45217 was no substantial breach to justify the rescission of said hereunder provided to wit:
against respondent spouses for rescission and damages contract or the return of the payments made. The By his own admission, Anthony Powers, General Manager
before the Regional Trial Court of Pasig, Branch 159. dispositive portion of said Decision reads: 11 of petitioner-corporation, did not ask the corporation's
Then, in its reply to PNB's letter of February 19, 1982, WHEREFORE, the Decision appealed from is hereby lawyers to stipulate in the contract that Respondent
petitioner demanded the return of the payments it made REVERSED and the complaint filed by Power Commercial Reynaldo was guaranteeing the ejectment of the
on the ground that its assumption of mortgage was never and Industrial Development Corporation against the occupants, because there was already a proviso in said
approved. On May 31, 1983,8 while this case was spouses Reynaldo and Angelita Quiambao and the deed of sale that the sellers were guaranteeing the
pending, the mortgage was foreclosed. The property was Philippine National Bank is DISMISSED. No costs. peaceful possession by the buyer of the land in
subsequently bought by PNB during the public auction. Hence, the recourse to this Court. question. 15 Any obscurity in a contract, if the above-
Thus, an amended complaint was filed impleading PNB as Issues quoted provision can be so described, must be construed
party defendant. Petitioner contends that: (1) there was a substantial against the party who caused it. 16 Petitioner itself caused
On July 12, 1990, the trial court9 ruled that the failure of breach of the contract between the parties warranting the obscurity because it omitted this alleged condition
respondent spouses to deliver actual possession to rescission; and (2) there was a "mistake in payment" when its lawyer drafted said contract.
petitioner entitled the latter to rescind the sale, and in view made by petitioner, obligating PNB to return such If the parties intended to impose on respondent spouses
of such failure and of the denial of the latter's assumption payments. In its Memorandum, it specifically assigns the the obligation to eject the tenants from the lot sold, it
of mortgage, PNB was obliged to return the payments following errors of law on the part of Respondent Court: 12 should have included in the contract a provision similar to
made by the latter. The dispositive portion of said decision A. Respondent Court of Appeals gravely erred in failing to that referred to in Romero vs. Court of Appeals, 17 where
states: 10 consider in its decision that a breach of implied warranty the ejectment of the occupants of the lot sold by private
IN VIEW OF ALL THE FOREGOING, the Court hereby under Article 1547 in relation to Article 1545 of the Civil respondent was the operative act which set into motion
renders judgment in favor of plaintiff and against Code applies in the case-at-bar. the period of petitioner's compliance with his own
defendants: B. Respondent Court of Appeals gravely erred in failing to obligation, i.e., to pay the balance of the purchase price.
(1) Declaring the rescission of the Deed of Sale with consider in its decision that a mistake in payment giving Failure to remove the squatters within the stipulated
Assumption of Mortgage executed between plaintiff and rise to a situation where the principle of solutio period gave the other party the right to either refuse to
defendants Spouses Quiambao, dated June 26, 1979; indebiti applies is obtaining in the case-at-bar. proceed with the agreement or to waive that condition of
(2) Ordering defendants Spouses Quiambao to return to The Court's Ruling ejectment in consonance with Article 1545 of the Civil
plaintiff the amount of P187,144.77 (P108,000.00 plus The petition is devoid of merit. It fails to appreciate the Code. In the case cited, the contract specifically stipulated
P79,145.77) with legal interest of 12% per annum from difference between a condition and a warranty and the that the ejectment was a condition to be fulfilled;
date of filing of herein complaint, that is, March 17, 1982 consequences of such distinction. otherwise, the obligation to pay the balance would not
until the same is fully paid; Conspicuous Absence of an Imposed Condition arise. This is not so in the case at bar.
(3) Ordering defendant PNB to return to plaintiff the The alleged "failure" of respondent spouses to eject the Absent a stipulation therefor, we cannot say that the
amount of P62,163.59 (P41,880.45 and P20,283.14) with lessees from the lot in question and to deliver actual and parties intended to make its nonfulfillment a ground for
12% interest thereon from date of herein judgment until physical possession thereof cannot be considered a rescission. If they did intend this, their contract should
the same is fully paid. substantial breach of a condition for two reasons: first, have expressly stipulated so. In Ang vs. C.A.,18 rescission
No award of other damages and attorney's fees, the same such "failure" was not stipulated as a condition — whether was sought on the ground that the petitioners had failed to
not being warranted under the facts and circumstances of resolutory or suspensive — in the contract; and second, fulfill their obligation "to remove and clear" the lot sold, the
the case. its effects and consequences were not specified either. 13 performance of which would have given rise to the
The counterclaim of both defendants spouses Quiambao The provision adverted to by petitioner does not impose a payment of the consideration by private respondent.
and PNB are dismissed for lack of merit. condition or an obligation to eject the lessees from the lot. Rescission was not allowed, however, because the
No pronouncement as to costs. The deed of sale provides in part: 14 breach was not substantial and fundamental to the
SO ORDERED. We hereby also warrant that we are the lawful and fulfillment by the petitioners of the obligation to sell.
On appeal by respondent-spouses and PNB, Respondent absolute owners of the above described property, free As stated, the provision adverted to in the contract
Court of Appeals reversed the trial court. In the assailed from any lien and/or encumbrance, and we hereby agree pertains to the usual warranty against eviction, and not to
Decision, it held that the deed of sale between respondent and warrant to defend its title and peaceful possession a condition that was not met.
spouses and petitioner did not obligate the former to eject thereof in favor of the said Power Commercial and The terms of the contract are so clear as to leave no room
the lessees from the land in question as a condition of the Industrial Development Corporation, its successors and for any other interpretation. 19
249
Sales – Chapter 3-12 Cases
Furthermore, petitioner was well aware of the presence of then fiction yields to reality — the delivery has not been public auction. But this deprivation is due to petitioner's
the tenants at the time it entered into the sales effected. fault, and not to any act attributable to the vendor-
transaction. As testified to by Reynaldo, 20 petitioner's Considering that the deed of sale between the parties did spouses.
counsel during the sales negotiation even undertook the not stipulate or infer otherwise, delivery was effected Because petitioner failed to impugn its integrity, the
job of ejecting the squatters. In fact, petitioner actually through the execution of said deed. The lot sold had been contract is presumed, under the law, to be valid and
filed suit to eject the occupants. Finally, petitioner in its placed under the control of petitioner; thus, the filing of the subsisting.
letter to PNB of December 23, 1980 admitted that it was ejectment suit was subsequently done. It signified that its Absence of Mistake In Payment
the "buyer(s) and new owner(s) of this lot." new owner intended to obtain for itself and to terminate Contrary to the contention of petitioner that a return of the
Effective Symbolic Delivery said occupants' actual possession thereof. Prior physical payments it made to PNB is warranted under Article 2154
The Court disagrees with petitioner's allegation that the delivery or possession is not legally required and the of the Code, solutio indebiti does not apply in this case.
respondent spouses failed to deliver the lot sold. execution of the deed of sale is deemed equivalent to This doctrine applies where: (1) a payment is made when
Petitioner asserts that the legal fiction of symbolic delivery delivery. 24 This deed operates as a formal or symbolic there exists no binding relation between the payor, who
yielded to the truth that, at the execution of the deed of delivery of the property sold and authorizes the buyer to has no duty to pay, and the person who received the
sale, transfer of possession of said lot was impossible due use the document as proof of ownership. Nothing more is payment, and (2) the payment is made through mistake,
to the presence of occupants on the lot sold. We find this required. and not through liberality or some other cause. 27
misleading. Requisites of Breach of Warranty Against Eviction In this case, petitioner was under obligation to pay the
Although most authorities consider transfer of ownership Obvious to us in the ambivalent stance of petitioner is its amortizations on the mortgage under the contract of sale
as the primary purpose of sale, delivery remains an failure to establish any breach of the warranty against and the deed of real estate mortgage. Under the deed of
indispensable requisite as our law does not admit the eviction. Despite its protestation that its acquisition of the sale (Exh. "2"), 28 both parties agreed to abide by any and
doctrine of transfer of property by mere consent. 21 The lot was to enable it to set up a warehouse for its asbestos all the requirements of PNB in connection with the real
Civil Code provides that delivery can either be (1) actual products and that failure to deliver actual possession estate mortgage. Petitioner was aware that the deed of
(Article 1497) or (2) constructive (Articles 1498-1501). thereof defeated this purpose, still no breach of warranty mortgage (Exh. "C") made it solidarily and, therefore,
Symbolic delivery (Article 1498), as a species of against eviction can be appreciated because the facts of primarily 29 liable for the mortgage obligation: 30
constructive delivery, effects the transfer of ownership the case do not show that the requisites for such breach (e) The Mortgagor shall neither lease the mortgaged
through the execution of a public document. Its efficacy have been satisfied. A breach of this warranty requires the property. . . nor sell or dispose of the same in any manner,
can, however, be prevented if the vendor does not concurrence of the following circumstances: without the written consent of the Mortgagee. However, if
possess control over the thing sold, 22 in which case this (1) The purchaser has been deprived of the whole or part not withstanding this stipulation and during the existence
legal fiction must yield to reality. of the thing sold; of this mortgage, the property herein mortgaged, or any
The key word is control, not possession, of the land as (2) This eviction is by a final judgment; portion thereof, is . . . sold, it shall be the obligation of the
petitioner would like us to believe. The Court has (3) The basis thereof is by virtue of a right prior to the sale Mortgagor to impose as a condition of the sale, alienation
consistently held that: 23 made by the vendor; and or encumbrance that the vendee, or the party in whose
. . . (I)n order that this symbolic delivery may produce the (4) The vendor has been summoned and made co- favor the alienation or encumbrance is to be made, should
effect of tradition, it is necessary that the vendor shall defendant in the suit for eviction at the instance of the take the property subject to the obligation of this mortgage
have had such control over the thing sold that . . . its vendee. 25 in the same terms and condition under which it is
material delivery could have been made. It is not enough In the absence of these requisites, a breach of the constituted, it being understood that the Mortgagor is not
to confer upon the purchaser the ownership and warranty against eviction under Article 1547 cannot be in any manner relieved of his obligation to the Mortgagee
the right of possession. The thing sold must be placed in declared. under this mortgage by such sale, alienation or
his control. When there is no impediment whatever to Petitioner argues in its memorandum that it has not yet encumbrance; on the contrary both the vendor and the
prevent the thing sold passing into the tenancy of the ejected the occupants of said lot, and not that it has been vendee, or the party in whose favor the alienation or
purchaser by the sole will of the vendor, symbolic delivery evicted therefrom. As correctly pointed out by Respondent encumbrance is made shall be jointly and severally liable
through the execution of a public instrument is sufficient. Court, the presence of lessees does not constitute an for said mortgage obligations. . . .
But if, notwithstanding the execution of the instrument, the encumbrance of the land, 26 nor does it deprive petitioner Therefore, it cannot be said that it did not have a duty to
purchaser cannot have the enjoyment and material of its control thereof. pay to PNB the amortization on the mortgage.
tenancy of the thing and make use of it himself or through We note, however, that petitioner's deprivation of Also, petitioner insists that its payment of the amortization
another in his name, because such tenancy and ownership and control finally occurred when it failed was a mistake because PNB disapproved its assumption
enjoyment are opposed by the interposition of another will, and/or discontinued paying the amortizations on the of mortgage after it failed to submit the necessary papers
mortgage, causing the lot to be foreclosed and sold at for the approval of such assumption.
250
Sales – Chapter 3-12 Cases
But even if petitioner was a third party in regard to the
mortgage of the land purchased, the payment of the loan
by petitioner was a condition clearly imposed by the
contract of sale. This fact alone disproves petitioner's
insistence that there was a "mistake" in payment. On the
contrary, such payments were necessary to protect its
interest as a "the buyer(s) and new owner(s) of the lot."
The quasi-contract of solutio indebiti is one of the concrete
manifestations of the ancient principle that no one shall
enrich himself unjustly at the expense of another. 31 But
as shown earlier, the payment of the mortgage was an
obligation petitioner assumed under the contract of sale.
There is no unjust enrichment where the transaction, as in
this case, is quid pro quo, value for value.
All told, respondent Court did not commit any reversible
error which would warrant the reversal of the assailed
Decision.
WHEREFORE, the petition is hereby DENIED, and the
assailed Decision is AFFIRMED.
SO ORDERED.

251
Sales – Chapter 3-12 Cases
G.R. No. L-42636 August 1, 1985 previously registered under Transfer Certificate of Title On August 18, 1967, petitioners, as plaintiffs, filed a
MARIA LUISA DE LEON ESCALER and ERNESTO No. 42999 issued in the name of A. Doronilla Motion for Summary Judgment, alleging the facts already
ESCALER, CECILIA J. ROXAS and PEDRO Development, Inc. Petitioners as vendees filed their averred in the complaint, and further alleging that the
ROXAS, petitioners, opposition to the said petition. defendants were summoned and were given their day in
vs. On June 10, 1964, an Order 2 was issued in the said case, court at the instance of plaintiffs in Case No. 4252. In
COURT OF APPEALS, JOSE L. REYNOSO, now the dispositive portion of which reads: support of their said motion, the plaintiffs attached the
deceased, to be substituted by his heirs or legal IN VIEW OF THE ABOVE CONSIDERATIONS, this Court affidavit of Atty. Alberto R. Avanceña who had
representatives and AFRICA V. is constrained to set aside Decree No. 62373 issued in represented the plaintiffs in Case No. 4252 and had filed a
REYNOSO, respondents. LRC. Rec. No. N-13783 and the Register of Deeds of joint opposition in behalf of all the vendees. The pertinent
Avanceña Law Office for petitioners. Rizal is directed to cancel OCT No. 1526 of his office and portion of that affidavit, states—
Bauza, Ampil, Suarez, and Paredes Law Office for all Transfer Certificates of Title issued subsequently 4. That he has furnished a copy of said joint opposition to
respondent Africa V. Reynoso. thereafter to purchaser of said property or portions Africa Reynoso, wife of Jose L. Reynoso, at her given
thereof, the same being null and void, the expenses for address at c/o Antipolo Enterprises, Antipolo, Rizal and
CUEVAS, J.: such cancellation to be charged to spouses Angelina the latter had received the same, as evidenced by the
This is a Petition for Review on certiorari of the Decision Reynoso and Floro Reynoso. The owner's duplicates in photostatic copy of the Registry Return Receipt thereto
of the then Court of Appeals (now the Intermediate the possession of the transferees of the property covered affixed as Annex "C-l";
Appellate Court) and of its Resolution denying petitioners' by OCT No. 1526 are declared null and void and said xxx xxx xxx
Motion for Reconsideration, in CA G.R. No. 41953-R, transferees are directed to surrender to the Register of 6. That he hereby executed this Affidavit to prove that said
which was an appeal from the judgment of the Court of Deeds of Rizal, said owner's duplicates for cancellation. defendants Africa Reynoso and Jose L. Reynoso were
First Instance of Rizal in Civil Case No. 9014 entitled The other reliefs sought for by the party oppositors are given their day in Court and/or were afforded their
"Maria Luisa de Leon Escaler, et al vs. Jose L. Reynoso denied the same not falling within the jurisdiction of this opportunity to be heard in Case No. 4252 aforecited.
and Africa Reynoso." Court under this proceeding. On September 27, 1967, judgment was rendered by the
The following are the pertinent background facts: SO ORDERED. trial court, the pertinent portion of which reads—
On March 7, 1958, the spouses Africa V. Reynoso and On August 31, 1965, herein petitioners, spouses Maria de Considering the foregoing motion for summary judgment
Jose L, Reynoso sold to petitioners several others, a Leon Escaler and Ernesto Escaler and spouses Cecilia J. and it appearing that the defendants under a Deed of
parcel of land, situated in Antipolo, Rizal with an area of Roxas and Pedro Roxas, filed Civil Case No. 9014 before Absolute Sale (Annex "C") have expressly warranted their
239,479 square meters and covered by TCT No. 57400 of the Court of First Instance of Rizal against their vendors, valid title and ownership of the said parcel of land and
the Register of Deeds of the Province of Rizal. The Deed herein private respondents, spouses Jose L. Reynoso and further warranted to defend said property from any and all
of Sale 1 contained the following covenant against Africa Reynoso for the recovery of the value of the claims of any persons whomever in favor of plaintiffs; that
eviction, to wit: property sold to them plus damages on the ground that the said warranties were violated when on June 10, 1964,
That the VENDOR is the absolute owner of a parcel of the latter have violated the vendors' "warranty against an Order was promulgated by the Court of First Instance
land ... the ownership thereof being evidenced by an eviction." of Rizal in Case No. 4252 (Related to LRC Case No.
absolute deed of sale executed in her favor by registered The complaint among others, alleged that the Order 1559, LRC Record No. N13293). In Re: Petition for
owner ANGELINA C. REYNOSO, ...; issued in Case No. 4252 which cancelled the title of Cancellation of Original Registration, etc., covering the
That the VENDOR warrants valid title to and ownership of Angelina C. Reynoso and all subsequent Transfer parcel of land in question; that said order of June 10, 1964
said parcel of land and further, warrant to defend the Certificates of Title derived and/or emanating therefrom has become final and executory there being no appeal
property herein sold and conveyed, unto the VENDEES, and which includes the titles of petitioners, is now final, interposed thereto and defendants were summoned and
their heirs, and assignees, from any and all claims of any and by reason thereof petitioners lost their right over the were given a day in court at the instance of the plaintiffs in
persons whatsoever. property sold; and that in said Case No. 4252, the Case No. 4252, the Court hereby grants the motion for
On April 21, 1961, the Register of Deeds of Rizal and A. respondents were summoned and/or given their day in summary judgment, and hereby orders the defendants to
Doronilla Resources Development, Inc. filed Case No. court at the instance of the petitioners. 3 jointly and severally return to the plaintiffs Maria Luisa de
4252 before the Court of First Instance of Rizal for the The respondents, as defendants, filed their answer Leon Escaler and Ernesto Escaler, Cecilia J. Roxas and
cancellation of OCT No. 1526 issued in the name of alleging, among others, by way of affirmative defenses Pedro Roxas, the value of the property sold to them at the
Angelina C. Reynoso (predecessor-in-interest of private that "the cause of action, if any, of plaintiffs against time of eviction which is not to be less than P5,500.00 to
respondents-vendors) on February 26, 1958 under defendants have been fully adjudicated in Case No. 4252 reimburse to each one of the plaintiffs the expenses of
Decree No. 62373, LRC Record No. N-13783, on the when plaintiffs failed to file a third-party complaint against contract and litigation and the amount of P2,250.00 to pay
ground that the property covered by said title is already defendants." 4 the attorney's fees of P1,000.00 plus the costs of suit.
252
Sales – Chapter 3-12 Cases
SO ORDERED. In the case at bar, the fourth requisite—that of being
Private respondents appealed the aforesaid decision to summoned in the suit for eviction (Case No. 4252) at the
the then Court of Appeals 5 assigning as sole error—that instance of the vendee—is not present. All that the
the lower court erred in finding that they were summoned petitioners did, per their very admission, was to furnish
and were given their day in court at the instance of respondents, by registered mail, with a copy of the
petitioners-plaintiffs in Case No. 4252. opposition they (petitioners filed in the eviction suit.
In reversing the decision of the trial court and dismissing Decidedly, this is not the kind of notice prescribed by the
the case, the then Court of Appeals found and so ruled aforequoted Articles 1558 and 1559 of the New Civil
that petitioners as vendees had not given private Code. The term "unless he is summoned in the suit for
respondents-vendors, formal notice of the eviction case as eviction at the instance of the vendee" means that the
mandated by Arts. 1558 and 1559 of the New Civil Code. respondents as vendor/s should be made parties to the
Hence, the instant recourse, petitioners contending — suit at the instance of petitioners-vendees, either by way
1) that the Court of Appeals erred in applying strictly to the of asking that the former be made a co-defendant or by
instant case the provisions of Articles 1558 and 1559 of the filing of a third-party complaint against said vendors.
the new Civil Code; and Nothing of that sort appeared to have been done by the
2) that the decision of the Court of First Instance of Rizal petitioners in the instant case.
should have been affirmed by the Court of Appeals or at IN VIEW OF THE FOREGOING CONSIDERATIONS, the
least, the, Court of Appeals should have remanded the petition is DISMISSED and the appealed decision of the
case to the trial court, for hearing on the merits. then Court of Appeals is AFFIRMED.
The petition is devoid of merit. Consequently, it must be No pronouncement as to costs.
dismissed. SO ORDERED.
Article 1548, in relation to Articles 1558. and 1559 of the Concepcion, Jr., Abad Santos, Plana, Escolin, Relova,
New Civil Code reads as follows: Gutierrez, Jr. and De la Fuente, JJ., concur
Art. 1548, Eviction shall take place whenever by a final
judgment based on a right prior to the sale or an act
imputable to the vendor, the vendee is deprived of the
whole or of a part of the thing purchased.
The vendor shall answer for the eviction even though
nothing has been said in the contract on the subject.
The contracting parties, however, may increase, diminish,
or suppress this legal obligation of the vendor.
Art. 1558. The vendor shall not be obliged to make good
the proper warranty, unless he is summoned in the suit for
eviction at the instance of the vendee. (emphasis
supplied)
Art. 1559. The defendant vendee shall ask, within the time
fixed in the Rules of Court for answering the complaint
that the vendor be made as co-defendant.
In order that a vendor's liability for eviction may be
enforced, the following requisites must concur—a) there
must be a final judgment; b) the purchaser has been
deprived of the whole or part of the thing sold; c) said
deprivation was by virtue of a right prior to the sale made
by the vendor; and d) the vendor has been summoned
and made co-defendant in the suit for eviction at the
instance of the vendee. 6

253
Sales – Chapter 3-12 Cases
G.R. No. 152219 October 25, 2004 22054 July 27, 1993 45,246.00 consequently, dishonored because respondent Maura
NUTRIMIX FEEDS CORPORATION, petitioner, Evangelista had already closed her account. The
vs. 22186 August 2, 1993 84,900.00 petitioner made several demands for the respondents to
COURT OF APPEALS and SPOUSES EFREN AND settle their unpaid obligation, but the latter failed and
MAURA EVANGELISTA, respondents. refused to pay their remaining balance with the petitioner.
DECISION Total: ₱275,631.00 On December 15, 1993, the petitioner filed with the
CALLEJO, SR., J.: =========== Regional Trial Court of Malolos, Bulacan, a complaint,
For review on certiorari is the Decision1 of the Court of respondents incurred an aggregate unsettled account with docketed as Civil Case No. 1026-M-93, against the
Appeals in CA-G.R. CV No. 59615 modifying, on appeal, the petitioner in the amount of ₱766,151.00. The respondents for sum of money and damages with a prayer
the Joint Decision2 of the Regional Trial Court of Malolos, breakdown of the unpaid obligation is as follows: for issuance of writ of preliminary attachment. In their
Bulacan, Branch 9, in Civil Case No. 1026-M-933 for sum answer with counterclaim, the respondents admitted their
of money and damages with prayer for issuance of writ of Check unpaid obligation but impugned their liability to the
Bank Due Date Amount
preliminary attachment, and Civil Case No. 49-M-944 for Number petitioner. They asserted that the nine checks issued by
damages. The trial court dismissed the complaint of the United respondent Maura Evangelista were made to guarantee
respondents, ordering them to pay the petitioner the Coconu the payment of the purchases, which was previously
unpaid value of the assorted animal feeds delivered to the BTS05208 July 30, determined to be procured from the expected proceeds in
t ₱ 47,760.00
former by the latter, with legal interest thereon from the 4 1993 the sale of their broilers and hogs. They contended that
Planter
filing of the complaint, including attorney’s fees. s Bank inasmuch as the sudden and massive death of their
The Factual Antecedents animals was caused by the contaminated products of the
On April 5, 1993, the Spouses Efren and Maura BTS05208 July 30, petitioner, the nonpayment of their obligation was based
Evangelista, the respondents herein, started to directly -do- 131,340.00 on a just and legal ground.
7 1993
procure various kinds of animal feeds from petitioner On January 19, 1994, the respondents also lodged a
Nutrimix Feeds Corporation. The petitioner gave the BTS05209 July 30, complaint for damages against the petitioner, docketed as
-do- 59,700.00
respondents a credit period of thirty to forty-five days to 1 1993 Civil Case No. 49-M-94, for the untimely and unforeseen
postdate checks to be issued in payment for the delivery death of their animals supposedly effected by the
of the feeds. The accommodation was made apparently BTS06272 August 4, adulterated animal feeds the petitioner sold to them.
-do- 47,860.00
because of the company president’s close friendship with 1 1993 Within the period to file an answer, the petitioner moved to
Eugenio Evangelista, the brother of respondent Efren BTS06272 August 5, dismiss the respondents’ complaint on the ground of litis
Evangelista. The various animal feeds were paid and -do- 43,780.00 pendentia. The trial court denied the same in a
0 1993
covered by checks with due dates from July 1993 to Resolution5 dated April 26, 1994, and ordered the
September 1993. Initially, the respondents were good BTS06277 August 6, consolidation of the case with Civil Case No. 1026-M-93.
-do- 15,000.00
paying customers. In some instances, however, they 4 1993 On May 13, 1994, the petitioner filed its Answer with
failed to issue checks despite the deliveries of animal Counterclaim, alleging that the death of the respondents’
feeds which were appropriately covered by sales invoices. BTS06274 Septembe animals was due to the widespread pestilence in their
-do- 47,180.00
Consequently, the 8 r 11, 1993 farm. The petitioner, likewise, maintained that it received
Sales Invoice information that the respondents were in an unstable
BTS06276 Septembe
Date Amount -do- 48,440.00 financial condition and even sold their animals to settle
Number 3 r 11, 1993
their obligations from other enraged and insistent
21334 June 23, 1993 ₱ 7,260.00 BTS06276 Septembe creditors. It, moreover, theorized that it was the
-do- 49,460.00 respondents who mixed poison to its feeds to make it
6 r 18, 1993
21420 June 26, 1993 6,990.00 appear that the feeds were contaminated.
A joint trial thereafter ensued.
21437 June 28, 1993 41,510.00 ₱490,520.00 During the hearing, the petitioner presented Rufino
Total:
21722 July 12, 1993 45,185.00 ========= Arenas, Nutrimix Assistant Manager, as its lone witness.
= He testified that on the first week of August 1993, Nutrimix
22048 July 26, 1993 44,540.00 When the above-mentioned checks were deposited at the President Efren Bartolome met the respondents to discuss
petitioner’s depository bank, the same were, the possible settlement of their unpaid account. The said
254
Sales – Chapter 3-12 Cases
respondents still pleaded to the petitioner to continue to improbable incident to the animal feeds supplied by the had no way of ascertaining whether the feeds were
supply them with animal feeds because their livestock petitioner, and asked Mr. Bartolome for indemnity for the indeed manufactured by the petitioner.
were supposedly suffering from a disease.6 massive death of her livestock. Mr. Bartolome disavowed Another witness for the respondents, Aida Viloria
For her part, respondent Maura Evangelista testified that liability thereon and, thereafter, filed a case against the Magsipoc, Forensic Chemist III of the Forensic Chemist
as direct buyers of animal feeds from the petitioner, Mr. respondents.12 Division of the National Bureau of Investigation, affirmed
Bartolome, the company president, gave them a discount After the meeting with Mr. Bartolome, respondent Maura that she performed a chemical analysis17 of the animal
of ₱12.00 per bag and a credit term of forty-five to Evangelista requested Dr. Rolando Sanchez, a feeds, submitted to her by respondent Maura Evangelista
seventy-five days.7 For the operation of the respondents’ veterinarian, to conduct an inspection in the respondents’ and Dr. Garcia in a sealed plastic bag, to determine the
poultry and piggery farm, the assorted animal feeds sold poultry. On October 20, 1993, the respondents took ample presence of poison in the said specimen. The witness
by the petitioner were delivered in their residence and amounts remaining from the feeds sold by the petitioner verified that the sample feeds yielded positive results to
stored in an adjacent bodega made of concrete wall and and furnished the same to various government agencies the tests for COUMATETRALYL Compound,18 the active
galvanized iron sheet roofing with monolithic flooring.8 for laboratory examination. component of RACUMIN, a brand name for a
It appears that in the morning of July 26, 1993, three Dr. Juliana G. Garcia, a doctor of veterinary medicine and commercially known rat poison.19 According to the
various kinds of animal feeds, numbering 130 bags, were the Supervising Agriculturist of the Bureau of Animal witness, the presence of the compound in the chicken
delivered to the residence of the respondents in Sta. Industry, testified that on October 20, 1993, sample feeds feeds would be fatal to internal organs of the chickens, as
Rosa, Marilao, Bulacan. The deliveries came at about for chickens contained in a pail were presented to her for it would give a delayed blood clotting effect and eventually
10:00 a.m. and were fed to the animals at approximately examination by respondent Efren Evangelista and a lead to internal hemorrhage, culminating in their inevitable
1:30 p.m. at the respondents’ farm in Balasing, Sta. Maria, certain veterinarian.13 The Clinical Laboratory Report death.
Bulacan. At about 8:30 p.m., respondent Maura revealed that the feeds were negative of salmonella 14 and Paz Austria, the Chief of the Pesticide Analytical Section
Evangelista received a radio message from a worker in that the very high aflatoxin level15 found therein would not of the Bureau of Plants Industry, conducted a laboratory
her farm, warning her that the chickens were dying at cause instantaneous death if taken orally by birds. examination to determine the presence of pesticide
rapid intervals. When the respondents arrived at their Dr. Rodrigo Diaz, the veterinarian who accompanied Efren residue in the animal feeds submitted by respondent
farm, they witnessed the death of 18,000 broilers, at the Bureau of Animal Industry, testified that sometime Maura Evangelista and Dr. Garcia. The tests disclosed
averaging 1.7 kilos in weight, approximately forty-one to in October 1993, Efren sought for his advice regarding the that no pesticide residue was detected in the samples
forty-five days old. The broilers then had a prevailing death of the respondents’ chickens. He suggested that the received20but it was discovered that the animal feeds
market price of ₱46.00 per kilo.9 remaining feeds from their warehouse be brought to a were positive for Warfarin, a rodenticide (anticoagulant),
On July 27, 1993, the respondents received another laboratory for examination. The witness claimed that the which is the chemical family of Coumarin.21
delivery of 160 bags of animal feeds from the petitioner, feeds brought to the laboratory came from one bag of After due consideration of the evidence presented, the
some of which were distributed to the contract growers of sealed Nutrimix feeds which was covered with a sack. trial court ruled in favor of the petitioner. The dispositive
the respondents. At that time, respondent Maura Dr. Florencio Isagani S. Medina III, Chief Scientist portion of the decision reads:
Evangelista requested the representative of the petitioner Research Specialist of the Philippine Nuclear Research WHEREFORE, in light of the evidence on record and the
to notify Mr. Bartolome of the fact that their broilers died Institute, informed the trial court that respondent Maura laws/jurisprudence applicable thereon, judgment is hereby
after having been fed with the animal feeds delivered by Evangelista and Dr. Garcia brought sample feeds and four rendered:
the petitioner the previous day. She, likewise, asked that a live and healthy chickens to him for laboratory 1) in Civil Case No. 1026-M-93, ordering defendant
technician or veterinarian be sent to oversee the untoward examination. In his Cytogenetic Analysis,16 Dr. Medina spouses Efren and Maura Evangelista to pay unto plaintiff
occurrence. Nevertheless, the various feeds delivered on reported that he divided the chickens into two categories, Nutrimix Feeds Corporation the amount of ₱766,151.00
that day were still fed to the animals. On July 27, 1993, which he separately fed at 6:00 a.m. with the animal feeds representing the unpaid value of assorted animal feeds
the witness recounted that all of the chickens and hogs of a different commercial brand and with the sample feeds delivered by the latter to and received by the former, with
died.10 Efren Evangelista suffered from a heart attack and supposedly supplied by the petitioner. At noon of the legal interest thereon from the filing of the complaint on
was hospitalized as a consequence of the massive death same day, one of the chickens which had been fed with December 15, 1993 until the same shall have been paid in
of their animals in the farm. On August 2, 1993, another the Nutrimix feeds died, and a second chicken died at full, and the amount of ₱50,000.00 as attorney’s fees.
set of animal feeds were delivered to the respondents, but 5:45 p.m. of the same day. Samples of blood and bone Costs against the aforenamed defendants; and
the same were not returned as the latter were not yet marrow were taken for chromosome analysis, which 2) dismissing the complaint as well as counterclaims
cognizant of the fact that the cause of the death of their showed pulverized chromosomes both from bone marrow in Civil Case No. 49-M-94 for inadequacy of evidence to
animals was the polluted feeds of the petitioner.11 and blood chromosomes. On cross-examination, the sustain the same. No pronouncement as to costs.
When respondent Maura Evangelista eventually met with witness admitted that the feeds brought to him were SO ORDERED.22
Mr. Bartolome on an undisclosed date, she attributed the merely placed in a small unmarked plastic bag and that he
255
Sales – Chapter 3-12 Cases
In finding for the petitioner, the trial court ratiocinated as In dismissing the complaint in Civil Case No. 1026-M-93, The provisions on warranty against hidden defects are
follows: the CA ruled that the respondents were not obligated to found in Articles 1561 and 1566 of the New Civil Code of
On the strength of the foregoing disquisition, the Court pay their outstanding obligation to the petitioner in view of the Philippines, which read as follows:
cannot sustain the Evangelistas’ contention that Nutrimix its breach of warranty against hidden defects. The CA Art. 1561. The vendor shall be responsible for warranty
is liable under Articles 1561 and 1566 of the Civil Code gave much credence to the testimony of Dr. Rodrigo Diaz, against hidden defects which the thing sold may have,
governing "hidden defects" of commodities sold. As who attested that the sample feeds distributed to the should they render it unfit for the use for which it is
already explained, the Court is predisposed to believe that various governmental agencies for laboratory examination intended, or should they diminish its fitness for such use
the subject feeds were contaminated sometime between were taken from a sealed sack bearing the brand name to such an extent that, had the vendee been aware
their storage at the bodega of the Evangelistas and their Nutrimix. The CA further argued that the declarations of thereof, he would not have acquired it or would have
consumption by the poultry and hogs fed therewith, and Dr. Diaz were not effectively impugned during cross- given a lower price for it; but said vendor shall not be
that the contamination was perpetrated by unidentified or examination, nor was there any contrary evidence answerable for patent defects or those which may be
unidentifiable ill-meaning mischief-maker(s) over whom adduced to destroy his damning allegations. visible, or for those which are not visible if the vendee is
Nutrimix had no control in whichever way. On March 7, 2002, the petitioner filed with this Court the an expert who, by reason of his trade or profession,
All told, the Court finds and so holds that for inadequacy instant petition for review on the sole ground that – should have known them.
of proof to the contrary, Nutrimix was not responsible at all THE HONORABLE COURT OF APPEALS ERRED IN Art. 1566. The vendor is responsible to the vendee for any
for the contamination or poisoning of the feeds supplied CONCLUDING THAT THE CLAIMS OF HEREIN hidden faults or defects in the thing sold, even though he
by it to the Evangelistas which precipitated the mass PETITIONER FOR COLLECTION OF SUM OF MONEY was not aware thereof.
death of the latter’s chickens and hogs. By no means and AGAINST PRIVATE RESPONDENTS MUST BE DENIED This provision shall not apply if the contrary has been
under no circumstance, therefore, may Nutrimix be held BECAUSE OF HIDDEN DEFECTS. stipulated, and the vendor was not aware of the hidden
liable for the sundry damages prayed for by the The Present Petition faults or defects in the thing sold.
Evangelistas in their complaint in Civil Case No. 49-M-94 The petitioner resolutely avers that the testimony of Dr. A hidden defect is one which is unknown or could not
and answer in Civil Case No. 1026-M-93. In fine, Civil Diaz can hardly be considered as conclusive evidence of have been known to the vendee.26 Under the law, the
Case No. 49-M-94 deserves dismissal. hidden defects that can be attributed to the petitioner. requisites to recover on account of hidden defects are as
Parenthetically, vis-à-vis the fulminations of the Parenthetically, the petitioner asserts, assuming that the follows:
Evangelistas in this specific regard, the Court does not sample feeds were taken from a sealed sack bearing the (a) the defect must be hidden;
perceive any act or omission on the part of Nutrimix brand name Nutrimix, it cannot decisively be presumed (b) the defect must exist at the time the sale was made;
constitutive of "abuse of rights" as would render said that these were the same feeds brought to the (c) the defect must ordinarily have been excluded from the
corporation liable for damages under Arts. 19 and 21 of respondents’ farm and given to their chickens and hogs contract;
the Civil Code. The alleged "callous attitude and lack of for consumption. (d) the defect, must be important (renders thing UNFIT or
concern of Nutrimix" have not been established with more It is the contention of the respondents that the appellate considerably decreases FITNESS);
definitiveness. court correctly ordered the dismissal of the complaint in (e) the action must be instituted within the statute of
As regards Civil Case No. 1026-M-93, on the other hand, Civil Case No. 1026-M-93. They further add that there limitations.27
the Court is perfectly convinced that the deliveries of was sufficient basis for the CA to hold the petitioner guilty In the sale of animal feeds, there is an implied warranty
animal feeds by Nutrimix to the Evangelistas constituted a of breach of warranty thereby releasing the respondents that it is reasonably fit and suitable to be used for the
simple contract of sale, albeit on a continuing basis and from paying their outstanding obligation. purpose which both parties contemplated.28 To be able to
on terms or installment payments.23 The Ruling of the Court prove liability on the basis of breach of implied warranty,
Undaunted, the respondents sought a review of the trial Oft repeated is the rule that the Supreme Court reviews three things must be established by the respondents. The
court’s decision to the Court of Appeals (CA), principally only errors of law in petitions for review on certiorari under first is that they sustained injury because of the product;
arguing that the trial court erred in holding that they failed Rule 45. However, this rule is not absolute. The Court the second is that the injury occurred because the product
to prove that their broilers and hogs died as a result of may review the factual findings of the CA should they be was defective or unreasonably unsafe; and finally, the
consuming the petitioner’s feeds. contrary to those of the trial court. Conformably, this Court defect existed when the product left the hands of the
On February 12, 2002, the CA modified the decision of the may review findings of facts when the judgment of the CA petitioner.29 A manufacturer or seller of a product cannot
trial court. The fallo of the decision reads: is premised on a misapprehension of facts.25 be held liable for any damage allegedly caused by the
WHEREFORE, premises considered, the appealed The threshold issue is whether or not there is sufficient product in the absence of any proof that the product in
decision is hereby MODIFIED such that the complaint in evidence to hold the petitioner guilty of breach of warranty question was defective.30 The defect must be present
Civil Case No. 1026-M-93 is dismissed for lack of merit. due to hidden defects. upon the delivery or manufacture of the product;31 or
So ordered.24 The petition is meritorious. when the product left the seller’s or manufacturer’s
256
Sales – Chapter 3-12 Cases
control;32 or when the product was sold to the And when the chickens died, they stopped naturally It bears stressing, too, that the chickens brought to the
purchaser;33 or the product must have reached the user or feeding it to the chickens. Philippine Nuclear Research Institute for laboratory tests
consumer without substantial change in the condition it Atty. Cruz: were healthy animals, and were not the ones that were
was sold. Tracing the defect to the petitioner requires Q You mean to say, Madam Witness, that although you ostensibly poisoned. There was even no attempt to have
some evidence that there was no tampering with, or believe (sic) that the chickens were allegedly poisoned, the dead fowls examined. Neither was there any analysis
changing of the animal feeds. The nature of the animal you used the same for feeding your animals? of the stomach of the dead chickens to determine whether
feeds makes it necessarily difficult for the respondents to A We did not know yet during that time that the feeds the petitioner’s feeds really caused their sudden death.
prove that the defect was existing when the product left contained poison, only during that time when we learned Mere sickness and death of the chickens is not
the premises of the petitioner. about the same after the analysis. satisfactory evidence in itself to establish a prima facie
A review of the facts of the case would reveal that the Q Therefore you have known only of the alleged poison in case of breach of warranty.36
petitioner delivered the animal feeds, allegedly containing the Nutrimix Feeds only after you have caused the Likewise, there was evidence tending to show that the
rat poison, on July 26, 1993; but it is astonishing that the analysis of the same? respondents combined different kinds of animal feeds and
respondents had the animal feeds examined only on A Yes, Sir. that the mixture was given to the animals. Respondent
October 20, 1993, or barely three months after their Q When was that, Madam Witness? Maura Evangelista testified that it was common practice
broilers and hogs had died. On cross-examination, A I cannot be sure about the exact time but it is within the among chicken and hog raisers to mix animal feeds. The
respondent Maura Evangelista testified in this manner: months of October to November, Sir. testimonies of respondent Maura Evangelista may be thus
Atty. Cruz: Q So, before this analysis of about October and summarized:
Q Madam Witness, you said in the last hearing that November, you were not aware that the feeds of Nutrimix Cross-Examination
believing that the 250 bags of feeds delivered to (sic) the Feeds Corporation were, according to you, with poison? Atty. Cruz:
Nutrimix Feeds Corporation on August 2, 1993 were A We did not know yet that it contained poison but we Q Because, Madam Witness, you ordered chicken booster
poison (sic), allegedly your husband Efren Evangelista were sure that the feeds were the cause of the death of mash from Nutrimix Feeds Corporation because in July
burned the same with the chicken[s], is that right? our animals.34 1993 you were taking care of many chickens, as a matter
A Yes, Sir. Some, Sir. We find it difficult to believe that the feeds delivered on of fact, majority of the chickens you were taking care [of]
Q And is it not a fact, Madam Witness, that you did not, as July 26 and 27, 1993 and fed to the broilers and hogs were chicks and not chickens which are marketable?
according to you, used (sic) any of these deliveries made contained poison at the time they reached the A What I can remember was that I ordered chicken
on August 2, 1993? respondents. A difference of approximately three months booster mash on that month of July 1993 because we
A We were able to feed (sic) some of those deliveries enfeebles the respondents’ theory that the petitioner is have some chicks which have to be fed with chicken
because we did not know yet during that time that it is the guilty of breach of warranty by virtue of hidden defects. In booster mash and I now remember that on the particular
cause of the death of our chicks (sic), Sir. a span of three months, the feeds could have already month of July 1993 we ordered several bags of chicken
Q But according to you, the previous deliveries were not been contaminated by outside factors and subjected to booster mash for the consumption also of our chicken in
used by you because you believe (sic) that they were many conditions unquestionably beyond the control of the our other poultry and at the same time they were also
poison (sic)? petitioner. In fact, Dr. Garcia, one of the witnesses for the used to be mixed with the feeds that were given to the
A Which previous deliveries, Sir[?] respondents, testified that the animal feeds submitted to hogs.
Q Those delivered on July 26 and 22 (sic), 1993? her for laboratory examination contained very high level of Q You mean to say [that], as a practice, you are mixing
A Those were fed to the chickens, Sir. This is the cause of aflatoxin, possibly caused by mold (aspergillus chicken booster mash which is specifically made for chick
the death of the chickens. flavus).35 We agree with the contention of the petitioner feeds you are feeding the same to the hogs, is that what
Q And you stated that this last delivery on August 2 were that there is no evidence on record to prove that the you want the Court to believe?
poison (sic) also and you did not use them, is that right? animal feeds taken to the various governmental agencies A Yes, Sir, because when you mix chicken booster mash
Atty. Roxas: for laboratory examination were the same animal feeds in the feeds of hogs there is a better result, Sir, in raising
That is misleading. given to the respondents’ broilers and hogs for their hogs.37
Atty. Cruz: consumption. Moreover, Dr. Diaz even admitted that the …
She stated that. feeds that were submitted for analysis came from a sealed Re-Direct Examination
Atty. Roxas: bag. There is simply no evidence to show that the feeds Atty. Roxas:
She said some were fed because they did not know yet of given to the animals on July 26 and 27, 1993 were Q Now, you mentioned that shortly before July 26 and 27,
the poisoning. identical to those submitted to the expert witnesses in 1993, various types of Nutrimix feeds were delivered to
Court: October 1993. you like chicks booster mash, broiler starter mash and hog

257
Sales – Chapter 3-12 Cases
finisher or hog grower mash. What is the reason for evidence, or if evidence be so slight as not reasonably to In the second week of December 1988, Filemon Flores,
simultaneous deliveries of various types of feeds? warrant inference of the fact in issue or furnish more than respondent, purchased from Supercars Management and
A Because we used to mix all those together in one materials for a mere conjecture, the court will not hesitate Development Corporation, petitioner, an Isuzu Carter
feeding, Sir. to strike down the evidence and rule in favor of the other Crew Cab for P212,000.00 payable monthly with a down
Q And what is the reason for mixing the chick booster party.41 This rule is both fair and sound. Any other payment equivalent to 30% of the price or P63,600.00.
mash with broiler starter mash? interpretation of the law would unloose the courts to The balance was to be financed by the Rizal Commercial
A So that the chickens will get fat, Sir. meander aimlessly in the arena of speculation.42 Banking Corporation (RCBC). The sale was coursed
… It must be stressed, however, that the remedy against through Pablito Marquez, petitioner's salesman.
Re-Cross Examination violations of warranty against hidden defects is either to Upon delivery of the vehicle on December 27, 1988,
Atty. Cruz: withdraw from the contract (accion redhibitoria) or to respondent paid petitioner the 30% down payment, plus
Q Madam Witness, is it not a fact that the mixing of these demand a proportionate reduction of the price (accion premium for the vehicle's comprehensive insurance policy
feeds by you is your own concuction (sic) and without the quanti minoris), with damages in either case.43 In any amounting to P7,374.80. The RCBC financed the balance
advice of a veterinarian expert to do so? case, the respondents have already admitted, both in their of the purchase price. Its payment was secured by a
A That is common practice among raisers to mix two testimonies and pleadings submitted, that they are indeed chattel mortgage of the same vehicle.
feeds, Sir. indebted to the petitioner for the unpaid animal feeds A day after the vehicle was delivered, respondent used it
Q By yourself, Madam Witness, who advised you to do delivered to them. For this reason alone, they should be for his family's trip to Bauang, La Union. While traversing
the mixing of these two types of feeds for feeding your held liable for their unsettled obligations to the petitioner. the national highway in Tarlac, Tarlac, the fan belt of the
chickens? WHEREFORE, in light of all the foregoing, the petition vehicle snapped. Then its brakes hardened after several
A That is common practice of chicken raisers, Sir.38 is GRANTED. The assailed Decision of the Court of stops and did not function properly; the heater plug did not
Even more surprising is the fact that during the meeting Appeals, dated February 12, 2002, also function; the engine could not start; and the fuel
with Nutrimix President Mr. Bartolome, the respondents is REVERSED and SET ASIDE. The Decision of the consumption increased.4
claimed that their animals were plagued by disease, and Regional Trial Court of Malolos, Bulacan, Branch 9, dated Upon their return to Manila in the first week of January
that they needed more time to settle their obligations with January 12, 1998, is REINSTATED. No costs. 1989, respondent complained to petitioner about the
the petitioner. It was only after a few months that the SO ORDERED. defects of the vehicle. Marquez then had the vehicle
respondents changed their justification for not paying their repaired and returned it to respondent that same day,
unsettled accounts, claiming anew that their animals were assuring the latter that it was already in good condition.
poisoned with the animal feeds supplied by the petitioner. But after driving the vehicle for a few days, the same
The volte-face of the respondents deserves scant G.R. No. 148173 December 10, 2004 defects resurfaced, prompting respondent to send
consideration for having been conjured as a mere SUPERCARS MANAGEMENT & DEVELOPMENT petitioner a letter dated January 30, 1989 rescinding the
afterthought. CORPORATION, represented by its President Benigno contract of sale and returning the vehicle due to breach of
In essence, we hold that the respondents failed to prove Chan, petitioner, warranty against hidden defects. A copy of the letter was
that the petitioner is guilty of breach of warranty due to vs. furnished RCBC.
hidden defects. It is, likewise, rudimentary that common THE LATE FILEMON FLORES, substituted by his In response to respondent's letter, petitioner directed
law places upon the buyer of the product the burden of surviving spouse, NORA C. FLORES,1 respondent. Marquez to have the vehicle fixed. Thereafter, he returned
proving that the seller of the product breached its the vehicle to respondent with the assurance that it has no
warranty.39 The bevy of expert evidence adduced by the more defects. However, when respondent drove it for a
respondents is too shaky and utterly insufficient to prove DECISION few days, he found that the vehicle was still defective.
that the Nutrimix feeds caused the death of their animals. Hence, on February 7, 1989, respondent sent petitioner
For these reasons, the expert testimonies lack probative another letter restating that he is rescinding the contract of
weight. The respondents’ case of breach of implied SANDOVAL-GUTIERREZ, J.: sale, a copy of which was furnished RCBC. On February
warranty was fundamentally based upon the Before us is a petition for review on certiorari assailing the 9, 1989, he returned the vehicle to petitioner. Later,
circumstantial evidence that the chickens and hogs Decision2 dated November 29, 2000 and Resolution3dated Marquez and Mamerto Catley, petitioner's salesman, tried
sickened, stunted, and died after eating Nutrimix feeds; April 26, 2001, both issued by the Court of Appeals in CA- to convince respondent to accept the vehicle as it had
but this was not enough to raise a reasonable supposition G.R. CV No. 40419, entitled "Filemon Flores vs. been completely repaired. But respondent refused.
that the unwholesome feeds were the proximate cause of Supercars Management & Development Corporation, On March 1, 1989, respondent sent petitioner a letter
the death with that degree of certainty and probability Mamerto Catley, Pablito Marquez, and Rizal Commercial demanding the refund of his down payment, plus the
required.40The rule is well-settled that if there be no Banking Corporation." premium he paid for the vehicle's insurance.
258
Sales – Chapter 3-12 Cases
Petitioner failed to comply with petitioner's demand. motor vehicle insurance plus interests at the rate of 14% The dispositive portion of the same Decision insofar as
Consequently, respondent stopped paying the monthly per annum from date of filing of this complaint on October the rest of the defendants are concerned is hereby
amortization for the vehicle. 30, 1989 until fully paid; maintained and affirmed in toto.
On March 21, 1989, RCBC sent respondent a letter 2. the sum of P50,000.00 as moral damages; SO ORDERED."8
demanding that he settle his past overdue accounts for 3. the sum of P25,000.00 as exemplary damages; From the above Decision and Order, petitioner, Marquez
February 15 and March 15, 1989. In reply, respondent, 4. the sum of P20,000.00 as attorney's fees; and and Catley interposed an appeal to the Court of Appeals,
through a letter dated March 31, 1989, informed RCBC 5. the costs of suit. docketed as CA-G.R. CV No. 40419. In a Decision dated
that he had rescinded the contract of sale and had SO ORDERED."7 November 29, 2000, the Appellate Court affirmed the RTC
returned the vehicle to petitioner. This prompted RCBC to Upon motion for reconsideration by RCBC, the RTC, in an Decision with modification in the sense that the complaint
file with the Office of the Clerk of Court and Ex- Order dated December 21, 1992, modified its Decision by against Marquez and Catley was dismissed, thus:
Officio Sheriff, Regional Trial Court, Quezon City, absolving RCBC from any liability and dismissing the xxx
a Petition for Extra-judicial Foreclosure of Chattel complaint against it, thus: "It is with respect to appellants Catley and Marquez'
Mortgage. xxx liability that we are minded to modify the (appealed)
On June 2, 1989, a Notice of Sheriff's Sale of the vehicle "Going into the merits of defendant bank's contention that Decision. The two being mere employees (of appellant
was set. it has valid and meritorious defense which should Supercars Management and Development Corporation),
On June 1, 1989, respondent filed with the same Office ultimately exculpate it from any liability, jointly and they cannot be held liable to refund the amount claimed
a Manifestation/Motion asking for the postponement of the severally, with the other defendants, the Court, after a by Flores. Nor can they be made liable for damages and
scheduled auction sale until such time that petitioner careful review of the evidence on hand, reconsiders its attorney's fees, there being no clear evidence that they
and/or RCBC shall have reimbursed him of the amount he Decision insofar as the said bank is concerned. The valid had a hand in giving rise thereto.
paid for the vehicle; and that should the auction sale be exercise by the plaintiff of its right to rescind the contract WHEREFORE, the appealed Amended Decision is
conducted, the proceeds thereof equivalent to the amount of sale for the purchase of the motor vehicle in question AFFIRMED, with the MODIFICATION that the complaint
he spent be withheld and turned over to him. does not apply to defendant bank. Said contract is insofar as defendants-appellants Mamerto Catley and
The auction sale proceeded as scheduled. RCBC, being effective only as against defendant Supercars Pablito Marquez is hereby DISMISSED.
the highest bidder, purchased the vehicle. Subsequently, Management and Development Corporation, which must SO ORDERED.9
RCBC sold the vehicle to a third party. principally suffer the consequence of its breach of the Petitioner filed a motion for reconsideration but denied in a
On November 3, 1989, respondent filed with the Regional contract. Resolution dated April 26, 2001.10
Trial Court (RTC), Branch 150, Makati City a complaint5for This Court likewise takes notice of the fact that since the Hence, the instant petition.
rescission of contract with damages against petitioner, motor vehicle was voluntarily surrendered by the plaintiff Petitioner contends that respondent has "no right to
Marquez, Catley and RCBC, docketed as Civil Case No. and that defendant bank merely exercised its right under rescind the contract of sale"11 because "the motor vehicle
89-5566. the chattel mortgage law, no fault can be attributed to the in question, as found by the RTC and the Court of
In their separate answers, petitioner, Marquez and Catley latter. The fact that the plaintiff sent a letter to the Office of Appeals, is already in the hands of a third party, one Mr.
denied having committed any breach of warranty against the City Sheriff of Quezon City, copy furnished the bank, Lim – an innocent purchaser for value."12 Thus, both
hidden defects, claiming that the vehicle had only "minor seeking the postponement of the auction sale of the courts erred in ordering petitioner to refund respondent of
and inconsequential defects" which "were promptly and subject motor vehicle, will not and cannot be considered the amounts he paid for the vehicle.
satisfactorily repaired by petitioner Supercars pursuant to as a valid ground to hold said bank liable for only The issue here is whether respondent has the right to
its warranty as the seller."6 For its part, RCBC claimed that exercising its rights under the law. At most, the liability rescind the contract of sale and to claim damages as a
it has no liability whatsoever against respondent because must really be imputed only against defendants Supercars result thereof.
it merely enforced its right under the chattel mortgage law. Management and Development Corporation, Mamerto We rule for respondent.
All the defendants prayed for the dismissal of the Catley and Pablito Marquez. Respondent's complaint filed with the RTC seeks to
complaint. "WHEREFORE, considering the foregoing premises, the recover from petitioner the money he paid for the vehicle
On April 13, 1992, the RTC rendered its Decision in favor Decision of this Court dated April 13, 1992, insofar as it due to the latter's breach of his warranty against hidden
of respondent and against the defendants, thus: holds defendant Rizal Commercial Banking Corporation defects under Articles 1547,13 1561,14 and 156615 of the
"WHEREFORE, judgment is hereby rendered in favor of jointly and severally liable to the plaintiff, is hereby Civil Code. The vehicle, after it was delivered to
the plaintiff and against the defendants, ordering the latter MODIFIED and the case against said bank DISMISSED. respondent, malfunctioned despite repeated repairs by
to jointly and severally pay the plaintiff as follows: Similarly, the compulsory counterclaim against the plaintiff petitioner. Obviously, the vehicle has hidden defects. A
1. the amount of P70,974.80 representing the 30% down is likewise dismissed. hidden defect is one which is unknown or could not have
payment and premium paid for one year comprehensive been known to the vendee.16
259
Sales – Chapter 3-12 Cases
The findings of both the RTC and Court of Appeals that benefits received.18Petitioner is thus mandated by law to are eliminated.21 Moreover, the trial court did not give any
petitioner committed a breach of warranty against hidden give back to respondent the purchase price upon his justification for granting it in its decision. It is now settled
defects are fully supported by the records. The Appellate return of the vehicle. Records show that at the time that awards of attorney's fees must be based on findings
Court correctly ruled: respondent opted to rescind the contract, the vehicle was of fact and law, stated in the decision of the trial court. 22
"The evidence clearly shows that Flores [now respondent] still in his possession. He returned it to petitioner who, WHEREFORE, the petition is DENIED. The assailed
was justified in opting to rescind the sale given the hidden without objection, accepted it. Accordingly, the 30% down Decision dated September 20, 1999 and Resolution dated
defects of the vehicle, allowance for the repair of which he payment equivalent to P63,600.00, plus the premium for February 1, 2000 of the Court of Appeals in CA-G.R. CV
patiently extended, but which repair did not turn out to be the comprehensive insurance amounting to P7,374.80 No. 52177 are AFFIRMED with MODIFICATION. The
satisfactory. paid by respondent should be returned by petitioner. award of moral and exemplary damages and attorney's
xxx As further stated by the Court of Appeals: fees are DELETED. Costs against petitioner.
For when by letters of January 30, 1989 and February 7, "Appellant's invocation of Article 1191 of the Civil Code in SO ORDERED.
1989, which were followed up by another dated March 1, support of his argument that as the vehicle had been sold
1989, Flores declared his rescission of the sale, which to a third party, rescission can no longer ensue is
rescission was not impugned or opposed by appellants as misplaced.
in fact they accepted the return of the vehicle on February For, Flores is asking for the refund of the downpayment
9, 1989, such extra-judicial rescission x x x produced legal and payment for insurance premiums. This brings us to
effect (UP vs. de los Angeles, 35 SCRA 102 appellant's final argument.
[1970]; Tolentino Commentaries and Jurisprudence on the Appellant's professed excuse from their inability to give
Civil Code, citing Magdalena Estate v. Myrick, 71 Phil. 344 refund – that refund would necessitate the return of the
[1940-1941]). subject motor vehicle which is impossible because it is
x x x"17 now in the hands of an innocent purchaser for value –
It is well within respondent's right to recover damages miserably fails.
from petitioner who committed a breach of warranty x x x appellant Supercars was paid the balance of the
against hidden defects. Article 1599 of the Civil Code purchase price by RCBC and, therefore, in addition to the
partly provides: downpayment given by Flores, it had been fully paid for
"Article 1599. Where there is a breach of warranty by the the vehicle.
seller, the buyer may, at his election: Ergo, Supercars had nothing more to do with the
xxx vehicle."19
(4) Rescind the contract of sale and refuse to receive the However, the lower court's award of P50,000.00 as moral
goods, or if the goods have already been received, return damages and P25,000.00 as exemplary damages to
them or offer to return them to the seller and recover the respondent is erroneous. While no proof of pecuniary loss
price or any part thereof which has been paid. is necessary in order that moral damages may be
When the buyer has claimed and been granted a remedy awarded, the amount of indemnity being left to the
in anyone of these ways, no other remedy can thereafter discretion of the court, it is nevertheless essential that the
be granted, without prejudice to the provisions of the claimant satisfactorily prove the existence of the factual
second paragraph of Article 1191. basis of the damage and its causal relation to defendant's
x x x." (Underscoring supplied) acts. Moral damages are in the category of an award
Petitioner's contention that under Article 1191 of the Civil designed to compensate the claimant for actual
Code, rescission can no longer be availed of as the injurysuffered and not to impose a penalty to the
vehicle was already in the hands of an innocent purchaser wrongdoer. This has not been proved by respondent.
for value lacks merit. Rescission is proper if one of the In contracts and quasi-contracts, the court may award
parties to a contract commits a substantial breach of its exemplary damages if the defendant acted in a wanton,
provisions. It creates an obligation to return the object of fraudulent, reckless, oppressive, or malevolent
the contract. It can be carried out only when the one who manner.20 Likewise, respondent failed to establish that
demands rescission can return whatever he may be petitioner acted in such manner.
obliged to restore. Rescission abrogates the contract from As to the award of attorney's fees, the same must be
its inception and requires a mutual restitution of the deleted since the award of moral and exemplary damages
260
Sales – Chapter 3-12 Cases
G.R. No. 162365 January 15, 2014 execution of the deed of sale based on the purchase price Roberto appealed to the CA.
ROBERTO R. DAVID, Petitioner, agreed upon, plus 12% interest per annum. Ruling of the CA
vs. In April 1997, Roberto and Edwin executed a On October 10, 2003,10 the CA promulgated its decision
EDUARDO C. DAVID, Respondent. memorandum of agreement (MOA)5 with the Spouses affirming the RTC. It opined that although there was no
DECISION Marquez and Soledad Go (Spouses Go), by which they express exercise of the right to repurchase, the sum of all
BERSAMIN, J.: agreed to sell the Baguio City lot to the latter for a the relevant circumstances indicated that there was an
In a sale with right to repurchase, title and ownership of consideration of ₱10,000,000.00. The MOA stipulated that exercise of the right to repurchase pursuant to the deed of
the property sold are immediately vested in the vendee, "in order to save payment of high and multiple taxes sale, that the findings of the RTC to the effect that the
subject to the resolutory condition of repurchase by the considering that the x x x subject matter of this sale is conditions for the exercise of the right to repurchase had
vendor within the stipulated period. mortgaged with DBP, Baguio City, and sold [to Roberto], been adequately satisfied by Eduardo, and that no
The Case Edwin will execute the necessary Deed of Absolute Sale novation as claimed by Roberto had intervened.
Under review at the defendant's instance is the decision in favor of [the Spouses Go], in lieu of [Roberto]." 6 The On February 16, 2004,11 the CA denied Roberto’s motion
promulgated on October 10, 2003,1 whereby the Court of Spouses Go then deposited the amount of for reconsideration.12
Appeals (CA) affirmed the judgment rendered on ₱10,000,000.00 to Roberto’s account.7 Hence, this petition for review on certiorari.
December 5, 2001 by the Regional Trial Court (RTC), After the execution of the MOA, Roberto gave Eduardo Issues
Branch 61, in Baguio City ordering him to return to the ₱2,800,000.00 and returned to him one of the truck Roberto seeks a reversal, claiming that the CA erred:
plaintiff the motor vehicle and trailer subject of the tractors and trailers subject of the deed of sale. Eduardo x x x IN HOLDING THAT THE RESPONDENT HAS
complaint, or to pay their value of ₱500,000.00 should the demanded for the return of the other truck tractor and EXERCISED THEIR RIGHT TO REPURCHASE;
return not be effected, and to pay the plaintiff ₱20,000.00 trailer, but Roberto refused to heed the demand. x x x IN HOLDING THAT THERE WAS NO NOVATION
as litigation expenses, ₱50,000.00 as attorney's fees, and Thus, Eduardo initiated this replevin suit against Roberto, OF THE DEED OF SALE WITH ASSUMPTION OF
the costs of suit.2 alleging that he was exercising the right to repurchase MORTGAGE WHEN THE PARTIES EXECUTED A
Antecedents under the deed of sale; and that he was entitled to the MEMORANDUM OF AGREEMENT FOR THE SALE OF
Respondent Eduardo C. David (Eduardo) initiated this possession of the other motor vehicle and trailer. THE SUBJECT HOUSE AND LOT AND, THEREAFTER
replevin suit against Roberto R. David (Roberto), his first In his answer, Roberto denied that Eduardo could SOLD THE SAID PROPERTY TO THIRD PERSONS;
cousin and former business partner, to recover the repurchase the properties in question; and insisted that x x x IN RESOLVING THE INSTANT CASE IN FAVOR
possession of one unit of International CO 9670 Truck the MOA had extinguished their deed of sale by novation. OF RESPO[N]DENT.13
Tractor and Mi-Bed Trailer. Judgment of the RTC Ruling of the Court
It appears that on July 7, 1995, Eduardo and his brother On December 5, 2001,8 the RTC rendered judgment in The petition for review has no merit.
Edwin C. David (Edwin), acting on their own and in behalf favor of Eduardo, holding that the stipulation giving A sale with right to repurchase is governed by Article 1601
of their co-heirs, sold their inherited properties to Roberto, Eduardo the right to repurchase had made the deed of of the Civil Code, which provides that: "Conventional
specifically: (a) a parcel of land with an area of 1,231 sale a conditional sale; that Eduardo had fulfilled the redemption shall take place when the vendor reserves the
square meters, together with all the improvements existing conditions for the exercise of the right to repurchase; that right to repurchase the thing sold, with the obligation to
thereon, located in Baguio City and covered by Transfer the ownership of the properties in question had reverted to comply with the provisions of Article 1616 and other
Certificate of Title No. T-22983 of the Registry of Deeds of Eduardo; that Roberto’s defense of novation had no merit; stipulations which may have been agreed upon."
Baguio City (Baguio City lot); and (b) two units and that due to Roberto’s bad faith in refusing to satisfy Conformably with Article 1616,14 the seller given the right
International CO 9670 Truck Tractor with two Mi-Bed Eduardo’s claim, Eduardo should be awarded litigation to repurchase may exercise his right of redemption by
Trailers.3 A deed of sale with assumption of mortgage expenses and attorney’s fees. The dispositive portion of paying the buyer: (a) the price of the sale, (b) the
(deed of sale)4 embodied the terms of their agreement, the judgment reads: expenses of the contract, (c) legitimate payments made
stipulating that the consideration for the sale was WHEREFORE, premises considered, judgment is hereby by reason of the sale, and (d) the necessary and useful
₱6,000,000.00, of which ₱2,000,000 was to be paid to rendered for the plaintiff and against the defendant expenses made on the thing sold.
Eduardo and Edwin, and the remaining ₱4,000,000.00 to ORDERING the latter to return to the former the motor The deed of sale entered into by Eduardo and Roberto
be paid to Development Bank of the Philippines (DBP) in vehicle and trailer subject matter of the case or to pay its contained the following stipulation on the right to
Baguio City to settle the outstanding obligation secured by value in the amount of ₱500,000 in case manual delivery repurchase, to wit:
a mortgage on such properties. The parties further agreed can not be effected; to pay plaintiff the amount of ₱20,000 x x x the Vendors are given the right to repurchase the
to give Eduardo and Edwin the right to repurchase the as litigation expenses; the amount of ₱50,000 as aforesaid described real property, together with the
properties within a period of three years from the attorney's fees and the costs of this suit. improvements thereon, and the two (2) motor vehicles,
SO ORDERED.9 together with their respective trailers from the Vendee
261
Sales – Chapter 3-12 Cases
within a period of three (3) years from the execution of this Court affirms the judgment of the CA upholding Eduardo’s the vendee, subject to the resolutory condition of
document on the purchase price agreed upon by the exercise of the right of repurchase. Roberto could no repurchase by the vendor within the stipulated
parties after considering the amount previously paid to the longer assail the factual findings because his petition for period.23 Accordingly, the ownership of the affected
Vendors in the amount of TWO MILLION PESOS review on certiorari was limited to the review and properties reverted to Eduardo once he complied with the
(₱2,000,000.00), Philippine Currency, with an interest of determination of questions of law only. A question of law condition for the repurchase, thereby entitling him to the
twelve percent (12%) per annum and the amount paid exists when the doubt centers on what the law is on a possession of the other motor vehicle with trailer.
with the Development Bank of the Philippines with an certain set of undisputed facts, while a question of fact WHEREFORE, the Court AFFIRMS the decision
interest of twelve percent (12%) per annum.15 exists when the doubt centers on the truth or falsity of the promulgated on October 10, 2003; and ORDERS the
The CA and the RTC both found and held that Eduardo alleged facts.18Whether the conditions for the right to petitioner to pay the costs of suit.
had complied with the conditions stipulated in the deed of repurchase were complied with, or whether there was a SO ORDERED.
sale and prescribed by Article 1616 of the Civil Code. tender of payment, is a question of fact. With both the
Pertinently, the CA stated: RTC and the CA finding and holding that Eduardo had
It should be noted that the alleged repurchase was fulfilled the conditions for the exercise of the right to
exercised within the stipulated period of three (3) years repurchase, therefore, we conclude that Eduardo had
from the time the Deed of Sale with Assumption of effectively repurchased the properties subject of the deed
Mortgage was executed. The only question now, of sale.
therefore, which remains to be resolved is whether or not In Metropolitan Bank and Trust Company v. Tan,19 the
the conditions set forth in the Deed of Sale with Court ruled that a redemption within the period allowed by
Assumption of Mortgage, i.e. the tender of the purchase law is not a matter of intent but of payment or valid tender
price previously agreed upon, which is Php2.0 Million, of the full redemption price within the period. Verily, the
plus 12% interest per annum, and the amount paid by the tender of payment is the seller’s manifestation of his
defendant to DBP, had been satisfied. desire to repurchase the property with the offer of
From the testimony of the defendant himself, these immediate performance.20 As we stated in Legaspi v.
preconditions for the exercise of plaintiff's right to Court of Appeals,21 a sincere tender of payment is
repurchase were adequately satisfied by the latter. Thus, sufficient to show the exercise of the right to repurchase.
as stated, from the Php10 Million purchase price which Here, Eduardo paid the repurchase price to Roberto by
was directly paid to the defendant, the latter deducted his depositing the proceeds of the sale of the Baguio City lot
expenses plus interests and the loan, and the remaining in the latter’s account. Such payment was an effective
amount he turned over to the plaintiff. This testimony is an exercise of the right to repurchase.
unequivocal acknowledgement from defendant that On the other hand, the Court dismisses as devoid of merit
plaintiff and his co-heirs exercised their right to Roberto’s insistence that the MOA had extinguished the
repurchase the property within the agreed period by obligations established under the deed of sale by
satisfying all the conditions stipulated in the Deed of Sale novation.
with Assumption of Mortgage. Moreover, defendant The issue of novation involves a question of fact, as it
returned to plaintiff the amount of Php2.8 Million from the necessarily requires the factual determination of the
total purchase price of Php10.0 Million. This only means existence of the various requisites of novation, namely: (a)
that this is the excess amount pertaining to plaintiff and there must be a previous valid obligation; (b) the parties
co-heirs after the defendant deducted the repurchase concerned must agree to a new contract; (c) the old
price of Php2.0 Million plus interests and his expenses. contract must be extinguished; and (d) there must be a
Add to that is the fact that defendant returned one of the valid new contract.22 With both the RTC and the CA
trucks and trailers subject of the Deed of Sale with concluding that the MOA was consistent with the deed of
Assumption of Mortgage to the plaintiff. This is, at best, a sale, novation whereby the deed of sale was extinguished
tacit acknowledgement of the defendant that plaintiff and did not occur. In that regard, it is worth repeating that the
his co-heirs had in fact exercised their right to factual findings of the lower courts are binding on the
repurchase.16 x x x Court.
Considering that the factual findings of the trial court, In sales with the right to repurchase, the title and
when affirmed by the CA, are binding on the Court,17 the ownership of the property sold are immediately vested in
262
Sales – Chapter 3-12 Cases
G.R. No. 92248 December 9, 1992 To stave off the foreclosure, the Illuscupideses sold the WHEREFORE, premises considered, by preponderance
VICENCIO T. TORRES and SOCORRO S. properties to Vivencio Torres and Socorro Torres of evidence, judgment is hereby rendered:
TORRES, petitioners, (petitioners in G.R. No. 92248), as evidenced by the Deed 1. Dismissing the complaint for rescission filed by plaintiff;
vs. of Sale dated October 19, 1973 2 for P130,000.00, of 2. Ordering the dismissal of the cross-claim and
COURT OF APPEALS, CEFERINO ILLUSCUPIDES, which the vendees paid the vendors P10,000.00, counterclaims of defendants Illuscupides against
ARACELI ILLUSCUPIDES and EMILIO P6,000.00 and P3,000.00. The vendees likewise paid defendants Torres and plaintiff;
OLORES, respondents. P51,498.97 to the GSIS. The aforesaid payments were in 3. Ordering defendants Illuscupides and/or Torres to
G.R. No. 93390 December 9, 1992 accordance to the schedule found in the promissory note deliver the P41,000.00 withheld by them as part of the
CEFERINO ILLUSCUPIDES and ARACELI CAMACHO- executed by the parties on October 19, 1973, 3 which purchase price of the lots and apartments for the
ILLUSCUPIDES, petitioners, provided — satisfaction of the claim of plaintiff;
vs. Downpayment (paid on October 1973) .......... P10,000.00 4. Ordering defendants Illuscupides to pay plaintiff and
COURT OF APPEALS, VICENCIO T. TORRES and Payment to the GSIS defendants Torres the sum of P5,000.00 as attorney's
SOCORRO S. TORRES, respondents. (assumption of mortgage) .......... 51,000.00 fees each; (and)
Cash payment upon issuance of 5. Ordering the defendants Illuscupides to pay the costs.
NOCON. J.: title in the name of vendee ......... 25,000.00 Olores and the Illuscupides then appealed to the Court of
Before Us is the petition for review on certiorari of Balance payable as follows: Appeals, where the case was docketed as CA-G.R. CV
petitioners Ceferino Illuscupides and Araceli Camacho- Dec. 30, 1974 .................... 11,000.00 No. 14779. On January 18, 1990, the appellate court
Illuscupides in G.R. No. 93390 from the decision of the Dec. 30, 1975 .................... 11,000.00 rendered a decision, 6 the dispositive portion of which
Court of Appeals dated January 18, 1990. The petition for Dec. 30, 1976 .................... 11,000.00 reads as follows:
review on certiorari in G.R. No. 92248, Vivencio T. Torres, Dec. 30, 1977 .................... 11,000.00 WHEREFORE, the decision dated October 7, 1986 is
et al. vs. Court of Appeals, et al., was dismissed by the Provided that no installment hereby AFFIRMED insofar as the dismissal of the
Court on June 18, 1990 1 for failure to show that a shall be paid until after the complaint of plaintiff-appellant Olores, the cross-claim and
reversible error was committed by the Court of Appeals, final adjudication of claim of counter-claim of defendants-appellants Illuscupides, and
and no motion for reconsideration was taken therefrom. Engr. E. Olores against vendor .................... 44,000.00 the counter-claim of defendant-appellees Torres;
The facts are undisputed: the Illuscupideses are the ————— REVERSED insofar as Nos. 3, 4 and 5 of the dispositve
owners of two (2) adjoining parcels of lands located in the P130,000.00 portion of the Decision are concerned; and the
Tapuac District, Dagupan City. The parcels are covered The parties also executed on the same day an defendants-appellees spouses Vivencio Torres and
by TCT Nos. 14874 and 15167, and have a combined agreement 4 whereby the Torreses would "RESELL, Socorro Torres are ordered to reconvey in favor of the
area of 465 square meters. The said properties were RETRANSFER, and RECONVEY" to the Illuscupideses defendants-cross-claimants spouses Ceferino Illuscupides
mortgaged to the Government Service Insurance System "that certain building, more particularly designated as a and Socorro Illuscupides "that certain building more
(GSIS). ten-door concrete apartment." particularly designated as a ten-door apartment in the
Sometime in 1965, the Illuscupideses contracted Emilio Olores found out about the transaction and, fearing that Deed of Sale executed by and between the above-named
Olores for the construction of a nine (9) door apartment on he would not be able to collect from the Illuscupideses, in parties on October 19, 1973." Without pronouncement as
the parcels of land for the sum of P79,400.00. While case the Court of Appeals would uphold the decision of to costs.
construction was going on, another door was added, the trial court in his favor, filed a new case for rescission With regard to the appeal of the Illuscupideses, the Court
thereby increasing the cost of the construction to of the sale against the Illuscupideses and the Torreses. of Appeals did not agree with their contention that the sale
P97,000.00. However, the Illuscupideses could only pay The Illuscupideses filed a counter-claim against Olores, of the properties to the Torreses was actually a pacto de
Olores P54,390.51, thus compelling the latter to sue them and a cross-claim against the Torreses, alleging that the retro sale, since the terms of the Deed of Sale did not
for the balance before the Court of First Instance of Deed of Sale was a pacto de retro sale. provide for the redemption of the property by the vendors.
Pangasinan in Civil Case No. D-1955. On November In 1977, the Court of Appeals upheld the decision in the However, the appellate court discovered that the land and
1969, judgment was rendered in favor of Olores for the collection case for the unpaid balance of the construction the apartment were sold separately, and only the land
unpaid balance with interests and costs. The costs in favor of Olores. When said judgment became final appears to have been fully paid. And since the Agreement
Illuscupideses then appealed the decision to the Court of and executory. Olores tried to execute the same but was (Annex "F") provided that the apartment should be resold
Appeals. unable to do so. to the Illuscupideses, the appellate court held that the
Meanwhile, the Illuscupideses received a notice from the Meanwhile, trial in the rescission case continued until Torreses should reconvey the apartment to the
GSIS that it was going to foreclosure the mortgage for judgment was rendered on October 7, 1986, 5 the Illuscupideses.
their failure to pay the loan when the same became due. dispositive portion of which provided:
263
Sales – Chapter 3-12 Cases
The Illuscupideses filed a motion asking that the Court of WHEREFORE, the decision appealed from is hereby
Appeals rule upon the apartment rentals collected by AFFIRMED in toto. Costs against petitioners
Torreses, since it had ruled that the apartment be Illuscupideses.
reconveyed to them. The appellate court denied the SO ORDERED.
motion on the ground that the matter of the rentals was
not raised as assignment of error in their brief.
From said resolution, the Illuscupideses elevated the case
to this Court on a petition for review for certiorari. The
Torreses filed a separate petition for review on certiorari,
but the same was dismissed by this Court on June 18,
1990. The dismissal of the Torreses' petition is now final
in view of their failure to file a timely motion for
reconsideration.
In their petition, the Illuscupideses allege that the Court of
Appeals erred in (1) not construing the Deed of Sale of
October 19, 1973 to be a pacto de retro sale; and (2) in
not ruling upon the rentals collected by the Torreses from
the apartment after it had ordered the reconveyance of the
apartment to the Illuscupides.
The petition is totally devoid of merit.
The Court of Appeals was correct in construing the Deed
of Sale as an absolute sale inasmuch as the terms thereof
are clear on the matter. The Illuscupideses argue,
however, that the appellate court should have taken into
account the circumstances surrounding the execution of
the deed, particularly the fact that an Agreement to resell
the apartment was executed on the very same day as the
deed of sale.
The argument is unavailing. Even if this Court were to
agree with the Illuscupideses that parole evidence may be
allowed to add to the terms of the deed of sale, this Court
has held in the case of Villarica, et al. vs. Court of
Appeals, et al., 7 that —
[t]he right of repurchase is not a right granted the vendor
by the vendee in a subsequent instrument, but is a right
reserved by the vendor in the same instrument of sale as
one of the stipulations of the contract. Once the
instrument of absolute sale is executed, the vendor can no
longer reserve the right to repurchase, and any right
thereafter granted the vendor by the vendee in a separate
instrument cannot be a right to repurchase but some other
right like an option to buy in the instant case. 8
As with regard to the Illuscupideses' second contention,
the Court of Appeals acted correctly in not passing upon
the rentals collected by the Torreses since the
Illuscupideses did not ask for the same in their original
cross-claim.
264
Sales – Chapter 3-12 Cases
G.R. No. 171250 July 4, 2007 said Register of Deeds to annotate in the new Transfer registered in the name of Jocelyn and her husband
SPS. CARLOS AND EULALIA RAYMUNDO and SPS. Certificate of Title in the names of the spouses Bandong a Angelito Buenaobra (Angelito).
ANGELITO AND JOCELYN BUENAOBRA, Petitioners, real estate mortgage in favor of the spouses Carlos and After the TCT of the subject property was transferred to
vs. Eulalia Raymundo reflecting the terms of this Decision. their names, the Spouses Buenaobra instituted before the
SPS. DOMINADOR and ROSALIA 4. AWARDING – moral damages in the amount of Metropolitan Trial Court (MeTC) of Caloocan City, an
BANDONG, Respondents. ₱50,000.00; exemplary damages of ₱20,000.00; and action for ejectment against the Spouses Bandong,
DECISION attorney’s fees and expenses of litigation of ₱20,000.00, docketed as Civil Case No. 20053, seeking the eviction of
CHICO-NAZARIO, J.: plus ₱500.00 per proven appearance of the plaintiffs- the latter from the subject property, which the Spouses
This is a Petition for Review on Certiorari under Rule 45 of appellants’ counsel in court – all solidarily payable by the Bandong opposed on the ground that they are the rightful
the Revised Rules of Court, filed by petitioners Spouses spouses Carlos and Eulalia Raymundo and the spouses owners and possessors thereof. The MeTC ruled in favor
Carlos and Eulalia Raymundo and Spouses Angelito and Angelito and Jocelyn Buenaobra, to the spouses of the Spouses Buenaobra which, on appeal, was affirmed
Jocelyn Buenaobra seeking the reversal and setting aside Dominador and Rosalia Bandong. in toto by the RTC7 and subsequently, by the Court of
of the Decision1 of the Court of Appeals dated 26 5. ORDERING the payment of the costs of the suit, Appeals.8 Finally, when the case was raised on appeal
September 2005 and its Resolution2 dated 24 January payable by the spouses Carlos and Eulalia Raymundo before us in G.R. No. 109422, we issued a
2006 in CA-G.R. CV No. 59557. The Court of Appeals, in and the spouses Angelito and Jocelyn Buenaobra.4 Resolution9dated 12 July 1993, finding that no substantial
its assailed Decision and Resolution, reversed the The factual and procedural backdrop of this case are as arguments were raised therein to warrant the reversal of
Decision3 of the Regional Trial Court (RTC) dated 28 follows: the appealed decision.
January 1998, in Civil Case No. C-14980, declaring the Eulalia was engaged in the business of buying and selling To assert their right to the subject property, the Spouses
Deed of Sale executed by respondent Dominador large cattle from different provinces within the Philippines. Bandong instituted an action for annulment of sale before
Bandong (Dominador) in favor of petitioner Eulalia For this purpose, she employed "biyaheros" whose the RTC against Eulalia and Jocelyn on the ground that
Raymundo (Eulalia) as valid and binding. The dispositive primary task involved the procuring of large cattle with the their consent to the sale of the subject property was
portion of the asailed Court of Appeals Decision reads: financial capital provided by Eulalia and delivering the vitiated by Eulalia after they were served by Jocelyn’s
WHEREFORE, premises considered, we hereby GRANT procured cattle to her for further disposal. In order to counsel with the demand to vacate. This was docketed as
the appeal. The January 28, 1998 decision of the RTC, secure the financial capital she advanced for the Civil Case No. C-14980. The Spouses Bandong alleged
Branch 126, Caloocan City is hereby REVERSED and "biyaheros," Eulalia required them to surrender the that there was no sale intended but only equitable
SET ASIDE and a new one entered: Transfer Certificates of Title (TCTs) of their properties and mortgage for the purpose of securing the shortage
1. ANNULLING the Deed of Absolute Sale dated February to execute the corresponding Deeds of Sale in her favor. incurred by Dominador in the amount of ₱70,000 while
3, 1989 as a deed of sale, and considering it instead as a Dominador had been working for Eulalia as one of her employed as "biyahero" by Eulalia.
real estate mortgage of the disputed property to secure biyaheros for three decades. Considering his long years of Eulalia countered that Dominador received from her a
the payment of the ₱70,000.00 the plaintiffs-appellants service without any previous derogatory record, Eulalia no significant sum of money, either as cash advances for the
spouses Bandong owe the defendants-appellees spouses longer required Dominador to post any security in the purpose of procuring large cattle or as personal loan, and
Raymundo. The spouses Bandong are given one (1) year performance of his duties.5 when he could no longer pay his obligations, the Spouses
from the finality of this Decision within which to pay the However, in 1989, Eulalia found that Dominador incurred Bandong voluntarily ceded the subject property to her by
₱70,000.00 owed to the spouses Raymundo, at 12% shortage in his cattle procurement operation in the amount executing the corresponding deed of sale in her favor.
interest per annum computed from July 17, 1991 until its of ₱70,000.00. Dominador and his wife Rosalia Bandong Indeed, the Spouses Bandong personally appeared
full payment. (Rosalia) then executed a Deed of Sale6 in favor of Eulalia before the Notary Public and manifested that the deed
2. ANNULLING the Deed of Absolute Sale dated on 3 February 1989, covering a parcel of land with an was their own voluntary act and deed.
September 25, 1990, between the spouses Raymundo as area of 96 square meters, more or less, located at For her part, Jocelyn maintained that she was a buyer in
vendors and the spouses Buenaobra as vendees. Caloocan City and registered under TCT No. 1421 good faith and for value for she personally inquired from
3. ORDERING the Register of Deeds of Caloocan City to (subject property), in the name of the Spouses Bandong. the Register of Deeds of the presence of any liens and
issue a new Transfer Certificate of Title covering Lot 18, On the strength of the aforesaid deed, the subject encumbrances on the TCT of the subject property and
Block 2 of the subdivision plan PSD 16599, a portion of property was registered in the names of Eulalia and her found that the same was completely free therefrom. While
Lot 1073 of the Cadastral Survey of Caloocan, in the husband Carlos Raymundo (Carlos). The subject property she admitted that she had previous notice that Dominador
names of the spouses Dominador and Rosalia Bandong, was thereafter sold by the Spouses Raymundo to Eulalia’s and a certain Lourdes Santos (Lourdes) were in
after the cancellation pursuant to this Decision of TCT No. grandniece and herein co-petitioner, Jocelyn Buenaobra possession of the subject property, Jocelyn claimed that
222871 currently in the names of the spouses Angelito (Jocelyn). Thus, the subject property came to be the said possessors already acknowledged her ownership
and Jocelyn Buenaobra; and FURTHER ORDERING the
265
Sales – Chapter 3-12 Cases
thereof and even asked for time to vacate. In the end, has not been proven that the consideration of sale (6) In any other case where it may be fairly inferred that
though, they refused to leave the premises. between Dominador and Eulalia was grossly inadequate. the real intention of the parties is that the transaction shall
On 28 June 1998, the RTC rendered a Decision 10 in Civil Elaborating, petitioners maintain that the amount of secure the payment of a debt or the performance of any
Case No. C-14980 in favor of Eulalia and Jocelyn by ₱110,000.00 (which they claimed they have given to other obligation.
declaring that the Deed of Sale between Dominador and Dominador), or even the sum of ₱70,000.00 (which Art. 1604. The provisions of Article 1602 shall also apply
Eulalia was valid and binding and, consequently, the respondents admitted receiving), was a substantial to a contract purporting to be an absolute sale.
subsequent sale between Eulalia and Jocelyn was also consideration, sufficient to support a sale contract. Mere For Articles 1602 and 1604 to apply, two requisites must
lawful absent any showing that Jocelyn was a buyer in inadequacy of the price is not sufficient to invalidate a concur: one, the parties entered into a contract
bad faith. The dispositive portion of the said decision sale; the price must be grossly inadequate or utterly denominated as a contract of sale; and two, their intention
reads: shocking to the conscience in order to avoid a contract of was to secure an existing debt by way of an equitable
WHEREFORE, judgment is hereby rendered sale. mortgage.13
DISMISSING the complaint filed by the [Spouses Petitioners further aver that the alleged market value of There is no question that Dominador and Eulalia entered
Bandong] and ordering said [Spouses Bandong] to pay the subject property as submitted by the appraiser, one of into a contract of sale as evidenced by the document
[herein petitioners] spouses Raymundo and Buenaobra respondents’ witnesses, would not serve as an objective denominated as Deed of Sale14 signed by them. As to
the amount of ₱50,000 and ₱30,000, respectively, as basis in determining the actual value of the subject whether the parties intended to transfer ownership of the
attorney’s fees and costs of the suit. property, much less the supposed amount of its purchase subject property or merely to constitute a security for an
On appeal in CA-G.R. SP No. 59557, the Court of price, in the absence of any logical and valid basis for its existing debt is an issue that needs to be addressed by
Appeals reversed the RTC Decision and found that the determination. this Court.
transaction entered into by Dominador and Eulalia was Finally, petitioners contend that so long as the contract In resolving this kind of controversy, the doctrine in Reyes
not one of sale but an equitable mortgage considering that was voluntarily entered into by the parties and in the v. Court of Appeals15 directs us to give utmost
the purchase price was grossly inadequate and the absence of a clear showing that their consent thereto was consideration to the intention of the parties in light of the
Spouses Bandong remained as possessors of the subject vitiated by fraud, mistake, violence or undue influence, relative situation of each and the circumstances
property after Eulalia’s alleged purchase thereof. The such as in the case at bar, the said contract should be surrounding the execution of the contract, thus:
appellate court likewise charged Jocelyn with knowledge upheld. In determining whether a deed absolute in form is a
that the Spouses Raymundo were not the absolute We do not agree. mortgage, the court is not limited to the written memorials
owners of the subject property negating the presumption An equitable mortgage is one that - although lacking in of the transaction. The decisive factor in evaluating
that she was an innocent purchaser for value. some formality, forms and words, or other requisites such agreement is the intention of the parties, as
The Court of Appeals found the Motion for demanded by a statute - nevertheless reveals the shown not necessarily by the terminology used in the
Reconsideration filed by petitioners unmeritorious and intention of the parties to charge a real property as contract but by all the surrounding circumstances,
denied the same in its Resolution11 dated 24 January security for a debt and contains nothing impossible or such as the relative situation of the parties at that
2006. contrary to law.12 time, the attitude acts, conduct, declarations of the
Hence, this instant Petition for Review on Certiorari filed The instances when a contract - regardless of its parties, the negotiations between them leading to the
by the petitioners assailing the Decision dated 26 nomenclature - may be presumed to be an equitable deed, and generally, all pertinent facts having a tendency
September 2005 and the Resolution dated 24 January mortgage are enumerated in the Civil Code as follows: to fix and determine the real nature of their design and
2006 rendered by the Court of Appeals. For the resolution Art. 1602. The contract shall be presumed to be an understanding. x x x16 (Emphasis supplied.)
of this Court are the following issues: equitable mortgage, in any of the following cases: By applying the aforestated principle to the case at bar,
I. (1) When the price of a sale with right to repurchase is we are constrained to rule that in executing the said Deed
WHETHER OR NOT THE DEED OF SALE BETWEEN unusually inadequate; of Sale, Dominador and Eulalia never intended the
DOMINADOR AND EULALIA IS VALID AND BINDING. (2) When the vendor remains in possession as lessee or transfer of ownership of the subject property but to burden
II. otherwise; the same with an encumbrance to secure the
WHETHER OR NOT JOCELYN IS A BUYER IN GOOD (3) When upon or after the expiration of the right to indebtedness incurred by Dominador on the occasion of
FAITH. repurchase another instrument extending the period of his employment with Eulalia.
In arguing that the sale between Dominador and Eulalia is redemption or granting a new period is executed; By Eulalia’s own admission,17 it was her customary
valid, petitioners posit that gross inadequacy of the price (4) When the purchaser retains for himself a part of the business practice to require her biyaheros to deliver to her
is not sufficient to invalidate the sale, and granting purchase price; the titles to their real properties and to execute in her favor
arguendo that insufficient consideration may void a sale, it (5) When the vendor binds himself to pay the taxes on the the corresponding deeds of sale over the said properties
thing sold. as security for the money she provided for their cattle
266
Sales – Chapter 3-12 Cases
procurement task, and since Dominador worked for Moreover, granting that the purchase price is adequate, claim that he acted in good faith under the belief that
Eulalia’s business for years, he was allowed to advance the fact that respondents remain in possession of the there was no defect in the title of the vendor or
the money without any security. Significantly, it was only subject property after its supposed sale is sufficient to mortgagor. His mere refusal to face up to the fact that
after he incurred a shortage that the sale contract was support our finding that the contract is one of equitable such defect exists, or his willful closing of his eyes to the
executed. mortgage and not of sale. To reiterate, the existence of possibility of the existence of a defect in the vendor’s or
We are not inclined to believe the contention of the any one of the conditions under Article 1602, not a mortgagor’s title, will not make him an innocent purchaser
petitioners that Dominador ceded his property to Eulalia concurrence, or an overwhelming number of such for value, if it afterwards develops that the title was in fact
as payment for his obligation for it is contrary to human circumstances, suffices to give rise to the defective, and it appears that he had such notice of the
experience that a person would easily part with his presumption that the contract is an equitable defect as would have led to its discovery had he acted
property after sustaining a debt. Rather, he would first mortgage.19 with the measure of precaution which may be required of
look for means to settle his obligation, and the selling of a Having threshed the issue that there was no sale in a prudent man in a like situation.
property on which the house that shelters him and his favor of Eulalia but an equitable mortgage leads us to In the present case, we are not convinced by the
family stands, would be his last resort. The only an inevitable conclusion that she has no right to petitioners’ incessant assertion that Jocelyn is an innocent
reasonable conclusion that may be derived from subsequently transfer ownership of the subject purchaser for value. To begin with, she is a grandniece of
Dominador’s act of executing a Deed of Sale in favor of property, in consonance with the principle that Eulalia and resides in the same locality where the latter
Eulalia is that the latter required him to do so in order to nobody can dispose of what he does not have.20 One lives and conducts her principal business. It is therefore
ensure that he will subsequently pay his obligation to her. of the exceptions21 to this rule, however, can be found impossible for her not to acquire knowledge of her grand
This conclusion is in accord with the doctrine we in Article 1506 of the Civil Code, wherein the seller aunt’s business practice of requiring her biyaheros to
enunciated in Aguirre v. Court of Appeals,18 that: has voidable title to a property but his title has not yet surrender the titles to their properties and to sign the
The explicit provision of Article 1602 that any of those been nullified at the time of the sale, and the corresponding deeds of sale over said properties in her
circumstances would suffice to construe a contract of subsequent buyer of the property was in good faith. favor, as security. This alone should have put Jocelyn on
sale to be one of equitable mortgage is in consonance An innocent purchaser for value is one who buys the guard for any possible abuses that Eulalia may commit
with the rule that the law favors the least transmission property of another, without notice that some other person with the titles and the deeds of sale in her possession.
of property rights. To stress, the existence of any one of has a right or interest in the property, for which a full and The glaring lack of good faith of Jocelyn is more apparent
the conditions under Article 1602, not a concurrence, or fair price is paid by the buyer at the time of the purchase in her own admission that she was aware that Dominador
an overwhelming number of such circumstances, suffices or before receipt of any notice of claims or interest of and a certain Lourdes were in possession of the subject
to give rise to the presumption that the contract is an some other person in the property.22 property. A buyer of real property that is in the possession
equitable mortgage. Petitioners are harping on the contention that Jocelyn was of a person other than the seller must be wary. A buyer
While we agree in the petitioners’ insistence that an innocent purchaser for value. Invoking the who does not investigate the rights of the one in
inadequacy of the price is not sufficient to nullify the indefeasibility of a Torrens title, they assert that there is possession can hardly be regarded as a buyer in good
contract of sale, their persistence is, however, misplaced. nothing in the subject property’s TCT that should arouse faith.25 Jocelyn’s self-serving statement that she
It is worthy to note that the factual circumstances Jocelyn’s suspicion as to put her on guard that there is a personally talked to Dominador and Lourdes about her
attendant in the case at bar call not for the application of defect in Eulalia’s title. acquisition of the subject property and intention to take
the legal and jurisprudential principles on annulment of Again, we are not persuaded. The burden of proving the possession of the same, and that Dominador and Lourdes
contract per se, but more aptly, of the provisions of purchaser’s good faith lies in the one who asserts the even pleaded for time to vacate the subject property
Articles 1602 and 1604 of the Civil Code on the same. In discharging the burden, it is not enough to cannot be given credence in light of the prompt filing by
construction of the contract of sale as an equitable invoke the ordinary presumption of good faith.23 In Arrofo the Spouses Bandong of an action for the annulment of
mortgage. v. Quiño,24 we have elucidated that: the sale contract between Dominador and Eulalia after
Consequently, the agreement between Dominador and [A] person dealing with registered land, [is not required] to they received the demand to vacate from Jocelyn’s
Eulalia was not avoided in its entirety so as to prevent it inquire further that what the Torrens title on its face lawyer.
from producing any legal effect at all. Instead, we construe indicates. This rule, however, is not absolute but admits of In the last analysis, good faith, or the lack of it, is a
that said transaction is an equitable mortgage, thereby exceptions. question of intention.1awphi1 But in ascertaining the
merely altering the relationship of the parties from seller Thus, while it is true x x x that a person dealing with intention that impels one on a given occasion, the courts
and buyer, to mortgagor and mortgagee, while the subject registered lands need not go beyond the certificate of are necessarily controlled by the evidence as to the
property is not transferred but subjected to a lien in favor title, it is likewise a well-settled rule that a purchaser conduct and other outward acts by which the motive may
of the latter. or mortgagee cannot close his eyes to facts which be safely determined.26
should put a reasonable man on his guard, and then
267
Sales – Chapter 3-12 Cases
Petitioners question further the belated filing by the
Spouses Bandong of an action for the annulment of sale,
since the Spouses Bandong filed the same only after they
received the notice to vacate, and not immediately after
the execution of the assailed Deed of Sale. We have
repeatedly held that the one who is in actual possession
of a piece of land claiming to be the owner thereof may
await to vindicate his right. His undisturbed possession
gives him a continuing right to seek the aid of a court of
equity to ascertain and determine the nature of the
adverse claim of a third party and its effect on his own
title, which right can be claimed only by one who is in
possession.27
Finally, we agree with the Court of Appeals that the
ejectment case which had been litigated to finality by the
Spouses Buenaobra and the respondents need not alter
our conclusion in the present case. Well entrenched is the
doctrine that in ejectment cases, the sole question for
resolution is the physical or material possession of the
property in question, so that neither the claim of juridical
possession nor an averment of ownership can outrightly
prevent the court from taking cognizance of the case.28 In
ejectment cases, all the court may do is to resolve who is
entitled to its possession although, in doing so, it may
make a determination of who is the owner of the property
in order to resolve the issue of possession. But such
determination of ownership is not clothed with finality.
Neither will it affect ownership of the property or constitute
a binding and conclusive adjudication on the merits with
respect to the issue of ownership.29
WHEREFORE, IN VIEW OF THE FOREGOING, the
instant Petition is DENIED. The Decision dated 26
September 2005, and the Resolution dated 24 January
2006, rendered by the Court of Appeals in CA-G.R. SP
No. 59957, are hereby AFFIRMED. Costs against
petitioner.
SO ORDERED.

268
Sales – Chapter 3-12 Cases
G.R. No. 158377 August 13, 2010 The Spouses Francia both died intestate (i.e., Monica and Amado Reyes dela Cruz
HEIRS OF JOSE REYES, JR., namely: MAGDALENA C. Ajoco on September 16, 1963, and Benedicto Francia on
REYES, OSCAR C. REYES, GAMALIEL C. REYES, January 13, 1964). Jose, Sr. Alejandro Reyes (respondents'
NENITA R. DELA CRUZ, RODOLFO C. REYES, and Alejandro Reyes (Alejandro), the son of Jose, Sr., first predecessor)12
RODRIGO C. REYES, Petitioners, partially paid to the Spouses Francia the amount of
On September 2, 1993, Alejandro also died
vs. ₱265.00 for the obligation of Leoncia, his uncles and his
intestate.13 Surviving him were his wife, Amanda Reyes,
AMANDA S. REYES, CONSOLACION S. REYES, father. Alejandro later paid the balance of ₱235.00. Thus,
and their children, namely: Consolacion Reyes, Eugenia
EUGENIA R. ELVAMBUENA, LUCINA R. MENDOZA, on August 11, 1970, the heirs of Spouses Francia
Reyes-Elvambuena, Luciana Reyes-Mendoza, Pedrito S.
PEDRITO S. REYES, MERLINDA R. FAMODULAN, executed a deed entitled Pagsasa-ayos ng Pag-aari at
Reyes, Merlinda Reyes-Famodulan, Eduardo Reyes and
EDUARDO S. REYES, and JUNE S. Pagsasalin,5whereby they transferred and conveyed to
June S. Reyes (respondents herein).
REYES, Respondents. Alejandro all their rights and interests in the property for
In 1994, respondent Amanda Reyes asked the heirs of
DECISION ₱500.00.
Teofilo and Jose, Jr., to vacate the property because she
BERSAMIN, J.: On August 21, 1970, Alejandro executed a Kasulatan ng
and her children already needed it. After the petitioners
The petitioners1 assail the decision dated July 31, 2002 Pagmeme-ari,6 wherein he declared that he had acquired
refused to comply, she filed a complaint against the
rendered in C.A.-G.R. CV No. 53039,2 by which the Court all the rights and interests of the heirs of the Spouses
petitioners in the barangay, seeking their eviction from the
of Appeals (CA) affirmed the decision dated May 21, 1996 Francia, including the ownership of the property, after the
property. When no amicable settlement was reached, the
of the Regional Trial Court (RTC), Branch 9, in Malolos, vendors had failed to repurchase within the given period.
Barangay Lupon issued a certification to file action to the
Bulacan.3 On the basis of the Kasulatan ng Pagmeme-ari, Tax
respondents on September 26, 1994.14
Antecedents Declaration No. 3703 covering the property7 was canceled
In the interim, petitioner Nenita R. de la Cruz and her
Antonio Reyes and his wife, Leoncia Mag-isa Reyes by Tax Declaration No. 8715,8 effective 1971, issued to
brother Romeo Reyes also constructed their respective
(Leoncia), were owners of a parcel of residential land with Alejandro. From then on, he had paid the realty taxes for
houses on the property.15
an area of 442 square meters, more or less, located in the property.
RTC Proceedings and Ruling
Pulilan, Bulacan and covered by Tax Declaration No. Nevertheless, on October 17, 1970, Alejandro, his
On September 28, 1994, the respondents initiated this suit
7590. On that land they constructed their dwelling. The grandmother (Leoncia), and his father (Jose, Sr.)
for quieting of title and reconveyance in the RTC. 16 The
couple had four children, namely: Jose Reyes, Sr. (Jose, executed a Magkakalakip na Salaysay,9 by which
complaint, docketed as Civil Case No. 817-M-94 and
Sr.), Teofilo Reyes (Teofilo), Jose Reyes, Jr. (Jose, Jr.) Alejandro acknowledged the right of Leoncia, Jose, Jr.,
entitled Amanda Reyes, et al. v. Heirs of Jose Reyes, Jr.,
and Potenciana Reyes-Valenzuela (Potenciana). Antonio and Jose, Sr. to repurchase the property at any time for
et al., was later amended.17 They alleged that their
Reyes died intestate, and was survived by Leoncia and the same amount of ₱500.00.
predecessor Alejandro had acquired ownership of the
their three sons, Potenciana having predeceased her On October 22, 1970, Leoncia died intestate.10 She was
property by virtue of the deed Pagsasa-ayos ng Pag-aari
father. Potenciana also died intestate, survived by her survived by Jose, Sr., Teofilo, Jose, Jr. and the heirs of
at Pagsasalin executed on August 11, 1970 by the heirs
children, namely: Gloria ReyesValenzuela, Maria Reyes Potenciana. Even after Leonica's death, Teofilo and Jose,
of the Spouses Francia; that on the basis of such deed of
Valenzuela, and Alfredo Reyes Valenzuela. Jose, Jr., and Jr., with their respective families, continued to reside in
assignment, Alejandro had consolidated his ownership of
his family resided in the house of the parents, but Teofilo the property.
the property via his Kasulatan ng Pagmeme-ari; and that
constructed on the property his own house, where he and Subsequently, Tax Declaration 1228,11 under the name of
under the Magkasanib na Salaysay, Alejandro had
his family resided. Alejandro, was issued effective 1980. All of Leoncia's
granted to Leoncia, his father Jose, Sr., and his uncles,
On July 9, 1955, Leoncia and her three sons executed a sons eventually died intestate, survived by their respective
Teofilo and Jose, Jr. the right to repurchase the property,
deed denominated Kasulatan ng Biling Mabibiling heirs, namely:
but they had failed to do so.
Muli,4whereby they sold the land and its existing
Name of Surviving Heirs The respondents prayed for judgment in their favor, as
improvements to the Spouses Benedicto Francia and Decedent follows:
Monica Ajoco (Spouses Francia) for ₱500.00, subject to
WHEREFORE, it is respectfully prayed that judgment be
the vendors' right to repurchase for the same amount sa Teofilo Romeo Reyes, Leonardo Reyes, rendered:
oras na sila'y makinabang. Potenciana's heirs did not Jose, Sr. and Leonora C. Reyes Rodrigo 1. Quieting the title to the property by declaring the
assent to that deed. Nonetheless, Teofilo and Jose, Jr. Reyes, Nenita Reyes- dela Cruz, plaintiffs (respondents herein) as the rightful and lawful
and their respective families remained in possession of Rodolfo Reyes, Oscar Reyes, owners thereof;
the property and paid the realty taxes thereon. Gamaliel Reyes, Magdalena Reyes
Leoncia and her children did not repay the amount of (petitioners herein), Efren Reyes
₱500.00.
269
Sales – Chapter 3-12 Cases
2. Ordering the defendants (petitioners herein) to vacate WHEREFORE, on the basis of the evidence adduced and In its decision dated July 31, 2002, the CA ruled that the
subject premises and reconvey and or surrender the law/jurisprudence applicable thereon, judgment is transaction covered by the Kasulatan ng Biling Mabibiling
possession thereof to the plaintiffs; hereby rendered: Muli was not a pacto de retro sale but an equitable
3. Ordering the defendants to recognize the right of the a) sustaining the validity of the "Kasulatan ng Biling mortgage under Article 1602 of the Civil Code; that even
plaintiffs as the lawful owners of subject property; Mabibiling Muli" (Exh. B/Exh. 1) executed on July 9, 1955 after the deed's execution, Leoncia, Teofilo, Jose, Jr. and
4. Ordering the defendants to pay plaintiffs the following: by Leoncia Mag-isa and her sons Teofilo, Jose, Sr. and their families had remained in possession of the property
a. Moral damages in the amount of P50,000.00; Jose, Jr., all surnamed Reyes, in favor of Spouses and continued paying realty taxes for the property; that the
b. Exemplary damages in the amount of P20,000.00; Benedicto Francia and Monica Ajoco as well as purported vendees had not declared the property for
c. Attorney's fees of P20,000.00, acceptance fee of the "Pagsasa-ayos ng Pag-aari at Pagsasalin" (Settlement taxation purposes under their own names; and that such
P10,000.00 and P500.00 per recorded Court appearance of Estate and Assignment) [Exh. C/Exh. 4] executed on circumstances proved that the parties envisaged an
of counsel; August 11, 1970 by the heirs of spouses Benedicto equitable mortgage in the Kasulatan ng Biling Mabibiling
d. The costs of this suit. Francia and Monica Ajoco in favor of the spouses Muli.
Plaintiffs further pray for such other relief which the Alejandro Reyes and Amanda Salonga; The CA observed that the heirs of the Spouses Francia
Honorable Court may deem just and equitable under the b) declaring the aforementioned "Kasulatan Ng Biling had themselves admitted in paragraph 5 of the Pagsasa-
premises.18 Mabibili Muli" (Exh. B/ Exh. 1) to be a contract of sale with ayos ng Pag-aari at Pagsasalin that the property had
In their answer,19 the petitioners averred that right to repurchase and not an equitable mortgage; been mortgaged to their predecessors-in-interest, viz:
the Kasulatan ng Biling Mabibiling Muli was an equitable c) confirming the consolidation of ownership, by operation Na, sa oras ng kamatayan ay nakaiwan sila ng isang
mortgage, not a pacto de retro sale; that the mortgagors of law, of spouses Alejandro M. Reyes and Amanda lagay na lupang nakasanla sa kanila na makikilala sa
had retained ownership of the property; that the heirs of Salonga over the residential lot mentioned and referred to kasulatang kalakip nito sa halagang LIMANG DAANG
the Spouses Francia could not have validly sold the in Exhibit B/Exhibit 1 and Exhibit C/Exhibit 4; PISO (P500.00). Ngunit nuong nabubuhay pa ang
property to Alejandro through the Pagsasaayos ng Pag- d) allowing the registration with the Registry of Deeds for magasawang Benedicto Francia at Monica Ajoco ay
aari at Pagsasalin; that Alejandro's right was only to seek the Province of Bulacan of the "Kasulatan ng Pagmeme- nakatanggap na ng halagang P265.00 kay Alejandro
reimbursement of the ₱500.00 he had paid from the co- ari" (Document of Ownership) [Exh. E/Exh. 5] executed by Reyes - Filipino, kasal kay Amanda Salonga, may sapat
owners, namely: Leoncia, Teofilo, Jose, Jr. and Jose, Sr. Alejandro M. Reyes on August 21, 1970 or of any na gulang at naninirahan sa Pulilan, Bulacan.21
and the heirs of Potenciana; and that Alejandro could not appropriate deed of consolidation of ownership over the However, the CA held that the appellants' (petitioners
have also validly consolidated ownership through residential lot covered by Exhibit E/Exhibit 5 which the herein) failure to file an action for the reformation of
the Kasulatan ng Pagmeme-ari, because a consolidation plaintiffs, as eventual owners by succession of the the Kasulatan ng Biling Mabibiling Muli to reflect the true
of ownership could only be effected via a court order. aforementioned proeprty, may deem proper to execute; intention of the parties within ten years from the deed's
The petitioners interposed a counterclaim for the e) ordering the defendants and all persons claiming rights execution on July 9, 1955, pursuant to Article 1144 of the
declaration of the transaction as an equitable mortgage, under them to vacate the residential lot subject of the Civil Code,22 already barred them from claiming that the
and of their property as owned in common by all the heirs above-entitled case and to restore possession thereof transaction executed between Leoncia and her children,
of Leoncia, Teofilo, Jose, Jr. and Jose, Sr. unto the plaintiffs; on one hand, and the Spouses Francia, on the other
On May 21, 1996, the RTC ruled in favor of the f) directing the defendants (except the heirs of Potenciana hand, was an equitable mortgage. The CA agreed with the
respondents, declaring that Alejandro had acquired Reyes-Valenzuela) to pay unto the plaintiffs the amount of RTC that the Magkakalakip na Salaysay did not effectively
ownership of the property in 1965 by operation of law P20,000.00 as attorney's fees; and extend the period for Leoncia and her children to
upon the failure of the petitioners' predecessors to g) dismissing the complaint in so far as the defendant repurchase the property, considering that the period to
repurchase the property; that the joint affidavit executed heirs of Potenciana Reyes-Valenzuela are concerned as repurchase had long lapsed by the time the agreement to
by Alejandro, Leoncia and Jose, Jr. and Jose, Sr., to well as their counterclaim for damages and attorney's extend it was executed on October 17, 1970.
extend the period of redemption was inefficacious, fees.1avvphi1 Issues
because there was no more period to extend due to the No pronouncement as to costs. In this appeal, therefore, the petitioners insist that:23
redemption period having long lapsed by the time of its SO ORDERED. 20 I.
execution; and that the action should be dismissed insofar Aggrieved, the petitioners appealed to the CA. The Honorable Court of Appeals erred in finding that
as the heirs of Potenciana were concerned, considering CA Ruling respondents (were) already barred from claiming that the
that Potenciana, who had predeceased her parents, had In the CA, the petitioners assailed the RTC's dispositions, transaction entered into by their predecessors-in-interest
no successional rights in the property. except the dismissal of the complaint as against was an equitable mortgage and not a pacto de retro sale;
Accordingly, the RTC decreed as follows: Potenciana's heirs. II.

270
Sales – Chapter 3-12 Cases
The Honorable Court of Appeals erred in affirming the July 9, 1955. Consequently, they urge the upholding of confirmed so in the Magkasanib na Salaysay, whereby he
findings of the court a quo that the Magkasanib na the original intention of the parties to the Kasulatan ng acknowledged the co-owners' right to redeem the property
Salaysay (Joint Affidavit), executed by Alejandro, Leoncia Biling Mabibiling Muli, without taking prescription into from him at any time (sa ano mang oras) for the same
and Jose, Jr., wherein Leoncia and her children were account, because both parties did not enforce their redemption price of ₱500.00.
granted by Alejandro the right to repurchase the property respective rights within the ten-year prescriptive period, is It is worthy to note that Alejandro's confirmation in
at anytime for the amount of P500.00, was of no legal more in keeping with fairness and equity. the Magkasanib na Salaysay of the co-owners' right to
significance. We agree with the petitioners. redeem was made despite 15 years having meanwhile
Ruling of the Court Considering that sa oras na sila'y makinabang, the period elapsed from the execution of the original Kasulatan ng
The petition is meritorious. of redemption stated in the Kasulatan ng Biling Mabibiling Biling Mabibiling Muli (July 9, 1955) until the execution of
A. Muli, signified that no definite period had been stated, the the Magkasanib na Salaysay (August 21, 1970).
The CA correctly concluded that the true agreement of the period to redeem should be ten years from the execution D.
parties vis-à-vis the Kasulatan ng Biling Mabibiling of the contract, pursuant to Articles 1142 and 1144 of the Neither did the petitioners' failure to initiate an action for
Muli was an equitable mortgage, not a pacto de retro sale. Civil Code.25 Thus, the full redemption price should have reformation within ten years from the execution of
There was no dispute that the purported vendors had been paid by July 9, 1955; and upon the expiration of said the Kasulatan ng Biling Mabibiling Muli bar them from
continued in the possession of the property even after the 10-year period, mortgagees Spouses Francia or their insisting on their rights in the property. The records show
execution of the agreement; and that the property had heirs should have foreclosed the mortgage, but they did that the parties in the Kasulatan ng Biling Mabibiling
remained declared for taxation purposes under Leoncia's not do so. Instead, they accepted Alejandro's payments, Muli had abided by their true agreement under the deed,
name, with the realty taxes due being paid by Leoncia, until the debt was fully satisfied by August 11, 1970. to the extent that they and their successors-in-interest still
despite the execution of the agreement. Such established The acceptance of the payments even beyond the 10- deemed the agreement as an equitable mortgage despite
circumstances are among the badges of an equitable year period of redemption estopped the mortgagees' heirs the lapse of 15 years from the execution of the
mortgage enumerated in Article 1602, paragraphs 2 and 5 from insisting that the period to redeem the property had purported pacto de retro sale. Hence, an action for
of the Civil Code, to wit: already expired. Their actions impliedly recognized the reformation of the Kasulatan ng Biling Mabibiling Muli was
Art. 1602. The contract shall be presumed to be an continued existence of the equitable mortgage. The unnecessary, if not superfluous, considering that the
equitable mortgage, in any of the following cases: conduct of the original parties as well as of their reason underlying the requirement for an action for
xxx successors-in-interest manifested that the parties to reformation of instrument has been to ensure that the
(2) When the vendor remains in possession as lessee or the Kasulatan ng Biling Mabibiling Muli really intended parties to a contract abide by their true intended
otherwise; their transaction to be an equitable mortgage, not a pacto agreement.
xxx de retro sale. The Kasulatan ng Pagmeme-ari executed by Alejandro on
(5) When the vendor binds himself to pay the taxes on the In Cuyugan v. Santos,26 the purported buyer under a so- August 21, 1970 was ineffectual to predicate the exclusion
thing sold; called contract to sell with right to repurchase also of the petitioners and their predecessors in interest from
xxx accepted partial payments from the purported seller. We insisting on their claim to the property. Alejandro's being
The existence of any one of the conditions enumerated held that the acceptance of partial payments was an assignee of the mortgage did not authorize him or his
under Article 1602 of the Civil Code, not a concurrence of absolutely incompatible with the idea of irrevocability of heirs to appropriate the mortgaged property for himself
all or of a majority thereof, suffices to give rise to the the title of ownership of the purchaser upon the expiration without violating the prohibition against pactum
presumption that the contract is an equitable of the term stipulated in the original contract for the commissorium contained in Article 2088 of the Civil Code,
mortgage.24 Consequently, the contract between the exercise of the right of redemption. Thereby, the conduct to the effect that "[t]he creditor cannot appropriate the
vendors and vendees (Spouses Francia) was an equitable of the parties manifested that they had intended the things given by way of pledge or mortgage, or dispose of
mortgage. contract to be a mortgage, not a pacto de retro sale. them[;] [a]ny stipulation to the contrary is null and void."
B. C. Aptly did the Court hold in Montevirgen v. Court of
Are the petitioners now barred from claiming that the When Alejandro redeemed the property on August 11, Appeals:27
transaction under the Kasulatan ng Biling Mabibiling 1970, he did not thereby become a co-owner thereof, The declaration, therefore, in the decision of July 1, 1971
Muli was an equitable mortgage by their failure to redeem because his father Jose, Sr. was then still alive. Alejandro to the effect that absolute ownership over the subject
the property for a long period of time? merely became the assignee of the mortgage, and the premises has become consolidated in the respondents
The petitioners contend that prescription, if it must apply property continued to be co-owned by Leoncia and her upon failure of the petitioners to pay their obligation within
to them, should as well be applied to the respondents, sons Jose, Sr., Jose Jr., and Teofilo. As an assignee of the specified period, is a nullity, for consolidation of
who had similarly failed to enforce their right under the the mortgage and the mortgage credit, Alejandro acquired ownership is an improper and inappropriate remedy to
equitable mortgage within ten years from its execution on only the rights of his assignors, nothing more. He himself enforce a transaction declared to be one of mortgage. It is
271
Sales – Chapter 3-12 Cases
the duty of respondents, as mortgagees, to foreclose the The respondents' and the lower courts' positions cannot ownership, acquired ownership of the property through
mortgage if he wishes to secure a perfect title to the be sustained. prescription.
mortgaged property if he buys it in the foreclosure sale. The provisions of the Civil Code governing equitable The Court cannot accept the respondents' posture.
Moreover, the respondents, as Alejandro's heirs, were mortgages disguised as sale contracts, like the one In order that a co-owner's possession may be deemed
entirely bound by his previous acts as their predecessors- herein, are primarily designed to curtail the evils brought adverse to that of the cestui que trust or the other co-
in-interest. Thus, Alejandro's acknowledgment of the about by contracts of sale with right to repurchase, owners, the following elements must concur:
effectivity of the equitable mortgage agreement precluded particularly the circumvention of the usury law and pactum 1. The co-owner has performed unequivocal acts of
the respondents from claiming that the property had been commissorium.29 Courts have taken judicial notice of the repudiation of the co-ownership amounting to an ouster of
sold to him with right to repurchase.28 well-known fact that contracts of sale with right to the cestui que trust or the other co-owners;
E. repurchase have been frequently resorted to in order to 2. Such positive acts of repudiation have been made
What was the effect of the Magkasanib na Salaysay? conceal the true nature of a contract, that is, a loan known to the cestui que trust or the other co-owners;
Both the trial court and the CA declared that secured by a mortgage. It is a reality that grave financial 3. The evidence on the repudiation is clear and
the Magkasanib na Salaysay, which extended the distress renders persons hard-pressed to meet even their conclusive; and
redemption period of the mortgaged property, was basic needs or to respond to an emergency, leaving no 4. His possession is open, continuous, exclusive, and
inefficacious, because the period to redeem could no choice to them but to sign deeds of absolute sale of notorious.33
longer be extended after the original redemption period property or deeds of sale with pacto de retro if only to The concurrence of the foregoing elements was not
had already expired. obtain the much-needed loan from unscrupulous money established herein. For one, Alejandro did not have
In contrast, the petitioners submit that disregarding lenders.30 This reality precisely explains why the pertinent adverse and exclusive possession of the property, as, in
the Magkasanib na Salaysay made no sense,considering provision of the Civil Code includes a peculiar rule fact, the other co-owners had continued to possess it, with
that the respondents' predecessors-in-interest admitted concerning the period of redemption, to wit: Alejandro and his heirs occupying only a portion of it.
therein that the petitioners had a right to redeem the Art. 1602. The contract shall be presumed to be an Neither did the cancellation of the previous tax
property. equitable mortgage, in any of the following cases: declarations in the name of Leoncia, the previous co-
The respondents counter, however, that the Magkasanib xxx owner, and the issuance of a new one in Alejandro's
na Salaysay, which acknowledged the other co-owners' (3)When upon or after the expiration of the right to name, and Alejandro's payment of the realty taxes
right to redeem the property, was void; that the petitioners repurchase another instrument extending the period of constitute repudiation of the co-ownership. The sole fact
could no longer claim to be co-owners entitled to redeem redemption or granting a new period is executed; of a co-owner declaring the land in question in his name
the property, because the co-ownership had come to an xxx for taxation purposes and paying the land taxes did not
end by Alejandro having openly repudiated the co- Ostensibly, the law allows a new period of redemption to constitute an unequivocal act of repudiation amounting to
ownership; that Alejandro's acts of repudiation had be agreed upon or granted even after the expiration of the an ouster of the other co-owner and could not constitute
consisted of: (a) redeeming the property from the Spouses equitable mortgagor's right to repurchase, and treats such adverse possession as basis for title by
Francia; (b) acquiring the property from the heirs of extension as one of the indicators that the true agreement prescription.34Moreover, according to Blatero v.
Spouses Francia by virtue of a deed of assignment between the parties is an equitable mortgage, not a sale Intermediate Appellate Court,35 if a sale a retro is
denominated as Pag-aayos ng Pag-aari at Pagsasalin; (c) with right to repurchase. It was indubitable, therefore, that construed as an equitable mortgage, then the execution of
executing an affidavit of consolidation of ownership over the Magkasanib na Salaysay effectively afforded to an affidavit of consolidation by the purported buyer to
the property (Kasulatan ng Pagmeme-ari); (d) applying for Leoncia, Teofilo, Jose, Sr. and Jose, Jr. a fresh period consolidate ownership of the parcel of land is of no
the cancellation of the tax declaration of property in the within which to pay to Alejandro the redemption price of consequence and the "constructive possession" of the
name of Leoncia, and the subsequent issuance of a new ₱500.00. parcel of land will not ripen into ownership, because only
tax declaration in his name; (e) his continuous possession F. possession acquired and enjoyed in the concept of owner
of the property from 1955, which possession the Did Alejandro and his heirs (respondents herein) acquire can serve as title for acquiring dominion.36
respondents as his heirs had continued up to the present the mortgaged property through prescription? In fine, the respondents did not present proof showing that
time, or for a period of almost 50 years already; and (f) the It is true that Alejandro became a co-owner of the property Alejandro had effectively repudiated the co-ownership.
payment of the taxes by Alejandro and the respondents by right of representation upon the death of his father, Their bare claim that Alejandro had made oral demands to
for more than 30 years without any contribution from the Jose Sr.31 As a co-owner, however, his possession was vacate to his co-owners was self-serving and insufficient.
petitioners; and that such repudiation established that like that of a trustee and was not regarded as adverse to Alejandro's execution of the affidavit of consolidation of
Alejandro and his successors-in-interest had already his co-owners but in fact beneficial to all of them.32 ownership on August 21, 197037 and his subsequent
acquired sole title over the property through acquisitive Yet, the respondents except to the general rule, asserting execution on October 17, 1970 of the joint affidavit 38 were
prescription. that Alejandro, having earlier repudiated the co-
272
Sales – Chapter 3-12 Cases
really equivocal and ambivalent acts that did not manifest c) Finding the Magkakalakip na Salaysay executed on
his desire to repudiate the co-ownership. October 17, 1970 by and among Leoncia Mag-isa Reyes,
The only unequivocal act of repudiation was done by the Jose Reyes, Sr. and Alejandro Reyes valid and effective;
respondents when they filed the instant action for quieting c) Nullifying the Kasulatan ng Pagmeme-ari executed by
of title on September 28, 1994, nearly a year after Alejandro M. Reyes on August 21, 1970; and
Alejandro's death on September 2, 1993. However, their d) Dismissing the petitioners' counterclaim.
possession could not ripen into ownership considering Costs of suit to be paid by the respondents.
that their act of repudiation was not coupled with their SO ORDERED.
exclusive possession of the property.
G.
The respondents can only demand from the petitioners
the partition of the co-owned property and the
reimbursement from their co-owners of the amount
advanced by Alejandro to repay the obligation. They may
also seek from their co-owners the proportional
reimbursement of the realty taxes paid for the property,
pursuant to Article 488 of the Civil Code.39 In the
alternative, they may opt to foreclose the equitable
mortgage, considering that the petitioners' period to
redeem the mortgaged property, which was ten years
from the execution on October 17, 1970 of
the Magkakasanib na Salaysay, had already long lapsed.
We clarify, however, that the respondents may take these
recourses only through the appropriate actions
commenced in court.
H.
The petitioners' counterclaim for damages is dismissed for
their failure to prove their entitlement to it.40
WHEREFORE, we grant the petition for review on
certiorari.
The decision dated July 31, 2002 rendered by the Court of
Appeals is reversed and set aside, and another judgment
is rendered:
a) Upholding the validity of the Kasulatan ng Biling
Mabibiling Muli (Deed of Sale with Right of Repurchase)
executed on July 9, 1955 by Leoncia Mag-isa Reyes and
her sons Teofilo, Jose, Sr. and Jose, Jr., all surnamed
Reyes, in favor of the late Spouses Benedicto Francia and
Monica Ajoco as well as the Pagsasa-ayos ng Pag-aari at
Pagsasalin (Settlement of Estate and Assignment)
executed on August 11, 1970 by the heirs of the late
Spouses Benedicto Francia and Monica Ajoco in favor of
the spouses Alejandro Reyes and Amanda Salonga;
b) Declaring the Kasulatan ng Biling Mabibili Muli to be an
equitable mortgage, not a contract of sale with right to
repurchase;1avvphi1

273
Sales – Chapter 3-12 Cases
G.R. No. 170023 November 27, 2009 without the approval of the Secretary of Agriculture and Deeds to verify the records on file. Tronio ascertained that
KINGS PROPERTIES CORPORATION, Petitioner, Natural Resources; that it shall not be incumbered, OCT No. 535 was clean and had no lien and
vs. alienated, or transferred to any person, corporation, encumbrances. After the necessary verification, petitioner
CANUTO A. GALIDO, Respondent. association, or partnership not qualified to acquire public decided to buy the Antipolo property. 15
DECISION lands under the said Act and its amendments; x x x 6 On 14 March 1995, respondent caused the annotation of
CARPIO, J.: On 10 September 1973, a deed of sale covering the his adverse claim in OCT No. 535. 16
The Case Antipolo property was executed between Rufina Eniceo On 20 March 1995, the Eniceo heirs executed a deed of
Kings Properties Corporation (petitioner) filed this Petition and Maria Eniceo as vendors and respondent as vendee. absolute sale in favor of petitioner covering lots 3 and 4 of
for Review on Certiorari 1 assailing the Court of Appeals’ Rufina Eniceo and Maria Eniceo sold the Antipolo the Antipolo property for ₱500,000. 17
Decision 2 dated 20 December 2004 in CA-G.R. CV No. property to respondent for ₱250,000. 7 A certain Carmen On the same date, Transfer Certificate of Title (TCT) Nos.
68828 as well as the Resolution 3 dated 10 October 2005 Aldana delivered the owner’s duplicate copy of OCT No. 277747 and 277120 were issued. TCT No. 277747
denying the Motion for Reconsideration. In the assailed 535 to respondent. 8 covering lots 1 and 5 of the Antipolo property was
decision, the Court of Appeals reversed the Regional Trial Petitioner alleges that when Maria Eniceo died in June registered in the names of Rufina Eniceo, Ambrosio
Court’s Decision 4 dated 4 July 2000. This case involves 1975, Rufina Eniceo and the heirs of Maria Eniceo Eniceo, Rodolfo Calove, Fernando Calove and Leonila
an action for cancellation of certificates of title, registration (Eniceo heirs), 9 who continued to occupy the Antipolo Calove Bolinas. 18 TCT No. 277120 covering lots 3 and 4
of deed of sale and issuance of certificates of title filed by property as owners, thought that the owner’s duplicate of the Antipolo property was registered in the name of
Canuto A. Galido (respondent) before Branch 71 of the copy of OCT No. 535 was lost. 10 petitioner. 19
Regional Trial Court of Antipolo City (trial court). On 5 April 1988, the Eniceo heirs registered with the On 5 April 1995, the Eniceo heirs executed another deed
The Facts Registry of Deeds of Marikina City (Registry of Deeds) a of sale in favor of petitioner covering lots 1 and 5 of the
On 18 April 1966, the heirs of Domingo Eniceo, namely Notice of Loss dated 2 April 1988 of the owner’s copy of Antipolo property for ₱1,000,000. TCT No. 278588 was
Rufina Eniceo and Maria Eniceo, were awarded with OCT No. 535. The Eniceo heirs also filed a petition for the issued in the name of petitioner and TCT No. 277120 was
Homestead Patent No. 112947 consisting of four parcels issuance of a new owner’s duplicate copy of OCT No. 535 cancelled. 20
of land located in San Isidro, Antipolo, Rizal (Antipolo with Branch 72 of the Regional Trial Court (RTC) of On 17 August 1995, the Secretary of the Department of
property) and particularly described as follows: Antipolo, Rizal. The case was docketed as LRC Case No. Environment and Natural Resources (DENR Secretary)
1. Lot No. 1 containing an area of 96,297 square meters; 584-A. 11 approved the deed of sale between the Eniceo heirs and
Lot No. 3 containing an area of 25,170 square meters; On 31 January 1989, the RTC rendered a decision finding respondent. 21
Lot No. 4 containing an area of 26,812 square meters; that the certified true copy of OCT No. 535 contained no On 16 January 1996, respondent filed a civil complaint
and annotation in favor of any person, corporation or entity. with the trial court against the Eniceo heirs and petitioner.
Lot No. 5 containing an area of 603 square meters. The RTC ordered the Registry of Deeds to issue a second Respondent prayed for the cancellation of the certificates
The Antipolo property with a total area of 14.8882 owner’s copy of OCT No. 535 in favor of the Eniceo heirs of title issued in favor of petitioner, and the registration of
hectares was registered under Original Certificate of Title and declared the original owner’s copy of OCT NO. 535 the deed of sale and issuance of a new transfer certificate
(OCT) No. 535. 5 The issuance of the homestead patent cancelled and considered of no further value. 12 of title in favor of respondent. 22
was subject to the following conditions: On 6 April 1989, the Registry of Deeds issued a second On 4 July 2000, the trial court rendered its decision
To have and to hold the said tract of land, with the owner’s copy of OCT No. 535 in favor of the Eniceo dismissing the case for lack of legal and factual basis. 23
appurtenances thereunto of right belonging unto the said heirs.13 Respondent appealed to the Court of Appeals (CA). On
Heirs of Domingo Eniceo and to his heir or heirs and Petitioner states that as early as 1991, respondent knew 20 December 2004, the CA rendered a decision reversing
assigns forever, subject to the provisions of sections 118, of the RTC decision in LRC Case No. 584-A because the trial court’s decision. 24 Respondent filed a motion for
121, 122 and 124 of Commonwealth Act No. 141, as respondent filed a criminal case against Rufina Eniceo reconsideration, which the CA denied in its Resolution
amended, which provide that except in favor of the and Leonila Bolinas (Bolinas) for giving false testimony dated 10 October 2005.
Government or any of its branches, units or institutions, upon a material fact during the trial of LRC Case No. 584- Aggrieved by the CA’s decision and resolution, petitioner
the land hereby acquired shall be inalienable and shall not A. 14 elevated the case before this Court.
be subject to incumbrance for a period of five (5) years Petitioner alleges that sometime in February 1995, The Ruling of the Trial Court
next following the date of this patent, and shall not be Bolinas came to the office of Alberto Tronio Jr. (Tronio), The trial court stated that although respondent claims that
liable for the satisfaction of any debt contracted prior to petitioner’s general manager, and offered to sell the the Eniceo heirs sold to him the Antipolo property,
the expiration of that period; that it shall not be alienated, Antipolo property. During an on-site inspection, Tronio respondent did not testify in court as to the existence,
transferred or conveyed after five (5) years and before saw a house and ascertained that the occupants were validity and genuineness of the purported deed of sale
twenty-five (25) years next following the issuance of title, Bolinas’ relatives. Tronio also went to the Registry of and his possession of the duplicate owner’s copy of OCT
274
Sales – Chapter 3-12 Cases
No. 535. The trial court stated that as owner of a property The CA pointed out that laches has not set in. One of the WHEREFORE, the appealed decision of the Regional
consisting of hectares of land, respondent should have requisites of laches, which is injury or prejudice to the Trial Court of Rizal (Antipolo, Branch 71) is REVERSED
come to court to substantiate his claim and show that the defendant in the event relief is accorded to the and SET ASIDE and another rendered as follows:
allegations of the Eniceo heirs and petitioner are mere complainant or the suit is not held to be barred, is wanting 1. Declaring null and void Transfer Certificates of Titles
fabrications. 25 in the instant case. The CA added that unrecorded sales Nos. 277747, 277120 and 278588 of the Registry of
The trial court noticed that respondent did not register the of land brought under the Torrens system are valid Deeds of Marikina City (the last two in the name of
deed of sale with the Register of Deeds immediately after between parties because registration of the instrument is defendant-appellee Kings Properties Corporation), the
its alleged execution on 10 September 1973. Further, merely intended to bind third persons. 31 derivative titles thereof and the instruments which were
respondent waited for 22 long years before he had the The CA declared that petitioner’s contention regarding the the bases of the issuance of said certificates of title; and
sale approved by the DENR Secretary. The trial court validity of the questioned deed on the ground that it was 2. Declaring plaintiff-appellant Canuto A. Galido the owner
declared that respondent slept on his rights. The trial court executed without the approval of the DENR Secretary is of fee simple of Lot Nos. 1, 3, 4, 5 formerly registered
concluded that respondent’s failure to register the sale untenable. The DENR Secretary approved the deed of under Original Certificate of Title No. 535 in the name of
and secure the cancellation of OCT No. 535 militates sale on 17 August 1995. However, even supposing that the Heirs of Domingo Eniceo, represented by Rufina
against his claim of ownership. The trial court believed the sale was not approved, the requirement for the DENR Eniceo, and ordering the Register of Deeds of Marikina
that respondent has not established the preponderance of Secretary’s approval is merely directory and its absence City to issue new transfer certificates of title for said
evidence necessary to justify the relief prayed for in his does not invalidate any alienation, transfer or conveyance parcels of land in the name of plaintiff-appellant Canuto A.
complaint. 26 of the homestead after 5 years and before 25 years from Galido, upon payment of the proper fees and presentation
The trial court stated that Bolinas was able to prove that the issuance of the title which can be complied with at any of the deed of sale dated September 10, 1973 executed
the Eniceo heirs have remained in actual possession of time in the future. 32 by Rufina Eniceo and Maria Eniceo, as sole heirs of the
the land. The filing of a petition for the issuance of a new The CA ruled that petitioner is a buyer in bad faith late Domingo Eniceo, in favor of the latter. 36
owner’s duplicate copy requires the posting of the petition because it purchased the disputed properties from the The Issues
in three different places which serves as a notice to the Eniceo heirs after respondent had caused the inscription Petitioner raises two issues in this petition:
whole world. Respondent’s failure to oppose this petition on OCT No. 535 of an adverse claim. Registration of the 1. Whether the adverse claim of respondent over the
can be deemed as a waiver of his right, which is fatal to adverse claim serves as a constructive notice to the whole Antipolo property should be barred by laches; 37 and
his cause. 27 world. Petitioner cannot feign ignorance of facts which 2. Whether the deed of sale delivered to respondent
The trial court noted that petitioner is a buyer in good faith should have put it on guard and then claim that it acted should be presumed an equitable mortgage pursuant to
and for value because petitioner has exercised due under the honest belief that there was no defect in the title Article 1602(2) and 1604 of the Civil Code. 38
diligence in inspecting the property and verifying the title of the vendors. Knowing that an adverse claim was The Ruling of the Court
with the Register of Deeds. 28 annotated in the certificates of title of the Eniceo heirs, Validity of the deed of sale to respondent
The trial court held that even if the court were to believe petitioner was forewarned that someone is claiming an The contract between the Eniceo heirs and respondent
that the deed of sale in favor of respondent were genuine, interest in the disputed properties. 33 executed on 10 September 1973 was a perfected contract
still it could not be considered a legitimate disposition of The CA found no merit in petitioner’s contention that the of sale. A contract is perfected once there is consent of
property, but merely an equitable mortgage. The trial court questioned deed of sale is an equitable mortgage. The CA the contracting parties on the object certain and on the
stated that respondent never obtained possession of the stated that for the presumption of an equitable mortgage cause of the obligation. 39 In the present case, the object
Antipolo property at any given time and a buyer who does to arise, one must first satisfy the requirement that the of the sale is the Antipolo property and the price certain is
not take possession of a property sold to him is presumed parties entered into a contract denominated as a contract ₱250,000.
to be a mortgagee only and not a vendee. 29 of sale and that their intention was to secure an existing The contract of sale has also been consummated
The Ruling of the Court of Appeals debt by way of mortgage. 34 because the vendors and vendee have performed their
The CA ruled that the deed of sale in favor of respondent, The CA stated that the execution of the notarized deed of respective obligations under the contract. In a contract of
being a notarized document, has in its favor the sale, even without actual delivery of the disputed sale, the seller obligates himself to transfer the ownership
presumption of regularity and carries the evidentiary properties, transferred ownership from the Eniceo heirs to of the determinate thing sold, and to deliver the same to
weight conferred upon it with respect to its due execution. respondent. The CA held that respondent’s possession of the buyer, who obligates himself to pay a price certain to
The CA added that whoever asserts forgery has the the owner’s duplicate copy of OCT No. 535 bolsters the the seller. 40 The execution of the notarized deed of sale
burden of proving it by clear, positive and convincing contention that the Eniceo heirs sold the disputed and the delivery of the owner’s duplicate copy of OCT No.
evidence because forgery can never be presumed. The properties to him by virtue of the questioned deed. 35 535 to respondent is tantamount to a constructive delivery
CA found that petitioner and the Eniceo heirs have not The CA reversed the trial court’s decision. The dispositive of the object of the sale. In Navera v. Court of Appeals,
substantiated the allegation of forgery. 30 portion of the CA decision reads: the Court ruled that since the sale was made in a public
275
Sales – Chapter 3-12 Cases
instrument, it was clearly tantamount to a delivery of the apparent or legal grounds for the Secretary to disapprove existing debt by way of mortgage. In fact, mere tolerated
land resulting in the symbolic possession thereof being the sale of the Subject Land. possession is not enough to prove that the transaction
transferred to the buyer. 41 The failure to secure the approval of the Secretary does was an equitable mortgage. 52
Petitioner alleges that the deed of sale is a forgery. The not ipso factomake a sale void. The absence of approval Furthermore, petitioner has not shown any proof that the
Eniceo heirs also claimed in their answer that the deed of by the Secretary does not a sale made after the expiration Eniceo heirs were indebted to respondent. On the
sale is fake and spurious. 42 However, as correctly held by of the 5-year period, for in such event the requirement of contrary, the deed of sale executed in favor of respondent
the CA, forgery can never be presumed. The party Section 118 of the Public Land Act becomes merely was drafted clearly to convey that the Eniceo heirs sold
alleging forgery is mandated to prove it with clear and directory or a formality. The approval may be secured and transferred the Antipolo property to respondent. The
convincing evidence. 43 Whoever alleges forgery has the later, producing the effect of ratifying and adopting the deed of sale even inserted a provision about defrayment
burden of proving it. In this case, petitioner and the Eniceo transaction as if the sale had been previously authorized. of registration expenses to effect the transfer of title to
heirs failed to discharge this burden. (Underscoring supplied) respondent.
Petitioner invokes the belated approval by the DENR Equitable Mortgage In any event, as pointed out by respondent in his
Secretary, made within 25 years from the issuance of the Petitioner contends that the deed of sale in favor of Memorandum, this defense of equitable mortgage is
homestead, to nullify the sale of the Antipolo property. The respondent is an equitable mortgage because the Eniceo available only to petitioner’s predecessors-in-interest who
sale of the Antipolo property cannot be annulled on the heirs remained in possession of the Antipolo property should have demanded, but did not, for the reformation of
ground that the DENR Secretary gave his approval after despite the execution of the deed of sale. the deed of sale. 53 A perusal of the records shows that
21 years from the date the deed of sale in favor of An equitable mortgage is "one which although lacking in the Eniceo heirs never presented the defense of equitable
respondent was executed. Section 118 of Commonwealth some formality, or form or words, or other requisites mortgage before the trial court. In their Answer 54 and
Act No. 141 or the Public Land Act (CA 141), as amended demanded by a statute, nevertheless reveals the intention Memorandum 55 filed before the trial court, the Eniceo
by Commonwealth Act No. 456, 44 reads: of the parties to charge real property as security for a heirs claimed that the alleged deed of sale dated 10
SEC. 118. Except in favor of the Government or any of its debt, and contains nothing impossible or contrary to September 1973 between Rufina Eniceo and Maria
branches, units, or institutions, or legally constituted law." 47 The essential requisites of an equitable mortgage Eniceo was fake and spurious. The Eniceo heirs
banking corporations, lands acquired under free patent or are: contended that even assuming there was a contract, no
homestead provisions shall not be subject to 1. The parties entered into a contract denominated as a consideration was involved. It was only in the Appellees’
encumbrance or alienation from the date of the approval contract of sale; and Brief 56 filed before the CA that the Eniceo heirs claimed
of the application and for a term of five years from and 2. Their intention was to secure existing debt by way of a as an alternative defense that the deed should be
after the date of the issuance of the patent or grant x x x mortgage. 48 presumed as an equitable mortgage.
No alienation, transfer, or conveyance of any homestead In Lim v. Calaguas, 49 the Court held that in order for the In Philippine Ports Authority v. City of Iloilo, 57 we ruled
after five years and before twenty-five years after the presumption of equitable mortgage to apply, there must that a party who adopts a certain theory upon which the
issuance of title shall be valid without the approval of the be: (1) something in the language of the contract; or (2) in case is tried and decided by the lower court will not be
Secretary of Agriculture and Natural Resources, 45 which the conduct of the parties which shows clearly and beyond permitted to change the theory on appeal. A theory of the
approval shall not be denied except on constitutional and doubt that they intended the contract to be a mortgage case not brought to the attention of the lower court will not
legal grounds. and not a pacto de retro sale. 50 Proof by parol evidence be considered by a reviewing court, as a new theory
In Spouses Alfredo v. Spouses Borras, 46 the Court should be presented in court. Parol evidence is admissible cannot be raised for the first time at such late stage.
explained the implications of Section 118 of CA 141. to support the allegation that an instrument in writing, Although petitioner raised the defense of equitable
Thus: purporting on its face to transfer the absolute title to mortgage in the lower court, he cannot claim that the deed
A grantee or homesteader is prohibited from alienating to property, was in truth and in fact given merely as security was an equitable mortgage because petitioner was not a
a private individual a land grant within five years from the for the payment of a loan. The presumption of equitable privy to the deed of sale dated 10 September 1973.
time that the patent or grant is issued. A violation of this mortgage under Article 1602 of the Civil Code is not Petitioner merely stepped into the shoes of the Eniceo
prohibition renders a sale void. This , however, expires on conclusive. It may be rebutted by competent and heirs. Petitioner, who merely acquired all the rights of its
the fifth year. From then on until the next 20 years, the satisfactory proof of the contrary. 51 predecessors, cannot espouse a theory that is contrary to
land grant may be alienated provided the Secretary of Petitioner claims that an equitable mortgage can be the theory of the case claimed by the Eniceo heirs.
Agriculture and Natural Resources approves the presumed because the Eniceo heirs remained in The Court notes that the Eniceo heirs have not appealed
alienation. The Secretary is required to approve the possession of the Antipolo property. Apart from the fact the CA’s decision, hence, as to the Eniceo heirs, the CA’s
alienation unless there are "constitutional and legal that the Eniceo heirs remained in possession of the decision that the contract was a sale and not an equitable
grounds" to deny the approval. In this case, there are no Antipolo property, petitioner has failed to substantiate its mortgage is now final. Since petitioner merely assumed
claim that the contract of sale was intended to secure an the rights of the Eniceo heirs, petitioner is now estopped
276
Sales – Chapter 3-12 Cases
from questioning the deed of sale dated 10 September good faith when they bought and registered the Antipolo which, through due diligence, could have been done
1973. property. earlier, thus giving rise to a presumption that the party
Petitioner is not a buyer in good faith In Carbonell v. Court of Appeals, 63 this Court ruled that in entitled to assert it had either abandoned or declined to
Petitioner maintains that the subsequent sale must be double sales, the first buyer always has priority rights over assert it. 66
upheld because petitioner is a buyer in good faith, having subsequent buyers of the same property. Being the first Respondent discovered in 1991 that a new owner’s copy
exercised due diligence by inspecting the property and the buyer, he is necessarily in good faith compared to of OCT No. 535 was issued to the Eniceo heirs.
title sometime in February 1995. subsequent buyers. The good faith of the first buyer Respondent filed a criminal case against the Eniceo heirs
In Agricultural and Home Extension Development Group remains all throughout despite his subsequent acquisition for false testimony. When respondent learned that the
v. Court of Appeals, 58 a buyer in good faith is defined as of knowledge of the subsequent sale. On the other hand, Eniceo heirs were planning to sell the Antipolo property,
"one who buys the property of another without notice that the subsequent buyer, who may have entered into a respondent caused the annotation of an adverse claim.
some other person has a right to or interest in such contract of sale in good faith, would become a buyer in On 16 January 1996, when respondent learned that OCT
property and pays a full and fair price for the same at the bad faith by his subsequent acquisition of actual or No. 535 was cancelled and new TCTs were issued,
time of such purchase or before he has notice of the claim constructive knowledge of the first sale. 64 The separate respondent filed a civil complaint with the trial court
or interest of some other person in the property." opinion of then Justice Teehankee is instructive, thus: against the Eniceo heirs and petitioner. Respondent’s
In Balatbat v. Court of Appeals, 59 the Court held that in The governing principle here is prius tempore, potior actions negate petitioner’s argument that respondent is
the realm of double sales, the registration of an adverse jure(first in time, stronger in right). Knowledge gained by guilty of laches.
claim places any subsequent buyer of the registered land the first buyer of the second sale cannot defeat the first True, unrecorded sales of land brought under Presidential
in bad faith because such annotation was made in the title buyer’s rights except only as provided by the Code and Decree No. 1529 or the Property Registration Decree (PD
of the property before the Register of Deeds and he could that is where the second buyer first registers in good faith 1529) are effective between and binding only upon the
have discovered that the subject property was already the second sale ahead of the first. Such knowledge of the immediate parties. The registration required in Section 51
sold. 60 The Court explained further, thus: first buyer does bar her from availing of her rights under of PD 1529 is intended to protect innocent third persons,
A purchaser of a valued piece of property cannot just the law, among them, to first her purchase as against the that is, persons who, without knowledge of the sale and in
close his eyes to facts which should put a reasonable man second buyer. But in converso knowledge gained by the good faith, acquire rights to the property. 67 Petitioner,
upon his guard and then claim that he acted in good faith second buyer of the first sale defeats his rights even if he however, is not an innocent purchaser for value.
and under the belief that there were no defect in the title is first to register the second sale, since such knowledge WHEREFORE, we DENY the petition. We AFFIRM the 20
of the vendor. One who purchases real estate with taints his prior registration with bad faith. December 2004 Decision and 10 October 2005
knowledge of a defect or lack of title in his vendor cannot This is the price exacted by Article 1544 of the Civil Code Resolution of the Court of Appeals in CA-G.R. CV No.
claim that he has acquired title thereto in good faith as for the second buyer being able to displace the first buyer: 68828.
against the true owner of the land or of an interest therein; that before the second buyer can obtain priority over the SO ORDERED.
and the same rule must be applied to one who has first, he must show that he acted in good faith throughout
knowledge of facts which should have put him upon such (i.e., in ignorance of the first sale and of the first buyer’s
inquiry and investigation as be necessary to acquaint him rights) – from the time of acquisition until the title is
with the defects in the title of his vendor. 61 transferred to him by registration or failing registration, by
Petitioner does not dispute that respondent registered his delivery of possession. The second buyer must show
adverse claim with the Registry of Deeds on 14 March continuing good faith and innocence or lack of knowledge
1995. The registration of the adverse claim constituted, by of the first sale until his contract ripens into full ownership
operation of law, notice to the whole world. 62 From that through prior registration as provided by law. 65
date onwards, subsequent buyers were deemed to have Laches
constructive notice of respondent’s adverse claim. Petitioner contends that respondent is guilty of laches
Petitioner purchased the Antipolo property only on 20 because he slept on his rights by failing to register the
March 1995 and 5 April 1995 as shown by the dates in the sale of the Antipolo property at the earliest possible time.
deeds of sale. On the same dates, the Registry of Deeds Petitioner claims that despite respondent’s knowledge of
issued new TCTs in favor of petitioner with the annotated the subsequent sale in 1991, respondent still failed to
adverse claim. Consequently, the adverse claim have the deed of sale registered with the Registry of
registered prior to the second sale charged petitioner with Deeds.
constructive notice of the defect in the title of Eniceo heirs. The essence of laches is the failure or neglect, for an
Therefore, petitioner cannot be deemed as a purchaser in unreasonable and unexplained length of time, to do that
277
Sales – Chapter 3-12 Cases
G.R. No. 152168 December 10, 2004 of P1,000,000.00), with the following as co-owners, over 1. P30,000.00 – upon signing today of the document of
HEIRS OF THE LATE SPOUSES AURELIO AND the property described therein: sale.
ESPERANZA BALITE; Namely, ANTONIO T. BALITE, a) Each of the [petitioners] over an undivided portion of 2. P170,000.00 – payable upon completion of the actual
FLOR T. BALITE-ZAMAR, VISITACION T. BALITE- 975 square meters; relocation survey of the land sold by a Geodetic Engineer.
DIFUNTORUM, PEDRO T. BALITE, PABLO T. BALITE, b) The [respondent], with an undivided portion of 9,751 3. P200,000.00 – payable on or before May 15, 1996.
GASPAR T. BALITE, CRISTETA T. BALITE and square meters. 4. P200,000.00 – payable on or before July 15, 1996.
AURELIO T. BALITE JR., All Represented by GASPAR 3. The [respondent] is hereby ordered to pay to the 5. P200,000.00 – payable on or before September 15,
T. BALITE,petitioners, [petitioners] the amount of P120,000.00, within a period of 1996.
vs. five (5) months from the finality of the Decision of this 6. P200,000.00 – payable on or before December 15,
RODRIGO N. LIM, respondent. Court; 1996.
4. In the event that the [respondent] refuses or fails to "Only Esperanza and two of her children, namely, Antonio
remit the said amount to the [petitioner] within the period x x x and Cristeta x x x, knew about the said transaction. x
DECISION therefor, the rights and obligations of the parties shall be x x Geodetic Engineer Bonifacio G. Tasic conducted a
governed by Republic 6552 (Maceda Law)."3 subdivision survey of the property and prepared a "Sketch
The Facts Plan" showing a portion of the property, identified as Lot
PANGANIBAN, J.: The CA summarized the facts in this manner: 243 with an area of 10,000 square meters, under the
A deed of sale that allegedly states a price lower than the "The spouses Aurelio x x x and Esperanza Balite were the name Rodrigo N. Lim.
true consideration is nonetheless binding between the owners of a parcel of land, located [at] Poblacion "The "Sketch Plan" was signed by Rodrigo x x x and
parties and their successors in interest. Furthermore, a (Barangay Molave), Catarman, Northern Samar, with an Esperanza. Thereafter, Rodrigo x x x took actual
deed of sale in which the parties clearly intended to area of seventeen thousand five hundred fifty-one possession of the property and introduced improvements
transfer ownership of the property cannot be presumed to (17,551) square meters, [and] covered by Original thereon. He remitted to Esperanza x x x and Cristeta x x x
be an equitable mortgage under Article 1602 of the Civil Certificate of Title [OCT] No. 10824. When Aurelio died sums of money in partial payments of the x x x property
Code. Finally, an agreement that purports to sell in metes intestate [in 1985, his wife], Esperanza Balite, and their for which he signed "Receipts".
and bounds a specific portion of an unpartitioned co- children, x x x [petitioners] Antonio Balite, Flor Balite- "Gaspar, Visitacion, Flor, Pedro and Aurelio, Jr. x x x
owned property is not void; it shall effectively transfer the Zamar, Visitacion Balite-Difuntorum, Pedro Balite, Pablo learned of the sale, and on August 21, 1996, they wrote a
seller’s ideal share in the co-ownership. Balite, Gaspar Balite, Cristeta (Tita) Balite and Aurelio letter to the Register of Deeds [RD] of Northern Samar,
The Case Balite, Jr., inherited the [subject] property and became co- [saying] that they [were] not x x x informed of the sale of a
Before us is a Petition for Review1 under Rule 45 of the owners thereof, with Esperanza x x x inheriting an portion of the said property by their mother x x x nor did
Rules of Court, assailing the February 11, 2002 undivided [share] of [9,751] square meters. they give their consent thereto, and requested the [RD] to:
Decision2 of the Court of Appeals (CA) in CA-GR CV No. "In the meantime, Esperanza x x x [became] ill and was in "x x x hold in abeyance any processal or approval of any
65395. The decretal portion of the Decision reads as dire need of money for her hospital expenses x x x. She, application for registration of title of ownership in the name
follows: through her daughter, Cristeta, offered to sell to Rodrigo of the buyer of said lot, which has not yet been partitioned
"IN THE LIGHT OF ALL THE FOREGOING, the Decision Lim, [her] undivided share x x x for the price judicially or extrajudicially, until the issue of the
of the Court a quo subject of the appeal is hereby SET of P1,000,000.00. x x x Esperanza x x x and Rodrigo x x x legality/validity of the above sale has been cleared."
ASIDE AND REVERSED and another Decision is hereby agreed that, under the "Deed of Absolute Sale", to be "On August 24, 1996, Antonio x x x received from Rodrigo
rendered as follows: executed by Esperanza x x x over the property, it will be x x x, the amount of P30,000.00 in partial payment of [the]
1. The "Deed of Absolute Sale" (Exhibit "A") is valid made to appear that the purchase price of the property property and signed a "Receipt" for the said amount,
only insofar as the pro indiviso share of Esperanza Balite would be P150,000.00, although the actual price agreed declaring therein that "the remaining balance
over the property covered by Original Certificate of Title upon by them for the property was P1,000,000.00. of P350,000.00 shall personally and directly be released
No. 10824 is concerned; "On April 16, 1996, Esperanza x x x executed a "Deed of to my mother, Esperanza Balite, only." However, Rodrigo
2. The Register of Deeds is hereby ordered to cancel Absolute Sale" in favor of Rodrigo N. Lim over a portion of x x x drew and issued RCBC Check No. 309171, dated
Transfer Certificate of Title No. 6683 and to issue another the property, covered by [OCT] No. 10824, with an area of August 26, 1996, [payable] to the order of Antonio Balite
over the entirety of the property covered by Original 10,000 square meters, for the price of P150,000.00 x x x. in the amount of P30,000.00 in partial payment of the
Certificate of Title No. 10824, upon the payment of the [They] also executed, on the same day, a "Joint Affidavit" property.
capital gains tax due, as provided for by law, (based on under which they declared that the real price of the "On October 1, 1996, Esperanza x x x executed a
the purchase price of the property in the amount property was P1,000,000.00, payable to Esperanza x x x, "Special Power of Attorney" appointing her son, Antonio,
by installments, as follows: to collect and receive, from Rodrigo, the balance of the
278
Sales – Chapter 3-12 Cases
purchase price of the x x x property and to sign the "Subsequently, Rodrigo secured a loan from the Rizal The CA likewise rejected petitioners’ claim that the sale
appropriate documents therefor. Commercial Banking Corporation in the amount was void allegedly because the actual purchase price of
"On October 23, 1996, Esperanza signed a letter of P2,000,000.00 and executed a "Real Estate Mortgage" the property was not stated in the Deed of Absolute Sale.
addressed to Rodrigo informing the latter that her children over the [subject] property as security therefor. It found that the true and correct consideration for the sale
did not agree to the sale of the property to him and that "On motion of the [petitioners], they were granted x x x was P1,000,000 as declared by Esperanza and
she was withdrawing all her commitments until the validity leave to file an "Amended Complaint" impleading the bank respondent in their Joint Affidavit. Applying Article 13535 of
of the sale is finally resolved: as [additional] party-defendant. On November 26, 1997, the Civil Code, it held that the falsity of the price or
xxx xxx xxx [petitioners] filed their "Amended Complaint". consideration stated in the Deed did not render it void.
"On October 31, 1996, Esperanza died intestate and was The [respondent] opposed the "Amended Complaint" x x x The CA pointed out, however, that the State retained the
survived by her aforenamed children. contending that it was improper for [petitioners] to join, in right to recover the capital gains tax based on the true
"[Meanwhile], Rodrigo caused to be published, in the their complaint, an ordinary civil action for the nullification price of P1,000,000.
Samar Reporter, on November 14, 21 and 28, 1996, the of the "Real Estate Mortgage" executed by the respondent The appellate court rejected petitioners’ contention that,
aforesaid "Deed of Absolute Sale". Earlier, on November in favor of the Bank as the action of the petitioners before because of the allegedly unconscionably low and
21, 1996, Antonio received the amount of P10,000.00 the court was a special civil action. inadequate consideration involved, the transaction
from Rodrigo for the payment of the estate tax due from "On March 30, 1998, the court issued an Order rejecting covered by the Deed was an equitable mortgage under
the estate of Esperanza. the "Amended Complaint" of the petitioners on the Article 1602 of the Civil Code. Observing that the
"Also, the capital gains tax, in the amount of P14,506.25, grounds that: (a) the Bank cannot be impleaded as party- argument had never been raised in the court a quo, it
based on the purchase price of P150,000.00 appearing on defendant under Rule 63, Section 1 of the 1997 Rules of ruled that petitioners were proscribed from making this
the "Deed of Absolute Sale", was paid to the Bureau of Civil Procedure; (b) the "Amended Complaint" constituted claim, for the first time, on appeal.
Internal Revenue which issued a "Certification" of said a collateral attack on TCT No. 6683. The [petitioners] did The CA further held that the remaining liability of
payments, on March 5, 1997, authorizing the registration not file any motion for the reconsideration of the order of respondent was P120,000. It relied on the Receipt dated
of the "Deed of Absolute Sale" x x x. However, the [RD] the court."4 August 24, 1996, which stated that his outstanding
refused to issue a title over the property to and under the The trial court dismissed the Complaint and ordered the balance for the consideration was P350,000. It deducted
name of Rodrigo unless and until the owner’s duplicate of cancellation of the lis pendens annotated at the back of therefrom the amounts of P30,000 received by Antonio on
OCT No. 10824 was presented to [it]. Rodrigo filed a TCT No. 6683. It held that, pursuant to Article 493 of the August 27, 1996; and P200,000, which was the amount of
"Petition for Mandamus" against the RD with the Regional Civil Code, a co-owner has the right to sell his/her the check dated September 15, 1996, issued by
Trial Court of Northern Samar (Rodrigo Lim versus undivided share. The sale made by a co-owner is not respondent payable to Esperanza.
Fernando Abella, Special Civil Case No. 48). x x x. On invalidated by the absence of the consent of the other co- Finally, the appellate court noted that the mortgage over
June 13, 1997, the court issued an Order to the RD to owners. Hence, the sale by Esperanza of the 10,000- the property had been executed after the filing of the
cancel OCT No. 10824 and to issue a certificate of title square-meter portion of the property was valid; the excess Complaint. What petitioners should have filed was a
over Lot 243 under the name of Rodrigo. from her undivided share should be taken from the supplemental complaint instead of an amended complaint.
"On June 27, 1997, [petitioners] filed a complaint against undivided shares of Cristeta and Antonio, who expressly Contrary to respondent’s argument, it also held that the
Rodrigo with the Regional Trial Court of Northern Samar, agreed to and benefited from the sale. bank was not an indispensable party to the case; but was
entitled and docketed as "Heirs of the Spouses Aurelio Ruling of the Court of Appeals merely a proper party. Thus, there is no necessity to
Balite, et al. versus Rodrigo Lim, Civil Case No. 920, The CA held that the sale was valid and binding insofar as implead it as party-defendant, although the court a
for "Annulment of Sale, Quieting of Title, Injunction Esperanza Balite’s undivided share of the property was quo had the option to do so. And even if it were not
and Damages x x x, [the origin of the instant case.] concerned. It affirmed the trial court’s ruling that the lack impleaded, the appellate court ruled that the bank would
xxx xxx xxx of consent of the co-owners did not nullify the sale. The still have been bound by the outcome of the case, as the
"The [petitioners] had a "Notice of Lis Pendens", dated buyer, respondent herein, became a co-owner of the latter was a mortgagee pendente lite over real estate that
June 23, 1997, annotated, on June 27, 1997, at the dorsal property to the extent of the pro indiviso share of the was covered by a certificate of title with an annotated lis
portion of OCT No. 10824. vendor, subject to the portion that may be allotted to him pendens.
"In the meantime, the RD cancelled, on July 10, 1997, upon the termination of the co-ownership. The appellate Hence, this Petition.6
OCT No. 10824 and issued Transfer Certificate of Title court disagreed with the averment of petitioners that the Issues
[TCT] No. 6683 to and under the name of Rodrigo over registration of the sale and the issuance of TCT No. 6683 In their Memorandum, petitioners present the following
Lot 243. The "Notice of Lis Pendens" x x x was carried was ineffective and that they became the owners of the issues:
over in TCT No. 6683. share of Esperanza upon the latter’s death. "A

279
Sales – Chapter 3-12 Cases
"Whether or not the [CA] seriously erred in not deciding The Petition has no merit. agreement is absolutely binding and
that the Deed of Absolute Sale dated April 16, 1996 is null First Issue: enforceable12 between the parties and their successors in
and void on the grounds that it is falsified; it has an Validity of the Sale interest.
unlawful cause; and it is contrary to law and/or public Petitioners contend that the Deed of Absolute Sale is null Petitioners cannot be permitted to unmake the Contract
policy. and void, because the undervalued consideration voluntarily entered into by their predecessor, even if the
"B indicated therein was intended for an unlawful purpose -- stated consideration was included therein for an unlawful
"Whether or not the [CA] gravely erred in not finding that to avoid the payment of higher capital gains taxes on the purpose. "The binding force of a contract must be
the amount paid by [respondent] is only three hundred transaction. According to them, the appellate court’s recognized as far as it is legally possible to do
twenty thousand (P320,000.00) pesos and that reliance on Article 1353 of the Civil Code was erroneous. so."13 However, as properly held by the appellate court,
respondent’s claim that he has paid one million pesos They further contend that the Joint Affidavit is not proof of the government has the right to collect the proper taxes
except P44,000.00 as balance, is fraudulent and false. a true and lawful cause, but an integral part of a scheme based on the correct purchase price.
"C to evade paying lawful taxes and registration fees to the Being onerous, the Contract had for its cause or
"Whether or not the [CA] seriously erred in not deciding government. consideration the price of P1,000,000. Both this
that at the time the Deed of Sale was registered x x x on We have before us an example of a simulated contract. consideration as well as the subject matter of the contract
May 30, 1997, said Deed of Sale can no longer bind the Article 1345 of the Civil Code provides that the simulation -- Esperanza’s share in the property covered by OCT No.
property covered by OCT No. 10824 because said land of a contract may either be absolute or relative. In 10824 -- are lawful. The motives of the contracting parties
had already become the property of all the petitioners absolute simulation, there is a colorable contract but for lowering the price of the sale -- in the present case, the
upon the death of their mother on October 31, 1996 and without any substance, because the parties have no reduction of capital gains tax liability -- should not be
therefore such registration is functus of[f]icio involving a intention to be bound by it. An absolutely simulated confused with the consideration.14 Although illegal, the
null and void document. contract is void, and the parties may recover from each motives neither determine nor take the place of the
"D other what they may have given under the "contract."8 On consideration. 15
"Whether or not the [CA] seriously erred in not ruling that the other hand, if the parties state a false cause in the Deed of Sale not an
petitioners’ amended complaint dated November 27, 1997 contract to conceal their real agreement, such a contract Equitable Mortgage
was proper and admissible and deemed admitted to is relatively simulated. Here, the parties’ real agreement Petitioner further posits that even assuming that the deed
conform to evidence presented. binds them.9 of sale is valid it should only be deemed an equitable
"E In the present case, the parties intended to be bound by mortgage pursuant to Articles 1602 and 1604 of the Civil
"Whether or not the [CA] seriously erred in not declaring the Contract, even if it did not reflect the actual purchase Code, because the price was clearly inadequate. They
that TCT No. T-6683 in the name of Respondent Rodrigo price of the property. That the parties intended the add that the presence of only one of the circumstances
N. Lim is null and void and all dealings involving the same agreement to produce legal effect is revealed by the letter enumerated under Article 1602 would be sufficient to
are likewise null and void and/or subject to the decision of of Esperanza Balite to respondent dated October 23, consider the Contract an equitable mortgage. We
the case at bar in view of the notice of lis pendens 199610 and petitioners’ admission that there was a partial disagree.
annotated therein. payment of P320,000 made on the basis of the Deed of For Articles 1602 and 1604 to apply, two requisites must
"F Absolute Sale. There was an intention to transfer the concur: one, the parties entered into a contract
"Even assuming but without admitting that the Deed of ownership of over 10,000 square meters of the property . denominated as a contract of sale; and, two, their
Sale is enforceable, the respondent court seriously erred Clear from the letter is the fact that the objections of her intention was to secure an existing debt by way of
in not deciding that the consideration is unconscionably children prompted Esperanza to unilaterally withdraw from mortgage.16
low and inadequate and therefore the transaction between the transaction. Indeed, the existence of any of the circumstances
the executing parties constitutes an equitable mortgage. Since the Deed of Absolute Sale was merely relatively enumerated in Article 1602, not a concurrence or an
"G simulated, it remains valid and enforceable. All the overwhelming number thereof, suffices to give rise to the
"The [CA] greatly erred in not rendering judgment essential requisites prescribed by law for the validity and presumption that a contract purporting to be an absolute
awarding damages and attorney’s fee[s] in favor of perfection of contracts are present. However, the parties sale is actually an equitable mortgage. 17 In the present
petitioners among others."7 shall be bound by their real agreement for a consideration case, however, the Contract does not merely purport to be
In sum, the issues raised by petitioners center on the of P1,000,000 as reflected in their Joint Affidavit.11 an absolute sale. The records and the documentary
following: 1) whether the Deed of Absolute Sale is valid, The juridical nature of the Contract remained the same. evidence introduced by the parties indubitably show that
and 2) whether there is still any sum for which respondent What was concealed was merely the actual price. Where the Contract is, indeed, one of absolute sale. There is no
is liable. the essential requisites are present and the simulation clear and convincing evidence that the parties agreed
The Court’s Ruling refers only to the content or terms of the contract, the upon a mortgage of the subject property.
280
Sales – Chapter 3-12 Cases
Furthermore, the voluntary, written and unconditional Hence, the transaction between Esperanza Balite and Although the factual findings of the two lower courts were
acceptance of contractual commitments negates the respondent could be legally recognized only in respect to not identical, we hold that in the present case, the findings
theory of equitable mortgage. There is nothing doubtful the former’s pro indiviso share in the co-ownership. As a of the CA are in accord with the documents on record.
about the terms of, or the circumstances surrounding, the matter of fact, the Deed of Absolute Sale executed The trial court admitted in evidence the August 24, 1996
Deed of Sale that would call for the application of Article between the parties expressly referred to the 10,000- Receipt signed by Antonio Balite. Interestingly, he was
1602. The Joint Affidavit indisputably confirmed that the square-meter portion of the land sold to respondent as the never presented in the lower court to dispute the veracity
transaction between the parties was a sale. share of Esperanza in the conjugal property. Her clear of the contents of that Receipt, particularly the second
When the words of a contract are clear and readily intention was to sell merely her ideal or undivided share in paragraph that had categorically stated the outstanding
understandable, there is no room for construction. it. No valid objection can be made against that intent. balance of respondent as of August 24, 1996, to
Contracts are to be interpreted according to their literal Clearly then, the sale can be given effect to the extent of be P350,000. Furthermore, the evidence shows that
meaning and should not be interpreted beyond their 9,751 square meters, her ideal share in the property as subsequent payments of P30,000 and P200,000 were
obvious intendment.18The contract is the law between the found by both the trial and the appellate courts. made by the latter. Thus, we affirm the CA’s Decision
parties. Transfer of Property holding that the remaining unpaid balance of the price
Notably, petitioners never raised as an issue before the During her lifetime, Esperanza had already sold to was P120,000.
trial court the fact that the document did not express the respondent her share in the subject parcel; hence her WHEREFORE, the Petition is DENIED and the assailed
true intent and agreement of the contracting parties. They heirs could no longer inherit it. The property she had Decision AFFIRMED. Costs against the petitioners.
raised mere suppositions on the inadequacy of the price, transferred or conveyed no longer formed part of her SO ORDERED.
in support of their argument that the Contract should be estate to which her heirs may lay claim at the time of her
considered as an equitable mortgage. death. The transfer took effect on April 16, 1996 (the date
We find no basis to conclude that the purchase price of the Deed of Absolute Sale was executed), and not on May
the property was grossly inadequate. Petitioners did not 30, 1997, when the Deed of Absolute Sale was registered.
present any witness to testify as to the market values of Thus, petitioners’ claim that the property became theirs
real estate in the subject’s locale. They made their claim upon the death of their mother is untenable.
on the basis alone of the P2,000,000 loan that respondent Second Issue:
had been able to obtain from the Rizal Commercial Respondent’s Liability
Banking Corporation. This move did not sufficiently show Petitioners insist that the appellate court erred in holding
the alleged inadequacy of the purchase price. A mortgage that respondent’s outstanding liability on the Deed of Sale
is a mere security for a loan. There was no showing that was P120,000, when the Receipts on record show
the property was the only security relied upon by the bank; payments in the total amount of P320,000 only. They
or that the borrowers had no credit worthiness, other than argue that the August 24, 1996 Receipt, on which the
the property offered as collateral. appellate court based its conclusion, was unreliable.
Co-Ownership To begin with, this Court is not a trier of facts. 21 It is not its
The appellate court was correct in affirming the validity of function to examine and determine the weight of the
the sale of the property insofar as the pro indiviso share of evidence. Well-entrenched is the doctrine that only errors
Esperanza Balite was concerned. of law,22 and not of facts, are reviewable by this Court in a
Article 493 of the Civil Code19 gives the owner of an petition for review on certiorari under Rule 45 of the
undivided interest in the property the right to freely sell Revised Rules of Court. Philippine Airlines, Inc. v. Court of
and dispose of such interest. The co-owner, however, has Appeals23 has held that factual findings of the Court of
no right to sell or alienate a specific or determinate part of Appeals are binding and conclusive upon the Supreme
the thing owned in common, because such right over the Court. These findings may be reviewed24 only under
thing is represented by an aliquot or ideal portion without exceptional circumstances such as, among others, when
any physical division. Nonetheless, the mere fact that the the inference is manifestly mistaken;25 the judgment is
deed purports to transfer a concrete portion does not per based on a misapprehension of facts;26 findings of the trial
se render the sale void.20 The sale is valid, but only with court contradict those of the CA;27 or the CA manifestly
respect to the aliquot share of the selling co-owner. overlooked certain relevant and undisputed facts that, if
Furthermore, the sale is subject to the results of the properly considered, would justify a different conclusion. 28
partition upon the termination of the co-ownership.
281
Sales – Chapter 3-12 Cases
G.R. No. 170509 June 27, 2012 that she is now the owner of the subject premises as she 85664.12 The CA, in its decision of June 21, 2005,
VIEGELY SAMELO, represented by Attorney-in-Fact had been in possession since 1944.7 reversed and set aside the RTC decision, and reinstated
CRISTINA SAMELO, Petitioner, The MeTC Ruling the MeTC judgment. The CA held that the petitioner is
vs. The MeTC, in its judgment8 of March 28, 2002, decided in now estopped from questioning the right of the
MANOTOK SERVICES, INC., allegedly represented by favor of the respondent, and ordered the petitioner to respondent over the subject property. It explained that in
PERPETUA BOCANEGRA (deceased), Respondent. vacate the subject premises and to deliver their peaceful an action involving the possession of the subject
DECISION possession to the respondent. The MeTC held that the premises, a tenant cannot controvert the title of his
BRION, J.: only issue to be resolved in an unlawful detainer case is landlord or assert any rights adverse to that title, without
Before us is the petition for review on certiorari 1 filed by physical possession or possession de facto, and that the first delivering to the landlord the premises acquired by
Viegely Samelo (petitioner), represented by her attorney- respondent had established its right of possession over virtue of the agreement between themselves. The
in-fact Cristina Samelo, to challenge the decision dated the subject premises. It added that the petitioner’s right appellate court added that the petitioner cannot claim that
June 21, 20052 and the resolution dated November 10, under the lease contract already ceased upon the she repudiated the lease contract, in the absence of any
20053of the Court of Appeals (CA) in CA-G.R. SP No. expiration of the said contract. It further ruled that the unequivocal acts of repudiation.
85664. petitioner is already estopped from questioning the right of The CA further held that the only issue in an ejectment
Background Facts the respondent over the subject premises when she suit is physical or material possession, although the trial
Manotok Services, Inc. (respondent) alleged that it is the entered into a contract of lease with the respondent. The courts may provisionally resolve the issue of ownership for
administrator of a parcel of land known as Lot 9-A, Block dispositive portion of the MeTC judgment reads: the sole purpose of determining the issue of possession. It
2913, situated at 2882 Dagupan Extension, Tondo, WHEREFORE, premises considered, judgment is hereby explained that the issue of ownership is not required to
Manila. On January 31, 1997, the respondent entered into rendered for the plaintiff and against defendant, ordering determine the issue of possession since the petitioner
a contract with the petitioner for the lease of a portion of the latter and all persons claiming rights under her: tacitly admitted that she is a lessee of the subject
Lot 9-A, Block 2913, described as Lot 4, Block 15 (subject 1. To vacate the premises located at 2882 Dagupan premises.13
premises). The lease contract was for a period of one (1) Extension, Tondo, Manila, and deliver the peaceful The petitioner moved to reconsider this decision, but the
year, with a monthly rental of ₱3,960.00. After the possession thereof to the plaintiff[;] CA denied her motion in its resolution dated November
expiration of the lease contract on December 31, 1997, 2. To pay plaintiff the sum of ₱40,075.20 as compensation 10, 2005.14
the petitioner continued occupying the subject premises for the use and occupancy of the premises from January In presenting her case before this Court, the petitioner
without paying the rent.4 On August 5, 1998, the 1, 1998 to August 30, 1998, plus ₱4,554.00 a month argued that the CA erred in ruling that a tenant is not
respondent, thru its President Rosa Manotok, sent a letter starting September 1, 1998, until defendant and all permitted to deny the title of his landlord. She maintained
to the petitioner demanding that she vacate the subject person[s] claiming rights under her to finally vacate the that the respondent is not the owner or administrator of
premises and pay compensation for its use and premises[;] the subject premises, and insisted that she had been in
occupancy.5 The petitioner, however, refused to heed 3. To pay plaintiff the sum of ₱5,000.00 for and as possession of the land in question since 1944. She further
these demands. attorney’s fees; and added that she repudiated the lease contract by filing a
On November 18, 1998, the respondent filed a complaint 4. To pay the cost of suit.9 case for fraudulent misrepresentation, intimidation,
for unlawful detainer against the petitioner before the The RTC Decision annulment of lease contract, and quieting of title with
Metropolitan Trial Court (MeTC), Branch 3, Manila.6 The The petitioner filed an appeal10 with the Regional Trial injunction before another court.15
case was docketed as Civil Case No. 161588-CV. The Court (RTC), Branch 50, Manila. The RTC, in its The Court’s Ruling
respondent prayed, among others, that the petitioner and decision11 of July 1, 2004, set aside the MeTC’s decision, We find the petition unmeritorious.
those claiming rights under her be ordered to vacate the and dismissed the complaint for unlawful detainer. The Respondent has a better right of possession over the
subject premises, and to pay compensation for its use and RTC held, among others, that the respondent had no right subject premises
occupancy. to collect rentals as it failed to show that it had authority to "An action for unlawful detainer exists when a person
In her answer, the petitioner alleged that the respondent administer the subject premises and to enter into a unlawfully withholds possession of any land or building
had no right to collect rentals because the subject contract of lease with the petitioner. It also ruled that the against or from a lessor, vendor, vendee or other persons,
premises are located inside the property of the Philippine subject premises, which were formerly owned by the PNR, after the expiration or termination of the right to hold
National Railways (PNR). She also added that the are now owned by the petitioner by virtue of her possession, by virtue of any contract, express or
respondent had no certificate of title over the subject possession and stay in the premises since 1944. implied."16 "The only issue to be resolved in an unlawful
premises. The petitioner further claimed that her signature The CA Decision detainer case is physical or material possession of the
in the contract of lease was obtained through the Aggrieved by the reversal, the respondent filed a petition property involved, independent of any claim of ownership
respondent’s misrepresentation. She likewise maintained for review with the CA, docketed as CA-G.R. SP No. by any of the parties involved."17 "Thus, when the
282
Sales – Chapter 3-12 Cases
relationship of lessor and lessee is established in an week to week, if the rent is weekly; and from day to day, if right of possession to the subject premises than the
unlawful detainer case, any attempt of the parties to inject the rent is to be paid daily. lessee.
the question of ownership into the case is futile, except Since the rent was paid on a monthly basis, the period of The Court thus explained in Tamio v. Ticson:25
insofar as it might throw light on the right of possession."18 lease is considered to be from month to month, in Indeed, the relation of lessor and lessee does not depend
In the present case, it is undisputed that the petitioner and accordance with Article 1687 of the Civil Code. "[A] lease on the former’s title but on the agreement between the
the respondent entered into a contract of lease. We note from month to month is considered to be one with a parties, followed by the possession of the premises by the
in this regard that in her answer with affirmative defenses definite period which expires at the end of each month lessee under such agreement. As long as the latter
and counterclaim before the MeTC, the petitioner did not upon a demand to vacate by the lessor." 21 When the remains in undisturbed possession, it is immaterial
deny that she signed the lease contract (although she respondent sent a notice to vacate to the petitioner on whether the lessor has a valid title – or any title at all – at
maintained that her signature was obtained through the August 5, 1998, the tacita reconduccion was aborted, and the time the relationship was entered into. [citations
respondent’s misrepresentations). Under the lease the contract is deemed to have expired at the end of that omitted]
contract, the petitioner obligated herself to pay a monthly month. "[A] notice to vacate constitutes an express act on The issue of ownership
rental to the respondent in the amount of ₱3,960.00. The the part of the lessor that it no longer consents to the We are likewise unpersuaded by the petitioner’s claim that
lease period was for one year, commencing on January 1, continued occupation by the lessee of its property." 22 After she has "acquired possessory rights leading to
1997 and expiring on December 31, 1997. It bears such notice, the lessee’s right to continue in possession ownership"26 over the subject premises, having been in
emphasis that the respondent did not give the petitioner a ceases and her possession becomes one of detainer.23 possession thereof since 1944. We emphasize that aside
notice to vacate upon the expiration of the lease contract Estoppel of tenant from her self-serving allegation, the petitioner did not
in December 1997 (the notice to vacate was sent only on We find no merit in the petitioner’s allegation that the present any documentary evidence to substantiate her
August 5, 1998), and the latter continued enjoying the respondent had no authority to lease the subject premises claim that she stayed on the subject premises since 1944.
subject premises for more than 15 days, without objection because the latter failed to prove that it is its owner or That the petitioner presented certificates of title of the
from the respondent. By the inaction of the respondent as administrator. Manila Railroad Company over certain properties in
lessor, there can be no inference that it intended to The Rules of Court protects the respondent, as lessor, Tondo, Manila, which allegedly cover the subject
discontinue the lease contract.19 An implied new lease from being questioned by the petitioner, as lessee, premises, is of no moment. One cannot recognize the
was therefore created pursuant to Article 1670 of the Civil regarding its title or better right of possession over the right of another, and at the same time claim adverse
Code, which expressly provides: subject premises. Section 2(b), Rule 131 of the Rules of possession which can ripen to ownership, thru acquisitive
Article 1670. If at the end of the contract the lessee should Court states that the tenant is not permitted to deny the prescription. "For prescription to set in, the possession
continue enjoying the thing leased for fifteen days with the title of his landlord at the time of the commencement of must be adverse, continuous, public, and to the exclusion
acquiescence of the lessor, and unless a notice to the the relation of landlord and tenant between them. Article of [others]."27 Significantly, the RTC decision failed to state
contrary by either party has previously been given, it is 1436 of the Civil Code likewise states that a lessee or a its basis for concluding that the petitioner stayed in the
understood that there is an implied new lease, not for the bailee is estopped from asserting title to the thing leased subject premises since 1944.
period of the original contract, but for the time established or received, as against the lessor or bailor. At any rate, we hold that no need exists to resolve the
in Articles 1682 and 1687. The other terms of the original These provisions bar the petitioner from contesting the issue of ownership in this case, since it is not required to
contract shall be revived. respondent’s title over the subject premises. "The juridical determine the issue of possession; the execution of the
"An implied new lease or tacita reconduccion will set in relationship between x x x [a] lessor and x x x [a lessee] lease contract between the petitioner, as lessee, and the
when the following requisites are found to exist: a) the carries with it a recognition of the lessor's title. As [lessee, respondent, as lessor, belies the former’s claim of
term of the original contract of lease has expired; b) the the petitioner is] estopped [from denying the] landlord's ownership.1âwphi1 We reiterate that the fact of the lease
lessor has not given the lessee a notice to vacate; and c) title, or to assert a better title not only in [herself], but also and the expiration of its term are the only elements in an
the lessee continued enjoying the thing leased for fifteen in some third person while [she remains] in possession of action for unlawful detainer. "The defense of ownership
days with the acquiescence of the lessor."20 As earlier the subject premises and until [she surrenders] does not change the summary nature of [this] action. x x
discussed, all these requisites have been fulfilled in the possession to the landlord. This estoppel applies even x. Although a wrongful possessor may at times be upheld
present case. though the lessor had no title at the time the relation of by the courts, this is merely temporary and solely for the
Article 1687 of the Civil Code on implied new lease [the] lessor and [the] lessee was created, and may be maintenance of public order. The question of ownership is
provides: asserted not only by the original lessor, but also by those to be settled in the proper court and in a proper action." 28
Article 1687. If the period for the lease has not been fixed, who succeed to his title."24 Once a contact of lease is Interest on rentals due
it is understood to be from year to year, if the rent agreed shown to exist between the parties, the lessee cannot by Additionally, the petitioner is liable to pay interest by way
upon is annual; from month to month, if it is monthly; from any proof, however strong, overturn the conclusive of damages for her failure to pay the rentals due for the
presumption that the lessor has a valid title to or a better use of the subject premises.29 We reiterate that the
283
Sales – Chapter 3-12 Cases
respondent’s extrajudicial demand on the petitioner was
made on August 5, 1998. Thus, from this date, the rentals
due from the petitioner shall earn interest at 6% per
annum, until the judgment in this case becomes final and
executory. After the finality of judgment, and until full
payment of the rentals and interests due, the legal rate of
interest to be imposed shall be 12%.
WHEREFORE, in light of all the foregoing, we DENY the
petition. The decision and the resolution of the Court of
Appeals dated June 21, 2005 and November 10, 2005,
respectively, in CA-G.R. SP No. 85664 are AFFIRMED
with the MODIFICATION that the unpaid rentals shall earn
a corresponding interest of six percent (6%) per annum, to
be computed from August 5, 1998 until the finality of this
decision. After this decision becomes final and executory,
the rate of legal interest shall be computed at twelve
percent (12%) per annum from such finality until its
satisfaction.
SO ORDERED.

284
Sales – Chapter 3-12 Cases
G.R. No. 122544 January 28, 2003 Court. This notwithstanding, the cases were set for oral therein is through an agent, the authority of the latter shall
REGINA P. DIZON, AMPARO D. BARTOLOME, argument on March 21, 2001, on the following issues: be in writing; otherwise, the sale shall be void."
FIDELINA D. BALZA, ESTER ABAD DIZON and 1. WHETHER THERE ARE CIRCUMSTANCES THAT When the sale of a piece of land or any interest thereon is
JOSEPH ANTHONY DIZON, RAYMUND A. DIZON, WOULD JUSTIFY SUSPENSION OF THE RULES OF through an agent, the authority of the latter shall be in
GERARD A. DIZON and JOSE A. DIZON, COURT; writing; otherwise, the sale shall be void. Thus the
JR., petitioners, 2. WHETHER THE SUM OF P300,000.00 RECEIVED BY authority of an agent to execute a contract for the sale of
vs. ALICE DIZON FROM PRIVATE RESPONDENT WAS real estate must be conferred in writing and must give him
COURT OF APPEALS and OVERLAND EXPRESS INTENDED AS PARTIAL PAYMENT OF THE specific authority, either to conduct the general business
LINES, INC., respondents. PURCHASE PRICE OF THE PROPERTY, OR AS of the principal or to execute a binding contract containing
x---------------------------------------------------------x PAYMENT OF BACK RENTALS ON THE PROPERTY; terms and conditions which are in the contract he did
G.R. No. 124741 January 28, 2003 3. WHETHER ALICE DIZON WAS AUTHORIZED TO execute. A special power of attorney is necessary to enter
REGINA P. DIZON, AMPARO D. BARTOLOME, RECEIVE THE SUM OF P300,000.00 ON BEHALF OF into any contract by which the ownership of an immovable
FIDELINA D. BALZA, ESTER ABAD DIZON and PETITIONERS; is transmitted or acquired either gratuitously or for a
JOSEPH ANTHONY DIZON, RAYMUND A. DIZON, 4. (A) IF SO, WHETHER PETITIONERS ARE valuable consideration. The express mandate required by
GERARD A. DIZON and JOSE A. DIZON, ESTOPPED FROM QUESTIONING THE BELATED law to enable an appointee of an agency (couched) in
JR., petitioners, EXERCISE BY PRIVATE RESPONDENT OF ITS general terms to sell must be one that expressly mentions
vs. OPTION TO BUY WHEN THEY ACCEPTED THE SAID a sale or that includes a sale as a necessary ingredient of
COURT OF APPEALS HON. MAXIMIANO C. PARTIAL PAYMENT; the act mentioned. For the principal to confer the right
ASUNCION and OVERLAND EXPRESS LINES, (B) IF SO, WHETHER ALICE DIZON CAN VALIDLY BIND upon an agent to sell real estate, a power of attorney must
INC., respondents. PETITIONERS IN THE ABSENCE OF A WRITTEN so express the powers of the agent in clear and
RESOLUTION POWER OF ATTORNEY; unmistakable language. When there is any reasonable
YNARES-SANTIAGO, J.: 5. (A) WHETHER THERE WAS A PERFECTED doubt that the language so used conveys such power, no
On January 28, 1999, this Court rendered judgment in CONTRACT OF SALE BETWEEN THE PARTIES; such construction shall be given the document. 2
these consolidated cases as follows: (B) WHETHER THERE WAS A CONTRACT OF SALE AT It necessarily follows, therefore, that petitioners cannot be
WHEREFORE, in view of the foregoing, both petitions LEAST WITH RESPECT TO THE SHARES OF FIDELA deemed to have received partial payment of the supposed
are GRANTED. The decision dated March 29, 1994 and AND ALICE DIZON; AND purchase price for the land through Alice Dizon. It cannot
the resolution dated October 19, 1995 in CA-G.R. CV 6. WHETHER PRIVATE RESPONDENT'S ACTION FOR even be said that Alice Dizon's acceptance of the money
Nos. 25153-54, as well as the decision dated December SPECIFIC PERFORMANCE HAS PRESCRIBED. bound at least the share of Fidela Dizon, in the absence
11, 1995 and the resolution dated April 23, 1997 in CA- In order to resolve the first issue, it is necessary to pass of a written power of attorney from the latter. It should be
G.R. SP No. 33113 of the Court of Appeals are upon the other questions which relate to the merits of the borne in mind that the Receipt dated June 20, 1975, while
hereby REVERSED and SET ASIDE. case. It is only where there exist strong compelling made out in the name of Fidela Dizon, was signed by
Let the records of this case be remanded to the trial court reasons, such as serving the ends of justice and Alice Dizon alone.
for immediate execution of the judgment dated November preventing a miscarriage thereof, that this Court can Moreover, there could not have been a perfected contract
22, 1982 in Civil Case No. VIII-29155 of the then City suspend the rules.1 of sale. As we held in our Decision dated January 28,
Court (now Metropolitan Trial Court) of Quezon City, After reviewing the records, we find that, despite all of 1999, the implied renewal of the contract of lease between
Branch III as affirmed in the decision dated September 26, private respondent's protestations, there is absolutely no the parties affected only those terms and conditions which
1984 of the then Intermediate Appellate Court (now Court written proof of Alice Dizon's authority to bind petitioners. are germane to the lessee's right of continued enjoyment
of Appeals) and in the resolution dated June 19, 1985 of First of all, she was not even a co-owner of the property. of the property. The option to purchase afforded private
this Court. Neither was she empowered by the co-owners to act on respondent expired after the one-year period granted in
However, petitioners are ordered to REFUND to private their behalf. the contract. Otherwise stated, the implied renewal of the
respondent the amount of P300,000.00 which they The acceptance of the amount of P300,000.00, lease did not include the option to purchase. We see no
received through Alice A. Dizon on June 20, 1975. purportedly as partial payment of the purchase price of the reason to disturb our ruling on this point, viz:
SO ORDERED. land, was an act integral to the sale of the land. As a In this case, there was a contract of lease for one (1) year
Private respondent filed a Motion for Reconsideration, matter of fact, private respondent invokes such receipt of with option to purchase. The contract of lease expired
Second Motion for Reconsideration, and Motion to payment as giving rise to a perfected contract of sale. In without the private respondent, as lessee, purchasing the
Suspend Procedural Rules in the Higher Interest of this connection, Article 1874 of the Civil Code is explicit property but remained in possession thereof. Hence, there
Substantial Justice, all of which have been denied by this that: "When a sale of a piece of land or any interest was an implicit renewal of the contract of lease on a
285
Sales – Chapter 3-12 Cases
monthly basis. The other terms of the original contract of as they are to put an end to controversies, courts should
lease which are revived in the implied new lease under frown upon any attempt to prolong them.5
Article 1670 of the New Civil Code are only those terms ACCORDINGLY, the Motion to Suspend Procedural
which are germane to the lessee's right of continued Rules in the Higher Interest of Substantial Justice filed by
enjoyment of the property leased. Therefore, an implied private respondent is DENIED WITH FINALITY. No further
new lease does not ipso facto carry with it any implied pleadings will be entertained in these cases.
revival of private respondent's option to purchase (as SO ORDERED.
lessee thereof) the leased premises. The provision Davide, Jr., C .J ., see separate opinion.
entitling the lessee the option to purchase the leased Puno, J., concur.
premises is not deemed incorporated in the impliedly
renewed contract because it is alien to the possession of
the lessee. Private respondent's right to exercise the
option to purchase expired with the termination of the
original contract of lease for one year. The rationale of this
Court is that:
"This is a reasonable construction of the provision, which
is based on the presumption that when the lessor allows
the lessee to continue enjoying possession of the property
for fifteen days after the expiration of the contract he is
willing that such enjoyment shall be for the entire period
corresponding to the rent which is customarily paid — in
this case up to the end of the month because the rent was
paid monthly. Necessarily, if the presumed will of the
parties refers to the enjoyment of possession the
presumption covers the other terms of the contract related
to such possession, such as the amount of rental, the date
when it must be paid, the care of the property, the
responsibility for repairs, etc. But no such presumption
may be indulged in with respect to special agreements
which by nature are foreign to the right of occupancy or
enjoyment inherent in a contract of lease."3
There being no merit in the arguments advanced by
private respondent, there is no need to suspend the Rules
of Court and to admit the motion for reconsideration.
While it is within the power of the Court to suspend its own
rules, or to except a particular case from its operation,
whenever the interest of justice require it, however, the
movant must show strong compelling reasons such as
serving the ends of justice and preventing a grave
miscarriage thereof,4 none of which obtains in this case.
Litigation must end sometime and somewhere. An
effective and efficient administration of justice requires
that, once a judgment has become final, the winning party
be not, through a mere subterfuge, deprived of the fruits of
the verdict. Courts must, therefore, guard against any
scheme calculated to bring about that result. Constituted

286
Sales – Chapter 3-12 Cases
G.R. No. 174376 September 12, 2012 to Zosima was demolished to pave the way for the upon the termination of the lease contract in March 1997,
ZOSIMA IN CORPORA TED, Petitioner, construction of the Light Rail Transit (LRT) Line II Project. an implied new lease or tacita reconduccion was created
vs. She further alleged that she bought the warehouse for ₱ by operation of law between the parties, 6 and that from
LILIA SALIMBAGAT and all persons claiming rights 300,000.00 as evidenced by a Deed of Conditional Sale, March 1997 to March 2000, Salimbagat continued to pay
under her, Respondents. and she had declared the property for taxation purposes. Zosima the monthly rentals. Notwithstanding this finding,
DECISION On July 6, 2004, after the submission of the parties’ the CA was not convinced that Salimbagat had unlawfully
BRION, J.: position papers, the MeTC set the case for clarificatory possessed the property from April 2000 to June 2003.
Before us is the petition for review on certiorari, 1 filed by hearing. It sought to resolve the following factual issues: According to the CA, the records do not support this
Zosima Incorporated (Zosima) under Rule 45 of the Rules 1. Whether the office building subject of the expired conclusion and Zosima failed to introduce any evidence to
of Court, assailing the Decision2 dated June 26, 2006 of contract of lease is still existing vis-à-vis Salimbagat’s prove its allegations.7
the Court of Appeals (CA) in CA-G.R. SP No. 92475. The claim that it had already been demolished; Zosima moved for reconsideration of the CA decision but
CA reversed and set aside the decision 3 dated October 5, 2. Presuming it still exists, whether Salimbagat is the CA denied the motion in a resolution 8 dated August
2005 of the Regional Trial Court (RTC), Branch 20, presently occupying the office building; and, 25, 2006.
Manila. The RTC affirmed the decision4 dated May 4, 3. Whether the warehouse/factory erected on a dried The Petition
2005 of the Metropolitan Trial Court premises and to pay estero that Salimbagat now claims to occupy is part and Zosima now questions the CA’s ruling before us. Zosima
Zosima rental arrearages, attorney’s fees and costs of parcel of the land registered in the name of Zosima under posits that the CA erred in ruling on factual matters that
suit. Transfer Certificate of Title No. 262637. were not part of the proceedings in the lower courts.
The Antecedent Facts Zosima filed a motion to reset the clarificatory hearing, Zosima also insists that the subject matter of the unlawful
Zosima, a domestic corporation, has been the registered prompting Salimbagat’s counsel to submit the case for detainer complaint is the office building owned by Zosima,
owner of an office building situated at 2414 Legarda decision solely on the basis of the position papers that the not the warehouse on the dried estero.
Street, Sampaloc, Manila. Sometime in April 1993, parties had submitted. For her part, Salimbagat argues that the appellate court
Zosima entered into a contract with Salimbagat for the On May 4, 2005, the MeTC rendered a decision whose may review factual matters on appeal, to determine
lease of the office building. The lease was on a yearly dispositive portion reads: whether these factual findings are just and equitable in
basis with the initial monthly rate of ₱ 8,000.00 that is WHEREFORE, judgment is hereby rendered ordering accordance with the aim of justice. Salimbagat further
subject to an annual increase. In 1999, the monthly rental defendant Lilia Salimbagat and all other persons claiming argues that Zosima has no cause of action to file the
fee reached ₱ 14,621.00. In March 2000, no monthly fee rights under her: complaint for unlawful detainer, since the office building
was paid because the contract of lease was allegedly not 1) To vacate the office building subject of the expired she had lease had already been demolished and she
renewed. Contract of Lease located at No. 2414 Legarda Street, presently occupies a warehouse that does not belong to
On June 20, 2003, Zosima, through counsel, sent a formal Sampaloc, Manila covered by Transfer Certificate of Title Zosima.
letter of demand to Salimbagat, requiring her to pay her No. 262637 and peacefully surrender possession thereof The Court’s Ruling
arrears within fifteen (15) days from receipt of the demand to the plaintiff; We deny the petition for lack of merit.
letter and to vacate the property. Despite the receipt of the 2) To pay plaintiff rental arrearages in the amount of ₱ The complaint for unlawful detainer
demand letter, Salimbagat refused to vacate the property 14,621.00 per month counted from April 2000 until the The present petition is an action for unlawful detainer
and to pay her alleged rental obligations. time the office building was fully vacated by said governed by Section 1, Rule 70 of the Rules of Court. 9 As
On November 5, 2003, Zosima filed a case for unlawful defendant; the principal issue in an unlawful detainer case is the right
detainer against Salimbagat. Zosima alleged that from 3) To pay attorney’s fees fixed in the reasonable amount to possess a real property, the subject matter must refer
April 2000 to October 2003, Salimbagat had accumulated of ₱ 7,000.00; and to a particular property. In an unlawful detainer, the
arrears in her rental payments amounting to ₱ 4) To pay the costs of suit.5 defendant’s possession of the plaintiff’s property is based
628,703.00. Salimbagat appealed the MeTC decision to the RTC. In its on the plaintiff’s permission expressed through an express
On March 26, 2004, Salimbagat filed her answer alleging decision dated October 5, 2005, the RTC fully affirmed the or implied contract between them. The defendant’s
that she was not occupying the property of Zosima. MeTC decision. Salimbagat elevated the case to the CA possession becomes illegal only when the plaintiff
Salimbagat alleged that although she was occupying a which reversed the RTC’s decision on June 26, 2006, and demands the return of the property, either because of the
property using the same address denominated as "2414 dismissed the case for unlawful detainer. expiration of the right to possess it or the termination of
Legarda Street, Sampaloc, Manila," it was not the same The CA did not dispute the findings of both lower courts their contract, and the defendant refuses to heed the
office building that Zosima owned, but a warehouse on a on the existence of a contractual relationship between the demand.10
dried estero located at the back of the office building. parties, nor that the lease had been annually renewed Zosima’s complaint for unlawful detainer referred to the
Salimbagat argued that the office building which belonged from April 1993 to March 1997. The CA also agreed that office building located at "2414 Legarda Street, Sampaloc,
287
Sales – Chapter 3-12 Cases
Manila;" hence, we confine our ruling to the question of of the property between April 2000 and June 2003 when a months. In case of daily rent, the courts may also fix a
whether Salimbagat should be held liable for unlawfully demand to vacate was made. Zosima cannot reason out longer period after the lessee has stayed in the place for
occupying the office building that was the subject of their that Salimbagat was likewise not able to prove that she over one month. [emphasis ours]
lease agreement. had not been in possession of the property as the burden Thus, after the expiration of the contract of lease, the
It is not disputed that Salimbagat had been in possession of adducing proof arises only after Zosima, as plaintiff, implied new lease should have only been in a monthly
of the leased property from April 1993 to March 1997 and had proven that Salimbagat had been in possession basis. In this regard, we find it significant that it was only
had been diligently paying the monthly rentals. There is during the relevant time. Additionally, the party carrying on June 20, 2003, or three (3) years after the last payment
also no issue that at the time the lease contract expired in the burden of proof must rely on the strength of his own of the monthly rentals, that Zosima filed the complaint for
March 1997, no new contract of lease was executed evidence and not upon the weakness of the unlawful detainer against Salimbagat. It does not help that
between the parties for the period of March 1997 to March defendant’s.14 For us to justify a judgment in Zosima’s Zosima failed to adduce any additional evidence to rebut
2000. Salimbagat, however, continued to pay Zosima the favor, it must in the first place establish through the allegation that by April 2000, no office building stood
monthly rentals during that period. Beginning April 2000, preponderance of evidence the case it alleged – that to be leased because it had been demolished to pave way
Salimbagat stopped the payment of monthly rentals, Salimbagat possessed its property after Salimbagat’s right for the construction of the LRT Line II Project.16
alleging that she was no longer in possession of the of possession had lapsed or expired. We further note that Salimbagat was able to produce tax
property. Despite this claim, Salimbagat still used the In this light, Zosima’s contention – that although the lease declarations and a copy of the Deed of Conditional Sale
address of the property, alleging this time that she was contract had already expired, the principle of implied new as proof of her right to possess the warehouse located on
occupying not the office building itself that she used to lease or tacita reconduccion existed by operation of law a dried estero and adjoining the demolished building she
lease, but the warehouse on the dried estero behind the between the periods of April 2000 and June 2003 – is not used to lease.17 While tax receipts and declarations are
office building. correct. An implied new lease will set in if it is shown that: not incontrovertible proof of ownership, they constitute, at
The evidence on record does not contain any information (a) the term of the original contract of lease has expired; least, proof that the holder has a claim of title over the
supporting the allegation that Salimbagat has been in (b) the lessor has not given the lessee a notice to vacate; property.18 In practical terms under the circumstances of
actual possession of Zosima’s property from April 2000, and (c) the lessee continued enjoying the thing leased for this case, we see it absurd for Salimbagat to be occupying
but neither does it confirm Zosima’s allegation that 15 days with the acquiescence of the lessor. This a property and paying monthly rentals on it when she
Salimbagat then occupied the office building. This was acquiescence may be inferred from the failure of the owns and occupies the property just behind it.1âwphi1
precisely the reason why the MeTC set the case for a lessor to serve notice to vacate upon the lessee. 15 This Under the existing evidentiary situation, we see no
clarificatory hearing. Unfortunately, the hearing was principle is provided for under Article 1670 of the Civil evidence supporting Zosima's allegations and, thus,
cancelled due to Zosima’s failure to appear, and the case Code: cannot rule in its favor.
was submitted for decision solely on the basis of the Article 1670. If at the end of the contract the lessee should WHEREFORE, premises considered, we hereby DENY
parties’ position papers. The CA decision in fact noted continue enjoying the thing leased for fifteen days with the the petition for lack of merit. Accordingly, we AFFIRM the
that: acquiescence of the lessor, and unless a notice to the decision of the Court of Appeals in CA-G.R. SP No.
These issues were not at all resolved due to the contrary by either party has previously been given, it is 92475.
unavailability of the respondent’s counsel despite due understood that there is an implied new lease, not for the SO ORDERED.
notice. These matters are essential to establish its case period of the original contract, but for the time established
by preponderance of evidence for the burden of proof is in Articles 1682 and 1687. The other terms of the original
on the respondent as plaintiff in the original action for the contract shall be revived. [emphasis and underscoring
ejectment case. It leads us to conclude, therefore, that the ours]
respondent, as plaintiff in the unlawful detainer case, The cited Article 1687, on the other hand, provides:
failed to prove its case by preponderance of evidence Article 1687. If the period for the lease has not been fixed,
since the burden of proof rests on its side. 11 (emphasis it is understood to be from year to year, if the rent agreed
and underscoring ours) upon is annual; from month to month, if it is monthly; from
In civil cases, the rule is that the party carrying the burden week to week, if the rent is weekly; and from day to day, if
of proof must establish his case by a preponderance of the rent is to be paid daily. However, even though a
evidence,12 i.e., by evidence that is of greater weight, or monthly rent is paid, and no period for the lease has been
more convincing, than that which is offered in opposition set, the courts may fix a longer term for the lease after the
to it.13 lessee has occupied the premises for over one year. If the
In the present case, Zosima, as plaintiff, bears the burden rent is weekly, the courts may likewise determine a longer
of proving that Salimbagat has been in actual possession period after the lessee has been in possession for over six
288
Sales – Chapter 3-12 Cases
G.R. No. 179382 January 14, 2013 Gaddi. In support thereof, Sps. Mamaril averred that the 2. To pay jointly and severally to the plaintiffs the daily
SPOUSES BENJAMIN C. MAMARIL AND SONIA P. loss of the subject vehicle was due to the gross loss of the income/boundary of the said jeepney to be
MAMARIL, Petitioners, negligence of the above-named security guards on-duty reckoned fromits loss up to the final adjudication of the
vs. who allowed the subject vehicle to be driven out by a case, which is ₱275.00 a day;
THE BOY SCOUT OF THE PHILIPPINES, AIB stranger despite their agreement that only authorized 3. To pay jointly and severally to the plaintiffs moral
SECURITY AGENCY, INC., CESARIO PEÑA,* AND drivers duly endorsed by the owners could do so. Peña damages in the amount of ₱50,000.00;
VICENTE GADDI, Respondents. and Gaddi even admitted their negligence during the 4. To pay jointly and severally to the plaintiffs exemplary
DECISION ensuing investigation. Notwithstanding, BSP and AIB did damages in the amount of ₱50,000.00;
PERLAS-BERNABE, J.: not heed Sps. Mamaril's demands for a conference to 5. To pay jointly and severally the attorney's fees of
This is a Petition for Review on Certiorari assailing the settle the matter. They therefore prayed that Peña and ₱50,000.00 and appearances in court the amount of
May 31, 2007 Decision1 and August 16, 2007 Gaddi, together with AIB and BSP, be held liable for: (a) ₱1,500.00 per appearance; and
Resolution2 of the Court of Appeals (CA) in CA-G.R. CV the value of the subject vehicle and its accessories in the 6. To pay cost.
No. 75978. The dispositive portion of the said Decision aggregate amount of ₱300,000.00; (b) ₱275.00 SO ORDERED.10
reads: representing daily loss of income/boundary reckoned from The RTC found that the act of Peña and Gaddi in allowing
WHEREFORE, the Decision dated November 28, 2001 the day the vehicle was lost; (c) exemplary damages; (d) the entry of an unidentified person and letting him drive
and the Order dated June 11, 2002 rendered by the moral damages; (e) attorney's fees; and (f) cost of suit. out the subject vehicle in violation of their internal
Regional Trial Court of Manila, Branch 39 is hereby In its Answer,7 BSP denied any liability contending that agreement with Sps. Mamaril constituted gross
MODIFIED to the effect that only defendants AIB Security not only did Sps. Mamaril directly deal with AIB with negligence, rendering AIB and its security guards liable for
Agency, Inc., Cesario Peña and Vicente Gaddi are held respect to the manner by which the parked vehicles would the former's loss. BSP was also adjudged liable because
jointly and severally liable to pay plaintiffs-appellees be handled, but the parking ticket8 itself expressly stated the Guard Service Contract it entered into with AIB offered
Spouses Benjamin C. Mamaril and Sonia P. Mamaril the that the "Management shall not be responsible for loss of protection to all properties inside the BSP premises, which
amount of Two Hundred Thousand Pesos (₱200,000.00) vehicle or any of its accessories or article left therein." It necessarily included Sps. Mamaril's vehicles. Moreover,
representing the cost of the lost vehicle, and to pay the also claimed that Sps. Mamaril erroneously relied on the the said contract stipulated AIB's obligation to indemnify
cost of suit. The other monetary awards are DELETED for Guard Service Contract. Apart from not being parties BSP for all losses or damages that may be caused by any
lack of merit and/or basis. thereto, its provisions cover only the protection of BSP's act or negligence of its security guards. Accordingly, the
Defendant-Appellant Boy Scout of the Philippines is properties, its officers, and employees. BSP, AIB, and security guards Peña and Gaddi were held
absolved from any liability. In addition to the foregoing defenses, AIB alleged that it jointly and severally liable for the loss suffered by Sps.
SO ORDERED.3 has observed due diligence in the selection, training and Mamaril.
The Antecedent Facts supervision of its security guards while Peña and Gaddi On June 11, 2002, the RTC modified its decision reducing
Spouses Benjamin C. Mamaril and Sonia P. Mamaril claimed that the person who drove out the lost vehicle the cost of the stolen vehicle from ₱250,000.00 to
(Sps. Mamaril) are jeepney operators since 1971. They from the BSP compound represented himself as the ₱200,000.00.11
would park their six (6) passenger jeepneys every night at owners' authorized driver and had with him a key to the Only BSP appealed the foregoing disquisition before the
the Boy Scout of the Philippines' (BSP) compound located subject vehicle. Thus, they contended that Sps. Mamaril CA.
at 181 Concepcion Street, Malate, Manila for a fee of have no cause of action against them. The CA Ruling
₱300.00 per month for each unit. On May 26, 1995 at 8 The RTC Ruling In its assailed Decision,12 the CA affirmed the finding of
o'clock in the evening, all these vehicles were parked After due proceedings, the RTC rendered a negligence on the part of security guards Peña and
inside the BSP compound. The following morning, Decision9 dated November 28, 2001 in favor of Sps. Gaddi. However, it absolved BSP from any liability,
however, one of the vehicles with Plate No. DCG 392 was Mamaril. The dispositive portion of the RTC decision holding that the Guard Service Contract is purely between
missing and was never recovered.4 According to the reads: BSP and AIB and that there was nothing therein that
security guards Cesario Peña (Peña) and Vicente Gaddi WHEREFORE, judgment is hereby rendered ordering the would indicate any obligation and/or liability on the part of
(Gaddi) of AIB Security Agency, Inc. (AIB) with whom BSP defendants Boy Scout of the Philippines and AIB Security BSP in favor of third persons, such as Sps. Mamaril. Nor
had contracted5 for its security and protection, a male Agency, with security guards Cesario Pena and Vicente was there evidence sufficient to establish that BSP was
person who looked familiar to them took the subject Gaddi: - negligent.
vehicle out of the compound. 1. To pay the plaintiffs jointly and severally the cost of the It further ruled that the agreement between Sps. Mamaril
On November 20, 1996, Sps. Mamaril filed a vehicle which is ₱250,000.00 plus accessories of and BSP was substantially a contract of lease whereby
complaint6 for damages before the Regional Trial Court ₱50,000.00; the former paid parking fees to the latter for the lease of
(RTC) of Manila, Branch 39, against BSP, AIB, Peña and parking slots. As such, the lessor, BSP, was not an
289
Sales – Chapter 3-12 Cases
insurer nor bound to take care and/or protect the lessees' The petition lacks merit. pool of security guards or watchmen employed by the
vehicles. Article 20 of the Civil Code provides that every person, agency shall be assigned to it; the duty to observe the
On the matter of damages, the CA deleted the award of who, contrary to law, willfully or negligently causes diligence of a good father of a family in the selection of the
₱50,000.00 representing the value of the accessories damage to another, shall indemnify the latter for the same. guards cannot, in the ordinary course of events, be
inside the lost vehicle and the ₱275.00 a day for loss of Similarly, Article 2176 of the Civil Code states: demanded from the client whose premises or property are
income in the absence of proof to support them. It also Art. 2176. Whoever by act or omission causes damage to protected by the security guards. The fact that a client
deleted the award of moral and exemplary damages and another, there being fault or negligence, is obliged to pay company may give instructions or directions to the
attorney's fees for lack of factual and legal bases. for the damage done. Such fault or negligence, if there is security guards assigned to it, does not, by itself, render
Sps. Mamaril's motion for reconsideration thereof was no preexisting contractual relation between the parties, is the client responsible as an employer of the security
denied in the August 16, 2007 Resolution.13 called a quasi-delict and is governed by the provisions of guards concerned and liable for their wrongful acts or
Issues Before the Court this Chapter. omissions. Those instructions or directions are ordinarily
Hence, the instant petition based on the following In this case, it is undisputed that the proximate cause of no more than requests commonly envisaged in the
assignment of errors, to wit: the loss of Sps. Mamaril's vehicle was the negligent act of contract for services entered into with the security
I. security guards Peña and Gaddi in allowing an agency.20
THE HONORABLE COURT OF APPEALS SERIOUSLY unidentified person to drive out the subject vehicle. Nor can it be said that a principal-agent relationship
ERRED IN ABSOLVING RESPONDENT BOY SCOUT Proximate cause has been defined as that cause, which, existed between BSP and the security guards Peña and
OF THE PHILIPPINES FROM ANY LIABILITY. in natural and continuous sequence, unbroken by any Gaddi as to make the former liable for the latter's
II. efficient intervening cause, produces the injury or loss, complained act. Article 1868 of the Civil Code states that
THE HONORABLE COURT OF APPEALS COMMITTED and without which the result would not have occurred.15 "by the contract of agency, a person binds himself to
SERIOUS MISTAKE WHEN IT RULED THAT THE Moreover, Peña and Gaddi failed to refute Sps. Mamaril's render some service or to do something in representation
GUARD SERVICE CONTRACT IS PURELY BETWEEN contention16 that they readily admitted being at fault or on behalf of another, with the consent or authority of
BOY SCOUT OF THE during the investigation that ensued. the latter." The basis for agency therefore is
PHILIPPINES AND AIB SECURITY AGENCY, INC., AND On the other hand, the records are bereft of any finding of representation,21 which element is absent in the instant
IN HOLDING THAT THERE IS ABSOLUTELY NOTHING negligence on the part of BSP. Hence, no reversible error case. Records show that BSP merely hired the services of
IN THE SAID CONTRACT THAT WOULD INDICATE was committed by the CA in absolving it from any liability AIB, which, in turn, assigned security guards, solely for
ANY OBLIGATION AND/OR LIABILITY ON THE PART for the loss of the subject vehicle based on fault or the protection of its properties and premises. Nowhere
OF THE PARTIES THEREIN IN FAVOR OF THIRD negligence. can it be inferred in the Guard Service Contract that AIB
PERSONS, SUCH AS PETITIONERS HEREIN. Neither will the vicarious liability of an employer under was appointed as an agent of BSP. Instead, what the
III. Article 218017 of the Civil Code apply in this case. It is parties intended was a pure principal-client relationship
THE HONORABLE COURT OF APPEALS COMMITTED uncontested that Peña and Gaddi were assigned as whereby for a consideration, AIB rendered its security
SERIOUS ERROR IN THE INTERPRETATION OF LAW security guards by AIB to BSP pursuant to the Guard services to BSP.
WHEN IT CONSIDERED THE AGREEMENT BETWEEN Service Contract. Clearly, therefore, no employer- Notwithstanding, however, Sps. Mamaril insist that BSP
BOY SCOUT OF THE PHILIPPINES AND PETITIONERS employee relationship existed between BSP and the should be held liable for their loss on the basis of the
A CONTRACT OF LEASE, WHEREBY THE BOY SCOUT security guards assigned in its premises. Consequently, Guard Service Contract that the latter entered into with
IS NOT DUTY BOUND TO PROTECT OR TAKE CARE the latter's negligence cannot be imputed against BSP but AIB and their parking agreement with BSP.
OF PETITIONERS' VEHICLES. should be attributed to AIB, the true employer of Peña and Such contention cannot be sustained.
IV. Gaddi.18 Article 1311 of the Civil Code states:
THE HONORABLE COURT OF APPEALS SERIOUSLY In the case of Soliman, Jr. v. Tuazon,19 the Court Art. 1311. Contracts take effect only between the parties,
ERRED WHEN IT RULED THAT PETITIONERS ARE enunciated thus: their assigns and heirs, except in case where the rights
NOT ENTITLED TO DAMAGES AND ATTORNEY'S It is settled that where the security agency, as here, and obligations arising from the contract are not
FEES.14 recruits, hires and assigns the work of its watchmen or transmissible by their nature, or by stipulation or by
In fine, Sps. Mamaril maintain that: (1) BSP should be security guards, the agency is the employer of such provision of law. The heir is not liable beyond the value of
held liable for the loss of their vehicle based on the Guard guards and watchmen. Liability for illegal or harmful acts the property he received from the decedent.
Service Contract and the parking ticket it issued; and (2) committed by the security guards attaches to the If a contract should contain some stipulation in favor of a
the CA erred in deleting the RTC awards of damages and employer agency, and not to the clients or customers of third person, he may demand its fulfillment provided he
attorney's fees. such agency. As a general rule, a client or customer of a communicated his acceptance to the obligor before its
The Court's Ruling security agency has no hand in selecting who among the revocation. A mere incidental benefit or interest of a
290
Sales – Chapter 3-12 Cases
person is not sufficient. The contracting parties must have Moreover, the Court concurs with the finding of the CA Anent Sps. Mamaril's claim that the exculpatory clause:
clearly and deliberately conferred a favor upon a third that the contract between the parties herein was one of "Management shall not be responsible for loss of vehicle
person. lease25 as defined under Article 164326 of the Civil Code. or any of its accessories or article left therein"31 contained
Thus, in order that a third person benefited by the second It has been held that the act of parking a vehicle in a in the BSP issued parking ticket was void for being a
paragraph of Article 1311, referred to as a stipulation pour garage, upon payment of a fixed amount, is a contract of adhesion and against public policy, suffice it to
autrui, may demand its fulfillment, the following requisites lease.27 Even in a majority of American cases, it has been state that contracts of adhesion are not void per se. It is
must concur: (1) There is a stipulation in favor of a third ruled that where a customer simply pays a fee, parks his binding as any other ordinary contract and a party who
person; (2) The stipulation is a part, not the whole, of the car in any available space in the lot, locks the car and enters into it is free to reject the stipulations in its entirety.
contract; (3) The contracting parties clearly and takes the key with him, the possession and control of the If the terms thereof are accepted without objection, as in
deliberately conferred a favor to the third person - the car, necessary elements in bailment, do not pass to the this case, where plaintiffs-appellants have been leasing
favor is not merely incidental; (4) The favor is parking lot operator, hence, the contractual relationship BSP's parking space for more or less 20 years,32 then the
unconditional and uncompensated; (5) The third person between the parties is one of lease.28 contract serves as the law between them.33 Besides, the
communicated his or her acceptance of the favor before In the instant case, the owners parked their six (6) parking fee of ₱300.00 per month or ₱10.00 a day for
its revocation; and (6) The contracting parties do not passenger jeepneys inside the BSP compound for a each unit is too minimal an amount to even create an
represent, or are not authorized, by the third monthly fee of ₱300.00 for each unit and took the keys inference that BSP undertook to be an insurer of the
party.22 However, none of the foregoing elements obtains home with them. Hence, a lessor-lessee relationship safety of plaintiffs-appellants' vehicles.
in this case. indubitably existed between them and BSP. On this score, On the matter of damages, the Court noted that while
It is undisputed that Sps. Mamaril are not parties to the Article 1654 of the Civil Code provides that "the lessor Sonia P. Mamaril testified that the subject vehicle had
Guard Service Contract.1âwphi1 Neither did the subject (BSP) is obliged: (1) to deliver the thing which is the object accessories worth around !J50,000.00, she failed to
agreement contain any stipulation pour autrui. And even if of the contract in such a condition as to render it fit for the present any receipt to substantiate her claim. 34 Neither did
there was, Sps. Mamaril did not convey any acceptance use intended; (2) to make on the same during the lease all she submit any record or journal that would have
thereof. Thus, under the principle of relativity of contracts, the necessary repairs in order to keep it suitable for the established the purported ₱275.0035 daily earnings of their
they cannot validly claim any rights or favor under the said use to which it has been devoted, unless there is a jeepney. It is axiomatic that actual damages must be
agreement.23 As correctly found by the CA: stipulation to the contrary; and (3) to maintain the lessee proved with reasonable degree of certainty and a party is
First, the Guard Service Contract between defendant- in the peaceful and adequate enjoyment of the lease for entitled only to such compensation for the pecuniary loss
appellant BSP and defendant AIB Security Agency is the entire duration of the contract." In relation thereto, that was duly proven. Thus, absent any competent proof
purely between the parties therein. It may be observed Article 1664 of the same Code states that "the lessor is of the amount of damages sustained, the CA properly
that although the whereas clause of the said agreement not obliged to answer for a mere act of trespass which a deleted the said awards.36
provides that defendant-appellant desires security and third person may cause on the use of the thing leased; but Similarly, the awards of moral and exemplary damages
protection for its compound and all properties therein, as the lessee shall have a direct action against the intruder." and attorney's fees were properly disallowed by the CA for
well as for its officers and employees, while inside the Here, BSP was not remiss in its obligation to provide Sps. lack of factual and legal bases. While the RTC granted
premises, the same should be correlated with paragraph Mamaril a suitable parking space for their jeepneys as it these awards in the dispositive portion of its November
3(a) thereof which provides that the security agency shall even hired security guards to secure the premises; hence, 28, 2001 decision, it failed to provide sufficient justification
indemnify defendant-appellant for all losses and damages it should not be held liable for the loss suffered by Sps. therefor.37
suffered by it attributable to any act or negligence of the Mamaril. WHEREFORE premises considered, the instant petition is
former's guards. It bears to reiterate that the subject loss was caused by DENIED. The May 31, 2007 Decision and August 16,
Otherwise stated, defendant-appellant sought the services the negligence of the security guards in allowing a 2007 Resolution of the Court of Appeals in CA-G.R. CV
of defendant AIB Security Agency for the purpose of the stranger to drive out plaintiffs-appellants' vehicle despite No. 75978 are AFFIRMFED.
security and protection of its properties, as well as that of the latter's instructions that only their authorized drivers SO ORDERED.
its officers and employees, so much so that in case of loss may do so. Moreover, the agreement with respect to the ESTELA M. PERLAS-BERNABE
of [sic] damage suffered by it as a result of any act or ingress and egress of Sps. Mamaril's vehicles were Associate Justice
negligence of the guards, the security agency would then coordinated only with AIB and its security WE CONCUR
be held responsible therefor. There is absolutely nothing guards,29 without the knowledge and consent of BSP.
in the said contract that would indicate any obligation Accordingly, the mishandling of the parked vehicles that
and/or liability on the part of the parties therein in favor of resulted in herein complained loss should be recovered
third persons such as herein plaintiffs-appellees.24 only from the tort feasors (Peña and Gaddi) and their
employer, AIB; and not against the lessor, BSP.30
291
Sales – Chapter 3-12 Cases

292

Potrebbero piacerti anche